SBT Mech TH Vol 2

You might also like

Download as pdf or txt
Download as pdf or txt
You are on page 1of 592

Mechanics Vol II

Your Personal Coach

Physics


for
JEE Main and
Advanced
Mechanics Vol II

Your Personal Coach

Physics


for
JEE Main and
Advanced

Shashi Bhushan Tiwari

McGraw Hill Education (India) Private Limited


chennai
McGraw Hill Education Offices
Chennai  New York  St Louis  San Francisco  Auckland  Bogotá  Caracas
Kuala Lumpur  Lisbon  London  Madrid  Mexico City  Milan  Montreal
San Juan  Santiago  Singapore  Sydney  Tokyo  Toronto
McGraw Hill Education (India) Private Limited
Published by McGraw Hill Education (India) Private Limited,
444/1, Sri Ekambara Naicker Industrial Estate,
Alapakkam, Porur, Chennai - 600 116, Tamil Nadu, India

Your Personal Coach Physics for JEE Main & Advanced (Mechanics – II)

Copyright © 2019, McGraw Hill Education (India) Private Limited.

No part of this publication may be reproduced or distributed in any form or by any means, electronic, mechanical, photocopying, recording, or otherwise
or stored in a database or retrieval system without the prior written permission of the publishers. The program listings (if any) may be entered, stored
and executed in a computer system, but they may not be reproduced for publication.

This edition can be exported from India only by the publishers,


McGraw Hill Education (India) Private Limited

123456789 D101769 23 22 21 20 19

ISBN (13): 978-93-87572-61-4


ISBN (10): 93-87572-61-7

Information contained in this work has been obtained by McGraw Hill Education (India), from sources believed to be reliable. However, neither
McGraw Hill Education (India) nor its authors guarantee the accuracy or completeness of any information published herein, and neither McGraw
Hill Education (India) nor its authors shall be responsible for any errors, omissions, or damages arising out of use of this information. This work
is published with the understanding that McGraw Hill Education (India) and its authors are supplying information but are not attempting to render
engineering or other professional services. If such services are required, the assistance of an appropriate professional should be sought.

Cover Designer: Creative Designer

Visit us at: www.mheducation.co.in

Write to us at: info.india@mheducation.com

CIN: U22200TN1970PTC111531

Toll Free Number: 1800 103 5875


Dedicated to
All youngsters
Working hard to transform India
Preface

Physics is extremely hierarchical. It is essential to have a thorough understanding of the concepts of the previous chapters
in order to master a particular one. Also, the principles involved are often simple to state but very difficult to apply.
For the reasons cited above a Physics text book must have a great deal of continuity in what it discusses and must have
the relevant examples to illustrate every little principle. This book has been written keeping these two basic principles in
mind. I have tried my level best to unfold the concepts gradually, one by one; illustrating each of them with examples.
During my years of experience as a Physics teacher, I have realised that nothing contributes more to understand this
subject than a good example. This book is a translation of this philosophy of mine.
This is a student centric book. Its basic aim is to make a student learn the basic principles of Physics by himself/her-
self. By saying this, I am not discouraging discussions with friends and teachers. Discussions are essential part of learning
this subject. All I intend is to provide a book which will help a student understand the basics on his own while working
through the book, requiring minimum external help. For this reason, I have provided solutions to almost all problems in
the book.
To make problem solving enjoyable, I have tried to pick up examples from real life wherever possible.
I shall be grateful to all of you who point out any error or help me with useful suggestions.

S.B. Tiwari
How to use this Book

To make full use of this book one must go through the topics sequentially while working through the examples and
in-chapter problems given under heading “Your Turn”. By doing this you will have a fair amount of grasp over all the
essentials in a chapter.
Miscellaneous examples given at the end of each chapter have problems which involve multiple concepts or have some
mathematical complexity or are tricky. If you are studying the subject for the first time or are hard pressed for time, you
may skip the section on miscellaneous examples.
Almost every solved example starts with explanation of physical situation and basic principles involved. This feature
comes under heading “Concepts” at the beginning of each example.
I have highlighted the important points of learning under the heading “In short”. Here, I have also taken important
learning points from the examples. While going through the chapter it is essential to go through these points.
Physics cannot be mastered without practice. Keeping this in view I have given three Worksheets (exercises) after every
chapter. Worksheet 1 has multiple choice objective type questions with single correct answer. Worksheet 2 has multiple
choice questions having one or more than one correct answers. Worksheet 3 has subjective problems. A good number of
problems has been given in the Worksheets to give you a good practice on concepts learnt.
After few chapters, at regular intervals, you will find separate assignments on miscellaneous type problems. These are
problems based on latest trend of competitive examinations and contain Match the Column type questions and problems
based on a given paragraph. Attempt these questions only after you gain enough confidence in the related chapters.
I have kept these problems in separate chapters so that you have no bias or hint about the equation/s to use.
In the last chapter, you will find a collection of questions asked in competitive examinations since 2005.This is an ideal
collection of problems for revision.
In the end of the book, solutions to all questions has been given. Solutions are quite descriptive and easy to
understand.
Those who desire to practice at even higher level, I recommend my book – “Problems in Physics for JEE
Advanced”.
I hope you will enjoy this book.

S.B. Tiwari

Acknowledgements

My sincere thanks to all who supported, encouraged and helped me in preparation of the book. My special thanks to –
• My family members.
• My students, who have taught me a lot.
• Entire management team at McGraw Hill. In particular, I am grateful to Mr. Biswajit Das who worked tirelessly
in bringing out the book.

S.B. Tiwari
Contents

Preface  vii   Worksheet 1  1.25


How to use this Book  ix   Worksheet 2  1.29
Acknowledgements  xi   Worksheet 3  1.31
  Answer Sheet  1.33
1. Centre of Mass 1.1–1.33
1. Introduction 1.1 2. Momentum and Its Conservation 2.1–2.58
2. Importance of centre of mass 1.1 1. Introduction 2.1
3. Location of Centre of Mass 1.2 2. Momentum 2.1
3.1 Two Particles 1.2 • Your Turn  2.1
3.2 Many particles on a straight line 1.2 3. Force and Momentum (Newton’s
3.3 Three-Dimensional Distribution of Second Law) 2.2
particles 1.2 • Your Turn  2.4
• Your Turn  1.4 4. Impulse 2.5
3.4 Centre of Mass of groups of particles 1.4 4.1 Impulsive Force 2.6
3.5 Centre of mass of continuous bodies 1.5 • Your Turn  2.9
• Your Turn  1.8 5. Conservation of Linear Momentum 2.9
4. Motion of Centre of Mass 1.9 • Your Turn  2.14
4.1 Velocity of Com 1.9 6. Collision 2.15
4.2 Acceleration of Com 1.9 6.1 Line of Impact 2.15
• Your Turn  1.14 6.2 Elastic and Inelastic Collision 2.16
5. Gravitational Potential Energy of an 6.3 Coefficient of Restitution 2.17
Extended Body 1.15 6.4 Head on, Perfectly Inelastic Collision 2.18
• Your Turn  1.15 6.5 Head on Perfectly Elastic Collision 2.18
6. The Centre of Mass Frame 1.16 6.6 Head on, Inelastic Collision 2.19
6.1 Kinetic Energy 1.16 • Your Turn  2.23
• Your Turn  1.17 6.7 Oblique Collision 2.23
7. Centre of Gravity 1.18 • Your Turn  2.28
• Your Turn  1.18 7. System with Varying Mass 2.29
■ Miscellaneous Examples  1.18 • Your Turn  2.31
xiv  Mechanics II

■ Miscellaneous Examples  2.31 6. Rotational Dynamics 6.1–6.70


  Worksheet 1  2.42 1. Introduction 6.1
  Worksheet 2  2.50 2. Moment of Inertia 6.1
  Worksheet 3  2.54 2.1 Theorem of perpendicular axis 6.4
  Answer Sheet  2.57 2.2 Parallel axis theorem 6.4
3. Miscellaneous problems on • Your Turn  6.10
chapter 1 and 2 3.1–3.9 2.3 Radius of Gyration 6.11
Match the Column 3.1 • Your Turn  6.11
Passage-based Problems 3.3 3. Newton’s Second Law for Rotation 6.11
  Answer Sheet  3.9 • Your Turn  6.15
4. Role of Friction in Rolling 6.16
4. Torque and Equilibrium 4.1–4.23 • Your Turn  6.17
1. Introduction 4.1 4.1 Rolling on an incline 6.17
2. What is Rotation? 4.1 • Your Turn  6.19
3. Torque 4.1 4.2 Rolling with other forces, apart from
3.1 Torque of a force about a point 4.2 friction, producing torque 6.19
3.2 Torque of a force about an axis 4.2 4.3 Role of friction in motion of a car 6.20
• Your Turn  4.5 • Your Turn  6.21
4. Equilibrium 4.6 5. Kinetic Energy of Rotation 6.21
• Your Turn  4.8 5.1 Kinetic Energy in Combined rotation
5. Toppling 4.8 and translation 6.22
5.1 A cube on an incline 4.9 • Your Turn  6.23
• Your Turn  4.10 6. Work Done by a Torque and Work–
Energy Theorem 6.23
■ Miscellaneous Examples  4.11
• Your Turn  6.26
  Worksheet 1  4.18
7. Angular Momentum 6.27
  Worksheet 2  4.21
7.1 Angular momentum of a particle 6.27
  Worksheet 3  4.22
• Your Turn  6.28
  Answer Sheet  4.23
7.2 Angular Momentum of a Rotating
5. Kinematics of Rotation 5.1–5.17 Rigid Body 6.28
1. Introduction 5.1 7.3 Angular momentum in combined
2. Kinematics of Pure Rotation 5.1 Rotation and Translation 6.29
• Your Turn  5.2 • Your Turn  6.30
3. Kinematics of Rotation Plus Translation 5.3 8. Torque and angular momentum 6.30
• Your Turn  5.4 8.1 Angular Impulse – Momentum
theorem 6.31
3.1 Instantaneous Centre of Rotation 5.4
• Your Turn  6.33
• Your Turn  5.5
9. Conservation of Angular Momentum 6.33
3.2 Rolling 5.5
• Your Turn  6.35
• Your Turn  5.9
10. Collision of a Particle with a Rigid Body 6.36
■ Miscellaneous Examples  5.10
• Your Turn  6.39
  Worksheet 1  5.13
■ Miscellaneous Examples  6.39
  Worksheet 2  5.14
  Worksheet 1  6.54
  Worksheet 3  5.16
  Worksheet 2  6.60
  Answer Sheet  5.17
Contents  xv

  Worksheet 3  6.64 10.2 Irrotational flow 8.26


  Answer Sheet  6.69 11. Equation of Continutiy 8.26
• Your Turn  8.27
7. Miscellaneous Problems on
12. Bernoulli’s Equation 8.27
Chapter 4 to 6 7.1–7.9
12.1 Pressure as a form of energy 8.27
Match the Columns 7.1
13. Applications of Bernoulli’s Equation 8.28
Passage-based Problems 7.2
13.1 Speed of efflux 8.28
  Answer Sheet  7.9
13.2 Siphon 8.29
8. Fluid Mechanics 8.1–8.56 13.3 Venturi meter 8.30
1. Introduction 8.1 13.4 Flight of aeroplanes 8.30
2. Density and Relative Density 8.1 13.5 Magnus effect 8.31
• Your Turn  8.2 13.6 A Sprayer 8.31
3. Pressure in a Fluid 8.2 • Your Turn  8.34
3.1 Atmospheric Pressure 8.3 ■ Miscellaneous Examples  8.35

3.2 Variation of pressure in a static fluid 8.3   Worksheet 1  8.44


• Your Turn  8.4   Worksheet 2  8.49
3.3 Pressure in a Gas 8.4   Worksheet 3  8.52
4. Force on container surface 8.4   Answer Sheet  8.56
4.1 Force on Horizontal Base 8.4
9. Surface Tension and Viscosity 9.1–9.30
4.2 Force on a Vertical Wall 8.5
1. Introduction 9.1
4.3 Force on a Curved Surface 8.6
2. Tension in a Liquid Surface 9.1
• Your Turn  8.7
2.1 Demonstration of Surface Tension 9.2
5. Barometer 8.8
• Your Turn  9.3
• Your Turn  8.8
3. Tendency of a Liquid Surface to Contract 9.3
6. Manometer 8.8
• Your Turn  9.4
• Your Turn  8.10
4. Microscopic Description of Surface Tension 9.4
7. Pascal’s Law 8.10
5. Surface Energy 9.4
7.1 Hydraulic Lift 8.10
5.1 Surface Tension as Surface Energy 9.5
• Your Turn  8.12
• Your Turn  9.6
8. Fluid in Accelerated Container 8.12
6. Pressure Difference on two Sides of a
8.1 Variation of Pressure in a Liquid in Curved Liquid Surface 9.6
Horizontally Accelerated Container 8.13
6.1 Excess Pressure inside a Spherical
• Your Turn  8.16 Liquid Drop 9.7
9. Buoyancy 8.17 6.2 Excess Pressure inside a Soap Bubble 9.7
9.1 Floatation 8.18 6.3 Air Bubble 9.7
9.2 Apparent Weight 8.18 6.4 Water Jet from a Faucet 9.8
9.3 Hydrometer 8.18 • Your Turn  9.9
9.4 Buoyancy in fluid kept in Accelerated 7. Cohesive, Adhesive Forces and Contact Angle 9.9
Container 8.19
7.1 Shape of Meniscus 9.11
• Your Turn  8.22
• Your Turn  9.11
9.5 Centre of Buoyancy 8.23
8. Capillary Rise 9.11
• Your Turn  8.24
8.1 Tube of Insufficient Length 9.12
10. Flow of Fluids 8.25
8.2 Inclined Tube 9.13
10.1 Steady and Turbulent Flow 8.25
xvi  Mechanics II

8.3 Obtuse Contact Angle 9.13 7. Gravitational Potential Energy 11.12


8.4 Capillary Rise in Practical Life 9.13 7.1 Gravitational force is conservative 11.12
• Your Turn  9.15 7.2 Gravitational Potential Energy of two
9. Detergents 9.15 point-masses 11.13
10. Viscosity 9.16 7.3 Gravitational PE of a system of
multiple particles 11.14
10.1 Viscous Force 9.16
7.4 Spherical bodies 11.14
• Your Turn  9.17
• Your Turn  11.16
11. Stokes’ Law 9.17
7.5 Escape Speed 11.16
12. Terminal Speed 9.17
7.6 black holes 11.17
• Your Turn  9.18
• Your Turn  11.17
13. Reynolds’ Number 9.18
8. Gravitational Potential 11.18
■ Miscellaneous Examples  9.19

8.1 Potential due to a point mass 11.18


  Worksheet 1  9.24
8.2 Potential due to a uniform thin
  Worksheet 2  9.27
spherical shell 11.18
  Worksheet 3  9.28
8.3 Potential due to a solid uniform
  Answer Sheet  9.30 sphere 11.18
10. Miscellaneous problems on • Your Turn  11.20
chapters 8 and 9 10.1–10.8 9. Satellite 11.21
Match the Columns 10.1 9.1 Ellipitical Orbit 11.22
Passage-based Problems 10.2 • Your Turn  11.24
  Answer Sheet  10.8 9.2 Geostationary Satellites 11.25
10. Planets and Kepler’s Laws 11.25
11. Gravitation 11.1–11.43 • Your Turn  11.28
1. Introduction 11.1 11. Mars orbiter Mission (Mangalyaan) 11.28
2. Newton’s Law of Gravitation 11.1 • Your Turn  11.29
• Your Turn  11.3 12. Binary stars 11.29
3. Measurement of Gravitational Constant G 11.3 ■ Miscellaneous Examples  11.30

4. Principle of Superposition 11.4   Worksheet 1  11.36


• Your Turn  11.5   Worksheet 2  11.39
5. Gravitational Field 11.6   Worksheet 3  11.40
• Your Turn  11.7   Answer Sheet  11.42
5.1 Gravitational field due to spherical
bodies 11.8 12. Elasticity 12.1–12.16
• Your Turn  11.9 1. Introduction 12.1
6. Acceleration Due to Gravity (g) 11.9 2. Microscopic Reason for Elasticity 12.1
6.1 ‘‘Weighing’’ The Earth 11.9 3. Stress and Strain 12.2
6.2 Variation in ‘g’ on the surface of Earth 11.9 3.1 Longitudinal Stress and Strain 12.2
• Your Turn  11.10 3.2 Shear Stress and Strain 12.2
6.3 Acceleration due to gravity at a 3.3 Volume Stress and Strain 12.2
height from the surface of the Earth 11.10 • Your Turn  12.3
6.4 Acceleration due to gravity inside 4. Hooke’s Law 12.3
the Earth 11.11 4.1 Young’s Modulus (Y) 12.3
• Your Turn  11.12 • Your Turn  12.5
Contents  xvii

4.2 Shear Modulus or Modulus of 14. Past Years’ Questions 14.1–14.23


Rigidity (h) 12.5
Centre of Mass and Momentum 14.1
• Your Turn  12.5
AIEEE/JEE Main Questions 14.1
4.3 Bulk Modulus (B) 12.6
IIT JEE (Advanced) Questions 14.3
• Your Turn  12.6
Rotational Motion 14.5
5. Measurment of Young’s Modulus 12.6
AIEEE/JEE Main Questions 14.5
6. Poisson’s Ratio 12.7
IIT JEE (Advanced) Questions 14.7
7. Longitudinal Stress–Strain Curve for
Fluid Mechanics 14.14
a Metal Wire 12.7
AIEEE/JEE Main Questions 14.14
8. Longitudinal Stress–Strain Curve for
Rubber 12.8 IIT JEE (Advanced) Questions 14.15
9. Elastic Potential Energy 12.8 Surface Tension and Viscosity 14.17
• Your Turn  12.9 AIEEE/JEE Main Questions 14.17
■ Miscellaneous Examples  12.9 IIT JEE (Advanced) Questions 14.18
  Worksheet 1  12.12 Gravitation 14.19
  Worksheet 2  12.14 AIEEE/JEE Main Questions 14.19
  Worksheet 3  12.15 IIT JEE (Advanced) Questions 14.20
  Answer Sheet  12.16 Elasticity 14.21
AIEEE/JEE Main Questions 14.21
13. Miscellaneous Problems based on IIT JEE (Advanced) Questions 14.22
chapters 11 and 12 13.1–13.5
  Answer Sheet  14.23
Match the Columns 13.1
Passage-based Problems 13.2 Solutions S.1–S.165
  Answer Sheet  13.5
Chapter  1

Centre of Mass
“Your centre of mass is a place you cannot visit but you always carry with you. Like memories, it is part
of life’s baggage.’’
–Neil deGrasse Tyson

1.  Introduction to predict the motion of the centre of mass of the system. It
will be of immense use in the chapters to follow.
Think of a stick tossed in air. It spins while going up and
falling down. Motion of a point, like end A, is not along a
parabolic path. Moreover, paths taken by different points,
2.  Importance of centre of mass
like the paths of ends A and B, have different geometry. The centre of mass (COM) of a system of particles moves
Because every part of the stick moves differently and have as if the entire mass were concentrated there and all exter-
different speeds, we cannot represent the stick as a particle. nal forces were applied there. When a stick is tossed in air,
In fact, the stick is a collection of interacting particles all of each particle experiences its weight (m1g, m2g, m3g...) as an
which are moving differently. A particle at the end A does not external force. But, acceleration of particle of mass m1 is
move with an acceleration equal to g because it experiences not g, because it experiences forces applied by neighbour-
force due to neigh- ing particles also.
bouring particles,
apart from the grav-
ity. However, the
stick has one special
point – its centre of
mass – that does
move on a parabolic
path with accelera- The special point in the stick whose acceleration is
tion g, just like a decided solely by the external forces is called its COM.
particle projectile.
If mass of the stick is M = m1 + m2 + m3 + ... , then net
We will learn by the end of chapter on ‘Rotational motion’ external force on it is
that the motion of the stick can be described as combina-
tion of two motions – translation and rotation. We think Fext = m1g + m2g + m3g + ...
that the entire mass of the stick is concentrated in the form
= Mg
of a particle kept at its centre of mass. This special point -
Centre of mass - of the stick moves on a parabolic path like Acceleration of COM is
a projectile. All other points rotate about the Centre of mass.
Knowing the motion of centre of mass and superimposing Fext
acm = ____
​     
​= g
a rotational motion about the centre of mass, helps us in M
describing the most complex kind of motions of extended For describing translational motion of the stick, we
objects. consider its entire mass at its COM and the sum of all
In this chapter, we will learn the procedure of locating the external forces (m1g + m2g + m3g ...) acting at that point.
centre of mass of a system of particles and also learn how Without going into the details of internal interactions
amongst the particles, we can predict the acceleration of
COM by knowing the external forces only.
1.2  Mechanics II

3.  LOCATION OF CENTRE OF MASS If the position of COM is chosen as the origin,
n
then ​S  ​ ​   ​mi xi = 0
3.1  Two Particles i = 1

The given figure shows two particles of masses m1 and 3.3  Three-Dimensional Distribution of particles
m2, placed on the x-axis, When particles in a system are scattered in a three-dimen-
at co-ordinates x1 and x2 sional space, then the position of COM is defined by three
respectively. The position co-ordinates, given as:
of com for the system of
1 n 1 n 1 n
two particles is given by xcm = ​ __  ​ ​S  ​ ​ mi xi ; ycm = __
​   ​  ​S  ​ ​   ​mi yi ; zcm = __
​    ​ ​S  ​ ​ mi zi ...(3)
M i = 1 M i = 1 M i = 1
m1x1 + m2x2 m1x1 + m2x2
xcm = ​ __________
    ​
  = ​ __________
      
​ ...(1) We can also write the above relations in terms of a single
m1 + m2 M
vector relation. The position vector of the COM of a system
Where, M = m1 + m2 is the total mass of the system. The of particles is given by
COM of a two-particle system lies somewhere between them ​_› ​_› ​_›
​_› m1 ​r ​ 1  + m2 ​r ​ 2  + ... + mn ​r ​ n 
on the line joining the two. ​r ​ cm
  = ​  _____________________
   
    ​
M
r1 r2
1 n _
​›
x = ​ __  ​ ​S  ​ ​   ​mi ​r ​  i ...(4)
m1 COM m2 M i = 1
r
y
If the origin is chosen at the position of mass m1, then
m2
x co-ordinate of COM is the distance of the COM from the m1
particle of mass m1. Let us assume this distance as r1 and r2
r1
let the distance between the two particles be r.
CM
m1x1 + m2x2 rCM m3
xcm = ___________
​     ​  r3
m1 + m2 O x
0 + m2r m2r
or r1 = ​ _______  ​ = _______
  ​    
 ​ z
m1 + m2 m1 + m2
_
​› _ ​› _
​›
Distance of the COM from m2 is Where, ​r ​  1, ​r ​  2 ... ​r ​  n are position vectors of particles of
masses m1, m2 ... mn, respectively.
m1r
r2 = r – r1 = ​ _______
    ​ ​_›
m1 + m2 ​r ​ i  = xi  + yi  + zi 
​_›
r1 m2 and ​r ​  cm = xcm  + ycm  + zcm 
It is easy to see that __
​ r  ​  = ___
​ m  ​ 
2 1
If we choose the COM of the system as the origin,
fi m1r1 = m2r2 ...(2) then
The last equation leads us to conclude that the COM of xcm = ycm = zcm = 0
the system will be closer to the heavier mass.
fi S mi xi = S mi yi = S mi zi = 0
3.2  Many particles on a straight line
Particles of masses m1, m2, m3... mn are located on the x-axis Example 1   A particle of mass m1 = 1 kg is located at
at co-ordinates x1, x2, x3, ... xn respectively. The COM of (3, 0) m and another particles of mass m2 = 2 kg is located
this collection of particles lies on x-axis with x co-ordinate at (0, 4) m in xy-plane. Find the distance of the COM of the
of COM given by two particle-system from the particle of mass m1.

x4 x3
Solution
x1 x2 x5
m4 m3 m1 m2 m5
x Concepts
O
(i) The COM will lie on the line joining the two
m1x1 + m2x2 + ... + mn xn particles.
xcm = ______________________
​           ​
m1 + m2 + ... + mn (ii) Distance of COM from m1 is given by
1 n
= __ ​   ​ ​ m x   where, M is total mass of the system.
​     ​ S
M i = 1 i  i
Centre of Mass  1.3

1 × 3 + 4 (– 3) + 2 × 5 + 3 (– 5)
m2r  ​ = ​ ___________________________
         ​
r1 = ​ _______
   ​, 1+4+2+3
m1 + m2
14
where r = distance between the particles = – ​ ___ ​  m = – 1.4 m
10
Distance between the particles is m1y1 + m2y2 + m3y3 + m4y4
______ ycm = ________________________
​        
m1 + m2 + m3 + m4
r = ​÷3  2 + 42 ​ 
= 5 m
Distance of COM from m1 is 1 × 4 + 4 × 4 + 2 (– 4) + 3 (– 4)
 ​ = ​ ___________________________
      ​   
m2r 2×5 10 10
r1 = ​ _______
   ​ = ​ _____ 
 ​ = ___
​   ​  m = 0
m1 + m2 1 + 2 3
y 1 × 3 + 4 (– 3) + 2 × 5 + 3 (– 5) + 1 (0)
(ii) xcm = ​ _________________________________
          ​
1+4+2+3+1
14
m1 = – ​ ___ ​  m
r 11
r1 1 × 4 + 4 × 4 + 2 (– 4) + 3 (– 4) + 1(0)
3
CM ycm = ​ _________________________________
      ​    = 0
11
x
O 4 1×0+ 4 × 0 + 2 × 0 + 3 × 0 + 1 × 4
m2
zcm = ​ __________________________________
      ​    
11
4
Example 2   Four particles are located in xy plane at = ___
​    ​  m
co-ordinates indicated in the figure. The co-ordinates are in 11
metres and the masses are given as follows. Note:  For solving second part of the problem, one can
m1 = 1 kg,  m2 = 4 kg replace the four masses, m1, m2, m3 and m4, with a point
mass, equal to m1+ m2 + m3 + m4 = 10 kg, placed at the
m3 = 2 kg,  m4 = 3 kg
COM of the system, at (– 1.4, 0) m.
Now, the problem has only two point masses:
M = 10 kg, placed at (– 1.4, 0, 0) m
and m5 = 1 kg, placed at (0, 0, 4) m
10 × (– 1.4) + 1 × 0 – 14
\ xcm = __________________
​   ​      = ​ ____ ​ m
11 11
10 × 0 + 1 × 0
ycm = ______________
​   ​
     =0
11
10 × 0 + 1 × 4 ___ 4
zcm = ​ ______________
 ​
     = ​    ​ m
11 11
(i) Find the co-ordinates of the COM of the system.
(ii) A fifth particle of mass m5 = 1 kg is added to the
system and is placed at (0, 0, 4) m.
– 14
Co-ordinates of COM are ​ ____ ( 
​   ​ 
11
4
, 0, ___ )
​    ​  ​ m.
11
Find the co-ordinates of COM of the collection of Example 3   The centre of mass of a system of four par-
five particles. ticles is at the origin of the co-ordinate system. Masses of
the particles are 1g, 2g, 3g and 4g. The particles of mass
Solution 1g and 3g are displaced by 2 cm and 1 cm respectively in
Concepts positive x-direction. The particle of mass 2g is moved in
negative x-direction by 3 cm. Find the co-ordinates of COM
(i) The COM of the four-particle system will lie in
of the system in the new arrangement.
xy plane and equation (3) will be used to find its
co-ordinates – xcm and ycm. Solution
(ii) After addition of the fifth particle, the position of Concepts
the COM can be defined by finding xcm, ycm and m1 1 + m2x2 + m3x3 + m4x4
xcm = ________________________
​         ​
zcm. M
m1 x1 + m2 x2 + m3 x3 + m4 x4 m1D x1 + m2 D x2 + m3 D x3 + m4 D x4
(i) xcm = _______________________
​     
    D xcm = ______________________________
​           ​
m1 + m2 + m3 + m4 M
1.4  Mechanics II

D x1 = 2 cm ; D x2 = 1 cm ; D x3 = – 3 cm ; D x4 = 0 Solution


Change in x co-ordinate of COM is Concepts
1 × 2 + 2 × 1 + 3 × (– 3) + 4 × 0 _
​› 1 _
​› _
​›
D xcm = ​ ______________________________
         ​ (i) ​r ​  cm = ​ __  ​ (m1 ​r ​  1 + m2 ​r ​  2)
1+2+3+4 M
​_› ​_›
– 5 (ii) The required vector is ​r ​  cm – ​r ​  1
\ D xcm = ___ ​   ​ = – 0.5 cm
10
​_› m ( + + ) + 2m ( – – )
x co-ordinate of COM was zero before movement of the ​r ​ cm __________________________
  = ​     
 ​   
particles. It changed by D xcm = – 0.5 cm. 3m
\  New x co-ordinate is – 0.5 cm. 1 1
= – __
​   ​  – __
​   ​ 
There is no change in y and z co-ordinates. 3 3
m
\  New co-ordinates of COM are (– 0.5 cm, 0, 0). Vector connecting m to
com is R
r1
Example 4   A particle of mass m has position vector ​_› _
​› _
​› CM
_
​› ​R  ​   = ​r ​  cm – ​r ​  1 r CM
​r ​  1 = +
​_›
+ and another particle of mass 2 m has position
O 2m
vector ​r ​  2 = – – . Write the vector obtained by joining 4 4 r2
= – ​ __ ​  – __
​   ​ 
mass m to the COM of the system. 3 3

Your Turn
Q.1  Two stars, having mass m and 4 m, are separated by displaced in opposite directions, along the line joining them,
a distance d. Both of them are moving in circles about the by 6 cm and 2 cm respectively.
com of the system with equal angular speed. Find the radius
of the circular path of the star of mass m. Assume the stars
Q.4  Particles of masses m1 = 1 kg, m2 = 2 kg and m3 = 3 kg
are placed on x-axis, at x1 = 4 cm, x2 = 6 cm and x3 = – 1 cm
to be point masses.
respectively. Where shall we place a fourth particle of mass
Q.2  Three masses have A m m4 = 2 kg so that the COM of the system of four particles
been placed on the vertices is at
of an equilateral triangle of (i) the origin
side length a = 10 m (see
(ii) the location of fourth particle itself.
figure). You are standing at
vertex B. How far must you Q.5  Particles of mass m1 = 1 kg, m2 = 2 kg and
​_
m3 = 3 kg

travel to reach the COM of 2m 4m are placed at points having position vectors ​r ​ 1  = ( + 4
​_› ​_›
the three-particle system? B C + ) m, ​r ​ 2  = ( + + ) m and ​r ​ 3  = (2  – – 2  )m respec-
tively. Find the position vector of the COM of the system.
Q.3  Find the displacement of the COM of a system of
two particles, having masses m and 3 m, if the particles are

3.4  Centre of Mass of groups of particles the COM of the two-particle system, comprising masses
M1 and M2.
Consider a group of particles having masses m1, m2 and m3.
The COM of this group is at C1. Another group, comprising One can prove this as follows.
​_› ​_› ​_› ​_› ​_›
particles of masses m4 and m5 has its COM at C2. ​_› m1 ​r ​  1 + m2 ​r ​  2  + m3 ​r ​  3+ m4 ​r ​  4 + m5 ​r ​  5
________________________________
​r ​  cm = ​            ​
If you are asked to find the COM of the collection of m1 + m2 + m3 + m4 + m5

( 
​_› ​_› ​_›

)
five particles, you can m1 ​r ​  1 + m2 ​r ​  2 + m3 ​r ​  3
replace the first group m3
m2 ​ ____________________
 ​        
 ​    ​ (m1 + m2 + m3)
C1
m1 + m2 + m3
by a particle of mass

( 
​_› ​_›

)
m1
M1 = m1 + m2 + m3, m4 ​r ​  4 + m5 ​r ​  5
____________
placed at C1 and the    + ​ ​    
   ​  ​  (m4 + m5)
m4 + m5
second group can be =
replaced by a particle m5
m1 + m2 + m3 + m4 + m5
of mass M2 = m4 + m5, m4 _
​› _
​›
C2 M1 ​​r ​​  ​c​ ​​ + M2 ​​r ​​  ​c​ ​​
placed at C2. Now find = _____________
1 2
​    
    ​ ...(5)
M1 + M2
Centre of Mass  1.5

3.5  Centre of mass of continuous bodies Length of the element = R dq


An ordinary object, such as this book, contains countless If l (kg m–1) is the linear mass density of the ring then
number of particles. The summations in equation (3) are mass of the element is dm = l ◊ Rdq
performed with the help of integration. Co-ordinates of the element are (R cos q , R sin q).
A very small element of mass dm can be regarded as a Remember that element is of infinitesionally small size and
point mass and the formulae for co-ordinates of COM can can be treated like a point mass. Mass of the half ring,
be given as follows: M = l  ◊ p R.
1
Now ycm = __ ​    ​ Ú y dm
1 1 1 M
xcm = ​ __  ​ Ú x dm, ycm = __
​    ​   Ú y dm, zcm = __
​    ​  Ú  z dm ...(6)
M M M q = p
1
where (x, y, z) are co-ordinates of elemental mass dm. The    ​ ​Ú  ​ ​ ​(R sin q) (l Rd q)
= ​ _____
l p R q = 0
examples to follow will clarify the procedure.
[As q changes from 0 to p, all elements in the ring gets
3.5.1  Use of Symmetry covered]
p
l R2 R
We need not perform all the three integrals if a body has a \ _____
ycm = ​   ​ Ú​   ​ ​sin q dq = – ​ __
  p
p ​  [cos q​]0​ ​ ​
l p R 0
point, a line or a plane of symmetry. The COM of such a
body always falls on that point, on that line or on that plane. R R 2R
A uniform ball or a uniform cube has its geometrical centre = – ​ __ __ ___
p ​  [cos p – cos 0] = – ​ p ​  [– 1 – 1] = ​  p ​ 
as the point of symmetry. the COM of a uniform ball or a
2R
uniform cube is at its geometrical centre. Similarly, the COM Hence, COM is at a distance of ___
​  p ​ from O on y-axis.
of a uniform thin rod is at its geometrical centre. The axis of
a cone is a line of symmetry and its COM lies on its axis.
Example 6   Uniform semicircular disc
It is also important to note that the COM of an object
A semicircular disc has radius R. The disc is uniform. Locate
may lie outside the material. The COM of a ring is at its
its COM.
geometrical centre, where there is no material.
Solution
Example 5   COM of a uniform semicircular ring
Concepts
A uniform semicircular ring has radius R. Find the position
of COM of the ring. (i) In the figure shown, y-axis is the line of symmetry.
COM lies on this line. It means xcm = 0.
Solution
Concepts (ii) We will divide the disc into countless number of
ring elements. Each of these ring elements can be
(i) The mass is distributed in a plane. In general, we replaced with a point mass, placed at the respec-
will need two co-ordinates – xcm and ycm – to fix tive COMs (refer to article 3.4). The system now
the position of COM. reduces to a collection of large number of particles,
(ii) In the figure shown, y-axis is a line of symmetry. distributed along the y-axis.
Distribution of mass to the left and right of y-axis is
identical. The COM lies on y-axis. Hence, xcm = 0. Consider the co-ordinate axes as shown. We need to find
We need to find ycm only. ycm.
1
(iii) ycm = __
​    ​ Ú y dm Consider a ring element of radius r having infinitesi-
M mally small thickness
dr. Many such rings,
ABC is a half-ring of radius R. We choose the co-ordinate laid one after another,
axes as shown. Such y will make the disc.
choice of co-ordinate x = R cos q Let s (kg m–2) be
axes is intelligent, as B y = R sin q the surface density
y-axis happens to be of mass for the disc
the symmetry-axis (i.e., mass per unit
R
and xcm = 0. area of the disc).
dq
Consider a small y Circumference of the ring element = p r. Area of the ring
element subtending q
A O x C
x element (the strip shown in dark colour)
an angle d q at the
centre O. dA = p r ◊ dr
1.6  Mechanics II

mass of the ring element, dm = s dA = sp rdr A craftsman makes a doll by


y co-ordinate of COM of the ring element is joining a solid sphere to a cone,
2r as shown [Later he painted vari-
y = __
​ p ​  ous features like eyes, nose, etc.,
The ring element can be replaced by a point mass dm on the object]. The axis of the
2r cone coincides with the diameter
placed on the y-axis at y = __
​ p ​ . of the ball. Mass of the uniform
s p R2 ball is 2 m and that of the cone
Total mass of disc, M = ​ ______  ​ 
  is 3 m. Radius and height of the
2
cone are R and 3R respectively.
Ú​  ​ ​ y dm Find the position of the COM of
\ ycm = ​ _____  

   
M the doll.
R Solution
1
= ​ ______
 2  ​ ​
s p R
_____ r = 0
2r
 Ú   ​ ​  __ (  )
​ p ​  ​ (s p r dr)
Concepts
​   ​   
2 (i) Refer to Article 3.4. We can replace the ball with
R a point mass 2 m placed at its COM and the cone
4 4 ___ R3 4R
 2 ​ ​Ú ​  ​ r2 dr = ____
= ​ ____ ◊ ​   ​ = ___
​   2 ​   ​   ​  can be replaced with a point mass 3 m kept at its
p R 0 p R 3 3p
COM.
Note:  It is recommended to remember the position of the (ii) To find COM of the resulting two-particle system.
COM for the following objects. we will use
m1 x1 + m2 x2
1. COM of a uniform triangular plate is at its xcm = ​ ___________
   ​ 
centroid. m1 + m2
2. COM of a uniform solid cone of height h is on its
3h Consider O as the origin and the axis of the cone as
symmetry axis, at a distance ___ ​   ​ from the apex. x-axis. x-axis happens to be the line of symmetry and the
4
COM of the doll will lie on this line. We consider a point
3. COM of a uniform solid hemisphere is on its sym-
mass 2 m at the centre (C1) of the sphere. x co-ordinate of
3R
metry axis, at a distance ___ ​   ​ from the centre of its this mass is x1 = 4R.
8
base circle.
4. COM of a uniform hemispherical shell is on its sym-
R
metry axis, at a distance __
​   ​  from the centre of its base
2
circle.
One who is interested may try to prove these results by
use of integration.

CM
h CM
3h 3R R
4 8
O
Solid cone Solid hemisphere Point mass 3 m is assumed at the COM of the cone
(at C2). x co-ordinate of this mass is
1 3R
x2 = __
​   ​  (3R) = ___
​   ​ 
CM 4 4
R R x co-ordinate of the COM of the doll is
2
O 2M x1 + 3M x2
xcm = ​ ____________
  
  
  ​
Hemispherical shell 2M + 3M

Example 7   COM of combination of known geometrical


3R
2M (4R) + 3M ​ ___
​   ​  ​
4
= ​ ________________
       ​ =
(  ) 41R
____
​   ​  = 2.05 R
shapes 5M 20
Centre of Mass  1.7

A B
Example 8   A piece of paper
has been cut in the shape shown
in figure. ABCD is a square and E
BEC is an equilateral triangle. O
R/2
Find the position of COM of R
D C
the paper piece.
Solution
Concepts Solution
(i) The COM of the square piece will be at its geo- Concepts
metrical centre. The COM of the triangle will be (i) Mass of the sphere will be proportional to its
at its centroid. volume. If volume of the removed sphere of
(ii) We can replace the square and triangle with point R
radius ​ __ ​  is v, then volume of the original sphere
masses at their respective COMs. 2
of radius R will be 8v ( v µ r3 fi If radius is
(iii) A line through E and centre of the square is
doubled, volume becomes 8 times). Volume of
symmetry axis.
material in the given object will be 7v. If the mass
Let side length of A B of removed part is M, then mass of remaining part
the square be a and s is 7M.
(kg m–2) be the mass O
C1 C2
x (ii) Line AB shown in figure is the line of symmetry
of paper per unit area. F E for the given object. Its COM will lie on line AB.
Mass of the square Because mass to the left of O is more than the
D C mass to the right of O, the COM will be to the
m1 = s ◊ a2 a
right of O, say at C1.
mass of the triangle (iii) For the purpose of calculation of COM, an extended

(  )
__ __ body can be replaced with a point mass placed at
1   
​÷3 ​   
​÷3 ​
m2 = s __
​   ​  (a) ​ ___
​   ​ a  ​ = s ___
​   ​ a2 its COM.
2 2 4
The x-axis shown in the figure is the line of symmetry
and the COM lies on this line. With O as the origin, the x
co-ordinate of COM of the square (at C1) is
C1 C2
A B
a
x1 = ​ __ ​  O
2
COM of the triangle (at C2) has co-ordinate

[ 
__

a
x2 = a + ____
​  __
   ​ ​
  
2​÷3 ​

1
  FC2 = __
1 ÷
​   ​  FE = __
3
  
​ 3 ​
​   ​   ◊ ​ ___ ​ a  ​
3 2 ] A 7M

x
O

R
M B

Now, we think as if point masses m1 and m2 are at C1 2


and C2 respectively. Mass of removed part is M. Mass of the given object
m1x1 + m2x2 with cavity is 7M. Imagine that the cavity is filled back to
\ xcm = ​ __________
   ​  complete the sphere. We can consider the sphere to be made
m1 + m2
R
up of two objects – (i) a sphere of radius ​ __ ​ , having mass
(  )( 
__
a
(s a ) ​ __
2
2 (  ) ​÷3 ​
___________________________ 4
  
2​
1__
​   ​   ​ + ​ s ​ ___ ​ a2  ​​ 1+ ​ ____
÷
   ​  ​ a
  
3 ​ ) 2
M and (ii) a sphere of radius R, with a cavity, having mass
= ​          
__  ​ 0.74 a
7M.
÷   
​ 3 ​
s a2 + s ​ ___ ​  a2 The two masses – M and 7M – can be replaced with point
4
Example 9   COM of an object with a cavity
masses placed at their respective COMs, at C2 and C1. The
R system of these two masses has its COM at O.
A spherical cavity of radius __ ​   ​  is carved out from a uniform
2 From equation (2) in article 3.1 we can write
sphere of radius R. The cavity has its boundary passing R
through the centre of the sphere, as shown in figure. Locate 7M ◊ x = M ◊ ​ __ ​ 
2
the COM of the sphere with cavity. R
___
fi x = ​    ​ 
14
1.8  Mechanics II

Note:  A body has its COM (3) For a body having continuous mass distribution, the
at O. The shaded part of process of summation is done by integration.
the body has its COM at C1
O 1 1
C2. Now, the shaded part is xcm = __
​     ​ Ú x dm ; ycm = __
​    ​ Ú y dm ;
C2 M M
removed. The COM of the
1
remaining object is at C1. zcm = ​ __  ​ Ú z dm
M
(i) C1, O and C2 lie on
a straight line. (4) For a two-particle system, m1r1 = m2r2.
(ii) If mass of the removed part is m and mass of the Here, r1 and r2 are distances of the two particles (hav-
remaining object is M, then ing masses m1 and m2 respectively) from the COM
of the system.
m (OC2) = M (OC1).
(5) If the x co-ordinate of individual particles in a system
In short: are changed by D x1, D x2, D x3 ... D xn, then the change
(1) COM of a body (or collection of particles) is a point, in x co-ordinates of the COM of the system is
whose motion can be predicted by knowing the exter-
1  n
nal forces, only without worrying about the interac- ​     ​ ​S  ​ ​ ​mi D xi
D xcm = __
M i = 1
tion of the particles amongst themselves.
Similar equation can be written for change in y and
(2) For a discrete particle system, the COM is located
z co-ordinates.
by finding its co-ordinates as
(6) For the purpose of calculation of position of COM,
1 1
​    ​ S mi xi ; ycm = __
xcm = __ ​    ​ S miyi ; a part of a body (or a group of particles) can be
M M
replaced with a point mass, kept at its COM.
1
​    ​ S mizi
zcm = __ (7) It is useful to identify the line of symmetry (or a
M
plane of symmetry) while finding the position of the
where is the total mass of the system. COM of a body. The COM lies on the line (or plane)
of symmetry.

Your Turn
Q.6  A rod of length A B
L has its linear mass
density changing with distance (x) from one end A as
[  x
]
l = l0 ​ 1 + ​ __  ​  ​. Find the distance of the COM of the rod
L
from end A.

Q.7  Two identical rods are kept perpendicular to each


other, with one of their ends coinciding. Length of each rod
is L. Find the distance of the COM of the system from the
common end of the two rods. Q.10  A uniform metal plate is in the shape of a disc of
R
Q.8  A uniform plate is in radius R. A circular part of radius ​ __ ​ 
2
L shape, with dimensions as is stamped out (i.e., removed) from
shown. Find the co-ordinates the plate, such that the circumfer-
of the COM of the plate. C
ence of the hole passes through the O
R/2
Q.9  A ring is made by join- centre O of the disc. Find the dis-
tance of the COM of the ‘plate with R
ing two semicircular pieces of
radius R. Semicircle ABC has hole’ from O.
mass twice that of the other. Q.11  A uniform wire of length L is used to make a
Find the distance of the COM semicircular closed frame ABC. Find the position of the
of the ring from centre O. COM of the object.
Centre of Mass  1.9

Q.13 A metal wrench of uniform density is cut at the loca-


tion of its COM (at C), as shown in figure. Can we say that
the two pieces have equal mass?
C

Q.12  Where could the COM of the Earth’s atmosphere


be?

4. MOTION OF CENTRE OF MASS 4.2  Acceleration of COM


Differentiating equation (7) with respect to time

( 
4.1  Velocity of COM ​_› ​__› ​_› ​_›

Equation (4) gives the position of the COM for a collection


of particles. If one or more particles move and their position
d ​v  ​cm
​ _____
dt

  
​  = __
M
1 d ​v​ 1 
​    ​ ​ m1 ​ ____   
dt
d ​v  ​2 
​ + m2 ​ ____   
dt
d ​v  n ​
​ + ... + mn ​ ____   
dt )
​  ​

​_› 1 ​_› ​_› ​_›


changes, the position vector of the COM may also change. fi ​a  ​cm __
  = ​    ​  (m1 ​a  ​1  + m2 ​a  ​2  + ... + mn ​a  ​n  ) ...(9)
It means that the COM of the system has a velocity. M
​_› ​_› ​_› ​_› This equation gives the acceleration of a system of
__1
​r ​ cm   = ​    ​  (m1 ​r ​
 1  + m2 ​r ​ 2  + ... + mn ​r ​ n  ) particles, when the accelerations of individual particles are
M
known. Let us write the equation as
Differentiating this equation with respect to time, ​_› ​_› ​_› ​_›

( 
_ _ _ _ M ​a  ​cm = m1 ​a  ​1  + m2 ​a  ​2  + ... + mn ​a  ​n 
)
​› ​› ​› ​›  
d ​r ​  cm __ 1 d ​r ​  1 d ​r ​  2 d ​r ​  n _
​ _____ ​ = ​    ​ ​ m1 ​ ____   
   
  ​ + m2 ​ ____    ​ + ... + mn ​ ____    ​  ​ ​_› ​›
dt M dt dt dt m1 ​a  ​1 
  (= ​F 
 ​1  )​_› is the sum of all forces acting on particle of
_ _ _ _ mass m1. ​F   ​1 is the resultant of external forces acting on
​› 1 ​› ​› ​›
fi ​v  ​ cm = ​ __  ​ (m1 ​v  ​ 1 + m2 ​v  ​ 2 + ... + mn ​v  ​ n) m1 and the forces applied by other particles present in the
M _
​› ​_›
system. Similarly, m2 ​a   ​2 (= ​F   ​2) is resultant force on m2 due
​_› 1 n ​_›
fi ​v  ​cm __ ​   ​ ​   ​ mi ​v  ​i 
  = ​    ​  S ...(7) to external agencies and the other particles in the system
M i = 1 (viz., m1, m3, m4 ...).
The above equation helps us to find the velocity of the ​_› ​_› ​_› ​_›
COM of a system when velocities of its constituent particles \ M ​a  ​cm
  = ​F 
 ​1  + ​F  ​2  + ... + ​F  ​n 
are given. If you have solid objects in your system, you can The right side of the above equation includes all the
replace them with point masses moving with velocities of internal interactions amongst the particles. However, from
their respective COMs. Newton’s third law, these forces occur in pairs (equal and
Let us rearrange the terms in the above equation to write opposite) and their vector sum is zero. So the sum on the
it as right side represents the sum of external forces only.
​_› ​_› ​_› ​_› ​_› ​_›
M ​v  ​cm
  = m1 ​v  ​1  + m2 ​v  ​2  + ... + mn ​v  ​n  \ M ​a  ​CM
  = ​F 
 ​ext
  ...(10)
​_›
The right side of the equation is the sum of the linear Where, ​F  ​ ext
is the sum of all external forces on the sys-
momentum of the constituent particles of the system. It is tem. This is Newton’s second law for a collection of par-
the total momentum of the system. The above equation tells ticles. It states that the acceleration of the COM of a body
you that if you know the velocity of the COM of a system, (or collection of particles) is equal to the net external force
multiply it with the mass of the system (M) to get the total acting on the body divided by its total mass.
momentum of the system. Again, think of the rotating ceiling fan in your room.
​_› _
​› What is the total external force (i.e., sum of forces by the
​P  ​ system = M ​v  ​ cm ...(8) support, force due to air, weight) acting on the fan? The
answer is zero. Acceleration of the COM of the fan is zero
Switch on the ceiling fan in your room. What is momen- (as it never moves even a centimeter) and therefore, we
tum of the fan when it is running at its full speed? The fan conclude that the net external force on it is zero. Even when
has many particles. Do we need to add momentum of all the fan is speeding up (i.e., its angular speed is increasing),
the particles? Not really. The COM of the fan (it is at the its COM has no acceleration and the net external force on
geometrical centre of the fan) is at rest. vcm = 0. Hence, the fan is zero. Can we say that every particle in the fan
momentum of the fan is zero. has zero acceleration? No, a typical particle is going in a
1.10  Mechanics II

​_› _
​›
circle and must be experiencing unbalanced force towards _
​› m D ​r ​
   + m2 ​D 
 ​r  2
D ​r ​  cm = ______________
1  1
the centre. ​     ​
   = 0
m1 + m2
Equation (10) is equivalent to three component equations ​_› ​_›
fi   m1 D ​r ​  1 + m2 D ​r ​  2 = 0
along the three co-ordinate axes.
​_› ​_›
(Fext) x = M (acm) x ; (Fext) y = M (acm) y ; (Fext) z = M (acm) z fi m1 D ​r ​  1 = – m2 D ​r ​  2 ...(12)
The
​_›
negative
​_›
sign indicates that the displacement
...(11)
D ​r ​  1 and D ​r ​  2 must be in opposite directions. If you
Following are some important situations that need your knowingly take the two displacement in opposite
attention. _
​› directions then you need not write the negative sign
(1) An external force F  ​  ​ is applied A F and use the above equation as:
to a body. Irrespective of
P m1D r1 = m2 D r2 ...(13)
its point of application,
the acceleration of the Where D r1 and D r2 are magnitudes of displacements
aCM
C F of the two particles in opposite directions.
COM of_ the body will be
​›
​_› ​F   ​ The above result can be applied in a specific direction
​a   ​cm = ​ __ ​.  A stick of mass
M as well. If (ucm) x = 0 and (Fext) x = 0 [(ucm) y and
M is lying on a smooth B F (Fext) y may or may not be zero] then
​_›
floor. A force ​F  ​  is applied to it. No matter whether D xcm = 0
the force is applied at A, B_ or C, the acceleration of
​›
m1D x1 + m2 D x2 = 0 ...(14)
​F   ​
COM of the stick will be ​ __  ​.  We will learn later that where D x1 and D x2 are changes in x co-ordinates of
M the two masses.
applying force at A will produce a motion of point P,
If we consider the displacement D x1 and D x2 in
which will be very different from the motion of point opposite directions, then
p that we see when the same force is applied at C.
(2) Suppose you find that the COM of a system is at
1D x1 = m2 D x2 ...(15)
rest (i.e., the velocity of the COM, ucm = 0) and (3) The result obtained above can be extended to a
also observe that the net external force on the sys- system having more than two particles.
​_› ​_› ​_›
tem is zero. It implies
If ​u  ​ cm = 0 and ​a  ​ cm = 0 (fi ​F  ​ ext = 0) then
that a cm is zero. ​_› ​_› ​_›
Therefore, the COM m1D ​r ​   + m2D ​r ​  2 + m3D ​r ​  3 + ... = 0
of the system will not
get displaced. It is If (Fext)x = 0 and (ucm) x = 0, then
possible that the com- m1D x1 + m2D x2 + m3D x3 + ... = 0.
ponents of the system
are accelerated due to (4) Consider a diwali bomb, moving like a projectile.
internal interactions. It explodes at point P when moving with a velocity
Consider two charges v (this is the velocity of the COM of the bomb at the
– both positive – instant of explosion). The fragments move along dif-
held certain distance apart. They are released. They ferent directions due to the force of explosion. But for
repel one another and move away from one another the system as a whole, the forces of explosions are
with increasing speed. But the COM of the system, internal and should not affect the motion of the COM.
comprising the two charges, does not move. The COM still has
acceleration equal
When charges were released, each had zero initial
to g (Ø), as the net
velocity.
external force is
\  Initial velocity of the COM was zero (ucm = 0) Mg. The COM of
Fext = 0  fi  acm = 0 the fragments con-
fi  COM does not move. tinues on the origi-
nal parabolic path
How are the displacements of two particles related if
until the fragments
their COM has no motion?
_
​› _
​› begin to fall on the
Let D ​r ​  1 and D ​r ​  2 be the displacements of the two ground.
particles, having masses m1 and m2 respectively. If
the COM has not moved, then
Centre of Mass  1.11

Events like explosion, collision, etc., do not change the _


​› m1 g (Ø) + m2g (Ø)
motion of the COM of the system, as they are phenomena (ii) ​a  ​ cm = ​ _______________
   ​
   = g (Ø)
m1 + m2
of internal interactions amongst the particles.
Example 12   Two blocks of masses m1 and m2 are con-
Example 10   In the arrangement shown in the figure, the nected by an ideal spring, having force constant k. The
horizontal surface is smooth but there is friction between system is placed on a smooth
the two blocks. Coefficient of friction is m. Mass of B is floor, and pushed against a
twice that of A. When a horizontal force F is applied to B, wall with compression in the m1 m2
the two blocks slip on one spring equal to x. System is
another. Find the acceleration released from this position.
of the COM of the system of (i) Find the acceleration of the COM of the system,
two blocks, A and B. Mass immediately after its release.
of A is m.
(ii) Find the acceleration of the COM of the system, after
Solution m1 breaks-off the wall.
Concepts (iii) Find the velocity of the COM of the system, after a
long time.
Acceleration of the COM depends on external force
only. Solution

Friction force between the blocks is an internal interaction Concepts


when we are considering (A + B) as our system. F is the (i) The compressed spring pushes the block of mass
only unbalanced external force on the system. m1 against the wall. The wall applies a normal
force on m1. This is the external force that causes
F F
\ acm = ​ _______
     ​ = ___
​    ​  the COM to accelerate.
m + 2m 3m
(ii) Once m1 leaves contact with the wall, there is no
Example 11   A particle of mass m is projected vertically external force on the system. Now, acm = 0.
up from a point with initial velocity 20 ms–1. Another particle (iii) Velocity of the COM at the instant m1 leaves the
of mass 2m is projected simultaneously from the same point, wall will be constant thereafter, as no external force
with velocity 20 ms–1 at an angle of 30° to the horizontal. acts on the system.
(i) Find the velocity of the COM of the system of two
particles, one second after their projection. (i) Spring force on the
(ii) Final acceleration of the COM of the system, one block of mass m1, at
second after their projection. the instant the sys-
tem is released, is kx.
Solution Normal force exerted
Concepts _
​› _
​› _
​› _
​› by the wall on the block of mass m1 is N = kx
​_› m 1 ​v  ​ 1 + m2 ​v  ​ 2
___________ ​_› m 1 ​a  ​ 1 + m2 ​a  ​ 2
____________ \ For the system, Fext = N = kx
​v  ​ cm = ​      ​   and ​a  ​ cm = ​      ​  
m1 + m2 m1 + m2 Fext kx
\ acm = _______
​      ​ = ​ _______
    ​
(i) Velocity of the first particle after 1 s is m 1 + m2 m 1 + m2

v = 20 – gt = 20 – 10 × 1 = 10 ms–1 (≠) (ii) As long as the spring is compressed, it will keep the
block pressed against the wall. When the block of
Horizontal and vertical components of initial veloc- mass m2 moves to the right by a distance x, the spring
ity of the second particle are (Horizontal–x and attains its natural length. At this instant, the spring
vertical–y-direction) __ does not exert any force on the blocks. The normal
ux = 20 cos 30° = 10 ​÷3 ​     ms–1  and force by wall on m1 becomes zero. The system leaves
uy = 20 sin 30° = 10 ms–1 the wall. After leaving the wall, Fext = 0
Velocity components after 1 s fi a = 0
__ cm
    ms–1 ; vy = 10 – gt = 10 – 10 × 1 = 0
vx = 10 ​÷3 ​ (iii) Let us find velocity of m2 at the instant the spring
​_› ​_› __ regains its natural length. (At this instant, m1 is about
​_› m1 ​v  ​ 1 + m2 ​v  ​ 2 m (10     )
) + 2m (10​÷3 ​
\ ​v  ​cm
  = ​ ____________    ​ =​  ___________________
           ​ to move, its velocity is zero).
m1 + m2 m + 2m
The entire spring energy has got converted into KE
20
= ​ ___
10
( 
​  __  ​  + ​ ___ ​    ​ ms–1
÷   
​ 3 ​ 3 ) of m2.
1.12  Mechanics II

÷ 
___
1 1 k Initial velocity of COM, ucm = 0, as both the insect and
\ ​ __ ​  m2v22 = ​ __ ​  k x2  fi  v2 = ​ ___
​ m   ​ ​   ◊ 
2 2 2 the straw are at rest.
____
m1 (0) + m2v2   2 ​ 
​÷km  ◊ x
acm = 0,  as  Fext = 0
vcm = ​ ___________
   = ​ _______ 
 ​   
 ​
m1 + m2 m1 + m2 This means that the COM will not get displaced. The straw
Velocity of the COM will remain constant at the can be replaced with a point mass, placed at its centre.
above value, as there is no external force on the Let the straw move to the left by D x, when the insect
system after m1 leaves contact with the wall. moves from A to B. Displacement of the insect is (L – D x)
to the right.
Example 13   Force on a rotating rod Using equation 15.
A uniform rod of mass M and
length L is rotating in a horizontal m (L – D x) = M ◊ D x
plane, about a vertical axis pass- mL
fi D x = ______
​     ​
ing through its end O. Find the M+m
horizontal force that the axis must
apply on the rod. Angular speed of
the rod is w.
Solution
Concepts
Force applied by the axis is the only external horizontal
force on the rod. This must be equal to F = M acm, where
acm is the horizontal acceleration of the COM.
L
COM is moving in a circle of radius ​ __ ​  with angular As the insect moves from A to B, the staw moves to the left by
2
speed w. Dx. Displacement of insect is (L – Dx) to the right.
L
acm = w2 ​ __ ​ 
2 Example 15   A projectile is fired at a speed of u = 100 ms–1
M w2 L
\ F = M acm = ______
​   ​    at an angle of 37° above the horizontal. At the highest point,
2
the projectile breaks into two parts of mass ratio 1:4 and
Example 14   Insect on a straw the smaller piece comes to rest. Find the distance from the
A straw of m ass M and launching point to the point where the heavier piece lands.
length L is lying on a Solution
smooth horizontal sur-
Concepts
face. A small insect of
mass m is sitting at the (i) The COM continues to move on the usual parabolic
end A of the straw. The insect crawls to the other end B of path even after the projectile breaks. Total external
the straw. will the straw move? If yes, by what distance. force on the pieces is still the weight and hence,
acceleration of COM is still g (Ø).
Solution
(ii) At the highest point, the two pieces have no verti-
Concepts cal velocity. They will both land on the ground
(i) Frictional force on the insect (applied by the straw) simultaneously; at the same instant, the COM will
causes it to move to the right. Friction applied by also land on the ground.
the insect on the straw is to the left and makes the
straw move to the left. Had there been no breakage, the range of projectile would
have been
(ii) Friction is an internal force for the insect–straw
3 4
system. There is no external force on the system. 2  2 × 1002 × __
​   ​  × __
​   ​ 
2u sin q   cos q ________________
5 5
Hence, the COM of the system does not move. R = ​ ____________
g  ​     
= ​   ​       = 960 m
This can happen if m D x1 = m D x2, where x1 is 10
the displacement of the insect to the right and x2 The COM will land at 960 m distance from the point
is displacement of the straw to the left. of projection. The lighter piece (having mass M) lands at
Centre of Mass  1.13

960 Let the centre of the bigger sphere move a distance x


a distance ​ ____
 ​ 
  = 480 m from the point of projection O.
2 to the right, as the smaller sphere moves to the extreme
The heavier piece (having mass 4M) lands at a distance x left. From the figure, it is easy to see that the centre of the
from O. smaller sphere moves a distance (10R – x) to the left. For
m1x1 + m2x2 no motion of the COM of the system,
Using xcm = ___________
​      ​ 
m1 + m3 m1 D x1 = m2 D x2
M × 480 + 4M ◊ x
fi 960 = ​ _______________
       ​ fi 4M ◊ x = M (10R – x)
5M
fi x = 1080 m fi x = 2R

Example 16   A sphere of Mass M and radius R is held


on the inner wall of a hollow sphere of mass 4m and radius
6R. The bigger sphere is kept on a smooth horizontal table
and the centres of the two spheres lie on a horizontal line.
The system of two spheres is released from this position. The
smaller sphere slides on
the inner smooth wall to
reach the other extreme
position (shown in dot- Example 17   Rod, falling on a smooth table
ted line). What will be A uniform thin rod of length L is standing vertically on a
displacement of the cen- smooth table. Consider the lower end of the rod as the origin
tre of the larger sphere, and vertically upward as the positive y-direction. A slight
as the smaller sphere disturbance causes the lower end to slip on the smooth table
moves from one extreme along positive x-axis and the rod begins to fall.
to another? (i) What is path of the COM of the rod during its
Solution fall?
Concepts (ii) Write the equation of the path followed by a point P

(i) Initial velocity of both spheres = 0. Hence, initial


velocity of the COM is zero. There is no horizontal end of the rod.
(  )
L
on the rod, which is at a distance r ​ π ​ __ ​   ​ from lower
2
force on the system. Hence, Solution
(acm) x = 0. It means (vcm) x = 0, always. Concepts
This implies that the x co-ordinates of the COM There is no friction. It means there is no horizontal force
of the system will not change. on the rod. The COM does not have any horizontal motion.
It moves only in vertical direction.
(ii) When one sphere stops the other must also stop,
since (vcm) x = 0. When KE of the system is zero,
(i) The COM moves along vertical straight line.
its PE must be equal to its initial value. This ensures
that the smaller sphere will be at its initial height (ii) When the rod slides to position AB, its centre is on
when it stops moving. y-axis, at C¢.
1.14  Mechanics II

L
AC ¢ = __
​   ​  In short:
2
(i) Velocity of the COM of a system does not change
P is a point at a distance r from end A. Co-ordinates
if there is no external force on the system. Internal
of point P are:
forces can change the velocities of individual par-

L
(  )
x = C¢P cos q = ​ ​ __ ​  – r  ​ cos q
2
...(i) ticles but they cannot alter ​v  ​ cm.
_
​›

(ii) Momemtum
​_›
of a​_›system is ​_›
its total mass multiplied
with ​v   ​cm. When ​F   ​ext = 0, ​v   ​cm does not change. This
implies that the momentum of the system ​_does ›
not
change (provided it has fixed mass) when ​F   ​ext = 0.
this is law of conservation of momentum.
(iii) Acceleration of COM is decided by the resultant of
all external forces.
​_›
​_› ​F  ​ext
____  
​a  ​cm
  =​    ​ 
M
Point of application of external forces and shape and
size of the system are not important while using the
And y = r sin q ...(ii) above equation.
2 (iv) If (ucm) x = 0 and (Fext) x = 0, then there is no dis-
x 2 y
From (i) and (ii) _______
​    2   ​ + __
​  2 ​  = 1
(  )
L placement of the COM along x-direction.
​​ __
​   ​  – r  ​​ ​ r
2

m1 D x1 + m2 D x2 + m3 D x3 + ... + mn D xn = 0.
This is the trajectory equation of point P and it is an (v) Events like explosion, collision, etc., do not change
equation for an ellipse. the motion of COM.

Your Turn

Q.14  A long boat is moving at constant speed through moving with veloc- u
the water. People on the boat gathered at the front and ran ity u, hits the blocks
together towards the back end of the boat. and sticks. Find the
(a) When the people are running, is the speed of the boat velocity of the COM
greater than it was before? of the system of five blocks before and after the collision.
(b) People stop running when they reach the other end. Q.18  Two identical balls are
what can be said about the speed of the boat after moving with constant speed (v)
they stopped? along x and y axes, as shown in
Q.15  Position vector of a 2.0 g particle changes with time the figure. They collide at the
​_› origin and the ball travelling
as ​r ​ 1  = 3t  + 2t2 . Another​_ particle of mass 3.0 g has its

position vector changing as ​r ​ 2  = – 6t  + 3  , where t is in along x-direction gets devi-
seconds and r is in centimeters. At t = 1.0 s, find: ated by 45°. Find the trajec-
tory of the COM of the system
(i) Velocity of the COM of the system of two particles.
of two balls, before and after
(ii) Acceleration of the COM of the system. collision.
(iii) External force on the system.
Q.19  Block a of mass m is placed at the top of a smooth
Q.16  A uniform rod of length L is held vertically, with inclined surface of block
its lower end on a smooth horizontal table. The rod begins A
B, which has mass 5m and
to fall from this position. Find the distance travelled by the is placed on a smooth hori-
lower end of the rod, by the time it falls completely on the zontal surface. The system h
table. is released from rest. Find
30° B
Q.17  Four identical blocks are placed on a smooth hori- the distance moved by B
zontal surface, stuck to each other. Another identical block,
Centre of Mass  1.15

by the time __
A reaches the bottom. Neglect size of A and 2m 3m
and B – of masses ​ ___ ​  and ​ ___ ​  respectively. Taking the
    m.
take h = 10​÷3 ​ 5 5
point of explosion as the origin and knowing that D t = 3 s
Q.20  Three particles, each later, the position of fragment A is (300, 24, – 48) m. find
of mass m, are located at the the co-ordinates of B at the instant. Take y-direction to be
vertices of an equilateral tri- vertically up.
angle ABC.
(i) Where is the COM of Q.22  A small block is released from the top of a hemi-
the system? spherical metal object. Mass of the block is m and that of the
hemisphere is M. Find the displacement of the hemisphere
(ii) The three particles begin when the block reaches at angular position q. Radius of
to move with constant
​___› ​___› ​___› hemisphere is R and there is no friction.
velocities, along AB​
​   , BC​
​   and CA​
​  .  All of them have
the same speed v. How far will the COM move from
its original position in time D t?
Q.21  A​_ projectile of Mass M moving horizontally with

velocity ​v  ​ 0 = (120 ms–1)   explodes into two fragments A

5. GRAVITATIONAL POTENTIAL ENERGY In which case is the potential energy of the hemisphere
higher?
OF AN EXTENDED BODY
The potential energy of a body
is simply the sum of potential
energies of its constituent par-
ticles. In the figure shown, the
potential energy of the body Solution
can be written as
Concepts
u = m1gy1 + m2gy2 + ... For writing PE, we can assume that the entire mass is
concentrated at its COM.
(m1y1 + m2y2 + ...) g
= ​ _________________
       ​ ◊ M  [M = m1 + m2 + ...]
M 3R
Distance of COM from centre O is ___
​   ​ 
8
\ u = Mg ycm

It means, we can assume the entire body to be a point


mass placed at its COM, for the purpose of writing potential
3R
\ PE in position A is uA = Mg ​ ___ (  )
​   ​  ​
8

energy.
( 
3R
PE in position B is uB = Mg ​ R – ___
8 ) (  )
5R
​   ​  ​ = Mg ​ ___
​   ​  ​
8
Example 18   A uniform solid hemisphere is kept on a
horizontal table in two ways – shown as A and B in figure. \ uB > uA

Your Turn

Q.23  A uniform chain is placed over a fixed sphere of


radius R, as shown. The chain has a length equal to p R and
its mass per unit length is l. Write the potential energy of the
chain, taking the centre of the sphere as reference level.
1.16  Mechanics II

6.  THE CENTRE OF MASS FRAME to each other. For example, two blocks connected with a
spring, two stars moving under mutual gravitational pull,
A reference frame, which is moving with the COM of a H2O molecule, etc.
system is often advantageous to use in solution of problems. In general, the reduced mass of a n-particle system is
We will call such a reference frame as the COM frame. defined as
Velocity of the COM itself is always zero in this frame and
1 1 1 1 1
therefore, the linear momentum of the system in this frame ​ __ ​ = ___ ​    ​ + ___
​    ​ + ___
​    ​ + ... + ___
​ m    ​
​_› m m1 m2 m3 n
[= M ​v  cm
 ​ ] is always equal to zero. For this reason, the COM
frame is also known as ‘‘zero momentum frame’’. Example 19   Two identical billiard balls collide. Just
Consider a two-particle system, having_
​›
particles
_
​›
of masses before collision, they were moving with speeds u and 2u in
m1 and m2, moving with velocities ​v   ​1 and ​v   ​2 respectively. opposite directions. Each ball has mass m.
The velocity of the COM of the system is (i) Find the momentum of each ball in COM frame
_
​› _
​›
_
​› m 1 ​v  ​ 1 + m2 ​v  ​ 2
___________ before collision.
​v  ​ cm = ​      ​  
m1 + m2 (ii) Find the momentum of the system of two balls in
​_›
Consider an observer moving with velocity ​v  ​ cm. He will COM frame after collision.
find that the particle of mass m1 is moving at velocity Solution
​_› _
​› ​_›
​v  ​ 1 cm = ​v  ​ 1 – ​v  ​ cm Concepts
​_› ​_› ​_›
​_› ​_›
​_› m1 ​v  ​1 
+ m2 ​v  ​2  (i) |​P  ​ 1cm| = |​P  ​ 2cm| = m |​v  ​ 12|
= ​v  ​1  – ​ ___________
    ​   (ii) COM frame is zero-momentum frame.
m1 + m2
​_› ​__› ​_›
m2 (​v   ​1 – ​ v​  2) m2 ​v  ​ 12 m ◊ m m
= __________
​      ​ = ​ _______   
 ​ ...(16) (i) m = ______
​    ​ = __
  ​   ​ 
m m
m1 + m2 m1 + m2 m+m 2 2u u
v12 = 3u (= v21)
​_›
3
\ |​P  ​1cm
  | = m v12 = __
​    ​ mu
2
​_›
3
Also |​P   ​2cm| = m v21 = __
​    ​ mu
2
(ii) Momentum of a system is always zero in COM
frame.
​_›
where, ​v  ​ 12 = velocity of particle 1 relative to particle 2. 6.1  Kinetic Energy
Similarly, velocity of particle of mass m2 in reference frame
attached to the COM is Consider a system consisting of particles of masses m1, m2,
​_› ​_› _
​› _ ​› _ ​› _
​›
​_› ​_› ​_› ​_› m 1 ​v  1 ​ + m2 ​v  ​2 
m3 ... mn and velocities ​v  ​ 1, ​v  ​ 2, ​v  ​ 3 ... ​v  ​ n respectively. Let the
​v  2 cm ​ = ​v  2 ​ – ​v  cm
 ​ = ​v  2 ​ – ​ ___________     ​   ​_›
m1 + m2 velocity of the COM of the system be ​v  ​cm   .
​_› ​__› ​_›
m1 (​v  ​ 2 – ​ v​  1) m1 ​v  ​ 21 In reference frame attached to the COM, the velocity of
= ​ __________    ​  = ​ _______   
 ​ ...(17) ith particle will be
m1 + m2 m1 + m2
​_› ​_› ​_› ​_› ​_› ​_›
Momentum of the particles in COM frame will be ​v  ​i cm
      fi ​v  ​i  = ​v  ​i cm
= ​v  ​i  – ​v  ​cm   + ​v  ​cm

(  )
_
​› ​_› m1 m2 ​_› ​_› kinetic energy of the system of particles in ground frame
​P  1cm
 ​ = m1 ​v  1cm
 ​ = ​ _______
​    ​ ​v   ​ = m ​v  ​12
 ​    
m1 + m2 12 is
1 ​_› ​_›
​   ​  S mi (​v  ​i  ◊ ​v  ​i )
= (​  _______
_ k = __
m +m )
​› ​_› m  m 1 2 ​_› _
​›
and ​P  ​2cm
  = m2 ​v  ​2cm
  ​   
  ​  ​ ​v ​   = m ​v  ​21
 21   ...(18) 2
1 2 _ _ _ _
1 ​› ​› ​› ​›
m1 m2 = __ ​   ​  S mi (​v   ​i cm + ​v   ​cm) ◊ (​v  ​ icm + ​v  ​ cm)
where m = _______
​   
  ​ , is known as reduced mass of the 2
m1 + m2
system. 1 1 _
​› _
​›
_
​› _
​› ​_› ​_› ​   ​  S mi v 2icm + __
= __ ​   ​  S mivcm
2
+ (S mi ​v  ​ icm) ​v  ​ cm
Obviously, ​P  ​1 cm
  + ​P  ​2 cm
  = m (​v  ​12
  + ​v  ​21
  ) = 0. 2 2
​_› ​_›
[Since ​a  ​  . ​a  ​  = a2]
Momentum of the system is zero in COM frame. ​_›
The third term is zero, since S  mi ​v  ​i cm
  is the momentum
Note:  Reduced mass is an useful concept in dealing with
of the system in COM frame and it must be zero.
bonded system – a system in which particles are bound
Centre of Mass  1.17

1 1 Solution
\ ​   ​  S mi vicm
k = __ 2
​   ​  ​( S mi )​ vcm
+ __ 2
2 2 Concepts
1 1
= __ ​   ​  S mi vicm
2
+ __ 2
​   ​  Mv cm When no external force acts, change in KE of a system
2 2 in COM frame is same as the change in KE in ground
1 frame.
k = kwrt cm + __ ​   ​  Mvcm2
...(19)
2
kwrt cm is the kinetic energy of the system in COM frame and KE of system in COM frame before collision is

( 
1 2
__
​   ​ Mv cm
2
is the kinetic energy associated with the translational
motion of the system as a whole.

1
kwrt cm = __
2
1 m ◊ 2m 2 __
​   ​  m u2 = __
​   ​ ​  ______
2 3m
​     
1
)
​  ​ u = ​   ​  mu2
3

If no external force is acting on a system, vcm will not KE in COM frame after collision is
change. Only kwrt cm can change due to internal interactions.
This has a very important implication. When no external
force is acting on a system, change in KE in ground frame

1
k¢wrt cm = __
2 2(  )
u 2 1 __
​   ​  m ​​ __
​   ​   ​​ ​ = __ (  ) 2 u2
​   ​  ​ ​   ​  m  ​ __
2 3
1
​   ​  = ___
4
​    ​ mu2
12
will be same as the change in COM frame.
Loss in KE in COM frame = loss in KE in ground
Using equations (16) and (17), one can easily prove that frame
for a two-particle system 1 1 1
= ​ __ ​  mu2 – ___ ​    ​ mu2 = __
​    ​ mu2
1 3 12 4
kwrt cm = __ 2
​   ​  m vrel ...(20)
2
m1m2 ​_› ​_› In short:
where m = _______
​    
 ​ and |vrel| = |​v  1 ​ – ​v  2 ​ | (i) COM frame is ‘zero-momentum frame’. Linear
m1 + m2
momentum of a system in its COM frame is always
Example 20   Two identical particles, each of mass m are zero.
moving in perpendicular directions
(ii) In a system having two particles, the particles move
with speed v. Find the kinetic energy
with equal and opposite momentum in COM frame.
of the system in a reference frame
Magnitude of momentum of each particle in COM
attached to the COM of the system.
frame is
Solution
P = m vrel
Concepts m1 m2
1 where m = ​ _______     ​, know as reduced mass and vrel
kwrt cm = __ 2
​   ​  m vrel m1 + m2
2
is the relative speed of the two particles.
m m m
m = ______
​   ​ = __
   ​   ​  (iii) KE of a system in COM frame, for a two-particle
m+m 2
1
​_› ​_› ​_› ​_›
______ system, is kwrt CM = __ 2
​    ​ m vrel
​v  ​ rel = ​v  ​ 1 – ​v  ​ 2  fi  |​v  ​ rel| = ​÷v  12 + v22 
 ​ 2
(iv) KE of a system in ground frame is
1 1 m 2 1
\  kwrt cm = __ 2
​   ​  m vrel = __
​   ​  __
​   ​  (v + v 22) = __
​    ​ m (v12 + v22) 1
2 2 2  1 4 k = kwrtCM + __ 2
​   ​  Mv cm
2
Example 21    Two balls of masses, m and 2m, approach where M is the total mass of the system.
each other with a relative velocity u and collide. after col-
u (v) If Fext = 0, then change in KE in ground frame is
lision, they separate from one another at a relative speed __
​   ​ .
2 same as the change in KE in COM frame.
Find loss in kinetic energy of the system of the two balls
due to collision.

Your Turn

Q.24  A particle of mass m1 = 4 kg moves at 5  ms–1, Q.25 


_

Two projectiles of equal
_

mass move with veloci-
› ›
while a particle of mass m2 = 2 kg moves at 2  ms–1. Find KE ties ​v  ​ 1 = (2  + 4  ) ms–1 and ​v  ​ 2 = (6  – 3  ) ms–1. Find the
of the system in a reference frame attached to its COM. momentum of the system in a reference frame attached to
its COM.
1.18  Mechanics II

7.  CENTRE OF GRAVITY If acceleration due to gravity is uniform, then


w1 = m1g, w2 = m2g , w3 = m3g ...
Consider a body having many y
particles. Weights of the par- m1y1 + m2y2 + m3y3
ticles are w1, w2, w3 ..., etc., \ ycg = __________________
​    
   ​= ycm
m1 + m2 + m3
and their y co-ordinates are
W1 W3 It means there is no difference in COM and centre of
y1, y2, y3 ..., etc. The centre W2 gravity (CG), as long as acceleration due to gravity is
of gravity is a point where we
uniform. The two terms are used interchangeably for daily-
can assume the weight of the
life objects.
body to be effectively acting.
x However, when you consider very tall buildings, the
Its y co-ordinate is given O
acceleration due to gravity will decrease as one moves up.
by
In such a case, the COM and the CG will be different points.
w1y1 + w2y2 + w3y3 + ... We must use equation (21) to find the position of CG in
ycg = ​  ____________________
        ​ ...(21)
w1 + w2 + w3 + ... such situations.

Your Turn

Q.26  Is it right to assume that CG of a very tall building


will be slightly below the COM? Give reason.

Miscellaneous Examples
y y
Example 22   A quarter disc y
Disc being folded
Find the x and y co-ordinates of the about y-axis
COM of a uniform plate in shape of a
a b
quarter circle, of radius R. [see fig.]
CM
x 4R
Solution O 3p
x x
Concepts O O

(i) The COM will lie on the symmetry line, pass-


ing through O and making 45° with both x an y
y
axes.
(ii) It means that x and y co-ordinate of the COM will Line of
symmetry
be same.
xCM
(iii) Refer to example 6. The y co-ordinate of the COM CM
4R
of a semicircular disc is ___
​    ​. 45° yCM
3p x
O
Imagine a semicircular disc of radius R. Its COM is on
4R
y-axis, at a distance ___
​   ​ from O. If the disc is folded about Example 23   COM of a liquid-filled container
3p
y-axis to a make a quarter disc, the y co-ordinate of none A liquid has been poured into a cylindrical vessel of mass
of the particles (in the disc) change. A particle at ‘a’ (see M and height H. Mass of liquid column of unit height is a.
figure) moves to ‘b’. Therefore, ycm does not change. Find the height x of the liquid column for which the COM
of the liquid plus the container system is at the lowest posi-
4R
\  For quarter disc, ycm = ___
​   ​  tion. Assume the COM of the empty vessel to be at height
3p H
__
​   ​  from the base.
4R 2
And xcm = ycm = ___
​   
3p
 ​
Centre of Mass  1.19

Solution COM of the system of two particles, measured from the


Concepts point of projection.
H
(i) COM of the empty container is at height ​ __ ​ . When Solution
2
liquid is poured, the bottom gets heavier and the Concepts
COM of the system moves lower. After a lot of (i) We can find initial velocity of COM using
liquid has been poured, the com begins to shift
up. Think of a situation where the liquid has been m1u1 + m2u2
ucm = ___________
​      ​ 
poured to a great height. Mass of the liquid will be m1 + m2
much larger than the container and the COM will (ii) We can find acceleration of COM using
be close to the COM of the liquid (i.e., nearly at
m1a1 + m2a2
height ​ __
H
  ​)  . We can say that the COM goes down acm = ___________
​      ​ 
2 m1 + m2
and then rises as the liquid is poured.
(iii) We can use equation of kinaematics directly for
(ii) We will write the height of COM (xcm) when liquid motion of the COM.
column has height x. Then we will use the concept

xcm will be minimum.


( 
of maxima and minima ​ ​ ____   
dxcm
dx )
​ = 0  ​ to find when Initial velocity of COM is
mu + (2m) (2u) 5
ucm = ​ _____________
  
  
  ​= __
​   ​  u
m + 2m 3
Acceleration of COM is
m (g) + (2m) (g)
acm = ______________
​    
     ​ = g (Ø)
m + 2m
5
COM is an imaginary point having initial velocity ​ __ ​  u (≠)
3
and a constant acceleration g (Ø). when COM is at its
maximum height, its velocity is zero.
2 2
The container can be replaced with a point mass (M) vcm = ucm + 2acm ◊ xcm
H
at height __ ​   ​ . When liquid is poured to a height x, mass of
2
liquid is m = a x. It can be assumed to be a point mass at
fi (  )
5u 2
0 = ​​ ___
​   ​   ​​ ​ – 2g (xcm)
3
height __
x 25u2
​    ​ from the base. fi xcm = ____
​    
2 18 g


(  ) H
M ​ __
2 (  )
xcm = ​ ________________   
2
     ​
x
​   ​   ​  + (a x) ​ ​ __  ​  ​ Example 25   A uniform rope of length L
M + a x is placed over a smooth pulley and held in
a position, where 40% of its length lies on
dxcm
For xcm to be minimum, _____
​     ​ = 0 one side and the remaining 60% hangs on
dx the other side. The rope is released from this
H a x2
( 
fi (M + a x) (a x) – ​ M ​ __ ​  + ____
2 2 )
 ​ a = 0
​   ​  
position. [Neglect dimensions of the pulley]
(i) Find the vertical acceleration of the
A

COM of the rope, immediately after


fi a x2 + 2Mx – MH = 0 B
it is released.
____________
     
– 2M ± ​÷4M 2
+ 4MH a ​
  
(ii) Find the vertical velocity of the COM, at the instant
fi x = ​  ___________________
 ​    L
2a end A is __​   ​  above its initial position.
5
___________

​÷M (M + H a) ​
   – M Solution
fi x = ​  _______________
    
a Concepts
 ​
Example 24   Two particles of masses m (i) Pulley is of negligible dimension. The rope has two
2u
and 2m are projected vertically upward from segments – one to the left of the pulley and other
u
a point, with velocities u and 2u respectively. to its right. Each segment of the rope has a mass
Find the maximum height attained by the m 2m proportional to its length.
1.20  Mechanics II

These can be assumed as point masses kept at respective


(ii) Acceleration of the rope can be calculated using
COMs of the two segments. PE in this position is
Atwood machine formula and after that, acm can
L 2L
m1a1 + m2a2 Uf = – M1g ​ ___  ​ – M2g ___
​   ​ 
be calculated using acm = ​ __________
    ​  10 5
m1 + m2
(iii) Speed of the rope after certain displacement can
be calculated from conservation of energy. Then
l L
= – ​ ___ (  ) (  ) (  ) (  )
L 4L 2L
​   ​    ​ ​ g ​ ___  ​  ​ – ​ l ​ ___ ​   ​ ​ g ​ ___ ​   ​
5 10 5 5
m1v1 + m2v2 17
we can use vcm = ​ __________
   ​  = – ​ ___ ​ l gL2
m1 + m2 50
gain in KE of rope = loss in its PE
2
(i) Length of rope on left = __ ​   ​  L 1
5 ​ __ ​  (l L) v2 = Ui – Uf
2
3
Length of rope on right = __ ​   ​  L 1 2
5 ​ __ ​  l Lv2 = ___​    ​ l gL2
2 25
Mass of rope on left and right side of the pulley
2 ___
respectively is m1 = __
2
​   ​  L l and m2 = __
3
​   ​  L l. Where l fi v = __    ​
​   ​  ​÷gL 
5 5 5
is the mass per unit length.
Velocity of the COM of the rope is


(m – m
a = ​ _________
2 1 ) 
   ​ 
​ __
3 2
5 5
= ​ __________
(  )
​   ​  – ​ __  ​  ​L l g
     ​ 
g
= __
​   ​  M v – M v
4L
5 ( 
L 2 ___
​ l ​ ___ ​ – l  __
​   ​   ​ __
5 5 ÷ )  ​ 
​   ​  ​ gL 
vcm = ​ _________ ​ = __________________
2 1
m2 + m1 l L 5    ​         ​
M2 + M1 l L
6 ___
= ___   ​ 
​    ​ ​÷gL 
25

Example 26   Two blocks of equal mass m are connected


by an unstretched spring and the system is kept at rest on a
smooth horizontal surface. A constant horizontal force (F)
begins to act on block 1,
at time t = 0, so as to pull
it away from the other
block.
(i) Find displacement of the COM of the system of two
blocks at time t.
L
(ii) Initially, COM of the left side of the rope is ​ __ ​  below (ii) Extension in the spring is observed to be x0 at time
5 t. Find the displacement of the two blocks at this
3L
the pulley and that of the right side is ___
​   ​ below the time.
10
pulley.
Solution
Assuming gravitational PE to be zero at the reference
level of the pulley, we can write the PE of rope in its initial Concepts
position by assuming point masses at respective COMs of (i) Acceleration of COM is decided by external forces
the segments. only.
L 3L
ui = – m1g ​ __ ​  – m2g ​ ___ ​  (ii) If x1 and x2 are displacement of the two blocks,
5 10
displacement of COM will be
2
= – ​ __
​   ​  L l  ​ g
5 (  ) L
__
​   ​  –
5 (  )
3
​ __
5
3L
​   ​  L l   ​g ​ ___ ​ 
10
mx1 + mx2
xcm = _________
​ 
m+m
   
x1 + x2
​ = ​ ______
2
 ​ 

13 (iii) Extension in spring is x0 = x1 – x2
= – ​ ___ ​  l gL2
50
L Fext ___
After the rope has moved by ​ __ ​ , the masses on two sides (i) acm = ____
​     
F
​ = ​    ​ 
5 M 2m
are
L 4L Displacement of COM in time t is
M1 = l ​ __ ​  and M2 = l ​ ___ ​ 
(  )
1 1 F 2 ___ Ft2
5 5 xcm = ucm ◊ t + __ ​   ​  acm ◊ t2 = __
​    ​ ​ ___
​    ​  ​ t = ​    ​
2 2 2m 4m
Centre of Mass  1.21

x1 + x2
(ii) Since xcm = ​ ______
 ​ 
  xcm = displacement of COM when extension is
2
  maximum

[ 
1 Ft2
\ x1 + x2 = __
​   ​  ​ ___

2 m
 ​  

Extension in spring is x1 – x2 = x0
...(a)

...(b)
Mx1 + mx2 x1 and x2 are displacement of
= ​ _________
M+m
  
 ​ ​  ​

      
M and m respectively
  ​  ​
​ ]
Solving (a) and (b), we get 2F
\ v2 = ________
​      ​ (mx1 + mx2) ...(a)

1 Ft
(  2
x1 = ​ __ ​  ​ ___
2 2m ) 1 Ft
​    ​ + x0  ​ and x2 = __ (  2
​   ​  ​ ___
2 2m )
​    ​ – x0  ​
(M + m)2
Initially, ki = 0
1
Example 27   Maximum extension At maximum extension, kf = __
​   ​  (m + M) v 2
2
A block of mass m is connected to another block of mass M
Using work–energy theorem:
by a massless spring of force constant k. the blocks are kept
on a smooth horizontal plane. Initially, the blocks are at rest Work done by F = gain in KE + gain in PE
and the spring is relaxed. A 1 1
constant horizontal force F Fx1 = __
​   ​  (m + M) v2 + __ ​    ​ kx2
2 2
begins to act on the block
of mass M (see figure). Find the maximum extension in the
spring during the subsequent motion.
( 
Mx1 + mx2
fi Fx1 – F ​ ​ _________
M+m
     )
​  ​ = __
1
​   ​  kx2   [using (a)]
2

Solution Fm (x1 – x2) 1


fi ​ __________
     ​ = __
​   ​  kx2
M+m 2
Concepts
But  x1 – x2 = x (extension in spring)
(i) Initially, acceleration of M will be higher and it
will move away from the other block. The spring 2Fm
\ kx = ______
​    
 ​
expands. The stretched spring will exert more and M+m
more force on m and its acceleration will increase. 2Fm
fi x = _________
​      ​
In fact, when the spring stretches a lot, M will k ( + m)
begin to retard. There will be a point when both
Alternate:  Here, we will use work–energy theorem in the
the blocks will have the same velocity (i.e., their
RF attached to the COM. This RF has an acceleration
relative velocity will be zero). At this point, the
F
extension will be maximum. aCM = ______
​       ​
M+m
(ii) Motion of the COM can be predicted as external
force is known.
(iii) Displacement of the COM when the two blocks
move by x1 and x2 is
mx1 + mx2
xcm = _________
​    
 ​
M+m
We apply a pseudo-force on both the blocks.
(iv) Work–energy theorem can be applied. Net external force (leaving apart the spring force) on M
is
Initially, ucm = 0 mF
when the extension is maximum, velocity of both F1 = F – Macm = ​ ______
   ​ (towards right)
M+m
blocks = v (say).
mF
At this instant, Net external force on m is F2 = macm = ​ ______
    ​ (towards
M+m
left)
Mv  + mv
vcm = ________
​     
​= v In COM frame, initial KE is zero and final KE, when
M+m
spring has maximum extension (i.e., when v1 = v2 = vcm),
The COM moves with a constant acceleration is also zero.
F
[ 
acm = ______
]
​       ​ 1
M+m ​ kwrt cm = ​ __ ​  m v2rel = 0  ​
2 2 2
\ vcm = ucm + 2 acm xcm
Using work–energy theorem in COM frame:
fi ( 
F
v2 = 2 ​ ______
​       ​  ​ x
M + m cm ) WF1 + WF2 = DU + Dk.
1.22  Mechanics II

mF mF 1
fi  ​ ______
    ​ x – ______
​     ​ x = ​ __ ​  kx2
M+m 1 M+m 2 2
mF 1
fi  ​ ______
    ​ (x – x2) = __
​   ​  kx2
M+m 1 2
mF 1
fi  ​ ______
    ​ x = ​ __  ​ kx2
M+m 2
2mF
fi x = _________
​    ​ 
k (M + m)
The lower block will just leave the ground if
Example 28   Jumping blocks kx = mg
Two blocks, each of mass m, are con-
fi mg = kx0 – 2mg
nected by a compressed light spring of
spring constant k. the blocks are held 3mg
in position by a light thread with spring fi x0 = ____
​     
 ​
k
compressed by x0. The thread is burnt.
Find
(i) The smallest value of x0 for which
7mg
(ii) If x0 = ​ ____
k
  
3mg
(  )
 ​ ​ > ​ ____
k
  ​  ​, the lower block will bounce
the lower block will bounce up. off when the upper block is still moving up. Let
(ii) The height to which COM of the system will rise, if v = speed of the upper block at the instant the lower
mg
7mg block just begins to move (i.e., at x = ___ ​  k   ​)  .
x0 = ____
​    ​   
 .

(  ) (  ) (  )
k
1 7mg 2 1 1 mg 2 7mg mg
​ __ ​  k ​​ ____ ​  ​​ ​ = ​ __  ​ mv2 + __
​      ​   ​  k ​​ ___
​     ​   ​​ ​ + mg ​ ____
​     
​ + ___
​     ​   ​
Solution 2 k 2 2 k k k
Concepts mg2
(i) when the spring stretches, it pulls the lower block fi v 2 = 32 ​ ____
  ​   ...(a)
k
up. When this force becomes greater than mg, the
Velocity of COM at the instant the lower block begins to
lower block jumps.
move is
(ii) Energy conservation can be used to find the exten-
mv + 0 v
sion in the spring after the system is released (thread vcm = ​ ______
   ​=
  __
​    ​
2m 2
is burnt) and lower block remaining fixed.
(iii) After the lower block leaves the surface, the net Now the COM moves with a retardation g. If the COM
external force on the system is 2mg and accelera- further rises by x¢, then for motion of COM

(  )
tion of COM is g (Ø). v 2 v2 4mg
02 = ​​ __
​    ​  ​​ ​ – 2gx¢  fi  x¢ = ___
​    ​ = ____
​     
 ​
2 8g k
(i) Let the upper block come to rest when the spring is
By the time the lower block begins to move, the upper
stretched by x. Assume that the lower block has not 7mg ___ mg 8mg
moved till now. block has gone up by x1 = ____ ​      ​ + ​     ​ = ____
​     
​. 
k k k
Using conservation of energy, It means the COM has moved through a distance
1 1 mx1 + 0 ____4mg
​ __ ​  kx20 = __
​    ​ kx2 + mg (x0 + x) ​ ______
   
​ = ​     
 ​ before the lower block starts to move.
2 2 2m k
fi kx2 + 2mg x + 2mg x0 – kx20 = 0 4mg ____ 4mg ____ 8mg
\  Total rise of COM = ____ ​       ​ + ​     ​ = ​      ​. 
______________________ k k k
  2g2 – 4k (2mg x
– 2mg ± ​÷4m    
2
0 – kx0 ) ​ Example 29   A point-like ball of
fi x = _______________________________
​           ​
2k mass m is tied to the end of a light
– 2mg ± (2kx0 – 2mg) string, which is attached to the top
= ___________________
​     
    ​ of a vertical massless rod. The rod
2k
is fixed to a block of mass M lying
(kx0 – 2mg) at rest on a horizontal surface. The
= ​ __________
    ​  and x0 (Not acceptable).
k pendulum is displaced to a hori-
\ kx = kx0 – 2mg zontal position with string just taut
Centre of Mass  1.23

__
and released. Friction between the block and the horizontal M (o) + max  m
3​÷3 ​
  g
\ (acm)x = ​ __________
   
   ​ = ​ ________  
 ​
surface is large enough to prevent the block from sliding. M+m 4 (M + m)
__
(i) Find the magnitude of the horizontal component    mg
(ii) f = (M + m) (acm)x = 3​÷3 ​ 
of acceleration of the COM of the entire system
(iii) When the string is vertical, there is no tangential
when the string makes an angle of q = 60° with the
force on the ball (fi  at = 0) and the radial accelera-
vertical.
tion is vertical.
(ii) Find the friction force on the block when the string
\    (acm)x = 0   fi  f = 0
makes 60° with vertical.
(iii) Find the friction force on the block when the string Example 30   Two thin rods of identical material and
becomes vertical. equal cross-section are
A
connected (at A) by a
Solution
frictionless joint. Lengths 2
Concepts of the two rods are AB =
1
(i) Since the block does not move, friction does not l1 and AC = l2. The struc-
perform work on it. Mechanical energy of the ture is kept vertical on a
swinging pendulum is conserved. We will use smooth table, as shown. it
B C
energy conservation to find speed of the ball when is released from this posi-
q = 60°. tion and the ends B and C slide so that the angle between
the two rods decreases. Where will the joint A hit the table?
(ii) Acceleration of the bob has a radial and a tan-
Assume that the structure always remains vertical during
gential component. We can find these components
the fall.
separately. Then, we find the horizontal accelera-
tion (ax) of the ball by resolving ar and at in hori- Solution
zontal direction. Concepts
max
(iii) aCM = ______
​     
​ , since M is not moving. There is no horizontal force on the structure. Its COM
m+M
will not have any horizontal displacement. End B moves
(iv) Friction is the only horizontal force on the to the right (and C also moves to the right) so that the
system. angle between the rods decreases.
\ f = (M + m) acm.
(v) When the string is vertical, there is no horizontal
acceleration. A
2
1
(i) speed at q = 60° can P CM Q m2
m1
be calculated as
1
​ __ ​  mv2 = mg l cos 60° x
2 B D
__ C
fi   ​ 
v = ​÷gl 
x
\ Radial acceleration m2 C¢
is x
A¢ B m1 B¢
v2
ar = ​ __ ​  = g Mass of rod AB,  m1 = l l1
l
Tangential acceleration is Mass of rod AC,  m2 = l l2
__
÷   
​ 3 ​ AB can be replaced with a point mass at its centre P and
at = g sin 60° = ___
​   ​  g
2 AC can be replaced with a point mass at its centre Q.
Horizontal acceleration of the ball is: _____
​   ​  ​÷l 22 – l12 ​ 
1 1
BD = __​   ​  BC = __
ax = ar cos 30° + at cos 60° 2 2
__ __ __
   g Taking B as the origin, the x co-ordinate of the COM of
÷   
​ 3 ​     1 3​÷3 ​ 
​÷3 ​
= ___
​   ​  g + ___
​   ​  g ◊ ​ __ ​  = _____
​   ​    the structure in initial position is _____
2 2 2 4
2 ÷l 2 – l 1 
1 2
__ 2
l l ​   
 ​
​  ​
m 1 (0) + m2 (BD)
_____________ 2
___________
xcm = ​      
 ​ = ​    
  
 ​
m1 + m2 l l1 + l l2
1.24  Mechanics II

÷ 
______
l2 l2 – l1
= __
​   ​  ​ ​ _____ 
2 l2 + l1

 ​ ​ ...(i)
\
l
(  ) (  )
2
xcm = _____________________
​       ​
l2
​  1 ​  – x  ​ + m2 ​ __
m1 ​ __ ​   ​  – x  ​
2
  
When the structure collapses, let end a be at a distance m1 + m2

(  ) (  )
x from origin B (see figure). l1 l2

÷ 
l1 ______
l1 ​ __
​   ​  – x  ​ + l2 ​ __
​   ​  – x  ​
Now, m1 is a point mass at a distance __ ​   ​  from A¢. l2 – l1
l__2 _____
fi ​   ​  ​ ​ 
2
 ​ ​ = ​ ___________________

       ​
2
  
2 2 l2 + l1 l1 + l2
(  )
l__1
\  Point mass m1 is at a distance ​ ​   ​  – x  ​ from B.
2 l2 _____ __
fi ​ __ ​  ​÷l 22 – l12 ​  
1
= ​   ​  (l12 + l22) – (l1 + l2) x

( 
2 2
l2
2 )
Similarly, point mass m2 is at a distance ​ ​ __  ​ – x  ​

l2
x = ________
​      [ 
_____
]
 ​ ​ l12 + l22 – ​÷l 22 – l12 ​  ​
from B. 2 (l1 + l2)
Worksheet 1
1. All the particles of a system are situated at a distance
r from the origin. The distance of the centre of mass
of the system from the origin is
(a) = r (b) ≤ r
(c) > r (d) ≥ r
2. A uniform wire of length l is bent into a ‘V’ shaped
object as shown. The distance of its centre of mass
from the vertex A is

(a) (1/2, 1/2, 1/2) (b) (1/3, 1/3, 1/3)


(c) (3/4, 3/4, 3/4) (d) (1/2, 3/4, 1/2)
6. In a HCl molecule, the separation between the nuclei
of the two atoms is about 1.27 Å (1 Å = 10–10 m).
__
l​÷3 ​
   Given that a chlorine atom is about 35.5 times as
(a) l/2 (b) ​ ____ ​ 
  massive as a hydrogen atom and nearly all the mass
__ 4
l​÷3 ​
   of an atom is concentrated in its nucleus, the position
(c) ​ ____ ​  
  (d) None of these of COM of the molecule is
8
3. Find the x coordinate of the centre of mass of the (a) On the line joining H and Cl and at a distance
arrangement of bricks shown in figure: of slightly less than 1.27Å from H end
(b) On the line joining H and Cl and at a distance
of nearly 1.27Å from Cl end
(c) On the line joining H and Cl and at a distance
of 0.27Å from H end
(d) On the line joining H and Cl and at a distance
of 0.57Å from H end
7. An arrow sign is made by cut-
24 25 ting and rejoining a quarter part
(a) ​ ___ ​ l (b) ​ ___ ​ l
25 24 of a square plate of side ‘L’ as
15 16 shown. The distance OC, where
(c) ​ ___ ​ l (d) ​ ___ ​ l ‘C’ is the centre of mass of
16 15
the arrow, is
4. A composite body is made by joining a uniform disk
L L
of radius r and a uniform square plate of side length (a) ​ __ ​   (b) ​ __ ​ 
2r, as shown in fig- 3 4
3L
ure. Both plates have (c) ​ ___ ​   (d) None of these
same mass per unit 8
area. The co ordi- 8. The centre of mass of a system of particles is at the
nates of COM of the origin. From this, we conclude that
composite plate are (a) The number of particles on positive x-axis is

(  ) (  )
r(p + 3) r(p + 2) equal to the number of particles on negative
(a) ​ ​ _______  ​, 0  ​ (b) ​ _______
​   ​ 
, 0  ​ x-axis
(p + 1) (p + 1)
(b) The total mass of the particles on positive x-axis
( 
r(p + 1)
(c) ​ ​ _______ ​ 
(p + 1) )
, 0  ​ (d) none

is same as the total mass on negative x-axis
(c) The number of particles on x-axis may be equal
5. Find the coordinates of centre of mass of the structure to the number of particles on y-axis.
shown in figure made of four identical cubes. Length (d) If there is a particle on the positive x-axis, there
of each side of a cube is 1 unit. must be at least one particle on the negative
x-axis.

1.25
9. Distance of the centre of mass, from centre, of an half-rings of radius R and two straight parts of length
annular half-disc shown in figure is p R. The height of COM from the table is
2R R
(a) ​ ___
p ​  (b) ​ __
p ​ 
R
(c) ​ __ ​   (d) zero
2
13. A piece of paper (shown in figure 1) is in the form
4(​R3​2​​  – ​R3​1​)​  (​R3​2​​  – ​R3​1​)​  of a square. Two corners of this square are folded
(a) ​ __________   
    ​ (b) ​ __________      ​
3p(​R2​2​​  – ​R2​1​)​  3p(​R2​2​​  – ​R2​1​)​  to make it appear like figure 2. Both corners are put
together at the centre ‘O’ of the square. If O is taken
3p (​R2​2​​  – ​R2​1​)​  to be (0, 0), the centre of mass of the new system
___________
(c) ​    
   ​ (d) None will be at
3p(​R2​2​​  – ​R2​1​)​ 
10. An L-shaped (shaded piece) is cut from a rectangular
metal plate of uniform thickness. The point that cor-
responds to the centre of mass of the L-shaped piece
is:


Fig. 1 Fig. 2

(  a
)
(a) ​ –  ​ __ ​ , 0  ​
8 (  a
)
(b) ​ –  ​ __ ​ , 0  ​
6
( a
___
)
(c) ​  ​    ​,  0  ​
12 (  ___a
(d) ​ –  ​    ​,  0  ​
12 )
14. Five homogeneous bricks, each of length L, are
arranged as shown in figure. Each brick is displaced
with respect to the one
in contact by L/5. Find
the x-coordinate of the
(a) 1 (b) 2 centre of mass relative
to the origin O shown.
(c) 3 (d) 4
23L 33L
11. From the circular disc of radius 4 R, two small disc (a) ​ ____ ​   (b) ​ ____ ​ 
of radius R are cut off. The centre of mass of the 50 50
new structure will be at: 13L 29L
(c) ​ ____ ​   (d) ​ ____ ​ 
50 50
15. Eight solid uniform cubes of edge l are stacked
together to form a
single cube with
centre O. One
cube is removed
from this system.
R R R R Distance of the
(a) i​ __ ​  + i​ __ ​   (b) –i​ __ ​  + j​ __ ​  centre of mass of
5 5 5 5
the remaining 7
– 3R
(c) ​ ____ ​    (î + jˆ ) (d) None of these cubes from O is
__ __
14
____   l
7​÷3 ​   l 
​÷3 ​
___
12. A wire is bent (a) ​   ​     (b) ​   ​ 
16 16
into a structure __
as shown in ÷    l
​ 3 ​
___
(c) ​   ​   (d) zero
the figure, and 14
placed on a table. 16. A projectile is launched from the origin with speed
It consists of two v at an angle q from the horizontal. At the highest
point in the trajectory, the projectile breaks into two

1.26
pieces, A and B, of masses m and 2m, respectively. (a) 3v (b) v
Immediately after the breakup, piece A is at rest (c) 1.5v (d) zero
relative to the ground. Neglect air resistance. Which 20. A boy of mass 60 kg is standing over a platform
of the following sentences most accurately describes of mass 40 kg placed over a smooth horizontal sur-
what happens next? face. He throws a stone of mass 1 kg with veloc-
ity v = 10 ms–1 at an angle of 45° with respect to
the ground. Find the displacement of the platform
(with boy) on the horizontal surface when the stone
lands on the ground. Neglect the height of projection
(g = 10 ms–2)
(a) 10 cm (b) 1 m
(c) 1.2 m (d) 50 cm
(a) Piece B will hit the ground first, since it is more
massive. 21. Two particles of equal mass have initial velocities
2iˆ ms–1 and 2jˆ ms–1. First particle has a constant
(b) Both pieces have zero vertical velocity imme-
acceleration (iˆ + jˆ ) ms–2 while the acceleration of
diately after the breakup, and therefore they hit
the second particle is always zero. The centre of mass
the ground at the same time.
of the two particles moves in
(c) Piece A will hit the ground first, because it will
have a downward velocity immediately after the (a) Circle (b) Parabola
breakup. (c) Ellipse (d) Straight line
(d) There is no way of knowing which piece will hit 22. A uniform thin rod of mass M and Length L is stand-
the ground first, because not enough informa- ing vertically along the y-axis on a smooth horizontal
tion is given about the breakup. surface, with its lower end at the origin (0, 0). A
17. Three men A, B & C of masses 40 kg, 50 kg & 60 slight disturbance at t = 0 causes the lower end to
kg are standing on a plank of mass 90 kg, which is slip on the smooth surface along the positive x-axis,
kept on a smooth horizontal plane. If A & C exchange and the rod starts falling. The acceleration vector​_› of
their positions, then mass B will shift the centre of mass of the rod during its fall is: [​R   ​ is
reaction from surface]
​_› _
​› ​_› _
​›
​_› M​g  ​  + ​R  ​  ​_› M​g  ​  – ​R  ​ 
(a) ​a  ​CM
  = ​ _______    ​   (b) ​a  ​CM
  = ​ _______     

M M
​_› ​_›
(c) ​a  ​ CM = M​g  ​  – R (d) None of these
23. A uniform sphere is placed on a smooth horizontal
surface and a horizontal force F is applied on it at a
(a) 1/3 m towards left distance h above the centre. The acceleration of the
(b) 1/3 m towards right centre of mass of the sphere
(c) will not move w.r.t. ground (a) is maximum when = 0
(d) 5/3 m towards left (b) is maximum when h = R
18. An isolated particle of mass m is moving in horizontal (c) is maximum when h = R/2
plane (x–y), along the x–axis, at a certain height above (d) is independent of h
the ground. It suddenly explodes into two fragment 24. Two men ‘A’ and ‘B’ are standing on a plank which
m 3m is placed on a smooth surface. ‘B’ is at the middle
of masses __​   ​  and ___
​   ​ . An instant later, the smaller
4 4 of the plank
fragment is at y = +15 cm. The larger fragment at and ‘A’ is at
this instant is at: the left end
(a) y = –5 cm (b) y = + 20 cm of the plank.
(c) y = + 5 cm (d) y = – 20 cm Surface of
the plank
19. Two particles A and B, initially at rest, move towards
is smooth.
each other under the mutual force of attraction. At the
System is
instant when the speed of A is v and the speed of B
initially at
is 2v, the speed of the centre of mass of the system
rest and
is:

1.27
masses are as shown in figure. A and B start mov- (a) If B is closed and sand is poured from A, centre
ing such that the position of ‘B’ remains fixed with of mass first rises and then falls
respect to ground. The point where A meets B is (b) If shell is completely filled with sand and B is
located at opened then centre of mass falls initially
(a) the middle of the plank (c) If shell is slightly filled with sand and B is
(b) 30 cm from the left end of the plank opened, then centre of mass falls.
(c) the right end of the plank (d) None of these
(d) None of these 29. Imagine a massless rod of length R carrying two
25. A large wedge rests on a horizontal frictionless particles of mass m each at its end. The rod is placed
surface, as shown. A block starts from rest and slides inside a tunnel dug along a
down the inclined surface of the wedge, which is radius of the Earth. Find the
rough. During the motion of the block, distance between the centre
the centre of mass of the block and of gravity and the centre of
wedge system mass of a two-particle system
(a) does not move attached to the ends of a light
(b) moves vertically with increasing speed rod. R is the radius of earth. It
(c) moves horizontally with constant speed is given that acceleration due to
(d) moves both horizontally and vertically gravity at the centre of the Earth is zero.
26. Two uniform non-conducting balls A & B have (a) R (b) R/2
identical size having radius R but made of differ- (c) zero (d) R/4
ent density materials; density of A being twice the 30. Two particles approach each other with different
density of B. The ball A is positively charged & velocities. After _
​›
collision, one of the particles has a
ball B is negatively charged. The balls are released momentum ​p   ​ in their centre of mass frame. In the
on the horizontal smooth surface when their centres same frame, the momentum of the other particle is
​_›
are at separation 10 R. Because of mutual attrac- (a) 0 (b) – ​p  ​ 
​_› ​_›
tion, the balls start moving towards each other. They (c) – ​p   ​/2 (d) – 2​p   ​
will collide when the lighter ball has travelled a 31. Two identical cars start at the same point, but travel
distance in opposite directions on a circular path of radius R,
10R 16R each at speed v. While each car travels a distance less
(a) x = ____
​   ​  
  (b) x = ____
​   ​   
3 3 than (p/2) R, (one quarter circle) the centre of mass
7R of the two cars
(c) x = 5R (d) x = ___
​   ​ 
5 (a) remains at the initial point
27. A block of mass M is tied to one end of a massless rope. (b) travels along a diameter of the circle at
The other end of the rope is in the hands of a man of speed < v
mass 2M. Both the man and the block lie on a rough (c) travels along a diameter of the circle at
wedge of speed = v
mass M, as
(d) travels along a diameter of the circle at
shown in
speed > v
the figure.
The whole 32. When a block is placed on a wedge as shown in the
system is figure, the block
resting on starts sliding down
a smooth horizontal surface. The man pulls the rope. and the wedge
Pulley is massless and frictionless. The displacement also starts sliding
of the wedge when the block meets the pulley is on the ground. All
(Man does not leave his position during the pull) surfaces are rough.
The centre of mass
(a) 0.5 m (b) 1 m
of (wedge + block)
(c) Zero (d) 2/3 m the system will move
28. A uniform metallic spherical (a) leftward and downwards
shell is suspended from ceiling.
(b) rightward and downwards
It has two holes A and B at top
(c) leftward and upwards
and bottom respectively. Which
of the following is/are true: (d) only downwards

1.28
Worksheet 2
1. In which of the following cases, the centre of mass string makes an angle of 30° with the vertical. The
of a rod may be at its centre? mass of the bar is 4 kg.
(a) The linear mass density continuously decreases (a) The bar first moves to left and then to the right,
from left to right. as the ball oscillates.
(b) The linear mass density continuously increases (b) The maximum angle that the string makes with
from left to right. vertical on other side is less than 30°.
(c) The linear mass density decreases from left to (c) The displacement of the bar when the string
right upto centre and then increases. makes the maximum angle on the other side of
(d) The linear mass density increases from left to the vertical is 0.2 m
right upto centre and then decreases.
​_› ​_›
(d) The COM of the ball plus bar system experiences
2. An external force ​F   ​ (​F   ​ π 0) acts on a system of no acceleration as the ball oscillates.
particles. The velocity and the acceleration of the 6. A mercury thermometer is placed in a gravity-free
centre of mass are found to be vcm and acm at an hall without touching anything. As temperature rises
instant, then it is possible that mercury expands and ascends in thermometer. If
(a) vcm = 0, acm = 0 (b) vcm = 0, acm π 0 height ascended by mercury in thermometer is h
(c) vcm ≠ 0, acm = 0 (d) vcm π 0, acm π 0 then
3. Consider a particle at rest, which may decay into two (a) The COM of the ‘‘mercury and thermometer”
(daughter) particles or into three (daughter) particles. system may descend if mass of mercury is very
Which of the following is true in the two-body case small relative to the mass of glass.
but false in the three-body case? (b) The COM of the ‘‘mercury and thermometer”
(There are no external forces and the masses of system may ascend if mass of mercury is very
daughter particles are known.) large relative to the glass.
(a) Velocity vectors of the daughter particles must (c) The COM of the ‘‘mercury and thermometer”
lie in a single plane. system may ascend even if mass of mercury is
(b) Given the total kinetic energy of the system, it is very small relative to the glass.
possible to determine the speed of each daughter (d) All the above are false.
particle. 7. A man of mass M is hanging with a
(c) Given the speed (s) of all but one daughter light rope, which is connected to a bal-
particle, it is possible to determine the speed of loon of mass m. The system is at rest in
the remaining particle. air. The man climbs a distance h with
(d) The total momentum of the daughter particles respect to the rope.
is zero. (a) Due to gravity, the COM of the
4. An object comprises a uniform entire system (Balloon + man)
ring of radius R and its uniform will move down.
chord AB (not necessarily made mh
(b) The actual displacement of the man is ______
​     ​
of the same material) as shown. m+M
Which of the following cannot be (c) The actual displacement of the balloon
the centre of mass of the object? Mh
is ​ ______
   ​
(a) (R/3, R/3) (b) (R/3,__R/2) __ m+M
(c) (R/4, R/4)   ,  R/​÷2 ​
(d) (R/​÷2 ​   )  (d) If the man just releases the rope and sets himself
5. A ball of mass 1 kg is sus- into a free fall, the balloon will begin to rise
pended by an inextensible with an acceleration.
string, 1 m long, attached to 8. A ring of mass m and a particle of same mass are
a point O of a smooth hori- fixed on a disc of same mass such that the centre of
zontal bar resting on a fixed mass of the system lies at the centre of the disc. The
smooth supports A and B. system rotates such that the centre of mass of the disc
The ball is released from rest moves in a circle of radius R with a constant angular
from the position where the velocity w. From this, we conclude that

1.29
(a) An external force mw 2R must be applied to (c) can move in a horizontal line
central particle (d) will move in a straight line for any value of
(b) An external force mw 2R must be applied to the v1, v2, q1 and q2
ring 10. For a two-body system, in the absence of external
(c) An external force 3mw 2R must be applied to forces, the kinetic energy as measured from ground
central particle frame is ko and from the centre of mass frame is kcm.
(d) An external force 3mw 2R must be applied on Pick up the wrong statement/s
the system (a) The kinetic energy as measured from the centre
9. Two particles of equal mass m are projected form of mass frame is least.
the ground with speeds v1 and v2 at angles q1 and (b) Only the portion of energy k cm can be
q2 (q1, q2 π 0, 180°), as shown in figure. The centre transformed from one form to another due to
of mass of the two particles internal changes in the system.
(c) The system always retains at least ko – kcm
amount of kinetic energy, as measured from
ground frame, irrespective of any kind of
internal changes in the system.
(d) The system always retains at least kcm amount of
(a) will move in a parabolic path for any values of kinetic energy, as measured from ground frame,
v1, v2, q1 and q2 irrespective of any kind of internal changes in
(b) can move in a vertical line the system.

1.30
Worksheet 3
1. A uniform wire is in the shape other corner is at A on y
of a circular arc, subtending an the circumference. The A
angle q at the centre. Radius of diagonal OA of the 
the arc is R. Find the distance R square is y-axis and
of the COM of the arc from q O is the origin. Find
centre of curvature O. the co-ordinates of the O x
2. A uniform wire is bent into a COM of the disc with
O
shape shown in figure. Segments hole.
AB and ED are straight, having length 2R each, lying 6. A 75-kg man stands at
in a horizontal plane and the segment BCD is a semi- the rear end of a plat-
circle of radius R. Find the distance of the COM of form of mass 25 kg and
the wire from centre O of the circle. length 4m, which is moving on a smooth horizon-
tal surface, with velocity 4 ms–1. The man begins to
walk at 2 ms–1, relative to the platform, towards the
front end. He stops after reaching the front end. Find
the following for the interval the man walks on the
platform–

(i) Displacement of the COM of the man + plat-


3. A smooth rod of length L = 2R is leaning against form system.
a semi cylinder of radius R, which is placed on a
(ii) Displacement of the man, relative to the
smooth horizontal surface. In the position shown,
ground.
q = 45°. The system is released from this position.
Find the displacement of the semi cylinder by the (iii) Displacement of the platform, relative to the
time end B of the rod touches it. Both cylinder and ground.
rod have the same mass. 7. A flat railroad car has a boy standing at its end. The
boy rolls billard balls along the surface of the car,
B which get collected at the other end into a small
L bucket (See figure) attached to the car. Length of the
car is L and it stands on a frictionless track. Initially,
R the entire system was at rest when the boy rolled the
first ball. Prove that, irrespective of the number of
q A
balls the boy has or the speed of the balls, the car
O cannot travel more than L.

4. A hemispherical shell has mass M. A liquid is filled


in it upto the brim.
O R
Mass of the liquid is
2m. Find the height of
the COM of the liq-
uid-filled hemisphere, L
measured from the
ground.
8. A wedge of mass M carries two blocks A and B, as
5. A disc of radius r is uniform. A square-hole of side shown in figure. Block A lies on a smooth-incline
r__
length l = ​ ___   ​ is punched in the disc, as shown. One surface of the wedge inclined at angle q to the hori-
÷   
​ 2 ​ zontal. B lies inside a horizontal groove made in the
corner of the hole is at centre O of the disc and the wedge. The two blocks have mass m each and are

1.31
connected by a light string. Length of the groove
is l, as shown. The entire system is released from
rest. Find the distance travelled by the wedge on the
smooth horizontal surface by the time block B comes
out of the groove.

B 11. A conical container has a circular base of radius R



and height 4R. It is filled with a liquid of density d.
q
Below the cone, there is a cylindrical container of
radius R. Base of the cylinder is at a depth 2R below
9. A solid hemisphere has radius R. Density of the solid the apex of the cone. Cap at the apex of the cone is
changes with distance r from the centre O, as opened and the liquid slowly drains into the cylindri-
r = rO ​ __r  ​. Find the position of the COM of the cal containers. Calculate work done by gravity in the
R process.
hemisphere.

R
R

r 4R

10. A rope has length 3L and is made of three uniform


2R
segments of length L each. Mass of the two end-
segments are m and 2m. the mid segment has mass
1.5 m. The rope is placed over a smooth equilateral
R
wedge fixed on a horizontal table. The rope stays in
equilibrium. Find y co-ordinate of the COM of the
rope, measured from the apex point O.

1.32
Answers Sheet
Your Turn

4d 13
1. ​ ___ ​   2. 6.5 m 3. Zero 4. (i) – 6.5 cm (ii) ___ ​   ​ cm
5 6
( 
3
5. ​ __
2
1
​   ​  + __
2
1
​   ​  – __ ) 5L
​    ​   ​ m 6. ​ ___ ​  
2 9
L__
7. ​ ____
   ​  
  
2​÷2 ​
( 5 5
8. ​ __ )
​    ​ a, __
6 6
​   ​  a  ​

2R R 2R
9. ​ ___ ​  10. ​ __ ​   11. ​ _____     ​ from O 12. At the centre of the Earth
3p 6 2+p
13. No 14. (a) yes (b) Speed becomes the same as original speed
L
15. (i) (– 2.4  + 1.6  ) cms–1 (ii) 4  cms–2 (iii) 8  dyne 16. ​ __ ​ 
2
u ​_› v
__
17. vcm = ​   ​   18. COM continues to move along a straight line, with velocity ​v  ​ cm = __
​    ​ (–  + )
5 2
19. 5 m 20. (i) At the centroid of the triangle (ii) Will not move
mR sin q
21. (400, – 91, 32) m 22. ​ _______  ​ 23. 2lgR2 24. 6 J.
m+M
25. 0 26. yes

Worksheet 1
1. (b) 2. (c) 3. (b) 4. (d) 5. (c) 6. (a) 7. (b) 8. (c) 9. (a)
10. (b) 11. (c) 12. (b) 13. (d) 14. (a) 15. (c) 16. (b) 17. (b) 18. (a)
19. (d) 20. (a) 21. (d) 22. (a) 23. (d) 24. (c) 25. (b) 26. (b) 27. (a)
28. (b) 29. (b) 30. (b) 31. (b) 32. (b)

Worksheet 2
1. (c, b) 2. (b, d) 3. (b, c) 4. (b, d) 5. (a, c) 6. (d) 7. (b, c, d) 8. (d) 9. (b)
10. (d)

Worksheet 3

1. ​ ________
(  )
R sin ​ __
q
​   ​   ​
2
 ​  

   R
2​÷5 ​
2. ​ _____ 
__
 ​
__
(3​÷2 ​
3. ​ ___________
__
  )  R
    – ​÷5 ​
 ​  
___  
7R
4. ​ ___ ​ 
q
(  )
​ __
​   ​   ​
2
p+4    
2​÷10 ​ 12

( 
ml (1 – cos q)
​ 
– r
5. ​ O, ________   
2 (2p – 1)
 ​  ​ ) 6. (i) 8 m, (ii) 9 m, (iii) 5 cm 8. ​ ___________
2m + M
     ​

2R 52p 4 
9. ​ ___ ​ from O on symmetry axis 10. 0.63 L 11. Wg = ____
​   ​ 
 R ◊ d ◊ g
5 9

1.33
Chapter  2

Momentum and Its Conservation


“The world is wide, and I will not waste my life in friction when it could be turned into momentum.’’
–Frances E. Willard

​_› ​_›
1.  Introduction ​P  ​   = m​v  ​   ...(1)
–1
A heavy truck and a car are moving with same velocity. Its unit is kg m s which is same as Ns. It is a vector
Which has more “motion” in it? Certainly, the truck. It and any change in direction of motion implies a change in
is very difficult to stop it. If it hits a wall, it will cause momentum.
more damage then the car. We say that the truck has more Momentum of an extended body or a collection of
“quantity of motion” than the car. The physical quantity particles is the sum of momenta of individual particles.
measuring this “quantity of motion” is called momentum. As studied in last chapter, momentum for a collection of
Similarly, a fast-moving bullet can cause more harm than a particles is given by product of total mass and velocity of
slow-moving ball, though mass of ball is more. COM. ​_› _
​› _
​›
Therefore, “quantity of motion” (i.e, momentum) possessed ​P   ​ = Â mi ​v   ​i = M ​v   ​cm ...(2)
by a body depends on two things – its mass and its velocity.
The product, mass × velocity, is known as momentum of a Example 1   A ball of mass 1 kg hits a wall normally at a
body. Momentum of a body is closely related to force acting speed of 10 ms–1 and rebounds at a speed of 10 ms–1. Find
on it. change in momentum of the ball.
In this chapter, we will understand that momentum of a Solution
system remains conserved in absence of any external force.
Concepts
This is a very strong conservation principle in physics and
helps us in solving diverse kinds of problems. We will be Momentum is a vector. Change in direction of a vector
able to understand collision problems and rocket propulsion. implies that the vector has changed.
Later, you will learn that conservation of momentum helps
us in understanding physics of sub-atomic particles also. DP = Pf – Pi
= mv (¨) – mv (Æ)
2.  Momentum = mv (¨) + mv (¨)
​_›
Momentum of a particle of mass m moving with a velocity ​v  ​  = 2mv (¨) = 2 × 1 × 10 (¨)
is defined as = 20 kg ms–1 (¨)

Your Turn

Q.1  Which has larger momentum — a 50 g bullet travelling Q.3  A projectile of mass m is projected at an angle q to
at 500 ms–1 or a 80 kg bicycle (with rider) travelling at the horizontal at speed u. Find change in its momentum by
9 kmh–1? the time
Q.2  A car and a truck have same kinetic energy. Which (i) it reaches the top point of its trajectory.
has more momentum? (ii) it is about to hit the ground.
2.2  Mechanics II

3. FORCE AND MOMENTUM (NEWTON’S (ii) The force experienced by the ball will be in the
SECOND LAW) direction of change in its momentum.
(iii) From Newton’s third law, the force applied by the
Newton, originally, expressed his second law in terms of ball or the wall is same as the force applied by the
momentum. According to this law: wall on the ball.
The rate of change of momentum of a particle is equal
to the net external force acting on it and the change in
momentum always takes place in the direction of the force.
​_›
d ​P   ​ _​ ›
​ ___ ​ =   ​F  ​ext
  ...(3)
dt
This, means that a force is necessary to produce a change
in momentum of a particle. If there is no force, momentum
cannot change.
​_› _
​›
If we put ​P   ​ = m​v   ​ in the above equation and assume that
the mass (m) of the particle is constant, we get Momentum of the ball before collision is
​_› _
​›
​_›
m d ​v  ​ 
_____ ​P  ​ 1 = mu cos q   – mu sin q 
​     
​  = ​F  ​ ext
dt
​_› ​_› Momentum after collision is
fi m ​a  ​   = ​F  ​ext
  ...(4) ​_›
​P  ​ 2 = – mu cos q   – mu sin q 
Thus, the two relations (3) and (4) are equivalent when
mass of a particle stays constant. \  Change in momentum of the ball
_
​› _
​› _
​›
If we have a system of particles, its momentum is given D​P  ​   = ​P  ​ 2 – ​P  ​ 1 = – (2 mu cos q) 
by equation (2)
​_› ​_›
= 2mu cos q (¨)
​P  ​   = M ​v  ​ cm
This change in momentum of the ball happened during
​_› ​_›
d ​P  ​  d ​v  ​ cm its interaction with the wall for an interval D t. Hence, the
___
fi ​   ​   = M ​      _____
​, assuming M to be a constant average force experienced by the ball is
dt dt 1
​_› 2 × 0.5 × 10 × __
​   ​ 
D P 2m u cos q 2
= M ​a  ​cm
  F = ___
​   ​ = _________
​      ​ (¨) = ​  ______________
   ​
    
​_› D t D t 0.01
We already know that M ​a  ​cm
  is the net external force on
the system. Therefore, = 500 N (¨)
​_›
d ​P  ​  ​_› The wall experiences 500 N force towards right.
___
​   ​   = ​F  ​ext  
dt Note:  Actually, the force between the ball and the wall will
This relation is true for a single particle or a system of change in some complex
particles. way during the interac-
tion period of Dt (see
Example 2   A ball of mass m figure). What we have
impinges on a wall at a speed u, at calculated is the average
an angle of incidence q. The ball force during interval D t.
rebounds at the same speed at an angle The instantaneous value
of reflection q. The ball interacted with of the force must have
the wall for a time interval D t = 0.01 s. a peak value larger than
Find the average force applied by the 500 N. Colliding bodies
ball on the wall if m = 0.5 kg and usually exert large force
u = 10 ms–1, and q = 60°. on one another.
Solution Example 3   Bullets hitting a wall
Concepts A gun is firing bullets on a vertical mud wall. Bullets
(i) The ball experiences a change in momentum. This get embedded into the wall. Each bullet has mass m and
happens because it experienced a force (from the speed u. Bullets hit the wall while flying horizontally. Find
wall). Average value of this force is the average force experienced by the wall if the gun fires n bullets per

(  )
D P second.
rate of change of momentum of the ball ​ ___
​   ​  ​
D t
Momentum and Its Conservation  2.3

Solution What is the maximum force that the falling chain will
Concepts apply on the table at any instant?

(i) The rate at which momentum of impinging bullets Solution


change is equal to the force applied by the wall Concepts
on the group of bullets hitting it. Same amount of
(i) In principle, this problem is not much different
force is applied by the bullets on the wall.
from the last one. When one small piece of the
(ii) If D P = change in momentum of all bullets that falling chain hits the table, it comes to rest, just
hit the wall in a time interval D t, then force is like a bullet hitting the wall and coming to rest.
D p
F = ​ ___ ​  Momentum of a falling piece changes due to the
D t force it experiences from the table.
Change in momentum of one bullet when it hits the wall   In the last problem, we could count the number
is of particles (bullets) hitting the wall. In this prob-
lem, infinite number of tiny particles of the chain
D Pone = Pf – Pi = 0 – mu (Æ) = mu (¨)
hit the table one after another. Also, each succes-
Note that the bullet comes to rest inside sive particle hits the table with a slightly greater
the wall and hence its final momentum is speed, as the speed of falling chain is increasing.
zero. (ii) The part of the chain that is vertical (in air) is
Also, the bullet experiences a force falling down with acceleration g. There will be
towards left and change in its momentum no tension force between two successive links. (If
is towards left. you place two books one over another and release
Number of bullets hitting the wall in an interval D t is them, both fall with acceleration g and there is no
nD t. normal force between them.)
\  Change of momentum of all bullets hitting the wall
in time D t is
D P = (n D t) (D Pone) = (n D t) (mu) (¨)
D p
\  Force = rate of change of momentum = ​ ___ ​ 
D t
= n mu (¨)
This is the force experienced by the bullets (as a group)
due to the wall. The wall experiences same force towards
right.
\ F = n mu (Æ)
Consider the time when x length of the chain has already
Note:  Tracking the direction (using Æ or ¨ sign) was landed on the table. The chain that is in air has fallen through
not essential in this problem, as it is obvious that the wall a distance x. Its velocity is
will experience a force towards right. Just focusing on mag-
____
nitude of D P would have sufficed.   ​ 
v = ​÷2gx 
Example 4   A falling chain A very small element of length dx at the bottom of the
A uniform chain of mass M and length L is held straight chain will land on the table in next small interval of time
above a horizontal table with dt. Momentum of this element just before it hits the table
its lower end just touching the is

[ 
table. The chain is released
from this position. Find the
force exerted by the chain on
(  )
M
dp = ​ __
L
M
​   ​  dx  ​ v ​ __ ]
​   ​  dx is mass of the element  ​
L
the table when its length x has In next ‘dt’ time, its momentum becomes zero due to force
piled up on the table. Assume by the table. Hence, change in momentum has magnitude
that each link of the chain
comes to rest on hitting the
table and neglect the height of pile formed on the table.

M
(  )
dp = ​ __
​   ​  dx  ​ v
L
2.4  Mechanics II

dp M ___ dx M 2
\  Force F = ​ ___ ​ = __
​   ​  v . ​   ​ = __
​   ​  v
dt L dt L
M x
= __
​   ​  (2gx) = 2Mg ​ __ ​ 
L L
This is thrust force applied by chain links hitting the
table. Obviously, the part of chain (length x) already lying
Volume flow in unit time = Av. Mass of water that flows
on the table is exerting a force equal to its weight on the
through the pipe in unit time is r Av. In a time interval D t,
M
table. This force is __ (  )
​   ​  x . g = Mg ​ __
L
x
​   ​   ​.
L the mass of liquid that flows is
Thus, total force due to chain (hitting + weight) on the D m = r Av D t.
table is
Its momentum changes from P1 = D mv (Æ) to
x x x
2Mg __ ​   ​  + Mg __
​    ​ = 3Mg __ ​    ​
L L L P2 = D mv (Ø)
This force is maximum when x is maximum (= L), i.e., \  Change in momentum has magnitude
when the last links hits the table.
Maximum force is
L
Fmax = 3Mg ​ __ ​ = 3Mg
L
Example 5   Force on a bend in a water pipe
Water (density r) is flowing at
speed v through a pipe of cross-
sectional area A. The pipe has a
right angled bend as shown. Find
the necessary force to keep the __
bend fixed. DP = ÷     mv and is in a direction making 45° with either
​ 2 ​
Solution sides of the bend.
Concepts \  Force by pipe on water is
__ __
(i) Direction of momentum of liquid particles change D P ÷     D mv
​ 2 ​     r Av D t ◊ v
​÷2 ​
F = ___
​   ​ = ​ _______ ​ = ​ ___________
           ​
at the bend. This means that momentum of liq- D t D t D t
uid particles change as they cross the bend. This __
change in momentum happens due to force applied fi F = ÷     r Av2
​ 2 ​
by the pipe on the liquid. Liquid also applies equal ​___›
force in opposite direction on the pipe. To keep the This forces is in direction of DP​​   shown in figure. Water
pipe in place, we need to apply an external force exerts equal force on pipe in opposite direction (shown as
to balance this force. F in above figure). To__ keep the pipe in place, the external
(ii) Volume of liquid flowing through a cross-section     r Av2 in direction opposite to F.
agent applies Fext = ​÷2 ​
in unit time is Av.

Your Turn

Q.4  Car manufacturers undertake crash tests of cars to time interval of D t = 0.15 s and maintained its straight line
gauge the extent of damage caused when an accident occurs. trajectory. Just after the collision it was travelling at 18 kmh–1
In one such test, a car of mass M = 1500 kg travelling at in same direction.
54 kmph is made to hit a (i) Find the average force experienced by the car during
temporary wall. The video collision?
shoot of the event sug- (ii) Find loss in KE of the car.
gested that the car inter-
acted with the wall for a Q.5  A ball of mass m = 50 g falls from a height of
5 m and rebounds to a heights of 1.25 m. The ball was in
Momentum and Its Conservation  2.5

contact with the floor for an estimated time of D t = (0.11 Q.8  Wind is blowing horizontally at a speed v. Density of
± 0.01) s. Estimate the average force applied by the floor air is r. Calculate force experienced by a vertical wall of
on the ball. area A. Assume that the air spreads parallel to the surface
Q.6  Momentum of a of the wall after striking it. How will the force change if
particle, travelling on speed increases to 2v?
a straight line, changes Q.9  A wide container of negligible mass is placed on a
with time, as shown in weighing scale. A tank
the figure. Find the max- at height H pours water
imum force acting on the on it. Assume that
particle at any instant. water comes out of the
Q.7  A car of mass M tank at negligible speed
enters a semi-circular and at a rate of m kg s–1.
track at speed v and exits out of it at speed v. The track The falling water does
has radius R. Find the average force experienced by the car not splash and spreads
on the track. uniformly on the con-
tainer. Find the reading
of the weighing scale at
a time t after the water
starts pouring into the
container.
Q.10  An army person fires 50 g bullets horizontally from
a machine gun. Bullets leave the gun at a speed of 1 kms–1
and he fires 20 bullets in 4 s. Find the average horizontal
force that he exters on the gun to hold it.

D t ​_
4.  IMPULSE ​J  ​   = ​Ú ​   ​ ​ ​F  ​  . dt
​_› ›
...(6)
0
We known that a force produces acceleration in a body (of
fixed mass) and thereby, changes its momentum. The greater When a graph of F versus t is given, the area enclosed
the force acting on a body, the greater its change in velocity, between the graph and time axis gives the impulse.
and hence the greater its change in momentum. But one
more thing is important in changing momentum–time. By From newton’s second law,
​_›
applying a force over an extended period of time, one can d ​P   ​ _​ ›
​ ___ ​ =   ​F  ​ 
produce a large change in momentum of a body. Both force dt
and time interval for which the force is applied are important for a particle or a system of particles.
in changing momentum. ​_› ​_›
The quantity–force × time is called impulse. fi d ​P  ​   = ​F  ​  dt ...(7)
_
​›
​_› ​_›
Impulse (​J  ​)  = ​F  ​  ◊ D t ...(5) This equation says that a force ​F  ​  acting on a body for
an infinitesimally small time interval dt produces a change
Unit_​ ›of impulse is Ns. It is a vector in the direction of in momentum of the body, given by equation (7).
force (​F  ​)  .
When the force acts for a finite time, change in momen-
When force is changing with time, the impulse of the tum can be obtained by integrating the above equation on
force can be written as both sides. ​_›
​P  ​ f _
​› Dt _
​›
​_Ú  ​ ​d ​P  ​   = ​Ú ​  ​ ​ ​F  ​  dt
​›
​P  ​i  0

​_› ​_› D t _ ​›


Ú 
fi ​P  ​ f – ​P  ​ i = ​  ​  ​ ​ ​F  ​  dt
0
​___› _
​›
fi ​DP​ =    ​J  ​   ...(8)
This equation says that change in momentum of a body
is equal to impulse of external force applied on it. Impulse
and momentum both are vectors having same unit – kg ms–1
or Ns.
2.6  Mechanics II

We will refer to equation (8) as Impulse–momentum Example 7   A cricket ball of mass 150 g hits a bat normally
theorem. while travelling at a speed 144 kmh–1. It bounces back at a
speed of 108 kmh–1 along the original line of motion. Find
Example 6   A block of the impulse received by the ball.
mass M = 4 kg is kept on a
smooth horizontal surface. A Solution
horizontal force F starts act- Concepts
ing on it. Find velocity of the block after 2 s if
Though we do not known the exact nature of the force
(i) the force F remains constant at 10 N. between the ball and the bat and also we do not know
(ii) the force F varies with time as F = (2t) N. the duration for which they interacted, we can still find
(iii) the force F varies with time, as shown in the the impulse by knowing the change in momentum of ball.
graph. Actually, the force must have varied in a complex manner
during the interval of interaction and the interaction time
will be very small. But without getting into these details
we can find the impulse.

For ball:
Initial momentum is
Pi = mu = 0.15 × 40 (¨)
Solution
= 6 kg ms–1 (¨)
Concepts
[   144 kmh–1 = 40 ms–1]
(i) Change in momentum = Impulse
After the hit, final momentum of the ball is
(ii) Impulse = F Dt for constant force
D t Pf = mv = 0.15 × 30 (Æ) = 4.5 kg ms–1 (Æ)
= ​Ú ​   ​ ​F dt for variable force
0 [108 kmh–1 = 30 ms–1]
(iii) The problem can also be solved by writing Change in momentum of the ball is
F
acceleration a = __
​    ​  and then using v = u + at for
M v D t
D P = Pf – Pi = 4.5 (Æ) – 6 (¨) = 10.5 kg ms–1 (Æ)
constant acceleration and Ú​  ​  ​dv = Ú​  ​   ​ ​a dt for variable \  Impulse applied by the bat on the ball is
u 0
acceleration.
J = D P = 10.5 Ns (Æ)
(i) Impulse = F D t = 10 × 2 = 20 Ns
Note that the ball applies same impulse in opposite direc-
\ D P = 20 tion on the bat.

Pf – Pi = 20  fi  Mv – 0 = 20 4.1  Impulsive Force


When you kick a football, your feet interacts for a very
20
fi v = ___
​   ​ = 5 ms–1 small time (D t) with the ball. But the force experienced by
4
the ball is relatively large (compared to other usual forces on
2 2 the ball – like its weight). The impulse received by the ball
(ii) Impulse = ​Ú ​    ​ F dt = 2 Ú​  ​  ​ t dt = 4 Ns (F Dt) is significant and the momentum of the ball changes
0 0
almost abruptly (in very small time D t) by a magnitude
\ Pf – Pi = 4  fi  Mv – 0 = 4 D P = F Dt.
The ball certainly gets displaced during the intervals
fi v = 1 ms–1
(D t) when force acted. But this displacement is too small
(iii) Impulse = Area under F–t graph (to catch our attention) compared to the displacement that
1 the ball is going to have after the kick. For us, the process
= ​ __ ​  × 2 × 10 = 10 Ns of kick was nearly instantaneous. The ball suddenly acquired
2
a momentum, in no time, while being at the same location
\ Pf – Pi = 10  fi  Mv – 0 = 10 (almost).
10
fi v = ___
​   ​ = 2.5 ms–1
4
Momentum and Its Conservation  2.7

Such a force of relatively large magnitude acting on a Solution


body for a very short interval of time is called an impulsive Concepts
force.
The duration for which the foot applies force on the ball (i) Applied force is large. Normal reaction, which
being small, we can neglect the impulse of a finite force depends on the applied force is also large. Therefore,
like weight of the ball. For the interval D t, the applied force friction is large and has significant impulse. This
F was way too large compared to weight (mg). Change in time, we cannot neglect the impulse of friction
momentum of the ball during interval D t is during the small time interval (D t) for which the
​___› ​_› _
​›
force was applied.
​DP​ =
   ​F  ​  D t + m​g  ​  D t (ii) You must carefully note why the impulse of friction
_
​›
​F  ​  ◊ D t was insignificant in last example and why it has
become significant in the present problem.
A kick, a jerk, a punch
or a collision involve
forces of large magni-
tude. They are impul-
sive forces. They cause
large change in momen-
tum during a very small
interval of time without
causing appreciable dis- Impulse of mg is negligible
placement of the body. compared to impulse of F.
If the applied force is F, the normal reaction will be
Example 8   Neglect impulse due to friction N = F sin q + mg
A block of mass M = 2 kg is placed on a horizontal table. 3
Coefficient of friction is m = 0.2. A sharp horizontal blow is F sin q = ​ __ ​  F
5
applied to the block. Impulse We have neglected mg as it must be a small force com-
imparted is J = 10 Ns. Find pared to the large impulsive force F.
the velocity acquired by the
3
block due to the blow. \  Friction F = mN = __ ​   ​  mF
5
Solution
Net impulse in x-direction is
Concepts
Jx = (F cos q) D t – f D t
Friction is a finite force and we will neglect its impulse
4 3
during the small interval for which the blow lasted. Impulse = __
​   ​  (F D t) – __
​    ​ m (F D t)
due to the applied force is J = 10 Ns. Impulse due to fric- 5 5
tion (in the interval for which the force was applied) must The question directly gives us the value of impulse
be << 10 Ns.
F D t = J = 10 Ns
D P = J 4 3
Jx = __
\ ​   ​  (10) – __
​   ​  × 0.2 (10) = 8 – 1.2
5 5
10
fi Mv – 0 = 10  fi  v = ​ ___ ​ = 5 ms–1 = 6.8 Ns
2
Example 9   Significant impulse due to friction
D Px = 6.8
6.8
In the last example, consider the sharp impulse (J = 10 Ns) fi Mv – 0 = 6.8  fi  v = ___
​   ​  = 3.2 ms–1
2
to be applied at q = 37° to the horizontal. Find the velocity
of the block immediately after the impact. Example 10   Impulsive tension
In an Atwood machine, a block (B) of mass 2m rests on a
table. It is connected to a ball of mass m using a light string
passing over a smooth pulley. The ball of mass m is raised
to a height h from its initial position and released.
(i) Find the speed of block B immediately after the string
becomes taut.
2.8  Mechanics II

Impulse on both the bodies is upward and has same


magnitude J as tension on both is same at any point of
time.
It is important to note that impulse of weight on the two
objects can be neglected for the small interval of time that
we are considering
For B: J (≠) = Pf – Pi
fi J (≠) = 2m v (≠) – 0
fi J = 2mv ...(i)
(ii) Write the kinetic energy of the two-block system
just before and just after the string gets taut. What For A: J (≠) = Pf – Pi
happened to the kinetic energy? why?
fi J (≠) = mv (Ø) – mu (Ø)
Solution
fi J = mu – mv ...(ii)
Concepts
Eliminating J between (i) and (ii)
(i) The string is loose and ball A experiences free fall

÷ 
____
till its displacement becomes h. u 2gh
3mv = mu  fi  v = ​ __ ​  = ​ ____
​   ​ ​ 
  
(ii) During the period (a small time) in which the 3 3
string gains tension, it applies an impulse on both We can now write the impulse applied by the string on
the blocks. This impulse sets B in motion and the two object using (i)
decreases the downward momentum of A. Just after (ii) KE just before the string has tension is
the string is taut, the two objects will have same
1
speed – B going up and A going down. Ki = __
​   ​  mu2
2
(iii) We will write the impulse-momentum equation for
both the bodies for interval during which the string KE just after the string has tension is
gains tension. 1 1 3
Kf = __
​   ​  (2m) v2 + ​ __ ​  (m) v2 = __
​   ​  mv2
2 2 2
(i) Speed of the ball after falling through h is
1
____ = ​ __ ​  mu2
  ​ (first figure)
u = ​÷2gh  6
1 1
Loss in KE is D K = Ki – Kf = __
​   ​  mu2 = __
​   ​  m (2gh)
3 3
2
= ​ __ ​  mgh
3
Where goes the kinetic energy?
Actually, the string stretches a little when it is taut. Energy
is spent in deforming the string – just like the energy spent
to stretch a spring. No string can be perfectly inextensible.
Though the stretch may be small enough to make any sig-
nificant change in its length, it may require a lot of energy
to stretch a string, even by a small amount.

In short: ​_› ​_›


In the next interval D t, the string will acquire full ten- (i) Momentum of a particle is ​P  ​  = m​v  ​  and momentum
​_› ​_›
sion. After this, both objects will have same speed v (last of a collection of particles is ​P  ​  = M ​v  ​CM
  .
figure). (ii) Momentum of a system can change only when an
During the interval D t, the tension might have changed external force acts on it. _
​›
in a complicated way. Forget about the way the tension d​P  ​  ​_›
___
D t
(iii) Rate of change of momentum is given by ​   ​ = ​F  ​ ext.
dt
changes. Let us assume that its impulse ​Ú ​   ​ ​T dt is equal to J ​_› ​_› ​_›
0
This equation is same as ​F  ​ext
  = M ​a  ​  (= M ​a  ​CM
  for a
during this interval.
system is particles) if mass is constant.
Momentum and Its Conservation  2.9

(iv) If n particles strike a surface per second and each ​___›


(vi) A large force acting for a short interval of time is
particle suffers a change in momentum equal ​DP​ , 
to​___ often known as an impulsive force. Usually, finite
​_› ›
then force experienced by the surface is ​F  ​  = n ​DP​ .  forces like weight have negligible impulse during a
​_› D t ​_ quick process like collision.

(v) ​J   ​ = ​Ú  ​  ​ ​F   ​ dt is known as impulse of a force. Impulse (vii) When a string gets taut, it consumes energy. In fact,
0
deformation of all objects consume energy. When a
experienced by a body is equal to change in its
car meets an accident, its body gets deformed. This
momentum. This result is called Impulse – momentum
consumes a lot of kinetic energy and, in a way, pro-
theorem. _
​› tects the passengers.
D P = ​J   ​

Your Turn

Q.11  Give reason for following: Q.16  Two balls of masses m


(i) When boxing, if you move into a punch instead of and 2m are placed on a smooth
away, you are in trouble. surface connected with a loose
massless string. The ball of
(ii) You find it convenient to catch a high-speed cricket
mass m is imparted a sudden
ball while your hand moves away from the ball.
velocity u in a direction that is
(iii) When your car is out of control, if you drive it into parallel to the line joining the
a concrete wall instead of a haystack, you are really initial positions of the balls. Find the impulse imparted by
in trouble. the string to set the ball of mass 2m into motion.
Q.12  A block of mass m = 2 kg begins to slide on a Q.17  A particle of mass 1kg is moving in free
​ › space.
_
​_
smooth incline having inclination angle q = 30°. Find the ›
Its velocity is ​u  ​  = (2  + 3  ) ms–1 when a force ​F  ​  = 2t
impulse of all the forces acting on the block in a time of newton starts acting on it. Find the velocity of the particle
t = 2 s after the motion starts. at t = 2 s.
Q.13  How much impulse is needed to slow down a Q.18  A projectile of mass m is fixed from ground with
_
1,000 kg car from a speed of u = 20 ms–1 to a final speed ​ ›
a velocity ​u  ​.  Find its momentum after time t (< time of
v = 10 ms–1? flight).
Q.14  A block of mass M = 10 kg is kept on a horizontal Q.19  A particle of mass
surface having coefficient of friction m = 0.2. A stick gives m = 1 kg is moving along
a sharp horizontal blow to a straight line with speed
the block. The block begins u = 4 ms–1. A force (F)
to move and stops after trav- starts acting on it in the
elling a distance s = 1.0 m. direction of its motion.
Find the impulse that the stick imparted to the block. The force acts on the par-
Q.15  Consider the situation described in example 4. What ticle for 8 s and changes
is the impulse applied by the chain on the table during the with time as shown in the figures. Find work done by the
interval the complete chain falls on the table? force in 8 s.

5.  CONSERVATION OF LINEAR We also know​_› ​_›


that velocity of COM remains constant
(even when ​v  ​1  , ​v  ​2  ... etc. are changing) when net external
MOMENTUM force on the system is zero.
We studied, in the last chapter, that momentum of a system This implies that the total momentum of a system remains
having n number of particles can be written as constant (i.e., conserved) when sum of all external forces
​_› ​_› ​_› ​_› ​_› acting on the system is zero. This is known as principle of
​P  ​   = m1​v  ​1  + m2​v  ​2  + ... + mn​v  ​n  = M ​v  ​CM

conservation of momentum.
Where
​_›
M = m1 + m2 + ... + mn = total mass of the system It is important to note that momentum of individual
and ​v  ​ CM = velocity of COM of the system. particles (or parts of the system) may change due to internal
2.10  Mechanics II

interactions. But the total momentum of the system cannot Example 11   Block A, having mass m, is placed on a
change if there is no external force. smooth horizontal surface.
Consider two positively charged particles released from Another identical block (B) is
rest. Initial momentum of the system is zero. If the system sliding towards A at a veloc-
is isolated, i.e., free of any ity u. They collide. Find their
external force, momentum speeds if
will remain zero forever. If (i)  They stick  (ii)  B comes to rest
we look at individual parti- What happens to kinetic energy of the system?
cles, they acquire momen-
tum due to force applied by Solution
the other particle. But they Concepts
move such that Psystem = 0
(i) External force on the system (A + B) is zero. The
​_› ​_› momentum of system is conserved. Both A and
fi m​v  ​1  + m​v  ​2  =0
B apply large forces on one another and change
​_› ​_›
fi m ​v  ​ 1 = – m​v  ​ 2 each other’s momentum. But there is no change in
​_› ​_›
overall system’s momentum.
Here, negative sign indicates that ​v  ​ 1 and ​v  ​ 2 have opposite (ii) Internal forces can perform work and change KE
directions. of the system.
There may be a situation when there is no external force
acting on a system in a particular direction (say x-direction) (i) Let final velocity of two blocks after they stick
though there may be a force in other directions. One can be v.
always conserve momentum in x-direction, whenever such
Pf = Pi
situation occurs. If means:
u
Px = a consant if Fx = 0  [Fy and Fz may not be zero] fi 2m ◊ v = mu  fi  v = __ ​    ​
2
...(9) Kinetic energy of system has initial and final values
When a projectile moves through its path, horizontal as:
1 1 1
components of its momentum remains constant, as there is ki = ​ __ ​  mu2  and  kf = __ ​   ​  (2m) v2 = __
​   ​  mu2
2 2 4
no horizontal force on the projectile.
1
The conservation of momentum, as sated above, will be \  KE decrease by __ ​   ​  mu2.
4
very difficult to apply in practice, as it is difficult to find
systems which are completely free of any external force. Let (iii) Let final velocity of A be v.
us put the principle in a slightly different from. Velocity of B after collision = 0
​_› D t ​_
​_› ​___› ​_›
d​P  ​  › \ Pf = Pi  gives
​   ​   =
dt
___ ​F  ​   Ú 
fi ​DP​  = ​  ​  ​ ​F  ​  dt = ​J  ​ 
0
mv = mu
The above equation is impulse-momentum relation, which
says that change in momentum of a system is equal to the fi v = u
impulse of external force. Consider a system, which is being
There is no change in kinetic energy.
acted upon by some finite-sized (non-impulsive) external
force. If an event (like collision) takes place, which lasts
​_› for   Example 12   Block A, of mass m, rests on a rough
a very small interval of time (D t), then the impulse (​J   ​) of horizontal surface. Another identical block B moving along
the force can be neglected and we can still say that D P  0. the surface hits block A and sticks to it. Speed of B, just
This means momentum is (nearly) conserved. before collision is u. The combined block travelled a distance
If two projectiles collide in mid-air, we can say that the s along the surface and stopped. Find s if the coefficient of
momentum of system of two objects just before collision friction between the blocks and the surface is m.
will be equal to momentum of the system just after collision.
Remember, there is an external force – weight – acting on
the system. Still, we are saying that we will use momentum
conservation! The impulse of weight in the short duration
for which the projectiles interacts is negligible.
Momentum and Its Conservation  2.11

Solution Solution
Concepts Concepts
There is friction between the blocks and the surface. Shall Bomb is an isolated system – free of external force. It
we apply conservation of momentum? Yes, we can write blasts due to internal reasons (some chemical reactions).
Pbc = Pac, where Pbc and Pac are momentum of the system The momentum of system will remain conserved equal
‘just before collision’ and just after collision’. During the to zero.
short time for which the blocks applied force on one-
another, the impulse of friction (f D t) is negligible.

Momentum of first piece is


m
( 
P1 = (100 g) ​ 20 __ )
​ s ​   ​ = 2 kg ms–1 (North)
Just before collision
Momentum of second piece is
Pbc = mB uB + mA uA
= mu + 0 = mu

m
( 
P2 = (200 g) ​ 10 __ )
​ s ​   ​ = 2 kg ms–1 (East)

Sum of momentum of first two pieces is


Just after collision, let us assume that the blocks move _______ __
with a common velocity v.   12 + P22 
P = ​÷P    kg ms–1 (NE)
 ​ = 2​÷2 ​ 
Pac = 2mv Sum of momentum of all three pieces must be zero.
Momentum conservation:  Pac = Pbc Hence, momentum of third piece (P3) is equal to P in direc-
tion opposite to P.
u __
fi 2mv = mu  fi  v = __
​   ​  \     (SW)
P3 = 2​÷2 ​
2

(  )
50 __
The friction retards the two-block system to rest as they ​ _____   
​    ​  ​ v3 = 2​÷2 ​
travel through distance s. Using work-Energy theorem: 1000
__
fi     ms–1
v3 = 40 ​÷2 ​
Wf = kf – ki

(  )
1 u 2 Example 14   Monkey lands on a car
– m (2mg) s = 0 – ​ __ ​  (2m) ​​ __
​   ​   ​​ ​
2 2 A car of mass M is moving uniformly on a smooth road at
u 2 a velocity u. A monkey of mass m is hanging from a tree
fi s = ____
​     ​ branch. On seeing the
8 mg
car, the monkey releases
Example 13   A bomb blast
the branch and falls verti-
cally. It lands on the car
A bomb lies at rest. A remote is pressed and it blasts.
and quickly steadies itself
However, the blast turned out to be a failure and the bomb
on the car. Find the speed
could just split into three mass chunks. A piece of mass
of the car after the event.
100 g was found moving due north at a speed of 20 ms–1
How does height of the
and another piece of mass 200 g was moving towards east
tree affect you answer?
at a speed of 10 ms–1. Find the velocity of the third piece
of mass 50 g, immediately after the blast.
2.12  Mechanics II

Solution Solution
Concepts Concepts
(i) Just before the monkey lands on the car, it has (i) The monkey and the car, both are moving at
a vertically downward velocity and the car has a velocity u. The interaction between the two cannot
horizontal velocity. There is no horizontal force change the overall momentum of the system.
on the (monkey + car) system and the horizontal (ii) The monkey jumps vertically relative to the car. The
momentum remains conserved. monkey retains its horizontal motion during the jump
(ii) Initially, the monkey has no horizontal motion but (recall inertia). In addition to its horizontal motion,
as it lands on the car, friction applied by the car on it acquired a vertical motion also.
its body provides it a horizontal motion. Friction (by
monkey) on car slows down the car.
But we need not think about all these details. These
are internal force in our system. Conservation of
momentum helps us avoid digging too much into the
internal forces!
(iii) Where has the vertical momentum (possessed by
the monkey) gone? The road applied a big vertical After jump, the monkey still has a horizontal velocity u.
impulse on the car when the monkey lands on it. For no change in horizontal momentum, the car must
This impulse has caused the vertical momentum of continue to move with velocity (u), which is same as velocity
the system to change and become zero. before jump.
An observer on the ground will find that the monkey has
a horizontal as well as vertical velocity. For him, the path
of the monkey will be parabolic.

Example 16   Man jumps out of a moving car


A flat car of mass M is moving on a smooth horizontal
Consider monkey + car as our system. Momentum is surface with velocity u. A man of mass m stands at the
conserved in horizontal direction. front edge of the car. The man
jumps horizontally at a velocity
Momentum before landing = momentum after landing
ur relative to the car in forward
mu = (M + m) v direction. Find velocity of the car
mu after the jump.
fi v = ______
​     ​
M+m
Solution
Height of the tree has no relevance in deciding v.
Concepts
Example 15   Monkey jumps out of a car (i) Momentum of man + car system is conserved in
A car of mass M is travelling on a road. A monkey of mass horizontal direction.
m is sitting on it. The monkey sees a mango tree and jumps (ii) After the man jumps, velocity of the car changes.
vertically relative to the car. It catches a tree branch and Now, the velocity of man as seen from car is ur.
clings to it. How does the jump by the monkey affect the
speed of the car?

Let velocity of the man and the car be v and v0 respectively


after the jump. Given:
ur = v – v0
fi v = (ur + v0)
Momentum and Its Conservation  2.13

Momentum conservation in horizontal direction gives Momentum conservation gives:


u 3u
P after jump = P before jump mvA + m ​ __ ​   = mu  fi  vA = ___
​   ​ 
4 4
fi mv + mv0 = (M + m) u Kinetic energy of the system at this instant is

(  )
m (ur + v0) + Mv0 = (M + m) u
(M + m) u – mur mur
1 u 2 __
KE = ​ __ ​  m ​​ __
2 4 (  ) 1 3u 2 ___
​   ​   ​​ ​ + ​    ​ m ​​ ___
2
5
​   ​   ​​ ​ = ​    ​  mu2
4 16
fi v0 = ​ ______________
     u – ______
 ​ = ​    
 ​
(M + m) M+m Conservation of mechanical energy gives:

Example 17   Two identical balls, A and B, have mass Ui + ki = Uf + kf


m each. They are placed on a smooth horizontal surface 1 5
0 + ​ __ ​  mu2 = Uf + ___​    ​ mu2
connected by a spring of 2 16
force constant k. The spring 3
fi Uf = ___
​    ​ mu2
is relaxed. Ball A is kicked 16
toward B so as to impart it a
velocity u. Example 18   A wedge of mass M is placed on a smooth
horizontal table. It has a curved smooth track from A to B.
(i) Find velocity of two balls when the spring is
The track is horizontal at end
compressed the most.
B. A small block of mass m is
(ii) Find potential energy stored in the spring when released from the top of the track
u
velocity of B becomes ​ __ ​  towards right. at A. It slides down from A to B
4
and leaves the wedge while trav-
Solution
elling horizontally with velocity
Concepts v1. Find v1 if height of A above
(i) Momentum of the entire system is conserved, B is h.
though momentum of the individual balls change. Solution
(ii) The spring will have maximum compression when Concepts
both the balls have same velocity.
(i) As the block slides, the wedge will begin to move
Initially, the spring compresses and the spring force
towards left. [Think of the normal force between
accelerates B and retards A. As long as speed of
the two objects]
A is greater than that of B, the spring compresses.
When velocity of the two balls becomes equal, (ii) There is no external horizontal force on the system
further compression of the spring stops. (block + wedge). Hence, horizontal momentum of
the system will remain conserved.
(iii) Energy of the system is conserved, as there is no
non-conservative force performing work. (iii) Mechanical energy is conserved.

(i) Maximum compression takes place when both A and


B have same velocity (say v).

Momentum conservation gives


Let the velocity of the block (when it leaves the wedge)
u
mv + mv = mu  fi  v = __
​   ​  be v1 (Æ) and the velocity of the wedge be v2 (¨).
2
Initial momentum is zero. Momentum is conserved in
(ii) Let velocity of A be vA when horizontal direction.
u \  Final horizontal momentum is also zero.
vB = ​ __ ​ 
4
fi mv1 = Mv2 ...(1)

Conservation of energy gives


2.14  Mechanics II

1 1
​ __ ​  mv12 + __
​   ​  mv22 = mgh Speed of each block just before collision is given by
2 2 energy conservation as:

mv1 2
mv12 + M ​​ ____
​     
M (  )
​  ​​ ​ = 2mgh 1
​ __ ​  mu2 = mgR
2
2m Mgh 2Mgh ____ __________ ___
\ v12 = ________
​      ​ = ______
​    
 ​ fi    
u = ​÷2gR  ​ 2  × 10 ×
​= ÷   2 ​ = ÷    ms–1
​ 40 ​
mM + m 2 M +m

÷ 
______ Let v = speed of the blocks immediately after they stick
2Mgh
\ v1 = ​ ______
​    

  to each other. Momentum conservation gives
M+m
​ ​
P after collision = P before collision
___
Example 19   A fixed wedge has a hemispherical trough in u ÷     –1
​ 40 ​
5v = 3u –2u  fi  v = __
​   ​  = ​ ____
 ​ 
 ms
it. Radius of the hemisphere is R = 2m. Two small objects 5 5
of masses m1 = 3 kg and m2 = 2 kg are released at the Let the combined object rise to a height h after collision.
same moment at the rim of the hemispherical trough. The Energy conservation gives
two objects are initially
at diametrically opposite 1
Mgh = __
​   ​  Mv2 [Where,  M = 3 + 2 = 5 kg]
ends. The two objects 2
collide and stick. To
what height the com- v2 40
fi h = ___
​    ​ = __________
​   
    ​ = 0.08 m
bined mass will rise 2g 25 × 2 × 10
after collision. Neglect
friction. In short:
Solution (i) If external force on a system is zero, its momentum
remains conserved.
Concepts
(ii) If there is no external force on the system in a partic-
(i) The two blocks slide and collide at the bottom of ular direction, its momentum will remain conserved
the trough with same speed. along that direction.
(ii) Energy conservation can give us speed of either
Fx = 0 implies Px = a constant (Fy, Fz may not be
objects, just before they collide.
zero)
(iii) Momentum conservation can be used to find speed
of the combined object just after collision. (iii) If an event lasts for a very short duration (like col-
lision), the usual external forces (like weight etc),
(iv) After collision, energy conservation can once again
which are not impulsive, will not produce any sig-
be used to find the maximum height attained.
nificant change in momentum of the system.
Remember, kinetic energy just before collision is not
(iv) Conservation of momentum and conservation of
same as kinetic energy just after collision. Energy is lost
mechanical energy together help us in solving a wide
in deformation of the blocks.
variety of problems in mechanics.

Your Turn

Q.20  An empty gun of mass M is loaded with a bullet of Q.21  An empty gun of mass M is loaded with a bullet of
mass m. The gun is kept on a smooth surface and it fires. mass m. The gun is kept on a smooth surface and it fires.
Velocity of the bullet relative to ground is u. Velocity of the bullet relative to the gun is u. Find the recoil
(i) Find the recoil speed of the gun. speed of the gun.
(ii) The gun fires due to a small amount of the gun Q.22  An empty canon of mass M is kept on a smooth
powder getting ignited. What can you say about the horizontal surface. It is loaded with a shell of mass m. It
energy released due to ignition of the powder? fires with its barrel inclined at 60° to the horizontal. Speed
Momentum and Its Conservation  2.15

car in a direction opposite to the direction of motion of the


car. Find the change in speed of the car.
Q.27  In the situation given in the last problem, how should
the man run to bring the car to rest?
Q.28  A man is standing on a railroad car travelling on a
smooth track with velocity u. (see the figure in problem 26).
The man has lots of balls with him. He begins to throw
of the shell relative to the cannon is u. Find the recoil speed
the balls horizontally, in a direction perpendicular to the
of the gun.
motion of the car, relative to himself. He also throws some
Q.23  Two blocks A and B have masses 2m and m balls in vertical direction relative to himself. How will the
respectively. A spring of force constant k is tied to block velocity of car change if it is constrained to move along a
A and block B is moved towards A so as to compresss the straight line.
spring by x. In this position, the blocks are held fixed by
a string connected to both of
Q.29  A block of mass M is trav-
elling horizontally at a speed u. A
them. Surface is smooth and
liquid drop of mass m falls on it and
B is not tied to the spring.
sticks. Find the change in velocity
The string snaps suddenly.
of the block if the liquid drop was
Find the final velocities of
travelling vertically.
the two blocks.
Q.24  A toy ball is made of three pieces of masses in the Q.30  A long plank of mass M is kept on a smooth
ratio 1 : 2 : 1. The ball at rest explodes on its own. Reason horizontal surface. A small block of mass m is kept over
is not known. The two equal pieces move with velocities (2  it and imparted a velocity u.
+ 5 – 6 ) and (– 4  + 3  + 2  ) ms–1. Find the speed of Due to friction between the
the third mass. block and the plank, the slid-
ing stops after some time.
Q.25  Three particles of mass m Calculate:
each, are located at vertices (A, B (i) The final speed of the block.
and C) of an equilateral triangle.
All of them are travelling uni- (ii) Work done by the friction on the system.
formly with same speed v towards Q.31  A block of mass M has a long straight part and a
the centroid (G) of the triangle. The vertical part having a semi-circular cut in it. A small block of
three particles collide at G. After mass m is placed on the
collision, one of the particle was straight part and given
found to be at rest and the other was travelling along GB. a horizontal velocity
In which direction was the third particle travelling after u. Find the speed of
collision? bigger block when the
Q.26  A man of mass m is standing smaller block reaches
on a flat rail road car of mass M. The point P (Where radius
car is travelling on a smooth straight is horizontal) on the
track with speed u. The man begins curved part. Neglect friction.
to run with a velocity v relative to the

6.  COLLISION 6.1  Line of Impact


Collision is a short-term interaction between two bodies The line of common normal to the surfaces in contact during
during which they exert relatively large force on one another. the collision is called the line of impact. The two bodies
This sudden impulse causes a big change in momentum of apply impulsive force on one another along this line. Figures
the individual bodies. However, the total momentum of two given below show line of impacts in few cases. In case of
bodies remains conserved if the system is isolated from collision of two balls, the line of impact (LOI) is a line
external impulses. Momentum conservation plays a key role joining their centres.
in analysing collision problems. The two objects experience change in momentum along
the line of impact. Change in momentum of two bodies will
2.16  Mechanics II

When the colliding objects move along the line of


impact before and after collision, the collision is said to be
One-Dimensional or, Head on or, Direct Collision.
When velocities of the colliding objects, before and after
collision, are not all along the same line, collision is said to
be oblique collision or two-dimensional collision.
Consider two balls moving with velocities u1 and u2
along the line joining their centres (i.e., LOI). During col-
lision, they exert force on one another along this line only.
Obviously, they will continue to move along the same line
after impact. This is a direct collision (see fig. (a)).
Now consider a billiard ball hitting another ball at rest,
as shown in fig (b). Initial velocity u is not along the line
of impact. The two balls move in different directions after
collision. In this example, u, v1 and v2 are not along same
line. This is an example of oblique collision.

6.2  Elastic and Inelastic Collision


be equal and opposite (due to equal and opposite impulse External forces can deform a body. When deforming forces
applied on one another), but there is no change in momentum are removed, bodies have a tendency to regain their shape. A
of the system. body is said to be perfectly elastic if it can regain its original
shape completely. A body is said to be perfectly inelastic if
it has no tendency to go back to its original shape. However,
real-life objects lie between these two extremes and could
regain their shape to some extent. Such objects are called
inelastic. Steel is very close to being a perfectly elastic
material and a clay ball is very close to a perfectly inelastic
material.
A collision between perfectly elastic bodies is said to
be elastic. A collision between the two perfectly inelastic
bodies is said to be perfectly inelastic. All other collisions
are inelastic.
In an elastic collision between two bodies, the kinetic
energy remains conserved. In inelastic collision, there is loss
in kinetic energy and in perfectly inelastic collision, there
is maximum loss in kinetic energy. Let us make a simple
model and understand what happens during a collision.
Consider a ball A, travelling at speed uA being hit head on
by another ball B travelling at speed uB. Assume two imagi-
nary springs tied to the balls as shown. We are assuming
the springs to model the interaction between the balls. The
interaction between the balls can be understood in terms of
spring force. Deformation in the spring is actually represent-
ing deformation in the balls. Potential energy stored in the
spring is actually energy stored in deformed balls.
Initially, uB > uA (in figure shown). This means that the
two balls are getting closer. Springs (actually the balls) get
compressed (deformed). They push the balls as shown. Ball
A accelerates and B retards. At some point in time, velocities
of the two balls become equal. This is when deformation
stops. Now the springs (balls) try to regain their shape. Once
again they push each other. Ball A further speeds up and B
slows down. They get separated.
Momentum and Its Conservation  2.17

u12 = u1 – u2
After collision, the velocities of the two balls are v1 and
v2. Now, an observer in 2 sees ball 1 receding away. The
relative velocity is

v12 = v1 – v2
It is obvious that u12 and v12 are oppositely directed.

Now imagine that the springs (balls), completely regain The ratio of magnitude of v12 and u12 is known as
their natural shape as they separate. The energy that was coefficient of restitution (e).
stored as deformation energy for a moment, gets back as
kinetic energy. [Everything happens quite fast.] If so is the
case, collision is elastic.

|v12|
e = ____
​   
|u12|
 ​ =
|v1 – v2|
​ _______ 
|u1 – u2|
  ( v1 – v2
 ​ = – ​ ​ ______
u1 – u2 
  )
 ​  ​ ...(10)

If the springs (the balls), after experiencing maximum In perfectly inelastic collision, the two balls move with a
deformation (when both balls have same velocity), have no common velocity after collision. It means, v1 = v2.
tendency to go back to their original shape, the two balls will
Hence, e = 0 for perfectly inelastic collision.
keep moving with same velocity after deforming each other.
In this case, a large chunk of original kinetic energy is lost We known that kinetic energy for the system of balls can
as deformation potential energy. Such collision is perfectly be written as
inelastic. 1 1
K = __ 2
​   ​  mv rel + __ 2
​   ​  Mv cm
If the springs (the balls), after experiencing maximum 2 2
deformation try to regain their shape but fail to go back to Due to collision, vcm cannot change as there is no external
original state, then the two balls will move separately after force on the system. Kinetic energy of the system will remain
collision but some kinetic energy will be lost in deformation. same before and after collision if
Such collision is inelastic.
When two steel balls collide, they deform each other and ​| (urel) before collision |​ = ​| (vrel) after collision |​
quickly get back to their original shape. Collision is elastic.
when two aluminium balls collide they retain some defor- fi ​| u1 – u2 |​ = ​| v1 – v2 |​
mation. Collision is inelastic. Collision of two mud balls is
This implies that coefficient of restitution is e = 1 for an
almost perfectly inelastic. They deform each other but have
elastic collision.
no tendency to regain their shape.
All other collisions, which are called inelastic, result in
6.3  Coefficient of Restitution loss in kinetic energy. It means
Consider two balls having velocities u1 and u2 just before ​| (urel) before collision |​ > |​ (vrel) after collision |​
collision. To an observer attached to ball 2, the ball 1 appears \  ​| u1 – u2 |​ > ​| v1 – v2 |​
to approach at a velocity
2.18  Mechanics II

This implies that the coefficient of restitution is < 1 for Conservation of momentum gives:
inelastic collisions.
m1u1 + m2u2 = m1v1 + m2v2 ...(i)
6.4  Head on, Perfectly Inelastic Collision Use of equation (10) gives:
Consider two balls, A and B, made of perfectly inelastic
materials and hitting each other head on. u1 and u2 are
velocities before collision.
(  v1 – v2
– ​ _______
​ u – u  
1
 ​  
2
)
​ = 1 [   e = 1 for elastic collision]

fi v1 – v2 = u2 – u1 ...(ii)

Solving (i) and (ii) for v1 and v2 gives

( m1 – m2
v1 = ​ ​ _______  ) 2m2
 ​  ​ u + ​ _______
m1 + m2 1
​  ( 
  
 ​  ​ u
m1 + m2 2 ) ...(11)

Velocities of the two balls are same (v1 = v2 = v) after


v = ​( ​ _______ 
2 m +m )
m –m 2 1
 ​  ​ u
1 2
2 ( ​ 
2m1
+ ​ _______   )
 ​  ​ u
m1 + m2 1
collision. (It means e = 0).
Now, we need only conservation of momentum to In elastic collision, the KE of the system of balls is
find v. conserved.

(m1 + m2) v = m1 u1 + m2 u2 6.5.1  Special Cases


m1u1 + m2u2
fi v = ​ __________
    ​  It is worth remembering the following cases in head on
m1 + m2 elastic collision of two objects:
Loss in KE is large. We can find the loss as: (i) When both objects have same mass (i.e, m1 = m2),
Loss in KE = ki – kf final velocities are (using 11)

( 
v1 = u2  and  v2 = u1

1
= __
2
1
​    ​ m1u21 + __
2
1
​   ​  m2u22 – __
2
m1u1 + m2u2 2
​   ​  (m1 + m2) ​​ ​ __________
  
m1 + m2 )
 ​  
​​ ​
It means the velocities of the two objects get

[ 
exchanged.

2 2
1 (m1 + m2) (m1u1 + m2u2) – (m1u1 + m2u2)
= __
2
​   ​  ​ ​ ___________________________________
2
    
m1 + m2
 ​ 
    ​ ]
= __ ( 
1 m1m2
​   ​  ​ _______
​ 
2 m1 + m2 1
   ) 1
 ​  ​ (u – u2)2 = __
​   ​  m u2rel
2
1 2 1
This result is easy to guess using K = ​ __  ​ m v rel + ​ __ ​  Mv cm
2
2 2
(Try yourself).

6.5  Head on Perfectly Elastic Collision


Velocities of two balls before and after collision are as
shown in figure. (ii) When the body of mass m2 is originally at rest (i.e.,
u2 = 0), equation (11) gives:

( 
m1 – m2
v1 = ​ ​ _______ 
  )
 ​  ​ u   and
m1 + 2 1

​  ( 
2m1
v2 = ​ _______  
 ​  ​ u
m1 + m2 1 ) ...(12)

(iii) A light body hits a very heavy object at rest and col-
lision is head on – elastic. It means m2 >> m1. From
(12), we get
v1  – u1  and  v2  0
Momentum and Its Conservation  2.19

It means that light object bounces back with same Solution


speed. [Negative value of v1 indicates that direction Concepts

( 
of motion has flipped]. There is no effect on the very
heavy object, which was at rest (v2 = 0).
v1 – v2
(i) – ​ ​ _______
u1 – u2 )
 ​  
​= e
Imagine a ball hitting a heavy truck. The ball bounces   In words:  (speed of approach) × e = speed of
off with unchanged speed but the truck does not separation.
move.
(ii) For elastic collision, e = 1
(iv) A heavy body (a truck) hits a light object (a ball)
at rest and collision is head on-elastic. On putting Let velocity of the ball on rebounding be v¢ (¨). Velocity
m1 >> m2, equation (12) gives: of the wall will remain practically unchanged because it is
v1  u1  and  v2  2u1 heavy.
Taking left as positive direction and using
The heavy object (the truck) experiences no change
in its velocity and the lighter object (ball) flies away
with twice the velocity of the truck. ( 
v1 – v2
– ​ ​ _______
u1 – u2  )
 ​  
​ = e

fi ( v¢ – u
– ​ ______
​ – v – u 
  )
​  ​ = e

fi v¢ – u = ev + eu
fi v¢ = ev + (e + 1) u
For elastic collision e = 1
\  v¢ = v + 2u.

6.6  Head on, Inelastic Collision


If u1 and u2 refer to velocities before collision and v1 and
v2 refer to velocities after collision (see fig given in article
6.5), then
m1u1 + m2u2 = m1v1 + m2v2 ...(i)

And  (  v1 – v2
– ​ ​ ______
u –u 
1 2
)
 ​  
​ = e ...(ii)

Solving these two equation gives Example 21   Three blocks A, B and C have equal masses
and are placed on a horizontal smooth surface. Block A is

) ( 
imparted a velocity u towards right. If all collisions are
( 
m1 – em2
v1 = ​ ​ ________ 
m1 + m2 1
(1 + e) m2
 ​  ​ u + ​ ​ ________ ​ 


m1 + m2 2 )
​ u .
...(13)
elastic, find the final veloci-
ties of all three blocks.

= (​  ​ ________ ​  ​ u + ​( ​ ________ 


m +m )
Solution
m +m )
(1 + e) m 1 m – em2 1
v2    ​  ​ u .
1 2
1
1 2
2 Concepts
In head-on elastic collision of equal mass, velocities get
Example 20   A ball is flying horizontally at speed v. It exchanged.
hits a heavy wall travelling with velocity u in opposite direc-
tion. Find the speed at which the ball rebounds if coefficient When A hits B, A stops and B acquires velocity u. Now,
of restitution is e. What will be your answer if collision is B hits C. Block C acquires velocity u and B stops. Finally,
elastic? only C is moving and A and B are at rest.
2.20  Mechanics II

fi v1 + 2v2 = 2 ...(i)
Note:  Consider number of simple pendulums made of e = 0.5

( 
steel bobs and placed side by side as shown. All pendulums
are identical and touching one another. If one of the extreme
pendulum (A) is taken aside and released, we find that none
(  v1 – v2
– ​ ​ _______)
u1 – u2  ​  
v1 – v2
​ = 0.5  fi  – ​ ​ ________ 
4 – (– 1)
 ​   )
​ = 0.5

of the pendulums appear to move after A hits 1 and soon, fi  v1 – v2 = – 2.5 ...(ii)
the pendulum (B) at other extreme swings. Then B comes
back to hit the system. Only A moves. Therefore, only A Solving (i) and (ii) gives: v2 = 1.5 ms–1 and
and B appear to move and all other bobs remain stationary. v1 = – 1.0 ms–1 Negative sign of v1 indicates that
Actually, A transfers all its momentum to 1 and 1 transfers the 2 kg ball will travel to left with a velocity of 1.0
it to 2 and so on. (elas- ms–1.
tic collision of equal 1 1
masses). Finally, it is (ii) KEi = ​ __ ​  × 2 × 42 + __
​   ​  × 4 × 12 = 18 J
2 2
B which acquires all 1 1
__ __
KEf = ​   ​  × 2 × 1 + ​   ​  × 4 × 1.52 = 5.5 J
2
the speed and moves. 2 2
This puzzling device is
known as Newton’s Cradle. \  Loss in KE = 18 – 5.5 = 12.5 J
(iii) Maximum deformation (mini-
Example 22   Two balls are moving as shown. They collide
mum KE) occurs when v1 = v2 =
and coefficient of restitution is e = 0.5.
v (say)
(i) Find the velocity of the two balls after collision.
Conservation of momentum
(ii) Find loss in kinetic gives:
energy
(iii) Find the smallest (2 + 4) v = 2 × 4 – 4 × 1
kinetic energy of the 2
fi v = __
​   ​  ms–1
system when collision was going on. 3
Solution
1
2
2 2 4
\  KEminimum = ​   ​  (2 + 4) ​​ __
__
(  )
​   ​   ​​ ​ = __
3
​   ​  J.
3
Concepts
(i) Conservation of momentum and definition of coefficient Example 23   A ball dropped on a hard floor
of restitution give us two equations, which can be solved A ball of mass m is dropped from a height h on a hard floor.
for getting final velocities or else, you can directly use The ball hits the floor and rebounds. It again falls back and
equation (13). hits the floor. It again rebounds. The process continues till
When deformation is maximum, the two balls have least the ball comes to rest. Coefficient of restitution is e (< 1) and
kinetic energy. This happens when both have same velocity the ball keeps moving along a vertical straight line. Find
at a point during the ongoing collision process. (i) the time for which the ball keeps moving after it is
dropped.
(i) Let v1 and v2 be velocities of the two balls after col- (ii) the total distance travelled by the ball before it
lision. Momentum conservation gives: stops.
2v1 + 4v2 = 2u1 + 4u2
fi  2v1 + 4v2 = 2 × 4 + 4 × (– 1)
Momentum and Its Conservation  2.21

(iii) the total impulse imparted by the ball to the floor v2 = ev1 = e2u
before it stops. Time of flight before it returns back for next collision
Solution is

÷ 
___
Concepts 2v2 ____
___ 2e2u 2h
2 ___
t3 = ​  g   ​ = ​  g   
​ = 2e  ​ ​  g ​ ​  
(i) Coefficient of restitution is e (< 1). If speed before
a collision is u, it is simply the speed of approach And height attained after second collision is
as the floor is not moving. Speed after collision (v) v22
is nothing but speed of separation. h2 = ___
​    ​ = e4h
2g
v
\ ​ __
u ​ = e  fi  v = eu
(i) Total time is
T = t1 + t2 + t3 + ...
(ii) The balls will bounce repeatedly with its speed
getting reduced. If its speed is 0.001 ms–1 before
÷ 
___
2h
collision, it will become e (0.001) after collision. It = ​ ___
​  g ​ ​  [1 + 2e + 2e2 + 2e3 + ... • terms]
will again bounce and come back to hit the floor.

÷​  g ​ ​  [1 + 2 (e + e
___
When its speed becomes 0.000001 ms–1, then also 2h
it bounces and comes back! Theoretically, there = ​ ___  
2
+ e3 + ...)]
will be infinite many collisions.

÷ 
___
  But the time that the ball takes between two 2h
= ​ ___
​  g ​ ​  [1 + 2e/(1 – e)] [Sum of infinite G.P]
collisions goes on decreasing. When its speed is too
small, it will rise to a very small height and will

÷​  g ​ ​  ​[ ​ 1_____
___

– e]
take a very small time to return back. As we shall 2h 1 + e
= ​ ___   ​  ​
  
see, this makes the total time and total distance a
finite quantity.
(ii) Total distance travelled is
(iii) Initially, the ball is released from rest and ulti-
mately it will comes to rest. Impulse of all the s = h + 2h1 + 2h2 + ...
forces acting on it must sum up to zero.
= h + 2e2h [1 + e2 + e4 + ... • terms]

(  )
Velocity of ball just before it hits the
2e2 h 1 + e2
floor is = h + ​ _____  
2
 ​= h ​ ​ _____2 
 ​  ​
____ 1–e 1–e
u = ÷   ​ 
​ 2gh 
(iii) Total impulse on the ball = change in momentum of
÷ 
___
2h
Time of fall t1 = ​ ___
​  g ​ ​   the ball

Now the ball rebounds with speed Jfloor + Jgravity = 0

[  ]
____ 1 + e
v1 = eu after collision. \    
Jfloor = – Jgravity = – mg T = – m ​÷2gh ​ ​ ​ _____  
​  ​
1–e
Time of flight before it returns for next collision is Negative sign implies that impulse on the ball due to
2v1 the floor is upward.
t2 = ___
​  g   ​  The ball applies same impulse on the floor in
____

÷ 
___ downward direction.
2eu _______   ​ 
2e ​÷2gh  2h
fi t2 = ____
​  g   
​ = ​  g    ​ = 2e ​ ___
​  g ​ ​  
[  ]
____ 1 + e
  ​ ​ ​ _____  
\  J = m ​÷2gh  ​  ​ (Ø)
v12 e2u2 1–e
Height attained after first collision is h1 = ___
​   ​ = ____
​     

2g 2g  Example 24    Another way to define coefficient of
2
=eh restitution
Two balls, having masses m1
The balls comes back to the floor with speed v1 for the and m2 collide head on while
next collision. Speed after this collision is travelling with velocities u1 and
u2 respectively. During collision,
2.22  Mechanics II

both deform each other. By the time deformation becomes In short:


maximum, they apply an impulse JD (Impulse of deforma-
(i) Line along which two colliding objects apply force
tion) on one another. Now they begin to regain back their
on one another is called line of impact (LOI). For two
shape and push one another. During this phase of reformation,
balls, LOI is the line joining their centres. For two
they apply an impulse JR (Impulse of reformation) on one
blocks, LOI is a line normal to the common contact
another before separating. Prove that coefficient of restitu-
JR wall.
tion is e = ___
​   ​ When an object hits a wall, LOI is normal to the
JD
Solution wall.
Concepts (ii) If initial velocities of colliding objects (u1 and u2)
are along LOI, the collision is head on.
(i) Momentum of the system before collision =
(iii) Collision in which u1 and u2 are not along LOI, are
Momentum during collision = Momentum after
called oblique collisions.
collision.
(iv) Colliding objects apply huge imulse on one another
(ii) When deformation is maximum, both balls have
during collision and deform each other. Deformation
same velocity.
is maximum when both have same velocity along the
(iii) Impulse = D P LOI.
If the objects quickly regain their shape and KE of
the system remains conserved, the collision is called
elastic. In such a collision

Relative speed of separation after collision


​ _____________________________________
     
      ​= 1
Relative speed of approach before collision

This ratio is called coefficient of restitution (e).


(v) If colliding objects have no tendency to regain their
Let velocity of both balls at the instant of maximum shape after deforming each other, they travel with
deformation be v. equal velocity after head-on collision (v1 = v2). Such
Momentum conservation gives collision is called perfectly inelastic (e = 0). Loss in
KE is maximum.
m1u1 + m2u2 = m1v + m2v = m1v1 + m2v2 (vi) If colliding objects are able to regain their shape par-
tially, collision is called inelastic. In such a collision,
m1u1 + m2u2 m1v1 + m2v2 0 < e < 1. There is some loss in KE.
fi v = ​ __________
   = __________
 ​  ​      ​
  ...(i)
m1 + m2 m1 + m2 (vii) Usually, conservation of momentum and defining
equation of coefficient of restitution are sufficient to
During deformation phase, impulse on m2 is solve a problem on head-on collision.
JD = m2v – m2u2 = m2 (v – u2) (viii) If a ball hits a fixed wall (or floor) with velocity
u while travelling normally to the wall, velocity of
During reformation phase, impulse on m2 is ball after collision is v = eu with direction of motion
reversed.
JR = m2v2 – m2v = m2 (v2 – v) (ix) When two equal masses undergo head-on elastic
collision, there is exchange of velocity.
JR v2 – v
\ ​ ___ ​ = ______
​ v – u   ​ (x) In a perfectly inelastic head-on collision, loss in KE
JD 2
is
using (i)
m1v1 + m2v2 1
v2 – ​ __________
   ​  D k = ​ __ ​  m urel
2
  where
J m1 + m2 m1 (v2 – v1) 2
​ ___ ​ = ​  ______________
    ​= ​ __________
R
     
    ​
JD m 1u1 + m2u2
__________ m1 (u1 – u2)
​     ​ – u2 m1m2
m1 + m2 m = ​ _______
   ​  and  urel = u1 – u2.
m1 + m2
v1 – v2
= – ​ ​ ______
u1 – u2  ( 
 ​  ​ = e )
Momentum and Its Conservation  2.23

Your Turn

Q.32  Block A, travelling on a smooth horizontal surface the balls and that between a
with velocity u, hits another ball and the wall is e = 0.25.
identical block, B which is at Find distance from the wall
rest. Each block has mass m. where the balls collide for the
second time. All motion takes
(i) Find the kinetic energy of each block, after collision,
place on a straight line, per-
if the collision is elastic.
pendicular to the wall.
(ii) Find loss in kinetic energy of the system if the
collision is perfectly inelastic. Q.38  A ball is fixed from point A on ground. There is a
vertical wall at a distance x
Q.33  A cart of mass M rests on a smooth horizontal sur- from point A. The projectile
face. At one end of the cart there is a ball of mass M. The hits the wall while its veloc-
ball is imparted a sharp hit so as to ity is horizontal. The coeffi-
give it a velocity u towards the other cient of restitution is e = 0.5.
end of the cart. The ball travels along At what distance from the
the smooth cart surface and hits the wall will the ball land on
front wall (B) elastically. After how the ground?
much time (from the start) will the
ball hit the wall A? Q.39  Three balls of masses m1, m2 and m3 are lying on
a smooth horizontal surface, as shown. Ball of mass m1 is
Q.34  A ball of mass m moving with velocity v makes a given a velocity towards m2. It hits the ball of mass m2 and
head-on collision with an identical ball at rest. The system itself comes to rest. The middle balls hits the third ball of
loses 25% of its kinetic energy in collision. Find the coef- mass m3 and comes to rest. Coefficient of restitution for each
ficient of restitution. collision is e. Find m2 in terms of m1 and m3
Q.35  A ball is moving with velocity
2 ms–1 towards a heavy wall which is mov-
ing towards the ball with speed 1 ms–1.
They collide head on. Find velocity of ball
immediately after collision, if
Q.40  Two identical balls A and B lie on a smooth
(i) Collision is elastic horizontal surface. Ahead of B, there is a fixed smooth
(ii) Coefficient of restitution is e = 0.5 track of height 3.2 m. Ball A is given a velocity u = 10 ms–1
towards B. It collides head on with B, which then begins
Q.36  A ball of mass m is travelling at speed u along a
to climb the curved track. Find minimum coefficient of
smooth horizontal surface. It collides head on with another
restitution (e) between the balls so that B can reach the top
ball at rest. Find the mass (M) of the other ball so that
of the track.
the direction of motion of the first
balls gets reversed due to collision.
Coefficient of restitution is 0.8.
Q.37  A particle A of mass m moving on a smooth
horizontal surface collides head on with another station-
ary particle B of mass 2m, which is located at a distance d
from a wall. coefficient of restitution for collision between

6.7  Oblique Collision motion of two balls in a direction that is perpendicular to


LOI.
Consider two balls moving with velocities u1 and u2, as
shown. When they collide, they apply impulse along the line For ease of discussion, we will call the LOI as n-line
joining their centre (Line of Impact), assuming that the balls (‘n’ for normal) and the line perpendicular to LOI as t-line
are smooth. The effect of collision is to impart an impulse (‘t’ for tangent).
to each ball along the LOI. There will be no effect on the Before collision the velocity component of two balls
along n-line are u1n (= u1 cos a in shown figure) and u2n
(= u2 cos b in shown figure).
2.24  Mechanics II

u2, as shown. When A hit B, the floor applies a large impul-


sive normal reaction on B. This impulsive force has a com-
ponent along n-line. Momentum of the system of two balls
is not conserved along the n line. [However, it is conserved
along horizontal direction].
If v1 and v2 are velocities after collision and coefficient
of restitution is e then

(  v1n – v2n
e = – ​ ​ ________
u1n – u2n  )
 ​  
​ ...(14)

Note:  In present example, v1n = v1 cos q, v2n = v2 cos q,


u1n = u1 cos q, u2n = u2 cos q

\ (  v1 – v2
e = – ​ ​ _______
u1 – u2 )
 ​  

Example 25   Oblique elastic collision between balls of


equal mass
A billiard ball B is at rest. Another identical and smooth
ball A hit it while travelling with
a velocity u, making an angle
q with the line joining the cen-
tre of the two balls. Collision is
elastic. Find velocity of the two
balls after the impact.
Solution
Concepts
Velocity components along t-line are u1t (= u1 sin a) and (i) Line joining the centre (C1C2) is the n-line.
u2t (= u2 sin b) (ii) Velocity of ball will remain unchanged along t-line
The effect of collision is to change velocities along n-line (a line ^ to the n-line).
only. For the time being, forget about velocity components
(iii) In a head-on elastic collision of two balls, there
along t-line. Take this as a problem of one-dimensional
will be exchange of velocities.
collision with balls moving with velocities u1n and u2n. Use
what you have studied in last section about head-on colli-
sions and find velocities (v1n and v2n) after collision. Now,
you need to recall that u1t and u2t are unchanged. Add u1t and
v1n vectorially to get final velocity of the first ball. Similarly,
add u2t and v2n to get final velocity of the second ball.
The above procedure will work if the system is free of
external impulses. At times, you may face problems when
there are external impulses on the system. If there is an
impulse along n-line then momentum cannot be conserved
along this line. However, for writing coefficient of restitution,
we consider veloc-
ity components along
n-line in all cases.
Consider two balls
A and B moving hori-
zontally on a floor
with velocities u1 and Velocity component of A along the n-line and t-line are un
= u cos q, ut = u sin q. Forget about ut for a moment. Assume
Momentum and Its Conservation  2.25

that B is at rest and A hits it head on with a velocity un.


what will happen? Velocities will get exchanged. A comes to
rest and B moves along n-line with velocity un = u cos q.
Now recall that A has a velocity ut = u sin q, which
remained unaffected during collision.
Thus, A will have a final velocity u sin q along t–line and
B will move with velocity u cos q along n-line. Both balls Since there is no change in momentum of the ball along
will move in directions perpendicular to each other. the t-line
Example 26   A ball colliding obliquely with a smooth \ vt = ut = u sin a
floor vn
A ball hits a smooth floor at an angle of incidence a while And ​ __
un ​ = e  fi  vn = e un = eu cos a
travelling with velocity u (see figure). ______ ______________
Coefficient of restitution is e. Find the \ v = ​÷v  2t + vn2    2a + e2 cos
 ​ = u ​÷sin   
2
 a ​ ...(i)
velocity of the ball after the impact.
Discuss the extreme cases of elastic Angle b can be calculated as
and perfectly inelastic collision.
vt u sin a 1
tan b = __
​ v  ​  = ​ _______  = __
 ​  ​   ​ tan a ...(ii)
Solution n e u cos a e
Concepts When collision is elastic, e = 1
(i) The LOI (the n-line) is normal to the \ v = u
floor. It does not make any difference
and b = a [from (i) and (ii)]
whether the ball came from right or
left, whether it was incident at angle It means there is no change in speed and angle of
0° or 30°; the line along which the reflection = angle of incidence.
floor and the ball apply force on one another will When collision is perfectly inelastic (e = 0):
be normal to the floor passing through the center vn = 0 and vt = u sin a.
of the ball.
(ii) There is no horizontal impulse on the ball. Its The ball will move along the floor (will not bounce back)
horizontal momentum will not change. with velocity u sin a.

Example 27   A ball hitting an inclined plane


 relative velocity of separation along n-line
(iii) e = ​ ____________________________________
          ​ A ball is dropped from a point. after

 |
relative velocity of approach along n-line falling through a distance h = 5 m, it hits
an inclined plane. The inclination angle

|
velocity of ball in  vertically upward of the plane is q = 60°. Coefficient of
​          ​ restitution is e = 0.5. Find the veloc-
direction after impact
= ​ ​ ​________________________________
    
     ​  ​ ​
ity of the ball immediately after the
velocity of ball in vertically downward
 ​          ​ impact. What angle does the velocity of ball (after impact)
​ direction before impact ​ makes with incline surface?

Note that the floor does not move. Solution


Concepts

The n-line and t-line for the given collision are as The concepts required are essentially same as in the last
shown. example.
Component of velocity before collision are: Velocity of the ball just before impact is
____
ut = u sin a, un = u cos a u = ÷   ​ = 10 ms–1 (Ø)
​ 2gh 

Velocity component after collision are: n-line and t-line are shown in figure.
__
÷   
​ 3 ​ __
vt and vn. ut = u sin q = 10 × ___     ms–1
​   ​ = 5​÷3 ​
2
2.26  Mechanics II

1
un = u cos q = 10 × __
​   ​  = 5 ms–1
2
v is velocity after collision as there is no impulse on the
ball along the t-line
__
    ms–1.
vt = ut = 5​÷3 ​
5
And vn = e un = __ ​   ​  ms–1
2
vn ____ 1__
\ tan a = ​ __
vt ​  = ​ 2​÷3 ​
   ​ 
  

fi (  )
1
a = tan–1 ​ ____
​  __
   ​  ​
  
2​÷3 ​

Example 28   Impact parameter


A ball of mass m and radius R travelling with velocity u,
hits another ball of mass 2m and radius 2R. The impact
parameter of the collision is 1.8 R. Coefficient of restitution fi q = 37°
is 0.5. Find velocity of two balls after the collision. Velocity component of smaller ball along n-line and t-line
Solution are
Concepts un = u cos q = 0.8 u
(i) Draw
​_›
a diameter of the second ball ​_that is parallel ut = u sin q = 0.6 u

to ​u   ​. The distance between line of ​u   ​ (through the
centre of the first ball) and the diameter drawn is ut does not change due to collision.
known as impact parameter (b). Consider a head-on collision problem along n-line.
(ii) Knowing b, we can find the angle that u makes u1 = u cos q , u2 = 0
with the LOI (i.e., n-line).
(iii) We can treat the collision problem as a head-on Let v1 and v2 be velocities after collision.
collision problem, considering velocities along Momentum conservation gives:
n-line only. mv1 + 2mv2 = mu1
(iv) Velocity along t-line does not change.
fi v1 + 2v2 = u1 ...(i)
At the time of impact, the velocity u makes an angle q
with the n-line where And  (  v1 – v2
– ​ ​ _______
u – u 
1
 ​  
2
)
​ = e

( 
b 1.8R
sin q = ___
​     ​ = ____
)
​     
​ = 0.6 v1 – v2 1
3R 3R fi  – ​ ​ ______  ​  ​ = __
​   ​ 
u1 – 0 2
Momentum and Its Conservation  2.27

u1 Let the other ball move with velocity v making an angle


fi v2 – v1 = __
​   ​   ...(ii)
2 q with x-direction, as shown in the figure.
Solving (i) and (ii) gives Using conservation of momentum along x-direction:
u1 Pix = Pfx
v2 = ​ __ ​   and  v1 = 0
2
mu = 2mv cos q  fi  u = 2v cos q ...(i)
0.8 u
\ v1 = 0  and  v2 = ____ ​   ​  = 0.4 u
2 Using conservation of momentum along y-direction
[ u1 = u cos q = 0.8 u] Piy = Pfy
First ball has its u u
0 = m __
​   ​  – 2m v sin q  fi ​ __ ​  = 2v sin q ...(ii)
velocity along t-line 2 2
unchanged. Thus final Squaring and adding (i) and (ii) gives
velocities of the two
balls are as shown. The
two balls will be mov-
(  )
u 2
u2 + ​​ __
​   ​   ​​ ​ = 4v 2 (cos2 q + sin2 q)
2
__
ing perpendicular to one   
​÷5 ​
another. fi v = ​ ___ ​  u
4

Example 29   Line of impact not known And dividing (ii) with (i) gives
A ball of mass m travelling with velocity u hits another ball
of mass 2m, which was at rest. After impact, the first ball
u

1
tan q = __
2
1
​   ​   fi  q = tan–1 ​ __ (  )
​    ​  ​
2
was found to be travelling at velocity ​ __ ​  in a direction perpen-
2
Example 30   External impulsive force present on the
dicular to the original line of motion. Find the velocity and
system
direction of motion of the second ball after the impact.
A ball of mass 2m is suspended
Solution using an inextensible string. Another
Concepts ball of mass m hits it while travelling
with velocity u in a direction making
(i) In the last problem, we knew the angle between
an angle q with the horizontal. The
initial velocity (u) and the LOI. This allowed us
collision in elastic. Find the velocity
to resolve the velocities along n-line and t-line.
of the smaller ball after the impact.
In this question, geometry of collision is not known.
We do not know the angle between initial velocity Solution
of the ball and LOI. Concepts
(ii) If we have no knowledge regarding the motion (i) During collision, the string applies a large
of either balls after collision, the problem cannot impulsive tension on the ball. This tension force
be solved. However, the present question tells us has a component along n-line. Therefore, we can-
the velocity of first ball after collision. It can be not use momentum conservation along n-line.
solved.
(ii) However, we can use equation (14) along n-line.
(iii) Apply conservation of momentum along two
(iii) Momentum is conserved along horizontal direction,
perpendicular directions.
as there is no external force on the system of balls
in horizontal direction.

Let velocity of the bigger ball be v2 along horizontal direc-


tion. This ball cannot have vertical motion. Let velocity of
the smaller ball be v1 along its original direction of motion.
This ball has experienced an impact along this line (the
n-line) only. Thus, it will move along this line only. [There
is no t-component of velocity]
Momentum of conservation along x-direction gives
Let the initial direction of motion of the first ball be x-di- mu cos q = mv1 cos q + 2mv2
rection. After collision, this ball moves along y-direction.
2.28  Mechanics II

Negative value of v1 indicates that its direction is opposite


to that indicated in figure.
In short:
(i) The line that is normal to contact surface of the two
colliding bodies may be termed as n-line and a line
perpendicular to that is termed as t-line.
(ii) When angle between initial velocities (u1, u2) and
the n-line is known, we can resolve u1 and u2 along
n and t-lines.
  Velocity along t-line does not change if the system
is free of any external impulsive force along this
line.
  In this case, we can consider the problem as a
problem of head-on collision along n-line. After
finding velocities along n-line we add the unchanged
velocity along t-line to get the resultant final velocity
of the body.

(  )
v1n – v2n
(iii) In all cases, e = – ​ ​ ________
u1n – u2n  ​  
​ ...(14)

v1n, v2n are velocity components along n-line after


fi u cos q = v1 cos q + 2v2 ...(i) collision and u1n, u2n are velocity components along
n-line before collision.
And ( v1n – v2n
– ​ ​ ________
u1n – u2n   )
​  ​ = e (iv) When a ball hits a smooth fixed wall (vertical,
horizontal or inclined), velocity parallel to the wall
remains unchanged. The velocity normal to the wall

fi ( 
v1 – v2 cos q
– ​ __________
​     ​ 
u–0

  )
​ = 1
reverses in direction and is equal to vn = e ◊ un.
(v) When angle between velocities and LOI is not known
we cannot use equation (14). However, conservation
fi  v2 cos q – v1 = u ...(ii) of momentum can be used.
Solving (i) and (ii) gives (vi) When external impulsive force acts on the system of
colliding bodies, apply conservation of momentum
(2 – cos 2q) u 2u cos q
v1 = – ____________
​     ​ and  v2 = ________
   ​    ​
  in a direction perpendicular to the impulse and use
(2 + cos2q) 2 + cos2q equation (14).

Your Turn
​_
Q.41  A ball travelling with velocity ​u › ​  = (2  + 3  ) ms–1 impact, the ball was found to be moving vertically up. If
hits a smooth wall. The plane of the wall is xy plane. Find collision is elastic, find the inclination of the incline surface
velocity of the ball immediately after impact if collision is with horizontal.
(i) Elastic (ii) Perfectly inelastic
Q.44  A wedge of mass M = 4 kg is placed on a smooth
(iii) inelastic with coefficient of restitu-
horizontal surface. Its inclined face is inclined at q = 37°
tion e = 0.5.
to the horizontal. A ball of mass
Q.42  Two balls collide as shown in m = 1 kg hits the smooth inclined
figure. Find their velocities after colli- surface normally, with a velocity
sion if the collision is elastic. of u = 10 ms–1. The wedge begins
to move horizontally at a veloc-
Q.43  A ball is flying horizontally ity 2 ms–1. Find the coefficient of
when it hits an inclined surface. After
restitution for the collision.
Momentum and Its Conservation  2.29

Q.45  A smooth ball hits a vertical wall with its direction on the floor at C. Floor is smooth and coefficient of restitu-
L1
of motion making 30° with the normal to the wall. The ball tion is e. Find the ratio __ ​   ​ .
rebounds in a direction that is perpendicular to its original L2
direction of motion. Find the coefficient of restitution. Q.47  A ball of mass m is moving along x-axis with
speed u. It hits another ball of mass 2m at rest. As a result
Q.46  A projectile is launched of collision, the ball of mass m loses half its speed but
from point A and it lands back
maintains its line of motion. The other ball breaks into two
on the horizontal floor at B. It
equal pieces with one piece travelling in y-direction with
bounces at B and again lands back u
velocity __
​   ​ . Find the velocity of the other piece.
2

7.  SYSTEM WITH VARYING MASS \  Thrust force acting on the rocket is
dm
Application of Newton’s second law of motion is little tricky Fth = – u ​ ___ ​ 
when mass of a system is changing. A rocket is a very good dt
example of varying mass system. Most of its mass is fuel In the above expression, Fth is positive, i.e., towards right,
which is eventually ejected as exhaust gases. Consider a since dm/dt is negative.
rocket of mass m travelling at a velocity v in space. In an We can also have a variable mass system in which mass
infinitesimally small time dt its mass changes by dm. Note is getting added. The general formula for thrust force acting
that dm is negative as its mass is decreasing due to burning on a system having variable mass is:
of fuel and throwing it out. [In other words, during time dt, ​_› ​_› dm
a negative mass dm gets added to the rocket and a positive ​F  ​ th = ​u  ​  ___
​   ​   ...(15)
mass (– dm) is thrown out]. dt
​_›
Here, ​u  ​  = velocity of incoming/outgoing mass relative
dm
to the system and ​ ___ ​  = rate of change of mass of the
dt
system.
​_› dm
We must be very careful about the sign of ​u  ​  and ___ ​   ​ .
dt
If the
​_› variable mass system is experiencing an external
force (​F   ​ext) then equation for its motion becomes:
​_›
d​v   ​ ​_› ​_›
m ​ ___ ​   = ​F  ​ th + ​F  ​ ext
Speed of rocket change to v + dv. dt
_
​›
Let the exhaust gases come out of the rocket at a velocity d​v   ​ _​ › dm ​_›
m ​ ___ ​   = ​u  ​  ___
​   ​ + ​F  ​ ext ...(16)
u relative to the rocket. What we are saying is that the dt dt
engine shoots out the gases at a fixed speed relative to
itself whatever be its own speed. Actual velocity of exhaust Example 31   A rocket in space is having mass M0 and is
ve = (v + dv – u), which may be positive or negative depend- travelling at a velocity v0. It is ejecting gases at a velocity
ing on values of v and u. [Note that we have taken u to be u relative to itself. Find its velocity when its mass reduces
directed towards left in our diagram]. to m.
Change in momentum of ejected mass (– dm) (in time dt)
is Solution
dp = (– dm) ve – (– dm) v Concepts
= – dm [ve – v] = – dm [dv – u] = u dm (i) Rocket is in space. Hence, external force on it is
zero.
Where we have neglected dm ◊ dv dM
(ii) Thrust force is Fth = u ​ ___ ​ 
.
\  Force on ejected mass is dt
Here, u (velocity of exhaust, relative to the rocket)
dp dm
F = ___
​   ​ = u ​ ___ ​  is negative; i.e., opposite to the velocity of the
dt dt rocket.
Same force is applied by the outgoing mass on the rocket
dv dM
in opposite direction. \ M ​ ___ ​   = – u ​ ___  
dt dt

2.30  Mechanics II

dM Solution
fi dv = – u ​ ___ ​ 
M
Concepts
v m
dM (i) This is a variable mass system as the mass of tank
fi ​Ú  ​ ​dv = – u ​Ú  ​ ___
​  ​ 
v0 M0 M
decreases continuously.
(ii) Initial velocity of tank = 0
(  ) (  )
m M0
fi v – v0 = – u ln ​ ___
​    ​  ​  fi  v = v0 + u ln ​ ​ ___
m ​  ​     \ Relative velocity of outgoing mass = u
M0
(iii) Volume of liquid flowing out per unit time through
Example 32   A rocket of mass M0 is fired vertically from the hole = Au.
the ground. It ejects gases at a velocity u relative to itself
Mass flowing out per unit time = r Au
and throws out mass at a rate b kgs–1. Find its velocity at
time t. Assume that the acceleration due to gravity remains dM
constant at g. Fth = u ​ ___ ​ = u (Æ) ◊ (– r Au)
dt
Solution [   Mass is decreasing]
Concepts = r Au2 (¨)
2
(i) Apart from the thrust force, the rocket experiences dv dv r Au
an external force Mg. \ M ​ ___ ​   = r Au2 (¨)  fi ​ ___ ​ = _____
​      
​ (¨)
dt M
(ii) At time t, its instantaneous mass is M = M0 – bt
Example 34   Chain falling through a hole
dv dM A chain has mass per unit length equal to l. It is piled up
M ​ ___ ​   = – u ​ ___ ​ – Mg
dt dt on a table. There is a small hole in the table through which
the chain begins to fall. Find its acceleration when its free
dM
Here ​ ___ ​   = – b, thus the thrust force is positive. end has fallen though a distance y.
dt
dv u dM
​ ___ ​   = – ​ __   ​ ___
​   ​ – g
dt M dt

dM
fi dv = – u ​ ___ ​ – g dt
M
v m t
dM
fi ​Ú ​  ​dv = – u ​Ú  ​ ___
​  ​ – g ​Ú ​  ​ dt
0 M0 M 0

Where m = mass at time t = M0 – bt


m
fi v = – u ln ​ ___  ​ – gt
M0
Solution

(  )
M0 Concepts
fi v = u ln ​ _______
​     ​  ​ – gt
M0 – bt (i) We can always solve _​this problem using Newton’s

​_› d ​p   ​
second law as ​F  ​  = ​ ___ ​  ; but here, we will do it by
dt
Example 33   A container filled with a liquid of density treating the hanging part of the chain as a system
r has a mass M (including liquid). It is at rest on a smooth to which mass is continuously getting added (From
horizontal surface. A small the heap).
hole of area A is punched in
its side wall through which (ii) If speed of chain is v at an instant, then v dt length
the liquid gushes out at speed of chain is added form the heap to the falling chain
u. Find the initial acceleration (which is our system) in time dt.
of the container. \  Mass added in time dt is dm = l v dt
dm
\ ​ ___ ​ =
  l v
dt
Momentum and Its Conservation  2.31

dm m = l y
​ ___ ​ is positive since mass is getting added.
dt Thrust force due to incoming mass is
(iii) Velocity of incoming mass (a small segment on dm
table) = 0. Velocity of our system(hanging chain ) Fth = u ​ ___ ​ = v (l v) (≠) = l v2 (≠)
dt
= v (Ø)
dv
\  Relative velocity, u = 0 – v (Ø) = v (≠) \ m ​ ___ ​ =
  mg – l v2
dt
dv l l v2
When y length is hanging, mass of our system is fi ​ ___ ​ =
  g – __
​ m  ​ v2 = g – ___
​     ​ v2 = g – __
​ y  
dt l y


Your Turn

Q.48  A rocket has a mass of 500 kg (including its fuel).


It can have a maximum exhaust speed of 2000 ms–1 (relative
to itself). Find the minimum rate of fuel consumption for it
to just lift off the launch pad.
Q.49  In the last problem, find the rate of fuel burning if Q.51  A balloon of negligible mass has a gas of mass m
we want the initial acceleration of the rocket to be 5 ms–2. filled in it. It suddenly gets released and the gas leaks from
it at a constant velocity u = 2 ms–1 relative to it. Find the
Q.50  A cart is moving on a smooth horizontal road at velocity of the balloon when its mass is reduced to half.
speed u. It is carrying sand. Sand begins to leak at a constant
Neglect gravity and buoyancy.
rate b kgs–1 through a small hole at the bottom of the cart.
Find the acceleration of the cart.

Miscellaneous Examples

Example 35   A cannon of mass m starts sliding freely Speed at B is


____ ________
down a smooth inclined plane at an angle a to the horizontal.   ​
  ​ = ​÷2g l sin a 
v = ​÷2a l   
After the cannon covered a distance l, a shot was fired, the
shell leaving the canon in horizontal direction with momen-
tum P. As a consequence, the canon stops. Assume the mass
of the shell to be negligible compared to that of the canon
and find the duration of the shot.
Solution
Concepts At this instant, it fires a shell horizontally and comes to
rest. Change in momentum of the cannon + shell system
(i) Duration of the shot implies the time duration for
along x during the duration of shoot (D t) is
which the cannon exerted force on the shell. This
duration can be deliberately prolonged (by making D P = (P cos a + 0) – (mv cos a)
the barrel long) so as to impart huge impulse to ________
the shell.   ​
= P cos a – m cos a ​÷2g l sin a 

(ii) Change in momentum of the cannon + shell External force on the system along x-direction is
system along the incline = impulse of external Fx = mg sin a
force along the incline. We will use this relation
for the duration of the shot. \ Fx D t = D P
________
The cannon started from A and moved a distance AB = l   ​
D P P cos a – m cos a ​÷2g l sin a 

with acceleration a = g sin a. fi D t = ___
​   ​ = ​ _______________________
         ​
Fx mg sin a
2.32  Mechanics II

Example 36   Three particles A, B and C of masses m, 2m But v2x = v1. When string AB is taut
and 3m respectively lie on a smooth horizontal table at the \ v1 + v3x = u ...(ii)
vertices of an equilateral triangle. A and
B as well as B and C are connected by When BC is taut
light inextensible strings as shown. C is Velocity of B along BC = Velocity of C along BC
imparted a velocity u parallel to AB. Find
the final speed acquired by A. fi  v2x cos 60° + v2y cos 30°

Solution = – v3y cos 30° + v3x cos 60°


__ __
Concepts fi v2x + ​÷3 ​
    v2y = – ​÷3 ​
    v3y + v3x
(i) Momentum of the system (A + B + C) will remain __ __ 2
conserved. fi v1 + ​÷3 ​    ◊ ​ __ ​  v2y + v3x
    v2y = – ​÷3 ​ 
3
(ii) When C moves to position shown in fig. below, the 5
fi v1 – v3x = – ​ _____  ​  v2y ...(iii)
string BC suddenly acquires tension. It applies an ÷   
​ 3 ​
impulse on C and also on B.
For C: J2 cos 30° = 3m v3y
(iii) As B gets a tendency to move, the string AB also __
exerts an impulse on B as well as A. ÷   
​ 3 ​ 2
fi J2 ___   3m ◊ ​ __ ​  v2y
​   ​ =
  A receives impulse along AB. It will have its 2 3
final velocity along AB. 4
fi J2 = ___
​  __  ​  m v2y ...(a)
÷   
​ 3 ​
Let J1 be impulse developed in string AB and J2 be
impulse developed in BC after C moves to a position shown And 3mu – J2 sin 30° = 3m v3x
in fig. below. fi J2 = 6m (u – v3x) ...(b)
From (a) and (b)
4__
​ ___  ​ v2y = 6 (u – v3x) ...(iv)
÷   
​ 3 ​
2u
Solving (ii), (iii) and (iv) gives v1 = ___
​   ​ 
19
Let final velocity of A be v1 along AB. Let final velocity
Example 37   A cylindrical solid of mass m = 10 –2 kg
of B have two components v2x and v2y as shown. Velocity
and cross-sectional area A = 10 –4 m2 is moving parallel to
of C has two components v3x and v3y, as shown.
its axis (the x-axis) with a uniform speed of 103 ms –1 in
positive direction. At t = 0, its front face passes the plane
x = 0. The region to the right of this plane is filled with
stationary dust particles of uniform density, r = 10 –3 kg m–3.
When a dust particle hits the face of the cylinder, it sticks
to it. Assuming that the dimensions of the cylinder remains
practically unchanged, and that the dust sticks only to the
front face of the cylinder, find the x-coordinate of the front
face of the cylinder at t = 150 s.
Solution
Py = 0
Concepts
fi 2m v2y = 3m v3y
(i) When the cylinder moves through a distance x,
fi 2v2y = 3v3y ...(i) the dust particles present in the volume Ax will
Px = 3mu stick to the cylinder. Mass of the cylinder goes on
increasing.
fi  mv1 + 2m v2x + 3m v3x = 3mu
(ii) Momentum being conserved, the speed of cylin-
der decreases as more and more dust sticks to
fi  v1 + 2v2x + 3v3x = 3u
it. Using momentum conservation we can find its
speed when it has moved through a distance x.
Momentum and Its Conservation  2.33

upward motion. It keeps moving up as the spring


stretches. By the time it stops it must have stretched
the spring to an extent such that the spring force
exceeds the weight of B: then only B will bounce
off the floor.
(iv) mechanical energy is conserved for the system
after the collision has taken place.

Volume of dust swept when the cylinder moves a distance


x is A ◊ x
Mass of dust in this entire volume sticks to the cylinder.
This mass is D m = r A ◊ x
Momentum conservation gives
(m + D m) v = mv0
fi  (m + r A x) v = mv0
dx
fi  (m + r A x) ​ ___ ​ =
  mv0
dt
x t
fi  ​Ú ​  ​(m + r A x) dx = mv0 ​Ú ​  ​dt Speed of A just after B hits the floor is u = ÷   ​ 
​ 2gh 
____
0 0
1 Block A compresses the spring and then moves up. Let it
fi  mx + __
​   ​  r A x2 = mv0 t
2 stretch the spring by x when it comes to rest. B will leave
1 the floor if
fi  10 –2 x + ​ __ ​  × 10– 3 × 10– 4 x2 = 10– 2 × 103 × 150
2 kx ≥ m2g ..(i)
fi  x2 + (2 × 105) x – 3 × 1010 = 0
Conservation of energy gives:
_________________
– 2 ​ 4  × 1010 + 12  
× 10 ± ÷
5
× 1010 ​
_____________________________ 1 1
Solving x = ​      
 ​
   ​ __ ​  m1u2 = ​ __  ​ kx2 + m1gx
2 2 2
= 105 m ____
Putting   ​   and
u = ​÷2gH 
Example 38   Two blocks A and B are connected to an
m2g
ideal spring of force constant k. Masses of the blocks are m1 x = ____
​     
​ from (i) for limiting case, we get
and m2 respectively. Initially, the system is k
kept vertical with spring relaxed and block
B at a height H above the floor (see figure).
1
​ __ ​  m1 (2gH) = __
2
1 m2g 2
2 (  )
​   ​  k ​​ ____
k (  )
m2g
​  ​​ ​ + m1g ​ ____
​      ​     
k
​  ​
The system is released from rest. Collision
between B and the floor is perfectly inelas-
tic. Find minimum value of H for which B
will again bounce off the floor.

m2g m
H = ​ ____
k
   (  2 + 2m1
 ​ ​ ​ ________
2m1
 ​  

​)
Solution This is the minimum value of H needed.
Concepts Example 39   A uniform chain has mass per unit length
(i) The system falls freely and speed of____
both A and B equal to l and it is kept passing over
just before B hits the floor is u = ÷    .
​ 2gh ​ a smooth pulley (of negligible dimen-
sion), as shown in figure. height of
(ii) B does not rebound since collision is perfectly ____ pulley above the table is H and length
inelastic. But A still has a velocity u = ​÷ 2gh  ​
of the chain on other side of the pul-
immediately after B meets collision. A does not
ley is y. Height of heap on the table
experience any impulse when B hits the floor.
is negligible. The end of the chain is
(iii) Downward motion of A compresses the spring being pulled with a force F so that
and A will begin to retard after some time. After the chain moves at a constant speed
A stops, it is pushed up by the spring and starts v. Find F as a function of y.
2.34  Mechanics II

Solution Velocity of bead relative to the ground is


​_› ​_› ​_›
Concepts ​v  ​   = ​v  ​ 0 + ​u  ​ 
The heap applies force on the moving chain as small = (u – v0 cos q)  + (v0 sin q) 
pieces are ‘jerked’ into motion. \ vx = u – v0 cos q ; vy  = v0 sin q
Momentum is conserved in horizontal direction.
An elemental length dy of the chain (in the heap) is set
into motion in time dt. \ mvx + Mu = 0
Momentum imparted to the element is dp = (l dy) ◊ v fi  m (u – v0 cos q) = – Mu ...(i)
Force applied by the vertical chain on the element to put it Law of conservation of mechanical energy:
in motion is Gain in KE of ring + Gain in KE of bead = loss in PE
dp dy of bead
Fth = ​ ___ ​ = l v ​ ___ ​ = l v2
dt dt 1 1
fi ​ __ ​  Mu2 + __
​    ​ m (vx2 + vy2) = mgR (1 + cos q)
The heap applies equal force on the chain in downward 2 2
direction. For moving part of the chain to have constant fi Mu2 + m [(u – v0 cos q)2 + (v0 sin q)2]
speed:
= 2mgR (1 + cos q)
F + weight of length y = Fth + weight of length H
fi Mu2 + m [u2 + v20 – 2u v0 cos q]
2
fi F + l yg = l v + l Hg = 2mgR (1 + cos q)
fi  F = l v2 + l g (H – y) (m + m) u
From (i) v0 = ​  ________  
m cos q

[  ]
Example 40   A ring of mass M and radius R is standing
(m + m)2
on a smooth horizontal table.  ​ \ u2 ​ M + m + ​  ________  ​ 
– 2 (m + m)  ​
A bead of mass m can slide m cos2 q
smoothly along the ring. The = 2mgR (1 + cos q)
system is released from rest Simplifying further gives:

÷ 
with the bead at the top-most ___________________
2gR (1 + cos q)
point. Find the velocity (u) u = m cos q ​ ​  ___________________
   
       ​ ​
of the centre of the ring at (m + m) (M + m sin2 q)
the instant the bead reaches a
position P at an angle q with Example 41   2n number of identical elastic blocks are
vertical. kept on a smooth horizontal table at uniform separation (n is
an integer). All odd-numbered blocks are imparted a velocity
Solution u towards right and all even-numbered blocks are imparted a
Concepts velocity u towards left. All the blocks start simultaneously.
(i) Direction of velocity of the bead relative to the ring How many collision will occur in all?
is along the tangent on the ring.
(ii) Actual velocity of the bead is vector sum of relative
velocity and velocity of the ring (u).
(iii) Horizontal momentum remains zero.
Solution
(iv) Mechanical energy is conserved.
Concepts
In head-on elastic collision of equal masses, velocities get
exchanged.

In first round, there will be n collisions, as shown


below

Let v0 be velocity of bead relative to the ring.


​_›
​v  ​0  = – v0 cos q + v0 sin q
Momentum and Its Conservation  2.35

After first collision, block 1 and 2n will leave the system Components of initial velocity of moving ball are
and will not participate in any further collision. Now there u
are (2n – 2) blocks [Block 2 to (2n – 1)] and they are travel- un = u cos 45° = ___​  __  ​ 
÷   
​ 2 ​
ling just like the initial system. There will be (n – 1) colli-
u
sions in the next round (2 and 3, 4 and 5, ….. collide). After ut = u sin 45° = ___
​  __  ​ 
this, 2 and (2n – 1) will permanently leave and the system ÷   
​ 2 ​
will have (2n – 4) blocks left. They will further make (n – 2) When the balls are deformed by maximum amount, let
collisions. Process will continue and in the end, only two the common velocity along n-line be v1. [The second ball
middle blocks will collide and then move away from each has no velocity along t-line]. Momentum conservation along
other. n-line gives:
\  Total number of collisions
u__
2mv1 = m ​ ___  ​ 
= n + (n – 1) + (n – 2) + ... + 2 + 1 ÷   
​ 2 ​
= Sum of first ‘n’ natural numbers u
fi v1 = ____
​  __
   ​ 
n (n – 1)   
2​÷2 ​
= _______
​   ​    ______
2
​ v  12 + u2t 
At this instant speed of first ball is ÷  ​

÷ 
_______ __
Example 42   A ball moving translationally collides
u2 u2 ____   
​÷5 ​
elastically with another stationary ball of same mass. At = ​ __
​   ​  + __
​   ​ ​
  = ​  __  ​ u

8 2   
2​÷2 ​
the instant of collision, the angle between the straight line
passing through the centres of the balls and the direction of \  KE at the instant of maximum deformation is

(  )
initial motion of the striking ball is a = 45°. Assuming the __
balls to be smooth, find the fraction (h) of kinetic energy
of the striking ball that was used up in deformation at the
1
= __
​   ​  m ​​ ____
2
   2
​÷5 ​
​  __  ​ u  ​​ ​ +
  
2​÷2 ​
1
__
2 (  ) u 2
​   ​  m ​​ ____
​  __
   ​  ​​ ​
  
2​÷2 ​

( 
moment of maximum deformation.
Solution
3 1
= __
​   ​  ​ __
4 2
​   ​  mu2  ​ )
Concepts \  Fraction of KE stored as deformation energy is

(  )
(i) At the instant of maximum deformation, the veloc-
1
__ 3 1 2
ity components of two balls along the LOI (i.e., ​   ​  mu2 – __ ​   ​  ​ __
​   ​ mu   ​
2 4 2 1
n-line) will be same. h = ​ _______________  ​
      = __
​   ​  = 0.25
1 2
__ 4
Velocity component along t-line does not change. ​   ​  mu
2
(ii) Using momentum conservation along n-line will
give us the velocity (along n-line) of both balls Example 43   Ball over ball makes a super ball!
when maximum deformation occurs. A small ball of mass m is held above
(iii) Deformation PE = loss in KE another ball of mass M with a little gap
between them. The bigger ball is at height
h above the floor. The balls are released
simultaneously. How high will the smaller
ball rise after collision? Assume that the
collisions are elastic and M >> m.
Solution
Concepts
(i) Both balls fall freely to a distance h before the
bigger ball hits the floor.
(ii) On elastic collision with the floor, velocity of
the bigger ball gets reversed without change in
magnitude.
(iii) As the velocity of bigger block becomes vertically
upward, the smaller ball is still travelling down-
ward. Thus, relative velocity of approach is high
2.36  Mechanics II

for the two balls. Relative velocity of separation


will also be high.
(iv) Motion of bigger ball remains unaffected, when it
hits the smaller ball, due to its large mass.
____
Speed of the balls just before hitting the floor, u = ÷   ​ 
​ 2gh 

r
From geometry: sin q = __
​     ​ [r = radius of each ball]
2r
fi q = 30°
____
Velocity of bigger balls after colliding with floor   ​, as it is just
Velocity of A just after impact is v0 = ​÷5gL 
= u (≠) able to complete the circle.
__________ __
For collision between the balls, relative speed of approach fi v0 = ​÷5  × 10 ×        ms–1
1.5 ​ = 5 ​÷3 ​
= 2u. Since collision is elastic, relative speed of separa-
tion after collision = 2u. The bigger ball keeps moving Let velocity components of B along horizontal and vertical
with velocity u (≠). Thus, velocity of smaller ball becomes directions be vx and vy, as shown.
3u (≠). Using conservation of momentum along horizontal
(3u)2 direction
\  Height attained by smaller ball is H = _____​     ​ = 9h __
2g     ms–1
mvx = mv0  fi  vx = v0 = 5​÷3 ​
This is the reason we are calling it a super ball (!).
Velocity of B along n-line:
__
Example 44   A sheet ball A is suspended by an inextensible ÷   
​ 3 ​
before collision is u cos q = ___
​   ​  u
string of length L = 1.5 m, from point O. 2
Another identical ball B is projected down-
after collision is vy cos q – vx sin q
ward such that it moves while just remaining
__
in touch with the thread (see fig.). It collides ÷   
​ 3 ​ 1 __
elastically with A. Ball A just manages to = ___
​   ​ vy – __   
​   ​  × 5​÷3 ​
2 2
complete the vertical circle after collision.
Velocity of A along n-line:
Find the velocity (u) of the falling ball just
before collision. before collision = 0
Solution after collision = v0 cos (90 – q) = v0 sin q
__
Concepts 5​÷3 ​  
= ____ ​   ​   
(i) LOI is the line joining the centres of the two balls. 2

(  )
By looking at geometry, we can find the angle v1n – v2n
between initial velocity (u) of B and the LOI. Now, – ​ ________
​ u – u    ​  
​= 1

( 
1n 2n

)
(ii) Due to impulse applied by the string (during __ __ __
collision), we cannot use conservation of momen- ÷   
​ 3 ​
___     ____
5​÷3 ​   
5​÷3 ​
​   ​  vy – ____
​   ​   – ​   ​ 

tum along n-line. Momentum is conserved only in 2 2 2
fi – ​ ​  ________________
__   
 ​ 
   ​ = 1
horizontal direction. ÷   
​ 3 ​
___
​   ​  u – 0
( 
v1n – v2n
(iii) We will use – ​ ________
​ u – u   
1n 2n
)
​  ​ = 1 for elastic collision.
÷
__
  
​ 3 ​
2
÷
__
  
​ 3 ​ __
(iv) Velocity of A, immediately after impact is known, fi – ​ ___ ​  vy = ​ ___ ​ u – 5​÷3 ​
  
2 2
as it is given that it completes the circle. fi vy = 10 – u ...(i)
(v) Velocity of B does not change along t-line of
collision. Velocity of B will remain unchanged along t-line.
Momentum and Its Conservation  2.37

\ u sin q = vx cos q + vy sin q


__
u __ ÷   
​ 3 ​ vy
fi ​ __ ​ =    ◊ ​ ___ ​ +
  5​÷3 ​  ​ __ ​ 
2 2 2
fi vy = u – 15 ...(ii)

From (i) and (ii), u = 12.5 ms–1

Example 45   A beam of particles has each particle of \  Change in momentum of all particles hitting the wall
mass m moving with velocity v. The beam has n number of in time D t is
particles per unit volume. The beam strikes against a wall
D P = [2mv cos q] [n A cos q ◊ vD t]
at an angle q to the normal. Collisions are elastic. Find the
pressure exerted by the stream on the wall. Force applied by wall on particles = force by particles on
Solution the wall
D P
Concepts F = ___
​   ​ = 2mnv2 A cos2 q
D t
(i) If we consider a cross-
F
sectional area A normal \ Pressure = __
​   ​ = 2mn v2 cos2 q
A
to the beam, then all the
particles located inside
Example 46   Two bodies of same mass are tied with an
cylindrical volume (abcd)
inelastic string of length l. They lie together on a floor. One
___
will cross through the area
of them is projected vertically upwards with velocity ​÷6gl   ​. 
A in time D t.
Find the maximum height up to which the centre of mass
Volume (abcd) = A v D t of system of the two masses rise.
Number of particles crossing area A in time
Solution
D t = nAv D t
Concepts
(ii) Consider an area A that
is not exactly normal to (i) Initially, the projected body (call it body 1) moves
the beam but its projec- under gravity. There is no tension in the string.
tion in a direction per- (ii) Once body 1 moves through a distance l, the string
pendicular to the beam acquires tension. During the process of getting taut,
is A. Number of particles crossing through A¢ in the string applies an impulse on both the bodies.
time D t = number of particles crossing through A This sets the second body in motion. In fact, both
in time D t = nAv D t. will have same velocity after the string gets taut.
(iii) Each particle experiences change in momentum in (iii) The COM of the system moves with acceleration
a direction that is normal to the wall. There is no g (Ø) after both bodies are in air.
change in momentum parallel to the wall.
D P
(iv) F = ___
​   ​ 
D t

Let the area AB on the


wall where the beam strikes
be A. Projection of area A
normal to the beam is area
(AC) = A cos q
\  Number of particles
striking the wall in time
D t = number of particles
Let velocity of first body be v0 immediately before the
crossing area A cos q in time
string is taut.
D t = n (A cos q) v D t. _______ ___
Change in momentum of one particle is ​ u  2 – 2gl  
v0 = ÷   ​ 
​ = ​÷4gl 
l
D Pone = mv cos q (¨) – mv cos q (Æ) COM of the system is presently at height __
​    ​ .
2
= 2mv cos q (¨)
2.38  Mechanics II

__
Just after the string gains tension, let velocity of two ÷   
​ 3 ​ __
bodies be v. Momentum conservation gives ux = u cos 30° = 100 ◊ ​ ___ ​ = 50​÷3 ​ 
   ms–1
2
v0 __ 1
2mv = mv0  fi  v = __ ​   ​  = ÷    ​
​ gl  uy = u sin 30° = 100 ◊ ​ __ ​  = 50 ms–1 (≠)
2 2
__
   ​
Velocity of the COM at this instant is vcm = ​÷gl  Vertical displacement of the ball by the time it hits the
COM now moves with acceleration g (Ø) as the only carriage is 120 m (Ø)
external force on the system is weight. If COM rises further 1
by h then: \ y = uyt + ​ __ ​  ayt2 gives
2
02 = v cm
2
– 2g ◊ h 1
– 120 = 50 ◊ t – ​ __  ​ × 10 × t2  fi t = 12 s, – 2 s
2
2
v cm l
fi h = ​ ___ ​ = __
​    ​  Obivously, negative time in meaningless.
2g 2
l l \  time of flight, t0 = 12 s
\  Total rise of COM = __
​    ​  + __
​    ​  = l
2 2 When it hits the carriage, its horizontal velocity is still ux.
Conserving momentum along horizontal direction gives:
Example 47   A car P is moving with a uniform speed of
__
    ms–1 towards a carriage of mass 9 kg at rest kept on the
5​÷3 ​ (Mass of ball) ux = (Mass of ball + carriage) v1
rails at a point B, as shown in fig. The height AC is 120 m. __
  
1 × 50​÷3 ​ __
Cannon balls of 1 kg are fired from the car with an initial
fi v1 = ​ ________
 ​ 
      ms–1 [= velocity of carriage]
= 5​÷3 ​
velocity 100 ms–1 at an angle 30° with the horizontal. The 10
first cannon ball hits the stationary carriage after a time t0
and sticks to it. Determine t0. At t0, the second cannon ball The second ball is fired at t0 = 12 s after the __ first ball.
The car
__ moves horizontally a distance x = (5​     ms–1) (12 s)
÷3 ​
is fired. Assume that the resistive force between the rails and 0
    m between two shots.
= 60​÷3 ​
the carriage is constant and
ignore the vertical motion The trajectory of ball 2 is identical to that of ball 1
of the carriage throughout. \  Ball 2 will have same range as 1.
The second ball also hits __
and sticks to the carriage.    m
Thus, ball 2 will hit the carriage at a distance x0 = 60​÷3 ​
What will be the horizontal from the spot where first hit happened.
__
velocity of the carriage just \  Carriage travels 60​÷3 ​     m in 12 s. This will be pos-
after the second impact? sible only__ when resistive
__
force on the carriage is zero as
   × 12 = 60​÷3 ​
v1 t0 = 5​÷3 ​    m. The carriage will cover a distance
Solution __
  __  m in 12 s while travelling with constant velocity
of 60​÷3 ​
Concepts
    ms–1.
v1 = 5​÷3 ​
(i) Velocity of cannon balls is 100 ms–1 relative to the Again, using conservation of momentum in horizontal
ground (and not relative to the cannon). Unless it direction: Momentum of carriage + first ball in horizontal
is mentioned otherwise, we should take a given direction + momentum of second ball in horizontal direction
velocity as observed from the ground. = Momentum of carriage + 2 balls in horizontal direction.
(ii) Just considering the vertical motion of the projec- __ __
tile can give us t0 fi   )  + (1) (50​÷3 ​
(10) (5​÷3 ​   )  = 11 v2
__
(iii) The cannon ball has horizontal momentum when   
100 ​÷3 ​
it hits the carriage. Momentum conservation gives fi v2 = ______
​   = 15.75 ms–1
 ​ 
11
velocity of carriage after the ball sticks to it.
Example 48   A ball of mass m = 1 kg in falling vertically
(iv) The second ball will have path (and time of flight)
at a velocity v0 = 2 ms– 1 when it hits a smooth inclined face
identical to the first one. The second ball is fired at
of a wedge of mass M = 2 kg. The wedge is on a smooth
the instant the carriage starts moving. For a hit, it
table and is initially at rest. The coefficient of restitution
is necessary that displacement of the car between
1
two shots = displacement of carriage during time is e = __
​   ​ . Find the velocity of the
of flight of the second ball. 2
wedge and the ball after collision.
For cannon ball, horizontal and vertical components of Inclination angle of the wedge is
initial velocity are q = 30°.
Momentum and Its Conservation  2.39

Solution
Concepts
(i) Conservation of momentum along horizontal
direction, definition of coefficient of restitution and
the fact that velocity component of the ball along
t-line (i.e., a line parallel to wedge surface) does
not change; are sufficient to solve the problem.
(ii) To illustrate a different line of thinking, we will Solving (i) , (ii) and (iii) gives:
solve this problem in terms of impulse between the 1
v1 = ___
​  __  ​ ms–1
ball and the inclined face of the wedge. However, ÷   
​ 3 ​
we will still need the defining equation for coef- 2
ficient of restitution. v2 = ___
​  __  ​ ms –1  and
÷   
​ 3 ​
Let v1 = velocity of wedge (towards right) after v3 = 0
impact
v2 = velocity component of the ball in horizontal Thus, the ball moves horizontally to the left with
direction (towards left) 2
velocity ___
​  __  ​ ms–1and the wedge moves to the right with a
v3 = velocity component of the ball in vertical ÷   
​ 3 ​
direction (up) after collision. 1
J = impulse between the ball and the wedge velocity ___
​  __    ​ms–1.
÷   
​ 3 ​
(perpendicular to the incline).
Example 49   A ball hitting a rough
surface
A ball of mass m hits a flat horizontal
surface while travelling with a veloc-
ity u. The angle of incidence is q.
Coefficient of restitution is e and coefficient of friction is
m. Find the velocity of the ball after impact.
Solution
Concepts
(i) Vertical component of velocity of the ball before
Impulse = change in momentum collision is velocity of approach and vertical com-
ponent of ball’s velocity after collision is velocity
\ J = Mv1 = mv2 of separation. We can use (speed of separation) =
fi J = 2v1 = v2 ...(i) e (speed of approach).
(ii) Change in momentum of the ball in vertical direc-
Also, J cos 30° = mv3 – (– mv0) = m (v3 + v0)
__ tion is due to the impulse of normal reaction force
÷   
​ 3 ​ that has acted on it.
fi ​ ___ ​ J = (v3 + 2) ...(ii)
2 (iii) Impulse of normal force Jn = N D t
Also ( 
v1n – v2n
e = – ​ ________
​ u – u  
1n 2n
)
 ​  
​ Impulse of friction force Jf = f D t = m ND t
(iv) Horizontal impulse by friction causes change in

[  ]
v1 sin 30° – (– v2 sin 30° – v3 cos 30°) horizontal momentum of the ball.
1
fi ​ __ ​  = – ​ ​ _____________________________
         ​ ​
2 0 – v0 cos 30°
1
fi ​ __ ​  v0 cos 30° = (v1 + v2) sin 30° + v3 cos 30°
2
__ __
fi  v1 + v2 + ÷     v3 = ÷
​ 3 ​   
​ 3 ​ ...(iii)
2.40  Mechanics II

Speed of separation = e (speed of approach) Velocity of M in COM frame (before collision) is


fi vy = e uy = e u cos q ...(i) Mu0 mu0
u1¢ = u0 – ​ ______
   ​ = ______
​    
 ​
Impulse in vertical direction = change in momentum in M+m M+m
vertical direction Velocity of m in COM frame (before collision) is
Jn = mvy + muy [Jn = ND t] Mu0 Mu0
u¢ 2 = 0 – ______
​     
​= – ______
​    
 ​
= mu cos q (e + 1) ...(ii) M+m M+m
Impulse of friction, Jf = m n D t = m Jn After collision, the two masses will move with their speed
= m mu cos q (e + 1) unchanged when observed in COM frame. Their velocities
will be oppositely directed. This is necessary to conserve
Jf = change in horizontal momentum. momentum and energy.
m mu cos q (e + 1) (¨) = mvx (Æ) – m ux (Æ)
fi – m mu cos q (e + 1) = mvx – m u sin q
fi vx = u sin q – m u cos q (e + 1) ...(iii)
\  Speed of ball after impact is
______
v = ​÷v  x2 + vy2 
 ​ We do not know q. It can have any value. Thus, velocity
______________________________ u¢1 (velocity of M in coM frame) can have any direction. We
= u ÷  
​ [sin q + 1) cos q]2 + (e cos q)2 ​
– m (e      have drawn a circle of radius |​ u1¢ |​ and various radii represent
the various possible directions of u1¢.
Direction of motion makes angle f with vertical
vx sin q – m cos q (e + 1) ____
tan q m (e + 1)
tan f = ​ __
vy ​  =​ 
__________________
      ​ – ​ _______
  ​ = ​  e    e    ​ 
e cos q

Example 50   Maximum deflection


A mass M collides with a stationary mass m. Assume that
M > m and find the maximum deflection angle for M. ​_› ​_› ​_›
In ground frame, velocity of M is ​v  ​ 1 = ​u  ​ 1¢ + ​v  ​ CM.
Assume elastic collision. ​___› ​_›
Take ​PO​ =
   ​v  ​CM

Solution _​ __›
Concepts ​   represent u1¢ we have
Now, when OQ​
_​ __› _​ __› _​ __›
(i) If M < m, then it is possible for M to bounce ​PQ​ = ​   + OQ​
   PO​ ​   
directly backward (i.e, deflection = 180°). But
for M > m, there is a maximum angle less than ​_› ​_› ​_›
​v  ​ 1 = ​v  ​ CM + ​u  ​ 1¢
180°.
​___›
(ii) It is possible to solve this problem in ground frame Thus, ​PQ​  is velocity
_
​›
of M in ground frame. This way,
but we can save a lot of effort by considering what we can construct ​v  ​ 1 for different direction of u1¢.
​_› ​_›
happens in COM frame and then shifting back to Angle between ​v   ​1 and ​v   ​CM (which is original line of
the ground frame. motion of M) is maximum when the vector drawn from P
(iii) COM frame is zero momentum frame. Momentum to the tip of u1¢ becomes tangent to the circle shown.
of system is zero, before and after collision, in this u1¢
OR m
frame. \  sin qmax = ___
​    
​= ____
​ v   ​ = __
​   ​ 
PO CM M
(iv) If u is velocity in ground frame, then velocity in
COM frame is u¢ = u – vCM.
Example 51   Loading at a mine
Let initial velocity of M in ground frame be u0 In a coal mine, the produced coal is fed to a hopper, which
discharges it into railway wagons at a constant rate of
Mu0
vCM = ______
​    
 ​ b kgs–1. An empty wagon having mass m0 approaches the
M+m hopper while travelling at speed v0 on a smooth track. Length
Momentum and Its Conservation  2.41

of the car is L. As soon as the right end of the car is below Conserving momentum:
the hopper, it begins to discharge. mv = m0v0
Find the mass and speed of the wagon just after it moves
m0v0
out of the hopper. fi v = _______
​    
 ​ ...(i)
m0 + bt
L t
dx m0v0 dt
fi ​ ___ ​   = _______
​   ​  fi ​Ú ​   ​dx = m0v0 ​Ú ​    _______
   ​​      

dt m0 + bt 0 0 m0 + bt

m0v0
fi L = ____  ​ [ln (m0 + bt)​]t0​ ​​ 
​     
b

Solution

m0v0
L = ____
​     
b
m0 + bt
( 
 ​ ln ​ ​ _______
m0 ​  
 ​ )
Concepts (  bt
fi ln ​ 1 + ___ ) bL
​ m   ​  ​ = ____
0
​ m v  ​ 
0 0
(i) The falling coal brings no momentum with itself bL
____
bt
1 + ​ ___
​    ​ 
in horizontal direction. fi   e​ ​m0v0 ​
m0  ​ =
(ii) We can easily write velocity of the wagon at time
t by using conservation of momentum in horizontal bL
​ ____  ​ 
fi  m0 + bt = m0 ​e​m0v0 ​ = mass of the wagon at time t.
direction.

Let the wagon move through a distance x in time t. m0v0 bL


– ​ ____  ​ 
From (i) v = _______
​    
bL
 ​ = v0 e​ ​ m0v0 ​
____
Its mass at time t is m = m0 + bt. ​    ​ 
m0 ​e​m0v0 ​
Note:  Using the concept of variable mass will give same
result.
Worksheet 1
1. A system of N particles is free from any external (a) 1.3 Ns (b) 1.06 Ns
forces. Which of the following is true for the mag- (c) 1300 Ns (d) 105 Ns
nitude of the total momentum of the system? ​_›
8. An impulse
​_› ​_›
​I ​   changes the velocity of a particle
(a) It must be zero
from ​v  ​1  to ​v  ​2  . Kinetic energy gained by the particle
(b) It could be non-zero, but it must be constant is
(c) It could be non-zero, and it might not be 1 _​ › ​_› ​_› 1 _​ › ​_› ​_›
constant (a) ​ __ ​  ​I ​  . (​v  1 ​ + ​v  2 ​ ) (b) ​ __ ​  ​I ​  . (​v  1 ​ – ​v  2 ​ )
2 2
(d) The answer depends on the nature of the internal _
​› ​_› ​_› _
​› ​_› ​_›
forces in the system (c) ​I ​ .  (​v  ​1  – ​v  ​2  ) (d) ​I ​ .  (​v  ​1  + ​v  ​2  )
2. A system of N particles is free from any external 9. A super-ball is to bounce elastically back and forth
forces. Which of the following must be true for the between two rigid walls, at a distance d from each
sum of the magnitudes of the momenta of the indi- other. Neglecting gravity and assuming the velocity
vidual particles in the system? of super-ball to be v0 horizontal, the average force (in
large time interval) being exerted by the super-ball
(a) It must be zero
on one wall is:
(b) It could be non-zero, but it must be constant
1 ____m​v2​0​​  m​v2​0​​ 
(c) It could be non-zero, and it might not be __
(a) ​   ​  ​      ​   ____
(b) ​      ​ 
constant 2 d d
(d) It could be zero, even if the magnitude of the 2 m​v2​0​​  4 m​v2​0​​ 
(c) ​ _____
     
​ (d) ​ _____
     

total momentum is not zero d d
3. If the KE of a particle becomes four times its initial 10. A particle of mass 4 m, which is at rest, explodes
value, then the new momentum will be more than its into three fragments. Two of the fragments, each of
initial momentum by mass m, are found to move with a speed ‘v’ each
(a) 50% (b) 100% in mutually perpendicular directions. The minimum
(c) 125% (d) 150% energy released in the process of explosion is:
4. If the KE of a particle becomes 0.2% higher than its (a) (2/3) mv2 (b) (3/2) mv2
initial value, then the new momentum will be higher (c) (4/3) mv2 (d) (3/4) mv2
than its initial momentum by; 11. A 500 kg boat has an initial speed of 10 ms–1 as it
(a) 0.2% (b) 0.1% passes under a bridge. At that instant, a 50 kg man
(c) 0.125% (d) 1.50% jumps straight down into the boat from the bridge.
The speed of the boat after the man and boat attain
5. Two masses of 1 g and 4 g are moving with equal
a common speed is (neglect any resistance)
kinetic energy. The ratio of the magnitude of their
linear momentum is- 100 10
(a) ​ ____ ​ ms–1 (b) ​ ___ ​ ms–1
(a) 1:1 (b) 1:2 11 11
50 5
(c) 1:3 (d) 1:4 (c) ​ ___ ​ ms–1 (d) ​ ___  ​ ms–1
6. A train of mass M is moving on a circular track of 11 11
radius ‘R’ with constant speed v. The length of the 12. A shell is fired from a canon with a velocity v at an
train is half of the perimeter of the track. The mag- angle q with the horizontal direction. At the highest
nitude of linear momentum of the train will be point in its path, it explodes into two pieces of equal
masses. One of the pieces come to rest. The speed of
2 Mv
(a) 0 (b) ​ ____
p   

  the other piece immediately after the explosion is
(a) 3v cos q (b) 2v cos q
(c) MvR (d) Mv
3
7. A ball of mass 50 g is dropped from a height (c) ​ __ ​  v cos q (d) v cos q
2
h = 10 m. It rebounds losing 75% of its kinetic 13. Two pucks are initially moving along a friction-
energy. If it remains in contact with the ground for less surface, as shown in the diagram. The pucks
D t = 0.01 s, the impulse of the impact force is: (take have masses m1 < m2 and have equal magnitude of
g = 10 ms–2)

2.42
momentum. A constant force F is applied to each mvN
(b) ​ _______
    ​ and increasing
puck directly to the right for the same amount of Nm + M
non-zero time. After the pushes are complete, what mv
is the relationship for the size of the momenta of (c) ​ _______
    ​ and decreasing
Nm + M
pucks (p1 and p2)?
Nm + M
(d) ​ _______
mv    ​ 
and increasing

18. A gun fires small balls of mass 20 gm. It is firing


20 balls per second on a smooth horizontal table. The
collisions are perfectly elastic and balls strike at the
centre of table with a speed 5 ms–1 at an angle of
60° with the vertical. The force exerted by one of the
(a) p1 < p2 leg on ground is (assume total weight of the table is
(b) p1 = p2 0.2 kg and it is a four legged table):
(c) p1 > p2 (a) 0.5 N (b) 1 N
(d) More information about the masses, speeds, (c) 0.25 N (d) 0.75 N
force and time are required to answer the 19. Consider the following two statements:-
questions
A: Linear momentum of a system of particles is
14. A spacecraft of mass M moves with velocity v in zero.
free space at first, then it explodes breaking into two
B: Kinetic energy of a system of particles is
pieces. If after explosion, a piece of mass m comes
zero.
to rest, the other piece of space craft will have a
velocity: Then-
(a) Mv/(M – m) (b) Mv/(M + m) (a) A does not imply B and B does not imply A
(c) mv/(M – m) (d) mv/(M + m) (b) A implies B but B does not imply A
15. A bullet of mass m moving vertically (c) A does not imply B but B implies A
upwards with velocity ‘u’ hits a hang- (d) A implies B and B implies A
ing block of mass ‘m’ and gets embed- 20. A ball of mass m travelling with velocity 5 v collides
ded in it. The block is hanging with with and sticks to a ball of mass 5 m travelling in the
the help of a string, as shown in the same direction with velocity v. Impulse applied by a
figure. The height through which the ball on the other during collision is
block rises after collision is (assume 6 mv
sufficient space above block) (a) 5 mv (b) ​ ____ ​ 

5
(a) u2/2g (b) u2/g 8 mv 10 mv
(c) ​ ____ ​  
  (d) ​ _____ ​  
(c) u2/8g (d) u2/4g 10 3
16. A stationary body explodes into four identical 21. Which of the following does not hold when two parti-
fragments such that three of them fly off cles of masses m1 and 2 undergo elastic collision?
mutually perpendicular to each other, each with same (a) When m1 = m2 and m2 is stationary, there is
kinetic energy E0. The minimum energy of explosion maximum transfer of kinetic energy in head-on
will be: collision.
4E0
(a) 6E0 (b) ​ ____
 ​ 
  (b) When m1 = m2 and m2 is stationary, there is
3 maximum transfer of momentum in head-on
(c) 4E0 (d) 8E0 collision.
17. A continuous stream of particles of mass m and (c) When m1 >> m2 and m2 is stationary, after
velocity v, is emitted from a source at a rate of n head-on collision, m2 moves with twice the
per second. The particles travel along a straight line, velocity of m1.
collide with a body of mass M and get embedded into (d) When the collision is oblique and m1 = m2, with
it. If the body of mass M was originally at rest, its m2 stationary, after the collision the particle
velocity when it has received N particles will be move in opposite directions.
mvn
(a) ​ _______
    ​ and incresing 22. A particle moves in the X-Y plane under the influ-
Nm + n ence of a force such that its linear momentum is

2.43
p (t) = A [î cos (kt) – jˆ sin (kt)], where A and k are (c) M1 = M2
constants. The angle between the force and the (d) same for all values of M1 and M2
momentum is: 29. A drop of liquid moving vertically down at 2 ms–1
(a) 0° (b) 30° it hit by another drop of half volume, moving
(c) 45° (d) 90° horizontally at 1 ms–1. The drops coalesce and move
together. The velocity after impact will be:
23. A block of mass 2 kg is ___
pushed towards a very 1__ –1 ÷     –1
​ 17 ​
(a) ​ ___  ​  ms (b) ​ ____ ​ ms


heavy object moving with ÷   
​ 3 ​ 3
__ ___
2 ms–1 as shown. Assuming    ms–1
(c) 3 ​÷3 ​     ms–1
(d) 3 ​÷17 ​
elastic collision, the veloc-
ity of 2 kg mass after collision is: 30. A ball of mass m falls vertically to the ground from
(a) 12 ms–1 (¨) (b) 14 ms–1 (¨) a height h1 and rebound to a height h2. The change
in momentum of the ball on striking the ground is:
(c) 2 ms–1 (¨) (d) 8 ms–1 (Æ) ____ ____
24. A ball is dropped on a floor from a height h. If the (a) mg (h1 – h2) (b) m ​( ​÷2gh
  1 ​ + ​÷2gh )​
  2 ​  
coefficient of restitution is e, to what height will it __________ ___
rise after third collision:
  1 +  
(c) m ​÷2g (h h2) ​   ​  (h1 + h2)
(d) m ​÷2g 

(a) h (b) e2h 31. A spring scale is adjusted to read zero. Particles
(c) e3h (d) none of these of mass 1 g fall on the pan of the scale and collide
elastically and they rebound upward with the same
25. A sphere of mass m, moving horizontally with veloc-
speed. If the height of fall of particles is 2 m and their
ity v, enters a hanging bag of sand and stops. The
rate of collision is 100 particles per second, then the
bag was hanging with the help of a massless rope. If
scale reading in grams will be: (g = 10 ms–2)
the mass of the bag is M and it is raised by height
h, then the velocity of the sphere was (a) 1,000 g (b) 1,100 g
(c) 120.0 g (d) 126.5 g
M + m ____ M ____
(a) ​ ______
m      ​  
​ ​÷2 gh  (b) ​ __   ​ 
m ​  ​÷2 gh  32. Three interacting particles of masses 100, 200 and
____ 400 g have each a velocity 20 ms–1 along the positive
m m ____
(c) ​ ______
     ​  
 ​ ​÷ 2 gh  (d) ​ __ ​ ​ ÷2 gh 
  ​  direction of x-axis, y-axis and z-axis, respectively.
M+m M Due to the force of interaction, the third particle
26. A particle of mass m moving with horizontal speed stops moving and the velocity of second particle
6 ms–1 as shown in figure. If m << M, then for one- becomes (10  + 5  ) ms–1. Then the velocity of the
dimensional elastic collision, the speed of lighter first particle is
particle after collision will be:
(a) 20  + 20  + 70  (b) 10  + 10  + 5 
(c) 20  + 70  + 20  (d) 5  + 20  + 70 
33. A solid iron ball A collides
head on with another sta-
(a) 2 ms–1 in original direction tionary solid iron ball B.
(b) 2 ms–1 opposite to the original direction If the ratio of radii of the
(c) 4 ms–1 opposite to the original direction balls is n = 2, then the ratio of their speeds just after
the collision (e = 0.5) is:
(d) 4 ms–1 in original direction
(a) 3 (b) 4
27. A body falling from a height of 10 m rebounds from
hard floor. If it loses 20% energy in the impact, then (c) 2 (d) 1
coefficient of restitution is: 34. The magnitude of force (in Newton) acting on a body
(a) 0.89 (b) 0.56 varies with time (in micro second), as shown in the
figure. The magnitude of total impulse of the force
(c) 0.23 (d) 0.18
on the body from t = 4 m s to 16 m s is:
28. A body of mass M1 collides elastically with another
mass M2 at rest. There is maximum transfer of kinetic
energy when:
(a) M1 > M2
(b) M1 < M2

2.44
(a) 5 × 10–2 Ns (b) 5 × 10–3 Ns 39. A body of mass ‘m’ is dropped from a height of ‘h’.
(c) 5 × 10–4 Ns (d) 5 × 10–6 Ns Simultaneously another body of mass 2 m is thrown
35. Two blocks of masses m and M are moving with up vertically with such a velocity v that they collide
speeds v1 and v2 (v1 > v2) in the same direction on at the height h/2. If the collision is perfectly inelastic,
the frictionless surface, M being ahead of m. An ideal the velocity at the time of collision with the ground
spring of force constant k is attached to the backside will be:

÷ 
____
of M (as shown). The maximum compression of the 5 gh ___
spring when the blocks collide is: (a) ​ ____
​   ​ ​        ​ 
(b) ​÷gh 
4

÷ 
___ _____
gh  
​÷10 gh ​

(c) ​ ___
​   ​ ​    (d) ​ ______ ​ 

4 3
40. A ball collides with a smooth and fixed inclined
plane of inclination q after falling vertically through
÷___ 
__
m
(a) v1 ​ __
​   ​ ​   a distance h. If it moves horizontally just after the
k
impact, the coefficient of restitution is:

÷ 
M (a) tan2 q (b) cot2 q
(b) v2 ​ __
​   ​ ​  

÷ 
k _________
mM (c) tan q (d) cot q
(c) (v1 – v2) ​ _________
​      ​ ​ 
(M + m) k 41. A ball of mass m strikes the fixed inclined plane after
falling through a height h. If it rebounds elastically,
(d) None of above is correct.
the impulse on the ball is:
36. In the arrangement shown,
the pendulum on the left
is pulled aside. It is then
released and allowed to
collide with other pendu-
lum, which is at rest. A
perfectly inelastic collision
occurs and the system rises ____ ___
to a height h/4. The ratio of the masses (m1/m2) of   ​  
(a) 2 m cos q ​÷2gh    ​ 
(b) 2 m cos q ​÷gh 
the pendulum is: ____
  ​ 
2 m ​÷2 gh  ____
(a) 1 (b) 2 (c) ​ ________   

    ​ 
(d) 2 m ​÷2gh 
(c) 3 (d) 4 cos q
37. There are hundred identical sliders, equally spaced, 42. Two billiard balls undergo a head‑on collision. Ball
on a frictionless track, as shown in the figure. 1 is twice as heavy as ball 2. Initially, ball 1 moves
Initially, all the sliders are at rest. Slider 1 is pushed with a speed v towards ball 2, which is at rest.
with velocity v towards slider 2. In a collision, the Immediately after the collision, ball 1 travels at a
sliders stick together. The final velocity of the set of speed of v/3 in the same direction. What type of
hundred stuck sliders will be: collision has occured?
(a) inelastic
  (b) elastic
(c) completely inelastic
v v
(a) ​ ___  ​   (b) ​ ____
   ​  (d) Cannot be determined from the information
99 100
given
(c) zero (d) v
43. A sphere of mass m1 = 2 kg collides with a sphere of
38. A sphere of mass m moving with a constant velocity
mass m2 = 3 kg, which is at rest. Mass m1 will move
hits another stationary sphere of the same mass. If
at right angle to the line joining centres at the time
e is the coefficient of restitution, then ratio of speed
of collision, if the coefficient of restitution is:
of the first sphere to the speed of the second sphere
after head-on collision will be: 4 1
(a) ​ __ ​   (b) ​ __ ​ 

(  ) (  )
1–e 1+e 9 2
(a) ​​ _____  (b) ​​ _____ 
÷ 
__
​   ​  ​ ​ 2 2
1+e 1–e (c) ​ __ ​   (d) ​ __​   ​ ​  
3 3
(c) ​( ​ _____  (d) ​( ​ _____ 
e – 1) e + 1)
e+1 e–1
 ​  ​  ​  ​

2.45
44. A small ball falling (a) zero (b) (m1 + m1) gt0
vertically downward 1
(c) 2 (m1 + m2) gt0 (d) ​ __ ​  (m1 + m2) gt0
strikes elastically a 2
massive inclined cart 48. Two blocks of masses 10 kg and 4 kg are connected
moving with velocity by a spring of negligible mass and placed on a
4 ms–1 horizontally, as frictionless horizontal surface. An impulse gives
shown. At the instant of impact, the velocity of the a velocity of 14 ms–1 to the heavier block in the
ball was 4 ms–1. The velocity of the rebound of the direction of the lighter block. The velocity of the
ball is centre of mass is:
__ __
(a) 4​÷2 ​     ms–1     ms–1
(b) 4​÷__
3 ​ (a) 30 ms–1 (b) 20 ms–1
–1
(c) 4 ms     ms–1
(d) 4​÷5 ​ (c) 10 ms–1 (d) 5 ms–1
45. Two masses A and B of mass M and 2 M respectively 49. A projectile of mass “m” is projected from ground
are connected by a compressed ideal spring. The with a speed of 50 ms–1 at an angle of 53° with the
system is placed on a horizontal frictionless table horizontal. It breaks up into two equal parts at the
and given a velocity u  in the z-direction, as shown highest point of the trajectory. One particle comes to
in the figure. The spring also gets released. In the rest immediately after the explosion. The ratio of the
subsequent motion, the line from B to A is always radii of curvatures of the moving particle just before
along x-direction. At some instant of time, mass B and just after the explosion are:
has ​_an x-component of velocity as vx  . The veloc- (a) 1:4 (b) 1:3

ity ​v   ​A of mass A at that instant is
(c) 2:3 (d) 4:9
50. AB and CD are two smooth
parallel wall at separation l. A
child rolls a ball along ground
from A towards point P. Find
PD so that ball reaches point B after striking the wall
CD. Given coefficient of restitution, e = 0.5.
(a) 0.5 m (b) 1 m
(c) 1.5 m (d) 0.8 ms
(a) vx  + u  (b) – vx  + u  51. A ball is shot from the floor in a long hall having a
(c) – 2 vx  + u  (d) 2 vx  + u  roof at a height of 15 m. Initial velocity of the ball
46. A disc A of radius r moving on perfectly smooth is 25 ms–1 at an angle of 53° with the floor. The ball
surface at a speed v undergoes an elastic collision lands on the floor at a distance x m from the point
with an identical stationary disc B. Find the velocity of projection. Assume collision with roof as elastic,
of the disc B after collision, if the impact parameter if any, and find x.
is r/2, as shown in the figure.

(a) 20 m (b) 40 m
___ (c) 30 m (d) 15 m
   
​÷15 ​ v
(a) ​ ____ ​ v

  (b) ​ __  ​ 52. Two masses m1 and m2 are connected by a spring
4 4 of spring constant k and are placed on a frictionless
__
v ÷    v
​ 3 ​  horizontal surface. Initially, the spring is stretched
(c) ​ __  ​ (d) ​ ____ ​   through a distance x0 when the system is released
2 2
from rest. Find the distance moved by the two masses
47. Two particles of masses​_ m1 and ​_›
m2 in projectile
› before they again come to rest.
motion have velocities ​v ​  1 & ​v ​  2 respectively, at
time t =_​ 0. They_​ collide at time t0. Their velocities 2 m2 x0 _______
2 m1 x0 2 m2 x0 _______
2 m1 x0
› › (a) ​ _______  
 ​, ​   ​ (b) ​ _______ 

   
 ​, ​   

 ​
become ​v   ​¢1 and ​v   ​¢2 at _​ time 2 t0_​ while still_​ moving in m1 + m2 m1 – m2 m1 + m2 m1 + m2
› › › _
​›
air. The value of: [(m1 ​v   ​¢1 + m2 ​v   ​¢2) – (m1 ​v   ​1 + m2 ​v   ​2)] 2 m2 x0 _______2 m1 x0
is: (c) ​ _______
m1 – m     ​, ​ m – m     ​ (d) None of these
2 1 2

2.46
53. A ball of mass m is projected with speed v into the 57. A particle of mass m, moving in a circular path of
barrel of a spring gun of mass M, initially at rest radius R with a constant speed v2 is located at point
on a frictionless surface. The mass m sticks in the (2 R, 0) at time t = 0 and a man starts moving with
barrel at the point of maximum compression of the velocity v1 along the +ve y-axis from origin, at time
spring. No energy is lost in friction. What fraction t = 0. Calculate the linear momentum of the particle
of the initial kinetic energy of the ball is stored in wrt the man as a function of time. (w = v2/R).
the spring?

2 M M
(a) ​ ______
   ​ (b) ​ ______
    ​
M+m M+m
M
(c) ​ _______
     ​ (d) None of these
2 M + m (a) mv2 sin w t  – m (v2 cos w t – v1) 
54. A bullet of mass 20 g travelling horizontally with a (b) – mv2 sin w t  – m (v2 cos w t – v1) 
speed of 500 ms–1 passes through a wooden block (c) – mv2 sin w t  + m (v2 cos w t – v1) 
of mass 10.0 kg, initially at rest on a level surface. (d) None
The bullet emerges with a speed of 100 ms–1 and 58. Particle ‘A’ moves with
the block slides 20 cm speed 10 ms–1 in a fric-
on the surface before tionless circular fixed
coming to rest. Find horizontal pipe of radius
the friction coefficient 5 m and strikes particle
between the block and ‘B’ having twice the
the surface: mass of A. Coefficient
(a) 0.16 (b) 0.26 of restitution is 1/2 and
(c) 0.36 (d) None of these particle ‘A’ starts its journey at t = 0. The time at
55. A ball hits the floor and rebounds after inelastic which second collision occurs is:
collision. In this case: p 2p
(a) ​ __ ​  s (b) ​ ___ ​  s
(a) The momentum of the ball just after the colli- 2 3
sion is same as that just before the collision 5p
(c) ​ ___ ​  s (d) 4p s
(b) The mechanical energy of the ball remains same 2
in the collision 59. A particle of mass m is given initial speed u, as
(c) The total momentum of the ball and the earth shown in the figure. It trans-
is conserved fers to the fixed inclined
(d) The total energy of the ball and the earth is plane without a jump. Its
conserved trajectory changes sharply
from the horizontal line to
56. Block A of mass M is moving with a speed of v0 on
the inclined line. All the surfaces are smooth and
a frictionless surface that ends in a wall, as shown
90° ≥ q > 0°. Then the height to which the particle
in figure. Farther from the wall is a more massive
shall rise on the inclined plane (assume the length
block B of mass a M (a > 1), initially at rest. block
of the inclined plane to be very large)
A undergoes elastic collision with the block B and
the wall. If two blocks undergo only one collision (a) increases with increase in q
then maximum value of a is- (b) decreases with increase in q
(c) is independent of q
(d) data insufficient
60. Three blocks are placed on smooth horizontal surface
and lie on same horizontal straight line. Block 1 and
block 3 have
(a) 1 (b) 2 mass m each
(c) 3 (d) 4 and block 2 has

2.47
mass M (M > m). Block 2 and block 3 are initially 64. A ball of mass m collides horizontally with a stationary
stationary, while block 1 is initially moving towards wedge on a rough horizontal surface. Consider
block 2 with speed v, as shown. Assume that all collision in the two cases as shown. Neglect friction
collisions are head on and perfectly elastic. What between ball and wedge. Two students comment on
M system of ball and wedge in these situations
value of __​ m ​  ensures that Block 1 and Block 3 have
the same final speed?
__ __
(a) 5 + ÷   
​ __
2 ​   
(b) 5 – ​÷2 ​
__
  
(c) 2 + ​÷5 ​   
(d) 3 + ​÷5 ​
61. A particle of mass m is moving along the x-axis with
speed v when it collides with a particle of mass 2 m
initially at rest. After the collision, the first particle Student 1:  Momentum of system in x-direction will
has come to rest and the second particle has split change by significant amount in both cases.
into two equal-mass pieces. Which of the following
Student 2:  There are no impulsive external forces in
statements correctly describes the speeds of the two
y-direction in both cases; hence the total momentum
pieces? (q > 0)
of system in y-direction can be treated as conserved
in both cases.
(a) Student 1 is wrong and 2 is correct
(b) Student 1 is correct and 2 is wrong
(c) Both are correct
(d) Both are wrong
(a) Each piece moves with speed v. 65. Two identical carts are constrained to move on a
straight line. Ram and Shyam are two twins of same
(b) Each piece moves with speed v/2.
mass who are sitting on the two carts. The carts
(c) One of the pieces moves with speed v/2, the are moving with same velocity. At some time snow
other moves with speed greater than v/2 begins to pour uniformly in vertically downward
(d) Each piece moves with speed greater than v/2. direction. Ram throws off the falling snow sideways
62. AB is an L-shaped obstacle fixed on a horizontal and in the second cart shyam is asleep. (Assume that
smooth table. A ball strikes it at friction is absent)
A, gets deflected and restrikes it (a) Cart carrying Ram will speed up while cart
at B. If the velocity
​_›
vector before carrying shyam will slow down
collision is ​v ​   and coefficient of (b) Cart carrying Ram will remain at the same speed
restitution of each collision is ‘e’, while cart carrying shyam will slow down
then the velocity of ball after its
(c) Cart carrying Ram will speed up while cart
second collision at B is
​_› ​_› carrying shyam will remain at the same speed
(a) e2 ​v   ​ (b) – e2 ​v   ​
​_› (d) Cart carrying Ram as well as shyam will slow
(c) – e​v  ​   (d) data insufficient down
63. As shown in the figure a ball of mass m moving 66. A wagon filled with sand has a hole so that sand
vertically with speed 3 ms–1 hits a smooth inclined leaks through ​_the ›
bottom at a constant rate l. An
plane of a wedge, which external force ​F   ​ acts on the wagon in the direction
is placed on a smooth of motion. Assuming ​_›
instantaneous velocity of the
table with a back support. wagon to be ​v   ​ and initial mass of system to be m0,
The ball rebounds with a the force equation governing the motion of the wagon
velocity in the horizontal is:
​_› ​_› ​_› ​_›
direction. Inclination of the d ​v   ​ ​_› d ​v   ​ ​_›
(a) ​F   ​ = m0 ​ ___ ​ + l ​v   ​ (b) ​F   ​ = m0 ​ ___ ​ – l ​v   ​
incline is 30°. The impulse applied by the support on dt dt
​_› ​_› ​_› ​_›
the wedge during collision is d ​ v   ​ d ​v   ​ _
​›
(c) ​F  ​  = (m0 – l t) ​ ___ ​   (d) ​F  ​  = (m0 – l t) ​ ___ ​ + l ​v  ​ 

__
(a) 3 m (b) ​÷ 3 ​  m dt dt
__ 67. If the thrust force on a rocket which is ejecting gases
   m
(c) 1/​÷3 ​  (d) this is not possible with a relative velocity of 300 ms–1, is 210 N. Then
the rate of combustion of the fuel will be:

2.48
(a) 10.7 kgs–1 (b) 0.07 kgs–1 69. A rocket of mass 4000 kg is set for vertical firing.
(c) 1.4 kgs–1 (d) 0.7 kgs–1 How much gas must be ejected per second so
68. An open water tight railway wagon of mass that the rocket may have initial upwards accel-
5 × 103 kg coasts at an initial velocity 1.2 ms–1 eration of magnitude 19.6 ms–2. Exhaust speed of
without friction on a railway track. Raindrops fall fuel is 980 ms–1 and acceleration due to gravity is
vertically downwards into the wagon. The velocity 9.8 ms–2
of the wagon after it has collected 103 kg of water (a) 240 kgs–1 (b) 60 kgs–1
will be –1
(c) 120 kgs (d) None
(a) 0.5 ms–1 (b) 2 ms–1
–1
(c) 1 ms (d) 1.5 ms–1

2.49
Worksheet 2
1. In an elastic collision in absence of external force, (a) The impulse imparted by tension force to the
which of the following is/are correct? 3 mu
block of mass 3 m is ____ ​   ​   during the impact of
(a) The linear momentum is conserved. 5
bullet.
(b) The potential energy is conserved in collision. (b) The impulse imparted by tension force to the
(c) The final kinetic energy is less than the initial 3 mu
block of mass m is ____ ​   ​   during the impact of
kinetic energy. 5
bullet.
(d) The final kinetic energy is equal to the initial (c) Momentum of B + bullet system is conserved
kinetic energy. during the impact.
2. A small ball collides with a heavy ball initially at (d) The impulse imparted by tension force on the
rest. In the absence of any external impulsive force, 6 mu
it is possible that ceiling is ____ ​   ​ 
 during the impact of bullet.
5
(a) Both the balls come to rest. 6. Two particles A and B having masses mA and
(b) Both the balls move after collision. mB respectively, start moving due ​_to their ​_mutual
› ›
(c) The moving ball comes to rest and the stationary interaction only. At any time ​_‘t’, ​a   ​A ​_and ​a   ​B are
› ›
ball starts moving. their respective accelerations, ​v   ​A and ​v   ​B are their
(d) The stationary ball remains stationary, the respective velocities; and WA and WB are the work
moving ball changes its velocity. done on A and B respectively by the mutual force
upto that time. Which of the followings is/are always
3. A block moving in air explodes in two parts. then,
correct?
just after explosion (neglect change in momentum ​_› ​_› ​_› ​_›
due to gravity) (a) ​v   ​A + ​v   ​B = 0 (b) mA ​v   ​A + mB ​v   ​B = 0
​_› ​_›
(a) the total momentum of two parts must be (c) WA + WB = 0 (d) ​a  A ​ + ​a  B ​ = 0
equal to the momentum of the block before 7. A set of n-identical cubical blocks lie at rest along a
explosion. line on a smooth horizontal surface. The separation
(b) the total kinetic energy of two parts must be between any two adjacent blocks is L. The block at
equal to that of block before explosion. one end is given a speed v towards the next one, at
time t = 0. All collisions are completely inelastic,
(c) the total momentum must change.
then
(d) the total kinetic energy must increase.
(a) The last block starts moving at t = n (n – 1) ___ ​  L  ​. 
4. Two bodies of same masses collide head on elasti- 2 v
cally; then L
__
(b) The last block starts moving at t = (n – 1) ​   ​ .
(a) their velocities are interchanged. v
(c) The centre of mass of the system will have a
(b) their speeds are interchanged.
final speed v/n.
(c) their momenta are interchanged.
(d) The centre of mass of the system will have a
(d) the faster body slows down and the slower body final speed v.
speeds up.
8. A particle strikes a horizontal smooth floor with a
5. A system of two blocks A and B are connected by velocity u, making an angle q with the floor and
an inextensible massless string, as rebounds with velocity v, making an angle f with
shown. The pulley is massless and the floor. If the coefficient of restitution between the
frictionless. Initially, the system is at particle and the floor is e, then:
rest with block A resting on a table
and B hanging freely. A bullet of (a) the impulse delivered by the floor to the body
mass ‘m’ moving with a vertically is mu (1 + e) sin q.
downward velocity ‘u’ hits the block (b) tan f = e tan q
______________
‘B’ and gets embedded into it. The system is set into (c) v = u ​÷1  – (1 – e2) sin
  
2
 q ​
motion
(d) the ratio of the final kinetic energy to the initial
kinetic energy is (cos2 q + e2 sin2 q)

2.50
9. The figure shows a string of equally spaced beads of 12. Two beads, each of mass
mass m, separated by distance d. The beads are free m, are constrained to move
to slide without friction on a thin wire. A constant along the circumference of
force F acts on the first bead initially at rest till it a smooth circular hoop of
makes collision with the second bead. The second mass m. The beads are ini-
bead then collides with the third and so on. Suppose tially located at opposite
all collisions are elastic, then: ends of a diameter and given
equal velocities v0, as shown in the figure. The entire
arrangement is located in gravity-free space. Collision
between the beads is elastic.
(a) The hoop will be at rest when the two beads
(a) speed of the first bead immediately before and are about to collide.
immediately after its collision with the second
____ (b) Speed of the two beads immediately after
bead is ​ ____÷ 
2 Fd
​  m    ​ ​  and zero respectively. collision between them is __
2
​   ​  v0.
3
÷
__
  
​ 7 ​
(b) speed of the first bead immediately before and ____
(c) Speed of the bead just before collision is ​    ​. 
3 v0
immediately after its collision with the second
____ ____ (d) Centre of the hoop can move in a direction
÷ 
2 Fd
____
bead is ​ ​  m    __
÷ 
1 ____ 2 Fd
​ ​  and ​   ​  ​ ​  m   
2
​ ​  respectively. inclined to the initial direction of v0.
13. Given figure shows the
(c) speed of the second bead immediately after its
velocity of an object as
collision with third bead is zero. ____ a function of the time.
(d) the average speed of the first bead is __ ÷ 1 2 Fd
​   ​  ​ ____
2
​  m    ​ ​ .  The object with mass
10 kg is being pushed
10. A ball moving with a velocity v hits a massive wall along a straight line
moving towards the ball with a velocity u. An elastic on a frictionless sur-
impact lasts for a time D t. face by an external force (F). At t = 3 s, the force
(a) The average elastic force acting on the ball stops pushing and the object moves freely. It then
m (u + v) collides head on and sticks to another object of mass
is ​ ________     
​.
D t 25 kg.
(b) The average elastic force acting on the ball (a) External force F is 50 N.
2 m (u + v) (b) Velocity of the second particle just before the
is ​ _________
    ​.  collision is 1 ms–1.
D t
(c) The kinetic energy of the ball increases by (c) Before collision, both bodies are moving in the

same direction.
4 mu (u + v).
(d) Before collision, bodies are moving in opposite
(d) The kinetic energy of the ball remains the same
direction with respect to each other.
after the collision.
14. Consider a particle at rest, which may decay into two
11. Two blocks A and B each of mass ‘m’ are con-
(daughter) particles or into three (daughter) particles.
nected by a massless spring of natural length L and
Which of the following is true in the two-body case
spring constant k. The blocks are initially resting on
but false in the three-body case?
a smooth horizontal plane. A third block C, also of
mass m, moves on the plane with a speed ‘u’ along (There are no external forces and the masses of
the line joining A and B and collides elastically with daughter particles are known.)
A then which of the followings is/are correct: (a) Velocity vectors of the daughter particles must
(a) KE of the AB system at maximum compression lie in a single plane.
of the spring is zero. (b) Given the total kinetic energy of the system, it is
possible to determine the speed of each daughter
(b) The KE of AB system at maximum compression
2 particle.
is (1/4) mu .

÷ 
__
m (c) Given the speed (s) of all but one daughter
(c) The maximum compression of spring is u ​ __ ​   ​ ​  .
k particle, it is possible to determine the speed

÷ 
___ of the remaining particle.
m
(d) The maximum compression of spring is u ​ ___ ​     ​ ​ . (d) The total momentum of the daughter particles
2k
is zero.

2.51
15. A projectile is launched from the origin with speed (d) Oblique collision is that collision in which the
v at an angle q from the horizontal. At the highest colliding bodies do not move along the same
point in the trajectory, the projectile breaks into two straight-line path.
pieces– A and B, of masses m and 2 m, respectively. 18. In a one-dimensional collision between two identical
Immediately after the breakage, piece A is at rest particles A and B, B is stationary and A has momen-
relative to the ground. Neglect air resistance. Which tum p before impact. During the impact, B gives
of the following sentences most accurately describes impulse J to A.
what happens next? (A) The total momentum of the ‘A plus B’ system
is P before and after the impact, and (p – J)
during the impact.
(b) During the impact, A gives impulse J to B
2J
(c) The coefficient of restitution is ___​   ​ – 1
P
J
(D) The coefficient of restitution is __ ​   ​  + 1
(a) Piece B will hit the ground first, since it is more P
massive. 19. Two trolleys A and B of equal masses M​_are mov-
› ​_›
(b) Both pieces have zero vertical velocity ing in oppsite directions with velocities ​v   ​ and – ​v  ​ 
immediately after the breakup, and therefore respectively on separate horizontal frictionless paral-
they hit the ground at the same time. lel tracks. When they start crossing each other, a ball
(c) Piece A will hit the ground first, because it will of mass m is thrown from B to A and another of same
have a downward velocity immediately after the mass​_›
is thrown from A to B with velocities normal
breakup. to ​v   ​. Assume no lateral deviation of the trolleys.
(d) There is no way of knowing which piece will (a) If the two balls are thrown simultaneously, the
hit the ground first, because not enough infor- event will lead to equal change in speeds of the
mation is given about the breakup. two trolleys.
16. A particle of mass m makes a head-on elastic collision (b) When ball is thrown from A to B after the
with another particle of mass 2 m initially at rest. ball thrown from B reaches A, speed of B will
The velocity of the first particle before and after change more than that of A.
collision is given to be u1 and v1 and the velocity (c) The ball thrown from A to B missed the trol-
of second particle after the collision is v2. Which ley and fell on ground. Final speed of A will
of the following statements is true in respect of this become smaller than B.
collision? (d) None of the above
(a) For all values of u1, v1 will always be less than 20. A bomb of mass 3 m is kept inside a closed box of
u1 in magnitude. mass 3 m and length 4 L at its centre. It explodes in
(b) The fractional loss in kinetic energy of the first two parts of masses m & 2m. The two parts move
8 in opposite direction and stick to the opposite walls
particle is __ ​   ​ .
9 of the box. Box is kept on a smooth horizontal
(c) The gain in kinetic energy of the second particle surface.

(  )
8 th
is ​​ ​ __  ​  ​​ ​ of the initial kinetic energy the first
9
particle.
(d) There is a net loss in the kinetic energy of the
two-particle system in the collision.
17. Which of the following statements is/are correct? (a) Both parts of the bomb must hit the walls
(a) Most of the collisions on the macroscopic scale simultaneously.
are inelastic collisions. (b) If there is friction between the parts of the bomb
(b) In a perfectly inelastic collision, there is a com- and the floor of the box, then the box will get
plete loss of KE. displaced in the direction in which mass 2m
travelled.
(c) Forces involved in elastic collision are conser-
vative in nature. (c) The box will get displaced by L/3.
(d) None of the above

2.52
21. In the figure shown, the cart of mass 6 m is intially 23. Consider a block of mass 10 kg, which is on a
at rest. A particle of mass m is attached to the end smooth surface. It is subjected to a horizontal force
of the light rod, which can rotate freely about A. If of 6 N for 4 s. Observer A is attached to ground and
the rod is released from rest in a horizontal position another observer B is in a reference frame moving
shown, with velocity 2 ms–1 as shown in figure. Which of
the followings are correct?

(a) The cart will be moving with speed equal to


1/6 of the speed of the particle at the instant
the rod becomes vertical. (a) The final speed of the block is 7.4 ms–1, if it
(b) The centre of mass of the cart (not including has initial speed of 5 ms–1 as measured by A.
the particle) will stay at rest. (b) The final speed of the block is 7.4 ms–1, if it
(c) The centre of mass of the cart will oscillate as has initial speed of 5 ms–1 as measured by B.
the pendulum swings inside it. (c) For both the observers, change in momentum
(d) the velocity vrel of the particle with ____
respect to of the body is equal to impulse experienced

÷ 
7
__ by it.
the cart when the rod is vertical is ​ ​   ​ gl  
​.
3 (d) None
22. A cannon is mounted on a trolley. A small cannon 24. A ball of mass m is projected from a point P on the
ball is fired at an angle q and velocity vrel relative ground, as shown in the figure. It was supposed to
to trolley, which is free have a horizontal range R. However, it hits a fixed
to recoil frictionlessly on vertical wall elastically at a distance l from P. Which
a smooth surface. Which of the following are correct?
of the following trajecto-
ries (represented by dark
curves) is not possible for
the cannon ball?

(a) (b)
(a) The ball will return to the point P directly if
l = R/2.
(b) The ball will fall between point P and the wall
if l < R/2.
(c) (d) (c) The ball will fall between point P and the wall
if l > R/2.
(d) If l < R/2, the ball spends more time in air after
collision than before it.

2.53
Worksheet 3
1. A beam of neutrons has all the particles travelling angle q, and rebounds
towards a surface with a speed v. Separation between v
with speed ​ __  ​ . Find
the successive particles is 2
d. On hitting the surface the impulse imparted
normally, the particles by the floor to the
rebound along the original sphere during the
line of motion with same time intervals for which it interacted with the wall.
speed. Neglect the inter- Wall is smooth.
action of rebounding par- 6. An L-shaped block A is placed on a smooth horizon-
ticles with the incoming tal surface. A spring of force constant k is attached
particles. Mass per unit length of the incident beam to its vertical arm as shown. A smooth block B is
m
is __
​   ​ . Find the force experienced by the surface. placed on A and is moved to right so that the spring
d
2. During a hailstorm, stones of 1.0 cm diameter fall on gets compressed by
flat roof of a house at a speed of 20 ms–1. The stones x0 = 1 m. The system
hit the roof normally and 2000 stones fall on every is released from rest.
square meter area of the roof in 1 second. The hail- Find velocity of A
stones do not rebound. Find the force applied by the when B leaves it. Take
falling hailstones on the roof. Density of hailstones mA = 5 kg, mB = 1 kg and k = 100 Nm–1.
= 900 kg m–3, and area of roof is 100 m2. 7. A triangular wedge of mass M rests on a smooth
3. Two identical blocks A and B have mass m each and horizontal surface. It is connected to a relaxed spring
are kept on a smooth horizontal surface connected of force constant k. The other end of the spring
with a rope of mass m. The rope is straight and is secured to a wall and the spring is horizontal.
horizontal. A bullet of mass m flying horizontally at Inclination angle of inclined surface of the wedge is
velocity u hits blocks q. A ball of mass m is dropped from some height on
B and gets embedded the inclined face. It hits the inclined face at a height
into it. For the small h above the horizontal surface. Immediately after hit-
time interval for which ting the wedge, the ball moves horizontally. It lands
the bullet interacts on the floor at a distance from its original line of
with B, find fall. Find the maximum compression in the spring.
(i) Impulse applied by B on the rope.
(ii) Impulse applied by the bullet on B.
(iii) Impulse applied by the rope on block A.
4. Two particles A and B of mass m each are placed on a
smooth horizontal surface connected by a light string
of length 2L. Separation between the particles is L.
Particle B is imparted a velocity u in a direction that
is perpendicular to line AB (see figure). Find velocity
8. Two identical rail road cars (each of mass M) are
of each particle after the string gets taut. Also find
moving on same smooth track with velocity u. There
the impulse
is a man of mass m on the rear car. He jumps hori-
applied by the
zontally with
string on parti-
a velocity v
cle A. Express
relative to his
this impulse
car and lands
in terms of
on the front
unit vectors
car. Find the
and .
velocities of the two cars after this jump.
5. A ball of mass m slides with velocity v on a smooth
horizontal surface. It hits an inclined wall, inclined at

2.54
9. A simple pendulum is 14. A projectile is launched from a point O on a floor
suspended from a peg at velocity u, making an angle q with the horizon-
on a vertical wall. The tal. Coefficient of restitution between the projectile
pendulum is pulled and the floor is e. Assuming the floor to be smooth,
away from the wall to a find:
horizontal position and (i) Total horizontal distance travelled by the pro-
released. The ball hits jectile before it stops.
the wall, coefficient of
2 (ii) Total time elapsed before it comes to rest.
restitution being e = ___
​  __  ​.  Find the minimum number
  
​÷5 ​ (iii) The angle made by velocity vector of the pro-
of collisions after which the amplitude of oscillation jectile with horizontal after nth impact.
becomes less than 60°.
10. Two pendulums of same
length have bobs of
masses m and 2m. They
are suspended from
same point and raised to
a height H above their
lowest position (see fig-
ure). They are released 15. Two men, each of mass m, stand on the edge of
from this position and they collide head on, elasti- a stationary flat car of mass
cally at the lowest point. To what height will the balls m. The car is on a smooth
rise for the first time after collision? horizontal surface. Calculate
11. A wooden block of mass M is the velocity of the car after
suspended like a pendulum, using both men jump off with same
a string of length L. A horizon- horizontal velocity u relative
tally flying bullet has mass m and to the car.
speed u. It strikes the block and (i) Simultaneously  (ii) One after the other.
gets embedded. Find 16. Two identical discs of
(i) Loss in mechanical energy during collision. mass m are in contact on
(ii) Minimum value of u so that the combined mass a smooth table. A third
completes the vertical circle. identical disc travelling
with velocity u, hits them
12. A ball is pro- symmetrically. After col-
jected form point lision, the third disc comes to rest. Find coefficient of
A on the ground restitution (e) and percentage loss in kinetic energy.
at a velocity u,
making an angle 17. A ball of mass m moving downwards with a velocity
q with horizontal, v hits a wedge of mass M which is placed on a
It hits a smooth smooth horizontal table. The wedge is connected to
vertical wall at B a relaxed spring of force constant k as shown. Find
and returns to the maximum compression in the spring subsequent to
projection point A along the path BCA. Distance of collision. q = 45° and collision is elastic.
the wall from A is d. Find coefficient of restitution
for the collision of the ball with the wall.
13. A 5 kg projectile is fired from ground with an initial
velocity of 24 ms–1 making an angle of 60° with the
horizontal. When at the top, it explodes into two
fragments of equal mass. One of the fragments moves
vertically upward after the blast. After both of them 18. Two blocks of mass 2 kg and M are at rest on an
land on the ground, separation between them was inclined plane and are separated by a distance of
found to be 45 m. Find speed of the two fragments 6.0 m as shown. The coefficient of friction between
just after explosion. each of the blocks and the inclined plane is 0.25.

2.55
The 2 kg block is given constant equal to b kgs–1. Find time dependence of
a velocity of 10.0 ms–1 velocity and acceleration of the wagon in the process.
up the inclined plane. It Neglect friction.
collides with M, comes 23. A block of mass 4 kg is at rest on a horizontal
back and has a veloc- table. Another block of mass 2 kg is approaching it.
ity of 1.0 ms–1 when it When the separation between the blocks is 16 cm,
reaches its initial posi- the velocity of 2 kg block was 1 ms–1. Coefficient
tion. The other block M after the collision moves of friction for both the blocks with table is 0.2 and
0.5 m up and comes to rest. Calcualte the coefficient collision is elastic. Find the final separation between
of restitution between the blocks and the mass of the the blocks.
block M.
[Take sin q tan q = 0.05 and g = 10 ms–2]
19. A small ball of mass m = 2 kg is connected to a fixed
point O by an inextensible
cord of length L = 2.4 m. The 24. An astronaut is inside a satellite. There is effectively
ball is resting on a smooth no gravity inside the satellite (More about it in the
horizontal table at point A at chapter of Gravitation). The astronaut has a mass of
a distance of 1 m from point 50 kg and is at rest inside the satellite. He throws an
O. It is imparted a velocity objects of mass 5 kg towards a wall of the satellite.
u in a direction perpendicu- He finds that the object moves away at a velocity of
lar to the line OA. The string 12 ms–1 relative to him. The object collides with the
gets taut when the ball reaches point A¢. front wall and rebounds. How long after throwing
(i) Find maximum allowable u if the impulse the object the astronaut can catch the object? Assume
applied by the string on the support at O is not that collision of the object with the satellite wall is
to exceed 3 Ns. elastic and that the astronaut does not meet a collision
(ii) Find loss in kinetic energy as the strings gets till the object is back to him. The object was thrown
taut, if the ball is initially given a velocity at a distance of 8 m from the wall.
equal to maximum allowable value calculated
in part (i).
20. Two identical smooth balls are projected towards
each other from two points A and B on horizon-
tal ground with same speed of projection. The
angle of projection in each case is 30°and distance
AB = 100 m. The two balls collide in mid air and
return to their respective projection points. Find the 25. A block of mass m is slid on a smooth horizontal
speed of projection of either ball if coefficient of table with a velocity u towards a wedge of mass
restitution for the collision is e = 0.7. M = h ◊ m. The wedge has a smooth curved surface
21. A rocket is designed to move at constant acceleration on which the block climbs without any abrupt change
a in space. It shoots out exhaust gases at constant in its velocity when it hits the wedge. Height of the
speed u relative to itself. At time t = 0, mass of the wedge is h. Find minimum value of u so that the
rocket is M0. Find its mass at a later time t. block can climb to the top of the wedge. Remember,
the wedge is free to move!
22. An empty railway wagon of mass m0 starts moving
to the right due to a constant horizontal force F.
The moment force
begins to act, sand
begins falling on
the wagon from
a fixed hopper.
Rate of loading is

2.56
Answers Sheet
Your Turn
1. the cycle 2. Truck 3. (i) DP = mu sin q (Ø) (ii) D P = 2 mu sin q (Ø)
5 5
4. (i) 10 N (ii) 1.5 × 10 J 5. (6.88 ± 0.37) N 6. 20 N
2mv2 ____
7. ​ _____   

​ 8. r Av2 ; force becomes 4 times   ​ + mgt
9. m ​÷2gH 
p R
10. 250 N 11. Larger impact time means smaller average force in all three cases
____
12. 20 Ns 13. 10000 Ns 14. 20 Ns   ​ (Ø)
15. M ​÷2gL 
2mu ​_› ​_›
16. ​ ____ ​  
  17. (2  + 7  ) ms–1 18. m ​u  ​  + m ​g  ​  ◊ t 19. 3520 J
3
mu mu
20. (i) ___
​   ​   (ii) Energy released is greater than 1/2(mu2)(1 + m/M) 21. ​ ______
    ​
M M+m

÷  ÷ 
___ ___
mu k k ___
22. ​ ________     ​ 23. vA = ​ ___
​     ​ ​   ◊ x ; vB = 2 ​ ___
​     ​ ​   ◊ x 24. ​÷ 21 ​ ms–1
2 (M + m) 6m 6m
​___›
25. ​BG​   
mv
26. ​ ______
M+m
    ​
M+m
( 
27. With a velocity ​ ​ ______
m    )
​  ​ u in forward direction.

28. No change ​ 
M+m
mu
29. – ______
   ​
M+m
mu
30. (i) ​ ______
  
1 mM
 ​ (ii) – __
2 M+m
​  ( 
​   ​ ​  ______
   ​  ​ u2 )
mu 1 1 2L
31. ​ ______
    ​ 32. (i) KA = 0, KB = __
​   ​  mu2 (ii) __
​   ​  mu2 33. ​ ___
u ​ 
M+m 2 4
1__ 5m
34. ​ ___   ​   35. (i) 4 ms–1 (ii) 2.5 ms–1 36. M > ___
​   ​    37. 0.23 d
÷   
​ 2 ​ 4
x _____ 3
38. ​ __  ​ 39. m2 = ​÷m  1 m3 ​   40. ​ __ ​ 
2 5
41. (i) 2  – 3  (ii) 2  (iii) 2  – 1.5 
42. vB = u in a direction opposite to original direction of motion
vA = 2u in a direction at right angle to the original line of motion.
1 1
43. 45° 44. e = 0.453 45. ​ __ ​   46. ​ __
e ​
3
u
47. ​ ___
__  ​ making 45° with x-axis 48. 2.5 kgs–1 49. 3.75 kgs–1
÷   
​ 2 ​
50. 0 51. 2 ln2

Worksheet 1
1. (b) 2. (c) 3. (b) 4. (b) 5. (b) 6. (b) 7. (b) 8. (a) 9. (b)
10. (b) 11. (a) 12. (b) 13. (a) 14. (a) 15. (c) 16. (a) 17. (b) 18. (b)
19. (c) 20. (d) 21. (d) 22. (d) 23. (b) 24. (d) 25. (a) 26. (a) 27. (a)
28. (c) 29. (b) 30. (b) 31. (d) 32. (a) 33. (c) 34. (b) 35. (c) 36. (a)
37. (b) 38. (a) 39. (d) 40. (a) 41. (a) 42. (b) 43. (c) 44. (d) 45. (c)
46. (a) 47. (c) 48. (c) 49. (a) 50. (b) 51. (c) 52. (b) 53. (b) 54. (a)
55. (c) 56. (c) 57. (c) 58. (c) 59. (b) 60. (c) 61. (d) 62. (c) 63. (b)
64. (d) 65. (d) 66. (c) 67. (d) 68. (c) 69. (c)

2.57
Worksheet 2
1. (a,d) 2. (b,c) 3. (a,d) 4. (a,b,c,d) 5. (a,b,d) 6. (b) 7. (a,c) 8. (a,b,c,d)
9. (a,c,d) 10. (b) 11. (b,d) 12. (c) 13. (a,b,c) 14. (b,c) 15. (b) 16. (a,b,c)
17. (a,c,d) 18. (b,c) 19. (a,c) 20. (c) 21. (a,c,d) 22. (a,b,d) 23. (a,b,c) 24. (a,c,d)

Worksheet 3


m
(  )
1. 2 ​ __
​   ​   ​ v2
d
2. 1884 N 3. (i)
1
2
3
​ __  ​ mu (Æ) (ii) __
​   ​  mu (iii) ___
4
mu
​   ​ 
4

( 
__ __ __
4. vA = ___
÷   
​ 3 ​
​   ​ u, vB = ___
4
    ​_›
​÷7 ​
​   ​  u ; ​J  ​  = ​ ___
4
  
​÷3 ​
) 3
​   ​ mu  ​  + (​ __ ​  mu) 
8 8
3
5. ​ __ ​  mv cot q
2

÷  ÷ 
___ ______
10 g mv Mmv
6. ​ ___
​   ​ ​  ms–1 7. md ​ ______
​      ​ ​   8. v1 = u – ​ ______
    ​ ; v = u + ________
​      ​
3 2h k M M+m 2 (M + m)2

9. 4
25H __
10. ​ ____
 ​ 
H
 , ​   ​  
Mmu2
11. (i) ​ ________
   ( 
M + m ___
 ​ (ii) ​ ​ ______
m    )
   
​  ​ ​÷5gl ​

 (
9 9 2 (M + m)

) (  )
1 u2 sin2 q 2u sin q _____ 1
12. ​ __________
​  2      ​  ​ 13. 24.1 ms–1, 2.55 ms–1 14. (i) ​ _______   ​ (ii) _______
  ​  g     ​  ​ (iii) tan–1 (en tan q)
​ ​ ​     

u  sin 2q
_______ g (1 – e) 1–e
​     
​ 
–1
d ◊ g

) ÷ 
___

2mu
15. (i) _______
​ 
M + 2m
   ​ 
mu
 ​ (ii) _______
  
M + 2m M + m
​ 
mu
 ​ + ______
   ​ 16. 33.3% ( 
​ 
2mu
17. ​ _______
m + 2M
  
M
 ​  ​ ​ __
​   ​ ​  
K

18. 0.84, 15.01 kg 19. (i) 1.65 ms–1 (ii) 2.25 J 20. 37.5 ms–1
at
__ Ft Fm0
21. M = M0 ​e– ​ 
​ u ​ ​ 22. v = _______
​      ​ ; a = _________
​      ​ 23. 5 cm
m0 + bt (m0 + bt)2

÷  ( 
__________
24. 1.6 s 25. ​ 2gh ​ 1 +  
h
1
__
​   ​  ​  )

2.58
Chapter  3

Miscellaneous problems
on chapter 1 and 2

MATCH THE COLUMN Column I Column II

|  | |  |
_
​ ​_
1. A particle of mass m2 is at rest. Another particle of (a) M1 >> M2, collision is head-on (p) ​ ​​v › ​​ 2 ​  ​ > ​ ​​v › ​​ 1 ​  ​
f f

|  | |  |
mass m1 hits it. Collision is one‑dimensional elastic. ​_› ​_›
(b) M1 << M2, collision is head-on (q) ​ ​​p   ​​ 2 ​  ​ > ​ ​​p  ​​ 1 ​  ​
Suffix ‘f ’ represents final and ‘i’ represents initial in f f

following. (c) M1 = M2, collision is head-on (r) ​k​2 ​ > ​k​1 ​


f f

(d) M1 = M2, collision is oblique (s) ​k​2 ​ < ​k​1 ​


f f

3. A body initially moving towards right explodes into


two pieces, 1 and 2. Direction of motion of the pieces
is shown in column I and possible mass ratios are
shown in column II.
Column I Column II
Column I Column II
(a) (p) m1 > m2
(a) the curve in the figure, which (p) A
is the graph of the velocity ratio
v1f /v1i versus the mass ratio
m1/m2
(b) the value of the intercept of (q) C or D (b) (q) m1 = m2
the graph of v1f  /v1i versus the
mass ratio m1/m2 with the verti‑
cal axis is
(c) (r) m1 < m2
(c) the maximum value taken by (r) B
the graph of v2f  /vli versus m1/m2 , v1 < v2
is
(d) the curve that is the graph of (s) 0 (d) (s) Impossible for any
the velocity ratio v2f  /vli versus masses
m1/m2 is (t) – 1
(u) 2

2. A smooth disk (mass M1) moving on a smooth floor


undergoes elastic collision with another smooth 4. Two blocks A and B of masses m and 2m respectively
disk (mass M2) at rest. Quantities in column II are connected by
refer to possible values after collision. (v Æ speed, a massless spring
p Æ momentum, k Æ kinetic energy) of spring constant
k. This system
lies over a smooth horizontal surface. At t = 0, the
3.2  Mechanics II

block A has velocity u towards the right, as shown, (c) (r)


while the speed of block B is zero, and the length of
spring is equal to its natural length at that instant.

Column I Column II
(a) The velocity of block (p) can never be zero
A (d) (s)
(b) The velocity of block (q) may be zero at certain
B instants of time
(c) The kinetic energy (r) is minimum at maximum
of the system of two compression of spring
block (t)
(d) The potential energy of (s) is maximum at maximum
the spring extension of spring

5. A truck of mass M crashes into a tempo of mass 7. An initially stationary box on a frictionless floor
m < M and the two masses stick together. For each explodes into two pieces, piece A with mass mA and
pair of quantities below, state the relationship. piece B with mass mB. The two pieces then move
across the floor along x-axis. Possible graph of posi‑
Column I Column II tion versus time for the two pieces are given.
Quantity 1 Quantity 2
(a) Magnitude of Magnitude of (p) Quantity 1 >
force exerted by force exerted by Quantity 2
truck on tempo tempo on truck

(b) Magnitude of Magnitude of (q) Quantity 1 <


acceleration of acceleration of Quantity 2
truck during tempo during
collision collision
(c) Total kinetic Total kinetic (r) Quantity 1 =
energy of the energy of the Quantity 2
two-vehicle two-vehicle
system before system after
collision collision
(d) Kinetic energy of Kinetic energy (s) I m p o s s i b l e
the tempo before of the tempo to determine
collision after collision from infor‑
mation given Column I Column II
6. Assume that two bodies collide head on. The (a) I (p) Explosion could happen due
graph of their velocities with time are shown in to internal reasons
column I. match them with appropriate situation in (b) II (q) mA = mB
column II. (c) III (r) mA > mB

Column I Column II (d) IV (s) mA < mB


(e) V (t) Explosion must have hap‑
(a) (p)
pened due to an impulsive
external force
(f) VI

8. In each situation of column I, a system involving two


(b) (q) bodies is given. All strings and pulleys are light and
friction is absent everywhere. Initially, each body is
at rest. Then match the statements in column I with
the corresponding results in column II.
Miscellaneous Problems on Chapter 1 and 2  3.3 

Column I Column II Column I Column II


(a) The block plus wedge system is (p) Shifts towards (a) A and B are connected by (p) v
cm =
​ __v  ​ before and
placed over a smooth horizon‑ right an ideal spring (which is 3
tal surface. After the system is relaxed) after all collisions
released from rest, the centre of (b) A and B are connected by (q) There will be more
mass of the system
an ideal string than one collision.
(c) A and B are connected by (r) A and B always have
a light rigid rod equal and opposite
momentum after all
collisions.
(b) The string connecting both the (q) Shifts (d) B and C are connected (s) Final mechanical
blocks of mass m is horizon‑ downwards
by an ideal string. gap energy after all
tal. The left block is placed
between A and B is collisions is less than
over smooth horizontal table,
large. initial.
as shown. After the two-block
system is released from rest, the
centre of mass of the system
PASSAGE-BASED PROBLEMS
Passage 1
Two blocks are far apart and are approaching each other with
velocities as shown in figure. The coefficient of friction for
(c) The block and monkey have the (r) Shifts upwards both the blocks is m = 0.2.
same mass. The monkey starts
climbing up the rope. After the
monkey starts climbing up, the
centre of mass of monkey +
block system
1. Linear momentum of the system is
(a) conserved all the time
(b) never conserved
(c) is conserved upto 5 seconds
(d) none of these
2. How much distance will the centre of mass travel
(d) Both blocks of mass m are ini‑ (s) Does not shift before coming permanently to rest
tially at rest. The left block is
given initial velocity u down‑ (a) 25 m (b) 37.5 m
wards. Then, the centre of (c) 42.5 m (d) 50 m
mass of the two-block system
afterwards Passage 2
A boy of mass 10 kg is standing on a 4 kg block, as
shown. The whole system
is on smooth level ground.
Suddenly, the boy jumps to
the right with a velocity of
7 ms–1 relative to the 4 kg
9. Two spheres A and B are moving along a straight line block. Interaction time of the boy and the 4 kg block during
with same velocity v. A third sphere C is moving in the event of jump is very small and the displacement of the
the opposite direction on the same surface, with the bock can be neglected during that period.
same speed v. All spheres are identical and elastic. 3. For the system, comprising the boy and the 4 kg
vcm represents block:
velocity of (a) momentum is not coserved during the jump due
COM. Match to presence of the spring force.
the entries in column I to those in column II.
3.4  Mechanics II

(b) both blocks have finite velocities immediately Passage 4


after the boy jumps
A block of mass M with a semi-circular track of radius R
(c) speed of 4 kg block immediately after the boy rests on a horizontal frictionless surface. A uniform cylinder
jumps is 5 ms–1 of radius ‘r’ and mass ‘m’ is released from rest at the top
(d) none of the above point A. The cylinder slips in the semi-circular frictionless
4. The maximum velocity of 1 kg block (in ms–1) in the track.
subsequent motion is
(a) 0.8 (b) 0.4
(c) 0.1 (d) 1.0

Passage 3
A horizontal frictionless rod is threaded through a bead
of mass m. The rod is mounted horizontally inside a cart
of mass M, as shown. The length of the cart is L and the
radius of the bead, r, is very small in comparison to L (r 8. Which of the following is true?
<< L). Initially, the bead is at the right edge of the cart. The (a) Momentum of the system of the block and
cart is given a sharp blow (at t = 0) towards the right and the cylinder is conserved after the cylinder is
as a result, it begins to move with velocity u0. When the released.
bead collides with the cart’s walls, the collisions are always
(b) Momentum of the system of the block and the
completely elastic. (Given: m > M).
cylinder is equal to the initial momentum only
when the cylinder reaches the bottom of the
track.
(c) The cylinder will never get back to the topmost
position in the track from where it is released.
(d) Motion of the block is not oscillatory.
9. How fast is the block moving when the cylinder
reaches the bottom of the track?

5. What is the velocity of the cart after the first collision,


in the centre of mass frame?

M [ 
m 2gM (R – r) 1/2
(a) ​ __ ​ ​​  ​ __________
M+m
  
    ]
​  ​​ ​


– mv0
(a) ​ ______ 
m+M

 ​
Mv0
(b) ​ ______
m+M
   ​
M [ 
m 2gM (R – r) 1/2
(b) ​ __ ​ ​​  ​ __________
  
2M + m
   ]
​  ​​ ​


M–m
(c) ​ ______ 
M+m 0
 ​ v
2M
(d) ​ ______
  
m+M 0
 ​ v
M [ 
m gM (R – r) 1/2
(c) ​ __ ​ ​​  ​ _________
M+m
  
  ]
​  ​​ ​

6. The first collision takes place at time t1 and the


second collision takes place at t = t2. Find t2 – t1.

M [ 
2m 2gM (R – r) 1/2
(d) ​ ___ ​ ​​  ​ __________
M+m
  
   ​  ​​ ​]
2L L Passage 5
(a) ​ ___
v0 ​   (b) ​ __
v0  ​ 
A shell is flying with a velocity u = 500 ms–1. It bursts
L L
(c) ​ ___  ​   (d) ​ ___  ​  into three identical fragments and the kinetic energy of the
2v0 3v0
system increases k times. One of the fragments continues
7. What is the distance the cart travels from t = 0 till to move along the original line of motion with speed v1.
t = t2 Answer the following questions for k = 1.5.
10. For v1 to be maximum
2ML mL
(a) ​ ______ 

 ​ (b) ​ ________
    ​ (a) the other two particles must move in opposite
m+M 2 (m + M)
directions perpendicular to the original line of
2m ML motion
(c) ​ ______
    ​ L (d) ​ ________
    ​
m+M 2 (m + M) (b) the other two particles must move in directions
making equal angles with the original line of
motion
Miscellaneous Problems on Chapter 1 and 2  3.5 

(c) one of the particles should come to rest 14. u and v are related as
(d) None of the above (M + m) v (M + m) v
(a) u = ​ ________  ​ 

(b) u = ​ ___________
    ​
  
11. Maximum possible value of v1 is m cos b m cos (a – b)
(a) 1000 ms–1 (b) 500 ms–1 (M + m) v mv
(c) 1200 ms–1 (d) 800 ms–1 (c) u = ​ ________   (d) u = ​ ____________
 ​       ​
m cos a (M + m) cos b

Passage 6 15. The velocity of wagon (v) is

÷  [ 
A freight car is moving on a smooth horizontal track with _________________________
no external force acting in horizontal direction. Rain is fall‑
ing with a velocity u ms–1 at an
angle q with the vertical. Rain

2mM gl cos b
(a) v = ​ _______
​ 
M+m

– cos a) cos2b
 ​ ​ ​ ________________
        ​  ​
  
M + m sin2 b ]
÷  [ 
_________________________

]
drops are collected in the car
2m2 gl (cos b – cos a) cos2b
at the rate of m kgs–1. If initial  ​ (b) v = ​ ______
​       ​ ​ ​ _________________
          ​  ​ ​
  
mass of the car is m0 and veloc‑ M+m M + m sin2b
ity v0 then

÷  [  ]
_________________________
12. The instantaneous acceleration (a) of the car is related (cos b – cos a) cos b
2mM gl ________________
to instantaneous velocity v of the car as (c) v = ​ _______
​    ​ ​ ​          ​  ​ ​
  
M+m M + m sin b
(a) (m0 + m t) a = – (u sin q + v) m

÷  [ 
_________________________


(b) (m0) a = – (u cos q + v) m
(c) (m0 + m t) a = – (u cos q + v) m

2m2 gl (cos b
(d) v = ​ ______
​ 
M+m


– cos a) cos 2b
 ​ ​ ​ _________________
      
  
M + m sin 2b
 ​  ​ ​
]
(d) (m0) a = – (u cos q + v) m 16. The velocity of wagon when velocity of the bob is
13. Velocity of the car as a function of time is horizontal relative to the wagon is

÷ 
____

(  )
​ 
m t
(a) v = u cos q ​ _______
  
m0 + m t
​ 
m0v0
 ​  ​ + ________  
(m0 + m t)
 ​
a gl
(a) v = 2 m sin ​ __ ​  ​ ____
2 m M
​     
​ ​ 

÷ 
_________

(  )
m t m0v0 a gl
(b) v = u cos q ​ ___
​ m   ​  ​ + ________
​     ​ (b) v = m sin ​ __ ​  ​ _________
​      
 ​ ​
0 (m0 + m t) 2 (M + m) M

÷ 
_________

(  )
m t m0v0  a gl
(c) v = – u sin q ​ _______
​      ​  ​ + ________
​     ​ (c) v = 2 m sin​ __ ​  ​ _________
​      ​ ​

m0 + m t (m0 + m t) 2 (M + ) M

÷ 
_________

(  )
m t
(d) v = u sin q ​ ___
0
m0v0
​ m   ​  ​ + ________
​    
(m0 + m t)
 ​
a
​ 
2 (M + m) m
gl
(d) v = m sin ​ __ ​  ​ _________
    ​ ​

Passage 7 Passage 8
A wagon of mass M can move without friction along A cannon with shots has total mass M0. It is kept on a rough
horizontal rails. A simple pendulum consisting of a bob of horizontal surface. The coefficient of friction between the
mass m is suspended from the ceiling by a string of length cannon and the horizontal surface is m. The cannon begins
l. At the initial moment, the wagon and pendulum are at to fire the shots at a uniform
rest and the string is deflected frequency with a velocity u
through an angle a from the relative to it. All the shots
vertical. The system is released are fired horizontally.
from this position. 17. If mass of each shot is m, the smallest frequency of
The velocity of wagon, firing, which will cause the cannon to accelerate just
when the string forms an angle as the firing begins is
b (b < a) with vertical is v and m M0g 2m M0g
u is the speed of pendulum in (a) ​ _____ ​   (b) ​ ______
 ​ 

(mu) (mu)
the frame fixed to the wagon.
m M0g
(c) ​ _____
 ​  
  (d) difficult to decide
(u)
3.6  Mechanics II

÷  ÷ 
_____ ____
18. Which of the following is true? 2M2g M2g
(a) If the cannon does not recoil initially, it will not (a) ​ _____
​  2    
​ ​  
  (b) ​ ____
​  2  ​ ​ 

ru r  u
move till the end of shots.
2Mg Mg
(b) If the cannon does not recoil initially, it may (c) ​ ____   
​ (d) ​ ___ ​ 
move a little later as the shots are being fired. r u r u
(c) If the barrel of the gun is inclined so as to 23. The minimum power needed for take-off is
have a large range, the cannon will experience (Mg)3/2 (Mg)3/2
a greater thrust force. (a) ​ ______
___ ​  (b) ​ _______  ​
____ 
d ​÷p r ​
       
d ​÷2p r ​
(d) None of the above.
19. The velocity of the cannon when it possesses a total (Mg)3/2 (Mg)3/2
(c) ​ _______  ​
___  (d) ​ ________  ​ 
____ 
mass M (with the remaining shots), after time t from 2d ​÷p r ​
       
2d ​÷2p r ​
starting is
(a) ut ln (M0/M) – m gt (b) u ln (M0/M) – m gt Passage 11
(c) ut ln (M0/M) – m g (d) u ln (M/ 0) – m gt A small block A slides down a smooth hill of height H and
lands on a plank B, as shown. Mass of A is m. A quickly
Passage 9
stops moving relative to B and
Consider three identical discs, A, B and C, on a smooth stays on it. The plank B slides
horizontal floor (xy-plane). B and C are at rest. A is imparted on the horizontal surface and
a velocity u in positive x-direction. It first hits B and then the friction force acting on it
goes on to hit C. Finally, A was found to be travelling is found to be directly pro‑
u
along x-direction with velocity ​ __ ​ . Assume all collisions to portional to its speed. The
2 proportionality constant is k.
be elastic.
That is, friction is given by

f = – kv
24. Impulse of friction that acted on (A + B) combined,
if block B travels a distance L is
___ L ___ H2
  ​  ◊ m ◊ ​ ___
(a) ​÷2g    ​     ​  ◊ m ◊ ​ ___
(b) ​÷2g   ​ 
H2 L
20. Final velocity of B is
u u (c) 2 kL (d) kL
(a) ​ __ ​  (b) – __ ​   ​ 
2 2 25. Minimum value of H needed for B to travel through
u u
(c) ​ __ ​  ( – ) (d) ​ __ ​  ( – ) a distance L is
2 4
k2L2 k2L2
21. Final velocity of C is (a) ​ _____2 
 ​   (b) ​ ____2 ​ 
2gm gm
u u
(a) ​ __ ​  ( + ) (b) ​ __ ​  (–  + ) 2k2L2
4 4 (c) ​ _____ ​   (d) None
gm2
u u
(c) ​ __ ​  (d) ​ __ ​ 
2 4
Passage 12
Passage 10 Two blocks, A and B, of masses m1 and m2 respectively, are
A model of a helicopter has mass M. Its fan has diameter d. placed on a frictionless horizontal floor. They are connected
When switched on, the fan collects the motionless air above by a light cord, as shown. Block A is imparted a veloc‑
it and blow it downwards. Assume that density of air, above ity u towards right. During the very short time interaction
and below the fan, is the same and equal to r. when the thread is gaining tension, it gets slightly extended
and after acquiring a maximum extension, it again goes
22. Assuming that the speed imparted to air particles by
back to relaxed state. The short period when the thread is
the fan is u; volume of air that the fan must push
getting stretched is called
down in 1 second, so as to just lift the helicopter is
deformation period and
Miscellaneous Problems on Chapter 1 and 2  3.7 

the period in which the thread returns to its relaxed state is block of mass m = 1 kg is held at rest at the top of the
called ‘restitution period’. Let us define a number h as inclined surface of the wedge. System is released from this
position. Assume no friction and take g = 10 ms–2.
Impulse of tension during restitution period
  h =  ​ ______________________________________
       ​ 30. The ratio of horizontal and vertical components of
     
Impulse of tension during deformation period
velocity of the block of at any time is
26. Energy absorbed by the thread during its deformation __ __
____   
4​÷3 ​   
2​÷3 ​
____
period is (a) ​   ​     (b) ​   ​   
5 5

(  )
__ __
1 m1m2 1
(a) ​ __ ​  ​ ​ _______   ​  ​ u2
  (b) ​ __ ​  m1u2 ___  
​÷3 ​   
4​÷3 ​
____
2 m1 + m2 4 (c) ​   ​   (d) ​   ​ 
5 25

1 m1m2
( 
(c) ​ __ ​  ​ _______
​ 
4 m1 + m2
  )
 ​  ​ u2

1
(d) ​ __ ​  m1u2
2
31. The vertical component of velocity of the block after
it has come down by a distance y (< h) in vertical
direction is
27. Final velocity of B is

÷  ÷ 
____ _____
10
___ 100
m 1(1 – h) m 1 (1 + h)
(a) ​ ​   ​  y   ​ (b) ​ ____
​   ​  y ​ 
(a) ​ ________  ​ 
 u (b) ​ _________  ​
  u 17 17
m1 + m2 m1 + m2  

÷ 
___

÷ 
___
y 5y
(c) ​ ___
​    ​ ​    (d) ​ ​ ___  ​ ​ 
m  (1 – h) 17 17
(c) ​ _________ ​
2
  u (d) None
m1 + m2   32. Time taken by the block to reach the bottom of the
incline
___
Passage 13 1__ ÷    
​ 17 ​
(a) ​ ___   ​ s (b) ​ ____ ​ s


There are two identical smooth slides A and B, each of   
​÷5 ​ 5
height h and mass M. They are placed on a smooth floor, as ___
   
​÷8.5 ​ 1
shown. A small disc C of mass m (<< M) is released from (c) ​ ____  ​ 
 s (d) ​ __ ​  s
5 5
the top of slide A. The disc slides down, then slides up
the other slide, reaches Passage 15
a maximum height and
then slides down. The A room has a square floor of side length a. A particle is
sequence continues till projected from the centre of the floor (O), at an angle q with
all interaction between horizontal. All its collisions with the walls and the roof are
the slides and the disc elastic. The particle returns to its point of projection after
stops. time T.

28. Maximum height attained by the disc on slide B is

(  m 2
(a) ​​ ______
​    ​  
m+m )
  ​​ ​ h (  M 2
(b) ​​ ______
​     
m+m )
​   ​​ ​ h

Mm 2M
(c) ​ ________
    ​ h (d) ​ ______
   ​ ◊ h
(m + m)2 m+m

29. Final speed acquired by each slide is nearly

÷  ÷ 
____ ____
mgh mgh
(a) ​ ____
​     

​ ​   (b) 2​ ____
​     

​ ​ 
M M

÷​  M + m   ​ ​ ÷​  M + m   ​ ​


______ ______ 33. The speed of projection, if the particle collided with
2mgh mgh
(c) ​ ______   (d) ​ ______   two parallel walls and the roof is
a 2a
(a) ​ ______
   ​   (b) ​ ______
   ​ 
Passage 14 T cos q T cos q
a a
A triangular wedge of mass (c) ​ ______
   ​   (d) ​ ______________
     ​
___________
M = 4 kg is on a smooth T sin q   q – sin
T ​÷cos2 2
  q ​
horizontal table. Its angle of
inclination is q = 30° and its 34. The speed of projection, if the particle just makes
height is h = 0.5 m. A small one collision with a wall, is
3.8  Mechanics II

Tg 2Tg 1 _____ 3 _____


(a) ​ ____  ​ 
(b) ​ ____  ​  (a) ​ __ ​  ​÷6m gd 
  ​
  (b) ​ __ ​  ​÷6 m gd 
  ​

sin q sin q 2 2
5 _____ ____

a
(c) ​ ______
   ​ 
(d) None (c) ​ __ ​  ​÷6m gd 
  ​
    ​ 
(d) ​÷m gd 
u cos q 2
39. Assuming u = 2u0, the distance measured from point
Passage 16 P at which the mass m falls on the table after collid‑
A 3 kg shell moving in space explodes into three fragments ing with B is
___ ___
of equal masses. After explosion, the fragments move apart    
(a) 6 d​÷3m ​    
(b) 4 d ​÷3m ​
but interact through mutual attraction with each other. At
(c) 4 dm (d) dm
some instant after explosion, velocities of the three fragments
are
Passage 18
(30  + 20  ) ms–1 , – 20   ms–1 and (50  – 50  ) ms–1.
Two identical discs of mass M are lying on a smooth
35. Find the KE of the shell before explosion
horizontal surface. A small ball of mass M strikes one of the
(a) 150 J (b) 450 J discs, while moving along a line, making an angle q = 37°
(c) 750 J (d) 2250 J with the line joining the centres of the two discs. Velocity
36. Find the minimum energy released in explosion of the ball just before collision is v = 10 ms–1.
(a) 2600 J (b) 2150 J
(c) 1600 J (d) 450 J
37. Find the least KE of the system that is possible at a
time after explosion
(a) 750 J (b) 400 J
(c) 3500 J (d) 1600 J
Coefficient of restitution for collision between the ball
Passage 17 1
and disc A is ​ __  ​ and the coefficient of restitution for collision
2
A block A of mass 2m is placed on another block B of between A and B is 1.
mass 4m, which in turn is placed on a fixed table. Length 40. Relative velocity of separation along the line of
of each block is 4d and impact for the ball and disc A after collision is
they are placed as shown.
(a) 2 ms–1 (b) 8 ms–1
Coefficient of friction –1
between B and the table (c) 4 ms (d) zero
is m. There is no friction 41. Speed of ball after collision is
___ ___
between A and B. A small (a) ​÷ 10 ​ ms–1    ms–1
(b) 3​÷10 ​
objects of mass m moving ___
(c) 4 ms–1    ms–1
(d) 2​÷10 ​
horizontally along a line passing through the COM of B,
and perpendicular to its vertical face, with speed u collides 42. Speed of disc B after collision is
elastically with B at a height d above the table. (a) 6 ms–1 (b) 3 ms–1
38. Minimum value of u (say u0) so as to make block A (c) 4 ms–1 (d) None.
topple is
Answers Sheet
Match the Columns
1. (a) r;  (b) t;  (c) u;  (d) p 2. (a) p, s;  (b) q, s;  (c) p, q, r;  (d) p, q, r, s
3. (a) s;  (b) s;  (c) p;  (d) p, q, r 4. (a) q;  (b) q;  (c) p, r;  (d) q, s
5. (a) r;  (b) q;  (c) p;  (d) s 6. (a) s;  (b) r;  (c) q;  (d) p
7. (a) t;  (b) p, q;  (c) t;  (d) p, s;  (e) p, r;  (f) t 8. (a) q;  (b) p, q;  (c) r;  (d) s
9. (a) p, r, s;  (b) p, q, r;  (c) p;  (d) p, q, s

Passage-based Problems
1. (c) 2. (b) 3. (c) 4. (a) 5. (a) 6. (b) 7. (c) 8. (b) 9. (a)
10. (d) 11. (a) 12. (a) 13. (c) 14. (a) 15. (b) 16. (c) 17. (a) 18. (b)
19. (b) 20. (c) 21. (c) 22. (d) 23. (a) 24. (d) 25. (a) 26. (a) 27. (b)
28. (b) 29. (a) 30. (a) 31. (b) 32. (c) 33. (b) 34. (c) 35. (c) 36. (a)
37. (a) 38. (c) 39. (a) 40. (c) 41. (d) 42. (a)

3.9
Chapter  4

Torque and Equilibrium


“Living things have no inertia, and tend to no equilibrium.’’
–Thomas Henry Huxley

1.  Introduction C travel different distances in equal time intervals. The


pencil is said to be rotating about point B, or about an axis
We solved some problems on equilibrium of bodies in the passing through point B and perpendicular to the plane of
chapter on Newton’s laws of motion. Knowledge of torque the figure.
will let us solve diverse kinds of equilibrium problems, Similarly, think of the spinning of the Earth. All particles
which were beyond our range till now. are moving in circles. The centres of all such circles lie on
Moreover, our understanding of torque will help us in the line passing through the poles of the Earth. This line is
understanding the dynamics of rotational motion of rigid the rotation axis of the Earth. The Earth is rotating about its
bodies in the chapters to follow. We will ultimately realise axis.
that torque is rotational analogue of force.

2.  WHAT IS ROTATION?


When a body moves such that all its particles undergo iden-
tical motion, we say it is
translating. Think of a
pencil being moved along
a circle, as shown in fig-
ure. Its particles, A, B and
C, always have the same
speed and travel equal
distances in equal inter-
Spinning Earth
vals of time. The pencil is
Point A goes in Circle with centre at C1
performing translational point B moves in circle with centre at C2
motion.
The same pencil now Following diagram shows the effect of rotating a book
moves with its end B by 90° about three perpendicular axes. The diagram helps
fixed. Tip A goes in a in understanding the meaning of rotation of a body about a
circle. Its centre C also given axis.
goes in a circle. In fact,
all points in the pencil are 3. TORQUE
moving in circles, hav-
ing common centre at B. Torque is a physical quantity that measures the turning effect
All the points have the of a force.
same angular speed (w) When you switch on a fan, it begins to rotate with increas-
but different linear speeds ing angular speed. The fan is experiencing a torque (applied
(v = wr). Points A and by electric motor). Since there is no acceleration of the COM
of the fan, we know that the sum of all force on it is zero.
4.2  Mechanics II

This implies that forces may have zero resultant but non-zero Let us talk a little about the magnitude of the torque.
turning effect.
t = r F sin q
= r F^ ...(2)
_
​›
where F^ = F sin q
​_›
is a component of ​
F  ​  in a direction per-
pendicular to ​r ​ . 

F cos q

P q
F
r
F sin q
O

We can also interpret the magnitude of torque as


t = F (r sin q)
= F r^ ...(3)

Where r^ = r sin q
​_›
is the perpendicular distance of line of
action of force ​F  ​  from the point O.
R
Q
P q
3.1  Torque of a force about a point q
F
​_› r sin q
A force ​F  ​  is acting at point P. Torque of this force about a r
point O is defined as
​_› O
​_› ​_›
​t    ​ = ​r ​   × ​F  ​   ...(1)
​_›
​_› It is easy to see that torque of force ​F  ​  about O will not
Where ​r ​   = position vector of point of application relative
to O change if the point of application of the force is moved to
a point like Q or R that lies on its line of action.
In the figure shown, direction of t is perpendicular to the
plane of the figure. [This is Obviously, unit of torque is N-m. We should avoid writing
the direction in which your it as Joule.
thumb points when you curl _
​› 3.2  Torque of a force about an axis
your ​_› right-hand fingers from ​r  ​ 
to ​F   ​]. For ease of discussion, Consider an L-shaped body OAB that is free to rotate about
let us call the direction going point O. Arm OA is along y-axis and has length a. ​ › Arm AB
_
into the plane of the figure is parallel to z-axis and has length b. A force ​F  ​  = F  is
as z-axis. Our force has a torque about O pointing along applied at tip B. Torque of this force about O is
_ ​_›
z-axis. what does this mean? If​_› there was a body, free to
​›
t = ​r ​   × ​F  ​  = (a  + b  ) × (F  )
rotate about point O and ​_›force ​F   ​ acted at some point P in
the body, the torque of ​F   ​ about O would try to spin the = bF  – aF 
body about z-axis. The direction of rotation is in _​the direc-
› z
tion
​_› of curling fingers of your right hand (from ​r ​   towards
​F   ​). With reference
​_› to the figure shown, we can say that the
torque of ​F   ​ tries to produce a clockwise rotation about point
F
O. So we either say that torque is directed into the plane
r B
(along z-direction), or we simply say that torque of the force O
x b
about O is clockwise. _
​› a
Now, we know that torque of force ​F   ​ about point O will
try to​_ rotate_​ ›a body about an axis that is perpendicular to A
› y
both ​r ​   and ​F   ​.
Torque and Equilibrium  4.3

​_› ​_› ​_›


Actually,
​_›
this​_›torque is along an axis that is perpendicular (i) ​t    ​ = ​r ​   × ​F  ​  = (2 – ) × (2 + 3 )
to both ​r ​   and ​F  ​.  But in our co-ordinate system, we have = (3 – 2 + 6 ) N-m
two components of torque.
(ii) tz = 6 N-m
t y = bF  and  t z = – aF
It implies that the force will try to rotate the body about
Example 2   ABCD is a rectangular plate having sides of
y-axis as well as z-axis. However, t x = 0 means that the
length 4 m and 3 m. Forces F1, F2, F3 and F4 are applied
force will have no rotational effect about x-axis. Think of the
on the plate as shown. Each force has magnitude equal to
above-described situation practically by having an L-shaped
20 N and is in the plane of the figure. Given, AE = ED
body in your hand. You can easily see that such force will
and q = 37°.
have no turning effect about x-axis but will try to turn the
body about z-axis as well as y-axis. (i) Find torque of all forces about centre O.
If our body is constrained to rotate about y-axis only, then (ii) What is the direction of the torque?
we may not be interested in tz. In such cases, ty = bF is the (iii) What is the torque due to all forces about an axis
relevant quantity. We call this as torque about y-axis. through O that is perpendicular to the plane of the
While calculating torque of a force about an axis, one figure.
just needs to learn the following three points:
1. If the given force is parallel to the axis, torque about
[  3
​ Take sin 37° = __ ]
​   ​   ​
5
the said axis is zero.
2. If the line of action of the force intersects the axis,
there is no torque.
3. A force F has torque about an axis (say z) only
when F is neither parallel to z-axis nor its line of
action intersects z-axis. When a force is perpendicular
to z-axis but its line of action does not intersect it
(skewed lines), torque about z-axis is t = Fd, where
d is the perpendicular distance between the line of
force and z-axis. Solution
Think of a door that can rotate Concepts
about z-axis. if you apply a force Fx
(i) t = F r^  or  t = r F^
(See fig.) it will produce no rotation
[Try with the door in your room]. (ii) We will find torque due to individual forces and
Line of Fx intersects z-axis. It has no then add them to get the resultant torque
torque about this axis. If you apply a
(i) Torque due to F1 is zero as the line of action of the
force Fz, it is also not going to rotate
forces passes through O. F2 acts along DC.
the door. Fz is parallel to z-axis.
When you apply Fy, the door Its perpendicular distance from O is OF = 1.5 m
rotates; Fy has torque about z-axis \ t2 = F2 × 1.5
and its torque is ty = Fy. d, where d is the distance between
z-axis and line of Fy. = 20 × 1.5 = 30 N-m ( )
Note:  When many forces are acting on a body and we
are supposed to find the resultant torque about a point (or Torque due to F2 tries to produce a clockwise rota-
about an axis), we find torques due to individual forces and tion in the plate about O. ( ) sign denotes clockwise
add them. direction. We can also say that this torque
​___›
is directed
​_›
_
​› into the plane of the figure (along OC​
​   × ​F  ​2  )
Example 1   A_ force ​F  ​  = (2 + 3 ) N acts at a point having
​›
position vector ​r ​   = (2 – ) m. D F C
F4 F2
(i) Find the torque of the force about the origin.
q F4 cos q
(ii) Find the torque of the force about z-axis.
Solution F4 sin q E O F3
Concepts a q
​_› ​_› ​_›
(i) ​t   ​ = ​r ​   × ​F  ​  A B
(ii) z-component of t is known as torque about z-axis
4.4  Mechanics II

4
tan a = __
​   ​   fi  a = 53° Solution
3
Concepts
Given q = 37°
(i) Forces parallel to z-axis have no torque.
\ F3 is perpendicular to OB.

(  )
(ii) Forces having their line of action intersecting z-axis
5
\ Torque due to F3 is t3 = OB ◊ F3 = ​ __
​   ​  m  ​ (20 N) have no torque.
2
(iii) Forces which are along x- and y-direction and do
= 50 N-m ( )
not intersect z-axis have torque.
This torque has anticlockwise ( ) direction. You can t = F ◊ d, where d = perpendicular distance between the
also say that the torque vector is out of the plane force and z-axis.
of the figure. [If you still feel confused about this
direction, just curl you right-hand fingers in anti- F6 is parallel to z-axis. F2 and F5 have their lines inter-
clockwise direction; the upward-pointing thumb is secting z-axis. These force have no torque about z-axis.
the direction of torque]. F1 F1
F1 can be split into two components: ​ ___ __  ​ along EI and ___
​  __ ​  
Torque due to F4 is ÷   
​ 2 ​   
​÷2 ​
along EG. The later component is parallel to z-axis and has
t4 = (OE) (F4 cos q) [   t = r ◊ F^]
no torque.
F1 Fa
= 2 × 20 × __
4
​   ​  = 32 N-m ( ) \ t1 = ___
​  __  ​  ◊ (EB) = ___
​  __  ​  [Along negative z]
5 ÷   
​ 2 ​ ÷   
​ 2 ​
Resultant torque is t = t1 + t2 + t3 + t4 Direction of this torque is along negative z-direction. Curl
your right-hand fingers in the direction of rotation produced
= 0 + 30 ( ) + 50 ( ) + 32 ( ) F1
by ​ ___
__  ​;  the thumb points along negative z-direction.
= 12 N-m ( ) ÷   
​ 2 ​
(ii) Direction of resultant torque is clockwise or we can Similarly, component of F3 along IH is parallel to z-axis
say that the torque vector is directed into the plane and has no torque. Component along IC has torque given
of the figure. by
F Fa
(iii) Torque about O is directed along the given axis. t3 = ___
​  __  ​ (BC) = ___ ​  __  ​ [along negative z]
÷   
​ 2 ​ ÷   
​ 2 ​
\ Torque about the given axis is same as the torque
Torque due to F4 is
about point O. _
​_› ​›
t4 = F4 (DA) = aF [along negative z]
Note:  When all ​r ​   and ​F   ​ are in a plane (say xy plane),
torque about a point O in the plane is always perpendicular \  Resultant torque is
to the plane (along z-axis). In such cases, it hardly makes a Fa Fa __
t = ​ ___ ​ + ​ ___
__      + 1) in negative z
 ​ + Fa = Fa (​÷2 ​
__ 
difference whether we say ‘torque about point O’ or ‘torque ÷    ÷
​ 2 ​   
​ 2 ​
about z-axis’. direction.

Example 3   A cube has side length a. Six forces are Example 4   A rod AB is being acted by three forces per-
acting on it, as shown. Each force has magnitude equal to pendicular to its length, F1
F. Find the torque on the cube about the side AB marked as shown in figure. Sum F2
as z-axis. of the forces is zero. Find C
A B
resultant torque on AB a
G b
H about end A, about end B
F2
and point C. F3
F1
45° Solution
E I
F3 Concepts

A
t = F r^
F5
D
Given, F3 = F1 + F2 ...(i)
F4
B
C Torque about point A is
z tA = t1 + t2 + t3 = 0 + F2 (a + b) ( ) + F3 a ( )
F6
= F2 (a + b) – F3 a ( ) ...(ii)
Torque and Equilibrium  4.5

Torque about B
tB = t1 + t2 + t3

= F1 (a + b) – (F1 + F2)b ( )

= F1 a – F2b ( ) ...(iii)

Using (ii) and (i) we can write:

tA = F2 (a + b) – (F1 + F2) a ( ) _


​›
Torque of M​g ​
   about O is directed into the plane of the figure.
= F2b – F1a ( ) We may simply say that torque is clockwise. (Curl you right-hand
fingers clockwise and the thumb points into the paper.)
= F1a – F2b ( )
Torque about C (ii) In the above two figures, torque about z-axis is same
as torque about point O. Torque about O has no
tC = F1a – F2b ( ) component in x- or y-direction.
​_›
Note: one can see that tA = tB = tC . This is important. If (iii) ​F   ​ is a force acting at point P and we
​_› wish to find its
sum of all the forces is zero, we get same torque about any torque about O. Extend the line of ​F  ​  (if needed) and
point (A, B or C). drop a perpendicular from O on this line. OQ is this
perpendicular. Torque of F about O is t = r^ ◊ F =
In Short: (OQ) F. In the shown figure, the force has a tendency
to rotate a body clockwise about O. We can simply
(i) Torque is ​_directed ​_› perpendicular to the plane
› say that the torque is clockwise.
containing ​r ​   and ​F  ​. 

F Applied
force

(iv) A force parallel to z-axis or intersecting z-axis will


have no torque about z-axis. A force that is away
Torque from z-axis directed perpendicular to z-direction has
direction a torque about z-axis.
t
(v) If resultant of all forces acting on a body is zero and
the sum of their torques is zero about a point, then
Z the resultant torque must be zero about any other
point.

Your Turn
​_›
Q.1  A force ​F  ​  = 3 i.e., N acts at a point (2, 2) m. Find
torque of the force about the origin.
Q.2  In the figure shown, find torque F
of force F about O. Given F = 20 N,
30°
d = 1 m. P

Q.3  AB is a rod. Two forces of same Q.4  A pulley is free to rotate about an axis through its
magnitude F act at the two ends of the rod d center O that is perpendicular to the plane of the figure. A
in opposite directions. Show that torque on thread passing over the pulley is being pulled at its two ends,
the rod is same about A or B or any other O with force F. Mass of the pulley is M. Find resultant torque
point C. Such pair of equal and opposite forces are often on the pulley about its rotation axis.
known as a couple.
4.6  Mechanics II

F4

z F1
D
C

F2

F3
A B
F5

Q.6  A ball of mass m is projected from ground with


Q.5  A table is acted upon by five forces, as shown. [Do velocity u, making an angle q with horizontal. Find torque
not worry about equilibrium]. All forces have same magni- of weight of the ball about the point of projection when the
tude of 20 N. Find the resultant torque about z-axis shown. ball is at its maximum height.
Given AB = 2 m; AD = 1 m.

4. EQUILIBRIUM
A body is said to be in equilibrium if the sum of all forces
acting on it is zero and the sum of all torques acting on it
about a point is also zero.
Fnet = 0 [Translational Equilibrium]  ​ Complete      ​
​ equilibrium ​
t = 0 [Rotational Equilibrium]
net

In previous chapters, we have never actually cared


about the point of application of a force. Now, we need to
We will take torque on the rod about point C to be zero
be careful. Torque of a force will depend on its point of
[You can take any other point A, B or D].
application. In this context, it is very important to remember
that weight of a body must always be shown acting at the Tc = 0
COM of the body.
Mg has no torque about this point. Tension T1 has
Example 5   A stick suspended with two strings anti-clockwise and T2 has clockwise torque about C. They
A uniform stick of mass M must be of same magnitude for net torque to be zero.
and length L is suspended
L L
horizontally with the help 1 \ T1 ​ __ ​ =
  T2 ​ __ ​ 
of two vertical strings, as 2 4
2
shown. String 1 is tied to fi 2T1 = T2 ...(ii)
one end of the stick and L
String 2 is tied at a dis- 4 Solving (i) and (ii) gives:
L
__
tance ​    ​ from the other
4 Mg 2Mg
end. Find tension in the two strings. T1 = ___   and  T2 = ____
​   ​   ​   

3 3
 ​
Solution
Concepts Example 6   Line of action of normal reaction

(i) Net force on the stick is zero. A uniform block of mass M is held
against a vertical wall by applying a
(ii) Torque on the stick about any point is zero. horizontal force F on it. The line of
(iii) The above two facts help us write two equations, action of the force passes through its
which are enough to find the tension in the COM (C). The normal force applied by
strings. the wall on the block is acting effec-
tively along a line that is at a distance
Weight of the rod must be shown acting at its COM.
d from C. Find d. Width of the block
Fnet = 0
is b, as shown.
fi T1 + T2 = Mg ...(i)
Torque and Equilibrium  4.7

Solution (iii) We also have three equations to solve for the three
Concepts unknowns:
(i) The block must be in translational as well as rota- Fx = 0,  Fy = 0  and  t = 0
tional equilibrium.
(iv) Torque is zero about any point. Any point like
(ii) The instinctive answer to this
A, B or C (or any other point) will work fine. But
question is that normal force
a certain choice can make calculation easier. The
must be horizontal and passing
best choice, in general, is a point at which most
through C, but this is wrong. If
number of forces act. In this case, t = 0 equation
N were actually passing through
will have the least number of terms. In this prob-
C, then N, F and Mg will have
lem, two forces act at end A (friction and N1). We
no torque about C. This leaves
will choose A as the pivot point about which we
friction ( f ), producing an unbal-
will balance the torque.
anced torque about C.
It is easy to guess that N must act Fx = 0  gives
along a line that is below C.
N2 = f ...(i)
For translational equilibrium, f
b
Fy = 0  gives
N = F  and 2
N1 = Mg ...(ii)
f = Mg
C F tA = 0  gives:
For rotational equilibrium, torque
d
about point C must be zero. In fact, L
N N2 L sin q = Mg __
​   ​  cos q
torque is zero about any other point 2
also. It is just that we have chosen Mg
y
point C.
F and Mg have no torque about C. Friction (f) has a N2
clockwise torque and normal force N has an anti-clockwise B
torque. For zero torque about C, these two torques must
L C
balance. L sin q
b N1
N ◊ d = f ◊ ​ __ ​  q
2 x
Mgb f A
b
fi F ◊ d = Mg ◊ ​ __ ​   fi  d = ____
​      Mg L
2 2F cos q

2
Example 7   Leaning ladder
A ladder leans against a smooth [Note that line of N2 is at a perpendicular distance
wall, as shown. Coefficient of fric- L sin q from A and line of Mg is at a perpendicular distance
tion is m. Find the smallest angle L
that the ladder can make with the ​ __ ​  cos q from A]
2
floor and not slip. 1
q fi N2 tan q = __
​   ​  Mg
Solution 2
1
Concepts Using (i): f tan q = __
​   ​  Mg
2
(i) The lower end of the
ladder has a tendency Mg
y fi f = ______
​    ​ 
to slip towards right. 2 tan q
Hence, friction on it is
towards left. B N2 But  f < m N1  fi  f < m Mg  [using (ii)]
(ii) We have three unknown C Mg
N1
forces: the friction (f), \ ​ ______  ​ 
< m Mg
Mg 2 tan q
the normal forces N1
q
and N2 by the floor and x 1
f A fi  tan q > ​ ___  ​ 
the wall. 2m
4.8  Mechanics II

Your Turn

Q.7  A uniform rectangular block force when a 10 kg load is hanged from the end C of the
is placed on an incline and it stays beam.
at rest. C is the centre of the block.
The normal force by incline on the
Q.10  A seasaw has a length of 5.0 m and its fulcrum
(support) is at its centre. Two boys weighing 10 kg and 15 kg
table will be effectively acting along
are trying to keep it in balance. The 10 kg boy sits at one
a line as indicated by 1, 2 or 3.
of the ends. At what distance from the centre, the other boy
Which is correct?
must sit to keep the seesaw in equilibrium?
Q.8  A rod of length L = 6 m is hinged to the floor at its
lower end A and leans against a smooth wall. Mass of the
rod is 20 kg. A 40 kg ball is tied
at point C of the rod such that
AC = 4  m. Given, q = 53°
(i) Find the force applied
by the wall at B on the
rod.
(ii) Find the vertical compo- Q.11  A 160-cm tall boy is sleeping on a massless
nent of force applied by board. The board is horizontal, supported by two weighing
the hinge at A. scales – one right under the head of the boy and the other
under his toe. Reading of the two scales are 30 kg and 25 kg
Q.9  CD is a uniform A respectively. At what distance is the centre of gravity of the
5 kg beam. It is horizon- boy from his toe?
tal and is hinged at end
D. Cable AB supports it
with –ABD = 37°. Find
37° B C
tension in the cable and D
horizontal and vertical 1.0 m 0.4 m
components of hinge
10 kg

5. TOPPLING This makes Fnet = 0 and tnet = 0. The brick can stay in equi-
librium in this position forever. The brick is pushed slightly
Consider a brick placed on a table. In figure (a), the brick to the left (figure b) but the vertical line of Mg is still pass-
is shown slightly protruding out of the edge of the table. Its ing through the support region. Normal force will adjust so
weight (Mg) is a vertical force through its COM (C). For
equilibrium, the normal force (effectively) passes through C.

C
A

(a) Mg

C
A

(b)
Mg
Torque and Equilibrium  4.9

that it passes through the COM (C) and the body will stay and having the same
in equilibrium. if the brick is pushed a little more towards cross-sectional area
right, such that C lies just above the edge A. It will be in is welded to the
critical equilibrium with normal force, adjusting itself to pass centre of the first
through C. Any further push to the brick will cause the line bar so that angle
of Mg to fall outside the support region. Now, normal force between them is
cannot shift to the right of point A; at best, it can stay at A 37°. Find the maximum length l of the second bar so that the
(see fig. d). Clearly, there is an unbalanced torque on the body can stay in equilibrium in a vertical plane, as shown.
brick about A. This causes it to rotate and it topples.
Solution
This result is true in general. When you bend to touch
your feet, you body posture adjusts so that the vertical line Concepts
through COM of your body passes through the support The vertical line through COM of the body should pass
region between your feet. If you try doing this act while within the support region for equilibrium. In critical case,
your back rests against a wall, you will never be able to do the vertical line through COM will pass through end of
it. As you bend, your hip cannot go back (as there is wall) the horizontal bar.
and COM of your body moves forward. The vertical line of
Mg falls outside the support region between your feet.

CM

Mg Let l = linear mass density of the two bars.


  
Mass of horizontal bar, m1 = a l
5.1 A cube on an incline
Mass of inclined bar, m2 = l l
Consider a uniform cube placed on a rough incline. When
Taking O as orgin, the x co-ordinate of COM of the
inclination angle is very small (as shown in fig. (a)), the
system is
line of Mg passes within the support region, AB and the cube
will not topple. If friction is large enough (m > tan q1), the
cube will not slide also. It will stay at rest.
m1 (0) + m2 ​ __
2
xcm = ___________________
​     
(   ​   
)
​ l  ​  cos 37°  ​
m1 + m2
If inclination angle is increased to q2 (see fig. (1)). Such
that line of Mg passes through point A, the normal forces
shifts to point A. Now all three forces (Mg, N and friction
f ) are passing through point A. There is no torque about A. 2
l 4
(ll) ​ __
= _________
​ 
5
  
(  )
​    ​  ◊ ​ __ ​   ​

2l2
​ = _______
​     ​
Block will still not rotate. al + ll 5 (a + l)
a
N For equilibrium, xcm < ​ __ ​ 
f C 2
C B
C a
B
A
In critical case, xcm = ​ __  ​
B A 2
A Mg
Mg q2 q3 Mg
q 2l2 a
fi ​ _______   ​ = __
​   ​   fi  4l2 – 5al – 5a2 = 0
Fig. (a) Fig. (b) Fig. (c) 5 (a + l) 2
__________
If coefficient of friction m is large enough (m > tan q2), it    ​
5a ±  ​÷25a 2
+ 80a 2
    ​
will not slide as well. This is the critical case. If inclination fi l = _________________
​      
8
is increased beyond this (see fig (c)), line of Mg passes

( 
____
outside the support region AB. Normal force cannot move
beyond A. Now the block has a net torque about A due to

8
   
5 + ​÷105 ​
l = ​ ​ ________
 ​  
 ​ a )
Mg. If friction coefficient is still good enough (m > tan q3), a
the block will not slide but it will topple. If l is increased beyond this, xcm will exceed ​ __ ​  and the
2
body will topple.
Example 8   A uniform bar has length a. It is resting on
a horizontal table. Another bar made of the same material
4.10  Mechanics II

A N = Mg N
Example 9    Suspended semi- q
circular ring Both N and Mg have zero torque A
A semi-circular ring is suspended in O about A. q
2R
a vertical plane such that it can rotate x = ___
​  p ​ 
freely about its end A. Find the angle O
B q x
(q) that the diameter AB makes with x 2
cos q = __
​    ​ = __
​   ​
the horizontal in equilibrium. R p CM

Solution
Concepts
\
2
q = cos–1 ​ __(  )
​ p ​ ​
Mg
(i) The COM of the semi-circular ring is at a In short:
2R
distance ___
​  p ​ from O, lying on its symmetry line. (i) A body is in equilibrium if sum of all external forces
(ii) The ring will stay in equilibrium only if torque of acting on it is zero and the sum of all external torques
external force about A is zero. (about any point) is also zero.
This is possible only if COM of the half ring lies (ii) A body remains in equilibrium if vertical line through
vertically below point A so that Mg produces no torque COM of the body passes through the support region.
about A. If this line falls outside the support region, torque
due to Mg makes the body unstable.
The equilibrium position is as shown in the figure. (iii) A freely suspended body stays in equilibrium in a
CM is exactly below A. position where its COM is exactly below its point
Support (at A) applies a force (N) of suspension.

Your Turn

Q.12  Why must you bend forward while carrying a heavy Q.15  A cubical block is kept on a rough incline. The
load on your back? inclination angle (q) of the incline is increased gradually.
Find the maximum value of coefficient of friction (m) so
Q.13  Three trucks are parked on a hill. Their centre of that the block starts sliding before it topples.
gravity lies at A, B and C (see fig). Which truck will tip
over?

Q.16  A long smooth plank is balanced like a seesaw. It


supports a small ball and a more massive ball, with com-
pressed spring between the two. The system is balanced
with the balls at the centre of the plank. Now the spring is
released. The two balls move away from each other. Does
the plank tip clockwise, anti-clockwise or remain in balance,
Q.14  Two identi- L
as the balls move away from each other?
cal uniform bricks
have been arranged
on a table, as shown.
Is the system in L
4
equilibrium? A
L
4
Torque and Equilibrium  4.11

Miscellaneous Examples
Example 10   Rod with a bead Solution
A bead of mass m is threaded on a light rod that is pivoted Concepts
to a point O on a vertical wall. Initially, the rod is held in (1) Horizontal component of tension at A = F and
horizontal position with vertical component of tension at A = mg.
the bead at a distance
x from O. The rod is (2) For rotational equilibrium, the torque about any
released from this posi- point should be zero
tion and it rotates in ver-
tical plane about O. Calculate the angle q that the rod makes T sin q = F
with horizontal at time t after its release. 5F
fi t = ___
​   ​ 
Solution 3
Concepts Also mg = T cos q
5F 4
The massless rod cannot apply a normal force on the bead. fi m = ___
​   ​  ◊ ​ ____
    

If there is such a force, then the rod will experience a 3 5 ◊ g
finite torque (and hence infinite angular acceleration). If 4F
there is no normal force, there cannot be friction between = ___
​   ​ 
3g
the bead and the rod.
Now consider torque about end A.
The bead experiences free tA = 0
fall. In time t, it falls through a
fi mg ◊ x = FH
distance given by:
4F 3H
1 fi ​ ___ ​  x = FH  fi  = ___
​   ​ 
y = __
​   ​  gt2 3 4
2
y gt2 T T cos q
\ tan q = __
​ x ​ = ___
​   ​ 
2x A
T sin q

\ q = tan –1
(  )
gt2
​ ___
​   ​  
2x H
q
CM
(x, y)

F
Example 11   Flexible rope O B
One end (A) of a flexible rope is fixed to a vertical wall and mg
its other end (B) is pulled by a horizontal force F. The rope
makes an angle of q = sin–1 ​ __
3
(  )
​    ​  ​ with the wall and its top
5
Example 12   A balance is mounted on a stationary trol-
ley, with a weight (A) attached to its one end. The other
end is at a vertical height H above the other end. Find the
end of the balance is linked
mass of the rope and x co-ordinates of its centre of mass (in
to the floor of the trolley
the co-ordinate system shown in the figure.)
by a spring. The trolley
is accelerated to the right A
y
and the string holding the
A weight gets inclined to the
vertical. When the weight
q
gets stable and is not swing-
H
ing, does the tension in the
B spring change?
F x
O Solution
Concepts
The balance will remain balanced if torque of forces at
the two ends, about the pivot point, is zero.
4.12  Mechanics II

Tension in the string increase since


T cos q = Mg
T sin q = Ma
______
fi T = M​÷g  2 + a2 ​ 

B O C

q
T Notice the direction of friction on two rods. Friction on AC must
produce clockwise torque to balance the anti-clockwise torque of
M1g about A.
A a

Forces on the two rods are as shown. We have not shown


W = Mg forces acting at A and B, since we plan to take torque about
these points.
But torque of tension force acting at B (torque about O) For rotational equilibrium of rod AC, we take
is tA = 0
T cos q ◊ (BO) = Mg ◊ (BO)
Since torque is unchanged, the spring tension will also
f (AC) = M1g ​ ___( 
AC
)
​   ​  cos q  ​
2
not change. M g
f = ____
1
fi ​   ​   cos q ...(i)
2
Example 13   One thin rod leans on another in vertical For rotational equilibrium of BC, let us take tB = 0
plane, as shown. The right rod makes an angle q with the
horizontal floor and the left rod makes an angle (90 – q)
with the floor. The left rod extends infinitesimally beyond

BC
N(BC) = M2g ​ ___( 
​   ​  cos (90 – q )  ​
2 )
the end of the right rod. Coefficient of friction between M2g
fi N = ​ ____
 ​ 
 sin q ...(ii)
the two rods is m and both sticks have same mass per unit 2
length. The ends A and B on the ground are hinged so that We know that
the rods can rotate about A and B. Find minimum value of
q for which the rods do not fall.   f £ mN
M1g M2g
fi  ​ ____  cos q £ m ​ ____
 ​   ​ 
 sin q
2 2
M1
fi  ​ ___ ​  £ m tan q
M2
l AC 1
fi  ​ _____ 
 ​ £ m tan q  fi ​ ____    
​ £ m tan q
l BC tan q
Solution
Concepts fi 
m
1
tan2 q ≥ __
1
​    ​ fi  qmin = tan–1 ​ ___
(  )
​  __   ​ ​
​÷m 
   ​
(i) The angle between the rods is 90°.
Note:  Let us check our answer in two extreme cases:
(ii) Mass of each rod is proportional to its length. (i) When m Æ 0, q must be close to 90°. It makes sense
(iii) The two rods apply normal force (N) and friction as we need to have a very small angle between the
(f) on one another. rods, if friction between the sticks is very small.
(iv) To avoid hinge force in our equations, we will (ii) When m Æ • (i.e, very sticky rods), q can be very
balance torque about A and about B, for rods AC small. This also makes sense.
and BC respectively.
Example 14   A spool at rest
Let l = mass of unit length
A spool has an axle of radius r and outer disc of radius 2R.
Mass of rod AC is M1 = l (AC) A thread is tightly wrapped around the axle and is pulled
Mass of rod BC is M2 = l (BC) with force T at an angle q with horizontal. Friction between
ground and the spool is sufficiently large and the spool
Torque and Equilibrium  4.13

does not moves (it neither


translates nor rotates). T

(i) Find q R
r
(ii) If coefficient of friction q
between the spool and
the ground is m, find
the largest value of T
for which the spool
remains at rest.
r
From figure in D CPA: cos q = __
​    ​
Solution R
Concepts From equation (ii),

(i) We will write equations using followings: N = Mg – T sin q


FHorz = 0 ;  Fvert = 0 ;  tcentre = 0. Since f £ mN
(ii) Another quick way of solving this problem is to T cos q £ m (Mg – T sin q)  [f = T cos q from (i)]
realise that the friction force, the normal force and
the weight of the spool pass through the contact mMg
point. If line of action of T also passes through this fi T £ ____________
​      ​

cos q + m sin q
point, then there will be no resultant torque. ______
r¢ ​÷R  2 –  r2 
 ​
Since, cos q = __
​   ​    Hence, sin q = ​ _______  ​ 
(i) FHorz = 0 R R
fi f = T cos q ...(i) mMg mMgR
\ T £ ​ ____________      ​ = ____________
______ ​    
   ​
______
Fvert = 0
__r m ​÷R  2
– r 2
  ​ r + ÷
m ​  
R 2
– r2 
 ​
​    ​ + ​ _________
    ​ 
N + T sin q = Mg ...(ii) R R

Torque about centre, tcentre = 0


Example 15   A stick is semi-infinite, i.e, one end goes off
T ◊ r = f ◊ R ...(iii) to infinity. Linear mass density of the stick is given by l =
le– x/l, where x is the distance from one of its ends and l is
T sin q
a constant. It balances on a support at a distance x0 from its
T
end at x = 0. Find x0.

C 1
Mg q Given: ​Ú ​  ​ ​x e– ax = __
​  2  ​ 
T cos q 0 a
N Solution
Concepts
f A
The support must be at the COM of the stick. It means
[Force T is tangential at a distance r from C and f is xcm = x0
at a distance R. N and Mg pass through C]
Consider a small elemental length dx of the stick at co-
From (i) and (iii)
ordinate x.
f r Mass of element dm = l dx = l e–x/l dx
​ __  ​ = cos q = __
​    ​
T R • •

\
r
q = cos ​ __ –1
(  )
​    ​  ​
R
Ú xdm
0
=
l0 Ú xe – x / l dx
0
\ xcm = • •
Alternate:
Mg, N and f pass through A. If T also passes through A,
Ú dm l0 Ú e – x / l dx
0 0
net torque on spool will be zero. If line of T does not pass •
x
– ​ __ ​  1
from A it will create a torque about A and the spool cannot Ú 
​  ​ ​ ​x​e​ l ​ dx = ______
​   2 ​  = l2
remain in equilibrium. 0
(  ) 1
__
​​ – ​   ​   ​​ ​
l
4.14  Mechanics II


• B
And ​Ú ​  ​   ​e– x/l dx = – l ​​[ e –x/l ]​​0​ ​ = – l [0 – 1] = l
0
R
l2
\ xcm = ​ __ ​  = l
l q/2
A
Example 16   Rod on a hoop
A rod of linear mass density l leans against a hoop of radius
R. The rod makes an angle q with the horizontal and is tan- Torque on the rod about A is zero.
gent to the hoop at its upper end. Friction is large enough
to keep the system at rest. Find the friction force between
\
AB
N (AB) = mg ​ ___ ( 
​   ​  cos q  ​
2 )
the ground and the hoop.
1 l Rg
fi N = __
​   ​  _____
​   ​ cos q
  ...(ii)
2 q
tan ​ __ ​ 
2
For horizontal equilibrium of the hoop,
R
f1 + f2 cos q = N sin q
q
lRg
fi  f1 + f1 cos q = ______
​    ​  ◊ cos q ◊  sin q
Solution q
2 tan ​ __ ​ 
Concepts 2
lRg sin q cos q
(i) Mass of the hoop is not given. We will not write fi  f1 = ​ _______________        ​
q
the equation of vertical equilibrium for the hoop. 2 tan ​ __ ​  (1 + cos q)
We will consider its horizontal equilibrium and 2
rotational equilibrium only.
(ii) We will consider torque on the rod about its lower [  sin q
​  ​ ________
  
1 + cos q
 ​ = tan
q
__
]
​   ​   ​
2
end. This will remove the need to consider force 1
acting on the lower end of the rod. = __
​   ​  lRg cos q
2
Let us check the result under extreme condition
N p
__
f2 when q Æ ​   ​  ,  f1 Æ 0
2
This makes perfect sense.
N
f2 C
Mg Example 17   A hinged door
N1 A door in a showroom is 4 m tall and 2 m wide. It is hinged
q mg
at two locations on the wall. Hinge H1 is located 1 m from
A f1 the top and hinge H2 is located 1 m from the bottom of the
door. Mass of the door is 100 kg and each hinge applies force
Ground friction on hoop = f1
of equal magnitude on the door. Find force applied by each
Friction between the rod and the hoop = f2 hinge on the door.
Normal force between the rod and the hoop = N
For rotational equilibrium of the hoop, net torque on it
about its centre (C) must be zero.
N, N1 and Mg have no torque about C.
\  Torque due to f1 and f2 must be equal and opposite

fi f1R = f2R  fi  f1 = f2 ...(i)


R
Length of rod AB = _____
​     ​ 
q
tan ​ __ ​ 
2
l R
\  Mass of rod, m = _____
​    ​ 
q
tan ​ __ ​ 
2
Torque and Equilibrium  4.15

Solution Solution
Concepts Concepts
(i) Though the door is free to rotate about vertical (z) (i) Net force and torque on the (disc + particle)
axis, we are not concerned about the torque about combined system must be zero.
z-axis in this problem. (ii) We will balance forces along the incline and
(ii) Look at the figure. Mg has a tendency to rotate perpendicular to it and will take torque to be zero
the door clockwise about H2. If hinge H1 breaks, about the centre of the disc.
torque of Mg will turn the door as shown in the However, choosing contact point as pivot point [for
second figure. t = 0] can straightaway give the value of m. This method
is being left for students to try.
One other method, is to understand that COM of
the combined system must lie exactly above the contact
point.

  
This is rotation about y-direction. To prevent this
rotation, the horizontal force applied by H1 on
the door must be to the left. This will cause an
anti-clockwise torque on the door about H2. Net force along the incline is zero.
(iii) For horizontal equilibrium of the door, the horizon- \  (Mg + mg) sin q = f
tal components of two hinges forces must be equal 3
and opposite. fi  ​ __ ​  (M + m) g = f ...(i)
5
(iv) Since forces by two hinges are of equal magnitude Net force normal to the incline is also zero.
and their x components have same magnitude; the
4
z components must also be same. \  N = (Mg + mg) cos q = __
​   ​  (M + m) g ...(ii)
5
100 × 10
2Fz = Mg  fi  Fz = ​ ________
 ​  = 500 N Torque about centre (O) of the disc = 0
2
Net torque about H2 is zero. fi f ◊ R = mg ◊ R cos q

\ Fx (2 m) = Mg (1 m) Fz has no torque about [   Mg and N have no torque about O]


H2. Fx passing through 4
H2 also has no torque fi f = __
​   ​  mg ...(iii)
5
fi Fx = 500 N From (i) and (ii), 4m = 3 (M + m)  fi  m = 3M
_______ __
\  Force by one hinge = ÷
​F  x2 + Fz2     N
 ​ = 500​÷2 ​ Since f £ mN
4 4
Example 18   Keeping a disc at rest on an incline \  ​ __ ​  mg £ m ​ __ ​  (M + m) g
5 5
A uniform disc of mass M is kept on
a rough incline of inclination angle fi 3M £ 4M m
q = 37°. A small particles of mass m
3
is fixed on the circumference at point fi ​ __ ​ £
  m
B. When the disc is placed on the 4
incline with its diameter AB parallel Alternate for part (i)
to the incline, it stays in equilibrium. Find: Consider a vertical line through con-
(i) m tact point C. This line intersects the
(ii) minimum coefficient of friction between the disc and diameter AB at point D. This point D
the incline. must be the COM of the system.
4.16  Mechanics II

OD But f £ mN
From figure: ____
​    ​ = tan q
OC
\  F sin q £ m (Mg – F cos q)
3 3 R
fi OD = R × __ ​   ​  = __
​   ​  R. And DB = __
​   ​ 
4 4 4 F sin q
fi  ​ ___________
  ​ £ m
  
Mass (m) of the disc is assumed at O. For D to be COM Mg – F cos q
of the system,
sin q sin q
M(OD) = m (DB) fi  ​ _________     ​ £ m  fi ___________
  ​       ​ £ m
Mg
___ 2
_____
​     
​ – cos q ​     ​ – cos q
fi (  ) (  )
3
M ​ __
R
​   ​  R  ​ = m ​ __
4
​   ​   ​  fi  m = 3M
4
F cos q
[Using (i)]

Example 19   Lifting a rod sin q cos q sin q cos q


fi ​ _________  ​ £ m  fi ​ ____________________
          ​ £ m
A uniform rod is lying on a table. A boy applies force at 2 – cos2q 2(sin2 q + cos2q) – cos2q
its one end so as to slowly lift the rod. He keeps the force
always perpendicular to the rod. The other end of the rod sin q cos q 1
fi ​ _____________
        ​ £ m  fi ____________
​       ​ £ m
does not slide and the rod was brought to a vertical position. 2 sin2 q + cos2 q 2 tan q + cot q
Determine the minimum coefficient of friction between the
rod and the table. m is minimum when z = 2 tan q + cot q is maximum.
Solution dz
fi ​ ___   ​ = 0  fi  2 sec2 q – cosec2 q = 0
Concepts dq
1
(i) The rod is raised slowly. It means there is no fi tan q = ___
​  __  ​ 
÷   
​ 2 ​
acceleration and the rod is in equilibrium at any
1
____________ _______1
position. \ mmin = ​       ​ = ​    __ 
 ​
2 tan q + cot q ___2__
(ii) To do this, the torque due to applied force must ​    ​ + ÷   
​ 2 ​
÷   
​ 2 ​
always equal to the torque due to weight of the 1__
fi mmin = ​ ____
   ​ 
rod.   
2​÷2 ​
(iii) Considering the equation for horizontal and verti-
cal equilibrium will give us the normal force and Ex ample 20   Car on a circular track
friction force by the floor on the rod. A car is negotiating a circular track of radius r. Centre of the
track is O and the car is going into the plane of the figure.
AB is the original In the figure, you are seeing the rear wheels. Separation
position of the rod. between the wheels is 2a and height of the COM of the
Consider it at position car above the road is h; friction is large enough to prevent
AB¢. End A does not slipping. Find maximum speed (v0) of the car so that it does
slide and applied force not overturn about the outer wheels.
F is perpendicular to the
rod AB¢.
Rod is always in
equilibrium.
tA = 0

( 
l
Fl = Mg ​ __ )
​    ​  cos q  ​, where l = length of the rod
2
1 Solution
fi F = ​ __ ​  Mg cos q ...(i)
2 Concepts
For vertical equilibrium: (i) In reference frame of the car, it is a problem of
equilibrium. In this frame, we need to apply the
N + F cos q = Mg  fi  N = Mg –  cos q ...(ii) centrifugal force at the COM.
(ii) The car begins to overturn when normal contact
For horizontal equilibrium:
force between the inner wheel (A) and the ground
f = F sin q ...(iii) becomes zero.
Torque and Equilibrium  4.17

For the rotational equilibrium let us take torque about B


to be zero
Mv2
N1 ◊ 2a + ____ ​  ◊ h = Mg a
​     
R
Mg _____Mv2h
fi N1 = ___
​   ​ 
 – ​   ​ 
2 2aR
Car will not overturn if N1 ≥ O
Mg _____ Mv2h
fi ​ ___ ​  
 ≥ ​   ​ 
For horizontal equilibrium (in RF attached to the car): 2 2aR

÷ 
____
Mv2
f = ​ ____
  
​  aRg
fi ​ ____ ​ ​   ≥ v
​     
R h

÷ 
For vertical equilibrium: ____
aRg
N1 + N2 = Mg \ v0 = ​ ____
​     
​ ​  
h
Worksheet 1
a
1. When a ceiling fan rotates, (a) W ​ __  ​ anti-clockwise
2
b
(b) W ​ __  ​ anti-clockwise
2
(c) Wa anti-clockwise
(d) Wa clockwise
4. A weightless horizontal rod is acted on by upward
parallel forces of 2 N and 4 N at ends A and B respec-
tively. The total length of the rod AB = 3 m. To keep
the rod in equilibrium, a force of 6 N should act in
(a) all particles go in circles of same radius. the following manner.
(b) its weight has a torque about its rotation axis. (a) Downwards, at any point between A and B.
(c) all particles go in circles having their centres on (b) Downwards, at mid-point of AB.
the rotation axis. (c) Downwards, at a point C such that AC = 1m.
(d) None of the above. (d) Downwards, at a point D such that BD = 1m.
2. Four forces tangent to the circle of radius ‘R’ are act- 5. A solid cone hangs from a fric-
ing on a wheel, as shown in the figure. The resultant tionless pivot at the origin O, as
equivalent one-force system will be: shown. If , and are unit vec-
tors, and a, b, and c are positive
constants, which of the following
forces F applied at a point on
the lower rim of the cone results
in a torque t on the cone with a
negative z component tz?
(a) F = a  , at point (0, b, – c)
(b) F = – a , at point (0, – b, – c)
(c) F = a , at point (– b, 0, – c)
(d) None
(a) (b) 6. A uniform rod of length l is
placd symmetrically on two
walls, as shown in figure. The
rod is in equilibrium. If N1
and N2 are the normal forces
exerted by the walls on the
(c) (d)
rod, then
(a) N1 > N2
3. A person supports a book of weight W between the (b) N1 < N2
forefinger and the thumb, as shown.The point of grip (c) N1 = N2
is assumed to be at the cor- (d) N1 and N2 would be in the vertical directions
ner of the book. Think of an
7. Two persons of equal height are carrying a long, uni-
axis that is passing through
form wooden beam of length
the grip point and is perpen-
l. They are at a distance l/4
dicular to the plane of the fig-
and l/6 from the nearest ends
ure. The person is producing
of the rod. The ratio of normal
a torque on the book about
reactions at their heads is
this axis, which is equal to:

4.18
(a) 2 : 3 (b) 1 : 3 12. A uniform rod AB of
(c) 4 : 3 (d) 1 : 2 weight W is movable in
a vertical plane about a
8. An inverted V is made up of two uniform boards,
smooth hinge at A, and is
each weighing 200 N. Each side
kept in equilibrium by a
has the same length and makes
force P acting on a string
an angle 30° with the vertical, as
BCP passing over a smooth peg C, as shown. AC is
shown in the figure. What is the
vertical. If AC is equal to AB, then the force P is
magnitude of the static frictional
force that acts on each of the (a) W/cos q (b) Wcos q
lower end of the V ? (c) W/sin q (d) Wsin q
50 200 13. A right triangular plate ABC
(a) ​ ___
__  ​ N (b) ​ ____ __ ​ N
÷   
​ 3 ​ ÷
​ 3 ​   of mass m is free to rotate
100
____ 60
___ in the vertical plane about a
(c) ​  __ ​ N (d) ​  __  ​ N
÷
​ 3 ​   ÷   
​ 3 ​ fixed horizontal axis through
9. Two identical bricks of length L are piled one on A. It is supported by a string
top of the other, on a table, such that the side AB is hori-
as shown in the figure. The zontal. The reaction at the
maximum distance S that the support A in equilibrium is:
top brick can overhang the mg 2 mg
table with the system still (a) ​ ___ ​   (b) ​ ____  ​  
3 3
balanced is: mg
1 2 (c) ​ ___ ​   (d) mg
(a) ​ __ ​  L (b) ​ __ ​  L 2
2 3
14. A thin hoop of weight
3 7
(c) ​ __ ​  L (d) ​ __ ​  L 500 N and radius 1 m rests
4 8
on a rough inclined plane,
10. An automobile of weight W is standing on a smooth
as shown in the figure. The
road as shown. Distance between the front wheels
minimum coefficient of fric-
and the rear wheels is 2a and the height of COM
tion needed for this configu-
is b from the road. A
ration is
horizontal pull force P is
1__ 1__
applied on the automobile (a) ​ ____    ​   (b) ​ ___  ​ 
with its line of action pass- 3​÷3 ​    ÷   
​ 3 ​
1 1__
ing through the COM. The (c) ​ __ ​   (d) ​ ____   ​ 
reaction at the front wheel 2   
2​÷3 ​
(A) is 15. Same number of books are placed in four book
(a) (W/2) – (Pb/2a) (b) (W/2) + (Pb/2a) cases, as shown. Which bookcase is most likely to
topple forward if pulled a little at the top towards the
(c) (W/2) – (Pa/2b) (d) None of these
right?
11. A uniform disk of radius R and mass m is connected
to a wall by string of length 2R. The normal reaction
of wall is

(a) (b)

mg (c) (d)
(a) mg (b) ​ ___ ​

2
mg
(c) ​ ___
__ ​   (d) 2 mg
÷   
​ 3 ​

4.19
16. A uniform cube of side ‘b’ and mass M rests on a 19. A uniform cylinder of mass M lies on a fixed plane
rough horizontal table. A horizontal force F is applied inclined at an angle q with horizontal. A light string
normal to one of the faces is tied to the cylinder at the right-most point, and a
at a point, at a height 3b/4 mass m hangs from the string, as shown. Assume
above the base. What should that the coefficient of friction between the cylinder
be the coefficient of friction and the inclined plane is sufficiently large to prevent
(m) between the cube and slipping. For the cylinder to the remain static, the
the table so that is will tip value of m is
about an edge before it starts slipping?
2 1
(a) m > __
​    ​ (b) m > __
​    ​
3 3
3
(c) m > __
​    ​ (d) none
2
17. A homogeneous cubical brick lies

( 
motionless on a rough inclined
surface. The half of the brick,
which applies greater pressure on

sin q
(a) M ​ ​ ________
  
1 – sin q )
 ​  ​
M cos q
(b) ​ ________ 

1 + sin q
 ​

the plane is: M sin q M cos q


(c) ​ ________ 
  ​ (d) ​ ________ 
  ​
(a) left half 1 + sin q 1 – sin q
(b) right half 20. A cylinder of radius R and weight W is pulled against
(c) both apply equal pressure a step of height h (< R) by applying a horizontal force
at its centre, as shown in the figure. The cylinder does
(d) the answer depends upon coefficient of
not move.
friction
18. A tight-rope walker in a circus holds a long, flexible
pole to help stay balanced on the rope. Holding the
pole horizontally and perpendicular to the rope helps
the performer

(a) Maximum possible value of applied force is


________
  – h  2 
​÷2Rh
F = W ​ _________
 ​
   ​
R–h
(b) The cylinder cannot remain in rotational
equilibrium
(a) by lowering the overall centre of gravity
(c) Whatever be the value of F, the cylinder cannot
(b) by increasing the mass
move.
(c) in the ability to adjust the centre of gravity, to
(d) None of the above.
stay on the rope
(d) in providing something to hold

4.20
Worksheet 2
1. Two uniform, equal ladders AB and AC, each of
weight ‘W’ lean against each other and a string is tied
between E and F. They stand on a smooth horizontal
surface. Then:

(a) Maximum ratio m/M for which the cylinder will


not tilt is 1/4.
(b) As m is increased (and the cylinder does not
move) the line of normal force acting on the
cylinder moves to the right.
(c) As m is increased (and the cylinder does not
(a) the force exerted by one rod on the other at A move), the line of normal force acting on the
is equal in magnitude to the tension T in the cylinder moves to left.
string (d) None of the above.
(b) tension T = (W/2) cot q 3. A body is in equilibrium under the influence of a
(c) the normal reaction at B and C are equal number of forces. Each force has a different line of
action. The minimum number of forces required is
(d) the normal reaction at B or C is greater than
‘W’ (a) 2, if their lines of action pass through the centre
of mass of the body.
2. A cylinder of height h, diameter h/2 and mass M
is placed on a horizontal table. One end of a string (b) 3, if their lines of action are not parallel.
running over a pulley is fastened to the top of the cyl- (c) 3, if their lines of action are parallel.
inder; a body of mass m is hung from the other end (d) 4, if their lines of action are parallel and all the
and the system is released. Friction is large enough forces have the same magnitude.
to prevent slipping of M.

4.21
Worksheet 3
1. Four masses are suspended from a ceiling using
massless rods, as shown in figure. All rods are hori-
zontal and extend three times as far to the right of
the wire supporting it, as to the left. Find mass m1
if m4 = 96 kg.

6. Two cylindrical rollers of radii R and r rest on a


rough horizontal surface as shown. The larger roller
has a light string tightly wrapped around it and the
end of the string is being pulled horizontally with a
force F. coefficient of
F
friction at each contact
2. A uniform rod leans is m. Find the smallest
against a vertical wall value of m so that the
with its lower end smaller roller remains
on a smooth incline, in equilibrium.
as shown. The rod is 7. AB is a uniform rod of length L. Its end A lies
perpendicular to the on a rough ground and a smooth roller supports it
incline and inclination at C. Height of C above the ground is h. Find the
angle of the incline is friction coefficient
q. Find minimum coef- between end A and
ficient of friction between the rod and the incline so the ground, if the
that the rod does not slip. Given, q = 45°. smallest angle that
3. A rod in L-shape the rod can make
has two arms, hav- with horizontal in
ing lengths L and 2L. equilibrium is q.
It rests on a peg, as 8. A uniform disc-shaped pulley can rotate freely about
shown. Find angle q an axis through its centre O. A thread is wrapped
in equilibrium. Both tightly on the pulley and
arms have same linear density. supports a mass M at its
4. A uniform bar AB of length L is suspended by two free end. A small particle
light strings as shown. The string tied at end B makes of mass m is stuck at the
an angle a = 30° with the horizontal. Another string circumference of the pulley.
L The system stays in equilib-
is tied at point C, which is at a distance __
​   ​  from end
4 rium when the radius to m
A. This string makes an angle b in equilibrium if the makes an angle q with the
m
bar is horizontal. Find b. vertical as shown. Find __​   ​. 
M
9. A hemisphere
of mass M is
being pulled
by a constant
horizontal force
F, as shown
in the figure.
5. Five identical bricks, each of length 5 cm are arranged The hemisphere
as shown. Find maximum value of x so that the moves with constant velocity while maintaining an
system is in equilibrium. inclination angle q between the horizontal and its
circular base. Find the coefficient of friction between
the hemisphere and the floor.

4.22
Answers Sheet
Your Turn
__
1. 6 N-m     N-m ( )
2. 10​÷3 ​ 4. Zero
5. 60 N-m along negative z-direction. 6. mu2 sin q . cos q 7. 3
5
8. (i) 275 N (ii) 600 N 9. Tension = 292 N, FH = 234 N ; Fv = 25.2 N 10. ​ __ ​  m
3
11. 87.3 cm 13. Truck having its CG at A
14. yes 15. 1 16. In balance

Worksheet 1
1. (c) 2. (c) 3. (b) 4. (d) 5. (c) 6. (c) 7. (c) 8. (c) 9. (c)
10. (d) 11. (c) 12. (b) 13. (b) 14. (d) 15. (c) 16. (a) 17. (a) 18. (a)
19. (a) 20. (a)

Worksheet 2
1. (a,c) 2. (a,b) 3. (b,c,d)

Worksheet 3
1. 2 kg
1
2. ​ __  ​
3
3. tan–14
2
4. tan–1 ​ ___ (  )
​  __  ​  ​
÷   
​ 3 ​

÷ 
__
5 r L cos q ◊ sin2q
5. ​ __  ​ cm 6. m = ​ __
​    ​ ​   ________________
7. m ≥ ​         ​
6 R 2h – L cos2 q ◊ sin q
1 3 sin q
8. ​ ____    ​   9. m = __________
​   
  
sin q 8 (1 – sin q)

4.23
Chapter  5

Kinematics of Rotation
“We live-on a spinning planet in a world of spin.’’
–Christopher Buckley

1.  Introduction each particle is the same, though their speeds may not be the
same. The figure shows a rotating body. As point P1 moves
In a rotating body, all the particles do not move identi- to P2 and rotates through an angle q, any other point (like
cally. Description of motion is quite different from that of Q) in the body also moves through the same angle q. We
translational motion. say that the body has rotated by an angle q.
The centre of mass of a body will play a pivotal role in Obviously, the angular velocity (w) and angular
understanding the general motion of a body. acceleration (a) are also same for all particles in the body.
We will consider pure rotation of rigid bodies, as well as Angular velocity is a vector. Its direction is along the
rotation and translation, with a major thrust on understanding rotation axis. Curl your right hand fingers in the direction
rolling motion. In this chapter, we will discuss the basic con- of rotation (trying to hold the rotation axis) and _​_› the thumb
cepts related to kinematics of rotating bodies and then take gives the direction of angular velocity vector, w​ ​   .
​_› ​__›
up the dynamics parts in the next chapter. You are advised to If w is increasing then ​a   ​ is directed parallel to w​ ​    and the
quickly revise the chapter on Kinematics of Circular Motion two vectors are oppositely directed if w is decreasing.
before you move ahead in this chapter.
For a particle in the body, rotating in a circle of radius r,
the components of acceleration are:
2.  KINEMATICS OF PURE ROTATION ar = w2r [Radial acceleration]
A body is said to be in pure rotation if it is rotating about dw
at = r ​ ___ ​ = ra [Tangential acceleration]
a fixed axis. The axis may pass through the body or may dt
lie completely outside it. When w is constant, a is zero.

Example 1   Two wheels can rotate about their fixed


axles. The wheels have radii R and r and they rotate with
their edges touching. There is no slipping between the
wheels. Find the ratio of mag-
nitudes of acceleration of points
on the circumference of the two
wheels. Assume that the wheels O1 O2
rotate with uniform angular
speeds.
Solution
Concepts
(i) No slipping implies that the speed of particles on
the circumference of the wheels is same. This gives
In such motion, all particles move in a circle, with their us the ratio of angular speeds.
centre lying on the rotation axis. Angular displacement of
(ii) Acceleration of a particle on circumference = w2r
5.2  Mechanics II

Solution
Concepts
The phrase ‘thread is tightly wrapped’ means that the
thread will not slip on the pulley.
This implies that speed of a point on the thread is
same as the speed of a point on the circumference of the
pulley.
For no slipping, speed of point
A = speed of point B
fi w1R = w2r
w1 r
fi ​ ___ ​  = __
​    ​
w2 R
w21R
(  )
aA r 2 R __ r
\ ​ ___
aB  ​ =
____
​  2  ​ = ​​ __
​    ​  ​​ ​ __
​ r ​  = ​    
w2r R R

Example 2   Mass suspended to a thread wrapped around
a pulley
A block is hanging from a thread, which is wrapped tightly
around a pulley of radius R. (i) Speed of block (v) = Speed of points A, B, C or D
(i) Find the angular speed of the pulley if the block is on the thread
moving down at a speed v. Speed of point C (or D) = speed of point on the
(ii) Find the angular acceleration of the pulley if the circumference of the pulley = wR
block is moving down with an acceleration a. v
\ wR = v  fi  w = __
​    ​
R
(ii) wR = v

O R
dw
fi ​ ___
dt (  ) dv
​   ​  ​ R = ___
​   ​   fi  a ◊ R = a
dt

a dw
fi a = ​ __ ​   ​ ___ ​ = angular acceleration
R dt
of the pulley

Note:  One must note that if the pulley rotates through an


angle q, a thread of length x = Rq will unwind out of it and
the block will descend through distance x.

Your Turn

Q.1  A fan having diameter of 36 in is rotating uniformly


at an angular speed of w = 20 rad s–1. Find acceleration of
a point at the tip of the blade.
Q.2  The diagram shows two pulleys, which can rotate
about their central axes. Radii of the pulleys are 0.2 m and
0.1 m. A thread is tightly wrapped on the bigger pulley. It
passes over the smaller pulley and its free-end carries a load.
The load is moving down at a speed of v = 10  ms–1. Find
Q.3  In example 2, how far does the block fall if the pul-
the angular speed of the two pulleys. Assume that the thread
ley makes two complete turns?
does not slip on the smaller pulley.
Kinematics of Rotation  5.3

3. KINEMATICS OF ROTATION PLUS Example 3   A metre stick is tossed in air. At a cer-
tain instant, the stick is horizon-
TRANSLATION tal and its two end points have
When you toss a stick in air, its motion is quite complicated. velocities vA = 6 ms–1 upwards and
It is rotating as well as translating. Similarly, a spinning vB = 4 ms–1 downwards. Find the
cricket ball is rotating about an axis and the axis itself is velocity of COM of the stick and
translating. Any general motion of a rigid body involves its angular speed.
both rotation and translation. Description of such a motion Solution
becomes easier with the help of Chasle’s theorem.
Concepts
This theorem asserts that it is always possible to represent
an arbitrary displacement of a rigid body by a translation of (i) If all points in a body have same velocity, it implies
its centre of mass plus a rotation about its centre of mass. that there is no rotation. In this question, velocities
When you toss a stick in air, its COM translates along a of points A and B are different. It implies that there
parabolic path and all particles of the stick rotate about the is rotation.
COM with the same angular velocity (w). (ii) We will assume that COM is translating and the
body is rotating about its COM. We will take
velocity of COM (vCM) and angular velocity (w)
as two unknowns.
By writing the velocities of end points (A and B) using
equation (1), we will create two equations.

If we wish to write velocity of a point (like A) on the


stick, we can do it in two steps. First we write the velocity
of the point relative to COM. Relative to COM, all particles
are going in circles with angular speed w. Velocity of point
A relative to COM is written as
vACM = wr
where
​_›
r is the distance of A from the COM. Direction of
​v  Acm
 ​ is normal to r.
_
​›
_​In second step, we add the velocity of COM (= ​v  ​ cm)

to ​v   ​Acm to get velocity of point A.
​_› ​_› ​_›
​v  ​A  = ​v  ​ACM
  + ​v  ​CM
  ...(1)

Let the stick rotate about its COM with angular speed w,
and velocity of COM be v0, as shown. In COM frame, speed
of both A and B is wr as shown in the second figure.
1
Here r = __
​   ​  m
2
\ vA = wr + v0 (≠)


1
(  )
6 = w ​ __
​   ​   ​ + v0
2
...(i)

Similarly, for writing the acceleration of a point like A, And vB = wr – v0 (Ø)

(  )
we first write its acceleration relative to COM and then add 1
the acceleration of COM to it. 4 = w ​ __
​   ​   ​ – v0 ...(ii)
2
​_› ​_› ​_›
​a  ​A  = ​a  ​ACM
  + ​a  ​CM
  ...(2) Adding (i) and (ii) gives
5.4  Mechanics II

w = 10 rad s–1 Velocity of point A relative to


–1 point B is
Now, (i) gives v0 = 1 ms
Alternate Solution vAB = 10 ms–1 (≠).
Concepts This velocity is perpendicular to AB.
(i) If angular speed of a body is w about its COM, \  Angular speed of A relative to B is
then it has same angular speed (w) about any other vAB 10
point in the body. w = ___
​    ​ = ___
​   ​ = 10 rad s–1
AB 1
(ii) We can easily write the velocity of A relative to B
and then angular speed of A relative to B can be And this is the angular speed of the body about any point
vAB like, A or C. Now, to get the velocity of COM(v0), we again
written as w = ​ ___  ​. If needed, revisit the concepts go back to equation (i) in previous method and write
AB

(  )
learnt in the chapter of Kinematics of Circular
1
Motion 6 = v0 + w ​ __
​   ​   ​  fi  v0 = 1 ms–1.
2

Your Turn

Q.4  A uniform stick moving in air has its COM moving Q.5  A uniform stick is tossed in air. It moves under grav-
–1
horizontally at a velocity of 2 ms . At the instant one of its ity while rotating with constant angular speed w = 4 rad s–1.
ends is at rest, find the velocity of the other end. Find the acceleration of its top point when the stick is
vertical. length of the stick is 2 m.

3.1  Instantaneous Centre of Rotation Solution


A rigid body is said to be undergoing plane motion if a Concepts
particle in the body moves in a fixed plane. In the example Instantaneous centre is the point about which each and
given above (and also in the problems you solved just now), every particle in the stick is rotating in circle. If C is the
any particle in the stick moves in a fixed vertical plane. The centre of rotation, then vA is perpendicular to AC and vB
stick is undergoing a plane motion. In other words, you can is perpendicular to BC.
say that in a plane motion, the orientation of the axis of
This implies that the centre of rotation is somewhere
rotation of the body does not change. In the above example,
on the stick AB.
the stick keeps rotating about its COM and rotation axis is
horizontal, perpendicular to the plane of the figure. As the
stick moves, the position of axis changes but its direction
remains same.
When a body is undergoing a plane motion, we can find
an axis inside or outside the body about which the body–
can be considered to be in pure rotation. This axis is known
as instantaneous axis of rotation. The axis may itself keep
changing its position. Let centre of rotation (C) be at a distance x from A and
(1 – x) from B.
When the body is two dimensional and undergoing plane
motion, we can find a point– inside or outside the body about vA = w . x
which the body can be considered to be in pure rotation. fi 6 = wx ...(1)
This point is instantaneous centre of rotation.
And vB = w (1 – x)
 Example 4   In example 3, find the position of instantaneous fi 4 = w (1 – x) ...(ii)
centre of rotation of the stick at the instant shown.
Solving (i) and (ii)
x = 0.6 m  and  w = 10 rad s–1
Kinematics of Rotation  5.5

This means that all particles in the rod are going in a


circle with angular speed w = 10 rad s–1, with the centre of
each circle being at C. Point C itself has zero velocity.

Example 5   Instantaneous centre of a sliding stick


A stick AB is leaning
against a wall. It begins
to slide. Find the position
co-ordinates of its instan-
taneous centre of rotation
​_› ​_› ​_› ​_›
at the instant length OA ​v  ​ A and ​v  ​ B are shown. Lines perpendicular to ​v  ​ A and ​v  ​ B
and OB both are equal intersect at C. Thus, C is the centre of rotation.
to L. All particles in the rod are rotating about C at this instant
Solution [At a later time, centre of rotation will move to a different
point.] From Fig., co-ordinates of C are (L, L).
Concepts
(i) Velocity of end A is vertically downward and the Note:  If speed of end A is v, then angular speed of the
velocity of end B is horizontally rightward. rod about C is
(ii) Instantaneous centre is on a line that is perpendicular vA v
_
​› _
​›
to ​v   ​A. It is also on a line that is perpendicular to ​v   ​B. w = ___ ​ = __
​      ​    ​
CA L
Instantaneous centre is always on a line that is per-
pendicular to velocity of any point in the stick.

Your Turn

Q.6  A stick is moving in air. Ends A and B have velocities (i) Find the location of instantaneous centre of
as shown. Length of the stick is L. rotation.
(ii) Find angular velocity of the rod.

​_› ​_› ​_›


3.2  Rolling ​v  ​P  = ​v  ​PCM
  + ​v  ​ 
​_›
Rolling is possibly the most familiar kind of motion of a ​v  ​ PCM is velocity of a point P wrt COM
rigid body, which can be described as a combination of
translation and rotation. Motion of a motorcycle wheel can
be described as superposition of two independent motions:
(i) Translatory motion of the centre of mass
(ii) Rotation about the COM
An observer located at the centre of the wheel will find the
wheel to be rotating with an angular velocity (w). Velocity
of different points on the circumference of the wheel appear
to this observer as shown in fig. (a).
Velocity of any point (Like A, B, C or D) relative to
ground is obtained by adding the velocity of COM to the
velocity shown in fig (a). Velocity of any point (P) relative
to ground is given by
5.6  Mechanics II

Figure (c) shows the veloc-


ity of different points as a
combination of velocity of
centre (= v) and the veloc-
ity due to rotation relative to
centre.
Obviously, the speed of top
point A is maximum and equal
to vA = v + wR. Speed of the
lowest point C is the smallest
and equal to vc = |v – wR|.
If v > wR, velocity of con-
tact point C of the wheel is
forward. It is said to be for- In this model, velocity of any point, like B is perpendicu-
ward slipping on the ground. lar to CB and has magnitude wr where r = CB.
If wR > v, v e locity of Velocity of centre O is v = wR
contact point C is backward. Velocity of top point A is vA = w(2R) = 2wr = 2v
We say that the wheel is back- Velocity of a point written in this fashion is same as the
ward slipping on the ground. velocity of a point predicted by considering rolling as a
However, the most inter- combination of translation and rotation.
esting situation arises when It is important to note that in case of rolling with sliding,
v = wR. In this case, velocity the contact point cannot be regarded as instantaneous
of the contact point is zero. centre.
The contact point is not slid-
ing (rubbing) on the ground. Example 6   A wheel is rolling without sliding on a
We say that the wheel is roll- horizontal surface with its centre moving at a speed v. Find
ing without sliding or it is the speed of point P shown in figure.
performing pure rolling.
Velocities of different points on the wheel, when it is
rolling without sliding, have been shown in the figure.

Solution
Concepts
(i) In rolling without sliding, wR = v
3.2.1 Instantaneous axis of rotation in rolling
(ii) Relative to centre, velocity of point P is tangential
without sliding and equal to wR = v. We add to this the velocity
We have learned that rolling motion is a combination of (v) of the centre and get velocity of point P.
translation of COM and rotation about it. In case of roll- (iii) We can also think that the wheel is instantaneously
ing without sliding, we can view the motion in an entirely in pure rotation about the point of contact. Speed
different way. of point P is = wr, where r = distance of P from
The point of contact (C) is instantaneously at rest. An axis the point of contact.
through C, perpendicular to the plane of the figure can be
regarded as instantaneous axis of rotation for the body. At Velocity of P wrt centre is wr = v in a direction making
each instant, there is a new point of contact and therefore, a q with horizontal. We add velocity of centre v (Æ) to it.
q
new axis of rotation; but instantaneously, the motion of the Resultant velocity vp is directed making an angle __​   ​  with
entire body can be regarded simply as rotation about C. 2
horizontal and has magnitude
Kinematics of Rotation  5.7

Solution
Concepts
(i) Wheel is rotating about its centre at angular speed
v
w = __
​   ​ 
R
(ii) We will write displacement of point A relative
to centre and then add the displacement of the
centre to get the displacement of A relative to the
ground.
_______________ __________
vp = ​÷v  2 + v2 + 2v v cos q    + cos q) ​
   ​ = v ​÷2 (1   

q
= 2v cos ​ __ ​ 
2
Alternate:
We can consider the wheel to be rotating about its instan-
taneous centre C.
p – q __ p q
In figure: a = ​ _____  = ​   ​  – __
 ​  ​   ​ 
2 2 2

( 
Relative to the centre, the point A rotates to position A1

p q
CQ = R sin a = R sin ​ __
​   ​  – __
2 2 ) q
​   ​   ​ = R cos ​ __ ​ 
2
in time t.
v
q q = wt = __
​   ​  t
\ CP = 2R cos ​ __ ​  R
2
Displacement of A in x-direction is
v q q
\ vP = w (CP) = __
​     ​◊ 2R cos ​ __ ​  = 2v cos ​ __ ​ 
R 2 2 = BA1 = R sin q
Direction of vp is normal to CP. Displacement of A in negative y-direction is

= AB = R (1 – cos q)

In time t, the centre moves along x-direction by a


distance = vt.
\  Displacement of A wrt ground in x-direction
= vt + R sin q
Displacement of A wrt ground in negative y-direction is
= R (1 – cos q)
\  Co-ordinates of A, at time t (in ground frame), are:
Example 7   Path of a point in a rolling wheel
A wheel is rolling with-
out sliding on a horizontal

vt
x = vt + R sin q = vt + R sin ​ __
R (  )
​   ​   ​ ...(i)

ground. Its centre is moving and y = 2R – R (1 – cos q)

[ 
at constant velocity v. In a
co-ordinate system fixed to
the ground, co-ordinates of
vt
= R (1 + cos q) = R ​ 1 + cos ​ __
​   ​   ​  ​
R (  ) ] ...(ii)

top point A are (0, 2R) at Note: Equation (i) and (ii) together represent a curve
time t = 0. Write the co- known as cycloid. Path of point A (or any other point on
ordinates of this point at the circumference) is a cycloid. It has been represented in
any later time t. the figure given below.
5.8  Mechanics II

Cycloid Concepts
A
(i) For pure rolling, v = wR condition must always
be true. If v is increasing due to acceleration, w
must also increase. It means that there must be an
angular acceleration (a) of the wheel as it rotates
Path of a point on the circumference of the wheel is a cycloid.
about its centre.
v = wR
Example 8   Acceleration of the contact point
dv dw
A wheel of radius R is rolling without sliding on a horizontal fi ​ ___ ​ =   R ​ ___ ​   fi  a = R a
dt dt
surface with its centre moving uniformly at speed v. Find
the acceleration of the contact point. (ii) A point on the wheel appears to be in non-uniform
circular motion for an observer, who is standing at
Solution its centre.
Concepts (iii) We will find acceleration of A relative to the centre
We will write acceleration of the contact point wrt COM. and then add the acceleration of centre to get the
Then we will add the acceleration of COM to it. resultant acceleration of A.
This particular question is quite easy, as COM has no
acceleration.

In a reference frame attached to the centre, the whole


Acceleration of point A has radial and tangential compo-
wheel is just rotating with angular speed
nents in a reference frame attached to the centre.
v
w = ​ __ ​  at = R a
R

(  )
Acceleration of point C relative to COM is v0 2
v2 ar = w2R = ​​ __
​   ​   ​​ ​ ◊ R
acr = w2R = __
​   ​  (≠) R
R
v20
\  Actual acceleration of point C is = __
​   ​ 
R
v2
ac = acr + aCM = __
(  )
​   ​  (≠) since aCM = 0
R ​_› v20
\ ​a  ​ ac = (R a)  + ​ __
​   ​   ​ 
Note:  The contact point C is instantaneously at rest but its R
acceleration is not zero.
Acceleration of C is aC = a0 

(  )
Example 9   Rolling with acceleration
​_› ​_› ​_› v02
A wheel of radius R is rolling without sliding on a horizontal \ ​a  ​ A = ​a  ​ AC + ​a  ​ C = (R a + a0)  + ​ __
​   ​   ​ 
R
surface. Its centre has a constant horizontal acceleration a0.
Find the acceleration of
the top point A at the
instant speed of the cen-
= 2a0  + ​ __
v02
R (  )
​   ​   ​  ​  ​


[ 
Since a0 =  R a for
   
pure rolling ​
​  ​ ]
tre of the wheel is v0. Magnitude of this acceleration is

÷ 
____________

(  )
Solution v02 2
__
aA = ​ (2a0)2 + ​​   
​   ​   ​​ ​ ​
R
Kinematics of Rotation  5.9

Your Turn

Q.7  A disc is rolling on a flat ground. Its angular velocity Q.9  In the last problem, find the acceleration of point A
is w = 2 rad s–1 and velocity of its centre is 3 ms–1. Radius if the wheel is moving with a constant velocity v.
of the disc is 0.5 m. Is the disc slipping? Find the velocity
of contact point of the disc. Q.10  A wheel is rolling uniformly with its centre moving
at speed v.
Q.8  A disc is rolling without sliding with velocity (i) Find the speed of point P that lies on the circumfer-
v = 4 ms–1. Radius of the disc is R = 1.0 m. Find speed of
ence on horizontal radius.
points A, B and C shown in figure.
(ii) Which point – P or Q – is moving faster at the instant
OA = OB = OC = 0.5 m shown?

In Short: (v) Any general motion of a rigid body can be under-


(i) In a rigid body, the distance stood as translation of its centre of mass superim-
between two points never posed with rotation about the centre of mass. Velocity
changes. Velocity compo- and acceleration of a point in the body can be written
nent of a point A along using this approach as
AB = velocity component _
​› _
​› _
​›
​v  ​ P = ​v  ​ PCM + ​v  ​ CM  and
of point B along AB. ​_› _
​› ​_›
fi  vA cos a = vB cos b ​a  ​ p = ​a ​  PCM + ​a  ​ CM
(ii) In pure rotation, the rotation axis remains fixed and (vi) A rolling wheel is a typical example where we under-
all particles of the body go in circle about the fixed stand the motion as the rotation of the wheel about
axis. Speed of any particles is given by v = wr, where its centre plus the translational motion of its centre.
r is distance of the particles from the axis. (vii) When the contact point of a wheel is at rest (i.e.,
w is same for all particles. does not rub the ground) we say it is rolling without
(iii) In pure rotation, if the angular speed of the body sliding (or pure rolling). This happens when v = wR,
changes, then there is angular acceleration where v is velocity of the centre and w is the angular
speed of the wheel about the centre.
dw
a = ___ ​   ​    If the wheel is accelerated, then a = Ra must
dt
also be true for pure rolling to sustain. Here, a is
Now, particles in the body are performing non-uniform
the acceleration of the centre and a is the angular
circular motion.
acceleration of the wheel about the centre.
(iv) In case of a thread passing over a pulley (or wrapped
(viii) When v π wR, the contact point slides on the ground.
on a pulley) and not slipping on the pulley, speed of
It is rolling with sliding or impure rolling.
the thread (v) is related to angular speed of pulley
(w) as v = wR. R is radius of the pulley. If the thread (ix) A plane motion of a rigid body can also be under-
is accelerated, then a = Ra stood as rotation about a certain axis known as
instantaneous axis of rotation.
Where a = angular acceleration of the pulley and a
is rate of change of speed of the thread. (x) For a ball rolling without sliding, the instantaneous
axis passes through its point of contact with the
ground.
5.10  Mechanics II

Miscellaneous Examples
Example 11   A rod of length L is B
hinged at its end A. It begins to fall
from a vertical position, rotating about L
A. Find acceleration of its end B, at the
instant its angular velocity and angular A
acceleration are w and a respectively.
Solution
Concepts
Velocity of B perpendicular to AB is
(i) Point B is rotating in a circle of radius L, with its
3
centre at A. v2 sin q = 12.5 × __
​   ​  = 7.5 ms–1
5
(ii) Motion of B about A is non-uniform circular motion.
\  Angular velocity of B relative to A is
Point B has a radial and a tangential acceleration.
7.5 7.5
w = ___
​    ​ = ___
​   ​ = 15 rad s–1
ar = w2L AB 0.5
at = La Example 13   A wheel of radius R is rolling without sliding
Acceleration of point B has on a horizontal surface with velocity v. As discussed in
magnitude example 7, the path of a particle on its circumference is
______
a cycloid. Find the radius of curvature of this path at the
a = ​÷a  r2 + at2 ​  top-most point.
____________
  2L)2 + (La)
= ​÷(w 2
    ​ Solution
Concepts
It makes an angle q with the rod such that
(i) If you look at the path drawn in example 7, you
at ___ a
tan q = ​ __
ar ​ = ​ w2  ​ 
will get convinced that radius of curvature is cer-
tainly larger that the radius (R) of the wheel.
Example 12   A flat, rigid body is moving in the plane of (ii) We can write the acceleration of the top point and
the figure. At an instant, veloc- its velocity is also known. If radius of curvature
ity of its two points A and B are of the path is r, then radial acceleration of the top
as shown. Velocity of point A is vA2
point (A) can be written as ar = __​    ​.
v1 = 10 ms–1 along AB and veloc- 2r
ity of point B is v2 = 12.5 ms–1.
Find the angular speed of the
body. Length AB = 0.5 m.
Solution
Concepts
(i) Velocity component of B along AB must be equal
to v1. This is necessary as distance between two
points in a rigid body cannot change.
(ii) We will find angular speed of point B wrt A. This
will be our answer, as a rigid body has the same
angular speed about any point in the body. If RF attached to the centre, velocity of point A is wR = v.
v2
For length AB to remain unchanged, v2 cos q = v1 Acceleration of A in this frame is aACM = w2R = __ ​   ​  (Ø).
R
fi 12.5 cos q = 10 If the wheel is rolling uniformly, its COM has no
4 acceleration. It means that acceleration of point A in ground
fi cos q = __
​   ​  v2
5 frame is also ​ __ ​  (Ø). But speed of A in ground frame is
R
Kinematics of Rotation  5.11

v + wR = 2v. Thus, in ground frame, acceleration of point


v2
A is aA = __​   ​  (Ø) and its velocity is vA = 2v (Æ).
R
If radius of curvature is r (i.e.,
we think as if point A is going in
a circle of radius r at this instant),
then
vA2
___
​  r ​   = aA

(2v)2 __ v2
fi ​ _____
r   

​ = ​   ​   fi  r = 4R
R

Example 14   Pure rolling on a moving surface


A plank is moving horizontally
with a velocity v1. A cylinder of
radius R, having angular speed w,
is on the plank. Find the velocity
of the centre of the cylinder if it
does not slip on the plank.
Solution Solution
Concepts Concepts
The point of the cylinder that is touching the plank must (i) B falls by a distance that is equal to the length of
have a velocity v1 (Æ). Thus, velocity of the point of con- thread that unwinds from A plus the length of the
tact relative to the plank will be zero and it will not slide thread that unwinds from B.
(rub) the plank surface. (ii) C falls through a distance that is equal to the
displacement of B plus the length of the thread
Let velocity of centre of the cylinder be v2 (Æ) velocity that unwinds from the sleeve.
of contact point relative to the centre is wR (¨).
\  Velocity of the contact point of the cylinder wrt (i) Let A rotate by q1 and B rotate by q2 in time t. The
ground will be thread that unwinds from A is Rq1 and the thread that
v2 (Æ) + wR (¨) = v2 – wR (Æ) unwinds from B is Rq2.
\ Displacement of B in time t is
For no slipping this should be equal to v1
xB = Rq1 + Rq2
\ v2 – wR = v1  fi  v2 = wR + v1 dxB dq1 dq2
fi ​ ___ ​   = R ​ ___ ​ + R ​ ___ ​ 
dt dt dt
Example 15   Constraint relation among accelerations
fi vB = R w1 + R w2
In the system shown in figure, cylinder A can rotate freely
about its axle. Two threads are wrapped tightly on it at dvB dw1 dw2
symmetric locations. The two threads are also wrapped fi ​ ___ ​   = R ​ ____   
​ + R ​ ____   
​ 
dt dt dt
on another identical cylinder B, as shown. Both cylinders
have radius R. Cylinder B can rotate about its axis and fi aB = Ra1 + Ra2
can also translate. At the centre of B, there is a sleeve of
(ii) Thread that unwinds from the sleeve in time t is
radius 2R. The sleeve is fixed to B and does not slide on
= 2Rq2
B. The sleeve has another thread tightly wrapped on it,
which carries a block C at its free end. At a certain instant, \ Displacement of C in time t is
angular acceleration of cylinders A and B are a1 and a2 xC = xB + 2Rq2
respectively. Differentiating twice wrt time, as above, gives
(i) Find acceleration of axle of cylinder B at this aC = aB + 2Ra2
instant
\ aC = Ra1 + Ra2 + 2Ra2
(ii) Find acceleration of block C at this instant.
= R (a1 + 3a2)
5.12  Mechanics II

_
​›
Example 16   Distance travelled by a point on a rolling \ ​v  ​ AC = – v0 cos q  + v0 sin q 
wheel ​_› ​_› ​_›
A wheel rolls without sliding on a horizontal surface. Its Velocity of A wrt ground is ​v  ​A  = ​v  ​AC
  + ​v  ​C

centre moves uniformly with velocity v0. Consider a particle _


​›
on the circumference. How much distance does this particle fi ​v  ​ A = (v0 – v0 cos q)  + v0 sin q 
travel by the time the wheel makes one complete rotation ____________________
about its centre? Radius of the wheel is R. ​ v  02 (1 – cos q)
Speed of point A is vA = ÷ 2
    + v20 sin2 q ​
Solution _________ __ ________
fi ​= ÷
vA = v0 ​÷2  – 2 cos q   ​
  – cos q  
    v0 ​÷1
​ 2 ​
Concepts
We will write speed (v) of the particle as a function of
time. Distance travelled is
q
2
wt
= 2 v0 sin ​ __ ​  = 2v0 sin ​ ___ (  )
v0t
​   ​   ​ = 2v0 sin ​ ___
2
​    ​  ​
2R (  )
T
2p 2pR
S = ​Ú ​   ​vdt where T = time for one rotation Time for one rotation is T = ___
​   ​ = ​ ____
v0 ​ 

w
0
\  Distance travelled by A in one full rotation is

(  )
T T
v0t
S = Ú​  ​  ​vAdt = 2v0 ​Ú ​    ​ sin ​ ___
​   ​   ​ dt
0 0 dt

2v0
= ___
___0
​    ​ 
[  (  ) ]
v0t T
​  v  ​ ​​  – cos ​ ___
​    ​  ​  ​​ ​ ​
2R 0
2R

Consider a point A at the bottom of the wheel, at t = 0.


In time t, the wheel rotates through an angle q about the
v0 2pR
= – 4R ​ cos ​ ___ [  ( 
​    ​  ◊ ​ ____
2R v0
 ​  
 ​ – cos 0  ​ ) ]
centre, where q = wt. Velocity of point A, relative to centre
(C) is tangential and is equal to wR = v0. = – 4R [cos p – cos 0] = – 4R [– 1 – 1] = 8R
Worksheet 1
1. A rigid body is in rotation about a fixed axis. Which 6. A disc of radius R is rolling purely
of the followings must be true? on a flat horizontal surface, with
(a) No two particles in the body can have the same a constant angular velocity. The
speed angle between the velocity and
(b) No two particles in the body can have the same acceleration vectors of point P is
velocity (a) zero (b) 45°
(c) No two particles can have the same angular (c) 135° (d) tan–1(1/2)
speed 7. A cylinder of radius 5 m rolls on a horizontal surface.
(d) All particles have the same angular acceleration Velocity of its centre
2. A rigid body is in pure rotation. is 25 ms–1. Velocity
of the point A is (in
(a) You can find two points in the body, in a plane
ms–1)
perpendicular to the axis of rotation, having the
same velocity. (a) 40  + 20 
(b) You can find two points in the body, in a plane (b) 20  + 30 
perpendicular to the axis of rotation, having the (c) 20  + 30 
same acceleration. (d) 20  + 30 
(c) Speed of all the particles lying on the curved 8. A rod is pivoted at its one edge about point O. Other
surface of a cylinder whose axis concides with edge of rod is suspended from the ceiling through a
the axis of rotation is same. rope, as shown. If the rope is suddenly cut and its
(d) Relative acceleration of any two points in the initial angular acceleration
body is always zero. is found to be a.
3. Two points of a rigid body are moving as shown. (a) initial acceleration of
The angular velocity of the COM of the rod will be zero
body is: (b) initial acceleration of the COM of the rod will
v be more that of any other point in the rod
(a) ​ ___  ​ 
2R (c) initial acceleration of the COM of the rod will
v be in vertically downward direction.
(b) ​ __ ​ 
R (d) none of the above
2v 9. A chain couples and rotates two wheels in a bicycle.
(c) ​ ___ ​ 
R The radii of bigger and smaller wheels are 0.5 m
2v and 0.2 m respectively. The bigger wheel rotates at
(d) ​ ___ ​  the rate of 200 rotations per minute, then the rate of
3R
4. A ladder of length L is slipping with its ends against rotation of smaller wheel will be
a vertical wall and a horizontal floor. At a certain (a) 1000 rpm (b) 50/3 rpm
moment, the speed of the end in contact with the (c) 200 rmp (d) 500 rpm
horizontal floor is v and the ladder makes an angle 10. In the figure shown, the block of
a = 30° with the horizontal. Then the speed of the mass m is going down with a con-
ladder’s centre must be stant velocity of 2 ms–1. Radius of
__
(a) 2v/​÷3 ​    (b) v/2 the pulley is 0.1 m
(c) v (d) None (a) acceleration of a point on the
5. A sliding solid sphere, on a fixed flat surface, has a circumference of the pulley has
velocity (of centre of mass) v and angular velocity magnitude equal to 40 ms–2
w. Its contact point (b) acceleration of a point on the circumference of
(a) must be moving forward (in direction of v) the pulley is 0

(b) must be moving backward (opposite to v) (c) acceleration of a point on the circumference of
the pulley could be 10 ms–2
(c) cannot be at rest
(d) acceleration of a point on vertical segment of
(d) data is insufficient
the string is 40 ms–2

5.13
Worksheet 2
1. A wheel is rotating with angular velocity 2 rad s–1.
It is subjected to uniform angular acceleration
2.0 rad s–2.
(a) angular velocity of the wheel after 10 s is
22 rad s–1
(b) number of rotations made by the wheel in 10 s
is close to 10
(c) angular velocity of different points will be dif-
ferent in general
(d) linear acceleration of a point in the wheel must
be towards the axis of rotation
2. A ring rolls without slipping on the ground. Its centre (a) a possible value of the (constant) angular speed
C moves with a constant speed u. P is any point on of the wheel is nearly 3.8 × 103 rad s–1?
the ring. The speed of P with respect to the ground (b) a possible linear speed of a point on the edge
is v. of the wheel is 1.9 × 102 ms–1
(a) 0 ≤ v ≤ 2u (c) speed of light is too high for such experiment
(b) v = u, if CP is horizontal to be successful
(c) v = u, is possible if CP makes an angle of 30° (d) None of the above
with the horizontal and P is below the horizontal 5. A 50-cm long rod AB is in combined translation and
level of C. rotation motion on a table. At an instant, velocity
__
(d) v = ÷    u, if CP is horizontal
​ 2 ​ component of point A perpendicular the rod is 10
3. A wheel of radius r is rolling on a straight line, the cm s–1, velocity component of point B parallel to the
velocity of its centre being v. At a certain instant, the rod is 6.0 cm s–1 and the angular velocity of the rod
point of contact of the wheel with the grounds is M is 0.4 rad s–1 in anti-clockwise direction
and N is the highest point on the wheel (diametrically
opposite to M). The correct statements are:
(a) The velocity of any point P of the wheel is
proportional to MP.
(b) Points of the wheel moving with velocity greater
than v form a larger area of the wheel than (a) the x component of velocity of A is 6.0 cm s–1
points moving with velocity less than v. (b) the y component of velocity of B is 10.0
(c) The point of contact M is instantaneously at cm s–1
rest. (c) the instantaneous centre of rotation is located at
(d) The velocities of any two parts of the wheel, a distance of 15 cm from the rod.
which are equidistant from the centre, are (d) None
equal. 6. A round body of radius 10 cm starts rolling, without
4. An early method of measuring the speed of light sliding, on a horizontal stationary surface with uni-
makes use of a rotating slotted wheel. A beam of form angular acceleration 2 rad s–2.
light passes through a slot at the outside edge of the
wheel, as shown in figure below, travels to a distant
mirror, and returns to the wheel just in time to pass
through the next slot in the wheel. One such slotted
wheel has a radius of 5.0 cm and 500 slots at its
edge. Measurements taken when the mirror is at a
distance L = 500 m from the wheel indicate speed
of light to be 3.0 × 105 kms–1.

5.14
(a) initial acceleration of the centre C is 20 cms–2
(b) initial acceleration of the top point A is
20 cms–2

(c) initial acceleration of the top point is
40 cms–2

(d) initial acceleration of the centre C is 0
7. Two identical disks, each of radius r, are connected (a) acceleration of the centre of disk II will be
by a cord, as shown in the figure. The cord wraps 1/2ra
tightly on disc I and does not slip on disc II. disk (b) angular acceleration of disk II will be 1/2a
I rotates with constant angular acceleration a in
(b) angular acceleration of disk II will be a
anti-clockwise direction.
(d) acceleration of disk II is difficult to predict
unless its mass is known.

5.15
Worksheet 3
1. A thread is tightly wrapped on a disc. The free end 3. A spool has outer radius R = 0.2 m. Thread is wound
of the thread is fixed to the ceiling and the disc is tightly on its inner wheel, having radius r = 0.1 m.
allowed to fall. Radius is R = 0.2 m. The spool is kept on a horizontal table and the free
(i) Find the angular speed of the end of the thread is used to suspend a block A, as
disc at the instant its cen- shown. Pulley P is fixed. Find acceleration of the
tre is moving down at speed centre of the spool when acceleration of block A is a.
v = 4 ms–1. It is given that the spool does not slip on the table.
A
(ii) Is there any point in the disc
P
having zero speed at the r
R
instant speed of the centre
–1
is 4 ms ? What is speed of R
the top point of the disc at
this instant?
2. A belt moves on two cylinders without slipping.
Radius of one cylinder is R1 and it is spinning with A
angular velocity w1 about its central axis. Find the
angular speed of the other cylinder, if its radius 4. A stick of length L = 1.0 m is tossed up such that
is R2. its centre moves up vertically. Initially, the stick was
horizontal and it falls back to the projection point
again being horizontal. During its journey, the stick,
made one complete rotation. The total time of flight
of the stick is 2 s. Find the speed of the ends of the
stick 0.5 s after projection. Assume that the angu-
lar speed of the stick does not change during the
flight.

5.16
Answers Sheet
Your Turn
1. 182.9 ms–2 2. 50 rad s–1, 100 rad s–1 3. 4 p R 4. 4 ms–1
5. 14 ms–2 is instantaneous centre such that D ABC is equilateral (ii) v/L
6. (i) C
__
–1
7. Yes, 2 ms in forward direction 8. vA = 6 ms–1 ; vB = 2​÷5 ​ 
   ms–1 ; vC = 2ms–1
__
9. 8 ms–2 (Ø) 10. (i) ÷     v (ii) P
​ 2 ​

Worksheet 1
1. (d) 2. (c) 3. (b) 4. (c) 5. (d) 6. (b) 7. (a) 8. (c) 9. (d)
10. (a)

Worksheet 2
1. (a) 2. (a,c,d) 3. (a,b,c) 4. (a,b) 5. (a,b,c) 6. (a,c) 7. (a,b)

Worksheet 3
__
1. (i) w = 20 rad s–1 (ii) The point of disc in contact with vertical thread has no speed. vA = 4​÷2 ​
    ms–1
w1R1 2
2. ​ _____
 ​
   3. ​ __ ​  a 4. 5.24 ms–1
R2 3

5.17
Chapter  6

Rotational Dynamics
“It’s not magic! It’s physics. The speed of the turn is what keeps you upright.
It’s like a spinning top.’’
–Deborah Bull

1.  Introduction of the masses of the particles


with square of their respective
An unbalanced force produces acceleration in a body and distances from the line. [You
causes its momentum to change. An unbalanced torque will soon realise why it has been
produces angular acceleration and causes the angular defined this way. Right now, just
momentum of a body to change. This is the basic subject learn this definition].
matter of this chapter. We will also learn about rotational XX is a fixed line and particles
kinetic energy. of masses m1, m2, m3 ... are scat-
By the end of this chapter, we will be able to analyse a tered around it. Their distances
plane motion of a rigid body. We will pay special attention from the line are r1, r2, r3 ... Moment of Inertia of this col-
to rolling motion. lection of particles about line XX is
Conservation of angular momentum is a fundamental
conservation principle in physics just like conservation of I = m1r12 + m2r22 + m3r32 + ... = Â miri2 ...(1)
momentum and energy. We will learn this conservation prin-
SI unit of MI is kg m2.
ciple towards the end of the chapter.
If the distribution of mass is con- Axis
tinuous, we consider a small elemen-
2.  MOMENT OF INERTIA tal mass dm in the body. If distance
When a finite force is applied on a body, its acceleration is of this element from the concerned
r
decided by its inertia (mass). Similarly, when a finite torque axis is r, then MI of the element is
dm
acts on a body (about an axis), its angular acceleration is dI = r2 dm
decided by its rotational inertia. Rotational inertia of a body
about an axis is termed as its moment of inertia. MI of the entire body is obtained
by summing the MI of every such
Think of two fans having same mass but different diam-
element.
eters. If you want to spin them with the same angular speed,
the one with the larger diameter will require more effort. If I = Ú r2 dm ...(2)
both of them are rotating with the same angular speed, the Following examples will illustrate the process.
one with the larger diameter will need more effort to bring
it to rest. We say that the fan with larger diameter has more Example 1   Three point-masses, each of mass m, are
rotational inertia (i.e., moment of inertia) than the other placed at the vertices of an equi-
fan. lateral triangle of side length a. A
Rotational inertia of a body depends on mass as well as Find the MI of the system about m
geometry of distribution of mass around the rotation axis. (i) an axis through vertex A
Moment of inertia of a body is always defined with respect perpendicular to the plane a a
to a given rotation axis. If the axis changes, moment of of the figure.
inertia may also change. m m
(ii) line BC. B a C
Moment of inertia (MI) of an n-particle system, about a
fixed line (usually called as axis), is the sum of the products
6.2  Mechanics II

Solution (ii) Different mass-points are at different distances


Concepts from the axis.
(i) A line passing through A is at a perpendicular (iii) We will consider a small element on the rod, write
distance ‘a’ from both B and C. its MI and then integrate to find the MI of the
entire rod.
(ii) A particle that lies on the axis has no contribution
to MI.
Axis
(i) MI about an axis through A and perpendicular to the
plane of the figure is dm
A
x dx
IA = m (0)2 + m (a)2 + m (a)2
= 2ma2 __
  
​÷3 ​
(ii) Distance of A from line BC is r = a sin 60° = ​ ___ ​ a Axis is as shown in figure. Consider a small segment of
2 length dx at a distance x from the axis.
B and C lie on the axis itself.
M
3m Mass of the element, dm = __ ​   ​  ◊ dx
\ I = mr2 = ___
​   ​  a2 L
4
M
A
m MI of the element about axis, dI = x2 dm = __
​   ​  x2 dx
L

r
\  MI of the rod about the axis is
a a

[  ]
L L
M M x3 ML2
B 60° C I = Ú dI = ​ __ ​  ​Ú ​  ​ x2 dx = __
​   ​ ​​  __
​   ​   ​​ ​ ​ = ____
​   ​ 

m D m L 0 L 3 0 3

By taking the limits of integration from x = 0 to x = L,


Example 2   MI of a ring about central axis we have considered all elements in the rod.
Find MI of a ring of mass M and radius R about an axis
passing through its centre and perpendicular to its plane. Example 4   MI of disc about central axis
A uniform disc of mass M has radius R. Find its MI about
Solution an axis passing through its centre and perpendicular to
Concepts its plane.
Every mass point is at a distance R from the axis. Solution
Concepts
I = Ú R2 dm = R2 Ú dm = MR2
(i) We will divide our disc into a series of thin
rings.
(ii) According to the result obtained in example 2, we
R can write MI of such a ring about the given axis.
Integration is used to add MI of all such rings.

The disc and its axis is


Axis
shown in the figure. Mass
Example 3   MI of a thin rod about an axis perpendicular per unit surface area of the
to it through its end disc is R dx
x
A uniform thin rod has mass M and length L. Find its M
MI about an axis that is perpendicular to the rod and passes s = ​ ____2  ​ 
pR
through its end.
Consider a ring of
Solution radius x and width dx in the disc. [i.e., consider the mass
Concepts lying between radius x and x + dx].
(i) ‘Thin rod’ implies that mass can be assumed to be Area of the elemental ring = area of a rectangle of length
distributed on a line. 2p x and width dx
Rotational Dynamics  6.3

Radius of ring, r = R sin q


dx
Width of ring = Rdq
2 px Surface area of the ring

If you cut open the elemental ring and straighten it, dA = (2p r) (Rdq) = 2pR2 sin q ◊ dq
it will appear like a rectangular strip.
M
Mass per unit surface area, s = _____
​   2 ​ 
4pR
fi dA = 2p x dx
M
Mass of ring element is \  Mass of the ring element, dm = s ◊ dA = __ ​   ​  sin q dq
2
M M
dm = s dA = ​ ____ 2 ​  
◊ 2p x dx MI of the ring, dI = (dm)r = __
2 2 3
​   ​  R  sin  q dq
p R 2
p
2M
= ___ MR2 2
​  2 ​  x dx \ I = ​ ____  Ú​  ​  ​sin3 q dq = __
 ​  ​   ​  MR2
R 2 0 3
MI of this ring about the given axis is Take note of the limit of integration.
2
dI = x  ◊ dm  [Refer to example 2]
Example 6   Uniform solid sphere
2M
= ___
​  2 ​ x3 dx A uniform solid sphere has mass M and radius R. Find its
R MI about a diameter.
To get MI of the disc, we need to add MI of all such Solution
rings.
Concepts
[  ]
R R
2M 2M x4
\ ​  2 ​ Ú​  ​    ​x3 dx = ___
I = Ú dI = ___ ​  2 ​ ​​ __
​   ​   ​​ ​ ​ We will consider the sphere to be made up of infinite
R 0 R 4 0
shells, laid one over another.
MR2 To write MI of a shell, we will use the result obtained
fi I = ​ ____
 ​ 

2 in the last example.

Example 5   Uniform spherical shell Consider an infinitesimal shell ele- Axis
A uniform spherical shell has mass M and radius R. Find its ment of inner and outer radii x and
p (x + ) respectively.
4
MI about a diameter. Given, ​Ú  ​ ​sin3 q dq = __
​   ​ . Volume of shell element
0 3 dx
x
Solution = (Surface area) × (thickness)
R
Concepts \ dV = 4p x2 ◊ dx
We will divide the shell into multiple thin rings with their
M
axes along the given diameter. Density of sphere, r = _____
​     ​ 
4
__
​   ​  p R3
3
\  Mass of shell element is

3M 2
dm = r ◊ dV = ​ ___ ​ x  dx
R3
2
MI of the shell about diameter is dI = __
​    ​ (dm) ◊ x2
3
2M
fi dI = ​ ___ 4
 ​ x  dx
R3

(  )
R
2M 2M R5
\ ​  3 ​ ​Ú ​  ​ x4 dx = ___
I = Ú dI = ___ ​  3 ​ ​  ___
​   ​   ​
R 0 R 5
Consider a ring element between angular position q and
2
(q + dq), as shown in figure. \ I = __
​   ​  MR2
5
6.4  Mechanics II

y
2.1  Theorem of perpendicular axis From symmetry: Ix = Iy
perpendicular axis theorem:
This theorem is applicable for a laminar (two-dimensional)
body. The theorem states that: Ix + Iy = Iz
O x
The sum of moments of inertia of a laminar body, about
any two mutually perpendicular axes lying in the plane of the fi 2Ix = MR2
body, is equal to MI about a third axis normal to the plane 1
of the body and passing through the point of intersection of fi Ix = ​ __ ​  MR2
2
the two axes.
z-axis is ^ to the plane
x and y-axes are in the plane of a flat body. MI of the 2.2 Parallel axis theorem passing through O.
body about these two axes is Ix and Iy. This theorem is valid for objects
z-axis is perpendicular to the plane of the body. of all shapes. It states that the
MI about z-axis is MI of a body of mass M about
any axis is equal to ICM + Md2,
Iz = Ix + Iy ...(3)
where ICM is MI of the body
Proof:  Consider the body to be made of a number of about another axis that passes
particles, specified by mass mi and co-ordinates (xi, yi, 0). through COM of the body and
Distance of a particle from z-axis is is parallel to the first axis and d
______
is perpendicular distance between
ri = ​÷x  2i + y2i  ​  the two axes.
I = ICM + Md2 ...(4)
\ Iz = Â mir2i = Â mi (x2i + y2i )
I is MI about axis 1 shown in figure and ICM is MI about
axis 2 that is parallel to 1 and passes through COM.
= Â mi x2i + Â mi y2i
Axis 1 may be completely outside the body.
fi Iz = Iy + Ix  xi = distance of mi from y-xis] Proof:  Consider a rigid body shown in the figure. Our axis
yi = distance of mi from x-axis] is perpendicular to the plane of the figure and passes through
point O (call it z-axis).
z
y

x ¢I
O y mi
ri
xi
yi y ¢I
mi
O O¢
x x
d CM

Example 7   MI of ring about its diameter


A ring has mass M and radius R. Find its MI about a COM of the body is assumed to lie on x-axis, at a dis-
diameter. tance d from O. There is no loss in generality in assuming
the COM to be on x-axis.
Solution
Consider an axis through COM and parallel to the first
Concepts axis (z-axis).
(i) We know that MI of a ring about its central axis Let co-ordinates of a particle of mass mi be (xi¢, yi¢, zi¢) in
perpendicular to its plane is MR2. Let us take this a co-ordinate system having its origin at COM.
axis as z-axis. Distance of the particle from axis through COM will be
__________
(ii) Any two perpendicular diameters will be taken as
x and y-axes.
ri¢ = ÷   i¢)2 + (y  
​ (x 2
i¢)  ​

(iii) Due to symmetry, MI of the ring about any diam- \  MI of the body about this axis will be
eter must be same. ICM = Â mi (r¢i )2 = Â mi ​[ (x¢i )2 + ( ¢)2 ]​ ...(i)

But xi¢ = xi – d  and  yi¢ = yi


Rotational Dynamics  6.5

where (xi, yi, zi) are co-ordinates of mi in co-ordinate system From example 3, MI about an I ICM
with origin at O. MOI about z-axis through O is axis through one end and perpen-
Â mi ri2 = Â mi [(xi)2 + (yi)2]
I = dicular to the rod is

= Â mi [(xi¢ + d)2 + (yi¢)2] ML2 CM


I = ____
​   ​ 

3
= Â mi [(xi¢)2 + (yi¢)2] + [​  Â mi ]​ d2 + 2d  mi x¢i Other axis is parallel to this axis L
I = ICM + Md2 + 0 and passes through COM. 2
The third term is zero, as xi¢ refers to x co-ordinates of
the particle in COM frame. \ I = ICM + Md2 gives
Theorems of parallel axis and perpendicular axis are two
very useful tools, which help us in calculating MI of a body
about some axis if its MI is known about certain other axis.
ML2
​ ____
3
 ​  
L 2
 = ICM + M ​​ __ (  )
​   ​   ​​ ​
2
We need not perform elaborate intergration all the times.
ML2 ____ ML2 ____ML2
fi ICM = ​ ____
 ​ 
 – ​   ​ 
 = ​   ​ 

Example 8   A uniform thin rod has mass M and length 3 4 12
L. Find its MI about an axis that is perpendicular to the rod
and passes through it COM. In Short:
The following table presents the MI of important geometrical
Solution
shapes about various important axes. We have also incorpo-
Concepts rated the logic/steps involved in writing the MI. [All bodies
We will use the result obtained in example 3 and parallel are uniform.]
axis theorem.

Body Axis Moment of Inertia Logic/steps


1. thin rod ML2
(a) through one end perpen- I = ____ Direct integration. See example 3.
​   ​ 

dicular to rod 3
I

(b) through centre perpen- ML2 Parallel axis theorem


ICM = ____
​   ​ 

dicular to rod 12 I ICM
ICM

L
2   I = ICM + Md2
2. Rectangular (a) axis through centre, Ma2 Divide the plate into a number of thin strips.
I = ____
​   ​ 
plate parallel to its width 12 Each strip is like a rod. If masses of these strips
are m1, m2, m3 ..., etc.
a
b m1a2 _____
_____ m3a2
m2a2 _____
I = ​   ​   + ​   ​ 
 + ​   ​ 
 + ...
12 12 12 2
a
= (m1 + m2 + m3 + ...) ​ ___  ​ 
12
Ma2
fi I = ​ ____ ​ 
12

m1 a
m2
m3
6.6  Mechanics II

Mb2
(b) axis through centre, par- I = ____ Can be concluded from above result.
​   ​ 
allel to its length 12
a
b I

(c) axis through centre, M Perpendicular axis theorem


I = ___
​     ​(a2 + b2)
perpendicular to plane of 12 M
the plate Iz = Ix + Iy = ___
​    ​ (a2 + b2)
12
b a
I

z-axis is perpendicular to plane of figure


2
3. Square plate (a) passing through centre, Ma It can be concluded from MI of rectangular
I = ____
​   ​ 
parallel to side 12 plate

(b) passing through centre, Ma2 Put b = a in the formula for rectangular plate
I = ____
​   ​ 

perpendicular to plane 6

4. Rectangular Passing through centre, nor- M Divide the brick into number of thin rectangu-
I = ___
​    ​ (a2 + b2)
brick mal to two parallel surfaces 12 lar plates. If m1, m2, m3 ... are masses of these
slices, then
a
m1 m2
b
I = ​ ___ ​ (a2 + b2) + ___
​   ​ (a2 + b2) + ...
12 12
h
(m1 + m2 + ...) 2
= ____________
​   ​
     (a + b2)
12
M
= ​ ___  ​ (a2 + b2)
12

a m1
b m2
m3

5. Cube Passing through the centre, Ma2 Put b = a in the above expression for rectangular
I = ____
​   ​ 

normal to two parallel faces 6 brick.

a
Rotational Dynamics  6.7

6. Ring (a) passing through the I = MR2 All points are at distance R from the axis.
centre, normal to plane
I

MR2 Any two diameters are similar. Consider two


(b) a diameter I = ____
​   ​ 

2 perpendicular diameters as x and y-axes
y
I

I
x

Ix + Iy = Iz
2I = MR2
1
I = __
​   ​  MR2
2
(c) a tangent 3 Parallel axis theorem
I = ​ __ ​  MR2
I 2 I = ICM + MR2
MR2
= ____
​   ​   + MR2
2
R 3
= __
​   ​  MR2
2
I ICM

7. Cylindrical Central axis parallel to I = MR2 All particles are at distance R from the axis.
shell length

8. Disc (a) passing through centre, 1 By method of integration. See example 4.


I = __
​   ​  MR2
normal to plane 2
I

R
6.8  Mechanics II

(b) About diameter MR2 Using perpendicular axis theorem, as in case


I = ____
​   ​ 

4 of a ring.
9. Solid cylinder Passing through centre, 1 Divide the cylinder into many this discs of
I = __
​   ​  MR2
parallel to length 2 masses m1, m2, m3 ...etc.
I

m1 R
m2
R m3

1 1 1
I = __
​   ​  m1R2 + __
​   ​  m2R2 + __
​   ​  m3 R2 + ...
2 2 2
1 1
= ​   ​  (m1 + m2 + m3 + ...)R2 = __
__ ​    ​ MR2
2 2
10. Thin spherical (a) Diameter 2 Integration. See example 5.
I = ​ __ ​  MR2
shell I 3

(b) Tangent 5 Parallel axis theorem


I = __
​   ​  MR2
I 3 I

ICM
R

I = ICM + MR2
2 5
= ​ __ ​  MR2 + MR2 = __
​   ​  MR2
3 3
11. Solid sphere (a) Diameter 2 Integration See example 6
I = __
​   ​  MR2
I 5

(b) Tangent 7
I = __
​   ​  MR2 I = ICM + MR2
5 2 7
I = ​ __ ​  MR2 + MR2 = ​ __  ​ MR2
5 5
I
R
ICM
R
Rotational Dynamics  6.9

Example 9   ABC is a A Axis ICM


L shaped rod made of I
B
two perpendicular seg- m
ments – AB and BC D

– each of length l and A
mass m. Consider an a
C
axis passing through B m 
C and parallel to AB.
CM
Write the moment of inertia of the rod about this axis.
Solution
C
Concepts
(i) Write the MI of two segments (AB and BC) sepa- __
rately, about the given axis, and add them. AB ÷    a ___
​ 2 ​  a
d = AD = ​ ___ ​ = ____
​   ​   = ​  __  ​ 
(ii) For segment AB, entire mass is at a distance l from 2 2   
​÷2 ​
the axis.
MI of AB , I1 = ml2 Each particle is at distance
\ 2
I = ICM + Md = ​   ​ 
Ma2
____
6
 + M ​​ ___
(  )
a 2
​  __  ​  ​​ ​ =
  
​÷2 ​
2
__
​   ​  Ma2
3

l from the axis] Example 12   MI of a square plate about diagonal


1
MI of BC , I2 = ​ __ ​  ml2 A uniform square plate has mass M and side length a. Find
3 its MI about a diagonal.
4
\ I = I1 + I2 = __​   ​  ml2
3 Solution
Example 10   In the last problem, find the moment of Concepts
inertia of the L-shaped rod if line AB itself is the axis. (i) Diagonals of a square are perpendicular. The two
Solution diagonals can be regarded as x and y-axes in the
Concepts plane of the square. We will use perpendicular axis
theorem.
When mass lies on the axis, it does not contribute to Ma2
moment of inertia. (ii) Iz = ____
​   ​ 
.  See the table.
6
MI of rod AB about the said axis = 0 From symmetry, Ix = Iy
1 Perpendicular axis theorem:
MI of rod BC about the said axis = __
​    ​ ml2
3 Ix + Iy = Iz
1 2
__
\ I = ​   ​  ml Ma2
3 2Ix = ____
​   ​   
6
Example 11   MI of a cube about its edge 2
Ma
fi Ix = ____
​   ​ 
A uniform cube has mass M and side length a. Find its MI 12
about one of its edge. y

Solution
Concepts
(i) MI of a cube about its central axis (See the table
1
given above) is __
​   ​  Ma2
6
a
(ii) Use parallel axis theorem. x

Ma2 Example 13   Half disc


ICM = ​ ____
 ​ 

6 A semi-circular disc has mass M and
AC is an axis that is parallel to the axis passing through radius R. Find its moment of inertia
COM. about an axis, which is perpendicular
Distance between the two axes is to its plane passing through the centre
of the circle.
6.10  Mechanics II

Solution
Concepts
(i) Think of a disc of mass M and radius R. Its R
MI about central axis perpendicular to its plane R/2
MR2
is ​ ____
 ​ 
. 
2
(ii) If this disc is folded about its diameter to make it
a semi-circular disc, the distance of any mass point
from the axis does not change. Solution
Concepts
(i) Fill up the hole. Write MI of completed disc about
the given axis and then subtract the MI of disc of
R
radius __
​   ​ .
2
(ii) Remember that mass of the completed disc
(without hole) is larger than M.

M 4m
Mass per unit area is s = ___________
​      2 ​ = ​ _____    ​ 
If half-disc PQR is flipped over to cover the other half
PSR, a point like A1 moves to a position A2. Distance of this
2 R
p R – p ​​ __
​   ​   ​​ ​ 3p R
2
2
(  )
point from z-axis remains unchanged. A point B1 moves to 4M
Mass of disc without hole is, M1 = s ◊ pR2 = ___
​   ​ 

B2. Its distance from z-axis remains unchanged. 3
Thus, Â mi ri2 will not change. MI of disc without hole about the given axis is

\  MI of half-disc about z-axis = MI of full disc about M1R2 ___ 2M 2


I1 = _____
​   ​  = ​   ​ R


z-axis 2 3

( 
1
\ I = __
​   ​  MR2
2 MI of the removed disc ​ of radius __
axis is
R
)
​   ​   ​ about the given
2
Example 14   Disc with a hole
(  )
2
1 R 2 MR
R I2 = ​ __ ​  M2 ​​ __
​   ​   ​​ ​ = ____
​   ​   
A disc of radius R has a concentric hole of radius __
​   ​ . ‘The 2 2 24
2
disc with hole’ has mass M. Find its moment of inertia about
its central axis normal to its plane.
\ Required MI is I = I1 – I2 = ​ __
2
( 
​   ​  – ___
3 24
1
) 5
​    ​  ​ MR2 = __
​    ​ MR2
8

Your Turn

Q.1  Three particles of masses 10 g, 20 g and 30 g are Q.4  A half-ring has mass M and radius
kept in xy-plane at points (2, 0) cm, (0, 6) cm and (4, 3) cm R. Find its moment of inertia about an axis
respectively. Find moment of inertia of the particle system perpendicular to its plane passing through
about centre O of the circle.
(a) x-axis  (b) y-axis  (c) z-axis Q.5  A uniform disc has radius 0.1 m
Q.2  A uniform rod has mass m and length 2l. Two and its moment of inertia about an axis
particles of mass m each, are attached to the ends of the rod. perpendicular to its plane, passing through
Find moment of inertia of the system about the perpendicular its centre is 1.2 kg m2. A sector of
angle 30° is cut and removed from A
bisector of rod.
the disc. Find moment of inertia
Q.3  A ring has mass M and radius R. One half of the of the remaining disc, axis being O
30°
ring is twice as heavy as the other half. Find the moment B
same.
of inertia of the ring about its central axis perpendicular to
its plane.
Rotational Dynamics  6.11

Q.6  Two circular discs of same mass and thickness are of the rod is L. Find its moment of inertia about a line that
made from metals having different densities– d and 2d. is perpendicular to the rod and passes through x = 0.
Which disc will have larger moment of inertia about its Q.10  Find percentage change in moment of inertia of a
central axis? solid sphere about its diameter if its radius increases by 0.1%
Q.7  Four thin sticks, each of mass m and length l, are without change in its mass.
joined to form a square. Find moment of inertia of the square Q.11  Two particles of masses M and m are connected by
about one of its side. a rod of negligible mass and length L to form a dumbbell.
Q.8  Three rods, each of mass m and length l, are joined Find the smallest moment of inertia of this dumbbell about
together to form an equilateral triangle. Find the moment of an axis that is perpendicular to the rod.
inertia of the triangle about an axis passing through its centre Q.12  A uniform sphere of radius a has concentric
of mass and perpendicular to the plane of the triangle. spherical cavity of radius b. Find the moment of inertia of
Q.9  A thin rod has its linear mass density changing with this object of mass M about its diameter.
distance from one of its end (at x = 0) as l = l0 x. Length

2.3  Radius of Gyration


Example 15   Find radius of gyration of a uniform solid
Radius of gyration (K) of a body about sphere about its diameter.
an axis is the effective distance from I
the axis where the whole mass can be Solution
assumed to be concentrated so that the Concepts
M
moment of inertia remains same.
Mk2 = I
A body of mass M has MI equal

÷ 
__
to I about certain axis. A point mass 2 2
M kept at a distance k from the axis k
M
Mk2 = __
​   ​  MR2  fi  k = ​ __
​   ​ ​   ◊ R
5 5
also has the same MI. K is known as
radius of gyration of the body about A higher value of k implies that the mass is effectively
the axis. at a larger distance from the axis.

÷ 
___
I
Mk = I  fi  k = ​ __
2
​     ​ ​   ...(5)
M

Your Turn

Q.13  Find the ratio of radius of gyration of a solid cylinder and a cylindrical shell about the central axis, parallel to
length. Both have same radii.

3. NEWTON’S SECOND LAW FOR We will use equation (6) about an axis
(i) that is fixed
ROTATION
(ii) that passes through COM of the body, even if COM
A torque can cause rotation and produce angular accelera- is accelerated.
tion in a body. Angular acceleration (a) produced depends
There are other axes about which equation (6) can be
on moment of inertia (I) of the body.
used directly, without thinking about the torques due to
Consider a rigid body, which is free to rotate about a pseudo forces. But having said this, there is usually noth-
fixed axis (say z-axis). tz is net external torque acting on ing to be gained by picking such axes. We will better avoid
the body. It produces an angular acceleration a in the body, picking any such axis, which is neither stationary nor passes
which is given by through COM.
tz = Iz ◊ a ...(6) COM has this remarkable property that rotational motion
where Iz is the moment of inertia of the body about z-axis. about it is independent of translational motion of the body.
Equation (6) can be used about any axis passing through
6.12  Mechanics II

COM of the body, whatever be the state of motion of the (ii) a changes with q.
COM itself. w 90°
3 g
w dw = a dq  fi  ​Ú ​  ​ ​w dw = __
​   ​  ​ __ ​ ​Ú ​   ​ ​sin q dq
Example 16   Hinged rod released from vertical position 0 2 L 0
w2 3 g
A rod of mass M and length L is hinged at its lower end fi  ​ ___ ​   = __
​    ​ ​ __ ​ ​[ – cos q ]90°
​0
and held vertical. It is given a gentle push and 2 2 L

÷ 
B ___
released. The rod rotates without friction about 3g
a horizontal axis at the hinged end. fi  w = ​ ___
​   ​ ​  
L
(i) Find its angular acceleration when the
L Example 17   A pulley is in the shape of a uniform disc
rod rotates through an angle q.
of radius R. Its mass is M. It can rotate freely about its
(ii) Use integration to find angular speed of
fixed horizontal axle. A light thread is tightly wound on
the rod when it becomes horizontal. A
it. Free end of the thread is connected to a block of mass
Solution m. System is released. Find acceleration of the block and
Concepts angular acceleration of the pulley. Also find tension in the
string.
(i) Rotation axis is a horizontal line passing through
M
A and perpendicular to the plane of the figure.
(ii) The rod rotates about this fixed axis. We can use R
t = I a about this axis.
ML2
(iii) MI of the rod about rotation axis is I = ____
​   ​ 

3
(iv) Only Mg has torque about rotation axis.
dw m
(v) ​ ___ ​ = a
dt
Solution
dw dq dw
fi ​ ___ ​  . ​ ___ ​ = a  fi  w ​ ___ ​ = a Concepts
dq dq
fi  w dw = a dq (i) If a is acceleration of the block and a is angular
Integrating this will give w at any position. acceleration of the pulley, then a = Ra.
(ii) We will write equation of translational motion for
B the block using F = ma
(iii) We will write equation for rotational motion of the
pulley about its fixed axis using t = Ia.
B
(iv) solving the equations will give the desired
C results.
L
sin q
2
a = Ra ...(i)
q
For the block: mg – T = ma ...(ii)
A Mg For rotation of pulley (about A)
Ia = t [Weight of pulley and force by
(i) When the rod rotates through angle q, torque on it
axle have no torque about A]
is MR2
____
L ​   ​ ◊ a

  = T ◊ R
t = Mg ​ __ ​  sin q 2 a
2 fi M (Ra) = 2T
L
__
​   ​  = distance of line of Mg 
2 fi Ma = 2T [using (i)] ...(iii)
from the axis through A A R
We have three equations with three
Hinge force has no torque about A.
unknowns: T, a and a. Eliminating T
T
Using  I a = t between (ii) and (iii) gives
T
mg
1 L 3 g
​ __ ​  ML2 ◊ a = Mg ​ __ ​  sin q  fi  a = __
​   ​  __
​   ​ sin q a = ______
​      ​ a
M
3 2 2 L m + __​   ​ 
2 mg
Rotational Dynamics  6.13

a mg Mmg
\ a = ​ __ ​  = _________  ​  and  T = ​ _______
[ 
​         ​
R M
R ​ m + __ ]
​   ​   ​
2
2m + M
R
a
Example 18   Atwood’s machine with a massive pulley
A light thread, carrying two blocks of T1
masses m1 and m2 (< m1) at its end, R
T2
passes over a pulley of radius R. The
pulley can rotate about its axle without Tensions on two sides of the string are different.
friction and has a moment of inertia I Friction causes this difference
about its axle. T2 T1
Thread does not slip on the pulley. m2
m1
Find acceleration of the blocks and ten- a a
sion in the threads on two sides.
m2g m1g
Solution
Concepts For translational motion of blocks:
(i) Friction between the thread and the pulley is large,
m1g – T1 = m1a ...(iii)
which does not allow the thread to slip.
(ii) Pulley rotates and transfers thread from left side to T2 – m2g = m2a ...(iv)
right side.
(iii) Tension on two sides of the pulley produces a We have four unknowns (a, a, T1 and T2) and there are
torque on the pulley about O. torque on the pulley four equations as well. Adding (ii), (iii) and (iv)
is t = T1R – T2R (See figure).
(iv) If pulley is massless, net torque on it must always

I
m1g – m2g = ​ ___
( 
​  2   ​ + m1 + m2  ​ a
R )
be zero. Even a small torque will imply infinite
(m1 – m2) g
angular acceleration (   I = 0). fi a = ​ ___________
    ​
   ...(v)
I
(v) For a massive pulley, we cannot have angular m1 + m2 + ___
​  2   ​ 
acceleration without torque. This implies that R

[ 
T1 > T2; otherwise, the pulley cannot accelerate. Substituting in (iii) and (iv) gives T1 and T2

]
Tension on the two sides must be different. Friction I
between the thread and pulley cause this difference 2m2 + ___
​  2   ​ 
R
in tension. T1 = m1g ​ ​ ___________
       ​  ​ ...(vi)
I
m1 + m2 + ​ ___2   ​ 

[ 
(vi) For no slipping, a = Ra, where a is the acceleration
R

]
of the blocks and a is the angular acceleration of
I
the pulley. 2m1 + ___
​  2   ​ 
R
(vii) We will write t = Ia for pulley and F = ma for T2 = m2g​ ​ ___________
       ​  ​
I
the blocks. m1 + m2 + ​ ___2   ​ 
R
a = acceleration of blocks with m1 going down and m2 Note:  If pulley is massless, then I = 0 and equations (v)
moving up and (vi) give
a = angular acceleration of the pulley (m1 – m2) g T  =  2m1m2g
a = Ra for no slipping ...(i) a = ​ __________
   ​  and ​ __________
   ​

(m1 + m2) m1 + m2
For rotation of pulley:
Example 19   Combined rotation
T1R – T2R = Ia and translation of a body
I A light thread is wound tightly
fi T1 – T2 = __
​    ​ a
R around a disc of mass M and radius
I R. Free end of the thread is fixed R
fi T1 – T2 = ___
​  2   ​  a ...(ii)
R to ceiling of a room and the disc M
is released. Find the acceleration of
[   Ra = a] the disc.
6.14  Mechanics II

Solution Example 20   Hinged rod


Concepts A uniform rod of mass M and M, L
length L is hinged at its end. It
(i) ‘Acceleration of the disc’ implies acceleration of
can rotate freely about this end
the COM of the disc.
in a vertical plane. The rod is released from horizontal posi-
(ii) The disc rotates and thread unwinds. If the disc tion. Immediately after the rod is released, find
rotates by q, a length x = Rq unwinds. The COM
(i) its angular acceleration
falls by . Thus, displacement (x) of COM and
angular displacement (q) of the disc are related as (ii) force applied by the hinge on it
x = Rq. Solution
fi a = Ra, Concepts
where  a = acceleration of COM (i) Immediately after the rod is released, it is still
a = angular acceleration. horizontal and has zero speed.
(iii) Though COM is accelerated, we can use t = Ia (ii) Using t = Ia about the rotation axis is easy. Hinge
about an axis through COM. force does not have any torque about the rotation
(iv) We will write Fext = MaCM and t = Ia (about an axis. Only Mg has torque.
axis through COM perpendicular to the figure) for (iii) Once a is known, acceleration of COM of the rod
the disc. can be calculated. The COM will go in a circle
about the hinged end. Immediatedly after release,
a = R a ...(i) speed of COM is zero. It has no radial acceleration.
It only has tangential acceleration.
where a = acceleration of COM
(iv) Fext = MaCM will help us know the hinge force.
a = angular acceleration about COM
Motion of the disc is a combination T
of translation of its COM and rotation a
about it.
For translatory motion, point
of application of forces are not R a
important. (i) Torque on rod about A is
Fext = MaCM L
Mg
t = Mg __​   ​ 
2
Mg – T = M a ...(ii)
Angular acceleration (a) is given by
We can always use t = Ia about an axis through COM.
Ia = t
Mg has no torque about this axis. Only tension (T) has a
torque. ML2 L 3g
​ ____
 ​ ◊ a =

  Mg ​ __ ​   fi  a = ___
​    ​ ...(i)
3 2 2L
\ I a = t
1 1 (ii) COM moves in a
​ __ ​  MR2 ◊ a = T . R I = __
​   ​  MR2 for a disc L
2 2 circle of radius ​ __ ​ . Its
about central axis 2
1 initial speed is zero.
fi ​ __ ​  Ma = T ...(iii)
2 Radial acceleration
[   Ra = a] of COM is zero.
2g It has a tangential acceleration (directed vertically
From (ii) and (iii) a = ___
​   ​ 
3 downward)
Note:  We have considered translation of COM (Without L 3g
worrying about the fact that there is rotation also) and we at = a __​   ​  = ___
​   ​ 
2 4
have considered a rotation about COM (not worrying about 3g
the fact that the COM is accelerated). Superposition of these \ aCM = ___
​   ​ (Ø)
4
two independent motions is the resultant motion of the body.
The equation a = Ra is the constraint relation, which relates If FH is the hinge force in vertically upward
the rotational and translational motion. direction
Rotational Dynamics  6.15

3Mg
Mg – FH = MaCM  fi  Mg – FH = ____ ​   ​ 
  This is same as (i).
4
Mg In short:
fi FH = ___
​   ​ 

4 (i) Use t = Ia about a fixed axis or about an axis that
Hinge force has no horizontal component, as COM has passes through COM of the body.
no horizontal acceleration.
(ii) For hinged objects, using t = Ia about the hinge is
Note: easiest, as it does not involve writing torque due to
(i) Acceleration of COM depends on Fext and has got hinge force.
nothing to do with point of application of the force (iii) For a body that is rotating as well as translating,
(ii) If we consider rotation about COM, we can write use
L t = Ia about an axis through COM and
ICM ◊ a = FH ◊ ​ __ ​ 
2
Fext = MaCM
ML 2 Mg L 3g
fi ​ ____ ​    ◊ a = ___   __ ​   
​   ​ ◊ ​ 
    fi  a = ___
​     While writing the last equation, one need not worry
12 4 2 2L
about the point of application of the forces.

Your Turn

Q.14  A sphere of mass m = 2 kg and radius r = 0.5 m


is free to rotate about its vertical diameter. A horizontal
tangential force F = 10 N acts on it for t = 3 s. Find the
angular speed gained by the sphere.
Q.15  A uniform rod
of mass m and length l
is swinging about its one
end like a pendulum. In 60°
Q.18  A cylinder of mass M
its extreme position, it and radius R is kept on a smooth
makes an angle of 60° table. It is pulled by a horizontal F
with vertical. Find its force F that acts along a line
angular acceleration at passing through centre of the
(i) Extreme position cylinder.
(ii) Vertical position. (i) Find acceleration of centre of the cylinder.
Q.16  A disc-shaped pulley (P) has mass M and radius (ii) Find angular acceleration of the cylinder
R. It is fixed on top of a smooth incline and can rotate Q.19  A uniform disc of mass M
freely about its axle. A and radius R is free to rotate about
P
thread is tightly wound on a horizontal axis through its centre
the pulley. The free end of (O). A particle (P) of mass m is
the thread is connected to a O
stuck at the circumference at angu-
block of mass m kept on the A lar position q as shown. The system q
incline. system is released. is released from this position. P R
Find the acceleration of the
q Find the acceleration of the par-
block. Inclination angle of
ticle, immediately after release.
the incline is q.
Q.20  A uniform rod of length 4l and mass m is free
Q.17  In the arrangement of Atwood machine shown in to rotate about a horizontal axis passing through a point
figure, the pulley has radius R and moment of inertia I about (O) at a distance l from its one end. The rod is released
its rotation axis. There is no friction between the block of from horizontal position.
mass m2 and the incline surface. On releasing the system, Immediately after release, O
m1 goes down. Find its acceleration.
6.16  Mechanics II

(i) Find angular acceleration of the rod


(ii) Find acceleration of COM of the rod. M
(iii) Find hinge force on the rod. R
Q.21  Two identical disc, each of mass M, are connected by
a massless string wrapped around them as shown. The upper
disc is mounted on a frictionless horizontal axis about which
it can rotate. The other disc is free to translate as well as R
rotate. Assume that the axis of lower disc remains horizontal, M
find its acceleration after the system is released.

4. ROLE OF FRICTION IN ROLLING


Think of a perfectly rigid ball rolled on a flat horizontal
surface. The ball is
‘rigid’ implies that it
touches the horizontal
surface at a single point
(A). Normal force (N) on
the ball and its weight
(Mg) have their line of
action along line AC [C
is the centre of the ball].
These forces are verti-
cal, they cannot produce any horizontal acceleration. They
do not have any torque [about C or A]. They cannot change
angular speed (w).
Friction force ( f ) will assume a direction depending on
the direction of motion of contact point A. If v = wR, then During the period, the ball rolls with sliding, friction
the ball rolls without sliding and the contact point is instan- force is kinetic in nature. Heat is produced (in rubbing) and
taneously at rest. Friction will adjust itself to zero so that no mechanical energy is lost.
sliding condition prevails. The ball will keep rolling forever!
Then why does a rolling ball stops? keep pondering. We will Example 21   Why does a rolling ball stop?
return to this in a while.
If a perfectly rigid ball is rolled on a horizontal floor with
If the ball is rolled v = wR, our theoretical discussion indicates that it will keep
with v > wR, then the rolling forever. In practice, no ball rolls forever. why?
contact point A has a
Solution  In practice, no object is perfectly rigid. Objects
velocity in forward direc-
deform under action of forces. A ball deforms under action
tion. Friction does not
of its weight and normal force by the floor. This results in
like rubbing. It imme-
multiple contact points of the ball with floor. The shown
diately takes a direction
figure is a bit of exaggeration if you talk of a steel ball (but
opposite to the direction
quite OK for a soft rubber ball). The normal force on the
of motion of contact
ball has a line of action slightly shifted to the right of the
point A. Friction causes
centre (c). This produces a
a retardation and decreases v. It also produces a torque about
torque about C. The torque
C and provides an angular acceleration (a). This increases w.
tries to reduce the angular
Therefore, after some time, velocity (v0) will become equal
speed w. Friction immedi-
to w0 R and sliding will stop. Now, friction will adjust itself
ately takes leftward direction
to zero and the ball keeps rolling forever!
so as to reduce the speed (v)
Figure below shows another situation where friction also, and to maintain the no
converts an imperfect rolling into perfect rolling motion. sliding condition (v = wR).
Fact is that, if it is left to friction, any motion will eventually Thus, the ball will slow down
get transformed to a motion where no sliding occurs. and eventually stop.
Rotational Dynamics  6.17

Example 22   A solid ball is imparted a velocity v Let f be friction force on the ball.
f
(and no spin) and released on a rough horizontal surface. Linear retardation of the ball is a = __
​    ​ 
Find its velocity when pure rolling begins. M
Torque due to friction about C is t = fR
Solution Moment of inertia about an axis through C is
Concepts 2
I = __
​   ​  MR2
(i) Every point (including the contact point) of the ball 5
has a forward velociy v in the beginning. Friction Angular acceleration (clockwise) is
will act on the ball in backward direction. 5f
t
Friction decreases v and increases w. Soon, speed a = ​ __ ​ = _____
​    ​ 
I 2MR
will become equal to wR and pure rolling will
start Velocity of the ball at time t is
F t
(ii) We will write a = ​ __  ​ and a = ​ __ ​ to write the linear f
M I v0 = v – at = v – ​ __  ​  t ...(i)
and angular accelerations of the ball. M
(iii) Using v = u + at and w = w0 + a t, we will write Angular velocity of the ball at time t is
v and w after time t.
5ft
w0 = 0 + a t = ​ _____  ​   ...(ii)
2MR
When pure rolling begins, v0 = w0 R

f 5ft
fi v – ​ __  ​  t = _____
​    ​  
. R
M 2MR

7f
fi v = ___ ​ ◊ t
​     
2M

2v ft
fi ​ ___ ​   = __
​    ​ 
7 M

2v 5v
Putting this in (i) gives: v0 = v – ___
​   ​ = ___
​   ​ .
7 7
This is the speed of the ball when rolling begins.

Your Turn

Q.22  A disc is rolled on a rough Q.23  A disc is rolled on a horizon-


horizontal surface. At an instant, its tal surface with its centre moving at a
angular velocity is found to be increas- velocity (v) and the disc spinning about
ing. Which is larger–velocity (v) of the 2v
its centre at an angular velocity w = ___
​   ​ .
centre or wR? R
Find the velocity of the disc when slip-
ping ceases.

4.1  Rolling on an incline friction is sufficiently strong, it will ensure that v is always
equal to wR. In fact, this will be possible if
A round object of mass M and radius R is released on an
inclined plane, set at an angle q to the horizontal. Component a = Ra ...(i)
of Mg, down the incline (= Mg sin q) will try to accelerate the where a = acceleration of the centre and
body. To ensure that there is no slipping, friction assumes an
a = angular acceleration of the body
upward direction so as to spin the body about its COM. If
6.18  Mechanics II

N a Example 23   Sphere is fastest, ring slowest


A solid sphere, a cylinder, a disc, a hollow sphere and a ring
a are released on an incline from same height. Which body
C
f will be first to reach the bottom? Which is last?
sinq Solution
Mg
Concepts
g sin q Mg sin q
Mg cos q (i) a = ​ ______2   ​ = ​ _______ 
   ​
q Mg k
___ ___I
1 + ​  2  ​  M + ​     ​

R R2
Equation for translational motion can be written as
(ii) Acceleration does not depend on mass, it depends
Mg sin q – f = Ma ...(ii) on shape of the body.

For rotation of the body about its COM, we can write


2 k2 2
For solid sphere:  Mk2 = __
​   ​  MR2  fi ​ ___2  ​ = ​ __  ​
t = Ia [Mg and N have no torque] 5 R 5

fR = Ia g sin q 5 g sin q
\ asph = ______
​   ​ = _______
  ​   ​   
2 7
I I 1 + __
​   ​ 
fi f = ___
​  2   ​  (Ra)  fi  f = ___
​  2   ​  a ...(iii) 5
R R
2 k2 2
For shell:  Mk2 = __
​   ​  MR2  fi ​ ___2 ​ = ​ __ ​ 
Adding (ii) and (iii) gives 3 R 3

Mg sin q g sin q 3 g sin q


a = _______
​    ​ ...(7) \ ashell = ______
​   ​ = _______
  ​   ​ 

I 2 5
M + ___
​  2   ​  1 + __
​   ​ 
R 3
1 k2 1
If k is radius of gyration of the body about its rotation For cylinder or disc: Mk2 = __ ​   ​  MR2  fi ​ ___2 ​ = ​ __  ​
2 R 2
axis through COM, then I = Mk2.
g sin q 2g sin q
g sin q \ acyl = adisc = ______
​    ​ = _______
​   ​   
a = ______
​    
 ​ ...(8) 1
1 + __
​    ​
3
k2 2
1 + ​ ___2  ​ 
R k2
For ring: Mk2 = MR2  fi ​ ___2 ​ = 1
Writing the expression for acceleration in above form R
shows you that it does not depend on mass of the body. It g sin q g sin q
depends on k, i.e., shape of the body. \ aring = ______
​    ​ = ______
​   ​   
1+1 2
Substituting value of a in (iii) gives
\ asph > adisc = acyl > ashell > aring.
___I
​  2   ​ Mg sin q Solid sphere is fastest and the ring is slowest.
I R
f = ___
​  2   ​  a = ​ __________     ​   Note:  Acceleration does not depend on mass or radius of
I
R M + ___ ​  2   ​  the body.
R
Mg sin q Mg sin q Example 24   Minimum coefficient of friction for pure
fi f = ​ ________
2
  ​ = _______
  ​  2  
 ​ ...(9) rolling
MR
____ R
___
​      ​ + 1 ​  2  ​ + 1 A solid uniform cylinder is released on a rough incline. Find
I k the minimum coefficient of friction between the cylinder and
If friction coefficient between the incline and the object is the incline so that it does not slip.
small and friction cannot assume a value given by equation
Solution
(9), the body will begin to slip.
The friction force is static friction when there is no Concepts
slipping. This friction does not produce heat. Mechanical For pure rolling, friction force must be equal to the value
energy of the rolling body does not get dissipated. given by equation (9).
Rotational Dynamics  6.19

Mg sin q Normal force on the cylinder is N = Mg cos q.


For pure rolling, f = ________
​  2   
 ​
MR
____ f £ mN
​      ​ + 1
I
1 Mg sin q tan q
I = __
​   ​  MR2 fi ​ _______
 ​   £ m Mg cos q  fi ​ ____ ​ 
 £ m
2 3 3
Mg sin q 1
\ f = _______
​   ​    \ mmin = __
​   ​  tan q
3 3

Your Turn

Q.24(a)  A disc of mass M and radius R and a ring of Q.25  Find minimum coefficient of friction between a ring
same mass and same radius rolls down an incline. They and a 45° incline, so that the ring can roll without sliding
start from the same height at same time. Which will reach when released on the incline.
the bottom of the incline travelling at higher speed? Assume
Q.26  A ball is released on an incline, which is smooth.
that they do not slip.
Find the angular speed of the ball after it has travelled a
(b)  A small solid ball is released distance S.
from top of an incline having incli-
Q.27  A ball of mass M and radius R is released on a
nation angle q. It rolls down with-
rough incline plane of inclination q. Friction is not sufficient
out sliding. It reaches the bottom
to prevent slipping. Coefficient of friction is m. Find
of the incline after travelling a
distance L. Find its speed at bot- (a) acceleration of the centre of the ball
tom. What is its angular speed if (b) angular acceleration of the ball about its centre
its radius is r?

4.2 Rolling with Other forces, apart from friction, contact point will have a tendency of backward slipping.
producing torque Friction will act so as to prevent this slipping. It will take
a forward direction. This will increase a and reduce Ra to
A round object is kept on a horizontal rough surface with a ensure a = Ra.
light thread wound on it. The free end of the thread is pulled
If you still find it difficult to figure out the direction
horizontally with a force F, as shown. This force produces
of friction, do not worry. If a problem says that the body
linear acceleration as well as angular acceleration. Friction
rolls without slipping, assume friction to be in any direc-
adjusts itself so as to ensure pure a F tion you want. Imposing the condition a = Ra, will ensure
rolling (i.e., a = Ra). Slipping R
that mathematics corrects the direction of friction if it is
will occur only if friction is a
C wrongly chosen. On solving your equations, you may get a
small and cannot assume a
negative value for friction, implying the direction chosen to
value that is needed to ensure
be wrong.
a = R a.
For a moment, assume that friction is absent. Acceleration  Example 25   A string is wrapped around a uniform cylinder
(a) and angular acceleration (a) of the body will be given of mass M, which rests on a fixed incline plane of inclination
by angle q. The string passes over massless pulley (P) and
F F ◊ R FR2 R2 is connected to a block of
a = __
​    ​   and  a = ​ ____2 
 ​  fi  Ra = ​ ____2  ​ = ​ ___2  ​ a. P
M Mk Mk k mass m, as shown. Assume
that the cylinder rolls with-
where k is radius of gyration about the rotation axis through
out slipping on the plane and
C.
that the string is parallel to M
If the round object is a ring, then Mk2 = MR2 and the plane. R m
F
Ra = ​ __  ​  = a. Any value of force F, on a ring, will ensure (a) Find acceleration of
M q
mass m.
pure rolling in absence of friction. Friction will adjust itself M
to zero! (b) Find the ratio __
​ m ​  for which the cylinder accelerates
For a body having k < R [all the round objects that we down the plane.
have seen, apart from ring, have k < R] R a > a and the
6.20  Mechanics II

Solution (b) For cylinder to roll down, 'a' must be positive.


Concepts
M ____ 2
\  M sin q > 2m  fi ​ __    ​ 
m ​  > ​ sin q
(i) If we assume that the cylinder rolls down, it will
wrap the thread and the block will move up.
M
Velocity of top-point of the cylinder will be twice If q Æ 0, this gives __
​ m ​  Æ •, which makes sense.
that of its centre (v + wR = 2v). Thus, velocity of
the block will always be twice that of the centre Note:  You can calculate friction ( f) from the above equa-
of the cylinder. It means acceleration of the block tion. It turns out to be a positive number. This means we
is twice that of the centre of the cylinder. have chosen the correct direction of the friction force.
(ii) Friction on cylinder will adjust itself so as to 4.3  Role of friction in motion of a car
ensure pure rolling. Without thinking much, we
will assume this friction to be in any direction, say Consider a front-wheel drive car. Once you start the car, put
up the plane. it in gear and release the clutch, the engine imparts torque
(t) to the front axle. Rear wheel receives no power. If there
(iii) We will write equation for translation and rotation
is no friction between the front wheel and the road, the
of the cylinder and for translation of the block.
front wheel will spin but the car will not move! There is
no external force. If front wheels are placed on a slippery
surface, the car will really not move, however hard you may
T press the accelerator pedal.

a
a T Rear Front
f 2a
f2 t f1
sinq F
Mg A
q
mg
When friction is present, it will try to prevent slipping of
wheel at A. It will assume a forward direction (f1) and will
Figure shows forces on the cylinder along the incline. accelerate the car. If you do not press the accelerator a lot
Using F = MaCM for cylinder: (i.e., you do not give a large angular acceleration a to the
front wheel), friction will ensure that the wheels do not slip
Mg sin q – T – f = Ma ...(i) and acceleration (a) of the car is a = Ra (R = radius of the
Using t = Ia about COM: wheel).
1 Due to f1, car has a tendency to move forward. The
fR – TR = Ia  fi  fR – TR = __ ​   ​  MR2 ◊ a chassis transfers a force F on the rear axle. This force will
2
1 1 try to accelerate (and not rotate) the rear wheel. To prevent
fi f – T = ​    ​ M (Ra)  fi  f – T = ​ __  ​ Ma ...(ii)
__
the rear wheels from slipping, friction (f2) will act on it in
2 2
backward direction.
[For pure rolling, Ra = a]
The car will fail to move if f1 is not greater than f2.
Acceleration of the block is twice that of the cylinder. Engine, which is heavy, is put at the front so that normal
\  For block force on front wheel is larger. This ensures that f1 > f2.
T – mg = m (2a) ...(iii) In heavy trucks or SUVs, we need more traction
(forward force). In such automobiles, both axles are pow-
(i) + (ii) + 2 × (iii) gives ered. When engine rotates both of them, friction on both

( 
front and rear wheels is in forward direction.

3
Mg sin q – 2mg = ​ __
​   ​  M + 4m  ​ a
2 ) When you apply brakes (on all wheels), the brake shoes
clamp the rotating wheels to the body of the car and rota-
(M sin q – 2m) g tion slows down. To maintain pure rolling (i.e., no sliding),
\ a = ​ _____________     ​
  
3
( 
​ __ )
​   ​  M + 4m  ​ a
2
friction takes backward direction on all wheels and decreases
the velocity (v) and tries to maintain wR = v.
It is friction which accelerates, it is friction which retards
2 (M sin q – 2m) g a moving car!
(a) Acceleration of the block = 2a = ​ ______________   
  ​
  
3
__
​   ​  M + 4m
2
Rotational Dynamics  6.21

g sin q
In Short:
a = ______
​   
2
  ​
(i) A body rolls without sliding on a surface if its con- 1 + ​ k ___  ​ 
tact point does not have a velocity relative to the R2
surface. Friction tries its best to maintain rolling where k is radius of gyration about rotation axis
without sliding. through centre.
(ii) If a body rolls without sliding (pure rolling), friction Friction force acting on the body is given by
Mg sin q
acting on it is static in nature. f = _______
​   
 ​
(iii) If a body rolls with sliding, friction is kinetic. R2
___
1 + ​  2  ​
(iv) On a horizontal surface, friction converts all motion k
into no sliding if it is the only force. (vi) In pure rolling, static friction does not dissipate any
mechanical energy.
(v) Acceleration of a round object performing pure roll-
(vii) When a body is rolling without sliding, it is fine
ing on an incline is given by
to choose any direction for friction. Solving the
equations with condition a = Ra results in correct
magnitude and direction of friction.

Your Turn

Q.28  A disc of mass M stands F moment of inertia of the spool about the central axis per-
on a horizontal surface. A light pendicular to the plane of the figure is 2MR2. Find the
thread is wrapped on it and its free direction and magnitude of friction force on spool if it does
end is pulled horizontally with a not slip.
force F, as shown. The disc begins
to roll without slipping. Find its
Q.30  In the figure shown, a string is wrapped around a
cylinder of mass M and radius R. The string passes over a
acceleration.
smooth pulley (P) and its free end holds a mass m. Find
Q.29  A spool has F acceleration of m if the cylinder does not slip
mass M. It has two light
2F P
threads wound on it on
circles of radii 2R and M
R
R. Free ends of the two R
threads are pulled hori-
2R
zontally with forces F
m
and 2F, as shown. The

1
5.  KINETIC ENERGY OF ROTATION = __ ​   ​  [​  Â mi ri2 ]​ w2
2
A rigid body is an aggregate of 1
particles and its kinetic energy or, k = __
​   ​  Iw2 ...(10)
2
is just equal to the sum total of
Example 26   Find the kinetic energy of earth related
kinetic energies of the individual
to its rotation about its axis. Given: Mass of Earth =
particles.
6 × 1024 kg, Radius of earth = 6,400 km. Assume that the
Consider a rigid body in pure earth is a uniform sphere.
rotation about a fixed axis. Its
angular speed is w. A particle at Solution
a distance r from the rotation axis Concepts
has speed v = wr. Kinetic energy
of the rotating body is (i) Angular speed of Earth is
2p rad
k = Â __
1
​   ​  mi vi2 w = ______ ​   ​ 

2 24 h
1
= ​   ​  Â mi (riw)2
1
__ (ii) k = __​   ​  I w2
2 2
6.22  Mechanics II

2p rad Solution
Angular speed w = ____________
​     ​ = 7.3 × 10–5 rad s–1
  
24 × 60 × 60 s
Concepts
MI of Earth about its axis
(i) k = kR + kT or else
2 2
I = ​ __ ​  MR2 = __
​   ​  × 6 × 1024 × (6400 × 103)2 1
(ii) k = ​ __  ​ I w2 where I is MI about instantaneous
5 5 2
= 9.83 × 1037 kg m2 axis. We know that for a body under pure roll-
ing, instantaneous axis passes through the contact
1 1
\ k = ​ __ ​  I w2 = __
​   ​  × 9.83 × 1037 × (7.3 × 10–5)2 point.
2 2
= 2.62 × 1029 J v
For pure rolling, w = __
​    ​
R
5.1  Kinetic Energy in Combined rotation and 1 1
\ k = kR + kT = __
​   ​  ICM w2 + ​ __ ​  Mv2
translation 2 2

( 
Kinetic energy of any system of particles can be written
as
1 2
= __
​   ​  ​ __
2 5
v 2
​   ​  MR2  ​ ​​ __ ) (  )
​   ​   ​​ ​ +
R
1
__ 7
​   ​  Mv2 = ___
2
​    ​ Mv2
10
1
k = ​k​wrt ​+ __ 2
​   ​  MvCM
 ​       ​ 2 Alternatively:  [Refer to article
COM
​ ​
3.2.1 of previous chapter]
In context of a rigid body, the first term is the energy
associated with rotation of the body with respect to the The ball is under pure rotation​
COM.
1
( 
w = __
v
)
​   ​   ​ about an axis through con-
R
​k​wrt ​ = ​ __ ​  ICM w2 tact point A.
 ​    ​    2
COM
​ ​
IA = ICM + MR2 [Parallel axis theorem]
Here, ICM is moment of inertia of the body about rotation
axis through COM. We will call this energy as rotational
2 7
kinetic energy. = __
​   ​  MR2 + MR2 = __
​    ​ MR2
5 5
1
kR = ​ __ ​  ICM w2
2
The second term in the above equation is kinetic energy
\
1
k = __
2
1
​   ​  IA w2 = __
​    ​ ×
2
7
__
(  )
v 2
​   ​  MR2 × ​​ __
5
​    ​  ​​ ​ =
R
7
​ ___  ​ Mv2.
10
associated with translational motion. It is written assuming
Example 28   A rod of mass
the entire mass of the body to be concentrated at COM and
moving with speed of COM. We will call this energy as M and length L is moving
translational kinetic energy. in air. At an instant, its two
end points have velocities as
1 2
kT = ​ __ ​  MvCM shown. Find the KE of the
2
rod.
Total kinetic energy of a body that is translating plus
rotating is Solution
1 1 2 Concepts
k = kR + kT = __ ​   ​  ICM ◊ w2 + __
​   ​  MvCM ...(ii)
2 2
It is important to note that velocities w and vCM are (i) End points have different velocities. It implies that
independent of one another and one can give any amount the rod must have rotation.
of rotational kinetic energy and translational kinetic energy (ii) We will find vCM and w and then write the kE
to a body.
If one can locate the instantaneous axis of rotation of a Let velocity of COM be vCM (Ø) and angular velocity of
body, its kinetic energy can be simply written as the rod be w ( )
1 l
k = ​ __ ​  I w2  where I = moment of inertia of the body vA = w ​ __  ​  – vCM
2 2
about instantaneous axis of rotation.
wL
fi v = ​ ___ ​ – vCM ...(i)
Example 27   Kinetic energy of a rolling ball 2
A solid uniform ball of mass M and radius R is rolling L
vB = w ​ __ ​  + vCM
without sliding. Velocity of its centre is v. Write its kinetic 2
energy.
Rotational Dynamics  6.23

wL 1 1
2v = ​ ___ ​ + vCM ...(ii) = ​ __ ​  ICM ◊ w2 + __
​    ​ M​v2​CM
  ​ 
2 2 2

(  ) (  )
3v v
Solving (i) and (ii): w = ___
​   ​  ; vCM = __
L
​    ​
2
1 ML2 ___
= __
​   ​ ​  ____
2 12
​   ​  
3v 2 1
 ​ ​​ ​   ​   ​​ ​ + __
L 2 (  )
v 2 1 2
​   ​  M ​​ __
​    ​  ​​ ​ = __
2
​    ​ Mv
2
\ k = kR + kT

Your Turn

Q.31  A uniform cylinder of mass M is rolling without Q.33  A fan is made of three
sliding. Its centre has velocity v. Find its kinetic energy. thin sticks. Each stick has mass
m and length l. The fan spins
Q.32  A uniform ring of mass M is rolling without about its central axis with angu-
sliding. Find the ratio of its translational to rotational kinetic
lar speed w. Write its kinetic
energies.
energy.

6.  WORK DONE BY A TORQUE AND 2. Change in rotational KE = work done by torque
WORK–ENERGY THEOREM 1 1
q2
​    ​ ICM w2i = Ú​  ​ ​t dq
fi ​ __ ​  ICM wf2 – __ ...(14)
When a torque acts on a rigid body, free to rotate about 2 2 q1

a fixed axis, it produces an angular acceleration, thereby When system is conservative, we can use law of
increasing/decreasing the rotational KE. conservation of energy as usual. Now the kinetic energy
Power delivered by torque = Rate of change of KE means sum of translational and rotational kinetic energies.


dk
P = ___
​   ​ =
dt
d 1 2
__
(  )
​    ​  ​ __
dt 2
dw
​   ​  Iw   ​ = Iw ​ ___ ​ = I aw
dt
Example 29   A light tape is wrapped
around a cylinder of mass M and radius
R. One end of the tape is pulled along
or, P = t ◊ w ...(12) the incline so as to prevent the cylinder
from falling or climbing. The incline is
Work done in an infinitensimal angular displacement dq
smooth and applied force is parallel to
is
the incline. Find:
dWt = P dt = t (w dt) = t dq (a) Work done on the cylinder in
time t.
\  Work done in finite angular displacement (Dq) is
(b) Length of tape unwound by the time the cylinder
Dq
attains an angular speed w.
Wt = ​Ú ​   ​ ​t dq ...(13)
0 Solution
If torque is constant; W = t Dq Concepts

When a rigid body is having a general motion, which is (i) Cylinder does not translate. Net force on it is zero.
a combination of rotation and translation, we will consider Work done by all forces = 0. There is no change
torque about COM and its work done will be equal to change in translational KE.
in rotational kinetic energy of the body. (ii) There is an unbalanced torque (about COM). This
The work – energy theorem divides into two parts for a causes the cylinder to rotate. Torque performs
rigid body in general motion: work. This increases the rotational kinetic energy.
1. Change in translational KE = Work done by forces Net force = 0 since COM does not move.
acting on it
​_› F = Mg sin q
_› ​ ›
​r ​  
2  ​ __
1 1
​   ​  Mv2i = _Ú​  ​  ​ ​F  ​  ◊ ​dr​  
fi ​ __ ​  Mvf2 – __ Torque on cylinder (about its COM) is
2 2 ​›
​r ​   1
6.24  Mechanics II

F Let w = angular speed of the rod in vertical position.


Gain in kE = Loss in PE
1 L
a
​ __ ​  IA w2 = Mg ​ __ ​ 
nq 2 2
si
Mg
q
2 3 (  )
1 ML2 2
​ __  ​ ​ ____
​   ​  
L
 ​ w = Mg ​ __ ​ 
2

÷ 
___
t = FR = mg R sin q 3g
fi w = ​ ___
​   ​ ​  
Mg R sin q 2g sin q L
\  Angular acceleration a =  ​t    ​ = _________
​  = ​ _______
 ​ 
      ​ 
​I ​ 1
__ R
​   ​  MR2 Example 31   Energy conservation in rolling down an
2
(a) Angular speed of the cylinder after time t is incline
A cylinder of mass M is released on an incline plane of
2g t sin q
_______
w = 0 + a t = ​      ​  inclination angle q. Friction is large enough to prevent
R slipping and the cylinder rolls down the incline. Find its
Work–energy theorem says: speed after it has descended through a vertical height h.
Work done by torque = change in rotational kE Solution

(  4g2t2 sin2q Concepts
1
2
1 1
Wt = ​ __ ​  I w2 = __
​   ​ ​  __
2 2 )
​   ​  MR2  ​ _________
​ 
R2
 ​  = M2g2t2 sin2q

In pure rolling, friction is static. There is no rubbing and
friction does not dissipate any energy. Mechanical energy
A slight alternative approach can be as follows:
of the cylinder is conserved. Gain in its kE is equal to
Angular displacement in time t is loss in its PE.
1 gt2 sin q Note:  Actually, torque due to friction (about COM)
q = __
​   ​  a t2 = _______
​       
​ does positive work and increases the rotational kE. But
2 R
“force” friction does negative work (it is opposite to dis-
\  Wt = t q = Mg2 t2 sin2 q placement of the cylinder) and decreases the translational
kE. Net change in kE due to friction is zero if there is
w2 w2R
(b) w2 = w20 + 2 a q  fi  q = ___
​    ​ = _______
​    ​  no sliding.
2a 4g sin q
Length of thread that unwinds is Let velocity of cylinder be v
q wR 2 2 after its centre has fallen through
L = 2pRn = 2p R ​ ___  ​ = Rq = ​ _______
   ​  a height h.
2p 4g sin q
K = kT + kR
Example 30   A uniform rod of mass M and lengh L is
1 1
hinged at one of its ends and = ​ __ ​  Mv2 + __
​   ​  ICM w2
can freely rotate in a vertical M, L 2 2
plane about a horizontal axis
through this end. Rod is released from horizontal position.
1 1 1
= ​ __ ​  Mv2 + __
2
​   ​ ​  __
2 2 (  )
​   ​  MR2  ​ w2
Find its angular speed at the moment it becomes vertical.
1 1
= ​ __ ​  Mv2 + __
​    ​ Mv2  [   v = wR]
Solution 2 4
Concepts 3
= ​ __ ​  Mv2
4
(i) The system is conservative. Only Mg does perform
work. Mechanical energy of rod is conserved. It Energy Conservation:
loses PE as it falls and gains kE. Gain in kE = loss in PE

÷ 
(ii) Loss in PE = Mgh; where h = fall in height of ____
3 4
COM. ​ __ ​  Mv2 = Mgh  fi  v = ​ __
​   ​  gh ​ 
1 4 3
(iii) kE of rod = ​ __ ​  I w2; where I = MI about the fixed
2 1
axis. In fixed axis rotation, kE = __ ​   ​  I w2 Example 32   Figure shows a track in vertical plane. Part
2 AB of the track is rough and the rest of the part is smooth.
Rotational Dynamics  6.25

A From B to C, the translational kE gets converted into


C potential energy. Rotational kE does not change
h1 2
h2 \ mgh2 = __
​   ​  mgh1
3
B 2
fi h2 = ​ __ ​  h1
A small cylinder is released from A and it rolls without 3
sliding on the part AB. It climbs on the other side of the
Example 33   A uniform
track, up to point C.
disc-shaped pulley has mass
Height of A is h1 and height of C is h2 from the bottom M and radius R. A light
of the track. R
thread is tightly wrapped M
(a) Which is larger – h1 or h2? around it and its free end
(b) Express h2 in terms of h1 carries a block of mass m.
The block is released from
Solution a height h above the floor. m
Concepts Find the speed with which
(i) From A to B, friction is large enough to prevent the block hits the floor. h
slipping and the cylinder performs pure rolling. Solution
(ii) At B, kinetic energy of the cylinder has two Concepts
components–translational kE and rotational kE.
Total kE is mgh1 as energy is conserved. Friction (i) You can find acceleration of the block [using
does not dissipate energy in pure rolling. F = Ma and t = I a] and then use v2 = u2 + 2as
to find the speed when the block has travelled
(iii) After B, there is no torque on the cylinder about through a distance s.
its COM. Its angular velocity cannot change now.
Rotational kE does not decrease. Only transla- (ii) Just to demonstrate an altenative approach, we will
tional kE decreases and the cylinder gains potential use energy method.
energy. When the cylinder stops at C, it is still Mechanical energy of the pulley + block system
rotating. is conserved, as the thread does not slip on pulley.
[A slipping thread would mean rubbing and kinetic
friction will dissipate mechanical energy.]
(iii) While writing kE, remember that the pulley also
has kE

Let, v = speed of the block after falling through h.


At B cylinder has kR and kT w = angular speed of pulley at the instant the
At D, v1 < v; kT has decreased but kR has not changed. block is about to hit the floor.
At C, kT = 0 but kR is still same as that at B.
(a) Mechanical energy of the cylinder is conserved Obviously,  v = wR
throughout. Gain in kE of the system = Loss in gravitational PE of
EA = mgh1 the block.
At C
1 1
EC = mgh2 + rotational KT ​ __ ​  mv2 + ​ __ ​  (Ipulley) w2 = mgh
2 2
Since, EA = EC
1 1 1
\ h2 < h1 fi  ​ __  ​ mv2 + ​ __ ​  __
​   ​  MR2 w2 = mgh
2 2 2
(b) kB = mgh1
1 1 4mgh
3
fi ​ __ ​  mv2 = mgh1 [Refer to Example 31. kE of a fi  ​ __ ​  mv2 + __
​   ​  Mv2 = mgh  fi  v2 = _______
​    
 ​
4 2 4 2m + M
3
purely rolling cylinder is __
​   ​  Mv2]

÷ 
_______
4 4mgh
1
fi ​ __ ​  mv2 =
2
__
​   ​  mgh1 fi  v = ​ _______
​    
 ​ ​

2 3 2m + M
6.26  Mechanics II

In Short: Here, v is the speed of the COM of the body.


(iv) Work done by a torque (t) is given by
(i) kE of a body rotating about a fixed axis is
1 dWt = t d q
k = ​ __ ​  Iw2
2 where dq is the angular displacement of the body on
(ii) In case of a general motion of a rigid body, kE is which the torque acts.
written as: dWt is positive if t and dq are in same direction and
1 1 it is negative if they are in opposite directions.
k = kR + kT = __
​   ​  ICM ◊ w2 + __
​   ​  Mv2CM
2 2 D q

or else, if we can identify the instantaneous axis of For finite displacement:  Wt = ​Ú ​   ​ ​t dq
0
rotation, then kE of the body can be simply written
(v) Change in rotational kE of a body = work done by
1
as k = ​ __ ​  I w2 torque acting on it.
2
where I = moment of inertial about instantaneous axis (vi) Rate of work done by a torque (i.e., power of
of rotation. the torque) = t ◊ w
(iii) kE of a purely rolling solid ball is where w = instantaneous angular speed of the
body.
7
k = kT + kR = ___
​    ​ Mv2 (vii) In pure rolling motion, friction does not dissipate
10 energy.
3
kE of a purely rolling disc or cylinder is __
​   ​  Mv2. (viii) Use conservation of energy for a conservative
4
kE of a purely rolling ring is Mv2. system. Be careful while writing kE. You must write
rotational as well as translational kE.

Your Turn

Q.34  A rigid body can rotate about a fixed axis. Its Q.38  In example 22, find work done by friction on the
moment of inertia about the axis is I = p  kgm2. A constant ball. Mass of ball is M.
torque t = 20 Nm acts on it till it makes two full rotations.
Q.39  A carpet of mass M,
Find the kE of the body.
made of inextensible material,
Q.35  In the last problem, what is the maximum power is rolled along its length in the
of the torque? form of a cylinder of radius R
and is kept on a rough horizon-
Q.36  When you brake your car such that it comes to rest tal floor. A gentle push is given
while the wheels do not slip, where goes the kinetic energy
to it and it begins to unroll, without sliding on the floor.
of the car?
Calculate the horizontal velocity of the axis of the cylindrical
Q.37  A uniform rod of mass B part of the carpet, when its radius reduces to __
R
​   ​ .
M and length L is free to rotate 2
M
about its one end in a vertical L Q.40  A small ring rolls down without slipping from the
plane. It is released from posi- 30° top of a track in a vertical plane. The track has an elevated
A
tion shown in figure with the rod section and a horizontal part. Horizontal part is h above
making an angle of 30° with the the ground and the top of the track is at height H. Find
horizontal (see fig). the distance of the point (D), where the ring lands on the
(a) Find angular speed of ground, from point C.
the rod when it becomes
vertical.
(b) Find force applied by the
hinge on the rod when it becomes vertical.
Rotational Dynamics  6.27

7. ANGULAR MOMENTUM
We know the importance of linear momentum in dealing with
the translational motion of a particle or a system of particles.
Angular momentum is an analogous concept in dealing with
rotational motion.
Angular momentum, as we shall see, is moment of
momentum and an external torque is needed to change
angular momentum just like an external force is required to
change linear momentum.

7.1 Angular momentum of a particle


​_›
Consider a particle moving with momentum ​_›
​P   ​. Position
vector of the particle (at any instant) is ​r ​   with respect to a However, if the motion is not restricted to xy plane, angu-
point O. Angular momentum of the particle about point O lar momentum of the particle_​ about O can be in any direction.
_
​› ​_› ›
is defined as The component of ​L   ​ = ​r ​   × ​P   ​ parallel to z-axis is defined as
​_› _ ​_› _ _
​› ​› ​› angular momentum of the particle about z-axis (Lz)
​L   ​ = ​r ​   × ​P  ​  = ​r ​   × (m ​v  ​)  ...(15)
​_› You must have noticed that the concept of angular momen-
where m and ​v  ​  are mass and velocity of ​_›
the particle. tum is mathematically similar to that of a torque.
_
​›
Consider the plane​_›defined by ​r ​  and ​P  ​ to be our xy-plane. _ ​_› ​_› ​_›
​› ​_› ​_›
Angular
_
​›
mometum
​_› (​L   ​) is a vector directed perpendicular ​t ​    = ​r ​   × ​F  ​   and ​L  ​  = ​r ​   × ​P  ​ 
to ​r ​   and_
​›
​P   ​
 ​_

and it must be along z-direction. In the figure
shown, ​r ​  × ​P   ​(by right hand rule) is along positive z-direction. To find angular momentum of a moving particle about a
If the_​ particle were moving in exactly opposite direc- fixed line (axis), we need to remember the following three
​_›
› points [It is quite similar to calculation of torque about an
tion, ​r ​   × ​P   ​ would have been along negative z-direction.
axis.] ​_›
(i) If line of ​P   ​ intersects the given axis, there is no
angular
_
momentum about the axis.
​›
(ii) If ​P   ​ is parallel to the given axis, there is no angular
momentum about the axis.
​_›
(iii) When line of ​P   ​ is perpendicular to the axis (but not
intersecting it) at a distance r^ from it, the angular
momentum about the axis is Pr^.
_
​›
Example 34   A particle has momentum ​P  ​  = Px  + Py 
​_›
and its position vector at an instant is ​r ​   = x  + y . Find the
angular momentum of the particle
Magnitude of angular momentum of the particle
about O is (a) about the origin  (b) about the z-axis.
L = r P sin q =  P^ ...(16) Solution
​_›
where P^ = component of momentum perpendicular to ​r ​ .  Concepts
​_› ​_› ​_›
One can also interpret the above expression as ​L  ​   = ​r ​   × ​P  ​ 
L = P (r sin q) = P r^ ...(17) ​_› ​_›
_
​› (a) L = ​r ​   × ​P  ​  = (x  + y ) × (Px  + Py  )
where r^ is perpendicular distance of line of ​P  ​  from O.
= (x Py – yPx) 
Note: unit of angular momentum is kgm2s–1.
(b) Angular
​_›
momentum about z-axis = z component
In the examples cited above, angular momentum of the
of ​L   ​
particle about O is directed in z-direction. Angular momen-
tum about point O is same as angular momentum about \ Lz = xPy – yPx
z-axis. ​_›
Alternate:  Think of the two components of ​P  ​  separately.
6.28  Mechanics II

Angular momentum correponding to Px is


= (Px) (Perpendicular distance of line of Px from O)
= Px y Direction is clockwise (or along negative z)
L
Angular momentum corresponding to Py is Line of motion of both particles are at a distance ​ __  ​
2
= Py (perpendicular distance of line of Py from O) from O.
= Py x Direction is anti-clockwise L
\  Each has angular momentum = mu ​ __ ​  about O.
(or along positive z) 2
\  L = (xPy – yPx) in anti-clockwise direction Both angular momenta are anticlockwise (or along a
(or positive z). direction perpendicularly out of the plane of the paper). A
simple observation tells you that the particles are, in a way,
Example 35   A stick of length L is lying on a floor. Two rotating anticlockwise about O. This judgement is good
horizontally moving particles have mass m each. They are enough to write the direction of angular momentum.
moving perpendicular to the stick in opposite directions, as
L
shown. Find sum of angular momentum of the two particles \  Required angular momentum is 2 × mu ​ __ ​  = muL
2
about centre (O) of the stick. (anticlockwise).
Solution
Note:  If the particles strike the stick, they will cause it
Concepts to rotate. This is the significance of angular momentum.
(i) L = Pr^. Particles are having angular momentum about O, implies
that they can cause rotation about O.
(ii) Angular momentum in same direction will add.

Your Turn

Q.41  A particle of mass m is moving the maximum angular momentum of the particle about point
in a circle of radius r with velocity v. of suspension.
Find its angular momentum about (a) Q.43  A projectile of mass m is projected with velocity u
centre O (b) about a line passing through at angle q with horizontal. Find its angular momentum about
O and perpendicular to the plane of its point of projection:
motion.
(a) When the projectile is at highest point
Q.42  A pendulum has length l and bob of mass m. It is (b) when the projectile is about to hit the ground.
released from a position where the string is horizontal. Find

7.2 Angular Momentum of a Rotating Rigid Body where I = Â mir2i is the moment of inertia of the body about
z-axis.
Angular momentum of a rigid body (or any system of
particles) about an axis is the sum of angular momentum Direction of Lz is as shown in
of individual particles. the figure. Curl your right hand
fingers in the sense of rotation of
Consider a rigid body rotating with angular speed (w)
the body while trying to hold the
about a fixed axis (z-axis). Its angular momentum about
rotation axis in your hand. The
z-axis is
thumb points in the direction of
Lz = Â mivi ri = Â mi (w ri) ri
angular momentum vector.
or Lz = [​  Â mi r2i  ]​ w = I w ...(18)
Rotational Dynamics  6.29

Notice that linear momentum (P) is mass (inertia) times


velocity and its rotational analogue-angular momentum (L)
is rotational inertia (I) times angular velocity (w).

7.3 Angular momentum in combined Rotation and


Translation
1
Any plane motion of a rigid body can be understood as a LA = __
​   ​  MR2w + MvR
combination of rotation about COM and translation of COM. 2
We have already learnt how to write kinetic energy of a body 1
using this thought. Angular momentum of the body about a = __
​   ​  MR (Rw) + MvR
2
point is also written using this approach.
1 3
Angular momentum of a body about some fixed = __
​   ​  MvR + MvR = ​ __ ​  MvR [Clockwise].
2 2
point is
​_› ​_› ​_›
​L  ​   = ​L  ​spin
  + ​L  ​orbital
  ...(19) Both Lspin and Lorbital remain same for point B also.
​_›
where ​L  ​ spin = angular momentum associated with spin 3
\ LB = LA = ​ __  ​ MvR
_ (i.e., rotation) about COM. 2
​›
and ​L   ​orbital = angular momentum arising due to motion of Note:  LB = ICM ◊ w + MvR
COM.
Lspin = ICM w, if the body is rotating about a single Let us write this in terms of w
axis passing through COM. ICM is the moment
of inertia of the body about the axis. LB = ICM w + M (wR) R = (ICM + MR2) w
​_› ​_› ​_›
​L  ​ orbital = ​r ​   × (M​v  ​ CM) ...(20) fi LB = IB ◊ w
​_›
where ​r ​   is position vector of COM with respect to the Where IB is moment of inertia of the rolling body about
fixed point. ​_ its instantaneous axis of rotation through B.

Obviously, ​L  ​ orbital is the angular momentum of a particle Therefore, angular momentum of a body in pure rolling
​_›
(located at COM) of mass M moving with velocity ​v  ​ CM. about contact point can be written assuming the body to be
in rotation about instantaneous axis through the point of
Example 36   Angular momentum of a rolling disc contact.
A disc of mass M and radius R is
Example 37   A sliding block
rolling without sliding on a hori-
zontal surface. Write its angular A cube of Mass M and velocity v is moving on a horizontal
momentum about a point A on the surface. find its angular momen-
surface. tum about an axis through point A,
Also write its angular momen- which is perpendicular to the plane
tum about contact point B. of the figure.

Solution Solution

Concepts Concepts

LA = Lspin + Lorbital Lspin = 0 as there is no rotation about COM. Only Lorbital


is there.
Lspin = Angular momentum due to rotation about an
Consider a particle of mass M moving with velocity v
axis through COM.
a
along a line that is at a distance __ ​   ​  from A.
1 2
= ICM ◊ w = __
​   ​  MR2 ◊ w [Direction is clockwise]
2 a
L = Mv ​ __ ​  (clockwise)
Lorbital is written considering a particle (at COM) of mass 2
M moving with velocity v

Lorbital = (Mv) r^ = MvR [Direction is clockwise]


Angular momentum of disc about A is
6.30  Mechanics II

Your Turn

Q.44  A disc of mass M and Q.46  A ring of mass M and radius R is placed on a hori-
radius R is rotating about a fixed zontal surface and imparted an angular speed w in clockwise
R sense apart from a velocity v towards left. Find angular
axis at a distance of __
​   ​  from the momentum of the ring about A, assuming v = 3wR
2
centre of the disc that is perpen-
dicular to the plane of the disc.
Find angular momentum of the
disc about the axis if its angular
speed is w

Q.45  A disc of mass M and radius


R has a light thread wrapped on it. Q.47  A ball of mass M radius R is rolling without sliding
Free end of the thread is fixed at O with velocity v. Find its angular momentum about point A
and the disc is allowed to fall. Find shown in the figure.
the angular momentum of the disc
about point O at the instant its centre
has velocity v.

In Short: 8. Torque and angular momentum


​_›
(i) Angular momentum of a particle about a point
​_› _ ​_› d ​P   ​ ​_›
​›
is ​L   ​ = ​r ​   × ​P  ​.  Recall the relation ​ ___ ​ = ​F  ​ ext. There is a rotational analogue
dt
(ii) For a particle, L = r^P and L = r P^. to this. Just as a force causes momentum to change, a torque
causes angular momentum to change. Angular momentum of
(iii) Angular momentum of a particle about an axis is zero
a particle about a point is:
if its line of motion intersects the axis or is parallel
​_› ​_› ​_›
to the axis. ​L  ​   = ​  ​   × ​P  ​ 
L = Pr^ where r^ is the distance of line of motion ​› _​› ​_› _
of the particles from the axis. d ​L   ​
___ ___ d ​r ​   _​ ›
d ​P  ​  ___
fi ​   ​ =   r × ​   ​ + ​   ​ × ​P  ​ 
dt dt dt
(iv) For a rigid body in rotation about a fixed axis, ​_›
L = Iw where _I = moment of inertia about the axis. ​_› d ​P   ​ ​_› ​_›
​› = ​r ​   × ___
​   ​ + ​v  ​  × (m​v  ​) 
Direction of ​L   ​ is along the rotation axis. Proper dt
​_› _
​› ​_› ​_›
direction is given by right hand rule. Curl you right = ​r ​   × ​F  ​ ext + 0 [  ​v  ​  × ​v  ​  = 0]
hand fingers in the sense ​_›
​_› of rotation and the thumb
d ​L   ​ _​ ›
gives the direction of ​L   ​. \ ​ ___ ​ =   ​t ​  ext ...(21)
dt
(v) For plane motion of a rigid body, Thus, rate of change of angular momentum of a particle
​_› ​_› ​_›
​L  ​   = ​L  ​ spin + ​L  ​ orbital about a point is equal to the external torque acting on it
about the point.
Lspin = ICM ◊ w. This is the angular momentum of a
body related to its spin in COM frame. For a system of particles
_
​› ​_› ​_› _
​›
Lorbital is written using ​r ​   × ​P  ​  where ​P  ​  = M ​v  ​ cm is For a system consisting of N particles (the system can
linear momentum of COM. be a rigid body as well), angular momentum about a fixed
point is _ _ _ _
(vi) For a body in pure rolling, angular momentum about ​› ​› ​› ​›
point of contact can be written as Iw, where I is ​L   ​ = ​L   ​1 + ​L   ​2 + ... + ​L  ​ N
​_›
moment of inertia about instantaneous axis through where ​L  ​ i is angular momentum of ith particle about the fixed
point of contact. point.
Rotational Dynamics  6.31

​_› ​_› ​_› ​_›


d ​L  ​1  ____
d ​L  ​  ____ d ​L  ​2  d ​L   ​N
\ ​ ___ ​ =  ​      ​ + ​      ​ + ... + ____
​      ​ 
dt dt dt dt
​_› ​_› ​_›
= ​t ​ 1  + ​t ​ 2  + ... + ​t ​ N 
​_› ​_›
Here, ​t ​  i is net torque on ith particle. ​t ​  i may have two
sources–
(i) Torque due to internal interactions amongst
particles
(ii) Torque due to external forces. dL mu2 sin 2q
\ ​ ___ ​   = t = _________
​  g   
​ 
Torques due to internal interactions cancel out in pair and dt
the above relation is simply
​_› Example 39   An amusement park ride has a big wheel
​_›
​ ___ ​ =
d ​L   ​
  ​t ​ ext
  ...(22) having moment of inertia 104 kgm2 about its rotation axis.
dt When switched on, its electric motor sets it into motion. The
This relation is true for a single particle or a wheel acquires its full speed of 4 rad s–1 in 2 minutes. Find
collection of particles – rigid or non-rigid – interacting or the average torque applied by the motor on the wheel.
non-interacting.
Solution
For a rotating rigid body Concepts
For a rigid body rotating about a fixed axis, there is a
much simpler way to get the above relation. We can easily find change in angular momentum of the
D L
wheel and use tav = ___
​   ​ 
dL dw D t
L = Iw  fi ​ ___ ​ = I ​ ___ ​ 
dt dt
Li = 0 ; Lf = Iw = 104 × 4 kg m2 s–1
dL dL
fi ​ ___ ​ =   I a  fi ​ ___ ​ = text
dt dt D L = Lf – Li = 4 × 104
Note: 4 2  –1
dL
(i) Equation ___ D L 4 × 10 kg m s
​   ​  = t shall be used in an inertial frame \ tav = ___
​   ​ = ​  ______________
   ​
   = 333.3 Nm.
dt D t 2 × 60 s
of reference or in a frame attached to COM of the
body. 8.1 Angular Impulse – Momentum theorem
​_›
​_› ​_
d ​L  ​ 
___ ›
d ​L   ​ ​_› _
​› ​_›
(ii) ​   ​ = ​t ​   has three components ​ ___ ​  = ​t ​    fi  d ​L  ​  = ​t ​   dt
dt dt
dLx dLy dLz Integrating on both sides gives
​ ___ ​ = tx ; ___
​   ​ = ty ; ___
​   ​ = tz
dt dt dt _
​›
Dt
​L  ​ f _
​› ​_›
​Ú_  ​ ​d​L  ​   = ​Ú ​  ​ ​t ​​  dt

​›
Example 38   A projectile of mass m is projected with  ​L  ​i  0

velocity u at an angle q to the horizontal. Find the rate of ​_› ​_› Dt


_
​›
change of angular momentum of the particle about the point fi ​L  ​ f – ​L  ​ i = ​Ú ​  ​ ​ ​t ​   dt ...(23)
0
of projection when it is at the highest point.
The left side of the above equation is the change in angu-
Solution lar momentum and right side is known as angular impulse
Concepts of torque.
Recall that force × time is linear impulse and is equal to
Only force acting on the particle is its weight. Torque of
change in linear momentum.
weight is equal to rate of change of L. _
​›
The above equation implies that when a torque ​t ​   acts on
When the particle is at the top, torque acting on it about a​_›body (or a system), it causes change in angular momentum
O is (​L   ​) and the change depends on torque as well the interval
R of time for which it acts.
t = mg ​ __ ​ 
2 If a constant torque acts on a body for time D t, then
change in angular momentum is
u2 sin2 q ________
mu2 sin2 q ​_›
= mg _______
​     ​ = ​ 
   ​    ​_› ​_›
2g 2 D ​L  ​   = ​t ​   D​t ​  
6.32  Mechanics II

When a large torque acts on a body for a very small Also, line of action of N and Mg cos q are always same.
interval of time such that there is practically no rotation of They produce equal and opposite torque about A. Only force
the body in that time, we call it an impulsive torque. that has a torque about A, is Mg sin q.

Example 40   A fan has moment of inertia 4 kg m2 about its tA = (Mg sin q) R.
rotation axis. When switched on, its motor applies a constant
In time t, this will produce a change in angular momen-
torque of 10 Nm. Find the angular speed of the fan 4 s after
tum of the disc (about A) that is
it is switched on.
D L = tA D t
Solution
Lf – Li = (Mg R sin q) t
Concepts
fi Lf – 0 = Mg R t sin q.
D L = t D t
One can also find angular acceleration using t = I a \ Lf = Mg R t sin q.
and then use w = w0 + at
Our answer will not change even if the disc slides.
D L = t D t  fi  Lf – Li = t D t
Example 42   Hit a ball to roll it without sliding
fi Iwf – O = 10 × 4 A billiard ball of radius R is hit hor-
fi 4 wf = 40  fi  wf = 10 rad s–1 izontally to impart a sharp impulse.
At what height h above the centre
Example 41   A disc of mass M and shall it be hit so that it starts rolling
radius R is released on an inclined plane without sliding?
at point A. It rolls down without slip-
Solution
ping. Find its angular momentum about
point A on the incline after time t. Will Concepts
you answer change if there is no friction and the disc slides (i) There is a linear impulse on the ball, which causes
down the incline? its momentum to change.
Solution There is an angular impulse about the centre of the
Concepts ball as well. This imparts a spin to the ball.
(ii) The ball begins to move with its centre having a
(i) One way of doing this problem is to find its veloc-
velocity v and its angular velocity being w. For
ity (v) and angular velocity (w) after time t and then
pure rolling v = wR.
use L = Lspin + Lorbital to write its angular moment
about A. (iii) If h = 0, there is no torque of the applied force
about the centre and angular impulse is zero. It
But it is easier to solve the problem by
means w = 0 if h is small, angular impulse will
using D L = t D t.
be small and v > wR. If h is larger, wR > v.
(ii) Forces on the disc are Mg, friction (f) and nor-
There is an appropriate value of h for which
mal reaction (N). Torque of all these forces about
v = wR.
point A can be calculated easily as friction has no
torque and torque due to N and Mg cos q always If the applied force (F) acted for
cancel out. a small time D t, its impulse will
Forces on the disc are as shown. Friction has no torque be D t
about A. J = Ú​  ​   ​ ​F dt
N = Mg cos q 0

If centre of mass of the ball acquired a velocity v then,


D t
J = D P  fi ​Ú ​   ​ ​dt = Mv ...(i)
0
Torque of force (F) about centre is t = F ◊ h. This torque
also acted for time D t and its angular impulse is
Dt Dt
= Ú​  ​  ​ ​ t dt = h ​Ú ​  ​ ​F ◊ dt = h ◊ J.
0 0
Rotational Dynamics  6.33

2
Using angular–impulse momentum theorem about an axis fi h ◊ (Rw) = __
​   ​  R2w [For pure rolling v = Rw]
through centre gives 5
2
h ◊ J = D L  fi  h ◊ J = ICM ◊ w fi h = __
​   ​  R
5
2 Note:  It is safe to neglect impulse due to friction in the
fi h ◊ Mv = ​ __ ​  MR2 ◊ w [From (i) J = Mv]
5 small interval for which the force was applied.

Your Turn

Q.48  A variable torque centre. A light thread is wrapped around it.


(t) acts on a fan for a small Its free end is pulled vertically with a force
time interval D t = 0.01 s. that changes with time as F = at. Find the
The variation of torque with angular speed of the cylinder after time T.
time is as shown. Moment Q.50  A ball of mass M and radius R
of inertia of the fan about rests on a horizontal surface. It is
its rotation axis is 0.4 kgm2. given a sharp horizontal blow at
Find the final angular speed a height h (> 0) above the floor.
of the fan if initially it was For what value of h will the ball
at rest. acquire maximum kinetic energy?
Q.49  A solid cylinder of mass M and radius R is free For what value of h the kinetic
to rotate freely about the fixed horizontal axis through its energy is minimum?

9. CONSERVATION OF ANGULAR holding two dumbbells in his hands. Moment of inertia


of the man – platform
MOMENTUM system is I1 and angu-
› ​_
d ​L   ​ ​_› lar speed is w1. Now,
We known that ___
​   ​ = ​t ​   the man folds his arms
dt
​_› ​_› to bring the dumbbells
If ​t ​   = 0 then ​L  ​  = a constant. closer. Moment of iner-
When net external torque on a system is zero, the angular tia of the system about
momentum of the system remains conserved. This is principle the rotation axis is now
of conservation of angular momentum. equal to I2
Like conservation of linear momentum, we can use (a) Find the new angular speed (w2) of the system
angular momentum conservation principle in component (b) What happened to kinetic energy of the system?
form. If torque about z-axis on a system is zero then its What is change in kE? Give physical reason for this
angular momentum about z-axis is conserved. Lz = a constant change.
if tz = 0 [tx and ty may not be zero].
Solution
This conservation principle has wide applicability
in understanding of macroscopic as well as microscopic Concepts
world. (i) As the man fold his arms and brings the dumbbells
In problems in mechanics, we will use this conservation closer, the moment of inertia of the system about
principle even when t π 0 but when we are convinced that the rotation axis decreases. I2 < I1
the impulse of the external torque in the duration of an event (ii) Angular momentum of the system about the fixed
is very small and can be neglected. rotation axis is conserved, as there is no external
We will use this conservation principle about a fixed axis torque on it.
or about an axis passing through COM of a system.
(a) Conservation of angular momentum (about rotation
Example 43   Man on rotating platform axis) gives:
A man is sitting on a platform that is spinning freely Lf = Li
about its vertical axis. He has his arms stretched and is
fi I2 w2 = I1w1  fi  w2 = ​ __
I1
(  )
​   ​   ​ w1
I2
6.34  Mechanics II

1
(b) ki = ​ __ ​  I1w21 (a) Let final angular speed of the system be w. conserva-
2
tion of angular momentum gives:
1 I1
[  ] I1 1
(  ) I1
2
1
kf = ​ __ ​  I2w22 = __
​   ​  I2 ​​ __
​   ​  w1  ​​ ​ = __
​   ​  ​ __
​   ​  I w2  ​ = ​ __ ​  ki Lf = Li
2 2 I2 I2 2 1 1 I2
(I1 + I2) w = I1w1 (≠) + I2w2 (Ø)
I1
Since, __
​   ​  > 1, the kE of the system has increased. [Direction of I1w1 is up, along the rotation axis. Curl
I2

(  ) (  )
your right-hand fingers in the sense of rotation of the
I1 I1 – I2 __ 1
D k = kf – ki = ​ __​   ​  – 1  ​ ki = ​ ​ _____  ​ ​   ​  I w2
 ​   upper disc, trying to hold the axis. Thumb points up.
I2 I2 2 1 1 This is the direction of L for upper disc. Similarly, L
kE has increased due to work performed by the man for lower disc is directed down along the axis]
in pulling the dumbbells inward.
fi  (2 + 6) w = 2 × 3 (≠) + 6 × 4 (Ø)
Remember, internal forces can change kE of a system.
9
Note:  A gymnast uses this principle to increase/decrease fi  8w = 6 (≠) + 24 (Ø)  fi  w = __ ​   ​  rad s–1(Ø).
4
her spinning speed. When she pulls her arms close to her 9
__ –1
body, her rotational inertia decreases and angular speed Final angular speed is ​   ​  rad s in the sense of w2.
4
increases. 1 1
(b) ki = ​ __ ​  I1 w1 + ​ __  ​ I2w22
2

Example 44   Two spinning 2 2


discs 1 1
= __
​   ​  × 2 × 9 + __
​    ​ × 6 × 16 = 57 J
Two discs are rotating about their 2 2
common axis of symmetry as
1
shown. Moment of inertia of the kf = ​ __ ​  (I1 + I2) w2
2
two discs are I1 = 2 kg m2 and
I2 = 6 kg m2 and they have angu-
lar velocities w1 = 3 rad s–1 and
1 9 2
= ​ __ ​  (2 + 6) ​​ __
2 (  )
​   ​   ​​ ​ = 20.25 J
4
w2 = 4 rad s–1 in opposite direc-
tions as shown. The upper disc Loss in kE = ki – kf = 36.75 J
is allowed to gently fall over the
lower disc. The two discs sub- This is heat dissipated during the period the two discs
sequently spin with one common slip on each other.
angular speed (w).
Example 45   Alternative solution to Example 22
(a) Find w.
Use conservation of angular momentum to solve example 22.
(b) Find the amount of heat dis-
sipated during the period Solution
the two discs slip on one Concepts
another.
(i) As learnt earlier, friction will eventually convert
Solution
the sliding motion into a pure rolling motion.
Concepts
(ii) There is no external torque on the ball about a
(i) Initially, w2 > w1 and the discs slip. Kinetic fric- point (A) on the horizontal surface. Line of friction
tion between them develops a torque. This torque passes through this point and normal force and Mg
speeds up the upper disc and slows down the lower produce equal and opposite torque.
disc. thus, the two discs eventually acquire a com- (iii) Use equation (19) while writing the angular
mon angular speed. momentum of the ball about A.
(ii) During the process, external torque on the system
of two discs about the rotation axis is zero. Angular Angular momentum of the ball is conserved about a
momentum of the system is conserved. point on the horizontal surface (say point A). Initial angular
(iii) Due to rubbing between the two discs, kinetic momentum about A is Li = Lspin + Lorbital = 0 + MvR
energy gets dissipated as heat. Heat dissipated is Final angular momentum about A when pure rolling starts
equal to loss in kE. is
In the last problem, kE of the system increased and 2
Lf = Lspin + Lorbitals = __
​    ​ MR2 ◊ w0 + Mv0R
in this problem, it decreases. You must ponder over the 5
working of internal forces in the two cases.
Rotational Dynamics  6.35

2 7 Let angular speed of the platform


= __ ​   ​  MRv0 + Mv0R = __​   ​  Mv0R.
5 5 be w (clockwise) after the man begins
Lf = Li to walk.
7 5v
​ __ ​  Mv0R = MvR  fi  v0 = ___ ​   ​  vm = velocity of man in tangential
5 7 direction.
Velocity of a point on the circum-
ference of the platform just beneath
the shoes of the man is wR in a direc-
tion opposite to the velocity of the
man.
v = velocity of man relative
to the platform

fi v = vm – (– wR)  fi  vm = v – wR.
Example 46   A platform of mass M and radius R is free Lfinal = Linitial  [about rotation axis through O]
to rotate about its central vertical axis. Initially, the platform
MR2
is at rest with a man of mass m standing at the edge. The mvm ◊ R – ____
​   ​ w

  =0
man starts walking along the edge with a tangential velocity 2
v relative to the disc. Find angular velocity acquired by the 2mv
platform. Treat man as a point object. fi w = ​ __________
    ​

(2m + M) R
Solution
Note:
Concepts (i) In the figure shown, angular momentum of the man
(i) Angular momentum of (man + platform) system about the rotation axis is out of the plane of the figure
is zero, initially. It will continue to be zero even (or, simply anti-clockwise). Angular momentum of
when the man starts walking. the platform is into the plane of the figure (or, simply
(ii) If the man moves anti-clockwise, the platform clockwise).
will begin to rotate clockwise. This happens due (ii) kE of the system has increased. Why? The man has
to friction between the shoes of the man and the performed work. When you walk on a hard ground,
platform. But when we consider man + platform you work to impart kE to yourself. While walking
as our system, we need not worry about friction. on this platform, you have to work to impart kE to
It is an internal force. self as well as to the platform.

Your Turn

Q.51  If all of us – the inhabitants of Earth – move to spinning faster now, than when it was a large spherical
equator, how would this affect the length of the day? And cloud?
what if all of us move to the poles?
Q.53  A platform (in shape of a disc) of mass M and
Q.52  We believe that our galaxy was formed from a huge radius R is rotating freely about a vertical axis through its
cloud of gas. The original cloud was more or less spherical centre with an angular speed w0. An insect of mass m is
and very large in size. sitting at the centre of the platform. The insect walks along
It was spinning about a radius to the edge of the platform. Find the new angular
some axis. The present speed of the platform.
shape of the galaxy
is somewhat like as Q.54  A boy is standing on the edge of a rotating platform.
shown in the second He drops a ball. Will the angular speed of the platform
figure. Is it correct to increase or decrease due to this event?
say that the galaxy is Q.55  A uniform disc of mass M and radius R is spinning
about its central horizontal axis with angular speed w. A chip
6.36  Mechanics II

of mass m breaks off the edge of the shorten the radius of circular path of the block. Find the
disc at the instant its (chips’s) velocity radius of circular path when tension in the cord becomes
is vertically up. 600 N.
(a) What is the new angular veloc- Q.57  A disc-shaped pulley has
ity of the disc? mass M and radius R. A light thread
(b) How high does the chip rise? is tightly wrapped around it and its
free end carries a small ball of mass m.
Q.56  A small block of mass 4 kg The ball is held in a position where the
is attached to a cord passing through
thread is loose and the pulley is at rest.
a hole in a horizontal smooth table.
The ball is released from rest. It falls a
The block is revolving in a circle of
distance L before the thread gets taut.
radius 0.5 m about the hole with a
Find the speed of the ball immediately
speed of 4 ms–1. The cord is slowly
after the string gets taut.
pulled below the table so as to

10. COLLISION OF A PARTICLE WITH A Example 47   A hinged rod hit by a bullet

RIGID BODY A rod of mass M and length L is hinged


at its one end at O such that it can freely
When a moving particle hits a rigid body, it can impart it swing in the vertical plane. A horizontally
a rotation apart from translation. consider a rigid body that flying bullet of mass m hits its lower end
can rotate freely about an axis (perpendicular to the figure) and comes to rest. speed of the bullet just
through point O. A particle P before impact is u.
strikes it at A. In general, the (a) Find the angular speed of the rod
axis (at O) will exert a large immediately after the impact.
impulsive force on the body dur-
(b) Find the angle (q) with the vertical
ing the collision. [The axis will
that the rod will make in its extreme position after
be a pin or a shaft]. Therefore,
the impact.
the linear momentum of the
(particle + body) system will Solution
not be conserved. However, the Concepts
impulsive force by the axis will
have no torque about itself. It (i) The rod can rotate about a horizontal axis through
means, the angular momentum O that is perpendicular to the plane of the figure.
of the system will remain conserved about the axis through angular momentum of the (rod + bullet) system is
O. It is important to understand that the angular momentum conserved about the axis.
cannot be conserved about any other axis that is not passing (ii) After collision, the mechanical energy of the rod
through O. It will not be safe. The impulsive force can have remains conserved.
torque about any such axis.   Remember, the kinetic energy of the bullet before col-
It coefficient of restitution is e, then: lision is higher than that of rod after collision. Kinetic
Relative speed of separation of P and point A after energy is lost during collision.
collision
= e (relative speed of approach of P and A before
collision)
We are not concerned with velocity of any other point in
the body while writing equation for e. It is point A that is
important.
If a moving particle strikes a rigid body that is free
(not hinged) then linear momentum is conserved. Angular
momentum conservation can be applied about some fixed
point or the COM of the system under consideration.
Equation for coefficient of restitution shall be written as (a) Conserving angular momentum about axis through
explained above. O.
Rotational Dynamics  6.37

​L​before ​ = ​L​ after ​ One can also think of a point attached to the table,
 ​       ​ ​        ​
collision
​ ​ ​collision ​
just below the centre of the stick. This is a point in inertial
1
fi muL = ​ __ ​  ML2 ◊ w frame and yields same equation as obtained by considering
3 centre of the rod as pivot point for application of conserva-
3mu
____ tion of angular momentum.
fi w = ​   ​ 
mL
(b) After impact, the rod has a kE. It gets converted
into PE, as the rod rises. The rod stops after rotating
through angle q where its entire kE gets converted
into its PE.
L L
COM of the rod rises by __ ​   ​  – ​ __ ​  cos q
2 2
L
__
= ​   ​  (1 – cos q)
2
(a) COM of the system is at C (centre ​_›
of the stick)
L 1
\  Mg ​ __ ​  (1 – cos q) = __ ​   ​  I w2 after collision. Let its velocity be ​v  ​ 0. Conservation
2 2 of momentum.
L
2
1 ML2 ____
fi  Mg ​ __ ​  (1 – cos q) = __
​   ​ ​  ____
2 3
​   ​   (  ) (  )
3mu 2
 ​ ​​ ​   ​  ​​ ​
ML


Pf = Pi
Pf = 2mv – m (2v) = 0
2 2
3m u
fi  1 – cos q = ____
​  2 ​ ___
​    ​  \ v0 = 0
M gL
(b) Angular momentum is conserved about C.
3m2 u2
fi  cos q = 1 – ​ ______ 
 ​ Li = L f
M2 gL
2m va + m (2v) (2a) = I w
Example 48   A uniform bar of length 6a and mass 8 m lies
on a smooth horizontal table. Two point masses–m and 2m– fi 6m va = Iw ...(i)
moving in opposite directions hit the bar simultaneously. The I = moment of inertia of rotating system about an
velocities of the particles are 2v and v, perpendicular to the axis through C perpendicular to plane of the
bar as shown in figure. figure
The particles stick to the 8m (6a)2
bar. Find = ​ _______
 ​ + 2ma2 + m (2a)2 = 30 ma2

12
(a) velocity of centre
of the rod after Putting in (i) gives
collision.
v
(b) angular velocity w = ​ ___  ​ 
5a
of the rod after collision.
(c) Find kinetic energy of the system after collision. (c) The system is just rotating about C. There is no
translation.
Solution
Concepts \
1
2
1
2 (  )
5a
v 2 3mv2
K = ​ __ ​  Iw2 = ​ __ ​  × 30 ma2 × ​​ ___
​    ​  ​​ ​= _____
​   ​ 
5

(i) After the particles stick to the rod, the COM of the
system is at the center of the rod. This is because
Example 49   Conserve angular momen-
the particle of mass m is at a distance 2a from the
centre of the rod and the particle of mass 2m is at tum about any point!
a distance a from the centre. A particle of mass m travelling with speed
u on a smooth horizontal table, hits one
  v CM can be found using conservation of
end of a stationary stick of mass m and
momentum.
length l. The stick is free. The particle
(ii) We can use conservation of angular momentum strikes the stick perpendicularly and sticks
about any point, as there is no external force on to it. Find angular speed of the system
the system. We will take the COM of the system after collision.
(i.e., centre of the stick) as our preferred point.
6.38  Mechanics II

5
Solution Here, ICM = ​ ___  ​ ml2 (as in last method) and Lorbital is
24
Concepts written assuming a particle of mass 2m travelling at speed
l
(i) There is no external force on the system. Angular v0 at a distance __
​    ​  from the origin.
momentum is conserved about any point. To 4
demonstrate the process, we will solve the problem To get v 0, we need to apply conservation of
by selecting three different points for conserving momentum:
angular momentum. u
mu = 2mv0  fi  v0 = __
​   ​ 
(ii) v cm can be obtained using conservation of 2
momentum. Putting in (i) gives

Method 1
COM of the stick + particle system, just before the particle

2
l
(  5
mu ​ __  ​   = ​ ___ ) u l
​    ​ ml2  ​ w + 2m ​ __
24 (  )
​   ​   ​ __
2 4
​    ​ 

l
strikes, is at a distance ​ __  ​  from the centre of the stick. A is fi
6u
w = ___
​   ​ 
4
5l
a point on the table just beneath the COM. We will conserve
Method 3
the angular momentum about this point.
Let the origin (point selected for conservation of angular
Li = Lf momentum) be the COM of the system.
u
l Before collision, velocity of COM is ​ __  ​ towards right.
mu __
​    ​   = Lspin + Lorbital 2
4 In a reference frame attached to COM, the particle is seen
u
moving with velocity ​ __ ​  to right and the centre of the rod is
2 u
seen moving to left with velocity __ ​   ​ .
2

The velocity of COM (= v0) has its line passing through A.


Therefore, the second term on the right side of the above
equation is zero.
l
\ mu ​ __  ​   = ICM ◊ w
4 \  Initial angular momentum of the system before

[ 
collision, wrt COM of the system is

4
l ml2
mu ​ __  ​   = ​ ___
12 4
l 2
​   ​ + m ​​ __ (  ) (  ) ]
l 2
​    ​   ​​ ​ + m ​​ __
​    ​   ​​ ​  ​ w
4
u l
Li = m ​ __ ​  __
u l
​    ​  + m ​ __ ​  ​ __  ​ 
2 4 2 4
mu l ____5ml2 6u
fi ​ ____
 ​  
 = ​   ​ ◊ w 

  fi  w = ___
​   ​  l
4 24 5l = mu ​ __  ​  ( )
4
Method 2 After collision, in COM frame, angular momentum is
This time, we will choose the origin to be a fixed point on
the table just below centre (C) of the stick. 5ml2
Lf = ICM ◊ w = ____ ◊ w
​   ​ 

24
Li = Lf
5 l
l \ ​ ___  ​ ml2 w = mu __
​    ​ 
mu ​ __  ​   = Lspin + Lorbital 24 4
2
6 u

l l
mu ​ __  ​   = ICM ◊ w + 2mv0 __
​    ​   ...(i) fi w = ​ __ ​  __
​   ​ 
2 4 5 l
Rotational Dynamics  6.39

Your Turn

Q.58  A disc of mass M and radius R is fixed to rotate Q.60  A ball of mass M lies on a smooth horizontal
about its central vertical axis through surface. A particle of mass m, travelling horizontally at a
O. A horizontally flying bullet of speed u, collides and sticks to the ball at a height h above
mass m has speed u. It kisses the the surface. Assume that M >> m so that the COM of the
disc at the circumference and loses system practically remains at the geometrical centre of the
half its kinetic energy. The bullet sphere after the particle sticks to it. Find the following just
maintains its line of motion. Find the after collision:
angular speed acquired by the disc.
(a) Velocity of COM
Q.59  A rod of length L and mass (b) angular speed of the sphere
2 m lies on a smooth table. A particle
of mass m is moving with a velocity u,
making an angle q with the length of
the rod (see fig.). The particle hits one
end of the rod and stops. Find:
(a) speed of centre of the rod and
(b) angular speed of the rod after
the hit.

In Short: If t is constant, angular impulse is t D t.


​_›
d ​L  ​  _
​› (v) Angular impulse experienced by a body is equal to
___
(i) ​   ​   = ​t ​ ext
  the change in its angular momentum. This is known
dt
as angular impulse momentum theorem.
dLx dLy dLz
fi ​ ___ ​   = tx ; ​ ___ ​ = ty ; ​ ___ ​ = tz (vi) When you apply a force on a body, it can cause its
dt dt dt linear momentum as well as angular momentum to
(ii) Angular momentum of a body about a point changes change.
D t D t
only when there is an external torque on the body D P = Ú​  ​   ​ ​ F dt  and  D L = Ú​  ​   ​ ​ t dt
about that point. 0 0
(iii) When text = 0, L = a constant. (vii) If a flying particle strikes a hinged body, use
conservation of angular momentum about the hinge
This is known as law of conservation of angular
point.
momentum.
If a particle hits a free body, angular momentum is
(iv) If a torque t acts on a body for a time D t, its angular
D t conserved about any fixed point. We can also use
impulse is defined as Ú​  ​   ​ ​ t dt. conservation of angular momentum about COM of
0 the system.

Miscellaneous Examples
Solution
Example 50   (a) An isosceles triangle has mass M, vertex Concepts
angle 2 q and common side length L. Find its moment of
inertia about an axis through tip A and perpendicular to the (i) We will divide the D into thin strips, parallel to the
plane of the triangle. base.
(b) A regular polygon of N (ii) MI of each strip about an axis through its cen-
sides has mass M. Distance of tre and parallel to given axis is given by formula
any of its vertex from the cen- ml2
tre is R. Find its moment of I = ___
​   ​ – for a rod.
12
inertia about an axis through (iii) Using parallel axis theorem, we can write MI of
its centre and perpendicular to its plane. each strip about given axis.
6.40  Mechanics II

(iv) For part (b), the polygon is made up of N isosceles


p
ML2
= ____
​   ​ ​ 
2
2
  1 – __ [ 
​   ​  sin2q  ​
3 ] ...(2)
triangles with q = __
​   ​. 
N (b) The N–gon is made up of
N isosceles triangles. MI of
each D is given by equation
(2) above. When we add,
masses of each D adds and
we get.

ML2
I = ​ ____
2
2
 ​ 1 – __
 ​  [  p
​   ​  sin2 ​ __
3 (  ) ]
​   ​   ​  ​
N
Note:  For hexagon, N = 6
(a) h = L cos q


Consider a strip (like a thin rod) as shown.
Length of strip = 2x tan q

ML2
I = ​ ____
2
 ​ ​ [  2 1
  1 – __
​   ​  × __
3 4 ] 5
​    ​  ​ = ___
​    ​  ML2
12

Area of strip dA = (2x tan q) dx Example 51   A uniform cylinder has length L, radius R
Mass per unit area of the triangle: and mass M. Find its moment of inertia about an axis that
passes through its centre and is perpendicular to its length.
M 2M M
s = ​ ________ = _________
   ​  ​     ​ = _______
​  2   ​   ...(1) Solution
__1 h (2h tan q) h  tan q
​   ​  ◊ h (BC)
2 Concepts
Mass of the strip, dm = s dA = (2 s tan q) x dx (i) We will consider the cylinder to be made up of a
number of thin discs.
MI of the strip about an axis perpendicular to the
figure and passing through centre of the strip is (ii) MI of each disc of mass dm can be written about
1
1 its diameter as __
​   ​  (dm) R2.
= ___ ​    ​  (dm) (2x tan q)2 4
12 (iii) Using parallel axis theorem, we can write MI of
MI of strip about given axis through A is (using each disc about the given axis.
parallel axis theorem) (iv) Adding MI of all such discs will give the desired
1 result.
dI = ___
​    ​  (dm) (2x tan q)2 + (dm) x2
12
(2x tan q)2
= (2 s tan q ◊ x dx) ​ _________
​ 
12
 ​  + x2  ​
  [  ]
(  )
h
tan2q
\ I = 2 s tan q ​ 1 + _____  ​ Ú​  ​  ​ x3 dx
​   ​  
3 0

tan2 q __
= 2 s tan q ​ 1 + _____
​   ​  
3 ( 
h4
 ​ ​   ​ 
4 ) M
Mass of cylinder per unit length = __
​   ​ 
L
M
= ______
h  tan q
tan2 q __
 tan q ​ 1 + _____
​  2    ​  ​   ​  
3
h4
( 
 ​ ​   ​ 
2 ) Mass of disc of thickness dx is


M
dm = __
​   ​  dx
L
Mh2
= ​ ____
2
 ​ 
tan2q
 ​ 1 + ​ _____
3 ( 
 ​  
 ​ ) MI of this disc element about its own diameter is
1 1 M 2
dICM = __
​   ​  (dm) R2 = __
​    ​ __
​   ​  R  dx

[ 
4 4 L
]
2
M (L cos q) tan2q
= __________
​   ​
    ​ 1 + _____
​   ​  
 ​
2 3 MI of disc element about desired axis is

[ 
MR2
]
M
ML2 sin2q dI = dICM + (dm) x2 = ____ ​  dx + ​ __ ​  x2 dx
​     
= ____   cos2q + ​ _____
​   ​ ​   ​  
 ​ 4L L
2 3
Rotational Dynamics  6.41

L L Solving (i) and (ii) gives


L /2 2 2 2
MR M Mg mMg
\ I = Ú dI =
4L Ú dx + L Ú x 2 dx N1 = ______
​   ​ , N2 = ______
 2  ​    
 ​
x=-
L

L

L 1+m 1 + m2
2 2 2
Net torque on cylinder about its rotation axis through O

[  (  ) ] [ (  ) (  ) ]
MR2 __ L L M L 3 – L 3 is
= ​ ____   
​ ​ ​   ​  – ​ – ​ __ ​   ​  ​ +
  ___
​     ​ ​ ​​ __
​   ​   ​​ ​ – ​​ ___
​   ​  ​​ ​  ​ t = f2R + f1R ( )
4L 2 2 3L 2 2
mMg (1 + m) R
MR2 ____ ML2 = m (N2 + N1) R = ​ ____________   
   ​
I = ​ ____
 ​ 
 + ​   ​ 
  (1 + m2)
2 12
Angular retardation produced in the cylinder is given
Example 52   A spinning cylinder in a corner by
A uniform cylinder of radius R is I a = t
spun about its central axis at an
angular speed w0 and then placed 1 mMg (1 + m) R 2mg (1 + m)
​ __ ​  MR2 . a = ​ ____________
   ​  fi  a = ​ __________
        ​
in a corner as shown. Coefficient 2 2
(1 + m ) R (1 + m2)
of friction between the cylinder
and two surfaces is m. How many Since, a is constant, we can use
turns will the cylinder accomplish
before stopping. w2 = w02 + 2 a q
Solution
Concepts 2mg (1 + m)
fi 0 = w02 – 2 __________
​     ​ ◊ q
  
(i) It is obvious that the cylinder cannot translate (due R (1 + m2)
to presence of wall). Therefore, the cylinder rotates
about a fixed axis through its COM. w02 R (1 + m2)
fi q = ​  ___________
  
    ​ = angular displacement
(ii) Since acm = 0, net force on the cylinder must be 4g m (1 + m)
before the cylinder stops.
zero.
(iii) There is a net torque about COM that causes the q w20R (1 + m2)
spinning to retard. \  Number of turns, n = ​ ___  ​ = ​ ____________
  
    ​
2p 8 p g m (1 + m)
The normal forces (due to the floor and the wall)
cannot produce torque about the central axis. Their Example 53   Watch the relation between acceleration
line of action intersects the axis. A uniform cylinder A of mass M
Only friction produces torque. and radius R is suspended using
(iv) We can use w2 = w02 + 2 a q to find angular two symmetrically wrapped
displacement of the cylinder before it stops strings, as shown. Another iden-
rotating. tical cylinder B is suspended
from A using another string,
FBD of the cylinder is a which is tightly wrapped at the
as shown. Friction forces are centre of two cylinders. The
directed opposite to the veloci- string holding is wrapped on
ties of the rubbing contact points A in opposite sense to the other
of the cylinder. two strings. Axes of the cylin-
ders remain horizontal. During
For aCMx = 0
fall, find acceleration of B, and
Fx = 0 acceleration of A.

fi N2 = f1  fi  N2 = mN1 ...(i) Solution


Concepts
For aCMy = 0 ; Fy = 0
(i) As A rotates, threads at its two ends unwind. A
fi N1 + f2 = Mg moves down. if acceleration of A is a1 and its
angular acceleration is a1, then a = R a1.
fi N1 + mN2 = Mg ...(ii)
6.42  Mechanics II

(ii) As a rotates, thread at its centre rewinds. Note Solution


that this thread is wound in opposite sense. If a Concepts
falls through x, the length of middle thread that it
(i) The COM of the system is at the centroid of the
rewinds is also x. Thus, motion of A has no effect
triangle.
on motion of B.
(ii) COM is rotating in a circle about A. Hinge force
If a2 is angular acceleration of B and a2 is its linear
provides the necessary acceleration to the COM.
acceleration, a2 = R a2
(iii) When force F is applied, there is a torque about
Motion of B is not affected by A. Body acquires angular acceleration. The COM
motion of A. From example 19 gains a tangential acceleration apart from the cen-
tripetal acceleration.
2g
a2 = ___
​   ​   and In the position described, tangential acceleration of
3 COM is in x-direction and radial acceleration is along
M Mg y-direction.
T = __
​   ​  a2 = ___
​   ​ 

2 3
Let tension in upper string be T0
For A:

T + Mg – 2T0 = Ma1

4Mg
​ ____
 ​ 
 – 2T0 = Ma1 ...(i) (a) G is COM. distance of G from A is
3
1
And 2T0 R – TR = __
​   ​  MR2 a1
2

÷
l__
r = ​ ___
  
​ 3 ​ [  2
   ​ ​ = __
3 ]
​    ​ × height  ​

COM goes in a circle of radius r about A.


Mg __ 1
2T0 – ___
​   ​  
 = ​   ​  Ma1 ...(ii) Force on the body must be equal to
3 2
l__ __
2g F = MaCM = 3m w2 ◊ r = 3m ◊ w2 ◊ ​ ___   ​ = ÷    ml w2
​ 3 ​ 
Adding (i) and (ii) gives a1 = ___
​   ​  ÷   
​ 3 ​
3
This force is applied by the hinge on the body.

(  )
__
Example 54   Three particles A, B, and C each of mass m ÷   
​ 3 ​
are connected to each other by three massless rigid rods to (b) Torque about A is: t = F ​ ___
​   ​  l  ​
2
form a rigid, equilateral triangular body of side length l. This
body is placed on a horizontal frictionless table (xy plane) Moment of inertia about axis through A and
and is hinged at point perpendicular to the plane of the figure is
A so that it can rotate, I = ml2 + ml2 + 0 = 2 2
without friction, about
a vertical axis through \  Angular acceleration produced due to F is
A. The body is set into __
t ÷    F
​ 3 ​ 
rotational motion on the a = ​ __ ​ = ____
​   ​ 
I 4ml
table about the axis with
constant angular veloc- Now COM is moving
ity w. in a circle of radius
(a) Find the magnitude of horizontal force exerted by the r and at the given
hinge (at A) on the body. instant its angular
speed is w and angu-
(b) At time t, when BC is parallel to x-axis, a force
lar acceleration is a.
F is applied on B along BC. Obtain the x and y
Radial acceleration
components of the force exerted by the hinge on the
of COM is
body, immediately after time t.
ay = w2r
Rotational Dynamics  6.43

\  Radial force on it is Where spring 2 is cut, tension in 1 remains T = Mg


__ ​_›
   ml w2
Fy = 3may = ​÷3 ​  Let acceleration of COM be ​a  ​0  = ax  + ay 
[as calculated in part (a)]
Tangential acceleration of COM is
__
l ​÷3 ​    F F
ax = r a = ___
​  __   ​  ◊ ​ _____ ​ = ___
​    ​ 
÷   
​ 3 ​ 4ml 4m
\  Force on the body in
x-direction must be
Let angular acceleration of the bar be a.
3F
Fx = 3max = ___
​   ​ 
4 May = Mg – T sin 30°
x component of hinge force g
must be towards left equal to fi ay = ​ __ ​ 
2
F
FxH = __
​   ​  Max = T cos 30°
4 __
y component of hinge force is FyH = ÷    mlw2
​ 3 ​  __
÷   
​ 3 ​
ax = ___
​   ​  g
Example 55   A bar suspended with springs 2
__
A uniform slender bar AB of mass M is suspended using ​_› ÷   
​ 3 ​
___ g
two identical springs, as shown in figure. Spring 2 is cut. \ ​a  ​ 0 = ​   ​  g  + __
​      [Ans.]
2 2
Find acceleration of COM and end A of the bar immediately
after this. Writing t = I a about COM

L
(T sin 30°) ​ __(  )
​   ​   ​ =
2
ML2
____
12
3g
 a  fi  a = ___
​   ​   ​   ​ 
L

Acceleration of end A wrt COM is

​_› L 3g 3g
​a  ​ A CM = a ​ __ ​  (≠) = ___
​   ​ (≠) = – ​ ___ 
2 2 2
__
_
​› _
​› _
​› ÷   
​ 3 ​
\ ​a  ​ A = ​a  ​ A CM + ​a  ​ 0 = ​ ___ ​  g  – g
2
Solution
Concepts
(i) We can find tension in each spring when the bar
is in equilibrium.
(ii) When spring 2 is cut, the extension in spring 1 can-
not change suddenly. It means tension in spring 1
will remain uncharged immediately after 2 is cut. Example 56   Bicycle
(iii) Any motion of a rigid body can be understood as A bicycle is accelerating without slipping on a straight
translation of COM and rotation about
​_›
it. We will horizontal road. The
assume the COM has acceleration ​a  0 ​ and the bar mass of the bicycle along
has an angular acceleration about COM. with the rider is M and
(iv) Since spring 1 applies a force that has horizontal radius of each wheel is
component as well, the COM will have a horizontal R. The moment of iner-
component of acceleration as well. tia of each wheel about
axle is I. The accelerat-
In equilibrium: ing torque applied to the rear wheel by the pedal and gear
2T sin 30° = Mg system is t. Find the acceleration of the bicycle.
fi T = Mg
6.44  Mechanics II

Solution Solution
Concepts Concepts
As the pedal–gear system applies torque on the rear wheel, (i) The stick experiences only vertical forces. Its COM
it acquires angular acceleration (a). Friction force (f2) on will have vertical acceleration only.
it must be in forward direction so as to impart a linear (ii) Acceleration of end A will be directed perpendicular
acceleration (a) to the wheel. The frame of the bicycle to the string. End A cannot have vertically downward
exerts a push on the axle of the front wheel. Thus, friction acceleration since string is inextensible.
(f1) on the front wheel must be in backward direction so
as to provide it angular acceleration (a) and prevent the Let a = Vertically downward acceleration of COM and
wheel from slipping. a = angular acceleration of the stick
Using F = MaCM
Mg – T = Ma …(i)
Using t = I a about COM

M (2L)2
T . L cos 30° = _______
​   ​ ◊ a
  
12
For rear wheel, we can write
2MLa
fi T = ______
​  __ ​    ...(ii)
t – f2 R = I ◊ a ...(i)   
3​÷3 ​
For front wheel, we can write

f1R = I . a ...(ii)

For translation of the complete bicycle, we have

f2 – f1 = Ma

Using (i) and (ii), we get

t I a ___ I a


​ __  ​ – ___
​   ​ – ​   ​   = Ma
R R R
t 2Ia Acceleration of end A relative to COM is a L in a
fi ​ __ ​   = Ma + ____
​     
​ 
R R direction perpendicular to the rod. Vertical component of
this acceleration is a L cos 30°.
t 2Ia
fi ​ __  ​ = Ma + ​ ___ ​   [ a = Ra] For A to have no acceleration in ground frame
R R2 __
÷   
​ 3 ​
t/R a L cos 30° = a  fi ​ ___ ​  a L = a ...(iii)
fi a = _______
​      ​ 2
2I
M + ___​  2  ​  4Ma
R Putting this in (ii) gives T = ____ ​  g   
 ​ ...(iv)
4Ma
Example 57   A thin uniform bar AB of mass M and length Put this in (i) Mg – ____
​   ​   = Ma
2L is held at an angle of 9
30° to the horizontal by 9g
fi  a = ___ ​   ​ 
means of two vertical 13
strings at each end. The 4M ___ 9g 4Mg
string at the right end \ T = ___    ​ = ​ ____ ​ 
​   ​ ◊ ​ 
   
9 13 13
breaks. Find tension in
the left string immedi- Example 58   A flat spinning disc
ately after the right string A uniform disc of radius R is spun to an angular velocity w0
breaks. about its central axis normal to its plane and is then placed
Rotational Dynamics  6.45

carefully on a horizontal surface, as shown. How long will Example 59   A sphere of radius R and mass M has linear
the disc be rotating if friction coefficient between the disc velocity v0 directed to the left (and no angular velocity) as
and the surface is m? Pressure exerted by the disc on the it is placed on a flat car moving to right with a constant
surface can be regarded as uniform. velocity v. After sliding
on the car for some time,
the sphere finally stops
slipping.Length of the
car is sufficiently large.
Coefficient of kinetic fric-
tion between the sphere
and the car is m. Find v0 in terms of v, if the slipping ceases
at the moment velocity of the sphere relative to ground is
zero. Also find the time when slipping ceases.
Solution
Solution
Concepts
Concepts
(i) Friction acting on the disc produces a retarding
torque. Friction is kinetic and is directed opposite (i) Initially, the contact point has a velocity towards
to the velocity of a point. left relative to the car. Friction acts towards right.
(ii) We will consider a ring-shaped element and write (ii) v0 decreases and the sphere acquires an angular
torque on it due to friction. Integration to cover all speed (w).
such ring elements gives total torque on the disc. (iii) At some later time, speed of (centre of) the sphere
is v1 and it has an angular speed w (anticlockwise).
M
Mass per unit area s = ____
​   2 ​  The contact point has a velocity wR – v1 towards
p R right.
Consider a ring of radius x and (iv) Slipping stops when velocity of contact point
width dx. becomes equal to the velocity of the car, i.e.,
wR – v1 = v
Area of ring, dA = 2p x dx
(v) Friction is kinetic during this entire period.
Mass of ring, dm = s dA
f
2M Acceleration of sphere is a = __
​    ​  (Æ)
= ___
​  2 ​ x dx M
R Angular acceleration (a) of the sphere is
Normal force on the ring fR 5f
t
element is a = __
​   ​ = ​ ______    ​ = _____
​    ​ ( )
I 2
__ 2MR
​   ​  MR2
2Mg 5
) g = ____
dN = ( ​  2 ​ 
 x dx
R Velocity at time t is v = v0 – at
Friction will have a torque on this ring element given f
by v = v0 – __
​    ​  t ...(i)
M
2Mg m 2 
d tf = (m dN) ◊ x = ______
​  2 ​   x dx Angular speed at time t is w = a t = _____
5ft
​     ​( )
R 2MR
R
2Mgm 2 2MgmR Velocity of contact point at time t is:
Total torque on disc tf = Ú d tf = ​ _____  ​  Ú​  ​  ​ x dx = _______
​   ​  
2
R 0
Moment of inertia of disc about rotation axis, I = ​ ____
3
MR2
 ​ 

vA = (wR – v) (Æ) =
5ft
___
2M (  ft
​    ​ – ​ v0 – __ )
​    ​   ​
M
2 7 ft
tf 4 mg = __ ​   ​  ​ __  ​ – v0 (Æ)
Angular retardation is a = __ ​   ​  = ​ __ ​  ​ ___ ​  2 M
I 3 R
Slipping ceases when vA = v
Using w = w0 + a t gives
7 ft
fi ​ __ ​  __ ​    ​ – v0 = v
4mg
____ 3w0R
_____ 2 M
0 = w0 – ​   ​  . t  fi  t = ​   ​ 
3R 4mg
6.46  Mechanics II

But, f = mN = mMg Example 61   Rolling ball in a vertical circle


7 2 (v + v0) A small solid ball of mass
\ ​ __ ​  (mgt) – v0 = v  fi  = ​ ________
    
​ ...(ii) m starts to roll down a
2 7mg
loop track from height H,
According to the question, slipping ceases when v = 0 as shown in figure. The cir-
f v0 cular part of the track has
M
fi v0 – __
​     ​ t = 0  fi  t = v0 ____
​     
​= ___
​    ​.  radius R. The ball does not
M mMg mg
slip anywhere.
v0 2 (v + v0) (a) Find kinetic energy of the ball when it reaches point
\ ​ ___  ​  = ​ ________​   fi  7v0 = 2v + 2v0
    P where radius makes an angle q with horizontal.
mg 7mg
(b) Find the radial and tangential acceleration of the
2v
fi v0 = ___
​   ​  COM of the ball when it is at P.
5
(c) Find the normal force and friction force applied by
2v
( 
2 ​ v + ___
​   ​   ​
5
From (ii) t = ​ _________
    ​ 
)2 v
= ____
​    ​ 
the track on the ball when it is at P if
H = 1m, R = 0.1 m, q = 0° and m = 0.1 kg.
7mg 5 mg [g = 9.8 ms–2]
Solution
Example 60   Unwinding tape roll on an incline
A length L of a flexible tape is Concepts
tightly wound. The tape is allowed (i) Mechanical energy of the ball is conserved. Friction
to unwind as it rolls down an incline is static (as the ball does not slip) and it does not
making an angle q with horizontal, dissipate energy.
the free end of the tape is held (ii) The COM of the ball is moving on a circle of
fixed on the incline. In what time radius R – r  R [ Radius r of the ball is very
the tape unwinds completely? small] when the ball is on circular track.
Solution v2
(iii) Radial acceleration of COM is ​ __ ​  and its tangen-
Concepts R
tial acceleration is dv/dt where v is speed of the
(i) The unrolling tape is a system of varying mass. COM.
But the outgoing mass does not apply any thrust (iv) Normal force provides the radial acceleration to the
force since its velocity (relative to contact point) COM and tangential component of mg and friction
is zero. force decide the tangential acceleration of COM.
(ii) The rolling cylinder has decreasing mass and radius
but its acceleration remains same. Acceleration (a) Kinetic energy at P = Loss in potential energy between
does not depend on size or mass. It depends on A and P.
shape only. kP = mg [H – R (1 + sin q)]

g sin q
= ______
​    
2
 ​
K
___
1 + ​  2  ​ 
R
2
K 1
for cylinder ​ ___2  ​   = ​ __ ​ 
R 2

2g sin q
\ a = ​ _______
 ​ 

3
The tape unwinds completely when distance travelled by (b) At P, let the velocity of centre of the sphere be v.
rolling cylinder is L. The ball is rolling and it has an angular speed also. Its kE
7

÷ 
___ ______ is kP = ___
​    ​  mv2  [Refer example 27]

1
L = __
2 ÷ 
2L
​   ​  a t2  fi  t = ​ ___
3L
​  a ​ ​  = ​ ______
​    ​ ​ 
g sin q

10
7
\ ​ ___  ​ mv2 = mg [H – R (1 + sin q)]
10
Rotational Dynamics  6.47

10g (c) At q = 0°
fi  v2 = ____
​   ​ 
 [H – R (1 + sin q)] ...(i)
7
COM is going in a circle of radius R – r  R
10g __
ar = ____
​   ​ 
7 R
H
[ 
 ​ ​   ​  – 1  ​ ]
v2
\  Radial acceleration is ar = __
​   ​  Normal force is
R
10g __
fi  ar = ____
​   ​ 
7 R
H
[ 
 ​ ​   ​  – (1 + sin q)  ​ ]
mv2 ___
N = ​ ____   
R
10
7
H
​ = ​   ​ mg ​ __ [ 
​   ​  – 1  ​
R ]
Forces acting in tangential direction are mg cos q and
friction (f). If at is acceleration of COM along tangential
= ___
10
​   ​ × 0.1 × 9.8 ​ ___
7
1
​    ​ – 1  ​ = 12.6N
0.1 [  ]
direction: 5g
at = – ​ ___ ​ [Put q = 0° in (iv)]
mat = f – mg cos q ...(ii) 7

For rotation about COM \ m at = f – mg

( 
5
2
​ __
​   ​  mr2  ​ a = – fr
5 ) fi – ​ __ ​  mg = f – mg
7
2 2
fi f = __
​   ​  mg = __
​   ​  × 0.1 × 9.8 = 0.28 N
[Notice that for pure rolling, angular acceleration must 7 7
be clockwise, if tangential acceleration is assumed up along
the track. In fact, at is down the track (slowing the ball) and Example 62   Falling rod on a smooth table
a is anti-clockwise (slowing the spin). But it hardly makes A rod of mass M and length L, initially upright on a smooth
any difference to our analysis if we take them in directions table, starts falling when disturbed slightly. Find the speed
indicated. Mathematics corrects the directions and gives of COM of the rod as a function of angle q it makes with
negative values of at and a, as you can see below] the vertical.
2 2 Solution
fi  ​ __ ​  m (r a) = – f  fi ​ __ ​  m at = – f ...(iii)
5 5 Concepts
[   at = r a]
(i) In absence of friction, there is no horizontal force
Solving (ii) and (iii) gives
on the rod. The COM has no horizontal accelera-
5 tion. it falls on a vertical line.
at = – ​ __  ​ g cos q
7 (ii) The rod rotates about its COM as the COM falls.
There is a fixed geometric relation between the
Alternative method to find at
fall of COM and angular displacement of the rod.

÷ 
____________________
10g This will helps us relate speed of COM (= v) and
v = ​ ____
​   ​ 
 [H – R (1
    + sin q)] ​ angular speed (w).
7
(iii) Mechanical energy of the falling rod is

÷ 
____
dv 10g ________________
– R cos q dq conserved.
at = ___
​   ​ = ​ ____
​   ​ ​        ​ ___
  ​  ______________ ​   ​ 
dt 7 2 ​÷H   – R (1 + sin q) ​
   dt (iv) KE of the rod = kR + kT.

÷ 
____
10g __ cos q
1 ______________ d
= – ​ ____   ​   ​  ​ _______________
​   ​ ​         ​ __ ​    ​  (Rq) Figure shows the rod (AB) in its initial vertical position
7 2 ​÷H   – R (1 +   
sin q) ​ dt and any later position (A¢ B¢) making an angle q with the
dq vertical. Let v be velocity of COM of the rod and w be its
Look at the figure: ​ ___ ​  is angular speed of COM about angular velocity in position A¢ B¢.
dt
dq
O. R ​ ___ ​ is linear speed (v) of the COM L L
dt From figure: x = __
​   ​  – ​ __ ​  cos q
2 2
____ _________________

÷  ÷ 
10g __ cos q 10g
\ at = – ​ ___ ​ ​  ​ 1 ​ ______________
​             ​ ​ ___
​  ____________ ​     
​ [H – R (1 +  
sin q)] ​ dx L
fi ​ ___ ​   = __
​   ​  sin q
dq
___
​   ​ 
7 2 ​÷H   – R (1 +   
sin q) ​ 7 2 dt
5 L
= – ​ __ ​  g cos q ...(iv) fi v = w ​ __ ​  sin q ...(i)
7 2
6.48  Mechanics II

Example 63   Rolling off an edge


A solid homogeneous cylinder of
radius R is placed horizontally at rest
with its length parallel to the edge off
the table. There is sufficient friction at
the edge so that a small displacement
causes the cylinder to roll off the edge
without slipping. Determine–
(a) The angle qo through which the cylinder rotates
before it leaves contact with the edge.
(b) The speed of COM of the cylinder just before leaving
Kinetic energy of the rod in position A¢ B¢ = Loss in PE contact.
between position AB and A¢ B¢ (c) the ratio of translational to rotational kinetic energy
of the cylinder when its COM is in horizontal line
1 1
​ __ ​  Mv2 + __
​   ​  ICM ◊ w2 = Mg x with the edge.
2 2
Solution
1
fi ​ __ ​  Mv2 + __
2 2 12 (  ) (  )
1 ML2 ______
​   ​ ​  ____
​   ​  
 ​ ​​ ​ 
2v 2
   ​  
L sin q
L
​ ​ = Mg __
​   ​  [1 – cos q]
2
Concepts
(i) No slipping implies that friction does not per-

÷ 
_________________
3gL (1 – cos q) sin2 q form any work. Mechanical energy of the cylinder
fi v = ​ ​  _________________
  
    
 ​ ​ remains conserved.
3 sin2 q + 1
(ii) As long as the cylinder is in contact with the edge,
Alternate way of writing KE its COM is rotating in a circle of radius R.
We can write KE of the rod by assuming it to be in pure (iii) Writing equation for centripetal force (for motion
rotation about its instantaneous axis of rotation. Velocity of of COM) and putting normal reaction = 0, will give
point C (COM) is vertically down. The instantaneous cen- us the position where the cylinder leaves contact.
tre of rotation will be on a line that is ^ to velocity of C. (iv) After leaving contact, the rotational KE will not
Similarly, point B is moving horizontally and instantaneous change as there is no torque about the COM.
centre will be on a line that is ^ to vB. [In fact, velocity
of any point is normal to the line joining the instantaneous
centre to the point]. O (see fig.) is the instantaneous centre of
rotation. Axis is a line normal to the figure passing through
O. MI of the rod about this axis is

Consider the cylinder when it has rotated by an angle q.


It is in pure rotation about the edge of the table. Friction at
A is preventing it from slipping. The COM of the cylinder

ML2
I = ​ ____ ​ 
12 ( L
 + M ​​ __ ) 2 ML2
​   ​  sin q  ​​ ​ = ____
2
 (1 + 3 sin2q)
​   ​ 
12
has descended by (R – R cos q).
Loss in PE has been converted into KE. KE of the cylinder
\  KE of the rod is 1 1
can be written as k = __ ​    ​ MvCM 2
+ __
​   ​  ICM w2 or,
1 ML2 2 2
k = ​ __ ​  Iw2 = ____  (1 + 3 sin2q) w2
​   ​  1
2 24 k = __
​   ​  IA ◊ w2
When you equate this KE to loss in PE 2
Using the later expression,
L
[ 
= Mg ​ __ ​  (1 – cos q], you get w.
2
L
v can be calculated as v = w ​ __ ​  sin q
1 MR2
k = __
​   ​  ​ ____
2
​   ​ 
2 ] 3
 + MR2  ​ w2 = __
​   ​  M (Rw)2
4
2 since, speed of COM is v = Rw
Rotational Dynamics  6.49

3
\ k = ​ __ ​  Mv2 Example 64   You may miss to
4 see this rotation!
Using conservation of energy
A sphere of mass m and radius r
3 is suspended from point O using a
​ __ ​  Mv2 = Mg R (1 – cos q)
4 light thread. Distance of its centre
v2 4 from point O is l. The sphere is
fi ​ __ ​   = __​    ​ g (1 – cos q) ...(i) pushed so as to impart its centre a
R 3
horizontal velocity v. What is the
(a) Forces on the cylinder are kinetic energy of the sphere? Can
as shown. When it is on you write this kE using equation
verge of leaving contact, (11), i.e.,
N = 0.
1 1
COM is moving in a circle k = __
​   ​  ICM ◊ w2 + __ 2
​    ​ mvCM
2 2
of radius R about A.
Solution
Mv2
\ Mg cos q = ____
​     
​  Concepts
R 1
(i) kE is not equal to ​ __ ​  mv2.
4mg 2
Using (i) Mg cos q = ​ ____
 ​ 
 (1 – cos q)
3 All particles in the sphere

(  )
4 4 do not have the same
fi cos q = __
​   ​   fi  q = cos–1 ​ __ ​    ​  ​ velocity. For example,
7 7
4 point A will have veloc-
(b) From (i) Value of v when cos q = ​   ​  is __
7 ity w (l – r) as it moves in

÷  ( 
___________
a circle of radius (l – r).

4
v = ​ __
4
__
​    ​ gR ​ 1 –  
3
​   ​   ​ ​
7 ) Point B will have velocity
w (l + r) as it moves on a

÷​  7 ​  gR ​
____
4 circle of radius (l + r).
= ​ __  
v
(ii) w = __ ​   ​  = angular speed about O.
(c) At the instant the cylinder leaves contact l
(iii) Body is in pure rotation about an axis through O

÷ 
___

÷ 
____
4 v 4g that is perpendicular to the plane of the figure.
v = ​ __
​   ​  gR ​   and  w = __
​    ​ = ​ ___
​     ​ ​ 
7 R 7R
1
\ k = __
​   ​  I0w2
Rotational kE of the cylinder at this instant is 2

4g MgR Where, I0 = MI about the axis through O.


1 1 1
kR = ​ __ ​  ICM ◊ w2 = __
​    ​ ◊ ​ __ ​  MR2 ◊ ​ ___  ​ = _____
​   ​   (iv) An observer located at COM (C) sees that the ball
2 2 2 7R 7
is spinning about C. Look at the given figure. As
Once the cylinder loses the ball completes one revolution, an observer at
contact, Mg is the only C finds that a point (like B) spins about it. Point
force acting on it. There B is below C, then it is to right of C, then it is
is no torque about the above C, then it
centre and kR does not is to the left of
change. When COM is C and then again
on a horizontal line through A: it is below C.
clearly, a point
kR + kT = MgR [= loss in PE between
like B is spin-
   initial and final position]
ning around C.
MgR __ 6 Angular speed of
fi kT = MgR – _____
​   ​  = ​   ​  MgR this spin is same
7 7
as the angular
kT speed of the
\ ​ __ ​ = 6
kR body about O.
6.50  Mechanics II

Method I Let the insect be at P at any


Body is rotating about O. time t and its velocity relative to
the frame be v.
\
1
2
1 2 2
k = ​ __ ​  I0w2 = __
2 5 [ 
​    ​ ​ __ ]
​   ​  mr + ml2  ​ w2
If AP = x, then

1 2 2
= ​ __ ​  ​ __
2 5 [  v2
​   ​  mr + ml2  ​ ​ __2 ​ 
l
]
dx
v = ___
​   ​ 
dt

[ 
At this moment, angular speed
1
= __
2 r2
​   ​  mv2 ​ __
2
​   ​  ​ __2 ​  + 1  ​
5 l ] of the frame is w about O. If q is
the angle by which the frame has rotated by now, then
Method II dq
w = ___
​   ​ 
1 1 dt
k = __
​   ​  Icm ◊ w2 + ​ __ ​  mvCM
2
2 2 Angular momentum of frame Lframe = Iw ( )
1 2
= __
​   ​ ​  __
2 5 (  1
​   ​  mr2  ​ w2 + __ )
​   ​  mv2 ​
2 [  v
w = __ ]
​   ​   ​
l
4
= 2R3r ​ 2p + __ ( 
​   ​   ​ w ( )
3 )
[  ]
1 2 r2 Actual velocity of insect (in ground frame) = v (along AB)
= __
​   ​  mv2 ​ __
​   ​  ​ __2 ​  + 1  ​ + wb (perpendicular to AB)
2 5 l
\ Linsect =  (Due to v along AB) + L (Due to wb
Example 65   Crawling insect    perpendicular to AB)
A frame made of a uniform wire = movR ( ) + mo(wb) b ( )
of linear mass density r is in the
shape of a circle. The circle has a = movR – mowb2 ( )
diameter AB made of same wire.
The frame lies on a smooth table But Linsect + Lframe = 0

( 
and can rotate freely about a ver-
tical axis through point O on the
circumference. An insect of mass
4
fi movR – mowb2 = 2R3r ​ 2p + __
​   ​   ​ w
3 )
Rr
mo = ___
​   ​ [where R is radius of the
3
circle] is initially at rest at A and

1
[  dx
] ( 
4
)
​ __ ​  rR ​ R ​ ___ ​ – w {R2 + (R – x2)}  ​ = 2R3r ​ 2p + ​ __  ​  ​w
3 dt 3

begins to crawl along the diameter AB. By what angle does 1 dx


the frame rotate by the time the insect reaches B? fi ​ __ ​  rR2 ​ ___ ​ 
3 dt

[  (  ]
Solution rR 2
Concepts
4
= ​ 2R3r ​ 2p + __
3 )
​   ​   ​ +____
​   ​ 
3
dq
 [R + (R – x2)]  ​ ___
​   ​ 
dt
(i) Angular momentum of the frame + insect system dx dq
is conserved about the vertical axis through O. fi R ​ ___ ​ = [n2R2 + (R – x)2] ​ ___ ​   [where n2 = 12p + 9]
dt dt
(ii) As the insect moves from A to B, it generates
an angular momentum about O in anti-clockwise fi Rdx = [n2R2 + (R – x)2] dq
direction. The frame must rotate clockwise so as to
q0 2R
have equal and opposite angular momentum. This R
fi ​Ú ​  ​ ​ dq = ​Ú ​   ​ ​ _____________
​  2 2     2 ​ dx
will keep the total angular momentum = 0 0 0 n R + (R – x)

Mass of circle, M = 2p Rr


Mass of diameter AB, m = 2Rr
1
fi q0 = __
2n
​ n ​ tan–1 ​ _____
( 
​  2    
n –1
 ​  ​
)
MI of the frame about vertical axis through O is
Example 66   A rod of mass M and length L is positioned
m (2R)2 4 as shown in figure. Surfaces at A are smooth. The rod is
I = 2MR2 + _______
​   + mR2 = 4p R3r + 2R3r ​ __ ​ 
 ​ 
12 3 released from the shown position. The rod hits the peg P

( 
and sticks to it. Find the angular speed of the rod after it
4
= 2R3r ​ 2p + __
​   ​   ​
3 ) hits the peg at P.
Rotational Dynamics  6.51

After collision,
the rod sticks at P
and begins to rotate
about P with angular speed w.
Angular momentum about P after collision is

L after collision = IP ◊ w

ML2
= ​ ____ [  7
 + M (0.1 L)2  ​ w = ___
​   ​ 
12
​    ​  ML2w
75 ]
L after collision = L before collision
AP = 0.6 L where L is length of the rod. [Note that the
rod is not hinged at A] 7 1
​ ___  ​ ML2w = ___
​    ​ ML2w0
75 30
Solution

÷ 
______
__
5 5 3​    g
÷3 ​
Concepts fi w = ___
​    ​  w0 = ___ _____
​    ​ ​  ​   
​ ​
  

14 14 2L
(i) The rod is in pure rotation about A till it hits P.
Using energy conservation, we can find angular
Example 67   A uniform rod of mass m and length l rests
speed of the rod just before the collision.
on a smooth horizontal surface. One of the ends of the rod
(ii) Angular momentum about P remains unchanged is given a sharp blow in horizontal direction at right angles
during collision. The impact force does not produce to the rod. As a result, the centre of the rod beings to move
any impulse about P. with velocity v0. Find the force with which one half of the
rod will act on the other half during the course of subsequent
Let w0 = angular speed of the rod just before hitting P. motion.
conservation of energy gives
Solution
Loss in PE = gain in kE
Concepts
L 1 ML2 2
Mg ​ __ ​  sin 60° = ​ __ ​ ​  ____
 ​ ◊ w

  0 (i) The centre moves with velocity v0 and the entire
2 2 3 rod spins around it with some angular velocity w.
[Rod is in pure rotation about A] (ii) w can be calculated using angular – impulse

÷  2
______
__
g
    __
3​÷3 ​
____ momentum theorem.
fi w0 = ​ ​   ​  ​    
​ ​ ...(i)
L (iii) After the hit, the COM is moving with constant
velocity and an RF attached to it is inertial. In this
frame, each particle of the rod is seen rotating at
angular speed w. The centripetal force necessary
for rotation of one half of the rod is provided by
the other half.

Angular momentum of the rod about P, just before impact If J = impulse given, then
is Lbefore collsion = Lspin + Lorbitals
J = mv0 ...(i)
= ICM ◊ w0 ( ) + Mv0 (0.1 L) ( ) Angular impulse about COM is
l
Note that motion of COM produces an anti-clockwise J ​ __  ​   = ICM ◊ w
angular momentum about P. 2
L
v0 = w0 ​ __ ​  l ml2
Also, fi mv0 __
​    ​   = ___
​   ​  w
2 2 12
ML2 w0L 6v0
\ L before collision = ____  w – M ​ ____
​   ​    ​ 
 [0.1 L] fi w = ___
​      ​
12 0 2 l
1
= ML2 w0 ​ ___
​    ​ – 0.05  ​ =
12 (  ) 1
___
​    ​ ML2w0 ( )
30
In RF attached to C, segment AC is seen rotating with
angular speed w. COM of segment AC is at P.
6.52  Mechanics II

l
CP = ​ __  ​  \ I w0 – r Jf = I w
4
2 2
\ Centripetal force needed by AC is fi __
​   ​  mr2 w0 – r (0.4mv0) = __
​   ​  mr2 ◊ w
5 5

(  )
2 fi rw0 – v0 = wr  fi  2v0 – v0 = wr
m l 1 6v0 2 9 mv0
F = __
​   ​  w2 ​ __  ​  = __
​    ​ ml ​​ ___
​     ​   ​​ ​ = __
​   ​  ____
​     
​  v0
2 4 8 l 2 l fi w = ​ __
r ​ 
This force is applied by BC on AC. \ KE = kT + kR
Example 68   Spinning ball hits a rough floor 1 1
= ​ __ ​  m (v02 + vx2) + __
​   ​  ICM ◊ w2
A ball of radius r and mass 2 2
m is spinning about its cen-
tre at an angular speed w0. It
1
= ​ __ ​  m (v02 + 0.16 v02) + __
2
1 2
(  )
v0 2
​   ​  × ​ __ ​  mr2 ​​ __
2 5
​ r ​   ​​ ​
falls vertically and hits a solid
= 0.78 mv02
floor while its centre was trav-
elling at speed v0. Coefficient
of friction and coefficient of Example 69   A homogeneous rod AB of length L = 1.8 m
restitution are m = 0.2 and and mass M is pivoted at centre O is such a way that it
e = 1 respectively. Find the can rotate freely in
kinetic energy of the ball immediately after impact. Take vertical plane. The
rw0 = 2v0 rod is initially in
horizontally posi-
Solution
tion. An insect (S)
Concepts of same mass M
(i) e = 1, implies that the centre of the ball will have falls vertically with
a vertical velocity v0 (≠) after the impact. speed v at point C
on the rod. Point C
(ii) Normal force by the floor is impulsive. Since the L
ball is spinning, the contact point will slide on is at a distance ​ __ ​  from centre (O). Immediately after falling,
4
the floor and there will be a friction on the ball the insect moves towards end B so that the rod rotates with
towards right. This friction is impulsive since N is constant angular velocity w.
impulsive. (a) Find w in terms of v and L
(iii) Impulse of friction imparts horizontal velocity to (b) If the insect reaches end B when the rod has turned
the ball. by 90° determine v.
(iv) Friction also imparts an angular impulse. This
changes the angular speed of the ball. Solution
Concepts
If JN is impulse of normal force then (i) Weight of the insect produces a torque about O.
JN = 2mv0 ...(i) This will cause the angular momentum of the sys-
tem to increase.
(ii) Angular momentum is increasing but the angular
speed can be maintained constant by increasing
moment of inertia. As the insect moves towards B,
MI of the system about rotation axis increases.

Let us denote angular momentum by A ( L is used for


length of rod)
Impulse of friction Jf = mJN = 2mmv0 [ f = mN] Angular momentum about O before the insect hits the
rod = angular momentum just after the insect hits the rod.

[ 
\ mvx = 2mmv0
fi vx = 2mv0 = 0.4 v0 ...(ii)
\
L
Mv __
4
ML2
​   ​   = ​ ____
​   ​ 
12
L 2
 + M ​​ __ (  ) ]
​   ​   ​​ ​  ​ w
4
Angular impulse due to friction = r Jf ( ) 12v
fi w = ____
​   ​   ...(i)
7L
Rotational Dynamics  6.53

Let the insect be at a distance x from O when the rod the line AB. This distance
has turned by an angle q. is chosen so that the reac-
tion due to the hinges
on the laminar sheet is
zero during the impact.
Initially, the laminar sheet
hits one of the obstacles
with an angular velocity 1 rad s–1 and turns back. If the
impulse on the sheet due to each obstacle is 6 Ns.
(a) Find the location of the centre of mass of the laminar
sheet from AB?
Torque on system about O is (b) At what angular velocity does the laminar sheet come
t = Mg x cos q back after the first impact?
(c) After how many impacts does the laminar sheet come
Angular momentum of the system about O at this moment
to rest?
is

ML2
[ 
A = ​ ​ ____ ​ 
12
 + Mx2  ​ w ] Solution
Concepts
dA
Using t = ___
​   ​  (i) Hinges do not apply force. This means that impulse
dt
by an obstacle on the sheet is equal to change in
dx
Mg x cos q = 2Mx w ​ ___ ​   [w is a constant] linear momentum of the sheet.
dt
(ii) We will also apply angular impulse-momentum
fi dx = ​ ___
g
(  )
​     ​  ​ cos q ◊ dt
2w
theorem about AB.

dx dq ___ g Let r = distance of COM from line AB


fi ​ ___  ​ ___
​   ​  = ​     ​ cos q w = angular speed of the sheet after first
dq dt 2w
collision with the rubber obstacle
dx g
fi w ​ ___  ​ = ___
​     ​ cos q
dq 2w Velocity of COM before and after collision are
L
2 p/2 vi = r (1 rad s–1) = r  and  vf = rw
g
fi ​   2 ​ ​Ú ​   ​ ​cosq dq
Ú dx = ____ Linear impulse (J) = D P
L 2w 0
4 fi 6 = mvf – (– mvi) = m (vf + vi)
L g p/2
fi ​ __ ​   = ____
​   2 ​ [​  sin q ]​0 1
4 2w fi 6 = 30 (r + rw)  fi  r (1 + w) = __
​   ​   ...(i)
5

÷ 
___
2g Angular impulse about AB = DL
fi w = ​ ___
​   ​ ​  
L fi J◊ r = IAB w – (– IAB ◊ 1 rad s–1)

÷​  L ​ ​   fi 
___
7 ____
12v 2g fi 6 × 0.5 = IAB (w + 1)
From (i) ____
v = ___ ​   ​ = ​ ___
  ​ 
​    ​ ​÷2gL   
12 7L
fi 3 = (ICM + mr2) (w + 1)
7 __________
= ___ ​ 2  × 10 ×  
​    ​ ÷ 1.8 ​ fi 3 = (1.2 + 30r2) (w + 1) ...(ii)
12
= 3.5 ms–1 Solving (i) and (ii) for r gives
r = 0.4 m  and  r = 0.1 m
Example 70   Two heavy metallic plates are joined together For r = 0.4 m, w < 0 which is not possible
at 90° to each other. A laminar sheet of mass 30 kg is hinged \ r = 0.1 m,  [Ans to (a)]
at the line AB joining the two heavy metallic plates. The Putting r = 0.1 in (i) gives
hinges are frictionless. The moment of inertia of the laminar
w = 1 rad s–1  [Ans to (b)]
sheet about an axis parallel to AB and passing through its
centre of mass is 1.2 kg m2. Two rubber obstacles P and Q Since, the sheet returns with same angular velocity of
are fixed, one on each metallic plate at a distance 0.5 m from 1 rad s–1, it will make infinite number of collisions.
Worksheet 1
1. For the same total mass, which of the following will 6. Two identical rings, each of mass m,
have the largest moment of inertia about an axis pass- with their planes mutually perpendicular,
ing through its centre of mass and perpendicular to and radius R are welded at their point of
the plane of the body contact O. If the system is free to rotate
(a) a disc of radius a about an axis passing through the point
P perpendicular to the plane of the upper
(b) a ring of radius a
ring and parallel to the plane of lower
(c) a square lamina of side 2a ring, the moment of inertia of the system about this
(d) four rods forming a square of side 2a axis is equal to
2. A thin wire of length L and (a) 6.5 mR2 (b) 12 mR2
uniform linear mass density (c) 6 mR 2
(d) 11.5 mR2
r is bent into a circular loop
7. Two identical hollow spheres of mass M and radius R
with centre at O, as shown. The
are joined together, and the com-
moment of inertia of the loop
bination is rotated about an axis
about the axis XX¢
tangent to one sphere and perpen-
(a) rL3/8p 2 (b) rL3/16p2 dicular to the line connecting their
(c) 5rL3/16p2 (d) 3rL3/8p2 centres. The rotational inertia of
3. A thin uniform rod of mass M and length L has its the combination is
moment of inertia I1 about its perpendicular bisector. 4
(a) 10 MR2 (b) ​ __ ​  MR2
The rod is bent in the form of a semi-circular arc. 3
Now, its moment of inertia through the centre of the 32
___ 2 34
___
(c) ​   ​  MR (d) ​   ​  MR2
semi-circular arc and perpendicular to its plane is I2. 3 3
The ratio of I1 : I2 will be 8. AB and CD are two identi-
(a) < 1 (b) > 1 cal rods each of length L
and mass M joined to form
(c) = 1 (d) can’t be said a cross. Find the MI of the
4. Moment of inertia of a thin semicircular disc system about a bisector of
(mass = M and radius = R) about the angle between the rods
an axis AB through point O, is (XY)
given by
ML2 ML2
1 (a) ​ ____ ​  
  (b) ​ ____
 ​ 

(a) ​ __ ​  MR2 12 6
4
ML2 4ML2
1
(b) ​ __ ​  MR2 (c) ​ ____
 ​  
  (d) ​ _____
 ​ 

2 3 3
1 9. The moment of inertia of a thin square sheet of mass
(c) ​ __ ​  MR2
8 M about the axis shown is

(d) MR2

5. The figure shows a uniform


rod lying along the x-axis.
Consider moment of inertia of
the rod about different axes
parallel to z-axis. All the axes
about which the moment of
inertia of the rod is same,
interect the xy plane at points 7 5
(a) ​ ___  ​  Ma2 (b) ​ ___  ​  Ma2
whose locus is 12 12
(a) an ellipse (b) a circle 1 1
(c) ​ __ ​  Ma2 (d) ​ ___  ​  Ma2
(c) a parabola (d) a straight line 3 12

6.54
10. Eight point objects, each of mass M, are at the ver- approximate value of coefficient of kinetic friction
tices of a rigid, massless frame in the between the brake shoe and the cylinder?
form of a cube. each side is of length
a. What is the moment of inertia of
this arrangement about an axis passing
through one edge of the cube?
(a) 4 Ma2 (b) 6 Ma2
(c) 8 Ma2 (d) 10 Ma2
11. A rigid body has its COM on x-axis and can rotate
about an axis that intersects the x-axis and is parallel (a) 0.027 (b) 0.042
to y-axis. When the axis is chosen to pass through a (c) 0.065 (d) 0.140
point on x-axis that is at a distance x from the origin
of co-ordinate system, the moment of inertia is given 15. When a force of 6.0 N is exerted at 30° to a wrench
by I = 2x2 – 12x + 27. The x-coordinate of centre of at a distance of 8 cm from the nut, it is just able to
mass of the body is loosen the nut. What force F would be sufficient to
loosen it, if it acts perpendicularly to the wrench at
(a) x = 2 (b) x = 0
16 cm from the nut?
(c) x = 1 (d) x = 3
12. A flat triangular sheet of uniform material is shown in
the drawing. Angle B is smaller than angle A. There
are three possible axes of rotation, each perpendicular
to the sheet and passing through one corner - A, B,
or C. For which axis is the greatest external torque
needed to impart the triangle an angular speed of (a) 3 N (b) 1.5 N
1.0 rad s–1 in 1.0 s, starting from rest? (c) 6 N (d) 1 N
16. Two bodies have moment of inertia I and 2 I respec-
tively about their axis of rotation. If their kinetic
energies of rotation, about fixed axes, are equal, their
angular momenta will be in the ratio
(a) 2 : 1 (b) 1 : 2
__ __
(c) ​÷ 2 ​  : 1   
(d) 1 : ​÷2 ​
(a) A (b) B 17. A T-shaped object is made by
(c) C (d) All are equal welding two rods of same linear
mass density. It has dimensions
13. A solid sphere of radius R has moment of inertia I
shown in the figure. It is lying
​_›
about its diameter. It is melted into
a disc of radius r and thickness t. If on a smooth floor. A force ​F   ​ is
its moment of inertia about the tan- applied at the point P parallel
gential axis (which is perpendicular to AB, such that the object has
to plane of the disc) is also equal only translational motion with-
to I, then the value of r and t are out rotation. Find the distance
(respectively) of P from C
4

2
(a) ​ ___  ​  R, 5R
2__
(b) ​ ___   ​  R, 2R (a) ​ __ ​   (b) l
15   
​÷5 ​ 3

÷ 
____
2l 3l
3 R 3 R (c) ​ __ ​   (d) ​ __ ​ 
(c) ​ ____
___ R, __
   ​   ​   ​   (d) ​ ___   ​, __
​    ​R  ​   ​  3 2
   
​÷15 ​ 2 15 5
18. A cylinder of radius
14. The figure shows a 15 kg solid cylinder mounted on a
R is spinned about its
fixed axle, with a radius 25 cm, rotating at an angular
central axis with angu-
speed of 500 rotations per minute. If a 100 N braking
lar speed w and then
force is applied normal to the curved surface of the
placed on an incline
cylinder, bringing it to rest in 15 seconds, what is the
having inclination

6.55
angle q. The cylinder continues to spin without fall- 25. A uniform cylinder of mass M and radius R is placed
ing. The cylinder can stay at the same location for a on an incline of inclination q. It can rotate freely
time about its central axle,
Rw Rw which is connected to
(a) ​ _______
   ​ (b) ​ _______
   ​ an ideal spring of force
3g sin q 2g sin q
constant k. The cylin-
Rw 2Rw
(c) ​ ______  ​   (d) ​ ______  
 ​ der is released from rest
g sin q g sin q
with the spring relaxed.
19. A body rolls down an inclined plane without slipping. Friction is large enough
If its kinetic energy of rotation is 40% of its kinetic to prevent slipping. As
energy of translation, then the body is the cylinder rolls down
(a) solid cylinder (b) solid sphere the inclined surface, the
(c) disc (d) ring maximum elongation in
the spring is
20. A body rolling without sliding has a fraction ‘f ’ of
its entire KE as rotational KE. Maximum possible 3 Mg sin q 2Mg sin q
(a) ​ __ ​  ​ ________
    ​   (b) ​ _________
   


value of ‘f ’ is 4 k k
(a) 1/2 (b) 1/3 Mg sin q
(c) ​ ________      
​ (d) None of these
(c) 2/3 (d) 2/7 k
21. A particle having mass 2 kg is moving along straight 26. A cubical block of side a is moving with veloc-
line 3x + 4y = 5 with speed 8 ms–1. Find angular ity v on a horizontal smooth
momentum of the particle about origin. x and y are plane as shown. It hits a ridge
in metres. at point O. The angular speed
of the block just after it hits
(a) 16 kgm2s–1 (b) 8 kgm2s–1 O is
2 –1
(c) 36 kgm s (d) none 3v 3v
(a) ​ ___  ​ (b) ​ ___  ​
22. A uniform thin pole of length L and mass M is 4a 2a
__
pivoted on the ground with a frictionless hinge. The ÷    v
​ 3 ​
pole makes an angle q with the (c) ​ ____
__    ​ (d) Zero
÷    a
​ 2 ​
horizontal. If it starts falling from 27. In the situation shown, a heavy wheel with a small
the position shown in the accom- drum attached at its centre is sus-
panying figure, the linear accel- pended by its frictionless axle from
eration of the free end of the pole a ceiling. Attached to strings around
immediately after release would the rims of the wheel and drum are
be two blocks of equal mass. The system
(a) (2/3) g cos q (b) (2/3) g is originally at rest. When the blocks
(c) g (d) (3/2) g cos q are released
23. If the earth shrinks to half the present radius such (a) Nothing moves, since the blocks have equal
that mass remains constant, without any change in mass.
mass, then the duration of day and night becomes: (b) The right-hand block falls and the left-hand
(a) 24 hours (b) 12 hours one rises, with accelerations of the same
(c) 6 hours (d) 3 hours magnitude.
24. At any time t, ​_a particle of mass 0.01 kg has the (c) While the blocks are moving, the tension in the

position vector ​r ​   = 3 cos q  + 4 sin q  . Here, q is right-hand string is less than that in the left-hand
the angle that the position vector makes with positive string.
direction of x-axis. If its angular momentum with (d) Which block falls depends on the moment of
respect to origin is 0.6 kg m2s–1 and remain constant inertia of the wheel‑drum system.
then angular displacement of the position vector of 28. A rod is hinged at its centre and rotated by apply-
particle in time interval t is ing a constant torque starting from rest. The power
(a) zero (b) (5 t) developed by the external torque as a function of
–1
(c) sin (0.1/t) (d) t time is

6.56
(a) Up the incline while ascending and down the
incline while descending
(a) (b)
(b) Up the incline while ascending as well as
descending
(c) down the incline while ascending and up the
incline while descending
(c) (d) (d) down the incline while ascending as well as
descending
32. A unifrom cylinder of mass M and radius R rolls
29. A pulley is hinged at the without slipping down a slope of angle q with hori-
centre and a massless thread zontal. The cylinder is connected to a spring of force
is wrapped around it. The constant k at the centre, the other side of which is
thread is pulled with a con- connected to a fixed support at A. The cylinder is
stant force F starting from released when the spring is unstretched. The force
rest. As the time increases, of friction (f )
(a) its angular velocity increases, but force on hinge (a) is always upwards
remains constant
(b) is always downwards
(b) its angular velocity remains same, but force on (c) is initially upwards and then becomes
hinge increases downwards
(c) its angular velocity increases and force on hinge (d) is initially upwards and then becomes zero.
increases
33. An L-shaped thin uniform rod of
(d) its angular velocity remains same and force on total length 2l is free to rotate in a
hinge is constant vertical plane about a horizontal axis
30. A uniform disc of mass M = 2.50 kg and radius at P, as shown in the figure. The
R = 0.20 m is mounted on an axle supported on fixed rod is released from rest from the position shown.
frictionless bearings. A light cord wrapped around Neglect friction. The angular velocity at the instant
the rim is pulled with a force 5 N. On the same it has rotated through 90° and reached the dotted
system of pulley and string, instead of pulling it position shown is

÷ 
___
down, a body of weight 5 N is suspended. If the first 6g
process is termed A and the second B, the tangential (a) zero (b) ​ ___
​   ​ ​  

÷ 
___ 5l
acceleration of point P will be 3g
___
(c) ​ ​   ​ ​    (d) none
5l
34. A uniform cylinder of mass m and radius R is placed
on a rough horizontal surface having coefficient of
friction m = ½. A horizontal force F = 4 mg is applied
on sphere at point P (as shown in figure). The friction
between cylinder and the floor is

(a) equal in the processes A and B.


(b) greater in process A than in B. mg
(a) f = 0 (b) f = ___
​   ​ 
(c) greater in process B than in A. 4
mg
___
(d) independent of the two processes. (c) f = ​   ​   (d) none of these
2
31. A cylinder rolls up an inclined plane, reaches some 35. A spinning sphere is placed with its centre initially
height, and then rolls down (without slipping through- at rest in a corner as
out these motions). The direction of the frictional shown in figure (a)
force acting on the cylinder is & (b). Coefficient
of friction between
all surfaces and the

6.57
1 fa are each free to rotate about fixed axes through their
sphere is ​ __ ​ . Find the ratio of the frictional force __
​   ​
3 fb centres. Assume the hoop is connected to the rotation
by ground in situations (a) & (b). axis by light spokes. With the objects at rest, identi-
9 cal forces are simultaneously applied to the rims, as
(a) 1 (b) ​ ___  ​  shown. Rank the objects according to their angular
10
10
___ momentum after a given time t, least to greatest.
(c) ​   ​   (d) none
9
36. A disc is rolling without
slipping with angular velocity
w. P and Q are two points
equidistant from the centre
C. The order of magnitude of
velocity is (a) all tie (b) disk, hoop, sphere
(a) vQ > vC > vp (c) hoop, disk, sphere (d) hoop, sphere, disk
(b) vP > vC > vQ 41. A uniform thin rod of mass ‘m’ and length L is
vC
(c) vP = vC, vQ = ___ ​   ​  hinged about its upper end, and is free to swing in a
2 vertical plane. A tiny ball of mass m/4 hits the lower
(d) vP < vC > vQ end of the rod. The rod was initially vertical and
the ball was moving horizontally with velocity v0. If
37. Two uniform spheres of densities 2d and d have
radii R and 2R respectively. Each sphere is rotating the ball stops just after collision, then coefficient of
about a fixed axis through a diameter. The rotational restitution for the collision is
kinetic energies of the spheres are identical. What is (a) 0.75 (b) 0.25
the ratio of the magnitude of the angular momenta (c) 0.5 (d) none
of these spheres? 42. A ball is attached to a string that is attached to a
__
(a) 4   
(b) 2​÷2 ​ pole. When the ball is hit, the string wraps around
__
(c) 1/2 (d) ​÷ 2 ​  the pole and the ball spirals
38. A ball is placed on a plank after being given a inwards, sliding on the friction-
velocity v and angular velocity w such that the ball less surface. Considering angu-
rolls without sliding on a plank. lar momentum about centre of
The upper surface of the plank is pole, what happens as the ball
rough enough to prevent slipping swings around the pole?
but lower surface in contact with (a) The mechanical energy and angular momentum
the ground is smooth. No other force is acting on are conserved.
system. (b) The angular momentum of ball is conserved but
(a) The plank will recoil back the mechanical energy of the ball increases.
(b) The plank will also move forward but with a (c) The angular momentum of the ball is con-
lesser velocity than that of the ball. served and the mechanical energy of the ball
(c) The plank will also move forward but with a decreases.
greater velocity than that of the ball. (d) The mechanical energy of the ball is conserved
(d) The plank will remain at rest. and angular momentum of ball decreases.
39. A child is standing with folded hands at the centre 43. A circular hoop of mass m and radius R rests flat on a
of a platform rotating about its central axis. Kinetic horizontal frictionless surface. A bullet, also of mass
energy of the system is K. The child now stretches m and moving with a velocity v, strikes the hoop
his arms so that moment of inertia of the system and gets embedded in it. The thickness of the hoop
doubles.The kinetic energy of the system now is is much smaller
than R. The
K
(a) 2 K (b) ​ __ ​  angular velocity
2
K with which the
__
(c) ​   ​   (d) 4 K
4 system rotates
after the bullet
40. A uniform disk, a thin hoop (ring), and a uniform
strikes the hoop
sphere, all with the same mass and same outer radius,
is

6.58
(a) v/(4R) (b) v/(3R)
(c) 2v/(3R) (d) 3v/(4R) (c) (d)
44. A smooth sphere A is moving on a frictionless hori-
zontal plane with angular speed w and centre of
mass velocity v. It collides elastically and head on 47. Two particles, each of mass M, are con-
with an identical sphere B at rest. Neglect friction nected by a massless rod of length l. The
everywhere. After the collision, their angular speeds rod is lying on the smooth sufrace. If
are wA and wB, respectively. Then one of the particles is given an impulse
Mv, as shown in the figure, then angular
(a) wA < wB (b) wA = wB
velocity of the rod would be:
(c) wA = w (d) wB = w
(a) v/l (b) 2v/l
45. An equilateral triangle ABC formed from a uniform
(c) v/2l (d) None
wire has two small identical beads initially located at
A. The triangle is set rotat- 48. A child with mass m is standing at the edge of a
ing about the vertical axis horizontal disc with moment of inertia I, radius R,
AO. Then the beads are and initial angular velocity w about its central vertical
released from rest simulta- axis. The child jumps off the edge of the disc with
neously and allowed to slide tangential velocity v with respect to the ground. The
down, one along AB and the new angular velocity of the disc is

÷ 
_______________

÷ 
_________
other along AC, as shown.
Iw – mv 2 2 (I + mR2)w2 – mv2
Neglecting frictional effects, (a) ​ ​ ________
   
​ ​  
  (b) ​ ​ _______________
    
     ​ ​
I I
the quantities that are con-
served as the beads slide Iw – mvR (I + mR2)w – mvR
down, are (c) ​ ________
    ​   (d) ​ _______________
     ​ 
I I
(a) Angular velocity and total energy (kinetic and 49. A uniform rod of length l and mass M is rotating
potential). about a fixed vertical axis passing through its centre,
(b) Total angular momentum and total energy. on a smooth horizontal table. It elastically strikes a
(c) Angular velocity and moment of inertia about particle placed at a distance l/3 from its axis and
the axis of rotation. stops. Mass of the particle is
3M
(d) Total angular momentum and moment of inertia (a) 3M (b) ​ ___ ​  
4
about the axis of rotation.
3M 3M
46. A circular platform is free to rotate in a horizontal (c) ​ ___ ​    (d) ​ ___ ​  

2 3
plane about a vertical axis passing through its centre.
A tortoise is sitting at the edge of the platform. Now, 50. A uniform rod AB of mass m and length l is at rest
the platform is given an angular velocity w0. When on a smooth horizontal surface. An impulse J is
the tortoise moves along a chord of the platform with applied to the end B, perpendicular to the rod in the
a constant velocity (with respect to the platform), the horizontal direction. Speed of point P at a distance
angular velocity of the platform w(t) will vary with __l
​    ​ from the centre towards end A of the rod at time
time t as 6
pml
t = ____​     
 ​after impact is
12J
J __
(a) (b) (a) 2​ __
m   ​  m
(b) J/​÷2 ​

J __ J
(c) ​ __
m  ​   (d) ​÷ 2 ​​ __
    ​ 
m

6.59
Worksheet 2
1. Suppose you are standing on the edge of a spin- (a) The sphere will definitely stop sliding at a point
ning platform and step off at right angle to the edge during its upward journey even if m is small.
(radially outward). Now consider it the other way. (b) Pure rolling will definitely begin before the
You are standing on the ground next to a spinning sphere reaches the highest point but the sphere
platform and you step onto the platform at right angle will continue to roll purely after that (even while
to the edge (radially inward). coming down) only if m is greater than a certain
(a) There is no change in rotational speed of the value.
platform in either situation. (c) Friction will decrease once pure rolling starts
(b) There is a change in rotational speed in the first if m > (2 tan q)/7
situation but not the second. (d) None of the above
(c) There is a change in rotational speed in the 5. A small ball of mass m sus-
second situation but not in the first. pended from the ceiling at a
(d) There is a change in rotational speed in both point O by a thread of length
instances.  moves along a horizontal
2. A hollow spherical ball is given an initial push circle with a constant angular
up an incline of inclination angle a. The ball rolls velocity w.
purely. Coefficient of static friction between ball and (a) angular momentum is con-
incline = µ. During its upward journey, stant about O
(a) friction force decreases in magnitude as the ball (b) angular momentum is constant about C
slows down (c) vertical component of angular momentum about
(b) friction force must increase in magnitude as the O is constant
ball rolls down (d) Magnitude of angular momentum about O is
(c) friction force becomes zero at the top point constant
where the ball stops 6. A round body kept on a smooth horizontal surface is
(d) m ≥ 2/5 tan a pulled by a constant horizontal force applied at the
3. A disc has radius R, mass m and has moment of top point of the body.
inertia I1, I2, I3 and I4 about axes shown in figure. (a) If the body rolls purely on the surface, its shape
Moment of inertia of disc about z-axis passing can be that of a thin pipe
through O will be: (b) If the body rolls purely on the surface, its shape
5
(a) I1 + I2 (b) ​ __ ​  mR2 can be that of a solid uniform cylinder
2
(c) If the body moves with backward sliding
(c) 3mR2 (d) I1 + I4 (i.e., its contact point having a backward veloc-
ity relative to the ground) its shape can be that
of a sphere.
(d) All bodies, irrespective of the shape, will slip
7. Inner and outer radii of a spool are r and R
respectively. A thread is wound over its inner surface
and placed over a rough horizontal surface. Thread is
pulled by a force F, as
4. In the figure shown, friction coefficient between the
shown in fig. Friction
solid sphere and the incline is m. The centre of the
is large enough to pre-
sphere is given an initial
vent slipping. Which
upward velocity at t = 0, as
of the followings are
shown, without imparting
possible?
any initial angular velocity.
Then, which of the following (a) Thread unwinds, spool rotates anti-clockwise
statement (s) is/are true? and friction acts leftwards.

6.60
(b) Thread winds, spool rotates clockwise and 11. A ball rolls down an
friction acts leftwards. inclined plane, as shown
(c) Thread winds, spool moves to the right and in figure. The ball is
friction acts rightwards. first released from rest
at P and then later from
(d) Thread winds, spool moves to the right and
Q. Which of the follow-
friction does not come into existence.
ing statement(s) is/are
8. A thin tabecloth covers a horizontal table and a uni- correct?
form round body lies on top of it. The tabecloth
(a) The ball takes twice as much time to roll from
is pulled from under the body, and friction causes
to O as it does to roll from P to O.
the body to slide and
rotate. (Assume that (b) The acceleration of the ball at Q is twice as
the table is large and large as the acceleration at P.
the body does not (c) The ball has twice as much KE at O when roll-
fall off it during its ing from Q as compared to when it starts from
course of motion.) P.
(a) Body will finally roll towards left if it is a (d) In case of the ball released from Q, angular
ring momentum of the ball about point Q changes
(b) Body will finally roll towards right if it is a more during its journey from Q to P as com-
cylinder pared to the change during its motion from P
to O.
(c) Body will finally come to rest, irrespective of
its shape 12. ABCD is a square plate with
centre O. The moments of
(d) shape and size, both are important to decide the
inertia of the plate about the
final state of the body.
perpendicular axis through O
9. A hoop and a solid cylinder have the same mass is I and about the axes 1, 2,
and radius. They both roll, without slipping, on 3 and 4 are I1, I2, I3 and I4,
a horizontal surface. If their kinetic energies are respectively. It follows that:
equal
(a) I2 = I3 (b) I = I1 + I4
(a) the hoop has a greater translational speed than
(c) I = I2 + I4 (d) I1 = I3
the cylinder
13. A paritcle falls freely near the surface of the earth.
(b) the cylinder has a greater translational speed
Consider a fixed point O (not vertically below the
than the hoop
particle) on the ground.
(c) the hoop and the cylinder have the same angular (a) Angular momentum of the particle about O is
momentum relative to their respective COM increasing.
(d) if they begin climbing an incline, they will attain (b) Torque of the gravitational force on the particle
same height but the hoop will be first to reach about O is decreasing.
the maximum height (assume no slipping)
(c) The moment of inertia of the particle about O
10. A dumbbell is lying on a is decreasing.
smooth horizontal table. A (d) The angular velocity of the particle about O is
force F is applied to the increasing.
dumbbell for a small time ​_›
interval, t, first as in (i) and 14. The torque ​t ​   on_​ ›a body
​ › about _
_ ​a

given point is found
then as in (ii). to be_​ › equal to ​A   ​ × ​L   ​ where ​A   ​ is a constant vector
and ​L   ​ is the angular momentum of the body about
(a) In case (i), the dumb- that point. From this, it follows _that
bell’s COM acquires higher speed ​_› ​›
(a) d​L   ​ /dt is perpendicular to ​L   ​ at all instants of
(b) In case (ii), the dumbbell acquires higher kinetic time
energy ​_› ​_›
(b) the components of ​L   ​ in the direction of ​A   ​ does
(c) In case (ii), the dumbbell acquires higher kinetic not change with time
energy in its COM frame ​_›
(c) the
_
magnitude of ​L   ​ does not change with time
(d) In case (ii), the dumbbell acquires higher ​›
momentum in COM frame (d) ​L   ​ does not change with time

6.61
15. A uniform rod is fixed to a (a) the acceleration of the centre of mass of the
rotating turntable so that its cylinder is 2a.
lower end is on the axis of (b) the acceleration of the centre of mass of the
the turntable and it makes an cylinder is zero.
angle of 20° to the vertical.
(c) the angular acceleration of the cylinder is a/R.
(The rod is thus rotating with
uniform angular velocity about a vertical axis passing (d) the angular acceleration of the cylinder is
through one end.) The turntable is rotating clockwise zero.
as seen from above. 19. A uniform disc of radius R is roll-
(a) The rod’s angular momentum vector about ing, without sliding, on a horizon-
its lower end is directed down at 20° to the tal surface. B is a point attached
horizontal. to the ground at the contact and
(b) The rod’s angular momentum vector about A is a fixed point just above the
its lower end is directed down at 80° to the top point of the disc.
horizontal. (a) The magnitude of the angular momentum of the
(c) there is a torque acting on the system and it is disc about B is thrice that about A.
in horizontal direction. (b) The angular momentum of the disc about A is
(d) there is a torque acting on the system and it is anti-clockwise.
in vertical direction. (c) The angular momentum of the disc about B is
16. A yo-yo is resting on a rough clockwise
horizontal table. Friction is (d) The angular momentum of the disc about A is
large enough to prevent slip- equal to that about B.
ping. Forces F1, F2 and F3 are 20. A solid sphere of radius R has a light thread wrapped
applied separately on the thread around it. It is pulled hori-
as shown. Line of action of F2 zontally by a force F as
passes through the contact point. The correct state- shown in the figure. There
ment is is no slipping and initially
(a) when F3 is applied, the centre of mass will move the sphere was at rest.
to the right.
(a) work done by force F when the centre of mass
(b) when F2 is applied, the centre of mass will move moves a distance S is 2 FS.
to the left.
(b) speed of the COM when COM moves a distance
(c) when F1 is applied, the centre of mass will move

÷ 
______
to the right. 20 FS
(d) when F2 is applied, the centre of mass will move S is ​ ___
​   ​ ___
​   ​ ​ 

7 M
to the right.
17. A plank with a uniform sphere placed on it rests (c) work done by the force F when COM moves a
on a smooth horizontal plane. Plank is pulled to distance S is FS
right by a constant force F. If (d) speed of the COM when COM moves a distance
sphere does not slip over the

÷ 
____
4FS
plank, which of the following S is ​ ____
​     
​ ​  
is correct? M
(a) Acceleration of the centre of sphere is less than 21. A small sphere A of mass m and radius r rolls without
that of the plank. slipping inside a large fixed hemispherical bowl of
(b) Work done by friction acting on the sphere is radius R (>> r), as shown in figure. The sphere starts
equal to its total kinetic energy. from rest at the top point of the hemisphere.
(c) Total kinetic energy of the system is equal to
work done by the force F
(d) None of the above
18. In the figure shown, the
plank is being pulled to the
right with a constant accel-
eration a. If the cylinder
does not slip, then

6.62
(a) the angular momentum of the sphere about O some angular velocity and the other is stationary.
is maximum when it reaches the bottom of the Their rims are now brought in contact. There is fric-
bowl. tion between the rims.
(b) the normal force exerted by the small sphere on (a) The force of friction between the rims will dis-
the hemisphere when it is at the bottom B of the appear when the discs rotate with equal angular
17 speeds.
hemisphere is ___​   ​  mg
7 (b) The force of friction between the rims will dis-
(c) friction force acting on the sphere is maximum
just after it is released appear when they have equal linear velocities
at the point of contact.
(d) the sphere will come out of the bowl if the part
of the hemisphere to the right of B is smooth (c) The angular momentum of the system about the
contact point will be conserved.
22. Two horizontal discs of different
radii are free to rotate about their (d) The rotational kinetic energy of the system will
central vertical axes. One is given not be conserved.

6.63
Worksheet 3
__
÷   
​ 3 ​
1. Particles of masses 1g, 2g, 3g ... 100g are kept at friction m = ​ ___ ​ between them. Calculate (a) tension
10
marks 1 cm, 2 cm, ... 100 cm respectively on a in each theread and (b) acceleration of each block
metre scale. Find the moment of inertia of the sys-
tem of particles about a perpendicular bisector of the
scale.
2. The density of a sphere of radius R varies with dis-
( 
tance x from centre as r = r0 ​ 1 + __
x
)
​   ​   ​, where r0 is
R
a positive constant. calculate the moment of inertia
of the sphere about
its diameter.
7. A uniform stick of mass M and length L is held
3. A uniform rod has
horizontal with the help of two vertical strings as
mass M and length
shown. The right string is cut. Immediately after the
L. Consider an axis
right string is cut, find:
through its end inclined at an angle q to the rod. Find
moment of inertia of the rod about this axis. (a) acceleration of COM of the rod
4. A square of side length (b) tension in the other string.
a is cut from a square
plate of side length 2a.
The centre of the hole
is at the centre of the
square plate, as shown
in figure. Mass of the
square with hole is M. Find the moment of inertia
of the plate about axis AB. 8. The strings in the previous question are replaced by
5. A block x of mass m = 0.5 kg is held by a long two identical
massless string on a smooth incline plane of ideal springs.
inclination angle q = 30°. The string is wound on Find acceler-
a uniform solid cylin- ation of end
drical drum y of mass A of the rod
M = 2 kg and radius immediately
r = 0.2 m. The drum after the right
is given an initial spring is cut.
angular velocity, such 9. A disc of mass M = 4 kg and radius R = 75 mm can
that the block X starts rotate feely about its central axis. The disc, originally
moving up the plane. at rest, is placed in contact with a belt moving at
(a) Find tension in the string during motion. a constant velocity v = 18 ms–1. The coefficient of
friction between the belt and the disc is m = 0.25
(b) At a certain instant of time, magnitude of and normal force
angular velocity of Y is 10 rad s–1. Calculate between the disc
the distance travelled by x from that instant of and the belt is
time until it comes to rest. equal to half the
6. In the system shown, block A and B have masses weight of the
26
m1 = 2 kg and m2 = ___ ​   ​  kg respectively. Pulley has disc. Find the
7
moment of inertia I = 0.11 kg m2 about its frictionless number of revo-
axle. Pulley can only rotate about its axis and cannot lutions completed
translate. The pulley has two dises of radii a = 15 cm by the disc before
and b = 10 cm, on which two threads are wound it stops slipping
tightly. The inclined plane and A have coefficient of on the belt.

6.64
10. A ring of radius R has a light thread wrapped on it. (a) Find angle q where the ball
The free end of the thread is pulled so as to apply a breaks off the sphere.
constant horizontal force on the ring at the top. The (b) Find angular velocity of the
ring is standing on a smooth horizontal surface. Find ball (about its centre) at
distance travelled by the centre of the ring during the the instant it breaks off the
time it makes one full rotation. sphere.
11. A sphere of radius R is 16. A uniform heavy solid hemisphere
moving with velocity v0 and of radius R is held at rest with its
rotating with velocity w0 in base vertical and its curved surface
anti-clockwise direction as in contact with a horizontal plane.
shown. The surface is rough. The hemisphere is released from
v0
Find the ratio ​ ____  ​ if this position. The plane is rough
w0 R enough to prevent slipping. Write
(a) the sphere stops translating and rotating the kinetic energy of the hemi-
simultaneously. sphere when the base makes an
(b) velocity of the sphere becomes zero while it is angle q with horizontal.
still rotating anti-clockwise. 17. In the last problem, find the minimum coefficient
12. A plank of mass M, with a sphere of mass m placed of friction such that the hemisphere does not slip
on it, rests on a smooth immediately after it is released.
horizontal surface. A 18. A uniform sphere of radius R has a spherical cavity
R
constant horizontal of radius ​ __ ​  scooped out of it as shown in figure.
force F is applied to 2
Mass of the sphere with cavity is M. The sphere is
the plank. Calculate the
rolling without sliding
minimum coefficient of friction between the sphere
on a rough horizontal
and the plank so that there is no slipping between
floor. When the cavity
the sphere and the plank.
is at lowest position,
13. B and C are identical uniform cylinders, each having centre of the sphere
mass m and radius r. Uniform plank A has mass M has velocity v. Write
and is kept symmetrically on the two cylinders. A the kE of the sphere
horizontal force F pulls the plank such that there is at this moment.
no slipping anywhere. Find speed of A after it has
19. A block of mass m = 4 kg is attached to a spring of
moved through a distance L.
force constant k = 32 N m–1 by a string that passes
over a pulley of mass M = 8 kg. Pulley is a disc.
System is released
from rest with
spring unstretched.
Find the speed
of the block
14. A uniform disc of mass m can freely rotate about after it has fallen
a horizontal axis fixed to a through 1m.
mount of mass M as shown.
20. A rigid body is made of three identical thin rods,
A constant horizontal force F
each having length l, fastened together to form let-
is applied to the free end A
ter ‘H’. This body is free to rotate about a hori-
of the light thread wound on
zontal axis AB that runs along the length of one of
the disc. There is no friction
the arms of H. The
between the mount and the horizontal surface. Find
body is allowed to
(a) the acceleration of point A on the thread. fall from rest from
(b) the kinetic energy of this system t seconds after a position in which
the beginning of the motion. the plane of H is
15. A uniform ball of radius r rolls without slipping horizontal. What is
down the top of a fixed sphere of radius R. Initial the angular speed
velocity of the ball is negligible. of the body when
plane of ‘H’ becomes vertical?

6.65
21. A stick of mass m and length l is lying flat on a the edge of the disc as
horizontal table. It is struck with a hammer. The blow shown. The disc can
is made perpendicular to the stick at one end. Let the rotate freely about a
blow occur quickly, so that the stick doesn’t have fixed horizontal chord
time to move much while the hammer is in contact. PQ that is at a dis-
Due to the blow, the COM of the stick starts moving R
with a velocity v. Find velocities of the two ends of tance __​   ​  from the cen-
4
the stick immediately after the hit. tre C of the disc. The line AC is perpendicular to PQ.
22. Two particles of same mass Initially, the disc is held vertical with point A at its
m move on a circle of radius highest position. It is then allowed to fall so that it
r while always remaining at starts rotating about PQ. Find the linear speed of the
the ends of a diameter. Both particle as it reaches its lowest position.
have speed v. Find the angu- 27. A particle of mass m is projected at time t = 0 from
lar momentum of the system a point O with a speed u at an angle of 45° to the
about a point O that is at a distance R from the centre horizontal. Find the magnitude and direction of angu-
of the circle. lar momentum of the particle about the point O at
u
23. A spool has mass M and outer radius R. A light time t = __ ​ g ​.
R
thread is wound on its inner disc of radius r = ​ __ ​ . 28. A homogeneous rod __of
2
The thread is pulled vertically with a force F. Find mass M and length ​÷ 2 ​  R
acceleration of the centre of the spool in two cases is released inside a smooth
shown in (A) and (B). The spool does not slip and its cylindrical surface from the
radius of gyration (k) about an axis through centre position shown. Radius of
R__ the cylinder is R. The rod
perpendicular to the figure is ​ ___   ​.  Assume that the
÷   
​ 2 ​ slides along the cylindrical
spool remains on horizontal surface. surface in vertical plane. Find the velocity of COM
of the rod at the instant it becomes horizontal.
29. A uniform plank of
mass M and length
2L is placed on two
identical cylindrical
rollers, as shown
in the figure. Each
roller has mass
M and radius R.
Inclination of the
24. Figure shows a track which has a cylindrical part of
incline is q = 30°.
radius R attached to a horizontal part. A sphere of
Initially, the separation between centres of the two
radius r is set rolling with speed v on the horizontal
rollers is L and one end of the plank lies on roller
part. The sphere does
A. System is released from rest. There is no slipping
not slip anywhere
anywhere. Find the speed of the plank at the instant
and completes the
it is about to leave the rear roller.
vertical circle. Find
minimum value of 30. Figure shows two cyl-
v. inders of radii r1 and
r 2 having moment
25. A uniform slender rod AB of
of inertia I1 and I2
mass M and length L is sup-
about their respec-
ported by a frictionless pivot at
tive central axes. The
A. It is released from rest in its
cylinders are spinning
vertical position (See fig.). Find
about their axes with angular speeds w1 and w2, as
the force applied by the pivot on
shown. They are brought closer and put into contact,
the bar when it gets horizontal.
keeping their axes parallel. The cylinders first slip
26. A uniform circular disc has radius R and mass m. over each other but the slipping finally ceases. Find
A particle, also of mass m, is fixed at point A on

6.66
the angular speed of the cylinders after the slipping (a) Find the linear speed (i.e., speed of COM) of the
cease. cylinder immediately after it is on the inclined
31. A particle of mass plane.
m is rotating in a (b) Find maximum value of v0, which permits the
circle of radius R cylinder to rolls on the inclined plane without
with angular speed a jump.
w, as shown in fig- 35. A uniform thin rod with mass
ure. Centre of the M = 0.6 kg and length L = 0.3 m,
circle (C) has co-or- stands on the edge of a smooth table
dinates (0, 0, ) and as shown. The rod is given a sharp
the plane of rotation below at a height of 0.2 m from
is parallel to xy plane. At time t = 0, the particle is the table. The impulse imparted is
located at (R, 0, a). J = 6 Ns. Find kinetic energy of the
(a) Find the angular momentum of the particle rod t = 1.0 s after the blow.
about the origin (O) at time t. 36. Two identical discs can rotate freely
(b) Find the angular momentum of the particle about their central axles. The axles of
about z-axis at time t. the two discs are connected using a light rod. One
32. Two small balls – A and B – each of of the discs is given an angular speed w0 about its
mass m, are joined using a light rigid axle and the system in placed on a rough horizontal
rod of length L. The system is placed surface. Find the final linear velocity of the system
on a smooth table and is translating when the two discs stop sliding.
with velocity v0 in a direction perpen-
dicular to the length of the rod. A par-
ticle P of mass m kept at rest on the
table, sticks to the ball A, as the ball
collides with it. Find, after collision,
(a) velocity of COM of the system
(A + B + P) 37. A ball rolls without sliding on a rough horizontal
(b) the angular speed of the system about COM floor with its centre moving towards a wall at a
and velocity of 14 ms–1. The ball collides with the smooth
(c) linear speed of A and B. wall. Coefficient of restitution is e = 0.7. Calculate
the velocity of the centre of the ball a long time after
33. A solid cube of mass M and collision.
side length 2a is placed on
a horizontal surface. It can 38. A rod of length L is held above a hard floor making
rotate freely about an axis an angle q with the horizontal. It is released from
passing through point A and position shown. End A of the rod makes an elastic
perpendicular to the plane collision with the floor after falling through a height
of the figure. Any other motion of the cube is not h. Find angular velocity of the rod immediately after
allowed. A bullet of mass m hits the cube while fly- impact.
4a
ing horizontally with a speed v at a height ​ ___ ​ above
3
the horizontal surface. The bullet gets embedded into
the block. Find minimum value of v needed to topple
the cube. Assume m < < M.
34. A uniform solid
cylinder of
radius R rolls
over a horizontal
plane passing 39. A block of mass M = 1 kg rests on a smooth horizontal
into an inclined surface. There is a semi-cylindrical track of radius
plane forming R = 62.5 cm cut in the block. A cylinder of mass
an angle q with horizontal. Friction is large enough m = 0.5 kg and radius r = 10 cm is held at the top of
to prevent slipping. Linear velocity of the cylinder the track with its axis horizontal and at the same level
on the horizontal plane is v0.

6.67
as the axis of the track (see fig.). System is released 40. Two uniform thin rods, A and B,
from this position. The cylinder does not slip on the of length L each and masses M
track. When the cylinder reaches the bottom of the and 2M respectively, are rigidly
track joined end-to-end. The combi-
(a) Calculate velocity of its centre nation is pivoted at the lighter
end P, as shown, such that it
(b) Calculate force applied by the block on the
can freely rotate about point P
floor.
in vertical plane. A small object
of mass M, moving horizontally,
hits the lower end Q of the com-
bination. What should be veloc-
ity (u) of the object so that the system could just be
raised to horizontal position, if
(a) the particle sticks to end Q.
(b) the coefficient of restitution is e.

6.68
Answers Sheet
Your Turn
7
1. (a) 990 gcm2 (b) 520 gcm2 (c) 1510 gcm2 2. ​ __ ​  ml2 3. MR2
3
4. MR2 5. 1.1 kgm2 6. Disc with smaller density
5 ml2 l0 L4
7. ​ __ ​  ml2 8. ​ ___ ​  
  9. ​ _____
 ​  
  10. 0.2%
3 2 4

​ 
m+M
mM
11. I = mL2 where m = ______     ​
2M a______
12. ​ ___ ​ 
5 ( 
5
 ​ ​  3
– b5
a – b3
  )
 ​  ​
÷
1__
13. ​ ___  ​ 
  
​ 2 ​
__
g
    __
3​÷3 ​ mg sin q (m1 – m2 sin q) g
14. 75 rad s –1
15. (i) ​ ____  ​   ​  (ii) Zero 16. a = _______
 ​  ​    ​ 17. a = ​ _____________
   ​
  
4 l M I
m + __
​   ​  m1 + m2 + ___
​  2   ​ 
2 R
F 2 mg sin q 3g 3g 4Mg
18. (i) __
​     ​ (ii) Zero 19. ​ ________   ​ 20. (i) ​ ___ ​ (ii) ___
​   ​ (iii) ____
​   ​ 

M M + 2m 7l 7 7
4v
21. 4g/5 22. v is larger. 23. ​ ___ ​   24.(a) Disc
3

÷ 
_________
10 v
(b) v = ​ ___    ​ , w = __
​   ​ gL sin q  ​ r ​ 25. mmin = 0.5 26. Zero
7
5 m g cos q 4F F
27. (a) g (sin q – m cos q) (b) _________
​      ​   28. ​ ___  ​   29. ​ __ ​  towards right
2R 3M 3
8mg 3 1
30. ​ ________    ​ 31. ​ __ ​  Mv2 32. 1 33. ​ __ ​  ml2 w2
3M + 8m 4 2

÷ 
___
g 13
34. 80p J 35. 3,200 W 37. (a) 3 ​ ___
​     ​ ​ (b) ___
​   ​  Mg
2L 4

÷ 
_____
Mv2 14 _________
38. – ​ ____
 ​  
  39. ​ ___
​   ​  gR ​    
40. ​÷2h (H – h) ​

7 3
41. (a) mvr Directed out of the plane of figure (b) mvr Directed out of the plane of figure
___ mu3 cos q ◊ sin2q 2mu3 ◊ sin2q ◊ cos q 3
  ​  
42. ml ​÷2gl  43. (a) _____________
​       ​  (b) _______________
​  g     

​ 44. ​ __ ​  MR2w
2g 4
3 2 2
45. ​ __  ​ MvR 46. ​ __ ​  MvR anti-clockwise 47. ​ __ ​  MRv
2 3 5
aT2
48. 0.25 rad s–1 49. ​ ____   ​ 50. 2R, R
MR
51. Length of day will increase if we move to the equator and will decrease if we move to the poles.
Mw0 w2R2
52. yes 53. ​ _______   
 ​ 54. Will not change 55. (a) w (b) ​ _____
   

M + 2m 2g
__ __
2mu
_______ ÷    (​÷2 ​
    – 1) mu
​_____________
2 ​
56. 3 m 57. ​    
 ​ 58. ​       ​ 
2m + M MR
u 3u sin q mu mu (h – R)
59. (a) __
​   ​   (b) _______
​      ​   60. (a) ​ ______
    ​  (b) ​ ____________  
  
2 L M+m
( 
2
)
 ​
​ __
​   ​  M + m  ​ R2
5

6.69
Worksheet 1
1. (d) 2. (d) 3. (a) 4. (a) 5. (b) 6. (d) 7. (d) 8. (a) 9. (a)
10. (c) 11. (d) 12. (b) 13. (a) 14. (c) 15. (b) 16. (d) 17. (a) 18. (b)
19. (b) 20. (a) 21. (a) 22. (d) 23. (c) 24. (b) 25. (b) 26. (a) 27. (c)
28. (b) 29. (a) 30. (b) 31. (b) 32. (c) 33. (b) 34. (a) 35. (b) 36. (a)
37. (c) 38. (d) 39. (b) 40. (a) 41. (a) 42. (d) 43. (b) 44. (c) 45. (b)
46. (b) 47. (a) 48. (d) 49. (b) 50. (d)

Worksheet 2
1. (c) 2. (d) 3. (a,b,d) 4. (a,b,c) 5. (b,c,d) 6. (a,c) 7. (b) 8. (c) 9. (b)
10. (b,c) 11. (c,d) 12. (a,b,c,d) 13. (a,c,d) 14. (a,b,c) 15. (a,c) 16. (c) 17. (a,b,c) 18. (b,c)
19. (a,b,c) 20. (a,b) 21. (a,b,c) 22. (b,d)

Worksheet 3
44 ML2 17
1. 0.404 kgm2 2. I = ___
​   ​ pr0R5 3. ​ ____  sin2 q
 ​  4. ​ ___ ​ Ma2
45 3 12
5 3g mg
5. (a) __
​   ​  N (b) 7.5 m 6. (a) 14.28 N, 25.22 N (b) 2.14 ms–2, 3.21 ms–2 7. (a) ​ ___ ​ (b) ​ ___ ​ 
3 2 4
2 v 2
11. (a) __
​   ​  (b) ____ ​ < __
0
8. g (≠) 9. 137.6 10. 2p R ​      ​    ​
5 w0R 5

÷  [ 
________

2F
12. ​ _________
   
(7M + m)g
 ​ ​ 
8FL
13. ​ ________
  
4M + 3m
 ​ ​
  [ 
2 ______
14. (a) F​  ​ __
1
m  ​  + ​ M + m  ]
 ​  ​ (b) F2t2 ​ __
1 1
​ m  ​  + ​ ________
   
2 (M + m)
 ​  ​ ]
÷ 
__________


10
15. (a) q = cos–1 ​ ___
17 (  ) l 10g (R + r)
​   ​   ​ (b) w = _​ r ​  ​ ​ _________
17
 ​ ​
    
  
3R
16. ​ ___ ​  cos q
8
17. 0.27

÷ 
__
31 3 g
18. ​ ___ ​  Mv2 19. 2.4 ms–1 20. w = __ ​   ​  ​ ​ __ ​ ​  
40 2 l
21. The end that is hit moves with velocity 4v and the other end has a velocity 2v in opposite direction
F
22. 2mvr 23. a = ___​    ​ in both cases. Towards left in case (A) and towards right in case (B).
3M

÷ 
__________ ___
27 ÷    
​ 37 ​ ____
24. ​ ___
​   ​ g (R   
– r) ​ 25. ​ ____ ​ 
 Mg    
26. ​÷5gR ​
7 4
___________
__
mu__3 1
27. ​ _____   ​ directed perpendicular to the plane of the motion   ÷2 ​
28. ​ __  ​ ​÷3 (​     – 1) 
   gR ​
   g
2​÷2 ​ 2

÷  [  ] [  ]
____
4 I1 w1 r2 + I2 w2 r1 I1 w1 r2 + I2 w2 r1
29. v = ​ ​ __ ​  gL  
​ 30. ​ ​  ______________
  
   ​  ​ r2 and ​ ​  ______________
  
    ​  ​ r1
7 2 2
I1r2 + I2r1 I1 r22 + I2r12
2v0 v0 v0
31. (a) mR2 w – mRaw [cos (wt)  + sin (wt)  ] 32. (a) ​ ___ ​  (b) ___
​    ​ (c) vA = __
​   ​ ; vB = v0
3 2L 2
M __ 1
__
33. v = __
​   ​ 
    – 1​)​2 ​]
​ m ​  [3ag (​÷2 ​

÷ 
_______________

÷ 
_________________
4 7gR 4
34. (a) v = ​ v02 + __ cos q) ​ (b) v0 max = ​ ____
​   ​  gR (1 –   ​   ​   __
 cos q –  
​   ​  gR ​ 35. 70 J
3 3 3
____
w0R   ​ cos q
6 ​÷2gh 
36. ​ ____
 ​  
  37. 3 ms . –1
38. ​ ____________
  
    ​
6 L (1 + 3 cos2 q)

÷  ÷ 
_____ ______
11 gL 1 567gL
39. (a) 2 ms –1
(b) 26.67 N 40. (a) ​ _____
​   ​ ​ 
  (b) ​ ______
    ​ ​ ______
​   
  
72 (1 + e) 20

6.70
Chapter  7

Miscellaneous Problems
on Chapter 4 to 6

MATCH THE COLUMNS 2. A uniform disc performs pure rolling


motion on rough horizontal surface
Match the entries in Column I with those in Column II. An with constant angular velocity. O is the
item in Column I can match with any number of entries in centre of the disc and P is a point on
Column II. It may also happen that an item in Column I does the disc. then match the statements in
not match with any of the entries in Column II. Column I with Column II.
1. Square frame shown in figures is made of four
Column I Column II
uniform rods. Mass of each rod is m and length is
. Figures show rotation axis. Column II has moment (a) Velocity of point P on (p) Magnitude changes with
of inertia of the square frame about an axis. Match disc time
the entries in the two columns. (b) Acceleration of point P (q) Always directed from
on disc point P towards centre
Column I Column II O of the disc
(Axis of rotation) (Moment of inertia) (c) Tangential acceleration (r) must be directed from
(about axis of rotation) of point P on disc point P to the centre of
disc at some instant, not
(a) ​ 2 ​ m2
(p) __
always.
3
(d) Normal acceleration of (s) Non-zero and remains
point P on disc constant in magnitude.

3. A rigid cylinder is kept on a smooth horizontal


surface, as shown. If Column I indicates velocities of
(b) ​ 5 ​ m2
(q) __ various points (3-centre
3
of cylinder, 2-top point,
4-bottom point, 1-on the
level of 3 at the rim)
on it, choose correct
(c) (r) ​ 7 ​ m2
__
3 state of motion from
Column II.
Column I Column II
​_› _
​›
(a) ​v  ​ 1 = + , ​v  ​ 2 = 2  (p) Pure rotation about
centre
(d) (s) ​ 8 ​ m2
__ _
​ _
​›
3 (b) ​v › ​1  = + , ​v  ​3  = –  (q) Rolling without slipping
to left
_
​ ​_
(c) ​v › ​ 2 = , ​v › ​ 3 = 0 (r) Rolling without slipping
to right
_
​ ​_
(d) ​v › ​4  = 0, ​v › ​1  = –  – (s) Not possible
7.2  Mechanics II

_
​›
4. A particle moves
_
​›
with position given by ​r ​   = 3t  + Column I Column II
4  . Where, ​r ​   is measured in metres and t (> 0) in
(a) (p) centre of mass accel-
seconds. erates towards left
Column I Column II
(a) Rate of change of distance (p) Increasing with
from origin. time (b) (q) centre of mass accel-
(b) Magnitude of linear (q) Decreasing with erates towards right
acceleration of particle time
(c) Magnitude of angular velocity (r) Constant
of particle about origin
(d) Magnitude of angular (s) Zero
(c) (r) friction acts towards
momentum of particle about
left
origin

5. Assume that a spherical ball is kept on a rough


ground. Column I indicates
situation related to ball and
column II indicates the effect (d) (s) friction acts towards
that friction has on the ball. right

Column I Column II
(a) Ball is suddenly given (p) Increases vcm
clockwise w, with centre of
mass initially at rest.
8. Two solid uniform round objects of equal mass but
(b) Ball is given a velocity to (q) Decreases vcm different radii rA and rB, shown in figure, can rotate
the right without any w
freely about horizontal axles passing through their
(c) Ball is given clockwise w (r) Increases w respective centres. On each of them, light inextensible
and given velocity to the strings are wound. Free ends of both the threads are
right such that vcm < wR fixed to the ceiling and the objects are released dur-
(d) Ball is given clockwise w (s) Decreases w ing the subsequent
and velocity to the right motion; assume
such that vcm > wR their axles to
remain horizontal.
6.
 represent the
Column I Column II length of thread
(Initially) (When pure rolling begins) unwound in time
t after release and
(a) (p) vcm is towards left in
2v a is acceleration
w = ​ __  ​ case of ring
R of the centre.
(b) (q) vcm is towards left in Column I Column II
2v
w = ​ __  ​ case of solid sphere
(a) A is disc and B is ring with rB > rA (p) aA > aB
R
(b) Both A and B are discs with rB > rA (q) lA > lB
(c) (r) vcm is towards right in
​  v  ​ 
w = ___ case of ring (c) A is disc and B is ring with rA = rB (r) lA = lB
2R
(d) Both A and B are discs with rA = rB (s) aA = aB
(d) (s) vcm is towards right in
​  v  ​ 
w = ___ case of solid sphere
2R
PASSAGE-BASED PROBLEMS
7. A yo-yo is resting on a rough horizontal table where Every passage is followed by a series of questions. Every
friction is large enough to prevent slipping. Forces question has four options. Choose the most appropriate
F1, F2, F3 and F4 are applied separately on the free option for the questions.
end of the thread, as shown.
Miscellaneous problems on chapter 4 to 6  7.3 

Passage 1 5. When the sphere is rolling down, the man wants to


run along with the same acceleration, without exert-
A solid uniform cylinder of
ing any force on it.
mass m has a light thread wrapped
on it. It is kept in equilibrium on (a) friction on the man would be backwards and
a fixed incline of angle a = 37° equal to that on the sphere
with the help of a force F applied (b) friction on the man would be forwards and
on the free-end of the thread. greater than on the sphere
1. What force F is required to keep the cylinder in (c) friction on the man would be backwards and
balance when the thread is pulled vertically, as less than on the sphere
shown? (d) friction on the man would be forwards and equal
mg 3mg
(a) ​ ___ ​
  (b) ​ ____
 ​ 
  to that on the sphere
2 4
6. What is the minimum friction coefficient for the man
3mg to just push the sphere up when he pushes at the top
(c) ​ ____ ​
   (d) None of these
4 point in a direction parallel to the hill
2. If there is flexibility to apply force F in any direction, 3 q
(a) ​ __ ​  tan q (b) tan ​ __ ​ 
then the minimum force required to hold the cylinder 2 2
is 2 5
3mg (c) ​ __ ​  tan q (d) ​ __ ​  tan q
(a) ​ ____ ​ parallel to incline 7 7
10
3mg Passage 3
(b) vertical ____ ​   ​  
8 A cardboard strip is in the shape of a rectangle of length 3a
mg
(c) horizontal ___ ​   ​  and width a. The extreme one-third portions are bent at right
5 angles to the middle part so as to give the strip a shape as
(d) None of these shown in the figure. It is put on a rough inclined plane, as
3. The values of minimum coefficient of static friction shown in the figure.
between cylinder and incline in the cases mentioned
in above two questions are: ______ and _____
respectively:
4 3 4 3
(a) ​ __ ​ , __
​   ​   (b) ​ __ ​ , __
​   ​ 
3 4 3 8
3 3
(c) ​ __ ​ , __
​   ​   (d) None of these 7. At what minimum angle of inclination of the inclined
4 8 plane will the cardboard strip topple? (assume that it
does not slide).
Passage 2
In a mythological story, a worker was punished by a king.
2
(a) tan– 1 ​ __(  )
​   ​   ​
3 (  )
3
(b) tan –1 ​ __
​   ​   ​
2
The punishment was to roll a spherical stone up a hill from
(d) tan   ​( __
​    ​ )​
–11
where it would invariably roll down to the other side. He (c) 45°
3
would run after the stone; will catch it and roll it back
uphill. The sphere and the worker 8. What should be the coefficient of friction so that it
had the same mass. Coefficient of does not slide before toppling.
friction between the man and the hill (a) 0.66 (b) 0.75
and that between the sphere and the (c) 1.0 (d) 0.33
hill is the same. Neither the sphere 9. What should be the time taken to travel a distance a,
nor the man slips. if the inclined plane is made completely smooth?
4. At what height from the centre (measured normal to

÷  ÷ 
______ ______
the hill surface) should the man push, in a direction a 2a
(a) ​ ______
​       ​   (b) ​ ______
​      
​ 
parallel to the hill, to roll the sphere slowly, with g sin a g sin a

÷ 
minimum force. _______
2a
(a) 0 (b) R/2 (c) ​ _______
​     ​ 
  (d) None of these
3g sin a
(c) 2/5 R (d) R
7.4  Mechanics II

Passage 4 12. If several wrenches are used one after another to


apply the same torque on the nut, which graph best
A winch is mounted on an axle, and a six-sided nut is welded
expresses the relationship between the average force
to the winch. By turning the nut with a wrench, a person
the person must apply on the wrench, and the length
can rotate the winch. For instance, turning the nut clockwise
of the wrench?
lifts the block off the ground, because more and more rope
gets wrapped around the winch.
Three students agree that using a longer wrench makes it
easier to turn the winch. But they disagree about why. All (a) (b)
three students are talking about the case where the winch is
used, over a 10-second time interval, to lift the block one
metre off the ground.
Student A
By using a longer wrench, the person decreases the average (c) (d)
force he must exert on the wrench, in order to lift the block
one metre in ten seconds.
Student B Passage 5
Using a longer wrench reduces the work done by the A smooth hill has height h. A small solid cylinder of radius
person. r and mass M slides down from rest and gets onto a long
Student C rough plank of mass M, lying on the smooth horizontal
Using a longer wrench reduces the power that the person plane at the base of the hill, as shown in the figure. Due
must exert to lift the block. to friction between the cylinder and the plank, the cylinder
slows down and starts rolling with-
out sliding over the plank. When
the cylinder starts rolling without
sliding over the plank, the velocity
of the plank is vP, the velocity of
the centre of mass of the cylinder
is vC and angular velocity of the
cylinder is w.
13. Which of the following is correct?
10. Student A is (a) vC = wr (b) vC < w
(a) Correct, because the torque that the wrench (c) vC > w r (d) none
must exert to lift the block doesn’t depend on 14. The value of vC is
the wrench’s length 3 ____
(a) constant and equals to __
​   ​  ÷    
​ 2gh  ​
(b) Correct, because using a longer wrench decreases 4
1 ____
the torque it must exert on the winch. (b) variable with (vcm ) max = __ ​   ​  ÷    
​ 2gh ​
2
(c) Incorrect, because the torque that the wrench ____
must exert to lift the block doesn’t depend on (c) constant and equal to ÷   ​ 
​ 2gh 
the wrench’s length ____
(d) variable with (vcm)max = ÷   ​ 
​ 2gh 
(d) Incorrect, because using a longer wrench
decreases the torque it must exert on the 15. Which of the followings is incorrect?
____
winch.   ​ 
(a) 2vp + wr = ​÷2gh 
____
11. Which of the following is true about students (b) 2vC – w r = ÷   ​ 
​ 2gh 
B and C?
(c) vp = vC – w r
(a) Students B and C are both correct.
(d) none
(b) Student B is correct, but student C is incorrect.
(c) Student C is correct, but student B is incorrect. Passage 6
(d) Students B and C are both incorrect. A yo-yo is pulled by its string along a
horizontal surface without slipping. The
horizontal velocity of the end of the string
Miscellaneous problems on chapter 4 to 6  7.5 

remains equal to v. A smooth bar is pivoted as shown and 2v 02


certain surface ‘X’, the ball covers the distance AB = ___
​  g   
​. 
remains supported by the yo-yo. The outer and the inner
radii of the yo-yo are R and r, respectively. Assume that all the surfaces used in the experiment are per-
16. The velocity of the centre of mass of yo-yo is fectly elastic.
v (R + r)
(a) vr/R (b) ​ _______
    ​ 
R
v (R – r) vR
(c) ​ _______
    ​   (d) ​ _____
   ​
R R+r
21. The coefficient of friction between the surface ‘X’
17. The angular speed of the bar as a function of q is and the ball is

(  ( 
2
​     ) sin  (q/2)
2v ________
(a) ​ _____  ​  ​ ​ 
R + r cos (q/2)
v
 ​  (b) ​ _____
​      ) 1
 ​  ​ ​ _____
R + r sin2q
   ​ 


(a) 0.5
(c) 0.75
(b) 0.25
(d) 1.0
2v v (R + r) 22. The magnitude of impulse imparted by surface ‘X’
(c) ​ _____
   ​ sin2 (q/2) (d) ​ _______
 ​ sin q

  to the ball during the collision at A is
R+r R2
(a) mv0 (b) 2 mv0
__ __
Passage 7    mv0
(c) 2 ​÷2 ​  (d) ​÷ 3 ​  mv0
A uniform rod of mass m is supported on two rollers, each of 23. Change in KE of the ball during collision at A is
m
mass ​ __ ​ and radius r. The system is placed on mv 20 9
2 (a) ​ ____ ​  
  (b) – ​ __ ​  mv20
an incline, as shown, and released. Assume 4 2
2
no slipping at any contact and treat the rollers mv  5
(c) – ​ ____ (d) ​ __ ​  mv20
0
 ​  

as a uniform solid cylinder. 4 4
18. If acceleration of rod down the plane is aR and Passage 9
acceleration of the centre of mass of roller is a0,
An open-top, hollow wooden cylinder of outer radius
then
R = 0.25 m is stand-
(a) aR > a0 ing on a frictionless
(b) aR < a0 ice surface.
(c) aR = a0 The cylinder con-
(d) Information is insufficient to decide tains some ice inside
it. A rifle is placed
19. If the friction force on roller at the contact point with very close to the cyl-
incline is f1 and at the contact point with rod is f2, inder and a bullet of
then mass m = 0.5 kg is fired horizontally with a muzzle speed
(a) f1 > f2 of 800 ms–1 so as to pass the cylinder, just toucing it on its
(b) f1 < f2 outer surface, at a height 0.6 m above ground. Eventually,
(c) f1 = f2 the bullet hits a target placed at a distance of 48 m from
(d) data is insufficient to decide the cylinder at a height 0.4 m. After the bullet went past the
cylinder, the cylinder began to rotate with a speed of one
20. If the inclined surface is smooth then acceleration of
rotation per 4 s. [g = 10 ms–2]
roller is
12g Inner radius of cylinder = 0.23 m, density of wood
(a) 2g sin q (b) ​ ____ ​ sin q = 600 kgm–3 height of cylinder = 1 m, and thickness of
11
(c) g sin q (d) none of these base = 0.02 m
24. The velocity of bullet just after touching the cylinder
Passage 8 is
In an experiment, a spinning ball of radius r and mass m is (a) 480 ms–1 (b) 240 ms–1
–1
made to fall on a rough horizontal surface. While hitting the (c) 120 ms (d) 960 ms–1
5v0
​  r   ​.  25. The moment of inertia of the cylinder alonwith its
surface, the ball has a velocity v0 and angular velocity ___
content about the axis shown is
After hitting the surface at A, the ball moves on the path
(a) 2.46 kgm2 (b) 3.46 kgm2
shown and hits the surface again at B. This experiment is
repeated for many surfaces of varying roughness. For a (c) 4.46 kgm2 (d) 8.92 kgm2
7.6  Mechanics II

26. Use the data given and the calculated value of 30. Value of a, for which the solid cylinder has its axis
moment of inertia to find the mass of ice inside the motionless is
cylinder 3 g cos q 2g sin q
(a) ​ __ ​  ​ ______
   
​   (b) ​ _______
    ​ 
(a) 27 kg (b) 227 kg 2 R R
(c) 50 kg (d) 127 kg g sin q 3g sin q
(c) ​ ______
    ​   (d) ​ _______
    ​ 
R R
Passage 10
In an entertainment show, large number of light boards are Passage 12
placed, as shown. Each of them can rotate about a fixed
Three identical light rods –
fulcrum. The fulcrum of each board divides the length of
AC, BC and DC – are rigidly
the board in the ratio of 2:1. At the left end of the left-most
connected to form a T-shaped
board, there is a block of mass 30 kg and a team of joker
structure. At the free ends of
stand keeping their feet at the ends of adjacent boards. Mass
each rod are fixed three par-
of each jokers is 80 kg. The jokers are able to maintain
ticles, each of mass m. The
balance.
structure can rotate freely with-
out friction about a fixed hori-
zontal axis, through junction point C, perpendicular to the
plane of the figure. Structure is released from the position
shown with q = 37°.
31. Immediately after the structure is released
(a) acceleration of particle at A and D are
perpendicular to each other.
27. NLA represents the force applied by the left foot of
(b) acceleration of particle at A and B are in same
joker A on the board and NRA is the force applied by
direction.
the right foot of joker A on the board. Symbols NLB,
NRB, etc., have similar meaning. (c) magnitude of acceleration of COM of the system
is half in magnitude to the acceleration of the
(a) NLB = NRB (b) NRA > NLB
particle at B.
(c) NLA + NRA < NLB (d) NRB > NLB
(d) None of the above
28. Maximum number of jokers that can keep balance in
32. Magnitude of force applied by rod CD on the particle
this way is
at D is
(a) 3 (b) 4 mg
(a) 2mg (b) ​ ___
__ ​ 
(c) 2 (d) 5   
​÷5 ​
2mg
(c) ​ ____
__ ​   (d) 3mg
Passage 11   
​÷5 ​
A hollow cylindrical drum of radius
R is kept rotating with constant Passage 13
angular acceleration (a) about its A billiard player strikes a ball (a uniform sphere with mass
stationary horizontal axis, through M and radius R) with a stick at the middle (i.e, at a height R
its centre (O). A uniform solid above the table at A). The impact
cylinder is placed inside the drum is along a line making an angle q
with its axis horizontal. It was found with horizontal. The impact causes
that the solid cylinder rolls without the ball to move towards right as
sliding inside the hollow cylinder well as ‘‘back spin’’ – spin in anti
at angular position q and its axis clockwise direction
remains motionless. Coefficient of kinetic friction between the ball and the
29. If radius of the solid cylinder is r, then its angular table is m. Assume that the ball does not rebound off the
acceleration about its axis is table due to the strike and neglect the impulse of friction
ra
(a) a (b) ​ ___ ​  due to table on the ball during the period of impact. It was
R observed that the ball came to complete rest after travelling
Ra (R – r) a through some distance.
(c) ​ ___
r ​    (d) ​ ________
   ​ 
R
Miscellaneous problems on chapter 4 to 6  7.7

33. Value of q must be 2w0 w0


(a) ​ ____ ​    (b) ​ ___ ​ 
(a) 45° (b) 30° 3 3
–1 __2
(  )
(c) tan   ​ ​   ​   ​
5
1
(  )
(d) tan–1 ​ __
​   ​   ​.
5

w0
(c) ​ ___ ​  
4
(d) 0
34. distance that the ball travels before coming to rest
is proportional to Passage 15
1 R A solid cylinder of
(a) ​ ____
   ​   (b) ​ ____
   ​  radius R is rolling with-
mM2 m 2
out sliding on the inner
R2 1
(c) ​ ____  ​   (d) ​ ___   ​  surface of a stationary
mM2 mM cylinder of radius 2R.
At the instant shown,
Passage 14 velocity of the centre
Three identical cylinders of rough surfaces are mounted on of the rolling cylinder
frictionless horizontal axles passing through their centres. is v0. A is a massless
They are all spinning about dust particle stuck at
their axles in anti-clockwise point A on the solid
direction with same angular cylinder.
speed w0. They are brought 38. Co-ordinates of the centre of the solid cylinder, when
closer into contact and their the co-ordinates of the dust particle becomes (0,0),
axels are held stationary till are
slipping between them ceases. (a) (0, R) (b) (– R, 0)
[The cylinders are held sym- (c) (R, 0) (d) None
metrically relative to one
39. Speed of the centre of the solid cylinder when
another as shown, so that all
co-ordinates of the dust particle becomes zero is
of them touch one another]

÷  ÷ 
________ ________
4 2
35. Angular momentum of the system of three (a) ​ v02 – __
​   ​  gR 

  (b) ​ v02 – __
​   ​  gR  

cylinders is 3 3

÷ 
_______
gR
(a) conserved about the axle of any of the three (c) ​ v02 – ___
​   ​ ​  
  (d) zero
cylinders. 3
(b) conserved about contact point of any two Passage 16
cylinders.
A student is sitting on a chair that can rotate about the
(c) conserved about any random point P
vertical axis (z-axis), as shown. He is
(see figure).
holding a spinning wheel. Initially, the
(d) not conserved about any of the above-mentioned chair is not rotating but the wheel is
points. rotating about its central vertical axis
36. Final angular speed of cylinder B will be with angular speed w0. Distance of the
w0 w0 axis of the spinning wheel from the
(a) ​ ___ ​   (b) ​ ___ ​  rotation axis of the chair is a. Moment
3 2
w of inertia of the chair and student com-
(c) ​ ___ ​  
0
(d) 0 bined is I1 about the z-axis shown.
4
Moment of inertia of the wheel about
37. The arrangement of the three cylinders is changed its central rotation axis is I0 and mass
as shown in figure. Now, what is their final angular I0
velocity if they are brought into contact? of the wheel is equal to ​ ___ 2   ​. Take
3a
I1 = 4I0

[Neglect friction in bearings of the wheel and the chair]


40. Angular momentum of the entire system about z-axis
is
3
(a) I0w0  (b) ​ __ ​  I0w0 
2
1
__
(c) ​   ​  I0w0  (d) zero
2
7.8  Mechanics II

41. The student turns the spinning wheel upside down. 42. Speed of the centre when body A is at the highest
Now the angular speed of the chair and the student point is given by

÷  ÷ 
about z-axis will be: ________ _______
w0 w0 v02 – 2gR v02 – gR
(a) ​ ___ ​
  (b) ​ ___ ​  (a) ​ ​ ________
 ​ ​  

  (b) ​ ​ _______
 ​ ​ 


4 8 3 3

÷  ÷ 
3w 5w ________ _______
(c) ​ ____ (d) ​ ____
0 0
 ​

   ​   v20 – 2gR v20 – gR
8 8 (c) ​ ​ _______
 ​ ​  

  (d) ​ ​ ______
 ​ ​ 


2 4
Passage 17 43. Maximum value of v0 so that the hoop remains in
A small body A is fixed to the inside of a thin rigid hoop of contact with the horizontal plane when body A is at
radius R and mass equal to that of the the top, is
___ ____
body A. The hoop rolls without slip-   ​  
(a) ​÷gR    ​ 
(b) ​÷2gR 
ping over a horizontal plane. At the ____ ____
instant the body A is at lowest posi-   ​  
(c) ​÷4gR    ​ 
(d) ​÷8gR 
tion, the centre of the hoop moves
with velocity v0.
Answers Sheet
Match the Columns
1. (a) s  (b) q  (c) p  (d) r 2. (a) p  (b) q, s  (c) r  (d) r
3. (a) r  (b) s  (c) p  (d) q 4. (a) p  (b) r, s  (c) q  (d) r
5. (a) p, s   (b) q, r  (c) p, s  (d) q, r 6. (a) p, q   (b) q, r  (c) p, q  (d) p, q
7. (a) q, r  (b) p, r  (c) q, r  (d) r 8. (a) p, q  (b) r, s  (c) p, q  (d) r, s

Passage-based Problems
1. (c) 2. (a) 3. (c) 4. (d) 5. (a) 6. (a) 7. (a) 8. (a) 9. (b)
10. (a) 11. (d) 12. (d) 13. (c) 14. (a) 15. (d) 16. (d) 17. (c) 18. (a)
19. (a) 20. (c) 21. (d) 22. (c) 23. (b) 24. (b) 25. (c) 26. (d) 27. (a)
28. (a) 29. (c) 30. (b) 31. (a) 32. (c) 33. (a) 34. (a) 35. (d) 36. (d)
37. (b) 38. (c) 39. (a) 40. (a) 41. (c) 42. (a) 43. (d)

7.9
Chapter  8

Fluid Mechanics
‘‘Eureka! – I have found it!’’
–Archimedes

1.  Introduction Density of the most common liquid, water, at 4°C is


1 gcm–3 or 1,000 kgm–3. Though density of water changes
Fluid is a substance that can flow. Liquids and gases are (slightly) with temperature, we will disregard this variation
fluids. A solid has definite volume and shape. A liquid has in most of our discussions and assume its density to be
definite volume but no definite shape and an unconfined nearly 1 gcm–3 for a fairly wide range of temperature.
gas has neither a fixed volume nor a definite shape. Relative density of a solid or a liquid is defined as the
Intermolecular forces are strong in a solid. These forces are ratio of density of the substance to the density of water
relatively weak in a liquid and therefore, allow the shape to at 4°C.
be easily changed. However, it is rather difficult to change density of substance
the density of a liquid. In gases, intermolecular forces are \ Rd = ​ ____________________
   
    ​ …(2)
density of water at 4°C
very weak. This allows easy change in shape as well as
volume. Relative density (Rd) is a ratio and has no unit. It is also
In this chapter, we will mainly discuss the properties known as specific gravity.
of a liquid, though most of the results that we obtain are
applicable for gases as well. We will discuss only about an Example 1   Gold is heavy
ideal liquid. A liquid that is incompressible and non-viscous Relative density of gold is 19.6. Find mass of a gold bar
is said to be ideal. Incompressibility means that density of having volume of 1 litre.
the liquid cannot be changed even by changing pressure.
Solution
Liquid is non-viscous – this implies that liquid surfaces
in contact do not exert tangential force on one another. In Concepts
simple language, two layers of a liquid do not exert friction (i) Density of substance = (Rd) (density of water)
on one another. They apply force that is normal to their
surface. (ii) 1 litre = 1,000 ml = 1,000 cm3
Mechanics of fluids is important in many engineering
Density of gold = 19.6 gcm–3 [= 19,600 kgm–3]
design. It is equally important for a civil engineer, a
mechanical engineer, an aeronautical engineer and even a Mass of 1,000 cm3 gold is
g
nuclear engineer. m = r ◊ V = 19.6 __
​ cc  ​ × (1,000 cm3)
In this chapter, we will first study about the behaviour of
= 19600 g = 19.6 kg
a static fluid (a fluid that is at rest relative to the container).
Such study is often termed as hydrostatics or fluid statics. In
the later half of the chapter, we will discuss flowing fluids. Example 2   How pure is your jewellery?
Such study is called hydrodynamics or fluid dynamics. A jewellery piece weighs 34 g and has a volume of 2 cm3.
It was sold to you as 22 karat gold. 22 karat gold has
2. Density and Relative Density 91.67% gold and 8.33% other metal. Your jewellery piece
has gold (Rd = 19.6) and copper (Rd = 9.0). How much
Density of a substance is defined as mass per unit volume. gold is there in your jewellery? Do you think that you have
mass m been cheated?
Density = ​ _______
   ​  or,  r = __
​   ​   …(1)
volume v
8.2  Mechanics II

Solution (34 – x)  g 34  –  x


Volume of Cu = _________
​  g    = ​ ______
 ​   ​  cm3
Concepts __
9 ​ cc  ​  9
Mass
(i) Volume = _______
​    ​  x 34 – x
Density \ ​ ____
   ​ + ​ ______ ​ =  2

19.6 9

[ 
(ii) Volume of Au + Volume of Cu = 2 cm3

Let mass of Au in the jewellery be x grams and mass of


1
fi x ​ __
1
​    ​ – ____
9 19.6 ] 34
​     ​  ​ = ___
​   ​ – 2
9

Cu be (34 – x) grams. fi  x [0.11 – 0.05] = 3.78 – 2  fi x = 29.67 g


29.67
xg
Volume of Au = ​ ________
x
 ​ = ____
​     ​ cm3 Percentage of gold = _____
​   ​  × 100 = 87.26%
g  

____ 19.6 34
19.6 ​   3 ​ 
cm Yes, you were cheated!

Your Turn

Q.1  A beaker can hold 0.5 kg water when full. It is filled Q.2  Aluminium bronze is an alloy made by adding 10%
with mercury. What is the mass of mercury that it holds? (by weight) Aluminium to Copper. Find density of the alloy
Relative density of mercury = 13.6. in CGS unit. It is given that relative density of Cu and Al
are 9.0 and 2.7 respectively.

3. Pressure in a Fluid body immersed in it. This force per unit area is termed as
fluid pressure. This arises due to molecules of the fluid
In mechanics of solids, force was a physical quantity of hitting a surface.
immense importance. In mechanics of fluids, it is much Consider a small area DS
easier to talk in terms of pressure. surrounding a point A inside a
When a liquid is at rest, there are no tangential forces fluid. Force on this area due to
(or you can say that there are no shear forces). A tangential liquid molecules hitting this surface
force acting on a liquid layer will cause it to flow. Therefore, from one side is DF. This force is
liquid layers exert only normal force on one another, when normal to the surface. (The surface
there is no flow. If you submerge a solid in a static liquid, experiences equal and opposite
the force exterted by the liquid is always normal to the forces due to molecules on other
surface of the solid. The effect is to compress the solid from side of it. While writing pressure,
all sides (see fig.). you need to consider force from one side only).
Pressure at A is defined as
DF
P = ___
​   ​   …(3)
DS
Pressure is a scalar; it has no direction. The force that
a fluid exerts on a surface has direction. Force is always
normal to the surface. If you hold a book as shown in
figure (a), the force on its cover surface due to atmospheric
pressure is vertically down. When you hold the book as
shown in figure (b), the force on cover surface due to air
is horizontal. In both cases, force = (pressure due to air) ×
(area of cover surface).

Forget about a solid; think of the cube shown in the


figure as a volume of the liquid itself. The neighbouring
liquid compresses it from all sides in a manner identical to
it compressing a solid cube.
Therefore, a fluid at rest exerts force perpendicular to
any surface in its contact – such as a container wall or a
Fluid Mechanics  8.3 

Obviously we cannot assign a unique direction to pressure. Any difference in P1 and P2 will mean that the cylindrical
element is accelerated. [Remember, our fluid is static.]
Unit of pressure
SI unit of pressure is Nm–2 also known as pascal (Pa). A
3.2.2  Pressure Increases with Depth
commonly used unit in meterology is Bar.
1 Bar = 105 Pa Consider a vertical cylinder of
liquid inside a tank filled with
3.1  Atmospheric Pressure liquid of density r. Height of
the cylinder is h and its cross-
Pressure applied by atmospheric air is known as atmospheric sectional area is DS.
pressure. Its mean value at sea level is 1.013 × 105 Pa.
Pressure (P1) is uniform on
Sometimes, pressure is expressed in multiplies of its top surface (the surface is
atmospheric pressure also. Thus, horizontal) and downward force
1 atm = 1.013 × 105 Pa due to surrounding liquid on this
surface is
3.2  Variation of pressure in a static fluid
F1 = P1 D S
When you swim under water, you can feel the water pressure Similarly, pressure (P2) is uniform on its bottom
on your eardrums. The deeper you swim, the greater the surface and upward force due to surrounding liquid on this
pressure. When a balloon is taken deep inside water, we surface is
find that size of the balloon decreases substantially. This is F2 = P2 DS
due to higher water pressure. It means pressure in a fluid
increase with depth. However, pressure remains same at all Weight of the cylindrical volume of the liquid is
points on a horizontal level. Let us understand the variation W = volume × density × g = D S h r ◊ g
in pressure inside a static fluid in much detail. For vertical equilibrium of the cylindrical liquid element,
we must have
3.2.1  Pressure is Uniform on a Horizontal Plane F2 = F1 + W …(i)
fi P2 DS = P1DS + D S h rg
Consider a liquid of density r
kept in a tank. There is no flow. fi P2 = P1 + rgh …(4)
Total force on the entire mass of It means that pressure increases linearly with depth if we
the liquid is zero. If we consider a travel down into a fluid having constant density.
small volume of liquid, net force Look at equation (i). It is actually the weight of fluid
on it is also zero. Consider one column, which causes the pressure to increase with depth.
such volume of the liquid, which Heavier the fluid, higher is the change in pressure with
is in the shape of a horizontal depth.
cylinder having small cross-
sectional area DS. Net force on Example 3   You carry a lot of weight on your head
this element is zero as it has no acceleration. Consider
horizontal forces on this cylindrical fluid element. Consider your head top to be a flat square of side length
20 cm. How much force does atmospheric air apply on your
F1 = P1 DS = force on circular face at 1 due to
head top?
surrounding liquid (Force is towards
right) Solution
F2 = P2 DS = force on circular face at 2 due to Concepts
surrounding liquid (Force is towards
left) Atmospheric Pressure, P0 = 1.0 × 105 Nm–2
P1 and P2 are pressure inside the liquid at 1 and 2 Force due to atmosphere on a surface is F = P0 DS.
respectively. Direction of force is normal to the surface.

( 
For horizontal equilibrium
F1 = F2  fi  P1 DS = P2 DS
m
N
F = P0 DS = ​ 1.0 × 105 ___ )
​  2  ​  ​ (0.2 m × 0.2 m)
fi P1 = P2
= 4,000 N
Thus, pressure at points like 1 and 2, which are on same
You are carrying a load of 400 kg on your head. Why
horizontal level, is same.
don’t your legs crumble? Actually, atmospheric pressure
8.4  Mechanics II

squeezes you from all sides. On every small surface of our However, if one goes deep in a lake, the increased outside
body, force is normal to the surface. At certain locations, the pressure does cause a problem.
force is downward and at some other location, it is upward.
Net force is nearly zero. (We will shortly learn that net force Example 4   Find pressure at a depth of 20 m in a lake.
is upward, known as buoyancy. However, it is small). Atmospheric pressure is P0 = 1.0 × 105 Nm–2.

Solution
Concepts
(i) Pressure on the surface of the lake = Pressure due
to atmospheric air = P0
(ii) Pressure at depth h is P = P0 + rgh

P = P0 + rgh
kg
= 1 × 105 Nm–2 + ​ 103 ___ (  ) (  )m
​  3  ​  ​ ​ 10 __
m
​  2 ​   ​ (20 m)
s
= 1 × 10 5 + 2 × 105 = 3 × 105 Nm–2
Note:  In the above calculation, shape of the lake is not
important.
The next natural question that comes to our mind is-why In fact, for the container shown in the figure
don’t our stomach or lungs get flattened? The answer is Pa = Pb = P0 + rgh, where P0 is
that our lungs are filled with air and stomach also is not atmospheric pressure. [Note that a
empty. The air in lungs and stomach (or the food material in and b are points at same horizontal
stomach) is nearly at atmospheric pressure. Therefore, there level in a static liquid.]
is an outward force also on your stomach muscles. Net force
on our stomach wall is definitely zero. Also, Pc = Pa + rgh

Your Turn

Q.3  How does water pressure one metre below the surface Q.4  Air inside the middle ear is normally at atmospheric
of a small pond compare with water pressure one metre pressure. Surface area of a man’s eardrum is nearly 1 cm2.
below the surface of a huge lake? Estimate the force exerted on his eardrum due to water, when
he is swimming in a lake at a depth of 10 m.

3.3  Pressure in a Gas


pressure at Mt. Everest is much less than the pressure that
When we move down inside a water tank, the pressure we experience at the sea level.
increases according to equation (4). Is this also true for air
inside our room when we move from ceiling to the floor?
The air has too small a density for any significant change in
4.  Force on container surface
pressure over a height variation of few metres. The pressure 4.1  Force on Horizontal Base
inside the room, or inside any reasonable-sized container
is uniform. However, if you consider the large atmosphere Container A and B have same area (S) of their bottom surface.
the variation in pressure will be significant. Atmospheric They are filled to the same height (h) by a liquid of
Fluid Mechanics  8.5 

component. The container wall also exerts same force on


the liquid. As can be seen in the figure, the horizontal
component will add to zero.
There is a net vertical (downward) force (Fw) on the liquid
due to the walls. If F is the force due to the base on the
liquid, then for vertical equilibrium of liquid
F = Fw + weight of the liquid

density r. How much force acts on the bottom surface of 4.2  Force on a Vertical Wall
the two containers?
Consider a wall of width L. A Liquid of density r is filled
First consider container A. Liquid pressure at the bottom
to the right to the wall up to a height H. The force on the
is
wall due to the liquid
P = P0 + rgh, where P0 is atmospheric
cannot be calculated
pressure. Force applied by the liquid on
by multiplying area
the surface is normal to it (in vertically
of the wall twith
downward direction).
pressure. The pressure is
FL = P S = (P0 + rgh) S ...(i) changing with depth.
If air is present below the bottom We must write force
surface (when you keep a beaker on table, on a small patch and
air is present between the beaker and table then integrate to find
surface), it will exert an upward force on the base of the the resultant force.
container in upward direction. This force is Consider a strip of
width dy at a depth y
Fa = P0S ...(ii) from the liquid surface.
Thus, the resultant force that the base experiences (due Area of strip, dS = L dy.
to liquid + atmosphere) is Pressure of liquid at depth y is P = r g y
Force on the strip (in horizontal direction) due to the
F = FL – Fa = rghS ...(iii) liquid is
It is important to note that the force due to atmospheric dF = P ◊ dS = (r gy) (Ldy) = rgL y dy ...(i)
pressure is present on both sides and it cancels out. Total force on the wall is obtained by summing up the
Now, r (hs) g = density × volume × g = weight of liquid force on all such strips.
in the container. Thus, H
1
\ F = Ú dF = r gL ​Ú ​  ​ ​ ydy = __
​   ​  r gL H2 ...(5)
F = Weight of liquid in the container. 0 2
This result is intuitive. Just consider the liquid as your There are two important things that you must notice:
system and study its equilibrium. Its weight is balanced by 1. We have deliberately left atmospheric pressure in our
the force applied by the container base on it. calculations. Atmospheric pressure acts on the wall
On the vertical wall of the container the liquid applies from left to right as well. If you include atmospheric
horizontal force all of which sum up to zero. pressure in your calculations (by taking pressure as
P = P0 + r gy), then effect of atmospheric pressure
Now, consider container B. Pressure at the bottom is
on both sides just cancels out.
again P = P0 + r gh and equations (i), (ii) and (iii) hold true
in this case also. Force 2. Pressure varies linearly with depth. Therefore, aver-
on the base is still F = age pressure due to liquid over the surface of the
r gh S, which is higher wall is the average of the pressure at the top and the
than the weight of the pressure at the bottom.
liquid in the container Ptop + Pbottom 0 + rgH 1
(obviously, B contains Pav = ​ ___________
 ​
    = ​  _______
 ​  = ​ __ ​  rgH
2 2 2
less liquid than A). How
is the force on the base Force on the wall is equal to

( 
larger than the weight
of the liquid?
1
F = (Pav) (Area of the wall) = ​ __ ) 1
​   ​  rgH  ​ (LH) = ​ __ ​  rgLH2
2 2
The liquid exerts force on the slant wall of the container.
This force is normal to the surface and has a vertical Thinking this way, you can avoid integration.
8.6  Mechanics II

4.2.1  Torque on the Wall surface on the liquid balances the weight of liquid in volume
represented by FABCDE.
The thrust of liquid on the wall produces a torque about the
\  Fy = weight of liquid in
base line AB (see the last figures). How can we calculate
volume FABCDE.
this torque?
Torque of force dF about AB is Force on the curved wall will
have two components as shown in
d t = (H – y) dF = (H – y) r gLy dy the figure. The resultant force is
_______
= r gL (Hy – y2) dy
  x2 + Fy2 
F = ​÷F  ​
Total torque due to forces on all strips is
H
Example 5   A bucket is in the shape
t = Ú d t = r gL ​Ú ​  ​ ​(Hy – y2) dy
0 of a frustum of a cone with radii of top
and bottom surfaces equal to R and r
Hy2
= rgL ​​ ____
​   ​ 
2
 – [  y3 H
__
] [  H3 H3
​   ​   ​​ ​ ​ = rgL ​ ___
3 0
​   ​ – ___
2 ]
​   ​   ​
3
respectively. Height of the bucket is h.
Water (density = r) is filled in the bucket
to the brim. Find force on slant surface of
1
= ​ __ ​  rgL H3 ...(6) the bucket due to water.
6
This equation is an important Solution
consideration in design of a dam Concepts
wall. The torque due to water 1
(i) Volume of the frustum is V = ​ __  ​ p h (R2 + r2 + Rr)
pressure tries to topple the wall 3
about A. Weight of the wall (W) (ii) The bucket wall experiences force due to water that
must produce a higher counter is normal everywhere. Obviously, the horizontal
torque about A; otherwise, the components add to zero. Vertical component is
wall will topple to the left, downward. From Newton’s third law, the bucket
about A. wall applies the same force on water in upward
direction.
4.3  Force on a Curved Surface (iii) By considering the equilibrium of water in the
Consider a curved surface ABC of a container wall, which bucket, we can find the force applied by bucket
contains a liquid. We wish to write force on this curved wall wall on the water body.
due to the liquid. It is easy to work out the horizontal and
Pressure at base is
vertical components of the
force, separately. P = r gh
For finding horizontal Force on base, F1 = p r2 ◊ P
component (Fx) of the
force, consider the = p r2 rgh
horizontal equilibrium
Same force is applied by the base of
of liquid in the volume
the bucket on the water body.
represented by ABCD.
F2 = Force by bucket wall on the
Force by curved surface on the
water body.
liquid has a horizontal component
(Fx) that is balanced by horizontal W = weight of water = volume × density × g
force on vertical wall CD due to 1
= __
​   ​  p h (R2 + r2 + Rr) r g
the surrounding liquid. 3
For equilibrium of water
( 
PC + PD
\  Fx = ​ ​ _______
2
 ​  )
 ​ (Area of face CD) F1 + F2 = W  fi  F2 = W – F1
1
Now, consider vertical equilibrium of the liquid volume fi F2 = __
​   ​  p h (R2 + r2 + Rr) r g – p r2 r gh
3
FABCDE (See first figure). Vertical force by the curved
1
= __
​   ​  p r gh [R2 + r2 + Rr – 3r2]
3
Fluid Mechanics  8.7

1 Force on face AM of the


= ​ __ ​  p r gh [R2 + Rr – 2r2]
3 considered volume due to
Example 6   A tray is of the shape shown in figure. It is liquid on left is equal to
completely filled with a liquid of density r. Find the force horizontal force by wall on
on its rectangular wall ABCD due to liquid. The wall is the liquid.
inclined at an angle q to the vertical. \ Fx = force on face
AM by liquid on left
= (Pav) (Area of face)

( 
= ​ ​ _______
PM + PA
2 )
 ​   (  )
0 + r gh
 ​ (hb) = ​ ​ _______
2
 ​  
 ​ (hb)

1
= __ ​   ​  r g h2 b
2
Vertical force by the wall on liquid = Weight of liquid in
volume denoted by D AMB
Solution
fi Fy = volume × density × g
Concepts
1
= __
​   ​  b (AM) ◊ (BM) r ◊ g
(i) Like all perivous questions, we can forget about 2
atmospheric pressure. Its effect is on both sides of 1 1
the wall and balances out. = __
​   ​  bh (h tan q) ◊ r g = __
​   ​  r gh2 b tan q
2 2
(ii) We will write the horizontal and vertical compo- Magnitude of resultant force on the wall is
nent of force separately. This can be done easily ________
by considering the equilibrium of liquid contained F = ÷
​F  x2 + Fy2 
 ​
in volume having its vertical cross-section denoted ________
1
by D AMB (see figure below.) = __
​   ​  r gh2b ​÷1  + tan2 q ​

2
1
We will consider equilibrium of liquid present in volume, = __
​   ​  r gh2b ◊ sec q
having D AMB as cross-section and b as length. 2

Your Turn

Q.5  A container has a square base of length a = 0.5 m. H = 12 m and water in the dam
Its height is h = 1.0 m. Half has a depth of h = 10 m. The
of it is filled with a liquid bricks used in construction
of density 2r and upper half of the wall have density of
contains another liquid of 3 gcm–3. Find the width a if
density r = 2000 kgm–3. it is required that weight of
(a) Find force on base of the dam wall be 10 times the
the container due to horizontal force on it due to water.
the liquids. Q.8  An earthen pot has the
(b) Plot a graph showing variation of pressure with height shape shown in figure. Its base
(y) from the base of the container. Ignore atmospheric surface as well as the mouth has
pressure. area 20 cm2. Height of the pot
is 20 cm. It is completely filled
Q.6  In the last problem, find the force experienced by a with half a litre water.
vertical wall of the tank due to the liquids.
(a) Find the increase in force
Q.7  Figure shows a dam wall whose length perpendicular on the bottom surface
to the plane of the figure is L = 30 m. Height of the wall is due to filling of the container with water.
(b) Find force applied by the pot wall on water.
8.8  Mechanics II

5.  BAROMETER If you record the height of mercury column as h = 76 cm


then the atmospheric pressure is
Barometer is a device used to measure atmospheric pressure.
P0 = (13.6 × 103 kgm–3) × (9.81 ms–2) × (0.76 m)
Figure shows a mercury barometer, which was invented by
Torricelli in 1640 s. A vertical = 1.013 × 105 Nm–2
glass tube, closed at the top,
stands on an open mercury- Note: Density of Hg is 13.6 × 103 kgm–3
filled basin. Lower end of the Actually, barometer is a kind of balance. Atmospheric
tube, which is open, is held pressure pushes the liquid surface down. Inside the tube, it
dipped inside mercury. The top is weight of Hg-column that creates same pressure at point 2.
of the tube has no air. With the If somehow the atmospheric pressure increases, the balance
aid of a scale, height (h) of the is destroyed. More liquid gets pushed into the tube so that
mercury in the tube above the height of Hg-column increases to balance the pressure.
liquid surface in the basin is Many a times, pressure is expressed in terms of height
recorded. This gives us the value of Hg-column. A pressure equal to h height of Hg-column
of atmospheric pressure. is same as P = rHg ◊ g ◊ h.
Pressure at point 1 is zero as there is no fluid present.
Pressure at point 2 is Example 7   Why not a water Barometer?
Mercury is poisonous and costly too. Why should not we
P2 = P1 + rHg ◊ g ◊ h = 0 + rHg ◊ g ◊ h = rHg ◊ g ◊ h construct a barometer using water?
Pressure at point 3 (on surface of mercury outside the tube) Solution  The primary reason is height of liquid column in
is equal to atmospheric pressure (P0). The basin is open to the barometer tube. If water is used, its height in the tube
air and pressure on surface of liquid in the basin is P0. is given by
P0
We known that pressure at all points on same horizontal P0 = rw g ◊ h  fi  h = ____
​    ​ 
rw g
plane are equal in a static liquid.
1.013 × 105
h = ​ __________
  
   ​ = 10.3 m
\ P3 = P2 103 × 9.81
This is almost equal to height of a three-storey building!
fi P0 = rHg ◊ g ◊ h ...(7) This will make the barometer impractical to use.

Your Turn

Q.9  Suppose we don’t have air all around us but Q.11  A barometer has a rigidly fixed tube that extends 1m
have mercury instead. How high up would that mercury above the mercury surface in the basin. The basin is wide.
atmosphere extend from ground if it were to make the same if the barometer is placed inside a freely falling lift. What
pressure that we feel? reading will it show?
Q.10  Assume that the atmospheric pressure is same at Q.12  A faulty barometer has some air trapped in the
equator and at poles. Where will the barometer show higher space in the tube above the Hg-column. It shows a reading
reading – at equator or at poles? of 72 cm on a day when atmospheric pressure is equal to
76  cm height of Hg. Find the pressure of the air in the
tube in Pa.

6.  Manometer
of the tube connected to a gas cylinder and the other end
It is a device used to measure pressure difference between left open to atmosphere. If the pressure of the gas is larger
two points. Here, we will discuss a manometer which is than the atmospheric pressure (P0), then the liquid in the
commonly known as U-tube manometer. U-tube will appear as shown in the figure–elevated on right
A U-tube manometer is simply a transparent U-shaped and depressed on left side of the tube.
tube filled with a liquid (usually mercury). Imagine one end
Fluid Mechanics  8.9 

Pressure at point 1 in the tube is same as pressure (P) of Solution


the gas in the cylinder and pressure at point 2 can be written Concepts
as
(i) The method of writing equation (8) can be used to
P2 = P0 + r gh get value of h.
(ii) We will take a point (1) in Hg in left column just
Where r is density of manometer liquid. below the Hg-surface and another point (2) in right
column, which is at same horizontal level as 1.
Pressure at point 1 = pressure at point 2.

But 1 and 2 are two points on same horizontal level in a


liquid.
\ P1 = P2

fi P = P0 + r gh P2 = P1

P0 + rHg ◊ g ◊ h = P0 + rw ◊ g ◊ (40.8 cm)
fi   P – P0 = r gh ...(8)
rHg
The above equation gives pressure difference between two fi ​ ___ ​  h = 40.8 cm
rw
ends of the manometer.
The left side of the above equation is the difference in fi 13.6 h = 40.8 cm
absolute pressure of the gas and the atmospheric pressure. fi h = 3 cm
This is sometimes known as guage pressure of the gas.
Note: Value of h is independent of values of cross-sectional
Pguage = Pabsolute – Patmospheric area of the two tubes. Even if the tubes on left and right have
unequal cross-sectional area, value of h remains same.
Example 8   A U-shaped tube contains mercury. Water
Alternate way of writing the above equation:
is poured in the left limb to a height of 40.8 cm. Now,
Start from top (point 3) of the left tube and move down
mercury column has different surface levels on two sides of
to point 1 and then to point 4. Keep adding r gh terms to
the tube. Find difference (h) in height of Hg-level on two
write pressure at point 4. Now move up from point 4 to 5.
sides. Relative density of Hg is 13.6.
subtract gh terms to get pressure at 5. Pressure at 3 and 5
both are equal to atmospheric pressure.

P3 + rw g (40.8cm) + rHg gH – rHg g(H + h) = P5


 
Pressure at 1
  
Presssure at 4
fi  P0 + rwg (40.8 cm) – rHg g h = P0
rw 40.8
fi  h = (40.8 cm) ___
​    ​ = ____
​   ​ = 3 cm
rHg 13.6
One may also chose to write pressure at 4 while moving
down from 3 to 4 and then while moving down from
5 to 4. Equate the two values. You will get same equation
as above.
8.10  Mechanics II

Your Turn

Q.13  In the arrangement shown in the figure, find the


relative density of the oil. What is guage pressure at oil-
water interface?

Q.15  Bulbs A and B contain gas. Bulb A is connected


to mercury barometer and B is connected to a water
barometer.

Q.14  Left arm of a U-tube is inclined at 30° to the


horizontal. It contains mercury. The right end of the tube
is closed and contains a gas. In equilibrium position, the
length of Hg column on left and right of the tube is 50 cm (a) In which bulb is the pressure of the gas higher?
and 20 cm, as shown. Find pressure of the gas in Pa. (b) Which bulb has negative guage pressure?
Atmospheric pressure is P0 = 1 × 105 Nm–2. Density of Hg (c) Find difference in pressure of A and B. Express your
is 13.6 × 103 kgm–3. g = 10 ms–2 answer in terms of height of Hg column.

7. PASCAL’S LAW Consider a jar filled with some water. The jar is open
at the top and pressure at the water surface is equal to
If you have a well inflated balloon and you squeeze it at atmospheric pressue P0. Pressure at a point A at depth h is
some point (A), it may burst at some other point (B). The
pressure inside the balloon cannot PA = P0 + r gh
be locally increased at A, leaving all Now, a lot of air in pumped inside the jar using an air
other points unaffected. Same pressure pump. The pressure at the water surface now has increased
change happens everywhere inside the and is equal to P0 + D P. Pressure at A now is.
balloon and it brusts where the wall is
weakest. PA¢ = (P0 + D P) + r gh
Pascal’s law states that a change Thus, pressure at A has increased by PA¢ – PA = D P. This
in pressure applied to an enclosed is independent of h. Thus, pressure must have increased by
imcompressible fluid is transmitted D P at all points in the water. Pressure change on the surface
undiminished to every point of the fluid has been transmitted throughout the water. This is a simple
and to the walls of the container. proof of Pascal’s law.
When you squeeze the lower end of the a toothpaste tube,
you increase the presence everywhere inside the tube. This 7.1  Hydraulic Lift
forces the toothpaste out of the tube. A mechanic at a car workshop uses hydraulic lift to raise a
car. A hydraulic lift is a forces-multiplying device that works
on Pascal’s principle.
Consider a U-shaped tank shown in figure. It is filled with
an oil. Left side of the tank has a small cross-sectional area
(A1) and is fitted with a tight-fitting smooth piston. Right
side is also fitted with a movable smooth piston. Area of
right limb is A2, which is much larger than A1.
Fluid Mechanics  8.11 

Due to various frictional losses, in practice, we will get


W2 < W1.
It is easily possible to design a hydraulic lift in which a
series of up and down strokes of the left piston can be made
one after another, each time raising the car a little.

Example 9   Hydraulic Brake


Figure shows a schematic diagram of a hydraulic braking
system in an automobile. Main cylinder is fitted with piston
P having area of A = 4 cm2. The wheel cylinder has two
pistons – P1 and P2 – each of area A1 = 36 cm2. When the
brake pedal is pushed, the lever system exerts a force of
F = 20 N on piston P. As a result, the pistons P1 and P2
The heavy object (such as a car) that is to be lifted is push the brake shoes S1 and S2, which in turn press against
placed on the right piston and the person (such as mechanic) the wheel. Due to friction between the brake shoes and the
applies downward force F1 on the piston of area A1. wheel, the rotation of the wheel is arrested. Find the force
with which each brake shoe presses the wheel.
F1
Increase in pressure at 1 is D P = ___
​   ​ 
A1
The same change in pressure occurs everywhere. Pressure
at 2 increases by D P. Thus, the right piston experiences an
extra upward force equal to


A2
(  )
F2 = D P ◊ A2 = F1 ​ ___
​    ​  ​
A1
...(9)

If F2 is equal to weight (w) of the car, the car begins to


rise.
A2
​ ___  ​ >> 1; hence, F2 >> F1
A1
A2
For example, if ​ ___   ​ = 100 then applying a force of
A1
F1 = 100 N will result in

(  )
A2 Solution
F2 = F1 ​ ___
​   ​   ​ = 100 × 100 = 10,000 N
A1 Concepts
F
Pressure in the fluid increases by __
​   ​
This is good enough to lift a 1,000 kg car! A
You must pay attention to the fact that there is no gain Change in pressure is same everywhere; pistons P1
in terms of work. If the force F1 pushes the piston down and P2 experience outward force due to this increase in
by x1 = 100 cm, the other piston rises up by only x2 = 1 cm pressure.
(volume of liquid in 100 cm length of left column = volume
of liquid in 1 cm length of the right column). Increase in liquid pressures when the brake pedal is
pressed is
Work done in pushing the left piston is W1 = F1x1 and the
F 20 N
work output that we get in form of the right piston raising D P = ​ __ ​ = _____
​    ​ 
A 4 cm2

(  )
x1
our car is W2 = F2 x2 = (100 F1) ​ ____
​    ​  ​ = F1 x1 = W1
100
Force that acts on each of S1 and S2 is

Therefore, work input = work output.


(  )
​ 
20 N
F1 = D P ◊ A1 = ​ _____
4 cm2
  ​  ​ (36 cm2) = 180 N
8.12  Mechanics II

Your Turn

Q.16  The areas of cross-section of two arms of the Q.17  In the hydraulic lift shown, cross-sectional areas of
U-tube shown in figure are A1 = 2 cm2 and A2 = 10 cm2. the two pistons are A1 = 10 cm2, A2 = 100 cm2
A force F1 = 10 N is applied on the piston of area A1 as
shown. The mass of the piston itself is 0.5 kg. The piston
on the other side has a mass of 2 kg. How much force F2
must be applied on this piston of area A2 so that the entire
system remains in equilibrium?

The load has a mass of 100 kg and both pistons are


massless. When pistons try to move, a friction force of 20 N
acts on each of them. Find the forec F1 with which the left
piston shall be pushed down so as to make the load just
move up.

8.  FLUID IN ACCELERATed CONTAINER


Can you keep water in a glass as shown in figure (A)? The
answer is no, if the glass is at rest. But it is possible if the
glass is accelerated.

PB – PA = r geff h = r (g + a) h ...(10)


If the container is having a vertically downward
If water surface, in a glass at rest, were inclined (as in acceleration of a, then geff = (g – a) and
figure A), then a water particle on the surface will experience
a tangential force (mg sin q), which will cause it to slide. PB – PA = r (g – a) h ...(11)
[Remember, there is no viscous force (i.e., friction) between Recall that, no observer can differentiate between the
layers of an ideal fluid.] The water surface will ultimately effect of gravity and acceleration.
become horizontal, as shown in figure B. Now, a particle on
To re-emphasise the above point, consider a thought
the surface experiences no tangential force. We know that in
experiment. A man is riding a space rocket, which is moving
this position PB – PA = rgh.
with acceleration a. There is a bucket tied to its tail. Bucket
Imagine a glass of water kept inside an elevator that is contains water. The orientation of the bucket and the water
accelerating up with acceleration a. Any observer inside surface will be as shown in the figure. If the man releases a
the elevator will experience effective value of acceleration ball, it falls towards the tail of the rocket with acceleration
of free fall to be geff = (g + a). The water surface remains a. Everything in this environment behaves as if there is
horizontal (normal to geff) and pressure difference between gravity towards left. The water surface also orients itself
A and B is given by
Fluid Mechanics  8.13 

perpendicular to the direction of effective gravity. Note


that
PB – PA = r ga ...(12)
Now, consider an open, water-filled container travelling
horizontally with acceleration a. An observer attached to this
container is in an environment where effective acceleration
of free fall is Consider a plane (such as 1, 2, 3, 4 ...) in liquid that is
______ normal to geff. Pressure at all points on any such plane is
geff = ​÷g  2 + a2 
 ​ ...(13) same.
Making an angle q with vertical, where Consider two points A and B inside the liquid, as shown
a in figure. Pressure difference between the two points is
tan q = ​ __
g ​ PB – PA = r geff ◊ h ...(15)

= r (g sec q) . h ​ [  ___g
]
​ g   ​ = cos q  ​
eff

= r (g sec q) (y sec q)
[y = vertical separation between A and B]
= r g y sec2 q = r gy (1 + tan2 q)
= r gy + r (g tan q) (y tan q)

\ PB – PA = r gy + r ◊ a ◊ x ...(16)

[ 

a
__ x
​ g ​ = tan q and __ ]
​ y ​ = tan q  ​

The last equation gives us a simple method of writing


pressure difference between any two points in a fluid, which
If the observer attached to the container releases a ball, he is in an accelerated container.
finds it to move in the direction of geff shown in the figure.
The liquid also feels geff in the shown direction and orients
its surface perpendicular to it. Therefore, the liquid surface
gets inclined to the horizontal at an angle

a
q = tan–1 ​ __ (  )
​ g ​  ​ ...(14)

In this position, a liquid particle on the surface will


experience no tangential force. The particle will not flow.
It will remain static, relative to the container.

8.1 Variation of Pressure in a Liquid in Horizontally


Accelerated Container To write pressure difference between point B and A, move
vertically from A to C and write
Fluid in the container is in an environment where
______
geff = ​÷ g2 + a 
2
 ​ and direction of geff makes an angle PC – PA = r gy ...(17)

(  )
a
q = tan–1 ​ __
​ g ​  ​ with vertical. Now move horizontally from C to B and write
8.14  Mechanics II

PB – PC = rax ...(18) F = FA – FB

= PA D S – PB D S
fi  PB – (PA + rgy) = rax  fi  PB – PA = rgy + rax
Using F = ma
Remember that pressure increases when you move
horizontally opposite to a. PA D S – PB D S = (r L D S) a

fi PA – PB = r L a ...(i)
Example 10   Alternative way to get equation 14
A liquid of density r is being transported in a tanker that This is nothing but equation 18.
is moving horizontally with acceleration a. Consider a (b) Look into the figure. We can write
horizontal cylindrical element of the liquid inside the tanker,
as shown in the figure. Cross-sectional area of the cylinder PA = P0 + r gh1 and PB = P0 + r gh2
is small.
\ PA – PB = r g (h1 – h2)
(a) Write pressure difference between the ends of the
cylinder by considering its accelerated motion in
From (i) rg (h1 – h2) = r L a
horizontal direction.
(b) Using the above result, find the angle q that the liquid h1 – h2 __ a a
surface must make with horizontal. fi ​ ______   ​ g ​  fi  tan q = __
   
​ = ​ g ​
L

Example 11   A trolley containing a liquid slides down a


smooth incline of inclination angle q. Find the angle a that
the free surface of the liquid makes with horizontal

Solution
Concepts
(i) The cylindrical element of the liquid can accelerate
only if there is a net horizontal force on it. This
implies that pressure at the left end is higher than Solution
pressure at the right end. Concepts
(ii) Net horizontal force on cylinder due to pressure A particle on the surface of the liquid must not have any
difference at two ends = (mass) × (acceleration) tangential force acting on it in the reference frame of the
(iii) The pressure at left end is high because the depth trolley.
from free surface of liquid is more. The liquid
surface is slant, creating more depth at left end. The trolley has an acceleration
a = g sin q down the incline.
(a) Let cross-sectional area of the liquid cylinder be In RF attached to the trolley,
D S and its length be L. Horizontal force on the forces acting on a particle on liquid
cylinder is surface are as shown in the figure.
Tangential force will be zero, if
mg sin a = ma

fi g sin a = g sin q  fi  a = q

Example 12   Rotating tank


A tank is filled completely with a liquid of density r. It is
spinning about a vertical axis with angular speed w. Find
Fluid Mechanics  8.15 

difference in pressure at a point O on the axis and a point, A F = FA – F0


located at a distance x from the axis. 1
fi ​ __ ​  r D S w2 x2 = PA ◊ D S – P0 ◊ D S
2
1
fi PA – P0 = __​   ​  r w2 x2 ...(19)
2
Example 13   Water in spinning bucket

Water is kept in a bucket,


which is spinning about
its central vertical axis
with angular speed w. The
surface of the liquid forms a
depression as shown. Take the
lowest point of depression as
Solution origin (O) and prove that the
Concepts vertical cross-section of the
surface of water is a parabola.
(i) Consider a horizontal cylinder of the fluid. The
Find its equation.
centripetal force for its rotation is generated by the
pressure difference at its two ends. Solution
(ii) If a cylindrical rod of mass M and length L is rotat- Concepts
ing about an axis as shown, then centripetal force
on it can be written assuming the entire mass of We will solve this problem in two different ways. In the
the rod to be at its COM. first method, we will use the result obtained in the last
example.
In the second method, we will once again use the fact
that a water particle on the surface must not experience
a tangential force in the reference frame attached to the
bucket.
L
F = Mw2 ​ __ ​ 
2 Point A on the surface has
co-ordinates (x, y).
From result in the last
problem, we have
1
PB – P0 = __​   ​  r w2 x2
2
But PB = PA + rgy
1
\  PA + rgy – P0 = __ ​   ​  r w2 x2
2
1
fi r gy = __
​   ​  r w2 x2
2
Consider a horizontal cylinder of liquid of length x and [   PA = P0 = atmospheric pressure]

(  )
small cross-sectional area D S. Centripetal force needed for w2
x fi y = ​ ___
​   ​   ​ x2
its rotation is F = D mw2 ​ __  ​ 2g
2
Where D m = mass of cylindrical element This is an equation of a parabola. If we rotate this parabola
about y-axis, we get the three-dimensional surface of water.
= r ◊ D S ◊ x The shape of the surface (obtained by rotating a parabola)
1 is known as paraboloid.
\ F = ​ __ ​  r D S w2 x2
2 Alternate
This force is provided by the surrounding fluid. Pressure Consider a particle of mass m on the surface of the liquid
at A is larger than pressure at O and at point A. This particle is rotating in a circle of radius x.
m w2 x is the centrifugal force on this particle in RF of the
bucket.
8.16  Mechanics II

dy
\ ​ ___  ​ = tan q
dx
For no tangential force on the particle,
w2
mw2 x cos q = mg sin q  fi  tan q = ___
​  g ​  x

dy w2
\ ​ ___  ​ = ___
​  g ​  x
dx
x x
w2
fi ​Ú   ​ ​  dy = ​Ú   ​ ​   ___
​​  g ​  x dx
Line l is tangent to the surface at A and makes an angle y = 0 x = 0
q with x-axis. Obviously, tan q is the slope of the curved
w2
surface at point A. fi y = ___
​    ​ ◊ x2
2g

Your Turn
Q.18  A closed container is Q.20  A tube AB of length
completely filled with water. It L = 0.1 m is rotating about
is accelerated to the right with its end A at an angular speed
an acceleration of a = 4ms–2. w = 20 rad s –1. The tube is
A, B, C and D are points in the horizontal and is completely filled
liquid at the four corners, as with water. End A of the tube is
shown. Find [take g =10ms–2] open to atmosphere. Find pressure at end B. Atmospheric
(a) PB – PA  (b) PD – PA  (c) PA – PC. pressure is P0 = 1 × 105 Pa.
Q.19  What is pressure difference between two points, A Q.21  An L-shaped tube contains a liquid of density
and B, inside a freely falling water-filled container? r. It is moved horizontally with
g
acceleration a = __
​    ​. Find pressure
2
at End 1 of the tube. End 2 is
open and atmospheric pressure is
P0. Express L in terms of H, if
pressure at point 1 is equal to P0.

In Short: (v) A real-life container having a gas has same pressure


(i) A fluid exerts force on any surface, which comes into at all points. The
contact with it. The force is normal to the surface. term rgh becomes
This force per unit area is called pressure. Its SI unit significant for gases
is Nm–2 (also known as Pascal). when h is too large.
(ii) Any small volume of fluid experiences force from all (vi) Force on base of
sides, normal to its surface, due to surrounding fluid. container A due to
Force per unit area is pressure. liquid is more than weight of the liquid in the con-
(iii) In a static fluid, pressure is same at all points on a tainer. This is due to force applied by slant wall on
horizontal plane. the liquid. In container B, force on base is less than
(iv) In a static fluid, pressure increases with depth. If point weight of the liquid in the container.
B is at a depth h below A, then PB = PA + r gh (vii) Force on vertical wall due
to the liquid is

F = (Pav) (Area of wall in


contact with liquid)
Fluid Mechanics  8.17 

(  )
1
Ptop + Pbottom PA – P0 = __
​   ​  r w2 x
= ​ ​ ___________
 ​ 
     ​ (Area) 2
2
(xv) Tangential force on a liquid particle on its surface
(viii) If height of Hg-column in a barometer is h, then must be zero if the liquid is static, relative to the
atmospheric pressure is P0 = rHg g h container.
(ix) In the U-tube shown in figure
P3 π P4
9.  BUOYANCY
Points 3 and 4 are at same horizontal level but they
are not in the same liquid. When a body is dipped, partially or completely, into a
fluid, the fluid exerts force
on the body. On any small
surface area of the body, the
force due to the surrounding
fluid is normal to the surface.
The resultant of these forces
acting on all small patches
on the surface of the body is
known as force of buoyancy
or buoyant force. The force
arises because the pressure is
It is correct to write
not same everywhere in the fluid.
P1 = P2 We all have experienced this force in our life. Any
(x) Any change in pressure at a point in an enclosed fluid floating object has its weight balanced by buoyancy. A
is transmitted throughout the fluid. This is known as bucket inside water in a well feels lighter due to buoyancy. It
Pascal’s law. Using this, we can make a hydraulic was Archimedes who first quantified the buoyancy force.
lift, which is a force-multiplying device. Archimedes’ principle states that when a body is partially
(xi) If a liquid is kept in a vertically accelerated container, or completely immersed in a fluid at rest, the fluid exerts an
pressure difference between two points (B at depth h upward force of buoyancy that is equal to the weight of the
below A) is displaced fluid.
The principle is valid for a body of any shape. How could
PB – PA = r geff h
Archimedes conclude this? This is a bit tricky. Consider the
where  geff = g + a for a container having upward above diagram of a solid immersed in a fluid. Surrounding
acceleration and geff = g – a for a container having liquid molecules strike its surface from all sides, producing a
downward acceleration.  resultant force on the body. Now, suppose that we replace the
(xii) In a liquid kept in a solid body with the same fluid, occupying the volume occupied
horizontally accelerated by the solid. Nothing has changed for the surrounding liquid
container particles. They keep on hitting the boundary in the same
a way as they were doing with the boundary of the solid.
tan q = ​ __
g ​ Therefore, the
PB – PA = rgy and surrounding fluid
exerts a force on
PC – PB = r ax the solid that is
(xiii) If the container shown in the above figure is having same as the force
an upward acceleration (a0) apart from horizontal it exerts on the
acceleration (a), then replaced fluid. It
is very easy to
a
tan q = ______
​       ​ write the force
g + a0
on the replaced
fluid. As the
PB – PA = r (g+ a0) y
entire fluid now becomes homogenous, all parts will remain
in equilibrium.
and PC – PB = rax
The volume of the fluid that has been used to replace the
(xiv) For a tank filled with liquid and rotating about a solid also remains in equilibrium. This volume of fluid can
vertical axis
8.18  Mechanics II

stay in equilibrium only if surrounding liquid exerts a force The same holds true for a balloon in air. A balloon that
on it that balances its weight. Therefore, force applied by weighs 1 ton displaces 1 ton of air, if it is hovering at a
surrounding fluid is equal to weight of the volume of fluid constant altitude. If it displaces more, it rises; if it displaces
under consideration and is in vertically upward direction. less, it falls.
Force on submerged solid is equal to this force.
Buoyancy force on a solid dipped partially or completely 9.2  Apparent Weight
in a fluid can be written as When you suspend a solid block from a spring balance, it
FB = Weight of displaced fluid shows true weight of the body. The spring balance measures
= (volume of displaced fluid) × tension in the spring, which is equal to the weight of the
body.
(density of fluid) × g
= (volume of submerged part of solid) ×
(density of fluid) × g

or, FB = V rf g ...(20)


Where rf is density of fluid and V is the volume of
submerged part of the solid.

9.1  Floatation
A body is said to be floating when its
weight is balanced by the buoyancy
force. If you put a block of wood in
water, it does not sink. A part of it
gets submerged and displaces water.
It stays in equilibrium in a position FS = W
where the weight of displaced water If the body is submerged in a liquid, reading of the balance
(= FB) balances its weight (W). decreases. This is due to buoyancy acting on the solid. Now,
the tension in spring (= reading of spring balance) is
FB = W
FS¢ = W – FB
fi Vsub rL g = V rs g
Thus, Apparent weight = True weight – Buoyant force
Vsub rS
fi ​ ____
   
​ = ​ ___  ​ ...(21) ...(22)
V rL
9.3  Hydrometer
Where Vsub is the volume of submerged part of the solid
and V is the volume of the solid. rS and rL are densities of A hydrometer is a device used to measure specific gravities
the solid and the liquid respectively. of liquids. It has a glass bulb attached to a stem. The bulb is
loaded with mercury or lead shots to make it float upright.
If rS = rL, then buoyancy can balance the weight of the
[You will understand this after going through Section 9.5].
solid only if it is completely submerged. If we place a body of
relative density 1.0 inside water, it will remain at the location
where it is released. It neither sinks nor comes up.

When placed in water, it floats with a total volume V0


submerged. Weight of the entire hydrometer is equal to
If rS > rL, then weight of the solid is always greater than buoyancy.
buoyancy and the solid sinks.
\ V0 rw g = W ...(i)
Fluid Mechanics  8.19 

A mark is made on the stem where it intersects the surface From equation (21), one can see that fraction of volume
of water and 1.0 is written at the mark. Now, the hydrometer of the solid submerged in the liquid is independent of g.
is placed in a different liquid. Figure shows the hydrometer Therefore, in all the cases in the above diagram, the fraction
in a liquid of specific gravity larger than 1.0. To balance the of the volume of the solid that is submerged is same.
weight, a lesser volume of liquid needs to be displaced. The
hydrometer moves up by D h. We can write
Example 14  Ice Block in a beaker

(V0 – S D h) rl g = W ...(ii) An ice block (relative density = 0.9)


floats in water in a beaker.
Where S = area of cross-section of the stem (a) Find the percentage of the
volume of ice block that is
Dividing (ii) by (i) submerged.

( 
(b) How will the level of water in the beaker change if
V0 – S D h ___
​ ​ ________
V0
 ​  

rl
)
​ ​    ​  = 1
rw
Solution
the ice block melts?

rl V0
fi  Specific gravity of liquid = ___
​    ​ = ________
​      ​ Concepts
rw V0 – S D h
��������������������������������������������������
(i) Net force on ice blocks is zero. Buoyancy bal-
Using the above equation, marking can be done on the ances its weight. Using this gives equation (21).
stem to directly give the reading of specific gravity when Vsub
the hydrometer is placed in a liquid. Percentage of volume submerged is ​ ____    ​ × 100
V
(ii) To answer part (b), we must figure out how much
9.4  Buoyancy in fluid kept in Accelerated Container
volume of water will be produced by melting of
Article 8 explains the variation of pressure inside a fluid that the ice.
is kept in an accelerated container. Buoyancy force arises due The key is to realise that mass will remain same
to liquid pressure only. Therefore, it is easy to understand even if the ice melts. Since density of water >
that buoyancy is given by density of ice, the volume of water produced due
to melting of ice is less than the volume of ice.
FB = Vsub rL geff ...(23)

Where geff is the effective gravity in the environment of (a) From equation (21)
the fluid (i.e., in RF attached to the container). The figures
given below are self-explanatory. Vsub r ice
​ ____
  ​   = ____
​     ​ = 0.9
V r  w

Vsub
​ ____
   ​ × 100 = 90%
V
90% of the volume of the
ice block remains submerged
inside water. 10% of its vol-
ume is outside water.
(b) Weight of ice block = Weight
of displaced water
fi  Mass of ice block = Mass of displaced water
fi  Mass of ice block = Mass of (Vsub) volume of
water
Where Vsub = volume shown in dark shade in
figure.
8.20  Mechanics II

Thus, when the ice melts, it froms water which has Solution
volume equal to Vsub. The produced water exactly fills the Concepts
space shown in dark shade. The level of water does not
change. Initially, buoyancy = Weight of the cube
Finally, buoyancy = Weight of (the cube + Added
Example 15   A steel boat mass)
Steel is eight times heavier than water. Why doesn’t a boat
made of steel sinks? Let side length of the cube be x and its weight be W.
Area of its face = x2
Solution
Initially, FB1 = W
Concepts fi Vsub rw g = W ...(i)
The boat is made wide enough to displace water equal to Finally, FB2 = W + mg  [m = 200 g]
its weight before it sinks too deep in water.
fi V rw g = W + mg ...(ii)
Focus on understanding the meaning of “displaced
water”. [Submerged volume = total volume of cube]
(ii) – (i) gives
To balance the weight of V volume of steel, we need 8V
volume of water. A solid iron block, if left in water, can (V – Vsub) rw g = mg
only displace V volume of water and will definitely sink.
fi x2 (2 cm) rw = 200 
[V – Vsub = x2 × 2; see first figure]
1 g
fi  x2 (2 cm) ____
​  3  ​ = 200
cm
fi x = 10 cm

The same quantity of iron, if given the shape of a boat, Example 17   A solid at the interface of oil and water
can displace a lot of water. In the figure shown, (abcd) A solid cube has a side length of 10 cm. It floats at the
represents the volume of water displaced. Buoyancy can interface of water and oil, as shown in figure. Its 2 cm height
easily balance the weight. is inside water and remaining 8 cm is in oil. The layer of oil
is 10 cm thick. Relative density of the oil is 0.8.

Example 16   A cube of wood is floating in water with its


2 cm height above water surface. A 200 g mass is placed on (a) Find relative density of the solid.
the top surface of the cube and it is now observed to float
(b) Find pressure exerted by water on the lower surface
while remaining completely submerged (see figure). Find the
of the cube. Atmospheric pressure is
side length of the cube.
P0 = 1 × 105 Nm–2
g = 10 ms–2
Solution
Concepts
Archimedes’ principle works just fine in such situations.
Buoyancy = Weight of displaced oil + Weight of displaced
water.
Here, we will give a simple proof for this, though one can
use the result directly.
Fluid Mechanics  8.21 

Let A = area of cross-section of the solid. Vdg = 210 g ...(i)


Buoyancy is Loss of weight in water = Buoyancy in water
FB = F2 – F1 fi 30 g = Vrw g ...(ii)
= P2A – P1A Vdg 210
From (i) and (ii), _____
​  ​ = ____
    ​   ​ 
Vrw g 30
d
fi ​ ___  ​  = 7  fi  d = 7 gcm–3.
rw

When dipped in liquid


Buoyancy = loss in weight

VrL g = 90 g ...(iii)

VrLg ____90 g
where P2 is pressure at lower face EF and P1 is pressure at From (iii) and (ii), _____
​   ​ = ​   
   ​
Vrwg 30 g
top surface AD.
rL
\ FB = (P2 – P1) A = (r1gh1 + r2gh2) A fi ​ ___  ​ = 3  fi  rL = 3rw = 3 gcm–3
rw
= (r1 h1 A) g + (r2 h2 A) g
= weight of volume ABCD of oil + Weight of Example 19   A solid block has density (d), which is less
  volume of water equal to BEFC than the density of water (r). It is
kept submerged in water with the
(a) In the present question help of a string, as shown in the
Weight of cube = FB figure. Tension in the string is T0.
fi  (103 cm3) d ◊ g = (10 × 10 × 8 cm3) roil g (a) Find weight of the block in
terms of T0.
+ (10 × 10 × 2 cm3) rw ◊ g
(b) The container is given an upward acceleration a. Find
d 8 roil ___ 2 the tension in the string in terms of T0.
fi  ​ ___  ​  = ​ ___  ​ ___
​   ​ + ​    ​ 
rw 10 rw 10
Solution
fi  R ◊ d of solid = 0.8 × R ◊ d of oil + 0.2
Concepts
= 0.8 × 0.8 + 0.2 = 0.84
(i) When the solid is fully submerged, buoyancy
(b) Absolute pressure at lower surface of the cube is acting on it (FB = Vrg) is greater than its weight
P = P0 + roil g (10 cm) + rw g (2 cm) (W = Vdg).
= 1 × 105 + (0.8 × 103) (10) (0.1) The tension in the string (a downward force) is
+ (103) (10) (0.02) necessary for equilibrium.
(ii) When the container accelerates, geff = g + a.
= 1 × 105 + 0.8 × 103 + 0.2 × 103
Both the weight of the block (= M geff) and buoy-
= 1.01 × 105 Nm–2 ancy ( = Vrgeff) change.

Example 18   A metallic ball weighs 210 g in air, 180 g in (a) For equilibrium of the block:
water and 120 g in an unidentified liquid. Find the density W + T0 = FB
of the metal and of liquid.
M Wr
Solution But FB = Vrg = __​   ​  rg = ___
​     
​ 
d d
Concepts
From equation (22), loss in weight = Buoyancy force
r
\  W + T0 = W ​ __ ​  fi 
d [  ]
r
W ​ ​ __ ​ –1  ​ = T0
d
T0
fi W = _____
​       ​
If V is volume of ball and d is its density then r
__
​   ​ – 1
d
8.22  Mechanics II

(b) Initially, W + T0 = FB Solution


fi  Mg + T0 = Vrg Concepts
(i) We can find acceleration of the container using
fi  T0 = Vrg – Mg ...(i) a
equation (14). tan 45° = __
​ g ​
When the container is accelerated
(ii) In RF attached to the container, everything is in an
W¢ + T = F ¢B ______
environment where geff = ÷ ​ a  + g2 
2
 ​. geff makes an
fi  M geff + T = Vr geff
(  )
a
angle a = tan–1 ​ __
​ g ​  ​ with vertical.
fi T = (V r – M) geff ...(ii) (iii) Both the effective weight and buoyancy act along
the line of geff. String tension must also be along
From (i) and (ii), this line for equilibrium.
T ___ geff g + a
​ __  ​ =
  ​  g   ​ = ​ _____
g ​   a
(a) ​ __
T0 g ​ = tan 45°  fi  a = g

( 
Effective free fall

a
T = T0 ​ 1 + __
​ g ​ ) acceleration will be

______ __
r geff = ​÷a  2 + g2 
 ​ = ÷    g
Example 20   A solid of density ​ __ ​  (r = density of water) ​ 2 ​
2
and mass m is held inside a water tank, tied to a string, as
It makes an angle a with
shown in the figure. The tank is accelerated uniformly in
vertical
horizontal direction such that the water surface makes an
angle of 45°with the horizontal. a
tan a = __
​ g ​ = 1

fi  a = 45°
Buoyancy will act in direction opposite to geff.
String will align itself along geff as shown.
\  q = 45°
(b) T = FB – M geff
__ __
    g) – M (​÷2 ​
= Vr geff – M geff = Vr (​÷2 ​     g)
(a) Find the angle q that the string makes with the
base of the container in equilibrium. r
[  ]
__ __
    Mg ​   M = V __
    Mg – ​÷2 ​
= 2​÷2 ​ ​   ​  g  ​
(b) Find the tension in the string. __
2
= ÷     Mg
​ 2 ​

Your Turn

Q.22  A metal ball weighs 0.1 N. When suspended in Q.25  A log of wood (density = 6 gm–3) of mass 120 kg
water, it has an apparent weight of 0.07 N. Find the density floats in water. How much weight can be placed on the log
of the ball. to make it just sink?
Q.23  A block has density r1. When placed in an Q.26  A jewellery piece weighs 50 g in air and 46 g when
unidentified liquid, it floats with its three-fourth volume dipped in water. Assuming that copper (r ◊ d = 10) is mixed
submerged. Find the density of the liquid. with gold (r ◊ d = 20), find the mass of copper in the jewellery
piece.
Q.24  A block of wood is held deep inside water. Its
acceleration immediately after release is 2 ms–2 upwards. Q.27  A balloon and its contents have a total mass of 800
Find the relative density of the wood [g = 10 ms–2]. kg. It is floating in air and is in equilibrium. Its mass is
Fluid Mechanics  8.23

reduced by D m without changing its volume. Its acceleration Q.31  A solid ball of density half of that of water falls
is observed to be 2 ms–2. Find D m. [g = 10 ms–2] freely under gravity from a height of 19.6 m above the
Q.28  A block of ice has a piece of gold frozen inside it. surface of water in a lake. Upto what depth will the ball go
The ice block is floating in water in a beaker. How will the inside the lake? Neglect viscosity (i.e., friction due to water)
level of water in the beaker change if the ice melts? and take g = 9.8 ms–2.

Q.29  A glass tube (open at top) Q.32  An iron ball has cavity. The
has outer diameter of 1.6 cm. It ball weighs 6000 N in air and 4000 N in
floats in water in a large tank, as water. Find the volume of cavity inside
shown. Some sand is poured into the ball.
the tube so as to sink it further Density of iron, d = 7.87 gcm–3
by 2 cm. Calculate the mass of Density of water, r = 1 gcm–3 and g = 9.8 ms–2
sand added.
Q.33  A solid block is held by a spring inside a water tub,
Q.30  An open top glass tube as shown in the figure. The spring is compressed by 3 cm
is floating in water in a beaker, as shown. Some water from in equilibrium. The container is now moved up with an
the beaker is taken and poured into the tube. Will the water acceleration a = 2 ms–2.
level in the beaker change?

Find the compression in the spring in equilibrium. Take


g = 10 ms–2

9.5  Centre of Buoyancy the ship tilts a little towards right, the centre of buoyancy
moves to right
Consider a volume of liquid inside a tank full of liquid. W is
at COM of the
the weight of this volume of liquid. It is balanced by force
displaced water.
applied by the surrounding liquid
This happens as
on considered volume. This force
displaced water
(F) due to surrounding liquid
has higher
must be considered acting at the
volume to the
COM of the considered volume.
right of vertical
This will ensure rotational
line through G. FB and W form an anti-clockwise couple
equilibrium. If line of F and W
that restores the position of the ship. The ship is definitely
are separate, it will produce a
in stable equilibrium.
torque, implying that the considered volume is rotating.
If a solid is used to replace the volume of liquid shown Example 21   A ship has it centre of buoyancy below its
in the diagram, it will also be acted upon by the surrounding COM. Can it be in equilibrium for small tilts?
liquid in same fashion. Net force on the solid will be
F (= W = weight of displaced liquid) and it will be assumed Solution
to be acting at the COM of the displaced liquid volume. Concepts
Therefore, the buoyancy force acting on a solid submerged It can be in equilibrium if the couple formed by buoyancy
in a fluid must be considered to be acting at the COM of the and weight is restoring.
displaced fluid. This point is called centre of buoyancy.
Consider a ship having its COM at G. The water displaced If a small shift causes the centre of buoyancy to shift a
by it has its COM at B. Thus, B is centre of buoyancy. Weight lot (as shown in figure (b)), a restoring couple is developed.
of the ship acts at G and buoyancy force on it acts at B. If Ship will be stable.
8.24  Mechanics II

h 1 5
AC = ______
​     ​  = __ ​    ​ = __
​   ​  m
sin 37° __ 3 3
​   ​ 
5
Buoyancy acts at mid-point (M) of the submerged
part AC.
AC 5
AM = ___
​   ​  = __​   ​  m
2 6
Weight acts at mid-point (G) of AB.
L
AG = __
​   ​  = 1 m
2

If the tilt causes a situation as shown in figure(C), then


the couple formed due to FB and W will further tilt the ship
to right. This will drown it.

Example 22   Stick in water

(  )
A stick (AB) is L = 2 m long. It is hinged at its lower
5
end A inside a water tank. End A is h = 1.0 m below the Buoyancy force, FB = S (AC) rw g = S ​ __
​    ​  ​ rw ◊ g
3
surface of water. In equilibrium, the stick makes q = 37°
[S = cross-sectional area of the stick]
with horizontal. Find the relative density of the stick. Take
3 Weight, W = S L r ◊ g = S (2) r ◊ g
tan 37° = __
​   ​ 
4
For equilibrium, torque due to FB (about A) must be equal
Solution to torque due to W.
Concepts
\ tW = tB
(i) Buoyancy force acts at the centre of mass of W ◊ (AG cos q) = FB (AM cos q)
displaced water, i.e., at the centre of part AC of
the stick.
(ii) Torque of buoyancy balances the torque due to

5
S (2) rg ◊ (1) = S ​ __ (  ) (  ) 5
​   ​   ​ rw g ​ __
3
​    ​  ​
6
r 25
weight. fi ​ ___  ​  = ___ ​   ​ = 0.69
rw 36
\  Relative density = 0.69

Your Turn

Q.34  A non-uniform sphere floats in a liquid while Q.35  A uniform stick of mass m
remaining completely submerged. It has a particle of negligible volume
is in a position where its centre of attached to its end. It floats in a
mass (G) lies above its geometrical liquid, as shown, with half its length
centre (O), as shown in figure. submerged. Find mass of the particle if
What will happen if the sphere is the centre of buoyancy coincides with
disturbed a little? the centre of mass of the system.
Fluid Mechanics  8.25 

In Short: 10.1.1  Streamline


(i) When a solid is dipped in a fluid, the resultant force Path taken by a fluid particle is known as its line of flow. In
acting on it due to fluid pressure from all sides is case of a steady flow, a line of flow is known as a streamline.
known as buoyancy. In this case, all particles passing through a point follow the
(ii) Archimedes’ principle states that buoyancy is upward. same path and hence we have a unique line of flow passing
It is equal to weight of displaced fluid. through a point.
FB = Vsub rf g A tangent draw on a stream line at a point gives the
direction of motion at that point. Obviously, only one
Where Vsub = volume submerged part of the solid streamline can pass through a point. Two streamlines can
and rf is density of fluid. never intersect. Intersection of two streamlines will imply
(iii) In an accelerated container, we should take geff in two different directions of motion at a point.
place of g for writing buoyancy.
(iv) When a solid is dipped in a fluid, it appears to be
lighter.
Wapparent = Wtrue – FB
(v) A solid having density (rS) less than that of liquid
(rl) floats with a part of it remaining outside the
liquid. The fraction of volume that is submerged is
Vsub __ rS
​ ____   
  ​   ​ 
​ =
V rl
Consider water flowing steadily through a pipe. S
(vi) The buoyancy force acts at a point that is centre of represents an area in the cross-section of the pipe. Many
mass of the displaced volume of the fluid. This point streamlines are passing through this area S. These streamlines
is known as centre of buoyancy. form a tube. such a tube is often termed as a tube of flow.
The figure shows three tubes of flow in a water pipe having
cross-sections S, A and B. There is no physical wall between
10.  FLOW OF FLUIDS two tubes of flow but the liquid in tube A will not intermix
Study of a flowing fluid is far more complex than study with liquid in tube S. This is because two streamlines never
of a static fluid. However, we will make many idealistic intersect. The entire pipe itself is a tube of flow.
assumptions to simplify our analysis of a flowing fluid.

10.1 Steady and Turbulent Flow


A flow is said to be steady (or streamline flow) if flow
conditions do not change with time. Imagine a gentle flow
of water through a pipe. If you observe the fluid particles
passing through a specific
point A, you will find them
all to have same velocity
(say v1) when they reach A. Streamline diagrams are often used to represent the nature
Velocity at A does not change of flow. One more important feature of such diagrams is
with time. Similarly, all particles passing through another that density of the lines give an idea of flow speed. Higher
point B have a common velocity (say, v2) at B. If a particle density of lines means higher speed. In the streamline
passing through point A, later passes through B, then all diagram shown in the figure,
particles which pass through A will also pass through B. It speed of flow at A is higher
means that all particles passing through A follow the same than speed of flow at B.
path. Such a flow is called steady flow.
By injecting some dye in
Motion of water in a fall or in a fast-flowing river is a fluid, we can easily see the
not that simple. Velocities of particles passing through the streamlines. Path followed
same point may be different. In fact, the flow condition at by dye particles are the
a point changes erratically with time. Such a flow is called streamlines.
turbulent flow. In general, liquid flowing at high speeds will
be in turbulent flow.
8.26  Mechanics II

10.2  Irrotational flow D m2 = r2 A2v2 D t [r2 = density at A2]


Irrotational flow means there is no spinning of fluid particles. But mass of liquid in the pipe between two sections at
Put a tiny piece of paper in a flowing fluid. If the paper does A1 and A2 cannot change.
not rotate about its centre of mass, the flow is irrotational.
In practice, flow of viscous liquids is mostly rotational. \ D m1 = D m2
Small fluid elements, while moving, also rotate about fi r1 A1v1 = r2 A2v2 ...(24)
their own centre of mass. In a rapidly flowing river full
of boulders, the flow is highly turbulent and rotational. Since liquid is incompressible, its density cannot
formation of vortex and eddies are quite common in such change.
flow.
\ r1 = r2
We will study only about steady flow of an ideal fluid.
An ideal fluid is incompressible and non-viscous. Hence, A1v1 = A2v2 ...(25)
This is continuity equation. It can be applied for any tube
11. EQUATION OF CONTINUTIY of flow.
This important equation in fluid dynamics follows from You must have noticed that speed of water coming out of a
conservation of mass. It states that in a steady flow of an garden hose can be increased by partially closing the hose
ideal fluid, the product of area of cross section and speed opening (thereby decreasing the area) with your thumb. This
remain same at all locations in a tube of flow. is continuity equation in action.
Consider a fluid flowing through a pipe of varying
cross-section. Flow speed is v1 at a location where area
Example 23   A syringe has a piston of cross-sectional area
of cross-section is A1 and flow speed is v2 at location where
A1 = 5 cm2. The outlet of the
area of cross-section of the pipe is A2.
nozzle has area of A2 = 4 mm2.
The piston is pushed with a
speed of v1 = 1 cms–1. Find the
speed with which the liquid is
ejected out of the nozzle.

Solution
Concepts
The liquid in contact with the piston moves with
speed v1.
A1v1 = A2v2

Volume of liquid entering through A1 in time D t


is A1v1 D t. [Actually, the shaded volume of liquid will cross A2v2 = A1v1
section A1 in time D t]. fi   (4mm2) v2 = (5 × 102 mm2) × (1 cms–1)
Mass of liquid entering at A1 in time D t is
fi  v2 = 125 cms–1 = 1.25 ms–1
D m1 = r A1v1 Dt [r1 = density of liquid at A1]
Similarly, volume of liquid flowing out through section An important thing to realise is that the volume swiped by
A2 is A2v2D t. Mass flowing out is the piston will be equal to liquid pushed out of the nozzle.

Your Turn

Q.36  A water tank has the shape shown in figure. The


cross-sectional areas are A1 = 0.5 m2, A2 = 1.0 m2 and
A3 = 0.05 m2. The piston having area A1 is pushed with a
speed of v1 = 0.1 ms–1. With what speed (v3) the water flows
out of outlet having area A3? With what speed is a fluid
particle at point P moving?
Fluid Mechanics  8.27 

1 1
12.  BERNOULLI’S EQUATION \  D k = __
​   ​  rD V v 22 – __
2
​   ​  rD V v 12
2
...(ii)
The equation bears the name of Daniel Bernoulli, who
studied fluid flow in eighteenth century. It is not something Change in gravitational potential energy (D U)
fundamental, but a form of work–energy theorem which is As the fluid volume (abcd) shifts to (a¢b¢c¢d¢), change in
more suitable for use in study of flowing fluids. its PE is
Consider a fluid flowing through a tube having variable D U = PE of volume (c c¢d¢d)
cross-section and changing height. Consider a liquid volume
– PE of volume (a a¢b¢b)
contained in the volume abcd of the pipe. This liquid volume
advances to new position a¢ b¢ c¢ d¢ in a small time interval = Dm gh2 – D mgh1
D t. We will apply work–energy theorem on considered = rD V gh2 – r D V gh1 ...(iii)
volume of fluid, as it moves from its initial position abcd
to its final position a¢ b¢ c¢ d¢.
Work done by surrounding liquid (W)
The fluid to the left of boundary ab pushes with a force P1A1
and its point of application gets displaced by v1D t. It does
positive work P1A1v1 D t on our fluid volume considered.
The fluid to the right of boundary cd exerts a leftward
force P2A2 and the boundary cd moved by a distance v2D t
towards right. Work done by this force is negative equal
to – P2A2 v2 D t.
\ W = P1A1v1 D t – P2A2v2 D t

= P1D V – P2 DV  [using (i)] ...(iv)


If the fluid is ideal, there is no viscous force. Normal
force by the pipe wall does no work, as it is perpendicular
Let A1 and A2 be cross sectional area of the pipe at ab to the direction of motion.
and cd respectively. v1 and v2 are flow speeds at ab and cd. Using work-energy theorem, we can write
P1 and P2 and pressure at ab and cd respectively.
W = D K + D U
Volume a a¢b¢ b is A1v1 D t as the distance a a¢ = v1D t.
1 1
Similarly, volume c c¢ d¢ d = A2v2 D t. fi  P1D V – P2 D V = ​ __ ​  r D V v 22 – __ ​   ​  r D V v 21
2 2
From continuity equation: + r D Vgh2 – r D V gh1
A1v1 = A2v2 1 1
fi  P1 – P2 = __ ​   ​  r v 22 – __
​   ​  r v 12 + rgh2 – rgh1
fi  A1v1D t = A2v2D t = D V (say) ...(i) 2 2
1 1
fi Volume (a a¢b¢b) = Volume (c c¢d¢d) = D V fi  P1 + __ ​   ​  r v 12 + rgh1 = P2 + __ ​   ​  r v 22 + rgh2 ...(26)
2 2
1
Change in kinetic energy (D K) fi  P + __​   ​  r v 2 + r gh = constant
2
As the fluid moves from position (abcd) to new position This equation is known as Bernoulli’s equation.
(a¢b¢c¢d¢), its kinetic energy changes. This change is due to
the fact that KE of volume (cc¢d¢d) is different from KE of 12.1  Pressure as a form of energy
volume (a a¢b¢b). KE of fluid particle in the region a¢b¢cd is 1
The term ​ __ ​  rv2 in Bernoulli’s equation represents the kinetic
same in initial and final positions. In steady flow, speed at 2
a point does not change with time. energy of a unit volume of the fluid. [This is because r is
nothing but mass of unit volume.] The unit of this term is
\ D k = KE of volume (c c¢d¢d) Jm–3.
– KE of volume (ab b¢a¢) Similarly, rgh is nothing but gravitational potential
1 1 energy of unit volume of the fluid.
= ​ __ ​  D mv2 2 – __
​   ​  D m v 12
2 2 The third term–pressure P – can be interpreted as pressure
energy of unit volume of the fluid. Unit of pressure Nm–2
Where D m = rD V = mass of fluid in volume (a a¢b¢b) or
is same as Jm–3.
(c c¢d¢d)
8.28  Mechanics II

It is a bit difficult to realise that pressure is a form of 13. APPLICATIONS OF BERNOULLI’S


energy. Let us try to understand this in a simple way.
Imagine a small tank of water with a plugged outlet. If
EQUATION
you open the plug, water flows out with some speed. Where Bernoulli’s equation is valid for steady flow of an ideal fluid
does the water get kinetic energy from? You can argue that but it gives us a lot of meaningful insights into real-life
water in the tank is having gravitational potential energy. Ok, situations. Here, we will learn that some daily-life observation
let us accept this argument for a moment. Now imagine the can be easily understood in terms of this principle.
same tank having water at increased pressure.
13.1 Speed of efflux
Consider a liquid of density r filled in a tank of cross-
sectional area A1. The liquid is filled to a height h and
the tank is open to atmosphere. There is a small hole of
cross-sectional area A2 in the side wall near the bottom of
the tank. The liquid flows out of the hole and we wish to
find the speed (v) with which it comes out of the hole.
The top surface of the liquid moves down at speed u such
that
A1u = A2v ...(i)
You can think that the tank is closed with a movable This is continuity equation.
piston and a large weight is placed on the piston [think
yourself standing on the piston]. If the plug is removed,
the water flows out with a great speed. What resulted in
increased kinetic energy of the outgoing water?
The increased pressure caused the water to gain more
kinetic energy. Therefore, pressurised liquid has more
energy. It is nothing wrong to regard pressure as a form of
fluid energy.
Bernoulli’s equation can be simply stated as – Sum of
kinetic energy, pressure energy and gravitational potential For the flowing liquid, we will use Bernouli’s equation,
energy (per unit volume) is constant for a steady flow of an considering two points as 1 and 2 shown in figure. Pressure
ideal fluid. at 1 is atmospheric pressure (P0). Pressure at 2 (just outside
the hole) is also atmospheric pressure. [A jet of liquid in air
is subjected to atmospheric pressure only].
Example 24   PE increases, KE does not decrease
1 1
A liquid of density r is flowing in a vertical P2 + ​ __ ​  r v 22 + r gh2 = P1 + __
​   ​  r v 21 + r gh1
pipe of uniform cross-section. At point 1, speed 2 2
of flow is v. What is the speed of flow at point 1 1
2, which is at a height h above 1? Justify your fi P0 + __
​   ​  r v2 + 0 = P0 + ​ __ ​  r u2 + r gh
2 2
result in terms of energy of the fluid.
[We have chosen the level of point 2 as the reference
Solution level for measuring height. One can choose the reference
Concepts level elsewhere also. It makes no difference.]

(i) Use of continuity equation will give speed at 2 fi (v2 – u2) = 2gh

[  ] [ 
(ii) A flowing liquid has energy in three forms – kinetic
energy, potential energy and pressure energy.
A22
fi  v2 ​ 1 – ___
A2
​  2  ​  ​ = 2gh ​ using (i), u = ___
​   ​  v  ​ ]

A1 A1
Use of A1v1 = A2v2 tells us that speed at 2 is same as ______

÷
speed at 1 (   A1 = A2). 2gh
fi v = ​ ______
​   2 
  ​ ​ ...(27)
The gravitational PE of the liquid increases as it moves A
1 – ___
2
from 1 to 2 but its KE does not change. A liquid has a ​  2 ​ 
third from of energy – pressure energy. The liquid pressure A1
decreases at the liquid climbs up. Fall in pressure energy If the tank is wide, then A1 >> A2 and
results in rise in PE.
Fluid Mechanics  8.29 

A22
___ This equation is nothing but Bernoulli’s equation used
​  2 ​ <<
  1 between points just inside and just outside the hole.
A1 ____
____
\   ​  
v ​÷2gh  ...(28) \   ​ 
v = ​÷2gh 

This speed is known as speed of efflux and the result


13.2 Siphon
expressed by equation (28) is known as Torricelli’s theorem.
The theorem can be stated as: This is a simple device, which most of us have used in our
The speed of liquid coming out of a small hole at a depth daily life. It is a simple pipe used to drain liquid from a
h below the free surface of an open tank is same as the speed tank with the help of gravity only.
gained by a particle falling freely under gravity through a
height h.
Note:  Speed of efflux remains same even if the hole is
made at the bottom surface of the tank.
Position of the hole is not
important; it is height (h) of
liquid above the hole that
matters.
An alternative way of
finding the speed of efflux can
be as follows (though there is
no difference in basic principle
used):

Look at the figure. All air is sucked out of the pipe and
the liquid begins to flow. Let the speed at outlet D be v. If
the pipe has uniform cross-section, the continuity equation
requires that the speed of flow must be same everywhere in
the pipe.
Let us apply Bernoulli’s equation between points E
and D.
1
PD +  ​1    ​ rv D2 + r ghD = PE + __
​   ​  r v 2E + r ghE
Consider a small element of liquid at the bottom of the ​2 ​ 2
tank, just inside the hole. If tank is large (A1 >> A2), speed Taking reference level (for height) to be at D and assuming
everywhere inside the tank is zero. The considered element that the tank is wide (which means vE 0), we get
has no speed but it is subjected to large pressure. 1
P0 + __
​   ​  rv2 + 0 = P0 + 0 + rg (h3 + h1)
Pressure is P = P0 + r gh 2
__________
As the element moves out, pressure on it reduces to fi   1 +  
v = ​÷2g (h h3) ​ ...(29)
atmospheric pressure (P0). There is no change is gravitational
Note that the speed of outflow depends on the height
PE as the element is essentially at same height. The loss in
difference between the liquid surface and the outlet of the
pressure energy manifests as gain in KE.
pipe. It has got nothing to do with position of inlet. If
Gain in KE = Loss in pressure energy the inlet end of the pipe is pushed deeper into the tank, it
will not make any difference to the speed of liquid flowing
1
fi ​ __ ​  D mv2 = (P – P0) (volume) out at D.
2
What about pressure in the pipe? Where is it minimum?
[‘Pressure’ is energy per unit volume] At D, pressure is P0 (atmospheric pressure). As you move up
from D, the kinetic energy does not change but gravitational
1
fi ​ __ ​  r (Volume) v2 = (rgh) (volume) potential energy increase. It increases at the cost of pressure
2
energy. Higher you move above D, lower is the pressure.
1
fi ​ __ ​  r v2 = rgh Pressure is smallest at C. Using Bernoulli’s equation between
2 C and D gives:
8.30  Mechanics II

1 1
PC + __
​   ​  r v2 + rg (h1 + h2 + h3) = PD + __
​   ​  r v2 + 0
2 2
\  PC = P0 – rg (h1 + h2 + h3)

The siphon will stop working if pressure at C drops to


zero. For successful working of the device
PC > 0
fi  P0 > rg (h1 + h2 + h3)
P0
fi  (h1 + h2 + h3) < ​ ___   ​ ...(30)
rg
Are pressures at point A and B different? Let us look at
it. Pressure at A is
To measure pressure difference between a point in the
PA = P0 + rgh1 [Speed at A  0] pipe and at constriction, we use manometers, as shown in
At B (a point inside the pipe), the liquid has kinetic the figures. With regard to first figure we can write.
energy. Height of A and B are same. It means pressure at B P1 – P2 = rgh [r = density of flowing liquid]
is less than that at A.
1 fi D P = rgh ...(i)
PB + __
​   ​  r v2 = PA + 0
2 From continuity equation:
1
fi PB = PA – __
​    ​ r v2 A1v1 = A2v2 ...(ii)
2
= P0 + rgh1 – rg (h1 + h3) Using Bernoulli’s equation
1 1
[using (29)] P1 + __
​   ​  r v 12 = P2 + __ ​   ​  r v 22
2 2
= P0 – rgh3
1
Please note that if a fluid has more speed at a point fi P1 – P2 = __ ​   ​  r (v22 – v12)
2

(  )
compared to another point at same horizontal level, the 2
pressure is lower at the point of higher speed. 1 A1
fi D P = __
​   ​  r ​ ___
​  2  ​ – 1  ​ v12

  
2 A2
13.3  Venturi meter
_________

÷( )
Venturi meter is a device used to measure the speed and 2 D P
fi v1 = ​ _________
​       ​ ​ ...(30)
flow rate of fluid through a pipe. Consider a horizontal A12
___
r ​ ​  2 ​ – 1  ​
pipe of cross-sectional area A1 through which a liquid is
A2
flowing at some unknown speed v1. A constriction (i.e., a
throat) of cross-sectional area A2 (< A1) is attached to the Value of D P is known form (i). Thus, we know v1.
pipe. This narrow zone is known as venturi. As the liquid Volume flow rate of the fluid through the pipe is
flows through the narrow zone, its speed increases. The A1v1 (m3s–1).
pressure decreases, as required by the Bernoulli’s equation.
By measuring the drop in pressure, we can workout the value 13.4  Flight of aeroplanes
of flow speed v1. One of the most interesting applications of Bernoulli’s
equation is flight of aeroplanes. When an aeroplane runs
on a runway, it feels
air blowing against it.
The cross-section of the
wings is designed such
that the wind causes
less drag (resistnace to
motion) and more lift
(an upward force against
gravity). A term aerofoil
Fluid Mechanics  8.31 

is used to describe the cross-sectional shape of objects such must have seen a curling path of football kicked by Lionel
as aeroplane wings. Messi, bluffing the goalkeeper.
Consider an aeroplane running towards left. It feels air
passing over it towards right. The aerofoil design of the 13.6  A Sprayer
cross-section of the wing causes most of the air to glide Let’s understand the Physics behind a hand pump used to
above the wing. Very less air passes beneath the wing. spray insecticides. It has a cylinder A with a movable piston
The streamlines get crowded above the wing. Thus, speed B. The cylinder has a narrow barrel C. It is connected by
of air is higher at point 1 (above the wing) compared to a thin tube D to a tank (E) containing insecticide. There
speed at point 2 (below the wing). There is not a considerable may be a hole in the tank to maintain pressure equal to
height difference between 1 and 2 and therefore, pressure at atmospheric pressure inside it. when the piston (B) is pushed
2 is higher than pressure at 1. This gives an upward force to with speed v1. The air passes through tube C at a much higher
the wing, helping the aeroplane to take–off. This is a simple speed (v2) due to its small cross-sectional area. Due to high
explanation of the phenomena of take–off, though the reality speed of air inside C, there is a sharp drop in pressure. The
is more complex. pressure inside the liquid tank (E) is atmospheric pressure.
This high pressure
13.5  Magnus effect causes the liquid to
A spinning ball travelling through air does experience a get pushed up in tube
sidewise force, deflecting its path. This effect is known as D. [It is similar to
Magnus effect. the phenomena taking
place in a barometer].
The liquid rising in
tube D gets mixed with
lot of air coming from
the cylinder and gets
sprayed.

Example 25   Maximum range


A tank placed on ground has water filled in it, up to a height
H. A small hole is punched in its side wall at a height h
from the bottom. Water jet strikes the ground at a horizontal
distance x.

A ball travelling through air experiences a wind in opposite


(a) For what value of h is x maximum?
direction. In case of a non-rotating ball, the streamlines
separate evenly on both sides. It means speed of air is (b) For two value of h, we get same range (x). If one
same on both sides of the ball. For a rapidly spinning ball, value of h is H/4, what is its other value?
the situation is slightly different. At one side, the spinning Solution
surface of the ball drags air along with it and increases the
flow speed. On the other side, motion of the surface of the Concepts
____
ball is against the wind, thereby decreasing the flow speed. (i) For small hole, speed of efflux is v = ÷   ​ where
​ 2gy 
This difference in speed on two sides results in pressure y is height of water above the hole.
difference in accordance with Bernoulli’s equation.
(ii) Range is obtained as product of v and time of flight
In the figure shown, speed at A is higher than speed at B.
(T). Time of flight can be obtained by treating the
Thus, pressure at B is higher. The ball experiences a force,
water jet as a horizontal projectile.
due to this pressure difference that is perpendicular to its

÷ 
___
2h
direction of motion. This causes the ball to deviate. You T = ​ ___
​  g ​ ​  
8.32  Mechanics II

_________

(a) Speed of efflux; v = ​÷2g (H – h) ​

÷ 
___
2h
Time of flight; T = ​ ___
​  g ​ ​  
________

\  Range x = vT = 2 ​÷h (H – h) ​
  ...(i)

Range is maximum when the product Z = h (H – h)


is maximum. This happens when
dZ H
​ ___  ​ = 0  fi  H – 2h = 0  fi  h = __
​   ​  If liquid level in the tank decreases by dy in time interval
dh 2 dt, we have
(b) From (i), it is obvious that range will be same for
two values of h: h and (H – h). – A dy = av dt
H Negative sign is necessary as y is decreasing.
If one value of h is ​ __ ​ , the other must be
4 ____
H ____ 3H \   ​ dt
– A dy = a ​÷2gy 
H – __
​   ​ =​ 
   ​ 

4 4 t 0
1
A___ – ​ __ ​ 
This implies that if we make two holes at an equal fi ​Ú   ​ ​ dt = – ​ _____
    ​ ​Ú  ​ ​ ​y​ 2 ​ dy
vertical distance from top and bottom, both jets will t = 0    ​ H
a ​÷2g 
hit the ground at same spot.
2A___ __ 0
fi t = – ​ _____
     ​​]​
 ​ [​÷y   H ​​ 
   ​
a ​÷2g 
Example 26   Time to empty a tank

÷ 
___
A tank has cross-sectional area (A) and a liquid is filled in A 2H
it up to a height H. A small hole of cross-sectional area a fi t = __
​ a ​  ​ ___
​  g   ​ ​ 
is opened at the bottom of the tank. In how much time the
tank will get emptied?
Example 27    A large tank
contains water (density 2r) filled
up to height 2h. Oil (density r) is
poured over water to a height h. A
hole is punched in the side wall at
a height h from the bottom. Find
speed of efflux.
Solution
Concepts
Solution Use Bernoulli’s equation by considering two points in the
Concepts same liquid.
(i) Speed of efflux (v) depends on height (y) of Consider a point 1 just inside the hole and a point 2 just
the liquid in the tank. It is not constant. It is outside it.
decreasing.
(ii) Volume flow rate (m3s–1) is given by av. Volume
of liquid flowing out in time dt is given by av dt.
(iii) The hole is small, implying that speed of efflux (v)
can be written using Torricelli’s theorem. The water
level moves down at a much slower pace compared
to the speed of liquid coming out of the hole.
____
  ​  Flow speed at 1 is 0
When height of liquid is y, speed of efflux is v = ​÷2gy 
Volume of liquid leaving the tank in a small interval dt Pressure at 1 is P1 = P0 + rgh + 2rgh
is avdt. = P0 + 3rgh
Fluid Mechanics  8.33 

Bernoulli’s equation applied between 1 and 2 gives Example 29   Stream of water emerging from a tap
“necks down”
1 1 1
P1 + rgh1 + __
​   ​  rv 12 = P2 + __
​   ​  rgh2 + ​ __ ​  rv 22 Open a tap discharging water at a steady
2 2 2
rate. The stream of water narrows down as
h1 = h2  and  v1 = 0 it falls.
(a) Why does this narrowing of stream
Also, density of liquid is 2r. happens?
1 (b) The cross-sectional area of the tap
\ P0 + 3rgh = P0 + __
​   ​  (2r) v2
2 is A1 = 1.2 cm2 and the area of cross-section of water
____ stream at a depth h = 4.5 m below it is A2 = 0.35 cm2.
fi   ​ 
v = ​÷3gh  Calculate the volume flow rate from the tap.
Solution
Example 28   A liquid is flowing in a horizontal pipe.
Two tubes, A and B, are inserted in the pipe. Tube B is Concepts
L-shaped and its cross-section at 1 is held normal to the
(i) From Bernoulli’s equation, we can see that speed
flow direction. Liquid rises in both tubes. Height of liquid
increases as water falls down.
column in B is h more than that in A. Find the speed of
flow (v) in the pipe. (ii) From continuity equation, the area of cross-section
must decrease with rising speed.

(a) As the water falls, its speed increases. This is due to


loss in gravitational PE. Since volume flow rate is
same at any cross-section, the area must be smaller
if speed is higher.
(b) Bernoulli’s equation
1 1
P1 + __
​   ​  r v 12 + rgh1 = P2 + __
​    ​ r v 22 + rgh2
2 2
Taking the reference level to be zero at depth h below
Solution the tap,
Concepts h1 = h  and  h2 = 0
P1 = P2 = P0 (atmospheric pressure)
(i) Liquid in tube B is at rest. This means that speed
1 1
of flow at the mouth of tube at 1 is zero. But speed \  P0 + ​ __  ​ r v 12 + rgh = P0 + __ ​    ​ r v 22
at a point like 2 on the same horizontal level is v. 2 2
This implies that pressure at 1 is more than the 1
fi ​ __ ​  r (v22 – v12) = rgh
pressure at 2. 2
(ii) Pressure difference between 1 and 2 is measured fi v22 – v12 = 2gh
A1


by h. But A1v1 = A2v2  fi  v2 = ___ ​   ​  v1
A2

(  )
______

÷
P1 – P2 = rgh ...(i) ​A2​1​​  2 gh
\ ​ ___
​  2 ​ – 1  ​ ​v​21​ ​ = 2 gh  fi  v1 = ​ ​ ______   
 ​ ​

Using Bernoulli’s equation between points 1 and 2 ​A​2​​  ​ 2​1​​ 
A
___
​  2 ​ – 1
​A2​ ​​ 

  ( 
1 1
P1 + __
​   ​  r v 21 = P2 + __
​   ​  r v 22
2 2 _____________

÷
2 × 9.8 × 0.045
1 fi v1 = ​ ​  _____________     
    ​ ​
P1 – P2 = ​ __ ​  r v2

2
[   v1 = 0] 1.2 2
____
)
​​ ​    ​  ​​ ​  –  1
0.35
1
fi rgh = ​ __ ​  r v2 = 0.286 ms–1 = 28.6 cms–1
2
____ Volume flow rate is = A­1v1 = (1.2 cm2) (28.6 cms–1)
fi   ​ 
v = ​÷2gh 
= 34 cm3 s–1
8.34  Mechanics II

Example 30   A vertical pipe has 1 1


P1 + __
​   ​  r v12 + rgh1 = P2 + __ ​   ​  r v22 + rgh2
cross-sectional area A1 = 0.2 m2. Water 2 2
(density r = 103 kgm–3) is flowing 1
fi  P1 – P2 – rg (h2 – h1) = __ ​   ​  r (v22 – v12)
through it. A pressure gauge G1 shows a 2
reading of 200-Pa. There is a narrow
fi  200 × 103 – 140 × 103 – 103 × 10 × 2

[ 
zone of cross section A2 = 0.1 m2 at a
height of h = 2 m above the location
of G1. The reading of another pressure
1
= __
A21
​   ​  × 103 ​ ___
2
​  2  ​ v2 – v2  ​
A2 ]
[ 
gauge G2 located at the narrow zone
is 140 kPa. Find the flow speed (v) in the pipe.
Solution
A1
​ A2v2 = A1v  fi  v2 = ___
​   ​  ◊ v  ​
A2 ]
Concepts fi [ (  ) ]
0.2 2
80 = ​ ​​ ___
​   ​  ​​ ​ –1  ​ v2  fi  80 = 3v2
0.1
Use of continuity equation and Bernoulli’s equation.

÷ 
___
Don’t use equation (30). That equation was derived for 80
fi v = ​ ___
​   ​ ​  = 5.16 ms–1
a horizontal pipe. 3

Your Turn

Q.37  During wind storms, light roofs are blown off. Q.42  Water flows through a
Why? horizontal tube of variable cross-
section. The area of cross-section
Q.38  A man is standing near a railway line. When a fast at A and B area 4 mm2 and 2 mm2
moving train crosses him, he feels as if he is being pushed
respectively. 1 cm3 of water enters into the tube per second
towards the train. Explain.
at A.
Q.39  Hold a strip of paper at one end. By blowing air over (a) Find the speed of water at B.
the strip of paper, you can keep it horizontal. Explain. (b) Find pressure difference PA – PB.
Q.43  In the figure shown, find the speed of efflux from
the hole.

Q.40  Air is streaming past a horizontal airplane wing


such that its speed is 120 ms–1 at the upper surface of the
wing and 90 ms–1 at the lower surface. Density of air is
1.3 kg–3. The wing is of rectangular shape; 10 m long and
2 m wide. Calculate the lift force on the wing.
Q.41  When the tap shown in figure Q.44  Find the speed of efflux from end A of the pipe
is closed, pressure in the pipe at point 1 shown in figure. Length of pipe is
is 3.5 × 105 Pa. When the tap is opened l and it is inclined at an angle q to
the pressure drops to 3.0 × 105 Pa. Find the horizontal. Area of cross-section
the speed of water flowing through the of the pipe is small compared to that
pipe. of the tank.

In short: (ii) A steady flow can be represented by a streamline


(i) A flow is said to be steady if flow conditions do not diagram. A streamline represents path followed by
change with time. all particles crossing through a point.
Fluid Mechanics  8.35 

(iii) Density of streamline represents flow speed at a loca- 1


tion and tangent on a streamline gives direction of P + __ ​   ​  rv2 + rgh = a constant
2
flow. (vii) In a flowing fluid the pressure at a point is higher
(iv) In steady flow of an incompressible fluid, volume if flow speed is lower compared to another point on
flow rate through any cross-section is constant. same horizontal level where speed is higher.
A1v1 = A2v2 = volume flowing past a cross-section (viii) Speed____ of efflux from a small hole in a large tank is
per unit time v=÷    
​ 2gh  ​, where h is height of water surface above
(v) Pressure can be regarded as a form of energy for a the hole.
fluid. (ix) Time needed to empty an open tank of liquid through
Pressure (P) is energy per unit volume. a small hole (area = a) at the bottom is

÷ 
___
(vi) Sum of kinetic energy, potential energy and pressure A 2H
t = __
​ a ​  ​ ___
​  g   ​ ​ 
energy (per unit volume) remains constant in steady
flow of an ideal fluid. This is known as Bernoulli’s Where A = cross-section of tank and H is initial
equation. height of liquid.

Miscellaneous Examples
Example 31   Density of liquid in a large tank increases with
depth y from the surface as r = a + by, where a = 1.0 gcm–3
and b = 0.01 gcm–4. Two balls, each of volume V = 1.0 cm3
are connected by a light string of length l = 15 cm and
released in the liquid. Density of two balls are r1 = 1.2 gcm–3
and r2 = 1.4 gcm–3. At what depth do the two balls settle
in equilibrium?
Solution
Concepts
��������������������������������������������������
(i) Buoyancy on both balls must sum up to the com-
bined weight of the two balls.
(ii) The heavier ball must be in lower position.
Buoyancy (for same volume) increases with depth,
as density of liquid in increasing. The ball in lower
position experiences higher buoyancy. For string to
be taut, the lower ball must have its weight greater
than buoyancy acting on it. Example 32   A ball of volume V and density d is inside
a liquid of density r. It is moved
Mass of the balls are: m1 = r1V = 1.2 g vertically down by a further distance
h. Find the change in gravitational
m2 = r2V = 1.4 g PE of the system of the ball and the
For equilibrium liquid.

FB1 + FB2 = m1g + m2g Solution


Concepts
V (a + by) g + V [a + b (y + l)] g = (m1 + m2) g
When the ball moves down, a same volume of liquid
Let us write everything in CGS unit system. moves up to occupy the space emptied by the ball. PE of
1 (1 + 0.01 y) + 1 [1 + 0.01 (y + 15)] = 2.6 the ball decreases and that of the liquid increases.

fi y = 22.5 cm Ball moves from position A to B. Its PE changes by


\  Depth of the lighter ball is 22.5 cm and that of the D Uball = – mgh = – Vd . gh
heavier ball is 22.5 + 15 = 37.5 cm.
8.36  Mechanics II

Liquid in position B moves up Example 34   Recoiling tank


to occupy the vacant position at A. A tank of cross-sectional area A is filled with water to
change in PE of liquid. a height H. It stands on a smooth horizontal surface. A
D UL = mL gh = Vrgh small orifice is made at the
bottom of the vertical wall.
\ D U = D UL + D Uball The water flows out of the
= V (r – d) gh orifice and the tank recoils.
Find the speed of the tank
Example 33   In a flowing fluid, the velocity distribution when it gets emptied.
in xy plane is given by Assume mass of empty tank
​_› to be negligible.
​v  ​   = (kx) iˆ – (ky) jˆ
Where k is a positive constant. Draw the streamlines in Solution
xy plane for y > 0 and give a possible interpretation of the Concepts
pattern.
(i) Speed of efflux
____ (u) with respect to the tank is given
Solution by u = ​÷2gh    ​ where h is instantaneous height of
water in the tank.
Concepts
(ii) The thrust force experienced by the water tank is

|  |
Path of any particle is a streamline. Therefore, we need to dm dm
find the path equations of various particles. u ​ ___
​   ​  ​ where ​ ___ ​ is rate of leakage of mass from
dt dt
dx the tank.
vx = kx  fi ​ ___ ​ = kx ...(i)
dt [Refer to variable mass system in the chapter of
dy momentum]
And vy = – ky  fi ​ ___ ​ = – ky ...(ii)
dt
(ii) ÷ (i) When height of water in the tank is h, let the instantaneous
speed of the tank be u.
dy/dt
_____ y dy y
​  ​ = – ​ __x ​  fi ​ ___  ​ = – ​ __x ​
   ____
dx/dt dx   ​ = speed of outgoing mass wrt the tank
u = ​÷2gh 


dy
Ú ___ dx
  – Ú ___
​  y ​ = ​  x ​ 
dm
\ ​ ___
|  |
​   ​  ​ = rate of leakage of mass = au ◊ r
dt
Where a = cross-sectional area of orifice and r is density
fi ln y = – ln x + ln c  fi  ln (xy) = ln c
of liquid.
fi xy = c
This is an equation of hyperbola. We get different
hyperbolas for different values of c. These hyperbolas are dv
dm
\  Thrust force, Fth = u ​ ___
|  |
​   ​  ​ = au2r = 2g ar ◊ h
dt
\  m ​ ___ ​ = 2g ar ◊ h
streamlines. The arrowheads can be determined by looking dt
at (i) and (ii). For example, in first quadrant, ux is positive But instantaneous mass of liquid in the tank can be written
and vy is negative. as
m = Ahr where A = cross section of the tank
dv
\ ​ ___ ​ =
dt
a
  2g ​ __
​   ​   ​
A (  )
dv dh a
fi ​ ___  ​ ◊ ​ ___ ​ =
  2g ​ __ ​ 
dh dt A
dh
But volume flow rate = a ◊ u = – A ◊ ​ ___ ​ 
dt
dh
___ au
___
fi ​   ​ =   – ​   ​ 
dt A

(  )
The streamlines could simulate the flow of a downward dv au a
stream against a flat floor. \ ​ ___  ​ ​ – ​ ___ ​   ​ = 2g ​ __ ​ 
dh A A
Fluid Mechanics  8.37 

dv ____ Example 36   A U-shaped tube has two immiscible liquids


fi – ​ ___  ​ (​÷2gh 
  ​)  = 2g
dh filled in it. The liquid in right arm has density r and the
V
___ 0 density of liquid in the left arm is not known. The equilibrium
dh
fi ​Ú ​  ​ dv = – ​÷2g 
  ​ ​Ú  ​ ​ ___
​  __  ​  position has been shown in the diagram. Now, the tube is
0 H ÷    ​
​ h  accelerated horizontally to the right such that liquid columns
____ have same height on the two sides. Find the acceleration of
fi   ​ 
v = 2 ​÷2gH 
the tube.

Example 35   A ball balanced in a water jet


Water jet is coming out through a pipe of cross-sectional
radius r at a velocity u in upward direction.
The water jet widens as it rises, though it
is nearly correct to assume that velocity of
water particles remain vertical. At a height
H above the outlet, there is a ball supported
by the water jet. Weight of the ball is W. Solution
Density of water is r. Find H for which the Concepts
ball will remain in equilibrium. Assume that
water particles striking the ball momentarily (i) We can find the density of the other liquid by
come to rest. writing pressure at the interface of the two liquids;
working from both sides and equating them.
Solution (ii) By conserving the volume, it is easy to see that
3h
Concepts height of liquid columns in both limbs must be ___ ​   ​ 
2
(i) As the water jet rises, its speed decreases and when the two heights are equal.
therefore cross-sectional area widens. (iii) In new position, we will again write the pressure at
(ii) Using Bernoulli’s equation, we find speed at height the interface, working from two sides and equating
H. Volume flow rate is same throughout the jet. them.
(iii) Force on ball due to impinging water jet = rate (iv) In a horizontally accelerated container, pressure
of change of momentum of the water jet due to increases when we move horizontally inside liquid
striking the ball. opposite to acceleration. Refer to equation (18).

Let v = speed of water on reaching the ball. Let us write the pressure at the interface of two liquids
when the tube is at rest.
1 1
P0 + __
​   ​  r v2 + rgH = P0 + __
​   ​  r u2 + 0 If r¢ is density of other liquid, then
2 2
________
fi v2 = u2 – 2gH  fi  v = ​÷u  2 – 2gH  
​ r¢gh = rg ◊ 2h  fi  r¢ = 2r

Volume flow rate, Q = p r2 ◊ u When the tube is accelerated, height of liquid in both
3h
Mass of water hitting the ball per second limbs becomes equal. This height must be ​ ___ ​  for volume
2
to remain conserved.
= Q ◊ r = pr2 u r
Let us write pressure at liquid interface (1), working from
* 

Change in momentum per unit time = (p r2 ur) v right.

\  Force on ball = (p r2 ur) v


2

3h
P1 = P0 + rg ​ ___(  ) (  )
3h
​   ​   ​ + ar ​ ___
2
​   ​   ​
2
For equilibrium: (p r  ur) v = W
[Refer to equation 18]
________
Working from left, we can write
​ 
fi  p r2 u r ÷ u2 – 2gH  
​= W

[ 
3h h
P1 = P0 + 2rg ◊ ​ ___ ​ – a 2r ​ __ ​ 
]

1 W2 2 2
fi ​ ___   ​ ​ u2 – ________
​  2 4 2 2 ​  
​= H
2 g p   r  u  r
8.38  Mechanics II

Example 38   A right-angled


tube is connected to a large tank
as shown. The cross-sectional
area of the tube is S and its
discharge end is at a depth H
below the water surface. Find
the torque of reaction force of
flowing water, acting on the
Equating the two values bend of the tube relative to
point O. Density of water is r.
(  )
3h
rg ​ ___ (  )
3h
​   ​   ​ + ar ​ ___
2
3h
​   ​   ​ = 2 rg ​ ___
2 (  ) h
​   ​   ​ – a 2r ​ __
2 (  )
​    ​  ​
2 Solution
Concepts
3
fi  a = ​ __ ​  g
5 As water flows out, its speed is constant throughout the tube
but there is a change in direction of velocity at the
Example 37   Flow at a bend bend. Thus, momentum of flowing water changes at
An ideal liquid is flowing through a flat horizontal tube of the bend. Water experiences force due to pipe for this
uniform cross-section. The flow is steady. The tube has an change is momentum. From Newton’s third law, the water
L-shaped bend as shown. Consider two points 1 and 2 at the exerts equal force on the bend. This force causes torque.
bend. At which point the streamlines will be more crowded?
At which point is the pressure higher?

____
  ​ 
Speed of efflux v = ​÷2gH 
Volume flow rate,

Solution Q = Sv

Concepts Mass flow rate = Q ◊ r = rSv


(i) Streamlines have higher density at locations where Change in momentum of water per unit time is force
speed of flow is high. applied by the bend on it.
(ii) Pressure is lower at a point where speed is higher ​_›
if the points are on same horizontal level. \ ​F  ​   = (rSv) v jˆ – (rSv) v iˆ

= rSv2 ( jˆ – iˆ)


Consider a small
cylindrical fluid element Force applied by water on the bend is
negotiating the bend. As the ​_› ​_›
fluid element turns, it must \ ​F  ​ 0 = – ​F  ​  = rSv2 (î – jˆ)
have a radial acceleration.
For this, it must have radial Torque about O is
_ ​_›
force acting on it. › ​
​t ​    = (l iˆ) × ​F  ​0  = (l iˆ) × rSv2 (iˆ – jˆ)
Therefore, P2D S > P1D S  fi  P2 > P1
= – lrSv2 k̂
Higher pressure at 2 implies lower speed at 2. Therefore, ​_›
the streamlines will be more crowded near 1. \ |​t ​ |  = l rSv2 = lrS (2gH) = 2l rS gH
Fluid Mechanics  8.39 

Example 39   A horizontal tube of length L is filled with a Example 40   A balloon
liquid of density r. It is rotated about a vertical axis passing A gas-filled balloon is in equilibrium while floating in air.
through its open end. The liquid column occupies a length x Pressure inside it is P, which is greater
in the tube, as shown. A small hole develops at the other end than atmospheric pressure (P0). Density of
of the tube. Find the speed with which the liquid is initially gas in the balloon is r. The seal of the
ejected with respect to the tube. balloon breaks and a small hole of area A
gets developed. Find the initial thrust force
experienced by the balloon due to the gas
escaping out of the hole.
Solution
Concepts
(i) The speed of outgoing gases relative to the balloon
Solution can be calculated using Bernoulli’s equation.
Concepts (ii) Then using the concept of variable mass learnt in
(i) Pressure at open end is atmospheric pressure. the chapter of momentum, we can easily write the
thrust force.
(ii) Pressure at the other end can be calculated by
considering centripetal force on cylindrical liquid OR, one can simply write the force = rate at which
inside the tube. For writing centripetal force, we the outgoing gas is imparted momentum.
can assume the entire mass of the liquid at its
Let speed of outgoing gas (relative to the balloon) be v.
COM.
[Since the balloon is at rest, initial speed relative to balloon
(iii) After knowing pressure at the other end, we can will be same as speed relative to ground. You must not be
use Bernoulli’s equation between points just inside very fussy about it.]
and just outside the hole to find speed of ejected
1
liquid with respect to the tube. ​ __ ​  rv2 + P0 = P
2

÷ 
_________
2 (P – P0)
fi v = ​ ​ ________
    ​ ​ 
r
Volume flow rate, Q = Av

dm
Mass flow rate, ___
​   ​ = Q ◊ r = Arv
dt
dm
Let S = area of cross-section of the tube \ Fthrust = urel ◊ ​ ___ ​ 
dt
Mass of liquid, m = S ◊ x ◊ r
= v ◊ Arv = Arv2
Centripetal force needed for rotation of liquid is

(  ( 
= 2A (P – P0)

x
F = mw2 ​ L – __ ) x
​    ​  ​ = S r x w2 ​ L – __
2 )
​    ​  ​
2
Example 41   A pichkari (a water gun)
\ PS – P0S = F During the festival of Holi, we use a pichkari to drench others
with colour. One such pichkari has a horizontal cylinder
x
fi  P = P0 + rw2 x ​ L – __ ( 
​    ​  ​
2 ) fitted with a piston of area A. It contains water. The opening
has a small cross-section
Applying Bernoulli’s equation between a point just inside a. The piston moves
and outside the hole slowly under action of a
1 constant horizontal force
P + 0 + 0 = P0 + __
​   ​  rv2 + 0 F. Find the speed (v) at
2
which water erupts out of
( 
x
fi  rw2 x ​ L – __ ) 1
​    ​  ​ = __
2
​   ​  rv2
2
the mouth.
___________
fi   2 x (2L   
v = ​÷w – x) ​
8.40  Mechanics II

Solution (a) Consider a strip of width dy


on upper half of the gate.
Concepts
Area of strip dA = ady
(i) Piston is moved slowly. This implies that speed of Pressure at strip, P = rgy
liquid inside the cylinder is nearly zero.
[We need not consider
(ii) The applied force increases the pressure in the atmospheric pressure. It is acting
liquid. This high pressure energy gets converted on the gate from right side also.
into kinetic energy as the liquid is ejected. Its effect gets cancelled out.]
Force on strip,

dF = PdA = r ga y dy
a
Torque about O, d t = ​ __
2 (  ) a
​    ​ – y  ​ dF = rg a y ​ __
​   ​  – y  ​ dy
2 (  )
F \  clockwise torque on segment AO is
P1 = P0 + __
​   ​ ; v1 = 0
A
a/2 a/2

1
P2 = p0 ;
1
v2 = v

y = 0 0
( 
a
t1 = ​Ú   ​ ​ d t = rg a Ú​  ​   ​ ​ ​ __ )
​   ​  y – y2  ​ dy
2
P2 + __
​   ​  rv22 = p1 + __
​   ​  rv12
2 2
1
\ t1 = ___
​    ​ rg a4
1 F 48
P0 + __
​   ​  rv2 = p0 + __
​   ​ + 0
2 A
Similarly, torque on lower half can be calculated as

÷ 
___
2F
fi v = ​ ___
( 
​    ​ ​ 
rA ) a
dt = ​ y – __
​   ​   ​ dF
2

= rg a y ​( y – __
​   ​  )​ dy
Example 42   A square gate has dimension (a × a). It is a
hinged at the middle and 2
can rotate freely about a a
horizontal axis through its
centre. It is used to retain

a/2
( a
2 )
t2 = rg a ​Ú  ​ ​ y2 – ​ __ ​  y  ​ dy

water (density = r) on
5
one side. For equilibrium = ___
​    ​ r g a4
of the gate, a horizontal 48
force (F) is applied at its 1
lower edge. \ tnet = t2 – t1 = ___
​    ​ rg a4
12
(a) Find F
This torque must be balanced by the torque due to applied
(b) Find the force applied by the hinge on the gate.
force F.
Solution
a ___ 1 1
\ F __   ​    ​ rg a4  fi  F = __
​   ​ = ​   ​  rg a3
Concepts 2 12 6
(i) The pressure of water causes an anti-clockwise (b) Total force on the gate due to water is
torque on the gate about the hinge. This is equal to
torque on lower half minus the (clockwise) torque
on upper half of the gate. ( 
0 + rga 2 __
F0 = ​ ​ _______
2
1
)
 ​ a = ​   ​  rg a3
 ​  
2
(ii) Force F is applied to ensure rotational equilibrium.
Its value is obtained by making net torque about
the hinge equal to zero.
1
\  Hinge force, FH = F0 – F = ​ __
​   ​  –
2 (  1
__
)
​   ​   ​ rg a3
6
(iii) We also need to find total force applied by water 1
on the gate. This force is balanced by F plus hinge = __
​   ​  rg a3
3
force.
Fluid Mechanics  8.41 

_________
Example 43   Stable stick fi   
M + m ≥ ​÷M p L R2 
s ​ 
A wooden stick of length L and radius R (<< L) has density
_________
r. A small particle of mass m is
attached at its lower end and the
fi   
m ≥ ​÷M p L R2
 s ​ 
–M
stick floats in vertical position in _______________
water of density s. Find smallest fi   
m ≥ ​÷(r L p R2
) p L R
  
2
 s ​ – r L p R2
value of mass m for which the stick

[ ÷  ]
__
can be in stable equilibrium. s
fi  m ≥ p R2 L r ​ ​ __
​   ​ ​  – 1  ​
Solution r

[ ÷  ]
__
Concepts s
\  mmin = p R2 L r ​ ​ __
​   ​ ​  – 1  ​
r
(i) For stable equilibrium, the centre of buoyancy must
be above centre of mass.
Example 44   A thin triangular tube of uniform cross-section
(ii) Centre of buoyancy is at geometrical centre of the
has each of its side equal to l. The tube is in vertical plane
submerged part.
with one of its side horizontal. It contains equal volumes of
three immiscible liquids having densities r1, r2 and r3.
Find the length x of liquid of density r1 in the horizontal
limb.

Mass of stick, M = r L p R2


Solution
Distance of COM of stick + particle system from the
lower end is Concepts
L
M ​ __ ​  (i) Pressure at all points in the horizontal segment
______ 2 will be same, otherwise a segment of fluid will get
ycm = ​      ​
m+m accelerated.
Let length of submerged part be l. (ii) Each liquid covers length l along the perimeter.
Thus, liquid of density r1 must have a length (l – x)
(M + m) g = p R2 l s g
in side AB. It implies that liquid of density r2 has
(m + m) a length x in side AB and (l – x) in AC.
\ l = ​  _______  ​ 

p R2 s
l
Centre of buoyancy is at a distance __​    ​  from the lower end.
2
For stable equilibrium, the centre of buoyancy (B) should be
above the centre of mass (G).
l
\ ​ __  ​ ≥
  ycm
2
L
M __
( 
​   ​ 
1 M
__ +m
______
fi ​    ​ ​ ​  2  ​ 
2 p R  s ) 2
______
  ​ ≥ ​ 
M+m
    ​

fi  (M + m)2 ≥ M L ◊ p R2 ◊ s
8.42  Mechanics II

PB = PC 1 _____2x
fi ​ __ ​ =
  ​      ​  fi  x2 + 4x – 1= 0
2 1 – x2
r2 gh1 + r1 gh2 = r2 gh4 + r3 gh3
______
– 4 ± ​÷16
  + 4 ​
  __
fi  r2 x ◊ cos 30° + r1 (l – x) cos 30° fi x = ​  ___________
 ​
         – 2)
= (​÷5 ​
2
q __
= r2 (l – x) cos 30° + r3 x cos 30° \ tan ​ __ ​ =
  ​÷5 ​
   – 2 0.24
2
fi  r2x + r1l – r1x = r2l – r2 x + r3x
q ___________
0.24
\ sin ​ __ ​ =
  ​  __________

    ​ = 0.23
(r2 – r1) l (r1 – r2) l 2 ​ 1  2 + (0.24)
fi  x = ​ ___________
      ​ = ​ ___________
      ​ ÷ 2
    ​
2r2 – r1 – r3 r1 + r3 – 2r2

Example 46   A hemispherical cup


Example 45   Air bubble in accelerated tank A hemispherical cup of radius R
A closed spherical bulb of radius R is full of water. There is kept in water (density = r), as
is a small air bubble inside it. The shown. Find the vertical thrust
bulb is moved with a horizontal by the liquid on the concave
g surface of the cup.
acceleration a = ​ __   ​. Find the
2 Solution
displacement of the air bubble as
it moves to its new equilibrium Concepts
position. (i) Vertical force by water on upper half is upward and
Solution on the lower half is downward. Net vertical force
is downward.
Concepts
(ii) We can calculate this force by considering equilib-
(i) The air bubble has buoyancy force larger than its rium of a hemispherical volume of water inside the
weight. It remains buoyed at the top. tank.
(ii) As the bulb accelerates, the entire environment
inside the bulb sees an effective acceleration due Consider a hemispherical volume of
to gravity that makes an angle with vertical. water inside the water tank. Surrounding
(iii) Buoyancy and effective weight both act along the liquid exerts pressure on it from all
line of geff. The bubble settles in equilibrium in a sides. Since the volume of water is in
position, which is farthest from the centre along equilibrium, net force on it must be
the line of buoyancy. zero.
Vertically upward force on this
geff makes an angle q with vertical, hemispherical element due to surrounding
where water is equal to weight of water in the
1 hemisphere.
tan q = ​ __ ​ 
2
The bubble moves from A to B 2
\ FV = ​ __ ​  p  R3 r ◊ g
Displacement = AB 3


q
AB = 2R sin ​ __ (  )
​   ​   ​ = 2R (0.23)
2 Frow Newton’s third law, the hemispherical volume of
water applies equal vertical force on surrounding liquid in
= 0.46 R downward direction.
q
2 tan ​ __ ​  When the hemispherical cup is kept, the water molecules
2
Note: tan q = ________
​      ​ keep striking the surface in similar fashion. They do not
q
1 – tan2 ​ __ ​  differentiate between a cup wall and a “water wall”. Thus,
2 2
q
__ force on the inner surface of the cup will be ​ __ ​  p R3 r g in
Let   x
tan ​   ​ = 3
2 downward direction.
Fluid Mechanics  8.43 

Example 47   A uniform cylinder of Consider the system when the cylinder has moved down
mass 2m is connected to a block of by a distance x. Buoyancy on the cylinder is:
mass m though a string passing over a
smooth pulley. The system is released FB = Ax r g
from position where the lower end
If acceleration is a, we can write
of the cylinder just touches a liquid
surface in a tank. Length and cross- T – mg = ma ...(i) and
section area of cylinder are L and A
  2mg – Arg x – T = 2ma ...(ii)
respectively. The cylinder just stops at
the instant when its upper edge reaches Adding (i) and (ii) gives
the liquid surface. Find density of
liquid. Neglect viscosity. mg – Ar gx = 3ma
Solution
g Ar g
Concepts fi a = ​ __ ​  – ​ ____ ​ x = k1 – k2 x
3 3m
(i) Initially, the cylinder accelerates downwards.
Buoyancy force on it goes on increasing. At a g Arg
where k1 = ​ __ ​   and k2 = ____
​   ​ 
point, its acceleration becomes zero (when buoy- 3 3m
ancy becomes equal to mg) and thereafter, it turns
dv
negative. Finally, the cylinder stops. \ v ​ ___  ​ = k1 – k2 x
dx
(ii) The acceleration is changing. We need to express
acceleration as a function of displacement (x) of o o
K2L2
the cylinder. Then we will do integration to get fi ​Ú ​  ​v dv = ​Ú ​ ​(k1 – k2 x) dx  fi 0 = k1L – _____
​   ​  
the answer. Note that velocity becomes zero when o L 2
x = L.
k2L g Arg L
fi k1 = ___   fi ​ __ ​  =
​   ​   _____
​   

  
2 3 6 m

2m
fi r = ___
​    ​
AL
Worksheet 1
1. Equal masses of three liquids are kept in three (a) larger part in water (b) lesser part in water
identical cylindrical vessels A, B and C. The densi- (c) same part in water (d) it will sink
ties are rA, rB, rC with rA < rB < rC. The force on
6. A closed cubical box is completely filled with water
the base will be
and is accelerated horizontally towards right with
(a) maximum in vessel A an acceleration a. The resultant normal force by the
(b) maximum in vessel B water on the top of the box-
(c) maximum in vessel C
(a) passes through the centre of the top
(d) equal in all vessels
(b) passes through a point to the right of centre
2. The three vessels shown in figure have same base
area. Equal volumes of a liquid are poured in the (c) passes through a point to the left of the centre
three vessels. The force on the base will be (d) becomes zero
7. In a hydraulic lift, used at a service station, the radius
of the large and small pistons are in ratio 20:1. What
weight placed on the small piston will be sufficient
to lift a car of mass 1,500 kg?
(a) 3.75 kg (b) 37.5 kg
(a) maximum in vessel A (c) 7.5 kg (d) 75 kg
(b) maximum in vessel B
8. The pressure at the bottom of a tank of water is 3 P
(c) maximum in vessel C
where P is the atmospheric pressure. If the water is
(d) equal in all the vessels drawn out till the level of water is lowered by one fifth,
3. Figure shows a vertical cross-section of a vessel filled the pressure at the bottom of the tank will now be
with liquid of density r. The normal thrust per unit
(a) 2 P (b) (13/5) P
area on the wall of the vessel at point P, as shown,
will be (c) (8/5) P (d) (4/5) P
9. A boy carries a fish in one hand and a bucket (not full)
of water in the other hand. If he places the fish in
the bucket, the weight now carried by him (assume
that water does not spill):
(a) is less than before
(b) is more than before
(c) is the same as before
(d) depends upon his speed
(a) h r g (b) H r g
10. Two vertical cylindrical tubes contain mercury. They
(c) (H – h) r g (d) (H – h) r g cos q are connected by a horizontal tube at their bottom.
4. A beaker containing a liquid is kept inside a big The diameter of one tube is four times larger than the
closed jar. If the air inside the jar is continuously diameter of the other. A column of water of height
pumped out, the liquid pressure near the bottom of 70 cm is poured into the narrower tube. How much
the jar will will the mercury level rise in the other tube?
(a) increase (a) 0.3 cm (b) 4.8 cm
(b) decrease (c) 5.8 cm (d) 1.2 cm
(c) remain constant
11. A body of uniform cross-sectional area floats in a
(d) first decrease and then increase
liquid of density three times its own density. The
5. A piece of wood is floating in water kept in a bottle. portion of exposed height will be
The bottle is connected to an air pump. When more
(a) 2/3 (b) 5/6
air is pushed into the bottle from the pump, the piece
of wood will float with (Neglect the compressibility (c) 1/6 (d) 1/3
of water.)

8.44
12. The reading of a spring balance when a block is connected to it, as shown in figure. The weighing
suspended from it in air is 60 N. Reading changes pan will show a reading of
to 40 N when the block is submerged in water. The (a) 600 g (b) 550 g
specific gravity of the block must be (c) 650 g (d) 632 g
(a) 3 (b) 2 19. The area of cross-section of the wider tube shown in
(c) 6 (d) 3/2 figure is 800 cm2. The left end is open and right end
13. A hollow sphere of volume V is floating on water is covered with a massless piston. If a mass of 12 kg
surface, with half its volume remaining immersed. is placed on the piston, the difference in heights h in
What should be the minimum volume of water poured the level of water in the two tubes is:
inside the sphere so that the sphere now sinks into
the water?
(a) V/2 (b) V/3
(c) V/4 (d) V
14. A and B are two metallic pieces. They are fully
immersed in water and then weighed. Now they show
same loss of weight. The conclusion, therefore, is
(a) 10 cm (b) 6 cm
(a) A and B have same weight in air
(c) 15 cm (d) 2 cm
(b) A and B have equal volumes
20. A cylindrical beaker of cross-sectional area A contains
(c) the densities of the materials of A and B are the
water. Pressure exerted by water on the bottom
same
circular surface is P. A block of wood (relative
(d) A and B are immersed to the same depth inside density = 0.8) is put in the beaker and it floats. Its
water. mass is M. Now, the pressure exerted by water on
15. Two bodies are in equilibrium when suspended in the bottom surface of the container is P1. Then
water from the arms of a balance. The mass of one (a) P1 – P = 0.8 Mg/A (b) P = P1
body is 36 g and its density is 9 gcm–3. If the mass
(c) P < P1 (d) P1 – P = Mg/A
of the other is 48 g, its density in gcm–3 is:
21. A hemispherical portion of radius R is removed from
(a) 4/3 (b) 3/2
the bottom of a cylinder of radius R. The volume
(c) 3 (d) 5 of the remaining cylinder is V and
16. The mass of a balloon with its contents is 1.5 kg. It its mass is M. It is suspended by
is descending with an acceleration equal to half the a string in a liquid of density r,
acceleration due to gravity. If it is to go up with the where it stays vertical. The upper
same acceleration, keeping the volume same, its mass surface of the cylinder is at a depth
should be decreased by: h below the liquid surface. The
(a) 1.2 kg (b) 1 kg force on the bottom of the cylinder
(c) 0.75 kg (d) 0.5 kg by the liquid is
17. A hemispherical bowl just floats without sinking in a (a) Mg (b) Mg – V r g
liquid of density 1.2 × 103 kgm–3. If outer diameter
(c) Mg + p R2 h r g (d) rg (V + p R2h)
and the density of the bowl are 1 m and 2 × 104 kgm–3
respectively, then the inner diameter of the bowl will 22. A wooden block, with a metal
be (Given: (0.94)1/3 = 0.98) coin placed on its top, floats in
(a) 0.94 m (b) 0.97 m water as shown in figure. The
(c) 0.98 m (d) 0.99 m distances l and h are as shown.
18. A vessel with water is placed on a weighing pan After some time, the coin falls
and it reads 600 g. Now into the water. Then
a ball of mass 40 g and (a) l decreases and h
density 0.80 g cm–3 is increases
sunk into the water. It (b) l increases and h decreases
is held sunk inside water (c) both l and h increase
with the help of a pin
(d) both l and h decrease
of negligible volume

8.45
23. A cube of ice floats partly in bottom of the tank by two
water and partly in kerosene. wires, which make angle 45°
Find the ratio of the volume of with the horizontal, as shown
ice immersed in water to that in the figure. The tension T in
in kerosene. Specific gravity of each of the wires is
kerosene oil is 0.8 and that of 4
ice is 0.9. ​ __ ​  p R3 r w g – Mg
3
(a) ​ ______________ __ ​     ( b )
(a) 2 : 1 (b) 1 : 1 2   g  – Mg
​ __ ​  p R3 r ÷
​ 2 ​
w
(c) 3 : 2 (d) None of these 3
24. A cubical block of iron 6 cm on each side is float- 4
​ __ ​  p R3 r w g – Mg
ing on mercury in a vessel; water is poured into the 3 4
(c) ​ ______________     (d) ​ __ ​  p R3 r w g + Mg
 ​
vessel so that it just covers the iron block. What is 2 3
the height of water column? [Take relative density of 30. A cylindrical block of area of cross–section A and
Hg = 13.6 and that of Fe = 7.2] of material of density r is placed in a liquid of
(a) 4.5 cm (b) 4.75 cm density one–third the density of
(c) 3.05 cm (d) 2.5 cm block. The block compresses a
25. A liquid of mass 1 kg is filled in a spring and compression in the
flask as shown in figure. The force spring is one–third the length of
exerted by the flask on the liquid is the block. If acceleration due to
(g = 10 ms–2) [Neglect atmospheric gravity is g, the spring constant
pressure] of the spring is:
(a) 10 N (a) rAg (b) 2rAg
(b) greater than 10 N
(c) 2rAg/3 (d) rAg/3
(c) less than 10 N (d) zero
31. A dumbbell, having a light rod and two balls of
26. A U–tube having horizontal arm of length 20 cm, has
masses M and 2M (of equal volume V), is placed in
uniform cross-sectional area = 1 cm2. It is filled with
water of density r. It is observed that by attaching a
water of volume 60 cm3. What volume of a liquid of
mass m to the rod, the dumbbell floats with the rod
density 4 gcm–3 should be poured from one side into
horizontal on the surface of water and each sphere
the U-tube so that no water is left in the horizontal
exactly half submerged, as shown in the figure. The
arm of the tube?
volume of mass m is negligible. The distance l of
(a) 60 cm3 (b) 45 cm3 mass from the ball of mass M is
3
(c) 50 cm (d) 35 cm3
27. Two cylinders of same cross‑section and length L
but made of two material of densities d1 and d2 are
cemented together to form a cylinder of length 2 L.
The combination floats in a liquid of density d with a
length L/2 above the surface of the liquid. If d1 > d2
then:
3 d
(a) d1 > __ ​   ​  d (b) ​ __ ​  > d1
4 2
d
(c) ​ __ ​  > d1 (d) d < d1
4 d (Vr – 3 M) d (Vr – 2 M)
28. A small wooden ball of density r is immersed in (a) ​  ___________
    ​ (b) ​  ___________
    ​
2 (Vr – 2 M) 2 (Vr – 3 M)
water of density s to depth h and then released. The
height H above the surface of water up to which the
d (Vr +  2 M) d (Vr – 2 M)
ball will jump out of water is (c) ​ ___________
    ​ (d) ​  ___________
    ​
2 (Vr – 3 M) 2 (Vr +  3 M)

s h
(a) ​ ___   ​
r ( 
s 
(b) ​ __ )
​   ​  – 1  ​ h
r 32. A light semi-cylindrical gate of radius R is piovted
(c) h (d) zero at its mid-point O of the diameter as shown in the
figure, supporting a liquid of density r. The force F
29. A hollow sphere of mass M and radius R is immersed
required to prevent the rotation of the gate is equal
in a tank of water (density rw). The sphere would
to
float if it were set free. The sphere is tied to the

8.46
37. A cubical vessel with
edge L is placed on
a cart, which is mov-
ing horizontally with
an acceleration ‘a’ as
shown in figure. The
cube is completely filled with an ideal fluid having
density r. It is sealed so that no air remains inside it.
The pressure at the centre of the cubical vessel is
(a) 2 p R3r g (b) 2rgR3l L L
(a) ​ __ ​  r g (b) ​ __ ​  r (g + a)
2 R2 lrg 2 2
(c) _______
​   ​  
(d) none of these
3 L L
(c) ​ __ ​  r a (d) ​ __ ​  r (g – a)
33. A bucket contains water filled upto a height = 15 cm. 2 2
The bucket is tied to a rope, which is passed over a 38. There is a small hole near the bottom of an open tank
frictionless light pulley and the other end of the rope filled with a liquid. The speed of the water ejected
is tied to a block which has half the mass of bucket does not depend on:
+ water. The system is released to move. The gauge (a) area of the hole
pressure at the bottom of the bucket is
(b) density of the liquid
(a) 0.5 kPa (b) 1 kPa
(c) height of the liquid from the hole
(c) 5 kPa (d) None of these
(d) acceleration due to gravity
34. A cone of radius R and
39. An incompressible liquid flows through a Y-shaped
height H, is hanging inside
horizontal tube as shown in the figure. Then the
a liquid of density r by
velocity ‘v’ of the fluid in tube of cross-section
means of a string as shown
1.5 A is
in the figure. The force, due
to the liquid acting on the
slant surface of the cone
is (Neglect atmospheric
pressure)
(a) r pgHR2 (b) prHR2
4 2
(c) ​ __ ​  prgHR2 (d) ​ __ ​  p r gHR2
3 3
35. An open cubical tank was initially fully filled with (a) 3.0 ms–1 (b) 1.5 ms–1
–1
water. When the tank was accelerated on a horizontal (c) 1.0 ms (d) 2.25 ms–1
plane along one of its side, it was found that one-third 40. Water enters through end A with a speed v1 and
of volume of water spilled out. The acceleration was leaves through end B with a speed v2 of a cylindrical
(a) g/3 (b) 2 g/3 tube AB. The tube is always completely filled with
water. In case I, the tube is horizontal, in case II, it
(c) 3 g/2 (d) None
is vertical with the end A upward and in case III, it
36. A liquid of density r is is vertical with the end B upward. We have v1 = v2
contained in an open tube for
of shape shown in the
figure. The lower end of (a) case I (b) case II
the tube is closed by a (c) case III (d) each case
massless, smooth movable 41. A horizontal pipeline carries water in a steady flow.
piston of area A and the At a point along the pipe where the cross-sectional
upper end having area a is area is 10 cm2, the water velocity is 1 ms–1 and the
open to atmosphere. The force F needed to maintain pressure is 2000 Pa. The pressure of water at another
the piston in equilibrium is point where the cross-sectional area is 5 cm2, is (den-
(a) r g [ha – (H – h) A] (b) r g (H + h) A sity of water = 103 kg m–3).
(c) r g HA + r gha (d) r g (H – h) A (a) 200 Pa (b) 300 Pa
(c) 400 Pa (d) 500 Pa

8.47
42. Water from a tap (at the end of a horizontal pipe) 46. The cross-sectional area of a horizontal tube increases
emerges vertically downwards with an initial speed along its length linearly, as we move in the direction
of 1.0 ms–1. The cross-sectional area of the tap of flow. The variation of pressure, as we move along
is 10–4 m2. Assume that the pressure is constant its length in the direction of flow (x-direction), is best
throughout the stream of water and the flow is steady. depicted by which of the following graphs?
The cross-sectional area of the stream 0.15 m below
the tap is:
(a) 5.0 × 10–4 m2 (b) 1.0 × 10–5 m2 (a) (b)
–5 2
(c) 5.0 × 10 m (d) 2.0 × 10–5 m2
43. A large open tank has two holes in the wall. One is
a square hole of side L at a depth y from the top and
the other is a circular hole of radius R at a depth 4 y
from the top. When the tank is completely filled with
(c) (d)
water, the quantities of water flowing out per second
from both holes are the same. Then, R is equal to:
L 47. A vertical tube is attached to a closed vessel containing
(a) ​ ____   ​  
___ (b) 2 p L
÷    
​ 2 p ​ water. The vessel has rarefied air above the water sur-
L face. Lower end of the tube is held inside water in a
(c) L (d) ​ ___  ​ 
2 p position where the air bubbles start appearing at the
tip of the tube. The velocity
44. A pipeline is inclined at an angle q to the horizontal.
of water coming out from a
A liquid of density r flows through it. A mercury
small hole in the side wall of
manometer is connected as shown in the figure. The
the container at a depth h = 20
difference in height D h for the levels of manometer
cm from the tip of the tube
liquid is: (symbols have usual meaning) (rHg >> r)
is
__
(a) ​÷ 2 ​  ms–1
(b) 2 ms–1
(c) depends on pressure of air inside vessel
(d) None of these
48. A large tank is filled
with water (density
= 10 3 kgm –3 ). A
r d cot q r d tan q small hole is made at
(a) ​ _______
     
​ (b) ​ _______    
​ a depth 10 m below
r Hg r Hg
water surface. The
r d sin q horizontal range of
(c) ​ _______    
​ (d) none of these
r Hg water issuing out of
45. An ideal fluid flows through a tube of uniform cross- the hole is R. What extra pressure must be applied on
section. A and B are two points in the vertical section the water surface so that the range becomes 2 R (take
of the tube as shown. Liquid velocities are nA & nB 1 atm = 105 Pa and g = 10 ms–2)?
and pressures are PA & PB at the two points. Which (a) 9 atm (b) 4 atm
of the following is true? (c) 5 atm (d) 3 atm
49. A cyclindrical vessel of cross-sectional area
1000 cm2 is fitted with a frictionless piston of mass
10 kg, and filled with water completely. A small hole
of cross-sectional area 10 mm2 is opened at a point
50 cm deep from the lower surface of the piston. The
velocity of efflux from the hole will be
(a) 10.51 ms–1 (b) 3.46 ms–1
(a) PB > PA (b) PB < PA (c) 0.81 ms–1 (d) 0.21 ms–1
(c) PA = PB (d) nA > nB

8.48
 Worksheet 2
1. The spring balance A reads 2 kg
with a block m suspended from
it. A balance B reads 5 kg when
a beaker with liquid is put on the
pan of the balance. The two bal-
ances are now so arranged that
the hanging mass is inside the
liquid in the beaker, as shown in
the figure. In this situation,
(a) the balance A will read more than 2 kg. (a) The liquids are certainly not in static equilibrium
(b) the balance B will read more than 5 kg. in case II only.
(c) the balance A will read less than 2 kg and B (b) If the liquids are in equilibrium then the density
will read more than 5 kg. of the grey liquid must be greater than the
(d) the balances A and B will read 2 kg and 5 kg density of the black liquid in case IV only.
respectively. (c) The liquids are certainly accelerated in case
2. Figure shows a siphon. Choose the wrong statement. II.
(d) The liquids are certainly accelerated in all cases,
(P0 = atmospheric pressure)
except III.
5. A cylinder of radius R is kept embedded along the
wall of a dam as shown. Take density of water as r
and length of the cylinder as L. Neglect atmospheric
pressure

(a) Siphon works when h3 > 0


(b) Pressure at point 2 is P2 = P0 – r gh3
(c) Pressure at point 3 is P0
(d) Siphon will always work, regardless of the value
of h1 (a) The vertical force exerted by water on the
3. Water coming out of a horizontal tube at a speed n r p R2 Lg
cylinder is ​ ________
 ​  
strikes normally on a vertical wall close to the mouth 2
of the tube and falls down vertically after impact. (b) The torque exerted by liquid on the cylinder
When the speed of water is increased to 2n, r R3 Lg
(a) the thrust exerted by the water on the wall will about its axis is ______
​   ​   
2
be doubled. (c) The force exerted by liquid on the cylinder in
(b) the thrust exerted by the water on the wall will horizontal direction is 4 R2rgL
be four times. (d) The force exerted by liquid on the cylinder in
(c) the energy lost per second by water striking the horizontal direction is 2 R2rgL
wall will also be four times. 6. A wooden block (having
(d) the energy lost per second by water striking the cross‑sectional area A), with
wall will be increased eight times. a coin (having volume V and
4. The figure below shows four situations in which a density d) placed on its top
grey liquid and a black liquid (about which we known floats in water, as shown in
nothing) are in a U-tube. figure. Cross sectional area
of the container is A1 &

8.49
density of water is r. If the coin is lifted and then The water leaves the second section of radius
dropped into water. r2 = 1 cm with speed v2. Assume the flow is fric-
(a) change in the submerged length (l) of the block tionless, irrotational, and steady. Also, assume the
V r reservoir is so large compared to the pipe, that the
is D l = ___
​   ​  water level in the reservoir is almost constant.
Ad
(a) the speed, v2, of the water leaving the exit pipe
(b) change in the submerged length (l) of the block
is 10 ms–1
V d
is D l = ___
​    
​ (b) the speed, v1, in the first section of the pipe is
A r 2.5 ms–1
(c) change in the water level (h) in container is (c) the height, h, of the water in the vertical tube
V
Dh = ___ (  ) d
​    ​ ​ 1 – __
A1
​   ​  ​.
r
is nearly 4.69 m
(d) the volume flow rate of the water flowing
through the exit pipe is 3.1 ms–3
(d) None of the above
7. A cylindrical container of 9. A large tank (height h1) of water has a hose connected
length L is full to the brim to it, as shown in figure. The tank is sealed at the
with a liquid, which has top and has air at atmo-
density r. It is placed on a spheric pressure between
weigh‑scale, which reads the water surface & the
W. A light ball (which top. Initially, height of
would float on the liquid if allowed to do so) of water in the tank is h2 >
volume V and mass m is pushed gently down and h3. Assume that the air
held beneath the surface of the liquid with a rigid pressure above the water decreases with expansion
rod of negligible volume, as shown. of air. Water begins to flow out of the hose.
(a) the mass M of liquid, which overflowed while (a) flow of water will stop when height of water
the ball was being pushed into the container is surface becomes equal to h3
2 rV (b) flow of water will stop when height of water
(b) the reading R1 on the scale when the ball is fully surface is more than h3
immersed is W (c) flow speed will decrease with time
(c) If instead of being pushed down by a rod, the (d) none of the above
ball is held in place by a fine string attached to 10. A balloon filled with air is
the bottom of the container, the tension T in the tied to a weight, as shown
string is T = rVg – mg in figure. It barely floats in
(d) as described in part (c), the ball is held in water while remaining com-
place by a fine string attached to the bottom pletely submerged. The string
of the container. The reading R2 on the scale is is taut in the position shown.
W – rVg + mg It is pushed down so that it
8. Water fills a reservoir, open to the atmosphere, to height gets submerged a short dis-
H = 5 m above the horizontal exit pipe at the bottom tance in water.
of the reservoir. The first section of the pipe has (a) If the string breaks, the balloon will move up.
radius r1 = 2 cm, the unknown speed there is v1 and (b) If the weight is pulled downward so as to take
this section of the pipe has a vertical tube conneted the system deep and released, the system will
to it in which the water rises to an unknown height sink further.
h above the the pipe.
(c) If the weight is pulled downward so as to take
the system deep and released, the system will
remain in the position it is left.
(d) None of the above
11. An open pan P filled with water (density = dw) is
placed on a vertical rod, maintaining equilibrium. A
block of density d is gently placed on one side of
the pan as shown. Water depth is more than height
of the block.

8.50
Y
(c) the specific gravity of sea–water is ________
​       ​
Y+X+Z
(d) none of the above
14. A uniform solid cylinder of density 0.8 gcm–3 floats
in equilibrium in a combination of two non–mixing
liquids A and B with its axis vertical. The densities
(a) Equilibrium will be maintained only if d < dw of the liquids A and B are 0.7 gcm–3 and 1.2 gcm–3,
(b) Equilibrium will be maintained in case d = dw respectively. The height of liquid A is hA = 1.2 cm.
The length of the part of the cylinder immersed in
(c) Equilibrium will be maintained for all relations
liquid B is hB = 0.8 cm.
between d and dw
(d) Equilibrium will not be maintained in case
d < dw
12. Two bulbs X and Y contain gas at different pressures.
A long tube having horizontal and vertical segments
is connected to the bulbs as shown. The tube contains
alternate segment of water and mercury. Water just
could not enter the two bulbs. Specific gravity of
mercury is 13.6
(a) the total force exerted by liquid A on the cylinder
is zero
(b) the length of the part of the cylinder in air is
0.25 cm
(c) The cylinder is depressed in such a way that its
top surface is just below the upper surface of
liquid A and is then released. The acceleration
of the cylinder immediately after it is released
is g/6
(a) Pressure in two horizontal segments of mercury (d) none of the above
must be same.
15. A hemispherical bowl of mass m made of steel of
(b) Pressure difference between X and Y is nearly
density d is floating on the surface of water
248 kPa.
(density = r).
(c) Pressure difference between X and Y is 148 kPa.
(a) volume of water displaced is m/r
(d) Equilibrium is not possible in the given arrangement.
(b) the bowl is pushed down so as to submerge
13. A ship sailing from sea into a fresh water river further it deep inside water and released. Its initial

(  )
sinks X mm and on discharging, the cargo rises Y r
mm. On proceeding again into sea, the ship rises Z acceleration has magnitude g ​ 1 – ​ __ ​  ​
d
mm. The ship has uniform horiontal cross-sections.
(c) if the bowl is inverted, it will sink with an

(  )
Y
(a) the specific gravity of sea–water is _________
​       ​ r
Y – X  +  Z acceleration of g ​ 1 – __
​   ​  ​
d
(b) X = Z
(d) none of the above

8.51
 Worksheet 3
1. A cubical block of wood of side length 3 cm floats arm. Now, water is poured into the longer arm. What
in water. The lower surface of the cube just touches should be height of water (h) in the longer arm so that
the free end of a vertical spring fixed at the bottom the water begins to leak from the mouth of shorter
of the tank. Find maxi- arm? The area of top surface of seal is A0 = 0.015 m2
mum weight that can be and atmospheric pressure is P0 = 1 × 105 Nm–2.
put on the block with-
out wetting its upper
surface (it means that
the block is just not
completely submerged).
Density of wood d = 0.8 gcm–3, force constant of
spring k = 50 Nm–1 and g = 10 ms–2.
2. A piece of brass (an alloy of copper and zinc) weighs
12.9 g in air. When completely immersed in water, it
weighs 11.3 g. Find the mass of copper in the allow?
Specific gravities of copper and zinc are 8.9 and 7.1
respectively.
3. A cubical block of side length 5 cm is made of 6. A cubical box is constructed using iron sheets 1mm
iron (density = 7.2 gcm–3). It is floating in mercury in thickness. Density of iron is = 8 gcm–3 and density
(density = 13.6 gcm–3). of water is 1 gcm–3.
(a) Find the height of block above the mercury (a) Will the box sink in water if side length of the
level. cube is large?
(b) Water is poured into the vessel (above mercury) (b) What value of external side length of the box
so that it just covers the iron block. Find the will you recommend if the box is supposed to
height of water column. float in water.
4. A small solid ball is released from rest, deep inside 7. A concrete sphere has a cavity packed with foam.
a liquid tank. Its kinetic energy (k0) is measured after The specific gravities of the concrete and foam are
it travels through a distance of 8  cm in the liquid. 2.4 and 0.3 respectively. The sphere floats in water
Experiment is repeated with many non-viscous while remaining completely submerged. Find the
liquids. k0 is plotted versus density (rl) of liquids. ratio of mass of concrete to the mass of foam in the
Graph is shown in figure. sphere.
8. A 10 cm side cube weighing 5 N is immersed in a
liquid of relative density 0.8 contained in a square
tank of cross-section area 15 cm × 15 cm. The tank
contained liquid to a height of 8 cm before immer-
sion. Find the height of liquid surface in the tank
after the cube is immersed. Is the cube touching the
base of the tank? g = 10 ms–2.
9. A uniform rod OB is hinged
at O inside water tank so
that it can rotate freely in the
(a) Find mass of the ball. vertical plane. Length of the
(b) Find density of a liquid, which is heavier than rod is L = 1.0 m, its relative
water, in which k0 will be equal to 4 J. [g = 10 density is 2.0 and cross-sec-
ms–2] tional area is A = 0.012 m2.
5. A J-shaped tube has a uniform cross-sectional area It is fully submerged in water. Its end B is tied to a
of A = 0.01 m2. Its shorter arm has length of 0.8 m horizontal sting AB, which can sustain a maximum
and the other arm is quite tall. A steel plate of mass tension of 45 N. Find the maximum angle q that
M = 10 kg is used to seal the opening of shorter

8.52
the rod can make with vertical without breaking the
string.
10. A horizontal tube having length L is filled with a
liquid of density r. It is rotating about a vertical axis
as shown. Find pressure difference between point B
and A.

14. A uniform aluminum ball of radius r = 0.5 cm is sus-


pended on a weightless thread from the end of a uni-
form rod of mass M = 4.4 g. The rod is balanced at
the edge of the container in
a horizontal position. In the
equilibrium position shown,
half the ball is submerged.
11. The device shown in the figure is a simple pressure Find the ratio of length x
gauge. Area of the light and smooth piston is 1.0  cm2 and y. Neglect surface ten-
and force constant sion and take relative den-
of the spring is sity of aluminum to be 3.0.
k = 100 Nm–1. The 15. A water tank has water filled into it up to a height
chamber inside h. The tank is placed on the edge of a table that is
which the spring H high. A small orifice is made at a depth y below
lies has no air in the water surface in the side wall of the tank. For
it. The wall of the which value of y is the range x maximum, if
container is marked
with a mm Scale. The zero of the scale is at the
position of the piston when the device is kept in
atmosphere.
(a) Find the smallest gauge pressure that can be
measured using this device.
(b) Find the displacement of the piston if the device
is taken at a depth of 2m inside a swimming
pool.
12. A candle of cross-sectional area a = 1 cm2 and length
L = 20 cm is floating in equilibrium in a tank of
2 4
water having cross-sectional area A = 2 m2. Half the (a) H = __
​   ​  h  (b) H = __
​   ​  h
3 3
candle is submerged in this position. The candle
burns at a rate of 2 cmh–1. The burnt part is com- 16. Water in a large dam is 10 m deep. There is a
pletely converted into gas and goes to atmosphere. horizontal pipe of diameter 4.0 cm fitted in the dam
Calculate the rate of fall of top of the candle in unit wall at a depth of 6 m from the water surface. A plug
of cm/hour. in the pipe keeps the water from flowing out.

(a) Find friction force between the plug and the


13. Find force exerted by water on the slant wall shown pipe wall.
in figure. The wall makes an angle q with vertical
(b) If the plug is removed, what volume of
and its width perpendicular to the figure is b. Density
water will drain out of the pipe in 24 hours?
of water is r. Neglect atmosphere.
[g = 9.8 ms–2]
8.53
17. A cylindrical tank of base area A has a small hole is A2 and is kept constant. The cross-sectional area
of area a at its bottom. At time t = 0, a tap starts to (A1) of section – 1 is varied during an experiment
supply water into the tank. The cross-sectional area and the pressure difference between two segments
of the outlet of the tap is a0 and water flows out from (D P = P2 – P1) is measured. The graph of DP verus​

(  )
it at a speed v. Find the maximum level of water that 1
___
​  2  ​   ​ is as shown. Volume flow rate is held constant
one can have in this leaking tank.
A1
18. A siphon tube is used to discharge a liquid of relative
density 0.9 from a large reservoir as shown. during the experiment.


(a) Find A2
(a) Find speed of flow of liquid through the pipe
(b) Find the volume flow rate.
(b) Find pressure at highest point B.
21. A rectangular vessel when full of water takes
(c) Find pressure at A (just outside the tube 10 minutes to be emptied through an orifice at its
mouth) bottom. How much time will it take to be emptied
(d) Find pressure at A just inside the tube. if it is only half-filled?
(e) Would the rate of flow be more, less or same 22. A cylindrical tank has cross-sectional area A. Mass
if the liquid were water? of empty tank is M. It contains a liquid of density 2r
[Atmospheric pressure is P0 = 105 Nm–2] filled upto height 2h and another immiscible liquid
of density r upto a further height 2h. A small hole
19. Steady flow of water is happening through a pipe of
of cross-section S is made in the wall of the cylinder
variable cross-section. Area of cross-section of the
at a height h above the horizontal table. Find the
pipe at A and B are A1 = 4 × 10–3 m2 and A2 = 8 ×
minimum coefficient of friction between the cylinder
10–3 m2 respectively. Height of pipe at A and B above
and the ground so that it does not slide immediately
the reference level is 2 m and 5 m. Speed of flow
after the hole is opened and the liquid jet discharges
at A is v1 = 1 ms–1. Density of water is 103 kgm–3.
horizontally.
Find work done by surrounding liquid on a liquid
element of unit volume as it moves from A to B.
[g = 10 ms–2]

23. A water tank has shape shown in figure. Speed


of flow at the horizontal segment of cross-section
A2 is v. Height h1 and h2 are given. Find ratio of
20. Water flows through a horizontal pipe, which has two cross-section (A2) of the horizontal segment to that
sections – 1 and 2. Cross-sectional area of section 2 of the vertical segment (A1).

8.54
24. A tube is held vertically dipped in water in a beaker.
A fan is put on, which causes air to blow over the
tube. Density of air is 1.3 gcm–3. After the fan is
switched on, the water rises in the tube to a height
of h = 1 cm. Find the speed of air near the mouth of
the tube.

8.55
Answers Sheet
Your Turn

1. 6.8 kg 2. 7.3 gcm–3 3. Both are same 4. 10 N


5. (a) 7500 N 6. 6250 N 7. a = 9.26 m 8. (a) 4N (b) 1N
9. 76 cm 10 Equator 11. 1 m 12. **
13. 0.67, 4,000 Nm–2 14. 1.068 × 105 Pa 15. (a) A (b) B (c) 10.74 cm of Hg
16. 55 N 17. 122 N 18. (a) 4 × 104 Pa (b) 8 × 103 Pa (c) – 4.8 × 104 Pa

19. 0
1
20. 3 × 105 Pa
4 5
( 
L
21. P1 = P0 + rg ​ H – __ )
​   ​   ​; L = 2H
2
22. ​ __ ​  × 104 kgm–3 23. ​ __ ​  r1 24. ​ __ ​   25. 80 kg
3 3 6
26. 30 g 27. 133 kg 28. Level of water falls 29. 4 g
30. No 31. 19.6 m 32. 0.12 m3 33. 3.6 cm
34. Sphere will rotate. 35. m 36. V3 = 1.0 ms  ; VP = 0.05 ms–1
–1

40. 81,900 N 41. 10 ms–1 42. (a) 50 cms–1 (b) 93.75 Pa


____ ____________
  1 ​ 
43. ​÷2gh  
44. ​÷2g (H – l sin q) ​
  

Worksheet 1
1. (d) 2. (c) 3. (c) 4. (b) 5. (c) 6. (c) 7. (a) 8. (b) 9. (c)
10. (a) 11. (a) 12. (a) 13. (a) 14. (b) 15. (c) 16. (b) 17. (c) 18. (c)
19. (c) 20. (d) 21. (d) 22. (d) 23. (b) 24. (c) 25. (a) 26. (d) 27. (a)
28. (b) 29. (a) 30. (b) 31. (b) 32. (d) 33. (b) 34. (d) 35. (b) 36. (b)
37. (b) 38. (b) 39. (c) 40. (d) 41. (d) 42. (c) 43. (a) 44. (b) 45. (a)
46. (a) 47. (b) 48. (d) 49. (b)

Worksheet 2
1. (b, c) 2. (d) 3. (b, d) 4. (a, b, c) 5. (a, c) 6. (b, c) 7. (b, c, d) 8. (a, b, c)
9. (b, c) 10. (a, b) 11. (b, d) 12. (b) 13. (a) 14. (a, b, c) 15. (a, b, c)

Worksheet 3
1. 0.35 N 2. 7.6 g 3. (a) 2.35 cm (b) 2.54 cm
–3
4. (a) 5 kg (b) 3.0 gcm 5. 6.8 m 6. (a) No (b) Side length > 4.8 cm 7. 4
1
8. 97/9 cm = 10.8 cm 9. 37° 10. ​ __ ​  rw2 L2
6
1
11. (a) 1 kPa (b) 2 cm 12. 1 cmh–1 13. ​ __ ​  r g bh2 sec q 14. 0.46
2
5h a02 v2
15. (a) ___
​   ​ (b) h 16. (a) 74 N (b) 1175 m3 17. ​ _____2 ​ 
6 2g a
18. (a) 10 ms–1 (b) 41.5 KNm–2 (c) 0.9 × 105 Nm–2 (d) 4.5 × 104 Nm–2 (e) same 19. 29625 J
16 rSh
20. (a) 0.35 m2 (b) 12.25 m3s–1 21. 7.07 min 22. ​ _________
   ​
M + 6hAr

÷ 
______________
2g (2h1 – h2)
23. ​ 1 – ​ ___________
   ​ ​
     24. 12.4 ms–1
v2

8.56
Chapter  9

Surface Tension and Viscosity


“You get surface tension with water. That helps the sand grains stick together.”
–Mark Mason

1.  INTRODUCTION F be the magnitude of forces exerted by two sides on one


another perpendicular to line AB. Surface tension S of the
A liquid surfaces has properties very different from its bulk. liquid is defined as
Surface of a liquid is like a stretched membrane–it is under
tension. Surface tension of water–the most abundant liquid–is
higher than most other liquids. Many small insects live in a
world dominated by surface tension of water. Inkjet printing,
design of automobile windshields, many medicals devices,
rise of water from roots to leaves in a tree, etc., are just a
few examples where surface tension plays a vital role.
We studied Bernoulli’s equation in the last chapter. It is
applicable for steady flow of ideal fluids. But all real life
liquids are viscous. Layers of liquids exert tangential force
on one other. Viscosity is basically a measure of stickiness
of a liquid. It plays an important role in deciding the nature
of flow and design of mechanical systems using liquid
lubricants–such as a car engine and gearbox.
F
S = __
​   ​ ...(1)
L
2. TENSION IN A LIQUID SURFACE
SI unit of surface tension is Nm–1.
Surface of liquid is like a stretched sheet and can bear load. Surface tension of few common liquids are:
light insects, like mosquitores, can walk on the surface of Water: 0.075 Nm–1; Soap solution: 0.03 Nm–1;
water. A steel (relative density  8) needle can remain afloat
Glycerine: 0.06 Nm–1; Mercury: 0.465 Nm–1.
on the surface of water if placed properly.
Surface tension of water is larger than most of the
Think of a large bedsheet held at many points along its
common liquids but it is less than surface tension of metallic
periphery by many of your friends. They all pull the sheet
liquids like mercury.
towards themselves. The sheet is under tension. You can go
and keep a heavy ball on the sheet. A depression is formed When a solid boundary comes into contact with a liquid
and tension force acting on the ball from all sides has a surface, the liquid surface pulls it tangentially-just like the
vertical resultant, which balances the weight of the ball. A stretched bedsheet pulls all your friends towards itself.
liquid surface is somewhat like this stretched sheet. If you The surface tension force will be obtained by multiplying
imagine a line on the sheet dividing it into two parts, each length of the boundary (L) in contact with the surface
part pulls the other one towards itself. tension (S).
Consider a line AB of length L on the surface of a liquid. When a liquid surface is curved, a line draw on it will
The two parts of liquid surface (one on right of the line and be curved. Surface tension force is tangential everywhere.
the other on left of it) pull one another with a force that is Consider a spherical water drop. Imagine its equator. The
perpendicular to line AB and tangential to the surface. Let equator divides the entire spherical surface into two halves.
9.2  Mechanics II

Each half exerts force on another half pricking. If there were no thread and you prick the film, the
having magnitude. entire film will collapse.
F = S  ◊ 2p R, where R is radius of Example 1   Be careful, there are two surfaces
drop. Figure shows surface tension
In the example cited above, after the soap film inside the
force by lower half on the upper half.
thread loop is pricked, find the surface tension force acting
Force is tangential to the surface. In
on a small segment of thread having length D l. Surface
the given figure, it is downward. Figure shows
surface tention force tension of soap solution is S.
Force by upper half on the lower half
by lower hemispherical
is equal and opposite. part on the upper part. Solution
Concepts
2.1  Demonstration of Surface Tension There are two surfaces in the soap film.
A liquid surface has a property of surface tension. This fact
can be demonstrated using many simple experiments. Here, Surface tension force on segment of length D l is D l s.
we take up a few of those. Since, there are two surfaces of the liquid in contact with
the thread, the force will be
(i)  Loop of thread on soap film
Take a rectangular frame of wire and dip it is soap solution. D F = 2 D l s
When you take out the frame from the solution, a film of
soap solution bounded by the wire frame is formed. Now (ii)  Sliding wire on U-shaped frame
take a loop of light cotton thread, wet it and gently place A U-shaped wire frame has a light wire AB mounted on it so
it on the film. The loop will stay in the shape it has been that it can slide smoothly. If this frame
placed. Now prick a hole is dipped in a soap solution and taken
in the film inside the out, a film of soap solution is fomed in
loop, with the help of a the rectangular region. If the frame is
needle. If done carefully, kept horizontal, the sliding wire moves
the film inside the thread towards the parallel arm of the frame.
loop will burst but the The force of surface tension applied by the liquid on the
film outside the loop wire causes it to move.
remains intact. After the
inner film is bursted, Example 2   The U-frame and sliding
the thread loop takes a wire system in the above discussion is
circular shape. held vertical. Now, a mass m is hung
from the wire AB to prevent it from
Initially, there were liquid surfaces on both side of the
sliding. There is no friction, find m if
thread. A small segment of
length of AB is 4 cm and surface ten-
the thread was pulled by
sion of soap solution is S = 0.03 Nm–1.
the force of surface ten-
Wire AB has no mass. [g = 10 ms–2]
sion from inside as well
as outside. The two forces
Solution
(shown as D F in figure)
have equal magnitude and Concepts
opposite direction–perpen- Surface tension force on AB balances the weight
dicular to the segments of
the thread. All segments of
Surface tension force on the wire AB is
the thread experience zero
force and stay in their original position. F = 2lS [There are 2 surfaces]
When the inner soap film is gone, every segment of the
= 2 × 0.04 × 0.03 = 2.4 × 10–3 N
thread is pulled from outside only. Due to uniform pulling
by the liquid surface from all sides, the thread loop acquires For equilibrium: mg = 2.4 × 10–3
circular shape.
It is important to note that a film of liquid can survive fi m = 2.4 × 10–4 kg
only if it has a solid boundary to hold. The wire frame and
the thread from the boundary for the liquid surface after = 0.24 g
Surface Tension and Viscosity  9.3

(iii)  A needle on surface of water Example 3   Maxmium mass of needle that can stay on
Take a glass of water and keep a small piece of tissue paper surface of water.
on its surface. Now place a steel nee-
In the above discussion, length of the needle is l. What is
dle on it. Wait for some time. The
the maximum mass of needle that can stay on the surface of
tissue paper soaks water and sinks.
water? Take l = 4 cm, surface tension of water S = 0.075 Nm–1
However, the needle remains afloat.
and g = 9.8 ms–2.
The needle is not floating in usual
sense. It is not buoyancy that prevents Solution
if from sinking. Density of steel is
Concepts
almost eight times that of water. It is
the force of surface tension that keeps Angle q = 90° in extreme case.
it in quilibrium.
The stretched surface of water gets depressed due to load 2F = Mg
of needle. The surface tension force (tangential to the surface
is directed as shown. The force of surface tension acts on 2 l S = Mg
two sides. If l is length of the cylindrical needle, then
F = l S
For equilibrium,
2 F sin q = Mg
2 l S sin q = Mg
If the needle is made
heavier without changing its
length, then q increases to
balance the weight. It means 2 × 0.04 × 0.075
2 l s _______________
the depression becomes fi M = ​ ____​ = ​ 
g     ​
    
9.8
deeper. In extreme case, q
can become 90°. = 6.1 × 10– 4 kg = 0.61 g

Your Turn

Q.1  Water is kept in a cylindrical glass of diameter 10 cm. Q.2  A circular disc of radius 5 cm is placed on the
Consider a diameter on the circular surface of the water. Find surface of a liquid. It is slowly pulled upward, keeping the
the surface tension force by which the surface on one side disc surface horizontal. In addition to the ring’s weight, an
of the diameter pulls the surface on the other side. surface upward force of 3 × 10–2 N is required to lift the disc to the
tension of water is S = 0.075 Nm–1. point where it just breaks free of the surface. what is the
surface tension of the liquid?

3. TENDENCY OF A LIQUID SURFACE TO Large drops are not


perfectly spherical due
CONTRACT to effect of gravity. In
A stretched rubber sheet has a tendency to contract. Similarly, a gravity-free space,
stretched surface of a liquid has a tendency to contract. A all water drops (however
liquid always tries to keep its surface area to minimum large) will be spherical.
possible. Following example demonstrates this property of (ii) When a printing brush
a liquid surface. (or shaving brush) is dipped in water, its hairs spread
(i) Small waterdrops are spherical. For a given volume, out and move freely. When it is taken out of the
a sphere has the least surface area. water, the hairs stick together. Sticking of hairs
9.4  Mechanics II

minimises the surface of water. The water particles (iii) Demonstration titled as ‘loop of thread on soap film’
come together to reduce their overall surface area. in the last article also illustrated the fact the liquid
has the tendency to minimise its surface area.
After the inner soap film is burst, the thread takes a
circular shape. For a given perimeter of a plane fig-
ure, a circle has maximum area. Therefore, circular
shape of the thread means that the part where there
is no film (and has air) has maximum area. Thus, the
area covered by the soap film is minimum possible.
(iv) ‘Sliding wire on U-shaped film’ also demonstrates
that liquid will always try to reduce its surface area.
The sliding wire AB moves to reduce the area of the
film.

Your Turn

Q.3  An ice cube is kept in a gravity-free hall. What will be its shape when it melts?

4. MICROSCOPIC DESCRIPTION OF A molecule like C, which is a few layers below the top
layer, also experiences a net force towards the bulk. There
SURFACE TENSION are fewer molecules above C to attract it.
A liquid molecule that is very close to other molecule will Any molecule in the top layer of 10 to 20 molecular
repel it. Molecules, which are slightly away from a given diameter thickness will experience a net force directed into
molecule, will attract the molecule. And molecules which are the bulk of the liquid. This zone is known as the surface of
far away (usually 10 to 20 molecular diameter away) will the liquid. If we go 20 molecular layers deep, a molecule
have no interaction with the molecule. finds equal force from all sides. net force on it is zero.
A molecule A interacts with Less and less number of liquid molecules will be there
other molecules, which lie in its on the surface. They are being pulled into the bulk. For
r
sphere of influence. Its sphere of this reason, a liquid has the least surface area and a tension
influences has radius of the order A develops in its surface.
of 10 to 20 molecular diameter.
A molecule inside the bulk of a 5.  SURFACE ENERGY
liquid (molecule A) feels uniform Molecules outside the
sphere of influence do not Force between a liquid molecule and other surrounding
attractive forces from all sides due interact with molecule A.
to liquid molecules present around molecules is attractive (unless the molecules get extremely
it. Net force on it is zero. close). Attractive force means negative potential energy. The
A molecule on the top layer of liquid surface interior molecules have as many neighbours as they can
(like molecule B) feels uniform attraction due to molecules have and thereby, have a large negative potential energy. A
below it but above it there are air molecules or molecules of molecule on the surface has less number of neighbours. [A
liquid vapour, which have a small density and exert almost molecule like B, in last diagram, has no neighbor above it].
no force on B. Thus, B experiences a net force directed into Potential energy of such molecules is higher. [A negative
the bulk (interior) of the liquid. number but lesser magnitude] molecules on a liquid surface
have higher energy compared to molecules in the bulk.
Another way to understand this is that when a molecule
moves up from inside of the liquid to its surface, it has
moved up against an inward force while travelling through
layers in the surface zone. It needs energy (from somewhere)
to put extra molecules in the surface of a liquid (and thereby
increase the surface area).
Surface Tension and Viscosity  9.5 

You need to work to lift a book against gravity. Similarly, New surface area created is A = 2 l x
molecules need energy (from somewhere) to rise to the Thus, excess energy possessed by molecules on unit area
surface against the inward pull. Molecules on the surface of liquid surface is
must have this extra energy. The extra energy that a surface
W _____2 l S x
layer has is called the surface energy. ​ __ ​ =
  ​    ​ = S
  ...(2)
A 2l x
5.1  Surface Tension as Surface Energy
Therefore, surface tension of a liquid is equal to the
Consider a U-shaped wire frame with a sliding arm AB. surface energy per unit surface area.
The frame is dipped in a soap SI unit of surface tension is Nm–1 which is same
A A1
solution and taken out. A film as Jm–2.
of soap solution is formed in Molecules on 1 m2 area of water surface have 0.075 J
the rectangular frame.
F  extra energy compared to the same number of molecules in
The soap film so formed Fext
the bulk.
appear to be thin. But on
molecular scale, it may be B1 Example 4   Coalescing drops
B
lakhs of molecular layer thick. x A number of little droplets of water, each of radius r, coalesce
Nearly 20 molecular layers on to form a single drop of radius R. Calculate the decrease in
both sides form two surfaces and the remaining layers of surface energy. Surface tension of water is S.
molecules form the bulk of the soap solution.
Solution
Both the surfaces pull the sliding wire AB with surface
tension force. This force is Concepts
F = 2 l S (i) Using conservation of volume, we can find the
where l = length of wire AB number of smaller drops.
(ii) Due to decrease in surface area, the surface energy
S = Surface tension of liquid
decreases.
Decrease in surface energy is
D U = D AS
where D A is decrease in surface area of liquid.
Let n = number of smaller drops
r
4 4
n ◊ ​ __ ​  p r3 = __
​   ​  p R3
3 3

To keep the wire in equilibrium, an external agent must


apply equal and opposite force. fi (  )
R 3
n = ​​ __
​ r ​   ​​ ​
R
Now assume that you wish to stretch the film, thereby Decrease in surface area Smaller drops joining to form
increasing the liquid’s surface area. It implies that when you of liquid bigger drops decreases the
slide wire AB to the right, more and more liquid molecules surface area
will move from the bulk to the surface (the thickness of D A = surfaces area of n smaller drops)
film reduces). The molecules moving from the bulk to the
– (surface area of bigger drop)
surface gain energy. They gain energy as you perform work
in pulling the wire AB.
If you slide the wire AB slowly by a distance x, work
R3
= n ◊ 4 p r2 – 4p R2 = 4p ​ ___
​  3 ​  ◊ r2 – R2  ​
r [  ]
[ 
done by you is
W = Fext ◊ x = (2 l S) x
1 1
= 4 p R3 ​ ​ __r ​  – __
​   ​   ​
R ]
Entire work done is used by liquid molecules in moving \  Decrease in surface energy is

[ 
themselves from the bulk to the surface. We can think that
the surface energy possessed by molecules on the newly
created surface is equal to W.

1 1
D U = D AS = 4p R3 ​ __
​ r ​  – __ ]
​   ​   ​ ◊ S
R
9.6  Mechanics II

The lost surface energy goes to increase the internal the bigger drop will get heated. Its temperature will be higher
kinetic energy of the water molecules. In simple language, than that of smaller drops.

Your Turn

Q.4  What is the surface energy of a mercury drop of radius the amount of work needed to move the wire AB towards
r = 1 cm? Surface tension of mercury is 0.47 Nm–1. right by 1 mm. Surface tension = 0.03 Nm–1.
Q.5  A waterdrop of radius 1.0 cm is sprayed into 1000
equal-sized droplets. Calculate the gain in surface energy.
Surface tension of water is S = 0.075 Nm–1.
Q.6  When water is sprayed it gets cooler. why?
Q.7  A film of soap solution is formed in a U-shaped
frame with sliding wire AB. Length of AB is l = 0.1 m. Find

6. PRESSURE DIFFERENCE ON TWO SIDES and R2 – are known as principal radii of curvature of the
curved surface at point A.
OF A CURVED LIQUID SURFACE It can be proved that the pressure on concave side of such
Any stretched surface will remain flat if pressure is same a surface is greater than the pressure on convex side by

( 
on both sides of it. If there is a pressure difference on two
sides. It will get curved. obviously, pressure on concave
side will be higher.

1
D P = S ​ ___
R1 R2
1
​    ​ + ___ )
​    ​   ​ ...(3)

Where S = surface tension


For a spherical surface, R1 = R2 (= R, say). Hence,
pressure is higher on concave side of a spherical surface by
an amount
2S
D P = ___
​   ​  ...(4)
R

For a cylindrical surface,

In a waterdrop, inside pressure must be higher than out- R1 = R = radius of cylinder


side atmospheric pressure. Pressuer inside a soap bubble
is higher than outside pressure. A water jet from a faucet R2 = •
is nearly cylindrical in shape. Inside the curved cylindrical \  Pressure inside a cylindrical surface is higher than
surface, pressure is higher than the outside pressure. outside pressure by
In general, a curved surface may have different curvatures
in different directions. Consider a curved surface under
tension. A small patch of the
surface around point A is as
shown. If we draw arcs along
the surface passing through A,
we may get arcs of different
radii in different direction. BC is
one such arc along which radius
of curvature is minimum (equal
to R1). DE is another arc that is
perpendicular to BC. Radius of
curvature of arc DE is R2. The two radii of curvature – R1
1
D P = S ​ __(  1
​   ​  + __
R • )
​    ​  ​
Surface Tension and Viscosity  9.7 

S
fi D P = ​ __ ​   ...(5) fi
2 S
P = P0 + ___
​   ​ 
R R
Though we did not derive equation (3), we will derive Excess pressure inside the drop is
equation (4) in a simpler way, considering a spherical liquid 2 S
drop. D P = P – P0 = ___
​   ​   ...(6)
R
6.1  Excess Pressure inside a Spherical Liquid Drop This is same as the result stated in equation (4).
Consider a liquid drop. It will have spherical shape in absence 6.2  Excess Pressure inside a Soap Bubble
of gravity. Even in presence of gravity, a small drop will be
nearly spherical. A soap bubble has apparently a very thin wall, but it may
have lakhs of molecular layers in it. Therefore, it has two
surfaces and a bulk of liquid
between them. If the size of a
bubble increases, its wall will
get thinner but still, it is thick
enough to have two surface
layers.
Let pressure inside liquid
wall be P and the outside
atmospheric pressure be P1.
Let P be the pressure inside the drop. In absence of
The outer surface is a spherical
gravity, pressure will be same throughout (no change with
liquid surface of radius R.
depth) and in case of small drops, effects of gravity will
Pressure on concave side (P)
be too small to be noticeable and we can again assume the
is higher than the pressure on convex side (P1).
same pressure everywhere inside the drop.
Let the outside atmospheric pressure be P0 and surface 2 S
\ P – P1 = ​ ___ ​   ...(i)
tension of the liquid be S. R
We will consider the equilibrium of half of the drop ABC. The inner surface is also concave and therefore P2 > P.
There are three forces on the hemispherical part ABC:
2 S
(i)  Force due to atmospheric pressure P2 – P = ___
​   ​   ...(ii)
R
As we have seen in many problems, in the chapter of fluid
mechanics, the force is Note that there is not much of difference in radii of the
inner and outer surfaces. Both have same value R.
F0 = P0 (Projection of the area of curved hemispherical
surface ABC on the flat surface represented by AOC) Adding (i) and (ii) gives
= P0 p R2, where R is the radius of the drop 4 S
P2 – P1 = ​ ___ ​ 
This force is towards right in our diagram. R
(ii)  Force due to liquid pressure 4 S
D P = ​ ___ ​   ...(7)
F = P p R 2 R
This is excess pressure inside a soap bubble.
This force arises due to push of liquid in hemisphere ADC
applied on flat circular surface AOC. This force is towards 6.3  Air Bubble
left in our diagram.
(iii)  Force due to surface tension Consider an air bubble
inside a water tank. There
The hemispherical surface ADC pulls the hemispherical
is only one surface in this
surface ABC along the circular periphery. Force is
case. There is air on the
Fst = S ◊ 2p R towards right. concave side and liquid on
the convex side.
For equilibrium of hemisphere ABC, we must have Pressure inside the air
F = F0 + Fst bubble is higher than pressure outside it by
2 S
fi P ◊ p R2 = P0 ◊ p R2 + S ◊ 2p R D P = P2 – P1 = ___
​   ​   ...(8)
R
9.8  Mechanics II

Where R is radius of bubble and S is surface tension of The common wall is concave towards bubble of radius R2.
the liquid. If the radius of curvature of the common wall is R0 then

6.4  Water Jet from a Faucet 4S


P2 – P1 = ___
​   ​  
R0
A water jet from a faucet is nearly
2 S
[Pressure increases by ___
​   ​ every time your cross to a con-
cylindrical in shape when it begins R0
cave side]
to fall. Pressure inside the jet is D P

(  ) (  )
S
= ​ __  ​ higher than outside pressure, 4 S 4 S
\ ​ P0  + ​ ___ ​  ​ – ​ P0  + ​ ___ ​  ​ =
4 S
___
​   ​ 
R R2 R1 R0
where R is radius of the cylinder. As
R gets smaller, the pressure inside 1 1 1
the water jet gets higher. There are fi ​ ___  ​ – ___
​    ​  = ___
​    ​ 
R2 R1 R0
always some irregularities, which
cause R to decrease here and there. R1R2
The higher pressure in the thinner fi R0 = _______
​    
 ​
R1 – R2
region forces liquid away, into adja-
cent thicker regions. This amplifies
Example 6   Connected Bubbles
the irregularities–making thinner
region even thinner and the thick Two soap bubbles are blown at the ends of a tube. The
region thicker. Such a water jet bubbles have different radii. The valve (V) is opened to
is unstable and pinches itself into connect the bubbles.
drops.

Example 5   Double bubble


Two soap bubbles of radii R1 and R2 stick together, as shown.
Find the radius of curvature of the common wall.

(a) Which bubble will grow in size after the valve is


opened?
(b) What will be final state of the two bubbles?
Solution Solution
Concepts Concepts
(i) The common wall has two surfaces. The smaller bubble has a higher pressure inside it.
(ii) Air inside smaller bubble is at higher pressure than
that in larger bubble. Therefore, common wall is (a) As seen in the last example, the pressure inside the
concave towards the smaller bubble. smaller bubble is higher. Therefore, after opening the
valve, air will flow from the smaller bubble to the
Pressure inside bubble of radius larger one. The larger bubble will grow in size.
R1 is (b) As the smaller bubble decreases in size, pressure
4 S inside it becomes even higher and it continues to
P1 = P0 + ___
​   ​ 
R1 become even smaller. ultimately, the liquid in the
Where P 0 is atmospheric smaller bubble will form a film at the end of the tube
pressure. Pressure inside bubble of as shown in figure [something like a film one started
radius R2 is with to blow a bubble]. The radius of curvature of
4 S the film will be equal to the radius (R0) of the larger
P2 = P0 + ___
​   ​  bubble at the other end of the tube.
R2

If R2 < R1  then, P2 > P1.


Surface Tension and Viscosity  9.9

This will ensure that the air inside the system is at

( 
same pressure everywhere ​ = P0 + ___
4 S
​   ​  ​.
R0 )
Note: Our lungs contain hundreds of millions of mucus – lined
sacs called alveoli, which are about 0.1 mm in diameter. We
can exhale without muscle action by allowing surface tension
of the mucus to contract the sacs (just like the surface tension
caused the smaller bubble to contract). Medical patients
whose breathing is aided by a pressure respirator having air
blown into the lungs, but are generally allowed to exhale
on their own.

Your Turn

Q.8  Find pressure inside an air bubble of radius Q.10  A water jet from a tap has radius r = 2 mm at
0.1 mm that is located at a depth of 10 m inside water. position A and radius R = 4 mm at another position B. What
Surface tension of water S = 0.072 Nm–1, density of water is the pressure difference inside the jet at A and B? Surface
r = 103 kgm–3, atmospheric pressure P0 = 1.013 × 105 Nm–2 tension of water = 0.07 Nm–1.
and g = 10.0 ms–2.
Q.11  The pressure difference inside a soap bubble and
Q.9  Soap bubble A has twice the mass of bubble B. Both outside atmospheric pressure is P0. Surface tension of
the bubbles have same radii of 3 cm. Which bubble has more soap solution is S. Find the surface energy of liquid in the
pressure inside it? bubble.

In Short: (vi) Wall of a soap bubble has two surfaces. Excess


(i) Molecules on the surface of a liquid are surrounded pressure inside a soap bubble is
by less number of molecules compared to a molecule 4 S
deep inside the liquid. Due to this, they experience a D P = ___
​   ​ 
R
net inward force towards the bulk of the liquid. This
causes the surface to acquire a tension and a tendency
to contract. 7. COHESIVE, ADHESIVE FORCEs AND
(ii) Since molecules on the surface interact with less CONTACT ANGLE
number of molecules, they have higher energy. [Each
interaction adds negative energy]. Attractive force between molecules of a same substance are
called cohesive forces. Attractive forces between molecules
(iii) If we draw an imaginary line on a liquid surface it of different substances are known as adhesive forces.
divides the surface into two parts. Each part applies
When mercury drops fall on a clean glass plate they do
force on the other. This force, acting on unit length of
not spread. They remain intact as cohesive force in mer-
the line, is known as surface tension of the liquid.
cury is large compared to adhesive force between mercury
F
S = __
​   ​ (Nm–1) and glass. Mercury remains in shape of nearly spherical
L beads, which stand on
(iv) Extra energy possessed by molecules on unit area glass surface, maintaining
of liquid surface, is equal to surface tension in minimal contact with the
magnitude. surface. You can go and
(v) Pressure is higher on concave side of a curved liquid pick up a drop and no
surface. For a spherical surface of radius R, the mercury will remain on
2 S the glass. We say that mercury does not wet glass.
difference in pressure on the two sides is D P = ___ ​   ​ 
R When water drops fall on a clean glass surface, the liquid
and for a cylindrical liquid surface of radius R, the spreads out to form a thin, relatively uniform film over
S the surface. We say that water wets glass. This happens
difference in pressure on the two sides is D P = __ ​    ​.
R
9.10  Mechanics II

because of relatively stronger adhesive force between water of the surface itself. For a simpler case of flat liquid surface
molecules and glass. (which is not the case, usually) a liquid particle touching
Surface tension of a liquid is directly related to strength a solid wall will experience cohesive force in a direction
of cohesive force. Stronger cohesive force implies higher making 45° with the solid surface. Let us call this force
surface tension. as Fc.
When a liquid surface touches a solid surface, the shape
of liquid surface near the contact gets curved, in general.
This happens due to interaction of the liquid molecules with
the solid molecules. The curvature of the surface is decided
by the relative strength of cohesive and adhesive forces.
The angle between the tangent planes to the soid surface
and the liquid surface at
the contact is called contact
angle (q). Tangent plane to
the solid surface is drawn
going towards the liquid A liquid particle on surface, touching the solid boundary,
bulk and the tangent plane will be experiencing three forces: (i) adhesive force Fa (ii)
to the liquid surface is Cohesive force Fc and (iii) it own weight W.
drawn away from the solid. The direction of resultant (R) of the three forces decides
Figures show a water drop on a waxed surface and a water the shape of the liquid surface. A liquid surface in equilib-
drop on a clean painted surface of a car. rium cannot sustain tangential force. Therefore, the surfaces
In both the diagrams, l is tangent on the curved liquid curve so that R gets perpendicular to the surface. If the
surface at the contact, taken in resultant (R) of the three forces is directed through the solid
direction pointing towards the [as shown in fig (a)], the surface is concave. This is what
solid. S is a tangent plane on happens when water touches glass. If resultant (R) is along
solid with its direction pointing the solid surface, liquid surface is flat. Contact angle is 90°.
towards the liquid. Angle q is If R is directed into the liquid, the surface bulge out and
contact angle for given pair of contact angle is obtuse. This is what happens when mercury
liquid and solid. touches glass. [fig (c)]
We all known that water does not wet a waxed surface.
In this case, cohesive forces are much stronger than adhesive
force between wax and water. In situations like this, contact
angle is obtuse.
Water wets a dry and clean painted car surface. Adhesive
forces are stronger here. In such cases, contact angle is acute.
The larger q is, the larger is the ratio of cohesive to adhesive
forces.
Let us now see the reason behind curving of liquid sur-
face in contact with a solid. A liquid particle in contact with
solid is attracted by
nearby solid molecules.
Resultant of all these
forces is normal to the
solid surface due to
symmetry of attraction
from all sides. Let
us call the resultant
adhesive force on a
small liquid particle as
Fa that touches the solid surface.
The cohesive force (due to attraction of all surrounding
liquid molecules) on a liquid particle on surface is directed
into the liquid and its exact direction depends on the shape
Surface Tension and Viscosity  9.11

Contact angle of pure water with glass is 0° and that of


mercury with glass is close to 140°.

7.1  Shape of Meniscus


Take water in a narrow bore glass tube. The surface of water
gets curved. This curved surface is
known as meniscus. The curved
surface is nearly spherical if the
radius of tube is small. [If the tube
is wide, the surface will not be For q > 90°, the meniscus surface is convex upwards. In
spherical due to pronounced effect the diagram shown,
of gravity.] For contact angle of 0°,
a = – APO = q – 90°
this meniscus will be a hemisphere.
For contact angle 0° < q < 90°, it
PC
will be a smaller part of a sphere. \ ​ ___ 
 ​ = sin a
Radius of this (imaginary) sphere OP
is known as radius of curvature r
(R) of the meniscus. fi ​ __  ​ = sin (q – 90°)
R
As can be seen from the drawing
r
\ ​ __  ​ = – cos q ...(10)
r R
​ __  ​ = cos q ...(9)
R

Your Turn

Q.12  Simple water (without detergent) is unable to wash Q.14  It is difficult to break a mercury drop into small
grease and fat from a cloth. Why? droplets. Why?
Q.13  A chewing gum does not stick to your teeth. What
does it tell you about the relative strength of cohesive force
in gum and adhesive force between gum and teeth?

8.  CAPILLARY RISE


Literal meaning of capillary is a tube, which has internal
diameter of hair-like thickness.
When a glass tube of narrow bore is dipped in water, the
liquid rises in the tube. Whenever the contact angle is acute,
the liquid rises. This phenomena is known as capillarity.
However, the liquid gets depressed inside the tube if contact
angle is larger than 90°.
The adhesive force between water and glass is large and
water spreads on a horizontal glass plate to wet it. When
glass surface is vertical, water still tries to spread on glass
surface but due to gravity it cannot rise much. Water surfaces
forms a meniscus and water rises in the tube. Narrower the Surface of the liquid meets the tube wall along a circle
tube, higher is the rise. of periphery 2p r. The liquid surface pulls the tube wall
Consider a capillary tube of radius r dipped in a liquid of tangentially (inclined at q to the tube wall) along this circle.
density r. Let the contact angle be q. Surface of liquid inside From Newton’s third law, the tube wall must pull the liquid
the tube is (nearly) spherical, having radius R. From equation surface in opposite direction. The figure shows this force as
r
(9), __
​    ​ = cos q, where r is inner radius of the tube. Fst.
R
9.12  Mechanics II

On a small elemental length D l of the circumference, the Alternative way to find capillary rise
force is D Fst = S ◊ D l, making angle q with vertical. Vertical Pressure at Point 1 above meniscus surface is atmospheric
component of forces on all such small lengths adds to give a pressure.
resultant vertical force on the liquid surface, which is equal
P1 = P0.
to
Fst = ​S    ​ ​S​   D l cos q = S cos q (2p r)

This vertically upward force by glass wall on the liquid


surface supports the weight of liquid raised in the tube.
Neglecting the liquid present in the meniscus, the weight
of liquid in the tube is
W = p r2h ◊ r ◊ g
Thus, W = Fst

fi p r2h rg = S cos q (2p r) ...(i)

2 S cos q
fi h = _______
​     
​   ...(11)
r r g Meniscus is concave upward, therefore, pressure at point
This can also be written as 2 just below the surface is
2 S 2 S
h = ____
​     ​ ...(12) P2 = P1 – ___
​   ​   where R is radius of
R r g R
curvature of the
where R is the radius of curvature of meniscus surface. meniscus surface
2 S
Correction for Weight of Liquid in Meniscus fi P2 = P0 – ​ ___ ​ 
R
In writing the above expression, we neglected the weight
of liquid present in the meniscus. Correction for this is 2 S
Pressure at 3 is P3 = P2 + rgh = P0 – ___
​   ​ + rgh
easy–particularly when contact angle is 0°. In this case, R
shape of meniscus is hemispherical and R = r. But pressure at 3 is same as pressure at point 4, which
is atmospheric pressure.
2 S
\ P0 = P0 – ___
​   ​ + rgh
R

2 S 2 S cos q r
fi h = ​ ____   
​ = _______
​     ​  since, ​ __  ​ = cos q
Rrg r rg R
Volume of liquid in meniscus is
2 8.1  Tube of Insufficient Length
= p r2 ◊ r – __
​   ​  p r3
3 As discussed in Section 7, the force (Fa) applied by solid
1 surface on liquid particles touching it is perpendicular to
= __
​   ​  p r3
3 the solid surface. But this
is not the case when the
Therefore, equation (i) gets corrected as solid surface is just dipped
in liquid. As there is no
1
p r2 h r g + __
​   ​  p r3 r ◊ g = S cos 0° (2p r) solid above the liquid sur-
3
face, the adhesive force is
directed towards the solid,
2 S r
fi h = ____
​    ​ – __
​    ​   ...(13) as shown. In such situation, the angle between the solid and
rrg 3
liquid surface may be different from the standard value of
Where r is small, the correction is not very significant. contact angle for the solid–liquid pair.
Equations (11) and (12) give us the capillary rise when a
long tube is dipped in a liquid. What happens if the length
Surface Tension and Viscosity  9.13 

of tube above the liquid surface is less than the value given fi p r2 ◊ l r ◊ g ◊ sin a = S ◊ 2 p r ◊ cos q
by equation (11)? Will the liquid overflow out of the tube?
Answer is, No. The angle between the tube wall and liquid 2 S cos q
fi l sin a = _______
​      ​ 
surface (= q) changes so as to keep surface tension force rrg
(Fst) equal to weight of liquid in the tube. In a much simpler
2 S cos q
language, we can say that radius of curvature (R) of the fi h = _______
​      ​ 
rrg
liquid surface changes.
Vertical ascent of the
liquid does not change
even if the tube is tilted.
In fact, the vertical
height of liquid in the
capillary tube remains
unchanged even if the
tube in not straight.

8.3  Obtuse Contact Angle


In this case, the liquid gets depressed in the tube. An analysis
similar to the above shows that the fall (h) of liquid level in
the tube is still given by equations (11) and (12).
If the tube has sufficient length
2 S
h = ____
​     ​
Rrg

2 S
fi hR = ___
​    ​ = a constant
rg
When the tube has insufficient length h1 above liquid
surface, the radius of curvature (R1) of liquid surface changes
such that
2 S
h1 R1 = hR = ___
​    ​ ...(14)
rg
In an extreme case, when tube has no projection out of cos q is negative for 90° < q < 180° and a negative value
liquid surface (h1 = 0), the value of R becomes infinite. It of h in equation (11) shall be interpreted as a fall in liquid
means liquid surface is flat, it does not curve. level.

8.2 Inclined Tube 8.4  Capillary Rise in Practical Life


The resultant force of surface tension remains (i) Capillary action allows the drainage of tear fluid that
is constantly produced in eyes. In the inner corner
Fst = S ◊ 2 p r ◊ cos q of the eyes, behind the eyelids, there are two hair-
This balances the component of weight along the tube. like canals known as lacrymal ducts. Tear fluid from
within the eyes rises into those ducts until they get
\ W sin a = S ◊ 2 p r ◊ cos q expelled.
(ii) The tissue in roots, stems and branches of a tree
Fst have pores between them. These pores interconnect
to form long capillary tubes. Water can rise through
these tube to great heights.
 h (iii) Diwali ‘diyas’ keep burning, as oil keeps rising to
W the tip of burning wick due to capillary action.
a
(iv) Moisture in soil rises to the surface through
capillaries and gets evaporated. By ploughing the
9.14  Mechanics II

field, we can break many such capillaries and reduce Since our tube is only 5 cm tall, it is a tube of insufficient
evaporation. length.
Example 7   A liquid of density 1.5 g cc–1 rises 3.0 cm in h1 R1 = hR
a capillary tube of diameter 0.5 mm. Angle of contact for the
solid–liquid pair is 0°. Calculate the excess pressure inside fi (5 cm) (R1) = (30.58 cm) (0.005 cm)
a spherical bubble of 1.0 cm radius blown from the same
liquid, in unit of dyne cm–2. g = 980 cms–2 fi R1 = 0.03 cm
Solution
Example 9   A capillary tube with solid core
Concepts
A capillary tube of radius r2 has a coaxial solid core of
(i) Formula for capillary rise will give the value of radius r1 (<r2). It is dipped in water and the liquid rises in
surface tension. We will write everything in CGS the annular space. Find the height to which water rises in
unit the tube. Contact angle is 0° and surface tension is S.
4 S
(ii) Excess pressure inside bubble is ___
​   ​ 
R

2 S cos 0° hrrg
h = ________
​      ​  fi  S = _____
  ​   ​ 

rrg 2

(3.0 cm) (0.025 cm) (1.5 gcm–3) (980 cms–2)
fi S = ​ ___________________________________
      ​    
2

= 55 dyne cm–1
4 S
Excess pressure inside bubble is D P = ___
​   ​ 
R
4 × 55  dyne  cm–1
\ D P = ​ _______________
 ​
     = 220 dyne cm–2
1 cm
Solution
Concepts
Example 8   A 5 cm long capillary tube with 0.1 mm
internal diameter is slightly dipped in water. Find the radius (i) Contact angle is 0°. This means that the force by
of curvature of the meniscus surface in the tube. Surface tube wall on the water surface is vertical.
tension of water is S = 75 dyne/cm and contact angle for (ii) Water surface touches the tube wall along two
water and glass is 0°. Take g = 981 cm s–2. circular peripheries of lengths 2p r1 and 2p r2.
Solution Thus, upward pulling force on water surface is
Concepts Fst = 2 p r1 S + 2p r2 S.
(iii) The above force balances the weight of water in
(i) We need to check whether the tube is of insufficient
the annular space.
length.
(ii) If the tube is sufficiently long, then for q = 0°, Volume of water in the tube when rise is h is
the radius of meniscus surface is same as the tube
radius, i.e., R = r V = p (r22 – r12) ◊ h
If the tube is of insufficient length, then we will
use equation (14) to find R. For equilibrium of water in the tube:

0.1 mm Weight of water in tube = Fst


Height of capillary rise in a tube of radius r = ​ _______
 ​  is
2
p (​r2​2​​  – r​ 21​ ​)​   h rg = 2p S (r1 + r2)
2 S cos 0° 2 × 75 (dyne/cm)
h = ________
​      = ​ ____________________________
​      
     ​ 2 S
rrg (0.005 cm) (1 g cm–3) (981 cms–2) fi h = _________
​       ​
(r2 – r1) rg

= 30.58 cm
Surface Tension and Viscosity  9.15

Your Turn

Q.15  A capillary tube of inner radius 0.5 mm is dipped Q.17  Mercury has angle of contact of 120° with wall of a
in a liquid having surface tension S = 0.07 Nm–1. Angle of capillary tube. Inner radius of the tube is 1mm. Surface tension
contact is 60° and density of the liquid is 103 kgm–3. Find and density of mercury are 0.5 Nm–1 and 13.6 × 103 kgm–3
the height to which the liquid rises. g = 9.8 ms–2. respectively. What is level of mercury in the tube when its
Q.16  A glass tube of radius lower end is dipped in a tank full
r = 0.4 mm is dipped in water and of mercury?
held making an angle of 30° with Q.18  A capillary tube dipped
the horizontal. How much length of in mercury is shown in figure.
capillary is occupied by water? Surface Find the radius of curvature of the
tension of water S = 0.07 Nm–1 and curved mercury surface. Density
density of water is r = 1 × 103 kgm–3. of mercury and its surface ten-
Contact angle is 0°. sion are r and S respectively.
Depression in the tube is h. What is pressure just below
mercury surface in the tube?

9.  DETERGENTS
Washing your clothes with water does not remove grease
stains. This is because water does not wet greasy stains.
If water could wet grease, the flow of water could carry
some grease away. However, this can be achieved by mixing
detergent to water. Mixing of detergent reduces the surface
tension of water.
Properties of detergents arise from their complicated
molecular structure. A
typical detergent mol-
ecule has a shape like
a pin. Its head is hydro-
philic and gets attracted
to water molecules. Its
tail is hydrophobic (hates water molecules) but lipophilic
(gets attracted to grease, oils, fats).
When detergent is put into water, at surface the detergent
molecules have their hydrophobic ends protruding out. It
is easier to pull this surface apart than it is to pull apart a
surface of pure water.

In Short:
(i) Angle between tangent plane to liquid surface (away
from the solid) and tangent plane to solid surface
(going towards the liquid) is known as contact
angle (q).
(ii) Value of q is less than 90° when adhesive forces are
relatively much stronger than cohesive forces. In such
Sequence occurring during washing can be understood in cases, liquid wets the solid surface.
simple way be looking at the following diagram.
(iii) If q < 90°, liquid rises in the capillary tube. Capillary
rise is given by
9.16  Mechanics II


2 S
h = ____
​    
2 S cos q
 ​ = _______
​      ​
  10.1  Viscous Force
Rrg rrg
Consider a liquid film of thicknes d over a fixed solid
Where R = radius of curvature of meniscus surface surface A. A block B is placed over the liquid and dragged
and r = radius of the tube horizontally with velocity v.
The layer of liquid in contact
(iv) When cohesive forces are much stronger, the contact
with the solid moves with
angle is obtuse (q > 90°). In such cases, the liquid
velocity v.
does not wet the solid surface.
The lowest layer, in contact
(v) If q > 90°, liquid level falls in capillary tube. The with surface A, remain at rest.
fall (h) is given by the above formula only. Negative velocity of different layer
value of h indicates fall. increase from zero to v, as
(vi) In a capillary tube that is inclined to vertical, the we move up from surface A.
vertical rise of liquid is still given by the above We define velocity gradient of fluid layers as
formula. v
Velocity gradient = __​   ​ ...(15)
(vii) In a capillary tube of insufficient length, the radius d
of curvature of the liquid surfaces change (contact
The force (F) needed to keep the block moving uniformly
angle has no meaning in this case) and liquid does
with velocity v is proportional to velocity gradient and also
not overflow.
proportional to area of contact (A) of the block with the
h0 R0 = hR liquid. Thus, viscous force acting on the block is given by
where,
v
h0 = capillary rise in a tube of sufficient length fv = F = h ◊ A ​ __ ​ ...(16)
d
R0 = radius of menicus in a tube of sufficient length
Here, h is a constant known as coefficient of viscosity. SI
h = capillary rise in a tube of insufficient length unit of coefficient of viscosity is N–s m–2. Its CGS unit is
R = radius of meniscus in tube of insufficient length dyne – s cm–2, which is also known as poise. You can yourself
prove that 1 poise = 0.1 N–s m–2.
At 20°C, coefficient of viscosity for water is nearly 10–2
10.  Viscosity poise. At 80°C, value of h for water is 0.36 × 10–2 poise.
A feature which distinguishes one liquid from another is In fact, viscosity of liquids decrease with rise in temperature
their ‘thickness’ or the ease with which they pour. And and for gases, it increases with increasing temperature.
this property of a liquid certainly depends on how well the When velocity gradient is not uniform, the force between
liquid molecules adhere to one another. If we pour water two layers of fluids should be written as
and honey on the walls of two separate funnels water flows
dv
down readily but honey takes much longer to flow down. As fv = h A ​ ___  ​ ...(17)
dz
honey tends to flow down due to gravity, the relative motion
between its layers is opposed strongly. We say honey is more Where v = velocity of
viscous than water. fluid layer at height z from
When a layer of fluid slips or tends to slip on another the fixed surface A.
layer in contact, the two layer exert tangential force on one v + dv = velocity of
another. The direction of the forces are such that the relative layer at height z + dz
motion between the layers is opposed. This property of a dv
fluid to oppose relative motion between its layers is known ​ ___  ​ = velocity gradient in
dz
as viscosity. the direction perpendicular to layers.
Before we start, let us put an important fact. In all
circumstances, the velocity of a fluid is always zero
Example 10   A layer of castor oil, 1 mm thick, is spread
(relative to the solid surface) at the surface of a solid. You
on a horizontal floor. A block is placed on the floor and
must have noticed that blades of a fan collect a thin layer
pulled horizontally at a constant speed v = 2 ms–1. Find the
of dust–and it remains there even when the fan is rotated at
force applied to the block.
a high speed. Dust doesn’t get blown off by air. Reason is
The face of block touching
the fact that speed of air relative to the fan blade is zero,
the oil has area of 20 cm2.
right at the surface.
Coefficient of viscosity of
oil at given temperature is 10 poise.
Surface Tension and Viscosity  9.17

v 2  ms–1
Solution \  Velocity gradient = __
​   ​ = ______
​  = 2 × 103 s–1
 ​ 
d 1 mm
Concepts
Viscous force on the block is opposite to motion having
The required force must be equal to viscous force on the magnitude
block. This will ensure no acceleration v
Fv = h A ​ __ ​ = (10 × 0.1 NSm–2) (20 × 10–4 m2) (2 × 103 s–1)
d
Top layer of oil moves with velocity of the block =
2 ms–1. Lowest layer of oil has zero velocity as the floor is = 4 N
not moving. External agent must apply 4 N force to keep the block
moving.

Your Turn

Q.19  There is water in a bucket. It is stirred for some time and it is found that it moves with
and left on its own. Soon the spin of water stops. Why?
constant velocity down the incline.
Q.20  A block of mass 50 g lies on an incline with a Find viscosity (h) for the oil.
layer of oil 1mm thick. Area of the block in contact with
oil is 20 cm2. The block is imparted a velocity v = 2 ms–1

11.  STOKES’ LAW (iii) Viscous force, Fv = 6p h rv, where r is radius of the
ball and v is its speed.
When a solid moves through a fluid, it experiences a viscous The first two forces remain constant but the viscous force
drag. Magnitdue of the viscous force depends on the shape (Fv) increases in magnitude as the ball speeds up, since it
and size of the solid, its speed and coefficient of viscosity is directly proportional to speed. Initially, when velocity is
of the fluid. low, the viscous force (Fv) is low as well. Therefore, the net
English Physicist Stokes derived a formula for viscous downward force and acceleration are large. This causes the
force acting on a spherical body moving through a fluid. speed to increase. As the speed
Consider a sphere of radius r moving at speed v in a fluid increases, Fv also increases and
medium, having viscosity h. According to Stokes, the sphere in turn, the net downward force
experiences a viscous force given by and acceleration decrease.
Eventually, Fv becomes large
Fv = 6 p h rv ...(17)
enough to make net force on
One can find the dependence of Fv on h, r and v by the the ball equal to zero. Now,
method of dimensions. the velocity becomes constant.
This constant velocity is termi-
12. TERMINAL SPEED nal velocity (v0).
Therefore, when velocity of
Consider a body released from rest in a viscous medium. the ball is v0, net force on it is
For example, consider a ball released from a height in zero.
atmosphere. The body first accelerates and then acquires a
velocity, which remains constant during the remaining fall FV + FB = W
of the body. This constant speed (v0) is known as terminal
4 4
speed. 6p h rv0 + __
​    ​ p r3 rg = __
​   ​  p r3 dg
3 3
There are three forces on the ball as it falls through
2 r2
atmosphere: fi v0 = __ ​   ​  __
​   ​  g (d – r) ...(18)
(i) Its weight, W = mg = Vdg, where V is volume of the 9 h
ball and d is its density. When density of ball is large compared to density of the
medium (i.e., d >> r), buoyancy can be neglected and the
(ii) Buoyancy, FB = Vrg, where r is the density of the
expression of terminal speed reduces to
medium (air). 2
2 rg
v0 = __
​   ​  ___
​     ​ d ...(19)
9 h
9.18  Mechanics II

Example 11   A steel ball of radius r = 5 mm is released generation is proportional to 5th power of radius of the
on the surface of a deep lake. Find the terminal velocity ball.
attained by the ball. Density of steel is 8,000 kgm–3 and Solution
that of water is 1,000 kgm–3. Coefficient of viscosity is 10–3
N-sm–2 and acceleration due to gravity is 10 ms–2. Concepts

Solution Rate of loss of mechanical energy = rate of heat production


= – (power developed by viscous force).
Concepts
2r2g
Direct use of equation 18 Terminal speed is v0 = ____
​   ​ (d – r)
9 h
2 –3 2
2r g 2 (5 × 10 ) × 10
v0 = __
​   ​ ​  ___   ​ (d – r) = __
​   ​  × ​  _____________
 ​
    
(8000 – 1000) Viscous force is Fv = 6 p h r v0.
9 h 9 10–3
Power of viscous force is
= 389 ms–1 P = – Fv v0 (negative sign since Fv and v0 are in opposite
This is very high! No lake will be so deep to allow the direction)
ball to reach its terminal speed. \  Rate of heat generation = Fv ◊ v0

Example 12   Viscous force dissipates energy 8p g2


= 6 p h rv02 = _____
​   (d – r)2 r5
 ​ 
A small ball falls from rest in a viscous medium. Due to 27 h
viscous force, heat is produced. Show that rate of heat
\  Rate of heat generation µ r5.

Your Turn
Q.21  A spherical object falls in a viscous medium and Q.22  A rain drop attains terminal speed of 3 ms–1 while
attains a terminal speed v0. Mass of the object is m and falling through air. If the volume of rain drop were 8 times
buoyancy force on it is negligible. Find the viscous force on its present value, what would have been its terminal speed.
the object when it has attained terminal speed.

In Short: 13. REYNOLDS' NUMBER


(i) Viscous force between layers of a fluid is given by
Osborne Reynolds studied the condition in which flow of

(  )
v fluid in pipes transitioned from steady flow to turbulent
Fv = h A ​ __
​   ​  ​
d flow. He took a large pipe of transparent glass having water
v flowing through it. He introduced a small amount of dye
Where A = area of contact and ​ __ ​ = velocity gradient
d into the centre of the flow and observed the behavior of the
along the thickness of the fluid. layer of dyed stream. When the flow speed was low, the dyed
h is coefficient of viscosity. layer remained distinct and laminar throughout the length of
(ii) Viscous force on a spherical body is Fv = 6 p h rv the large pipe. When speed of flow was increased beyond a
point, the dyed layer broke up and diffused throughout the
(iii) When a body falls through a viscous medium, its tube, indicating turbulenece.
speed becomes constant after some time. This con-
He concluded that whether the flow will be steady or
stant speed is known as terminal speed (v0).
turbulent depends on density (r), speed (v) and viscosity (h)
For spherical bodies, of the fluid. It also depends on diameter (D) of the tube.
2 He suggested a dimensionless number, known as Reynolds'
2 rg
v0 = __
​   ​  ___
​     ​ (d – r) number, which can be used to predict the nature of flow.
9 h
rvd
d = density of body, r = density of fluid. R = ____
​     
​   ...(20)
h
Surface Tension and Viscosity  9.19 

When R < 2000, the flow is steady and when R > 3000, Solution
the flow becomes turbulent. For 2000 < R < 3000, the flow Concepts
is unstable. If may remain steady for a small interval of time
and then turn turbulent for some time. R has same value in Find Reynolds’ number.
any system of units, as it is dimensionless.
rvd 10 3
× 10 × 0.2
R = ____ ​ = ​ _____________
​       ​
     = 2 × 106
Example 13   Water is flowing at a speed of 10 ms through
–1 h 10–3
a pipe of dimater 0.2 m. Viscosity of water is 10–3 Nsm–2. This is a large number. Flow is certainly turbulent.
Is the flow steady?

Miscellaneous Examples
Example 14   There is a film of soap solution on a circular
wire frame. A light thread loop is placed on the film. A pin [  D q D q
]
​ For very small D q,  sin ​ ___ ​    Æ ​ ___ ​    ​
2 2
is used to prick the film inside the loop. The thread takes
For equilibrium of the segment we must have
a circular shape of radius r. Calculate tension developed in
the thread. D q
2T ___    2.(rD q).S
​   ​ =
2
fi T = 2 ◊ S ◊ r

Example 15   Two parallel plates


Two square plates have side length L each. They are kept
close to each other with a small gap d between them and
their lower edges are made to touch the water surface in a
tank. Water rises in the gap between the plates to a height h.
Solution Surface tension of water is S and it has a contact angle of 0°
Concepts with glass. Neglect edge effects, if any, and assume that the
water surface between the plates has uniform curvature.
Surface tension force on a small segment of the thread is
radially outward. This is balanced by the radial component
of thread tension on two sides of the segment.

Consider a segment of thread subtending a small angle


D q at the centre.
Outward force on the segment due to surface tension is

(a) Find h.
(b) Calculate the horizontal force on any one plate arising
due to pressure difference on two faces.
D Fst = 2 (r D q) ◊ S [There are two surfaces]
Solution
Let T = Tension developed in the thread, Concepts
D q D q
component of T towards centre = T sin ​ ___ ​   T ​ ___ ​   (i) The meniscus is cylindrical in shape.
2 2
9.20  Mechanics II

(ii) Upward pull on water surface due to two glass a thin film having thickness h. Diameter (D) of the flattened
plates is simply Fst = 2.SL drop is large compared to h. Contact angle is 0°. What is the
extra force acting on each plate due to pressure difference
(iii) Fst balances the weight of water between the
on two sides?
plates.
(iv) One can also find height h thinking in terms of
pressure. just below the curved menisus, pressure
S
is less than atmospheric pressure by D P = ​ __  ​ where
R
d
R = __​   ​ .
2
(v) Pressure on the plate due to atmospheric pressure Solution
is higher than pressure due to water. This pressure Concepts
difference causes a resultant horizontal force.
(i) Since, D >> h, the surfaces of water exposed to
(a) Fst = W atmosphere can be assumed to be nearly cylindrical.
pressure inside water is less than atmospheric
2SL = L ◊ d ◊ h ◊ r ◊ g S 2S
pressure by __
​   ​  = ___
​   ​ 
R h
2S
fi h = ​ _____
  ​   ...(i) (ii) The tangential surface tension force is horizontal
r d ◊ g and has no role in present problem.
(b) Radius of cylindrical water surface
Since, contact angle is 0°, the surface tension force on
d
is R = ​ __ ​  glass plate at every location is horizontal. for a circular drop,
2
resultant of this force on glass
Pressure at point 1 just below the cylindrical surface is plate is zero.
S 2 S Curved surface of water is
P1 = P0 – __
​    ​ = P0 – ___
​     ​
R d nearly cylindrical with radius
h
Pressure at point 2 is R = __
​   ​ 
2
2 S
P2 = P1 + rgh = P0 – ___
​     ​ + rgh \ P2 = P1 – __
S
​   ​ 
d R
Average outward pressure on a plate is 2 S
= P0 – ___ ​     ​
P1 + P2 2 S rgh h
Pav = ​ _______ = P0 – ___
 ​  ​     ​ + ____
​   ​ 

2 d 2 \  Force on a plate due to pressure difference on two
Outward force on a plate due to water is p D2
sides over an area A = ____
[ 
​   ​ 
 is

2S
F1 = Pav ◊ L2 = ​ P0 – ___
​   ​ +
d
rgh 2
____
2 ]
​   ​  
 ​ L

4
2S D2 p ____
F = (P0 – P2) A = ___
​   ​  ◊ p ​ ___ ​ = __
SD2
​   ​  ​     
 ​
h 4 2 h
Force due to atmospheric pressure is
This force is downward on upper plate and upward on
F2 = P0 L2
the lower plate.
\ Net inward force is It will be difficult to separate the two plates by pulling
F = F2 – F1 = ​ ___
2S
​   ​ –
d [  rgh 2
____
]
​   ​  
2
 ​ L
them away from one another.

[ 
Example 17   A liquid having surface tension S is form-
2S
= ​ ___
d
S
​   ​ – ​ __ ​   ​ L
d
2
] [iusing (i)] ing a film between a horizontal surface and a cylindrical
object of radius R, as shown
SL2 in figure. Cylindrical liquid
or, F = ___
​     
​ 
d surface has radius r. The
contact angle is q = 0°. Find
Example 16   Water drop between two glass plates the downward force acting
A drop of water lies between a pair of horizontal flat glass on unit length of the cylin-
plates. Figure shows the side view. The water drop spreads as der due to presence of the
Surface Tension and Viscosity  9.21 

liquid. Height of liquid above horizontal surface is greater Example 18   A monometer contains water of density
than r. r = 103 kgm–3. Its arms have different radii of r1 = 1.44 mm
Solution and r2 = 0.72 mm. The wider tube is connected to a bulb
containing a gas and the other
Concepts tube is open to atmosphere. The
There is a surface tension force, which has vertically difference in level of liquid in
downward resultant and there is one force acting due to the two arms is h = 0.2 m, as
drop in pressure inside the liquid (like in last example). shown. Find the gauge pressure
in the bulb. Surface tension of
From the figure water is 0.072 Nm–1 and contact
R–r angle is 0°.
cos a = ​ _____ 
 ​ ...(i)
R+r Solution
Consider a length L of the cylinder. Force due to surface Concepts
tension acts tangentially and has a vertical resultant given
(i) Since, radii of the two tubes are small, effect of
by
surface tension cannot be neglected.
2 S L ◊ sin a (ii) We will equate pressure in the two arms at the level
of liquid surface in the wider arm.

Pressure in water just below the curved surface in the


wider arm is
2 S since q = 0°
P1 = Pgas – ​ ___
r1  ​
  Radius of surface = radius of tube

Pressure in other arm at the same horizontal level is


2 S
P2 = P0 – ​ ___
r2 ​ + rgh
P1 = P2
Pressure inside the liquid (at 1) is less than atmospheric
S
pressure by D P = __
​ r ​  \ Pgas – P0 = 2 S ​ __( 
1
​ r   ​  –
1
__1
)
​ r   ​   ​ + rgh.
2
Pressure difference causes an additional downward force
on the cylinder given by
D P  [Rectangular area
\ ​ 
1
[ 
Pgas – P0 = 2 × 0.072 ​ __________
1
    ​– __________
​      ​  ​
1.44 × 10–3 0.72 × 10–3 ]
2 SRL
2R sin a . L] = _____
​  r   
​ sin a. + 103 × 9.8 × 0.2

\  Net downward force due to = 100 – 200 + 1960 = 1860 Nm–2


presence of liquid is
2SRL
Fnet = 2 SL sin a + _____
​  r   
​  sin a Example 19   Capillary tube open at both ends
A narrow capillary tube of radius r = 0.5 mm
Force per unit length is
contain some water in it. When the tube is held

[ 
vertical, the water remains inside the tube as
Fnet
​ ____
L
  ​ =
R
  2S ​ 1 + ​ __
r ​   ​ sin a ] shown. Find the radius of curvature of the upper
and lower water surfaces when

] ÷  ( 
__________
r+R
= 2 S ​ ​ _____ 
r    [  R–r 2
​  ​ ​ 1 – ​​ ​ _____
R+r
    )
 ​  ​​ ​ ​ [using (i)]


(a) h = 2 cm
(b) h = 3 cm (c) h = 4 cm
__
Given: Surface tension S = 0.075 Nm–1 and
R
= 4S ​ __
​ r ​ ​   ÷  contact angle = 0°. g = 10 ms–2
9.22  Mechanics II

Solution 2 S
​ ___ ​ =
  100
R
Concepts
fi R = 1.5 mm
(i) The upper meniscus will be hemispherical. The
upward force by glass on the upper meniscus tries
Example 20   Two soap bubbles are in contact. What is
to balance the weight of water column. If the
the angle between the tangent planes to their surfaces where
weight increases (i.e., h is high) the lower surface
they meet?
also gets concave up so as to have an upward force
on it. Solution
(ii) Working in terms of pressure is much easier. Concepts
Pressure just below the upper meniscus is (i) Soap solution is same, therefore surface tension is
same for both bubbles.
2 S 2 × 0.075
P1 = P0 – ___
​  r   ​  = P0 – ​ _________–3 

 ​ (ii) Consider forces on a small line segment along the
0.5 × 10 joint on the surface of the two bubbles.
= (P0 – 300) Nm–2 Consider a small arc of length
At depth h below the upper meniscus, the pressure will be D l along the circumference of
the common wall. This segment
P2 = P1 + rgh = P0 – 300 + 103 × 10 × experiences three surface tension
forces due to three surfaces touch-
fi P2 = P0 – 300 + 104 h ...(i) ing it. All three forces are of same
magnitude F1 = F2 = F3
(a) For h = 2 cm For equilibrium of the segment, resultant of three forces
P2 = P0 – 300 + 104 × 0.02 = P0 – 100 Nm–2 must be zero. This is possible only if the three forces are
inclined at 120° to each other.
Since, pressure inside the lower sur- Forces are tangential to the three surfaces. Thus, 120° is
face is less than P0, the surface must the required angle between the tangent planes on the two
be convex upwards. Radius of curva- bubbles.
ture R is such that
2 S Example 21   A capillary tube of radius r is dipped in
​ ___ ​ =
  100 Nm–2 water. The liquid rises in the tube. Find heat generated due
R
to dissipative force (like viscosity). Contact angle is 0° and
2 × 0.075 surfaces tension is S.
​ ________
     
​ = 100
R
Solution
fi R = 1.5 × 10–3 m = 1.5 mm
Concepts
(b) When h = 3 cm As soon as the capillary touches the water surfaces, the
4 upward surface tension force pulls the water into the tube.
P2 = P0 – 300 + 10 × 0.03
If there is no dissipative force, the water in the tube will
[using (i)] get accelerated. Its speed will increase till equilibrium
= P0 position (where surfaces tension balances weight of water
in the capillary). Having some kinetic energy, water will
\ surface will be flat with radius of continue to rise. Now, it retards and stops. Now its weight
curvature = • is greater than upward surface tension. It will accelerate
(c) When h = 4 cm down. Thus, water column in the tube will oscillate. But
this does not happen due to presence of dissipative forces.
P2 = P0 – 300 + 104 × 0.04 When the water moves viscous force dissipates energy.
= P0 + 100 Nm–2 2 S
Rise of water in the tube is h = ____
​     ​
rg r
Since, pressure at 2 is higher than
atmospheric pressure, the lower sur- Increase in gravitational potential energy of water
face will be concave up.
h
D U = (mass of water is capillary) ​ __ (  )
​   ​   ​ g.
2
Surface Tension and Viscosity  9.23 

(  )
2
h p 1 2gr d
= (p r2 h r) ​ __ ​   ​   ​ g = __
​   ​  r2 r g h2 where v0 = ​ __ ​ = _____
​    
​ 
2 2 k 9 h
2p S2
_____
= ​      ​ 
rg At time t Æ 0; v = 0
Work done by surface tension is
At time t Æ •; v Æ v0
W = (2p r . S). h [Contact angle is 0° and surface
tension force is vertical]
4p S2
_____
= ​      ​ 
rg
2p S2
Loss in energy ( = heat generated) = W – D U = _____ ​     ​ 
rg
Example 22   Falling raindrop
A raindrop of radius r begins to fall from a large height.
Write its speed as a function of time and plot speed (v) –
time (t) graph. Assume that density of drop (d) is large
compared to density of air and coefficient of viscosity is h.
Take acceleration due to gravity (g) to be constant. v0 is the final terminal speed. v-t graph is as shown.

Solution
Example 23   A cylinder is filled with a liquid of density
Concepts
r and viscosity h. A cylindrical piston
(i) Since density of drop is large compared to density has radius (r) slightly smaller than the
of medium, we can neglect buoyancy. inner radius (R) of the cylinder. Mass of
(ii) The drop moves under influence of gravity and the piston is m. The piston is released in
viscous force only. the liquid with its axis conciding with
dv the axis of liquid cylinder. Find the
(iii) We will use F = ma and write a = ___​   ​ .
dt terminal speed attained by the falling
Integrating the obtained expression after separating piston. Thickness of the piston is x.
the variables gives the result.
Solution
Let speed be v at time t after the drop begins to fall. Concepts
m ◊ a = W – Fv (i) The fluid in contact with the piston will move
with speed of piston and the fluid in contact with
4 dv __ 4
​ __ ​  p r3 ◊ d ​ ___ ​ =
  ​   ​  p r3 ◊ d ◊ g – 6 p h r v cylinder wall is at rest. Velocity gradient in the
3 dt 3 v
fluid layer between two walls is ​ ______     ​ where
dv 9 h (R – r)
fi ​ ___ ​ =
  g – __
​   ​  ___
​    ​  v v = speed of piston.
dt 2 r2 d (ii) When piston moves at terminal speed, net force on

[ 
it is zero.
9h
= g ​ 1 – ​ _____
2gr2d
   
​ v  ​
] Let v0 = terminal speed.
9h
= g [1 – kv]  where  k = _____
​  2   
2gr d
​ v0
Viscous force FV = h A ​ _____(  )
​     
R–r (  )
v0
 ​  ​ = h 2p r x ​ _____
​     
R–r
 ​  ​
v t
dv
\ Ú = g ​Ú ​  ​ dt FV + FB = mg
v=0 1 – kv 0

fi [ln (1 – kv)​]v0​ ​​  = – kgt (  )


v0
h . 2p r x ​ _____
​     
R–r
 ​  ​ + p r2 x rg = mg

fi ln (1 – kv) – ln 1 = – kgt (R – r)


fi v0 = ​ _______  ​ [mg – p r2 xrg]
  
h . 2p r x
1
fi 1 – kv = e– kgt  fi  v = __
​   ​ [1 – e– kgt]
k (R – r)g
= ​ _______ ​ 
[m – p r2 xr]
fi [ 
v0 = v0 ​ 1 –
– gt
___
]
​   ​ 
​e​v0 ​  ​ 2p h r x
Worksheet 1
1. A liquid will not wet the surface of a solid if the tension of the mixture is (assume contact angle zero
angle of contact is and g = 980 cms–2)
(a) 0° (b) 45° (a) 218 dyne cm–1 (b) 234.18 dyne cm–1
(c) 60° (d) > 90° (c) 107.9 dyne cm (d) 10.79 dyne cm–1
–1

2. A thread is tied slightly loose to a wire frame as 8. A glass capillary tube of inner diameter 0.28 mm is
shown in the figure. lowered vertically into water in a vessel. The gauge
And the frame is dipped pressure that must be maintained above the water
into a soap solution and surface in the capillary tube so that water level in
taken out. The frame is the tube is same as that in the vessel is (surface
completely covered with tension of water = 0.07 Nm–1 and atmospheric
the film. When the por- pressure = 105 Nm–2)
tion A is punctured with (a) 500 Nm–2 (b) 99 × 103 Nm–2
a pin, the thread: (c) 100 × 103 Nm–2 (d) 101 × 103 Nm–2
(a) becomes convex
9. The work done in increasing the size of a rectangular
towards A soap film with dimensions 8 cm × 3.75 cm to 10 cm
(b) becomes concave towards A × 6 cm is 2 × 10–4 J. The surface tension of the film
(c) remains in the initial position in Nm–1 is:
(d) either (A) or (B), depending on size of A w.r.t. B (a) 1.65 × 10–2 (b) 3.3 × 10–2
3. A liquid rises in a capillary tube when the angle of (c) 6.6 × 10–2 (d) 8.25 × 10–2
contact is 10. A capillary tube is dipped vertically in a liquid and
(a) an acute one (b) an obtuse one the liquid rises up to a height of 50 cm. The tube is
(c) p /2 radian (d) p radian now tilted to an angle of 45°. Length of liquid in the
4. In an experiment, water rises to a height 0.1 m in tube is
__
a capillary tube. If the same experiment is repeated (a) 50 cm     cm
(b) 50 ​÷2 ​
in an artificial satellite, which is revolving around (c) zero (d) none of these
the earth, water will rise in the capillary tube upto a 11. A thin metal ring of internal radius 8 cm and external
height of radius 9 cm is supported horizontally from the pan
(a) 0.1 m (b) 0.2 m of a balance so that it comes in contact with water
(c) 0.98 m (d) full length of tube in a glass vessel. It is found that an extra weight of
5. Neglecting gravity, the potential energy of a molecule 7.48 g is required to pull the ring out of water. The
on the surface of a liquid compared with PE of a surface tension of water is
molecule deep inside liquid is: (a) 80 × 10–3 Nm–1 (b) 75 × 10–3 Nm–1
(a) greater (b) less (c) 65 × 10–3 Nm–1 (d) 68 × 10–3 Nm–1
(c) equal 12. A soap film is formed on a vertical equilateral
(d) depending on the liquid sometimes more, triangular frame ABC. Side
sometimes less BC can move vertically always
remaining horizontal. If m is the
6. The surface tension of a liquid is 5 Nm–1. If a film
mass of rod BC, and S is surface
is held on a ring of area 0.02 m2, its surface energy
tension of soap film, the length of
is about:
BC in equilibrium is
(a) 5 × 10–2 J (b) 2.5 × 10–2 J mg mg
–1
(c) 2 × 10 J (d) 3 × 10–1 J (a)  = ___
​    ​ (b)  = ___
​     ​
2 S S
7. In a vessel, equal masses of alcohol (sp. gravity 0.8) 2 mg mg
and water are mixed together. A capillary tube of (c)  = ____
​      ​   (d)  = ___
​    ​
S 3 S
radius 1 mm is dipped vertically in it. If the mixture
rises to a height 5 cm in the capillary tube, the surface 13. There is a 1mm thick layer of glycerine between a
flat plate of area 100 cm2 and a big fixed plate. If the

9.24
coefficient of viscosity of glycerine is 1.0 kgm–1s–1 velocity of the sphere is proportional to which of
then how much force is required to move the plate the following?
with a velocity of 7 c ms–1? (a) r2 (b) r
(a) 3.5 N (b) 0.7 N (c) r3/2 (d) r1/2
(c) 1.4 N (d) None 18. Two drops of same radius are falling through air with
14. The displacement of a ball falling from rest in a steady velocity of v c ms–1. If the two drops coalesce,
viscous medium is plotted against time. Choose a the new terminal velocity will be
possible option. (a) 4 v (b) (4)1/3 v
(c) 2 v (d) 64 v
19. The figure shows a soap film in which a closed
(a) (b) elastic thread is lying. The film inside the thread is
pricked. Now the sliding wire is moved out so that
the surface area increases. The radius of the circle
formed by elastic thread will

(c) (d)

15. A tube of fine bore AB is connected to a manometer


M as shown. The stop cork S controls the flow of air.
AB is dipped into and taken out of a liquid whose
surface tension is s. A
film of liquid is formed,
(a) increase (b) decrease
which closes the end
B. On opening the stop (c) remains same (d) data insufficient
cork for a while, air is 20. There is an air bubble of radius R
forced into the tube and inside a drop of water of radius 3 R.
a bubble is formed at B. The ratio of gauge pressure at point
The manometer level is A to the gauge pressure at point B
recorded, showing a dif- is
ferene h in the levels in (a) 4 (b) 1
the two arms. If r be the density of manometer liquid (c) 3 (d) 2
and r the radius of curvature of the bubble, then the 21. A long vertical capillary of radius R is dipped in a
surface tension r of the liquid is given by liquid having surface tension S. The gauge pressure
(a) rhrg (b) 2rhgr at the mid-point of the liquid column in the capillary
is (take angle of contact = q)
rhrg
(c) 4 rhrg (d) ​ _____
 ​ 
  3 S cos q S cos q
4 (a) ​ _______
    ​   (b) – ​ ______
   


R 2 R
16. A viscous fluid fills the clearance between a shaft
and a sleeve. When a force of 800 N is applied to the S cos q 2 S cos q
shaft, parallel to the sleeve, the shaft attains a speed (c) – ​ ______
   

​ (d) ​ _______
    ​ 
R R
of 1.5 cms–1. If a force of 2.4 kN is applied instead,
the shaft would move with a speed of 22. A long capillary tube of radius ‘r’ is vertically
–1 –1 immersed inside a liquid such that its top end is at
(a) 1.5 cms (b) 13.5 cms the surface. Angle of contact for the solid - liquid
–1
(c) 4.5 cms (d) None pair is 0°. If the tube is slowly raised then relation
17. A solid metallic sphere of radius r is allowed to between radius of curvature of meniscus inside the
fall freely through air. If the frictional resistance capillary tube and displacement (h) of tube can be
due to air is proportional to the cross-sectional area represented by
and to the square of the velocity, then the terminal

9.25
The ratio of excess pressure in the drop to the excess
pressure inside the bubble is
(a) (b)
(  ) (  )
1
__ 1
__
R ​   ​  R ​   ​ 
(a) ​​ ___
​    ​  ​​3 ​ (b) ​​ ___
​    ​  ​​3 ​
3 d 6 d

(c) ​​( ____
1

24 d )
R ​ __ ​ 
​     ​  ​​ ​ 3 (d) None

24. When a ball is released from rest in a very long


(c) (d) column of viscous liquid, its downward acceleration
is ‘a’(just after release). Find its acceleration when
it has acquired two third of the maximum velocity:
a 2 a
(a) ​ __ ​   (b) ​ ___ ​ 
23. A soap bubble has radius R and 3 3
thickness d (<< R), as shown. It a
(c) ​ __ ​   (d) none of these
collapses into a spherical drop. 6

9.26
Worksheet 2
1. When an air bubble rises from the bottom of a deep tube so that the water in the wider tube is at level
lake to a point just below the water surface, the with the water outside.
pressure of air inside the bubble Take angle of contact as zero and surface tension of
(a) is greater than the pressure outside it water = 7 × 10–2 Nm–1.
(b) is less than the pressure outside it (a) height h of water in the narrower limb is nearly
(c) increases as the bubble moves up 0.028 m.
(d) decreases as the bubble moves up (b) if more air is pumped in, level of water in both
2. A sphere of mass m is released from rest in a station- limbs will not get affected.
ary viscous medium. In addition to the gravitational (c) if more air is pumped in, level of water in wider
force of magnitude mg, the sphere experiences a limb will go below the free surface of water in
retarding force of magnitude bn, where n is the speed the tank and water level in other limbs will not
of the sphere and b is a constant. Assume that the get affected.
buoyant force is negligible. Which of the following (d) if more air is pumped in, level of water in wider
statement/s about the sphere is correct? limb will not change but it will go down in the
(a) Its kinetic energy begins to decrease sometime other limb.
after it begins to fall. 5. A sphere of radius ‘a’, is moving in a medium hav-
(b) Its kinetic energy increases to a maximum, then ing coefficient of viscosity h. A force F starts acting
decreases to zero. on it along the line of motion. There is no gravity.
(c) Its speed increases monotonically, approaching (K is a numerical constant)
a terminal speed that depends on b but not on (a) the sphere acquires a constant velocity equal to
m. F
K ​ ___  ​,  if force F is a constant.
(d) Its speed increases monotonically, approaching ah
a terminal speed that depends on both b and (b) the sphere acquires a constant velocity equal
m. F
to K ​ ___   ​, if the force F remains constant in
3. Two balls of same material (density r) but radii ah
r1 and r2 are joined by a light inextensible vertical magnitude and changes its direction once.
thread and released from a large height in a medium (c) the sphere acquires a constant velocity equal to
of coefficient of viscosity h. Neglect buoyancy and
a
assume acceleration due to gravity to be a constant. Kh __ ​   ​,  if force F is a constant.
F
(a) the terminal velocity acquired by the balls (d) none of the above
2 rg 2 6. A metallic sphere of radius 1.0 × 10–3 m and density
is __
​    ​ ___
9 h 1
[  ]
​   ​ ​ ​r​ ​​  – r1 r2 + r​ 2​2​  ​
1.0 × 104 kgm–3 enters a tank of water, after a free
(b) tension in the string when balls are moving with fall through a height h in the earth’s gravitational
terminal velocity is __
4
|  |
​   ​  p r g ​ ​r2​1​ ​ r2 – ​r2​2​ ​ r1  ​
3
field. Its velocity remains unchanged after entering
water. Given: coefficient of viscosity of water = 1.0
× 10–3 Nsm–2, g = 10 ms–2 and density of water =
(c) just after the release, both balls will have same
acceleration 1.0 × 103 kgm–3.
(d) the string will get taut some time after the (a) Value of h is 20 m
release. (b) If water is replaced with a liquid having double
4. The figure shows an inverted U-shaped tube with the coefficient of viscosity of water but same
straight limbs of unequal radii r1 = 0.25 mm and density as water, value of h will be 5 m
r2 =  0.50  mm. Both the open ends (c) If the experiment is repeated on moon (assuming
of the tube are immersed below the we have water there), value of h will be
free surface of water. There is an 10 m (acceleration due to gravity on moon is
opening in the tube at the top. Air 10/6 ms–2)
is pumped into the upper part of the (d) None of the above

9.27
Worksheet 3
1. A thread ring is lying on surface of water in irregular of water = 73 dyne cm–1, contact angle = 0° and
shape. some detergent in added to the surface inside g = 980 cm s–2.
the loop. What happens? 7. A U-shaped glass tube has two limbs of different
radii. The radii are r1 = 1.5 mm and r2 = 3.0 mm. The
tube is inverted so that both ends are dipped inside
water. What is the difference in heights(h) to which
water rises in the two limbs? Surface tension of water
S = 0.07 Nm–1 and contact angle with glass is 0°.

2. Three matchsticks are floating on the surface of


water, forming a triangle. Some soap solution is
poured gently inside the triangle. what happens?

8. A water drop of radius R splits into n identical drops.


3. A cube of side length a and mass m floats in water. Calculate the increase in surface energy. surface
water perfectly wets tension is S.
the material of the cube 9. A narrow capillary tube of radius r = 0.5 mm con-
(it means that contact tains some water. The tube is held vertical. What is
angle is 0°). Find the dis- the height of water column h in the tube if the water
tance h between the lower is about to leak from the lower end?
face of the cube and water
surface. Surface tension of water = S. Surface tension S = 0.075 Nm–1; contact angle = 0°;
g = 10 ms–2.
4. A thin ring of mass m and radius r is resting on
a liquid surface. Surface tension of the liquid is S. 10. Two soap bubbles of same size come into contact.
The ring is slowly lifted while keeping its plane Find the radius of curvature of the common wall.
horizontal. Find the force needed to separate it from
the liquid surface. 11. A capillary tube has conical shape. Its radius decreases
with height. The cone wall makes an angle a with
5. A U-shaped tube has two vertical arms of small
vertical. The lower end
cross-sections. Radii of
of the tube is dipped in
the two arms are r1 and r2
a liquid, which rises in
(< r1). A liquid is poured
it to a height h. At the
in the tube and its height
location of liquid sur-
in two columns is h1 and
face in the tube, radius
h2. Surface tension of the
of circular section of
liquid is S and its contact
the tube is r. Contact
angle with the tube wall is q = 60°. Find h2 – h1.
angle is 90°. Find h if
6. A plate of length 20 cm, width 0.77 cm and thickness density of liquid is r.
2 mm has mass of 8.2 gram. It is held vertically with
long side horizontal and lower-half under water. Find
the apparent weight of the plate. Surface tension

9.28
12. A capillary tube is dipped in water and the liquid Density of glycerine r = 1260 kgm–3 and coefficient
rises to a height h. In an experiment, the capillary of viscosity = 8.0 poise.
tube is dipped in water and the whole experimental 15. A block (A) is placed on a table. There is a 3.0 mm
arrangement is inside an elevator. Length of the tube thick layer of oil between the table and the block.
above the water surface is 2 h. Face of the block in contact with oil has an area
(a) With what acceleration the elevator must be 0.1 m2. The block is connected to another block (B)
moved so that water rises to the brim in the of mass m = 0.01 kg by
capillary? massless string passing
(b) With what acceleration the elevator must be over a smooth pulley
h (see figure). The system
moved so that water rises only to a height __​   ​ ? is released from rest. It
2
accelerates and finally
(c) What is the radius of curvature of the water
attains a constant speed
surface in the tube if the elevator falls with
of 0.085 ms–1. Find coefficient of viscosity of the
acceleration g?
liquid. Neglect viscous force and buoyancy force due
13. With what terminal velocity will an air bubble 0.8mm to air on the blocks. g = 9.8 ms–2.
in diameter rise in liquid of viscosity 0.15 Nsm–2 and
relative density 0.9? Density of air is 1.29 kgm–3. 16. Spherical dust particles are shaken up in water and
allowed to settle. Depth of water in the container is
14. A metal sphere has radius r = 1 mm and mass 2 cm. Estimate the diameter of the largest particles
m = 50 mg. It falls vertically in glycerine. Find remaining in suspension 10 minute later. Density of
(a) Viscous force on the sphere when its speed is dust particles, is d = 1.8 × 103 kgm–3. Assume that
0.5 c ms–1 the dust particles quickly attain terminal speed.
(b) The terminal speed of the sphere

9.29
Answers Sheet
Your Turn
1. 7.5 mN 2. 0.096 Nm–1 3. Spherical 4. 5.9 m J
5. 0.85 mJ 6. Increase in surface area requires energy. 7. 6 m J
8. 2.0274 × 105 Nm–2 9. Both have same pressure 10. 17.5 Nm–2
S3
11. 128 p ___
​  2  ​   15. 1.43 cm 16. 7.14 cm
​P​0​​ 
17. Mercury gets depressed by 3.75 mm

2 S
18. R = ____
​    ​;  P0 + pgh 19. Due to viscous force between different layers 20. 0.6 poise
pgh
21. mg 22. 12 ms–1

Worksheet 1
1. (d) 2. (b) 3. (a) 4. (d) 5. (a) 6. (c) 7. (a) 8. (a) 9. (b)
10. (b) 11. (d) 12. (a) 13. (b) 14. (d) 15. (d) 16. (c) 17. (d) 18. (b)
19. (c) 20. (a) 21. (c) 22. (b) 23. (c) 24. (a)

Worksheet 2
1. (a, d) 2. (d) 3. (a,b,c,d) 4. (a) 5. (a, b) 6. (a, b)

Worksheet 3
1. Thread becomes circular. 2. Matchsticks get separated.
mg 4s S (r1 – r2)
3. ​ _____ ​ + ​ ____  ​  
    4. mg + 4p r S 5. ​ ________  ​ 

6. 9.67 gm. wt.
2
a  rg arg rg r1r2

7. 4.76 mm [  1
__
​   ​ 
8. 4p R2 ​ ​n​3 ​ –1  ​ S ] 9. 6 cm 10. Infinite

2 S sin a g
11. ​ _______    
​ 12. (a) __
​   ​  (Ø) (b) g (≠) (c) • 13. 0.21 cms–1
rrg 2
14. (a) 7.5 × 10–5 N (b) 2.9 cms–1 15. h = 0.034 Nsm–2 16. 8.8 m m

9.30
Chapter  10

Miscellaneous problems
on chapters 8 and 9

MATCH THE COLUMNS Column I Column II

Match the entries in Column I with those in Column II. An (a) Net force exerted by liquid A of (p) 9 r gRh2
density r on the cylinder.
item in Column I can match with any number
of entries in Column II. It may also happen (b) Net force exerted by liquid B of (q) pr gR2h
that an item in Column I does not match with density 2r on the cylinder.
any of the entries in Column II. (c) Net force exerted by liquids A and (r) 4prgR2h
1. A bottle is filled with water, above B on the left half of the curved part
which a little air at atmospheric pres- of cylinder.
sure is present. Plugs are blocking the (d) Net force exerted by liquid A and B (s) 3prgR2h
three small holes (A, B and C). on the cylinder.

Column I Column II 3. Bucket A contains only water. An identical bucket B


(a) A & C are opened (p) Air comes into the bottle contains water, but also contains a solid object in the
from upper-most open water. Consider the following four situations given in
hole column I. Which bucket weighs more?
(a) B & C are opened (q) Water flows out of holes
B and C Column I Column II
(c) Only C is opened (r) Some water comes out and (a) The object floats in bucket (p) Bucket A
the flow stops B, and the buckets have the
(d) All the holes are (s) Pressure of air inside bottle same water level
opened above the water remains (b) The object floats in bucket (q) Bucket B
unchanged. B, and the buckets have the
same volume of water
2. Shown below is a cylinder of radius R floating in (c) The object sinks completely (r) Both buckets have
vessel containing liquids A and B. Neglecting atmo- in bucket B, and the buckets the same weight
spheric pressure, match the quantities mentioned have the same water level
in Column I with corresponding expression in
(d) The object sinks completely (s) The answer can-
Column II. in bucket B, and the buckets not be determined
have the same volume of from the informa-
water tion given.

4. A cubical box is completely


filled with mass m of a liq-
uid and is given horizan-
tal acceleration a, as shown.
Match the force due to fluid
10.2  Mechanics II

pressure on the faces of the cube with their appro- (b) The horizontal range of (q) Is same for h = y and
priate values (assume zero pressure at the point of liquid jet ejected from h=H–y
minimum pressure inside the tank.) the tank

Column I Column II (c) Speed of efflux (r) Will change if the con-
tainer is accelerated
(a) force on face ABFE (p) ​ ma   ​
___ vertically up
2
mg (d) Time needed to empty (s) Is same if A o is
(b) force on face BFHD (q) ___
​     ​
2 the tank for h = H doubled.
ma mg
(c) force on face ACGE (r) ​ ___   ​ + ​ ___   ​ 7. Match Column I with entries in Column II
2 2
(d) force on face CGHD (s) ​ ma   ​ + mg
___ Column I Column II
2
mg (a) Surface tension (p) Conservation of energy
(t) ___
​     ​ + ma
2 (b) Viscous force in (q) Conservation of mass
liquid (r) Terminal velocity
5. A container contains a liquid of density r. It is accel-
erated so as to have a horizontal component of accel- (c) Continuity equation (s) Surface energy
eration equal to g and a vertically upward component (d) Bernoulli’s equation (t) Reynold’s number
also equal to g.
8. A block of ice floats in a liquid
with a part of the ice block remain-
ing outside the liquid. Density of
ice is d and that of the liquid is r.
Also, density of water is rw. Match
the situation described in Column I with results in
Column II.
Column I Column II
(a) r = rw and the ice (p) Level of liquid in the
Match the enteries in Column I with respect to entries melts container falls
in Column II.
(b) r = rw and half (q) Level of liquid in the
Column I Column II the ice melts container goes up
(a) Liquid surface makes an angle a with (p) 4 (c) r > rw and the ice (r) Level of liquid in the
horizontal. tan a = melts container does not change
(b) A wooden block is held under water (q) 2 (d) r < rw and the ice (s) No prediction can be made
with the help of a string tied to the melts about change in level of
base. The string makes an angle b with liquid
horizontal. tan b =
(c) A point A on the left wall is at a height (r) ​ __1 ​
2 cm above the base. B is another point 2 PASSAGE-BASED PROBLEMS
on the base at a distance x from the left Every passage is followed by a series of questions. Every
wall. Pressure at A and B are equal.
question has four options. Choose the most appropriate
Value of x in cm is
option for the questions.
(d) Angle between line AB and free surface (s) 0
of the liquid is f . tan f = Passage 1
6. A liquid is filled in a cylindrical container up to a Hydrometer is used to measure the density of liquids. A
height H and a small hole is made at a depth h below calibrated tube sinks into the liquid until the weight of the
the free surface of the liquid. Cross-section of the liquid it displaces is exactly equal to its own weight. It is
tank is A and that of the hole is Ao (<< A). weighted at its bottom end so that the upright position is
stable, and a scale in the top stem permits direct density
Column I Column II readings. Figure (b) shows a typical type of hydrometer that
(a) The time taken by the (p) Is independent of den- is commonly used to measure the density of battery acid or
liquid surface to reach sity of liquid antifreeze. A large tube has a calibrated hydrometer inside
level of orifice it. The large tube is fitted with a rubber bulb at the top and
Miscellaneous Problems on chapters 8 and 9  10.3 

its lower end has a small opening. The bottom of the large Passage 2
tube is immersed in the battery liquid. The rubber bulb is
A soap bubble is blown at the end of a tube of radius b. The
squeezed to expel air and is then released, like a giant medi-
liquid has surface tension S. Bubble having radius R(b << R)
cine dropper. The liquid rises into the tube. The hydrometer
is formed at the end of the tube. Assume that the bubble
floats in this sample of the liquid.
is essentially spherical in shape and the mouth of the tube
is a small circle intersecting the spherical bubble. Air is
blown inside the tube with velocity v, as shown, to grow
the size of the bubble. Density of air is d. Assume that the
air molecules entering the bubble travel straight and strike
the wall of the bubble. The air molecules collide perpen-
dicularly with the wall
of the bubble and stop.
The force applied by the
air molecules striking the
wall of the bubble pull
the bubble away from
the tube. When this force
exceeds the force applied
by the tube wall on the
bubble, the bubble sepa-
rates. Disregard gravity.
4. The force applied by the air molecules striking the
wall of the bubble is
1
(a) 2pb2 r v2 (b) ​ __ ​  p b2r v2
2
(c) p b2r v2 (d) None
1. For the case shown in figure (a):
5. The radius R at which the bubble separates from the
(a) the depth of the hydrometer submerged in denser ring is
liquid is more
2S S
(b) the apparent weight of hydrometer in denser (a) ____
​  2  ​   (b) ​ ____
   ​ 
liquid is more r v r v2
(c) the depth of hydrometer in denser liquid is (c) b (d) None
lesser
(d) the apparent weight of hydrometer in denser Passage 3
liquid is less A beaker containing water is placed on the pan of a balance.
2. In figure (b), pressure of air above the hydrometer The balance shows a reading of M kg.
is A lump of sugar having mass m is
suspended by a thread in such a way
(a) less than atmospheric
that it get completely submerged in
(b) atmospheric water. The upper end of the thread
(c) more than atmospheric is fixed to the ceiling. After time
(d) cannot be said t1, half the mass of sugar lump gets
3. The condition of stability of hydrometer in any fluid dissolved and at time t2, the complete
will be obtained if sugar lump get dissolved. Density of
(a) buoyant force is equal to weight and centre of sugar (d) is greater than that of water
buoyancy coincides with the centre of mass (r). The thread has negligible volume
and mass.
(b) buoyant force is more than weight and centre
of buoyancy coincides with the centre of mass 6. Reading of the balance at time t1 is
(c) buoyant force is more than weight and centre
of buoyancy lies above the centre of mass
mg r __
(a) Mg + ___ [  ]
​   ​ __
d
​   ​ ​ ​    ​ + 1  ​
2 d r
(d) buoyant force is equal to weight and centre of mg
(b) Mg + ___
​   ​ 
buoyancy lies above the centre of mass 2
10.4  Mechanics II

mg 11. Value of l is
(c) Mg – ___
​   ​ 
2 2F0 F0

mg
(d) Mg + ___
r d
​   ​ __ [  ]
​   ​ ​ __
2 d r
​    ​ – 1  ​
(a) L – _____
​ 
Ar0 g
F0
  ​   (b) L – _____
​    ​ 
Ar0 g
3F0
7. Reading of the balance at time t2 is (c) L – ______
​     ​   (d) L – ______
​     

2Ar0 g 2Ar0 g
r d
(a) Mg + mg __ ​   ​ (b) Mg + mg __ ​    ​
d r
Passage 5
mg
(c) Mg + mg (d) Mg + ___
​   ​  A thin-walled open box of height H floats in water with
2 its height h lying outside water. The cross-section of the
8. Reading of the balance tank is rectangular with dimension a × b. A small hole of
(a) Continuously increases from t = 0 (when sugar cross-sectional area A develops in the base of the box and
was put into water) to t = t2 water begins to enter the box.
(b) Increases in the interval t = 0 to t = t1 and then
decreases in the interval t1 to t2
(c) Decreases first and then increases
(d) Nothing can be said

Passage 4
A cylindrical piece of wood has non-uniform density and
its length and cross-sectional area are L and A respectively.
It floats in water in vertical position in stable equilibrium 12. The height difference between level of water outside
with its length l submerged. To make the wood piece float in and inside the box
horizontal position while remaining completely submerged, (a) remains constant till the brim of the box reaches
we need to apply a vertically downward force F0 at its end the outside water level
(see figure). Density of water is r0. (b) increases till the brim reaches the outside water
level
(c) decreases till the brim reaches the outside water
level
(d) increases or decreases depending on the relative
density of material of the box
13. Speed at which water enters the hole when water
h
inside the box collects to a height __​   ​  is
9. Centre of mass of the wood is 2

÷  ( 
________

)
(a) at the geometrical centre, when it is floating ________ h

(a) ​÷g (H – h) ​
  (b) ​ g ​ H – ​ __ ​    
​ ​
vertically 2
(b) above the geometrical centre, when it is floating _________ 1
vertically  
(c) ​÷2g (H – h) ​
  (d) ​ __ ​  g (H – 2h)
2
(c) closer to the end at which F0 is not applied in 14. Time t in which the box will sink is
horizontal position abh abh2
(a) ​ ____  ​  
___  (b) ​ __________  
    ​
(d) above the water surface in vertical position   ​ 
​÷gH  A2g (H – h)
10. The distance of centre of mass of the wood piece
abh abH
from the end to which F0 is applied is (c) ​ ___________
    ​

_________ (d) ​ __________
     ​
________

A ​÷2g (H – h) ​
   
A ​÷g (H – h) ​

F0L F0L
(a) ​ _____________
     ​ (b) ​ __________
    ​
  Passage 6
2 [ALr0 g – F0] ALr0 g + F0
A rectangular tank has length L and width b. A slit is made
F0L F0L at the bottom of the tank at its left edge. A wooden wedge
(c) ​ _____________
     ​ (d) ​ _______
   

2 [ALr0 g + F0] 2ALr0 g is used to close the slit, as shown in the figure. The width
of the wedge (perpendicular to the figure) is exactly b and it
fits perfectly in the slit. Wedge has mass m and apex angle q.
Miscellaneous Problems on chapters 8 and 9  10.5 

Its vertical surface is in contact with the dry left wall of the 19. Distance from point O where the water jet hits the
container. Coefficient of friction between the container wall horizontal ground is nearly
and the wedge is m. Density of water is r and it is filled to (a) 5.24 m (b) 6.27 m
a height h in the tank. Neglect atmospheric pressure.
(c) 4.90 m (d) 3.95 m
20. The tilted container is accelerated to right with a
horizontal acceleration of a = 2 ms–2. Now the water
surface makes an angle a with horizontal. Value of
a is
(  )
1
(a) tan–1 ​ __
​   ​   ​
8 (  )
1
(b) tan–1 ​ ____
​  __
   ​  ​
  
2​÷2 ​

15. Normal contact force between the wedge and tank


(c) 15°
1
5 (  )
(d) tan–1 ​ ​ __ ​   ​

wall is Passage 8
1 An object is in the shape of a truncated cone. It is floating
(a) ​ __ ​  r gbh2 (b) r gbh2
2 in a lake with its upper surface just exposed to the atmo-
1 sphere. The radii of upper and lower surfaces are r2 and r1
(c) ​ __ ​  r gbh2 tan2q (d) r gbh2 cot2 q respectively and the height of the object is h = 30 m. The
2
16. Vertical component of force applied by water on the slant surface of the object makes 37° with the horizontal.
wedge is It was observed that the magnitude of force applied by
water on the slant (curved) surface is equal to weight of
1
(a) ​ __ ​  r gbh2 (b) r gbh2 the object.
2
1

(c) ​ __ ​  r gbh2 tan2q (d) r gbh2 cot2q
2
17. Maximum height h to which water can be filled in
the tank without leakage from the slit is

÷  ÷ 
____________ ____________
m 2m
(a) ​ ___________
​       ​ ​ (b) ​ ___________
​       ​ ​
r b (tan q – m) r b (tan q – m)
Density of water, r = 103 kgm–3, Atmospheric pressure
​  
÷ 
_________ _________
P0 = 105 Nm–2.
÷ – m)  
m tan q 2m tan q
(c) ​ _________
​    
 ​ ​
  (d) ​ _________   ​ ​
r g (1 – m) r b (1 21. Magnitude of force applied by water on the slant
curved surface of the object
Passage 7 (a) will not change if the object is inverted upside
Water is filled in a container having square base (2 m × 2 m). down
Height of water in the container is h = 2 m. Now the container (b) will increase if the object is tilted upside
is tilted so that its base makes an angle of 30° with the hori- down
zontal. A small hole is made at the bottom of the right wall
of the container. Water ejects from the hole in horizontal (c) will decrease if the object is tilted upside
direction [g = 10 ms–2] down
(d) None of the above
22. Value of r1 is
(a) 10 m (b) 15 m
(c) 30 m (d) 40 m

Passage 9
18. The water surface in the tilted container makes angle Pitot tube is used in wind tunnel experiments and in airplanes
q with horizontal. Value of q is to measure flow speed. The diagram shows a very simple
pitot tube mounted along the axis of a gas pipeline having
(a) 30° (b) 60° cross-sectional area S. Flowing gas has density r. The pitot
(c) 15° (d) 0 tube has a liquid of density r0. The mouth A of the tube is
held normal to the direction of flow. The gas inside the tube
10.6  Mechanics II

is essentially at rest. Difference in height of liquid column Passage 11


in two arms of the pitot tube is h. A hollow cylinder has inner radius R and is held fixed. A
co-axial solid cylinder of radius r and length l is inserted
inside it. The solid cylinder has mass m and is free to rotate
about its axis. Space between the
two cylinders is filled with a liquid
of viscosity h. The solid cylinder
is imparted an angular speed w0
and released. Assume that veloc-
ity gradient in the liquid between
the walls of the two cylinders is
23. PA is pressure just inside the mouth of the tube at A uniform. Also, assume the viscous
and PB is pressure inside the mouth of the tube at B. force to act only on the curved sur-
Then, face of the cylinder.
(a) PA > PB (b) PA = PB 27. Retarding torque acting on the cylinder when its
(c) PA < PB (d) cannot be said angular speed reduces to w is given be t = kw. Value
24. Volume of gas flowing across the section of pipe per of k is
unit time is 2p h L r3 p h Lr3
(a) ​ _______ ​   (b) ​ ______   ​

÷  ÷ 
_____ ______
r0 gh 2r 0 gh (R – r) (R – r)
(a) S ​ _____
​   ​ ​ 
  
(b) S ​ ______
​   ​ ​  

r r 2p h Lr2k p h LR2r
(c) ​ ________ ​  
  (d) ​ _______   ​

÷  ÷ 
____ _____ (R – r) (R – r)
r gh r0 gh
(c) S ​ ____
​   ​ ​ 
  (d) S ​ _____
​   ​ ​ 
   28. Angular speed at time t after the start of motion is
2r0 2r
kt
____ 2kt
​    ​  – ​ ____  ​ 
(a) w0 ​e​mr2 ​ (b) w0 ​e​ mr2 ​
Passage 10
Consider a fluid flowing through a narrow tube in steady
flow. Because of viscosity, the layer in contact with the wall

[  kt
(c) w0 ​ 1 – ____
]
​   2 ​  ​
mr [  2kt
(d) w0 ​ 1 – ____
​  2  ​  ​
mr ]
of the tube remains at rest and the layers away from the wall
Passage 12
move fast. If pressure difference across the ends of a tube
D P Viscosity of a highly viscous liquid can be measured with
of length l is D P, then ___ ​     
​  is defined as pressure gradient an arrangement shown in the figure. A test tube (T) contains
l
along the tube. It has been found that the volume of fluid experimental liquid and is fitted into a water bath (B). A
flowing through the tube in unit time (call it volume flow thermometer is used to measure the temperature of the water
rate Q) is dependent on pressure gradient, radius of the tube bath (thermometer has not been shown in the diagram). A
(r) and coefficient of viscosity (h) of the fluid. tube A is fitted in the cap of test tube T. There are three
In one experiment, the speed of flow along the axis of marks – P, Q and R – on the test tube well below the
tube of radius r0 was measured to be v0. Length of the tube lower end of tube A. Distance between the marks is such
was l0 and viscosity of the fluid used was h. Velocity gradi- that PQ = QR = d◊ A spherical metal ball of mass m and
ent from the tube wall to its axis is assumed to be linear. radius r is dropped in
25. The volume flow rate (Q0) for the tube mentioned tube A. With the help of
above is a stop watch, time inter-
1 val taken by the ball to
(a) p v0r20 (b) ​ __ ​  p v0r20 cross the length PQ and
2
1 1 QR are recorded. Let
(c) ​ __ ​  p v0 r20 (d) ​ __ ​  p v0 r20 these time intervals be
3 4
t1 and t2 respectively.
26. The pressure difference across the tube mentioned
r is the density of the
above can be expressed as (R is a dimensionless
experimental liquid.
constant)
Different balls are tried
Q0l0h0 Q0l0h0
(a) ​ ______ 4
 ​  
  (b) ​ ______  ​ 
  till we get t1 = t2.
k r0 r03
Q0l02h0 Q0l02h0
(c) ​ _______  ​

  (d) ​ ______  ​ 

k r50 k r0
Miscellaneous Problems on chapters 8 and 9  10.7 

[  ]
29. Which of the following is true in the context of the 2
2 t0gr0 _____ 3m
experiment? (a) ​ __ ​  ​ _____
   ​ ​ ​     ​ – r  ​
9 d 4p r03
(a) We can do away with the step of measuring the

[ 
temperature of the water bath. This step is not
]
2
2 t0gr0 _____ 3m
relevant. (b) ​ __ ​  ​ _____
   ​ ​ ​  3  ​ – r  ​
9 d 2p r0
(b) If the time interval t1 π t2 then we should try a

[ 
different ball of slightly smaller radius.


(c) We can have t1 > t2 for some specific balls.
(d) Highly viscous liquid ensures that the balls

2 t0gr0 _____
(c) ​ __ ​  ​ ____
9 d
   
3m
​ ​ ​   
4p r02 ]
 ​ – r  ​

[ 
never attain terminal speed while falling. t0gr20 _____
30. For a ball of radius r0 and mass m0, it was recorded
that t1 = t2 (= t0, say). The coefficient of viscosity of
(d) ​ ____
d
   
m
​ ​ ​   3 ​ – r  ​
6p r0 ]
the liquid at the temperature of experiment is
Answers Sheet
Match the Columns
1. (a) p, s  (b) p, q  (c) r  (d) p, q, s 2. (a) q  (b) r  (c) p  (d) s
3. (a) r  (b) q  (c) q  (d) q 4. (a) p  (b) q  (c) t  (d) s
5. (a) r  (b) q  (c) p  (d) r 6. (a) p, r  (b) p, q, s  (c) p, r, s  (d) p, r
7. (a) s  (b) r, t  (c) q  (d) p 8. (a) r  (b) r  (c) q  (d) p

Passage-based Problems
1. (c) 2. (a) 3. (d) 4. (c) 5. (a) 6. (a) 7. (c) 8. (a) 9. (c)
10. (a) 11. (b) 12. (a) 13. (c) 14. (c) 15. (a) 16. (c) 17. (b) 18. (d)
19. (d) 20. (d) 21. (b) 22. (d) 23. (a) 24. (b) 25. (c) 26. (a) 27. (a)
28. (b) 29. (b) 30. (a)

10.8
Chapter  11

Gravitation
“Pick a flower on Earth and you move the farthest star.’’
–Paul Dirac.

1.  INTRODUCTION 4p2 × 3.85 × 108m


= ​  ___________________
        ​
(27.3 × 24 × 60 × 60 s)2
It will not be wrong to say that Newton did not discover
gravitation. What Newton discovered was that gravitation is 0.0027 ms–2
universal – that it is not special to Earth, as others of his time From his laws of motion, Newton knew that acceleration
assumed. Newton published his brilliant theory in 1687. can be produced only if there is a force and direction of
Before 1687, a large amount of data had been collected acceleration is in the direction of the force. A natural guess
regarding motions of the moon and the planets. Tycho Brahe is that the Earth is attracting the Moon.
(1546–1601) and Johannes kepler (1571–1630) need special According to popular legend, he was sitting under an
mention. This teacher – disciple pair studied the planetary apple tree when an apple fell from the tree. The falling apple
motion in great detail. Kepler formulated three laws of plan- (and the preoccupied mind of Newton, thinking about the
etary motion, based on large amount of data collected. He acceleration of the Moon) sparked the idea that the Earth
stated that all planets go around the sun in elliptical orbits. attracts all objects towards itself, including the Moon.
[We will take up the three laws later in this chapter.] But
The prevailing notion, at that point of time, was that there
clear understanding of the motion of the planets and moon
were two sets of natural laws: one for earthly objects, and
was not available till Newton gave his law of universal
another, altogether different, for heavenly objects. Newton’s
gravitiation.
intuition that the force between an apple and the earth is
In this chapter, we will study the law of universal gravita- the same force that pulls the moon towards earth was a
tion. We will realise that it is the same force that causes an revolutionary breakthrough.
apple to fall downwards and also causes the moon to revolve
Acceleration of a body (such as an apple) falling near the
around earth. It is the same universal gravitational force that
earth’s surface is 9.8 ms–2. Therefore,
is holding a galaxy of stars together and is trying to hold aapple 9.8
together the entire universe (which is expanding). ​ _____ amoon   ​ = ______
​    ​ 
= 3600 ...(i)
0.0027
We will learn about the motion of satellites and planets
and see how kepler’s findings could be explained in terms To understand how the acceleration is affected by distance,
of Newton’s law of gravitation. let us calculate the ratio of distance of the apple and the
moon from the centre of the earth.
rapple
2.  NEWTON’S LAW OF GRAVITATION ​ _____
6400  km
 ​ = ____________
1
 ​ ___
rmoon  ​      
5
3.85 × 10  km
​    ​  
60
...(ii)
Newton was aware that the Earth – Moon distance is close
to r = 3, 85,000 km and it takes T = 27.3 days for the moon Note that the apple is close to the surface of the earth
to make a revolution around the earth. Acceleration of the and its distance from the centre of the earth is equal to the
moon (directed towards the earth) is radius of the earth = 6,400 km.

(  )
2p 2 From (i) and (ii), it can be concluded that
a = w2r = ​​ ___
​   ​   ​​ ​ ◊ r

(  )
T aapple rmoon 2
​ _____
amoon   ​ = ​​ _____
​ r  ​  ​​ ​
apple
11.2  Mechanics II

Newton guessed that acceleration of a body towards earth


is inversely proportional to the square of distance from the
centre of the earth.
1
a µ ​ __2  ​ 
r
Since force is mass times acceleration, force on a body
of mass m due to the Earth is
m
F µ __
​  2  ​
r
From Newton’s third law, force applied by the earth on
a body must be equal to the force applied by the body on
(iii) Force between two objects is independent of the
earth. Therefore, this force must be proportional to the mass
medium between the particles.
of earth as well.
(iv) Gravitational force is conservative in nature.
mM
\ F µ ​ ____ ​ 

r2 Example 1   Gravitation is a weak force
Newton generalised the law by saying that all material Two small bodies, each of mass 1 kg, are placed 1m apart.
bodies in the universe attract one another. His famous law Find the gravitational force they exert on one another.
of gravitation can be stated as: Assuming that the only force acting on the particles is mutual
Any two particles in this universe attract each other along gravitation, find their initial acceleration.
the line joining them. Force between them is proportional
Solution
to the product of their masses and inversely proportional to
the square of the distance between them. Concepts
Gm1m2
m1m2 F = ______
​  r  ​ 

F = G _____
​  r  ​  
  ...(1) 2
2
Gravitational force applied by one particle on another is
G is a constant, called
Gm1m2
Universal Constant of Gravitation and F = ______
​  r  ​ 

2
its value is found to be 6.673 × 10–11 Nm2 kg–2. Nm2
6.67 × 10–11 ____
​  2 ​ × (1 kg) × (1 kg)
It is important to remember the following points: kg
= ​ _____________________________
       
(i) Newton’s law of gravitation gives the force between (1 m)2
two point-masses. For writing gravitational force between
two pens, you cannot directly use equation (1). The pens are  ​ = 6.67 × 10–11 N
not point masses. If the two pens are 1km apart, then they
can be treated as point masses, as distance between them F 6.67 × 10–11
Acceleration, a1 = ___
​ m   ​ = ​ __________
 ​
    
is large compared to their size. There is nothing wrong is 1 1
treating earth to be a point mass while writing the force on
it due to the sun. = 6.67 × 10–11 ms–2
(ii) A spherically symmetrical body can be replaced by Acceleration of the other particle is also of same
a point particle of equal mass placed at its centre for the magnitude.
purpose of calculating gravitational force on an outside Note:  Gravitational forces between objects of daily life
object. We will return to this point later in this chapter. It are too small to be noticeable. Gravitational force is a weak
needs a good amount of calculus to prove this. what this force. It becomes significant only when at least one of the
essentially means is that all celestial objects (which are two masses is huge. For example, the force between your
spherical) can be treated as point masses at their centres for body and the earth or the force between the sun and the earth
the purpose of calculating the mutual force between them. is large and meaningful. There is no point in discussing the
Force applied by the earth on an apple can be calculated gravitational force between a proton and an electron in an
by assuming the entire mass of earth to be located at its atom.
centre, like a point mass.
Gravitation  11.3

Your Turn

Q.1  Distance between two masses is halved. How does Q.4  Find gravitational force between the earth and the
the gravitational force between them change? sun.
Msun = 2.0 × 1030 kg ; Mearth = 6 × 1024 kg ;
Q.2  There is attractive force between all objects. Why do
we not feel ourselves gravitating towards massive buildings Earth–Sun distance, r = 1.5 × 1011 m
in our close proximity?
Q.3  Heavier bodies are attracted more strongly by Earth.
Why don’t they fall faster than lighter bodies?

3.  MEASUREMENT OF GRAVITATIONAL by a fine fibre or a thin metal wire. There is a small mirror
attached to the wire, which reflects a sharp beam of light
CONSTANT G incident on it on to the scale fixed at some distance D from
Value of G was first measured in 1798 by a gifted English the mirror. Initially, there is no twist in the wire.
scientist, Henry Cavendish (1731–1810). Cavendish made Two heavy spheres (A and B), of mass M each, are
many contributions to science but his measurement of G brought near the smaller spheres such that the centres of the
was the most prolific of all. The experiment required a very four spheres fall on a horizontal circle. Let distance between
delicate set up. the centres of a heavy ball and the smaller ball near it be
r.
Gravitational pull of larger ball on the smaller ball is
G Mm
F = _____
​  2 ​  

Torque on the dumb bell due to gravitational force is


t = 2Fl [2l = length of the dumb bell]
2GM ml
= _______ ​  2 ​   
r
This torque causes the dumb bell to rotate and the wire
gets twisted. The twisted wire produces a counter-torque.
After some time, an equilibrium is established with torque
produced by wire balancing the gravitational torque.
If a wire is twisted by q, the torque it develops is given
by t = kq, where k is a constant for the given wire, known
as its torsional constant.
If equilibrium is attained with wire twisted by q,
2GM ml
​ _______  ​  
 = kq
r2
kr2q
fi G = _____
​   
 ​   ...(i)
2Mml
How do we measure q? If the wire turns by q, the mirror
attached to it also turns by q. The spot of the light on the
scale moves from S1 to S2. We measure S1S2. If D is the
S1S2
distance between the mirror and the scale, then q ____​     ​. 
D
[Note that when mirror rotates by q, the reflected ray
rotates by 2q. We will learn more about it in optics]
Two small balls, each of mass m, are attached to the ends Since q is a very small angle, its measurement is going to
of a light rod to form a dumb bell. The rod is suspended be difficult. In an actual experiment, we allow the dumb bell
11.4  Mechanics II

to settle and then move the larger balls so as to place them


on opposites sides of the smaller balls, at same distance (r).
See the figure. Larger balls are moved from position A, B
to A¢, B¢. As the heavy balls are shifted to a new position,
the dumb bell rotates. It will settle at angle q from mean
position on the other side. Thus, the mirror rotates through
2q (and reflected light rotates by 4q) as the heavier balls are
moved to a new position. The spot of light moves a larger
distance and hence 4q can be easily measured. (a) Write net gravitational force (F) on M.
Putting all the values in (i) gives us the value of universal (b) Approximate the value of F for the condition y >> d.
gravitational constant, G. Does the result convince you?
Solution
4. PRINCIPLE OF SUPERPOSITION Concepts
Consider fixed particles (i) We will write force due to each m on M and add
having masses m1, m2, m3 them. We must add the forces as vectors.
... We wish to write net
(ii) Here, we will resolve both the forces into two
force due to all of them
perpendicular components and then add them.
on another particle of
mass M. (iii) y >> d does not mean that y Æ •.
_
​›
Let ​F   ​1 = force on M due It only means that y is large compared to d. We can
to m1, assuming that only use approximations like y ± d y, y2 ± d2 y2 etc.
m1 ​_and M are present (m2, m3 etc. are not there)

​F   ​2 = force on
​_› M ​_› due to m2 when only M and m2 are there (a) Force due to particle at A on the particle at P is
and similarly, ​F   ​3, ​F   ​4 ... are forces due to individual particles
GMm
on M when all other particles are missing. F = _____
​  2 ​  
  (along PA)
The principle of superposition says that​_ net​_ gravitational r
› › ​_›
force on M is obtained by vector sum of ​F   ​1, ​F   ​2, ​F  ​3  ... etc. Force due to particle at B is
​_› ​_› ​_› ​_› GMm
\ ​F  ​ net = ​F  ​ 1 + ​F  ​ 2 + ​F  ​ 3 + .... ...(2) F = _____
​  2 ​  
  (along PB)
r
In other words, the force between m1 and M is independent
of the presence or absence of other particles (m2, m3 ...
etc)
If we wish to find force due to an extended body on a
point mass M, then we divide the
body into parts small enough to
treat as particles. We need to write
force due to each small part on M
and add the forces vectorially.
We resolve the two forces along x- and y-direction shown.
​_› ​_› The x-components cancel out and the y-components add
​F  ​   = Ú d ​F  ​ 
up.
Integration is performed so as to cover all the particles \  Resultant force is F0 = 2F cos q (in y-direction
in the extended body. towards O)
Things would have been very difficult if force between 2GMmy
2GMm __y _______
two particles changed when a third particle was brought \ F0 = ______
​  2 ​ ◊ ​ 
 r ​ = ​  3 ​ 
   
r r
closer. ______
But r = ​÷d  2 + y2 
 ​
Example 2   Two point-masses, m each, are kept fixed at y
\ F0 = 2GMm _________
​  2  2 3/2 
 ​
a separation 2a. There is another point mass M placed on (d + y )
perpendicular bisector of the line joining two masses, at a (b) For y >> d
distance y.
y2 + d2 y2
Gravitation  11.5

y 2GMm
\  F0 2GMm _____
​  2    ​ = ______
​  2 ​ 
  Mass of each element = lrdq
(y )3/2 y Force due to each element on mass m at O is
When M is placed at a large distance (y) compared to d,
the two particles will appear to be very close, as seen from
M. They will appear like a single point of mass 2m, kept at
a distance y. Thus, the force they will exert on M is
GM (2m)
F0 = ________
​   ​ 

y2
Therefore, the result obtained is absolutely convincing.

Example 3   A circular arc


A circular arc of radius r subtends
an angle f at the centre (O). The
linear mass density of the arc is l G m (l r dq) _____
G ml
dF = ​ __________
 ​
    = ​  r   
​ dq
kgm–1. Find the gravitational force r 2
it exerts on a point mass m, kept Direction of the two forces are as shown.
at O.
Obviously, the x-components of these two forces cancel
Solution out and y-components will add.
Concepts x-component of forces by any two symmetrical elements
will cancel out. Resultant force is in y-direction.
(i) We will consider a small element on the arc y-component of force due to one element is
subtending a very small angle dq at the centre.
Gml
We can write force on m due to this element. dFy = dF cos q = _____
​  r   
​ cos q dq
(ii) We need to find vector sum of forces due to
all small elements. These forces have varying \  Resultant force is
directions.
(iii) Considering another symmetrical element (as F = Ú dFy
shown below) helps us to visualise that direction
Gm l f /2
of resultant force is along the line of symmetry = ​ _____
r   
​  Ú cos q dq
(y-axis). – f /2

We need to take component of force due to each Gml f/2


element in y-direction and add them. Addition will be done \ F = _____
​  r   
​ [ 
​​ sin q ]​​– f/2  ​ 
using integration.

Consider two elements at angle q from y-axis as shown.


Angular width of each element is dq.
Gml
= ​ _____
f
[  f
(  ) ]
​ ​ sin ​ __ ​  – sin ​ – ​ __ ​   ​  ​
r    2 2
Length of each element = rdq 2Gml f
F = ​ ______​ sin ​ __ ​ 
r    2

Your Turn

Q.5(a)  Three particle of masses (b)  Three point masses, m each, are placed at the vertices
m, m and M are placed at the vertices of an equilateral triangle of side a. Find net gravitational
of an isosceles right-angled triangle, force on any one mass.
as shown in figure. Equal sides of the
Q.6  Each of the five vertices of a regular pentagon has
triangle have length x. Find gravita-
a point mass m kept at them. A sixth particle of mass M
tional force on M.
is placed at the centre. Distance of each vertex from the
centre is x. Find
11.6  Mechanics II

(a) Gravitational force on M Q.7  A point mass m is at


(b) magnitude of gravitational force on M, if a particle a distance L from one end
is removed from one of the vertices. of a uniform rod of mass M
and length L. Find gravitational force on the point mass
due to the rod.

5.  GRAVITATIONAL FIELD


The sun applies force on the earth. They are not in contact.
Similarly, two magnets, which are at some distance, apply
force on one another. These are not contact forces (like
tension in a string or normal reaction). These are action at
a distance. In physics, such action at a distance forces are
often called as field forces. Such forces are described in
_
​› _
​› _
​› _
​›
terms of something known as field. ​E  ​   = ​E  ​ 1 + ​E  ​ 2 + ​E  ​ 3 + ...
The gravitational force between two masses is assumed to ​_› ​_› ​_›
be set up in two steps – (i) a body A creates a gravitational Where ​E  ​1  , ​E  ​2  , ​E  ​3  , etc., are fields due to individual point
field in the space around it. (Think of it as some sort of masses.
invisible change that the body creates around itself). (ii)
Example 4   What is gravitational field intensity due to
when a body B is placed near A, this fields exerts a force
the earth at a point close to its surface?
on it.
Solution  Force on a particle of mass m near the surface
If a point mass​_› m placed at a point P experiences a
of the Earth is
gravitational
​_› force ​F   ​, then we define intensity of gravitational
field (​E   ​) at point P as F = mg towards the centre of the earth (i.e., verti-
_
​› cally downward direction)
_
​›
__​F  ​ 
​E  ​   = ​ m  ​ ...(3) F
\  Gravitational field, E = ​ __
m ​  = g
In a much simpler language, the intensity of gravitational
Thus, intensity of gravitational field near the surface of
field at a point is the gravitational force experienced by a
the earth is equal to the acceleration due to gravity.
unit mass placed at that point. Quite often, the intensity of
gravitational field is abbreviated as gravitational field. Its SI Example 5   A square of side
units is N kg–1 (which is same as ms–2). length x has four particles placed at
its vertices, as shown in the figure.
Find gravitational field at the cen-
tre of the square. How much force
does point mass M, placed at the
centre of the square, experience?
Consider a fixed point mass M. It produces field around Solution
itself. What is the intensity of field at a point P, which is at
Concepts
distance r from M? A mass m placed at P will experience
GMm We will write field due to individual masses and then add
a force F = ​ _____ ​ 
 towards M. Thus, gravitational field at P them vectorially.
r2
due to M is
x
F GM Let AP = r = ___
​  __  ​ 
E = __
​ m ​  = ____
​  2 ​ 
 directed towards M ...(3a) ÷   
​ 2 ​
r Field at P due to m is
This is the gravitational field due to a point mass at a
distance r from it. It is always directed towards the point
mass.
The field obeys the principle of superposition. If M1,
M2, M3 .... etc., are fixed point masses, then together, they
produce a field at P, given by
Gravitation  11.7

​__›
GM Solution
E = ____
​  2 ​ 
 (along PA​
​  ) 
r Concepts
Field at P due to 2m, 3m and 4m are
​___›
(i) Due to symmetry, the field at P will be along the
G 2m
​ _____ ​

  = 2E (along ​  ) 
PC​ axis directed towards O. This can be seen easily
r2 by considering two identical mass elements on the
​___› ring at diametrically opposite ends.
G 3m
​ _____ ​

  = 3E (along ​  ) 
PB​
r2 (ii) Summing up the component of field along PO due
​___› to various element on the ring gives the resultant
G 4m
​ _____ ​

  = 4E (along PD​​  )  field.
r2
Consider a small element of mass dm at A. Field due to
this small mass at P is

Gdm
dE = _____
​  2 ​ 
 directed along PA
r

​___›
Resultant of E and
​___›2E is E along PC​
​   and resultant of 4E
and 3E is E along PD​
​  .  _​ __›
Resultant of these two fields is parallel to ​BC​  and has
magnitude
__ __ __
__ ÷     Gm
​ 2 ​ ÷     Gm
​ 2 ​     Gm
2​÷2 ​
    E = ​ ______  = ​ ______ = ​ _______
(  )
E0 = ÷
​ 2 ​  ​  x__ 2 ​   ​ 

r2 ___
​​ ​    ​  ​​ ​
2
÷   
​ 2 ​
If we place a unit mass at P, it will experience a _​ net __› Consider identical element at B. Field due to this element
gravitational force equal to E0 in the direction parallel to BC​
​  .  at P is also dE, directed along PB.
If a mass M is kept at P, force on it is Components of dE in direction perpendicular to the axis
__
    GmM
2​÷2 ​ ​___› cancel out. This is true for all pairs of masses at diametrically
F = ME0 = ​ _________ ​ 
 along ​
BC​    opposite ends.
x2
Resultant field is obtained by adding components along
Note:  Take a note of how we are thinking. We said that the the axis due to each elemental mass on the ring.
four masses at A, B, C and D created a field at P and now
dm x Gx
the field applies a force on a mass (M) placed at P. \ E = Ú dE cos q = Ú G ___​  2 ​  ◊ ​ __r ​ = ___
​  3 ​ Ú dm
r r
Example 6   Gravitational field on the axis of a ring
GMx _________GMx
A uniform ring has mass M = _____ ​  3 ​ 
 = ​  2     ​ along PO
r (a + x2)3/2
and radius a. Calculate the
gravitational field due to this Notice that r is same for all elemental masses on the
ring at a point lying on its ring.
axis, at a distance x form its Note:  If you place a mass m at P, it will experience a
centre. gravitational force mE towards O.

Your Turn

Q.8   An equilateral triangle Q.9  Find field at the centre of a uniform ring of mass M
has point mass placed at its each and radius R.
vertex. Each mass is m and side Q.10  Find field at the centre of a uniform semi-circular
length of the triangle is x. Find ring of mass M and radius R.
gravitational field intensity at
mid-point (D) of side BC.
11.8  Mechanics II

5.1  Gravitational field due to spherical bodies If x < r, the point P is outside the
shell. For writing field at P due to the
Celestial bodies are spherical in shape. In gravitation, spheri-
shell, we can replace the shell with a
cal geometry is most important. Here are two results related
point mass at O.
to spherical geometry, without proof.
If x > r, the point P is inside the shell.
We will learn how to prove this in chapter of
Field at P due to such a shell is zero.
Electrostatics.
Thus, field at P is contributed by shells having radii
1. A thin uniform spherical shell can be treated as a
0 < x < r. Mass of the all these shells can be through to be
point particle of same
at O. Combined mass of these shells is equal to the mass
mass, placed at its centre,
of solid sphere of radius r and is given by
for calculation of gravita-
tion field at an external m = (density) (volume)
point. Field at Point P due M __ 4 Mr3
to shell of mass M shown = _____ ​     ​   ◊ ​   ​  p r3 = ____
​  3 ​ 
4
__ 3 R
in the figure is ​   ​  p R3
3
GM Field at P = Field due to point mass m at O
E = ____
​  2 ​ 

x Gm GM
\ E = ____
​  2 ​ = ​ ____ ​  r ...(6)
2. Gravitational field inside a uni- r R3
form spherical shell is zero due to
Direction of the field is towards the centre of the
the mass of the shell. If a mass
sphere.
is placed inside a spherical shell
at point P, it will experience no Example 7   E is gravitation field at a distance r from the
force. centre of a uniform solid sphere of mass M and radius R. Plot
a graph showing variation of E with r, for 0 £ r < •.
5.1.1  Gravitational field due to uniform solid sphere
Solution
Using the above two results, we can easily find the gravita- Concepts
tional field indensity due to a uniform solid sphere. GM
For inside points:  E = ____
​  3 ​  r
Field at an external point R
GM
Consider a solid sphere of mass M and radius R. We need For outside points:  E = ____
​  2 ​  
to find the gravitational field r
at an outside point P at a dis-
tance r from centre O.
The sphere can be divided
into numerous thin shells
centred at O. One such shell
is shown in the figure. Field
at P due to any such shell can
be obtained by replacing the shell with a point mass at O.

(  )
Thus, all such shells can be replaced with a point mass at O. GM
It implies that the entire sphere of mass M can be replaced For 0 £ r £ R: E = ​ ____
​  3 ​  ​ r
R
with a point mass M placed at O. fi  E µ r
\  Field at P is At r = R  (i.e., on surface)
GM
E = ____
​  2 ​  
  ...(4) GM GM
r Esurface = ____
​  3 ​  ◊ R = ____
​  2 ​ 
R R
Field on the surface of the sphere is
GM GM
Esurface = ____
​  2 ​   ...(5) For  r ≥ R; E = ____
​  2 ​   
R r
Field at an internal point 1
P is a point inside a solid sphere of mass M and radius R. fi  E µ __​  2  ​ 
r
Distance of P from centre (O) is r.
Graph is as shown.
Once again, we will think of the sphere as composed of
many concentric shells. Suppose mass of one such shell is
dm and its radius is x.
Gravitation  11.9

Your Turn

Q.11  E is gravitational field at a distance r from the centre Q.13  Consider Earth to be a uniform sphere. At what
of a uniform thin spherical shell of mass M and radius R. Plot depth from the surface of the earth, the gravitational field
a graph showing variation of E versus r, for 0 £ r < •. intensity is half its value on the surface? Radius of the
earth is R.
Q.12  Mass of the Moon is 7.36 × 1022 kg and its radius
is 1.74 × 106 m. Assuming it to be a uniform sphere, find
gravitational field on its surface.

6. ACCELERATION DUE TO GRAVITY (g) 6.2  Variation in ‘g’ on the surface of Earth
The gravitational pull of earth on a body is often referred Acceleration due to gravity at a point on the surface of the
to as gravity. Acceleration produced by this force is called Earth differs from the value predicted by equation (7), due
acceleration due to gravity. When a body of mass m is near to various reasons. Prominent reasons are:
the surface of Earth, force of gravity acting on it is 1. Non-uniform Earth:  Earth is not a uniform sphere. On
GMm the surface, we have mountains, valleys, seas, etc. Moreover,
F = _____
​  2 ​ 
 towards the centre of earth the density of Earth is not uniform. The density in the inner
R core is as high as 14 kgm–3 and the density of the mantle
Thus, acceleration due to gravity (denoted by g) near the ranges from 3 kg m–3 to 6 kg m–3. The crust has density vary-
surface of Earth is ing from region to region over the surface of Earth. Thus,
varies from region to region over the surface.
2. Non-spherical Earth:  Equation (7) assumes that Earth
is a sphere. We known that it is not. Earth is flattened at
the poles and bulging at the equator. Its equatorial radius is
larger than its polar radius by nearly 21 kilometres. A point
on the pole is closer to the denser core of Earth as compared
F GM to a point on the equator. Accordingly, value of g is higher
g = __
​ m  ​ = ____
​  2 ​   ...(7)
R at poles than at the equator.

The above expression has been written assuming Earth to 3. Rotation of the Earth: Earth is rotating about an axis
be a uniform sphere. running through its poles. Consider a ball of mass m sus-
Measured value of g near the surface of Earth is pended from the ceiling of a room using a spring balance,
nearly 9.8 ms–2. at a place on the equator.
Acceleration due to gravity (g) has the same value as
gravitational field intensity due to Earth.

6.1  ‘‘Weighing’’ The Earth


Value of acceleration due to gravity (g) can be measured
easily with the help of simple experiments. Radius of Earth
(R) is also known. The day Cavendish measured the value of
universal constant G, the mass of the Earth became known.
It is often said that Cavendish was the first person to weigh
the Earth. Gravitational pull on the ball is

Putting g = 9.8 ms–2, G = 6.67 × 10–11 N m2 kg–2, GMm


Fg = _____
​  2 ​ 
 = mg
6
R = 6.4 × 10  m in equation (7) gives the mass of Earth R
as If Earth were not rotating, the reading of the spring bal-
M 6 × 1024 kg ance (i.e., tension in the spring) would have been mg.
11.10  Mechanics II

However, in the reference frame of the rotating Earth, Example 8   Effect of rotation is small
there is a centrifugal force (= mw2R) acting on the ball in Find the percentage change in value of acceleration of free
a radially outward direction. Thus, the force applied by the fall at a place on the equator due to rotation of Earth. Value
spring balance on the ball is GM
of ____
​  2 ​ = 9.80 ms–2. Radius of the Earth R = 6.4 × 106 m.
FS = Fg – mw2R R
Solution
FS = Weight recorded by the balance = mg¢
Concepts
Where g¢ is the effective acceleration due to gravity at the g¢ – g
place. (i) Percentage change is ​ _____​ × 100
g   
\ mg¢ = mg – mw2R 2p rad
(ii) Angular speed of Earth is w = ______
​    
​ 
24 h
fi g¢ = g – w2R ...(8)

The above equation gives the value of free fall accelera- Change in g due to rotation
tion at the equator. It is less than the gravitational accelera-
tion (g). D g = g¢ – g = (g – w2R) – g = – w2R
This effect is less pronounced at higher latitudes, mainly D g
because the radius of the latitude circle decreases as one Fractional change = ___
​  g   ​ 
moves from the equator towards the pole. At poles, there is
no effect of rotation of the Earth, as a point on the pole is D g w2R
Percentage change = ___
​  g   ​ × 100 = – ​ ____
g    ​ × 100
rotating on a circle of zero radius.

( 
Note that due to non-sphericity of the Earth also, value
of g is higher at poles. In general, measured value of g is ​ 
2p
= – ​​ ___________

24 × 60 × 60
2
) 6.4 × 106
    ​  ​​ ​ × ​ ________
9.8
 ​ 

× 100 = – 0.34%
highest at poles and smallest at the equator.
You can also refer to our discussion in this regard in the Negative sign indicates decrease.
chapter of circular motion.

Your Turn
Q.14  Suppose you could increase the angular speed of on the equator feels lighter and lighter. What will be the
rotation of Earth. You go on increasing the speed and a man length of the day if he starts feeling weightlessness?

6.3  Acceleration due to gravity at a height from the Note that acceleration due to gravity has the same value
surface of the Earth as the gravitational field intensity due to earth. Also note that
GM
Consider a body of mass m we have reserved the symbol g for ____​  2 ​ 
, value of acceleration
located at a point P at a dis- R
due to gravity on the surface. Any changed value of g is
tance x (> R) from the centre of
being represented by g¢.
the earth. Height of point P from
the surface is h. Using (i) in equation (9) gives

GM GM
x = R + h ...(i) g¢ = _______
​   2   ​ = __________
​    2 ​
[ 

Gravitational force on m due to earth (mass = M) is
(R + h) h
R2 ​​ 1 + __]
​    ​  ​​ ​
R
GMm g
F = _____
​  2 ​ 
  fi g¢ = ________
​    2   ​ ...(10)
x
\  Acceleration produced due to this force is
( 
h
​​ 1 + __ )
​   ​   ​​ ​
R
When h << R, we can approximate the above expression
GM in the following manner:
g¢ = ____
​  2 ​  
  ...(9)
x
[  h – 2
g¢ = g ​​ 1 + __ ]
​    ​  ​​ ​
R
Gravitation  11.11

Using binomial expansion: Solution


n (n – 1) 2 Concepts
(1 + x)n = 1+ n x + ​ _______
 ​ 
 x + ... g
2! (i) g¢ = ​ _______
  2   ​
[  2h
g¢ = g ​ 1 – ___ (  )
h 2
​   ​ + 3 ​​ __ ]
​   ​   ​​ ​ + ...  ​ (  )h
__
​​ 1+ ​   ​   ​​ ​
R

( 
R R

(  ) (  )
h 2 h 3
The terms having ​​ __
​    ​   ​​ ​, ​​ __
​   ​   ​​ ​, etc., are very small and can
(ii) When h << R,  g¢
2h
g ​ 1 – ___ )
​   ​   ​
R
R R g g
be neglected. 4
(a) g¢ = __________
​     2 ​ = _______
​    2   ​ = __
​   ​  g
\  g¢ g ​( 1 – ___
​   ​  )​
2h
R
...(11) (  3200
_____
​​ 1 + ​ 
6400 ) (  ) 1
__
 ​  ​​ ​ ​​ 1+ ​    ​  ​​ ​
2
9

For h = 3,200 km, one should not use equation (11),


Note that equation (10) is the true equation and can be
as h is not small.
used for any value of h. Equation (11) is an approximated
version of (10) for the ease of calculation and should be (b) 32 km << 6400 km
used only when h << R.
\  ( 2h
g¢ g ​ 1 – ___ ) (  2 × 32
​   ​   ​ = g ​ 1 – ​ ______ ​  
R 6400
​ )
6.4  Acceleration due to gravity inside the Earth
Imagine a particle kept inside a tunnel, at a depth h from = g (1 – 0.01) = 0.99 g
the surface of the earth. Distance of the particle from the
Example 10   At what distance from the centre of the earth
centre of the earth is
is acceleration due to gravity half its value on the surface?
Radius of Earth is R.
Solution
Concepts
Acceleration due to gravity
changes with distance from
the centre, as shown in the
x = R – h graph. There are two points
(one inside the earth and
Acceleration experienced by the particle due to gravity
the other outside it) where
will be same as the gravitational field at the point, which is
g is half its value on the
given by equation (6).
surface.
GM
\ g¢ = ​ ____ ​  ◊ x ...(12) For a point inside Earth, equation (12) gives
R3
x
g¢ = g ​ __ ​ 
or,
GM (R – h) ____
g¢ = ____
​  2 ​ ​ ______
R R
   
GM
R
( 
h
​ = ​  2 ​ ​ 1 – __
​    ​  ​
R ) g x1
R

fi ​ __ ​ =
  g ​ __ ​ 

( 
2 R

h
g¢ = g ​ 1 – __ )
​   ​   ​ ...(13)
R

R fi x1 = __
​   ​ 
2
At the centre of the earth, h = R and g¢ = 0. The varia-
tion of acceleration due to gravity with distance from the For a point outside Earth, equation (9) gives
centre of the earth is same as depicted in the graph of 2
GM ____ GM    ___
example 7. g¢ = ____
​  2 ​  ◊ ​ R  ​ 
 = ​  2 ​ 
x R x2
Example 9   Be careful while using equation (11)
R2
Acceleration due to gravity on the surface of Earth is g. Find fi g¢ = g ◊ ​ ___2 ​ 
its value at a height: x
(a) 3,200 km (b) 32 km g __
R2
from the surface of Earth. Radius of Earth is \ ​ __ ​ =
  g ____
​   2 ​   fi  x2 = ÷    R
​ 2 ​ 
2 (x2)
R = 6,400 km.
11.12  Mechanics II

Your Turn

Q.15  At what depth below the surface of Earth is the value Q.17  Assuming Earth to be spherical, at what height
of acceleration due to gravity same as its value at a height above the south pole, acceleration of free fall is same as
h = R, where R is radius of Earth? that on Earth's surface at the equator?
Q.16  At what height from the surface of Earth will the Q.18  Value of g on the surface of Earth is 9.8 ms–2. Find
value of g get reduced by its value on the surface of a planet whose density and radius
(a) 36%  (b) 0.36% both are twice that of Earth.
from its value at the surface? Radius of Earth = 6,400 km.

7.  GRAVITATIONAL POTENTIAL ENERGY In Short:


(i) any two masses in the universe attract each other by
Gravitational force is conservative. We can define a potential
a force known as gravitational force. Force between
energy corresponding to it.
two masses is given by
7.1  Gravitational force is conservative GMm
F = _____
​  2 ​ 

Consider a fixed mass M. A point mass m is at A, at a r
distance r1 from M. It is moved from A to B, where B is when (a) both masses are point masses,
a point at a distance r2 from M. The point mass is moved
(b) both masses are spherical
along a path ACDEFGHIB, where AC, DE, FG and HI are
(c) one mass is spherical and other is point
circular arcs centred at M and CD, EF. GH and IB are radial
mass.
displacements towards M.

Obviously, no work is done by gravitational pull of M on (ii) Gravitational force obeys principle ​_› of superposition.
m, in the segments AC, DE, FG and HI. Gravitational force If force
​_› due to m 1 on M is ​F  ​

1 and that due to m2 on
is perpendicular to these displacements. Gravitational force M is ​F   ​2, then net
​_› force
​_› on M in the presence of m1
does work in the segments CD, EF, GH and IB only. and m2 both is ​F   ​1 + ​F   ​2.
Note that CD + EF + GH + IB = r1 – r2. Work done (iii) Gravitational field at a point is defined as the force
will remain same for any other path between A and B. Any experienced by a unit mass placed at that point.
path can be approximated as number of arcs and radial Gm
displacements. If the path is a smooth curve, it can be (iv) Field due to a point mass is given by E = ____ ​  2 ​ 
r
approximated as infinite number of radial and tangential Field is directed towards m.
(circular arc) displacements. Work is done by gravitational
force only when there is a radial movement and this work
done will depend on the distance of initial and final posi-
tions. It will not depend on whether the movement has taken (v) Gravitational
_
​› ​field
_ › ​ › obeys the principle of superpo-
_
place along radial line l1 or l2. sition. If ​E   ​1, ​E  ​ 2, ​E  ​ 3 are fields at a point P due to
Therefore, we can say with certainty that work done by individual masses m1, m2 and m3, then resultant field
gravitational force between A and B is path independent and at P in presence of all three masses is
the force is conservative.
Gravitation  11.13

​_› ​_› ​_› ​_› r Æ •


​E  ​   = ​E  ​1  + ​E  ​2  + ​E  ​3  \ U• – Ur = – Wg

(vi) Field due to a uniform spherical shell at an inside r Æ •


fi 0 – Ur = – Wg
point is zero. Field at an outside point (r ≥ R) is
r Æ •
GM fi Ur = Wg ...(14)
E = ____
​  2 ​ 

r In words, the PE of the system of two point-masses at
(vii) Field at an inside point of a uniform solid sphere separation r is equal to work done by the gravitational force
is in moving the masses to infinite separation from present
GM separation r.
E = ____
​  3 ​  ◊ r  where r £ R
R One can also define it as

[ 
GM
At an outside point, E = ____  where r ≥ R.
​  2 ​ 
]
r Æ • r Æ • • Æ r
r Ur = Wg = – ​W​ext Agt  ​ ​ = ​W​ext   ​ 
Agt
​  ​ ...(15)
​         ​ ​         ​
(viii) Acceleration due to gravity has the same value as ​ slowly ​ ​ slowly ​
gravitational field intensity at a point.
In words, the PE of a pair of point masses at separation r
(ix) Acceleration due to gravity at various points is given
is negative of work done by external agent in slowly moving
by
the masses from present separation to infinite separation.
GM
(a) on surface of the Earth: g = ____ ​  2 ​  While calculating the work done, we can assume one of
R
the masses (say M) to be fixed. It makes no difference to the
(b) At height h above the surface: PE, as it will depend only on separation r and not on the
g fact whether the masses are located in Delhi or in London.
g¢ = ________
​    2   ​
h
​​ 1 + __ (  )
​    ​  ​​ ​
R
Keeping M to be
fixed, let us assume
that m is moved away
g ​[ 1 – ___
​   ​  ]​ 
2h starting from a separa-
when  h << R
R tion r.

h
(c) At depth h from the surface: g¢ = g ​ 1 – __ [  ]
​   ​   ​
R
When separation is x, the gravitational force is
GMm
(x) Due to rotation of the earth, the value of free Fg = _____
​  2 ​ 

fall acceleration that we feel is slightly less than x
acceleration due to gravity. At equator: Work done by gravitational force when m is further
moved by dx is
g¢ = g – w2R
GMm
At poles, there is no effect of rotation. dWg = – Fg ◊ dx = – ​ _____ ​ 
 dx
x2
Work done by gravitational force when m moves from r
7.2 Gravitational Potential Energy of two point-
to • is
masses •
Gravitational PE of two point-masses is assumed to be zero
when they are far apart (i.e., at infinite separation).
Wg
r Æ •

x = r x
dx 1 •
= – GMm ​Ú  ​ ​ ___2 ​  = GMm ​​ __ [  ]
​ x ​  ​​r​ ​

= GMm ​ __
1 1
​ •  ​ – __
GMm
[ 
​ r ​   ​ = – ​ _____
r   ​  ]
From equation (14)

Let PE of the system be Ur when the masses are at r Æ • GMm


Ur = Wg = – _____
​  r   

  ...(15)
separation r.
Based on your discussion in the chapter of work, energy Note:  1. we wrote the gravitational PE of a body near the
and power, we know that change in PE is defined as negative surface of Earth as mgh, where h was the height measured
of work done by the conservative force (here the gravita- from a reference level. While writing the expression U = mgh,
tional force). we considered the gravitational force to be constant, as h
was small.
11.14  Mechanics II

The expression (15) is very general in nature. It is not Earth partically remains unmoved. The entire KE belongs
exclusive for Earth and another object. It is true in general to the ball. Any decrease (or increase) in PE of the system
for any pair of point masses in this universe. results in an increase (or decrease) in KE of the ball only.
GMm
2. When two particles are brought closer, the PE We can say that the PE, U = – ​ _____ ​ belongs to the ball.
r   
decreases (just like U = mgh decreases when a mass falls
towards Earth). Since U = 0 for r = •, the PE is negative
Example 11   Work done in separating particles from one
(i.e., less than zero) for any finite separation and becomes
progressively more negative as the particles move closer. another
Three particles, each of mass m, are kept at the vertices
7.3 Gravitational PE of a system of multiple of an equilateral triangle of side length x. Find the work
particles needed to separate the particles and place them far away
from one another.
When a system has more
than two particles, the PE Solution
of the system is the sum of Concepts
PE of each pair considered
separately. (i) We can write PE of the system using equation
For a system having three (16).
particles of masses m1, m2 and (ii) PE of the system is negative. When all the particles
m3 are placed at infinite separation, the PE becomes
zero.
U = U12 + U13 + U23
To increase the PE, from a negative number to

(  )
Gm1m3 _______
Gm1m2 ______ G m2m3 zero, an external agent has to do work.
= – ​ ______
​  r  ​ 
 + ​  r  ​ 
 + ​  r  ​  
 ​ ...(16) (iii) Amount of work done is equal to change in PE of
12 13 23
the system.
7.4 Spherical bodies Wext = Uf – Ui
How will we write the potential energy of a system
comprising the spherical Earth (mass M) and a particle (mass
m) kept at a distance r (≥ R) from the centre of Earth?
PE of this system is the work done by gravitational force
in moving the particle from distance r to •, while keeping
Earth fixed. The expression of PE will be same as equation
(15), as the spherical Earth applies the same force on m as
a point mass placed at the centre (O).
PE of the system is
Gmm _____ Gmm _____ Gmm
Ui = – ​ _____​ – ​  x   
x    ​ – ​  x   

3Gm2
= – ​ _____
x   
​ 

Final PE at • separation is: Uf = 0

3Gm2
GMm \ Wext = Uf – Ui = ​ _____
x   


\ U = – ​ _____
r   ​   ...(17)

Note:  A ball of mass m is placed at a distance r from Example 12   Two particles are in space, separated by a
the centre of the earth. PE of the system (Earth + ball) is distance r. Masses are 2M and M. With the particle of mass
GMm
– ​ _____
r   ​.  However, in such cases, where one mass is fixed, 2M held stationary, the other
particle is released. It moves
we can take the liberty of saying that this PE belongs to
under gravitational pull of mass
the movable object. If the ball is allowed to fall, PE of the
2M. Find the speed of M when separation between the par-
system decreases and KE increases. Owing to its large mass, r
ticles reduces to __
​    ​. 
2
Gravitation  11.15 

Solution 2GM
fi 3v2 = _____
​  r   
​ 
Concepts

÷ 
______
2 GM
(i) Gravitational force is conservative. Mechanical fi v = ​ __
​   ​  ____
​     
​ ​ 

3 r
energy of M is conserved.

÷ 
______
2 GM
(ii) We can take the liberty of calling the PE as PE of Thus, speed of M is 2v = 2 ​ __
​   ​  ​ ____
  ​ ​ 

mass M, since 2M is fixed. 3 r
Note:  Speed of M obtained in this example is less than the
speed of M obtained in last example. Now, the same loss
in PE has been shared between two particles as KE. In last
example, mass M was the only beneficiary.

Example 14   Reaching a great height


A body is projected from the surface ___ of Earth
(mass = M, radius = R) with a speed u = ÷    ​ (  8 kms–1)
​ gR 
in vertically upward direction. To what height (above the
Ki + Ui = Kf + Uf surface) will the body reach before it begins to fall down?

( 
Neglect atmospheric resistance.
( 
G 2M ◊ M
0 + ​ – ​ ________
r     ) 1
2
G 2M ◊ M
​  ​ = ​ __ ​  Mv2 + ​ – ​ ________
)
r  ​  
__
​    ​ 
2
 ​ Solution
Concepts
1 2GM
fi ​ __ ​  v2 = _____ ​  r    ​ 
2 As the projectile moves up, it slows down. Its KE decreases
____ and gravitational PE increases. At the top point, its KE is
fi ÷ 
GM
v = 2 ​ ____
​  r   
​ ​   zero.
We will use conservation of energy to solve the problem.
Particle is projected from A, it stops at B.
Example 13   In the last example, both particles are released
from separation r. Find speed of mass M when separation Let OB = x
r
reduces to __
​    ​. 
2
KA + UA = KB + UB
Solution
1 GMm GMm
Concepts ​ __ ​  mu2 – _____
​     ​ = 0 – ​ _____
  x   ​

2 R
(i) Mechanical energy is conserved. In this case,
1 GM GM
PE is strictly said to belong to the system of the fi  ​ __ ​  (gR) – ​ ____​   = – ​ ____
   ​ 
x   
2 R
particles. Any change in PE affects the KE of both
particles.
(ii) The system of particles move under mutual
1 GM ____
fi  ​ __ ​  ​ ____
2 R
GM
​ – ​     
  
R
GM
​   = – ​ ____ 
​  ​
x    [  GM
  g = ​ ____
R2
 ​  ​
]
interaction. There is no external force. Momentum GM GM
fi – ​ ____ ​  = – ​ ____ ​ 
x   
of the system is conserved. Since initial momentum 2R
r
is zero, momentum at separation __ ​    ​  is also zero. fi x = 2R
2

If velocity of 2M is v (Æ)
then the velocity of M must be
2v (¨) for momentum of the
system to remain zero.
Conservation of energy:
Ki + Ui = Kf + Uf

G 2M ◊ M __ 1 1 G 2M ◊ M
0 – ​ ________
r      ​   ​  (2M) v2 + ​ __ ​  (M) (2v)2 – ________
​ = ​  r  ​   
2 2 __
​    ​ 
2
11.16  Mechanics II

\  Height attained above the surface is h = x – R = R


If we want them to have kE when at •, we need
Example 15    Two particles, each of mass m, are at a to do more work on them.
separation r. They are moved apart from one another. On (ii) Wext = Change in mechanical energy of the
reaching infinite separation, both of them have speed v. How system.
much work was done by the external agent?
Solution \  Wext = (Kf + Uf) – (Ki + Ui)

( 
Concepts

GMm
(i) Potential energy of the system is – _____
​  r   ​

1
= ​ __
2
1
​   ​  mv2 + __
2 ) ( 
Gmm
​   ​  mv2 + 0  ​ – ​ 0 – _____
​  r   
​  ​
  )
GMm
If an external agent delivers + ​ _____ r    ​ energy, the   [   Uf = U• = 0]
total energy of the system will become zero. The
two particles can be just separated by large distance Gm2
\  Wext = mv2 + ____
​  r   

GMm


if + _____
​  r   

​ work is done on them.

Your Turn

Q.19  Three particles of masses m, 2m and 3m, are placed


on the vertices of a right-angled isosceles triangle, as shown. Q.20  Two particles of mass m each are initially at rest at
They are moved so as to bring them on the vertices of an very large separation. They are gently pushed and they begin
equilateral triangle of side length a. Find the work done by to move towards one another due to mutual gravitational
gravitational force in the process. pull. Find their speed when separation between them
becomes r.
Q.21  A body is dropped from a height R above the surface
of Earth. Find its speed on reaching the surface. R is the
radius of Earth and g is the acceleration due to gravity on
its surface.

7.5  Escape Speed


GMm
gravity is + ​ _____
R

    ( 
GMm
​. This energy ​ = ​ _____
R )

​  ​ is known as
   
If a body is projected up, it will slow down, stop and then fall binding energy of the Earth-particle system.
back to Earth. If we go on increasing the projection speed, it If the particle is given a speed v, its total energy
will go higher and higher. There is a certain minimum initial becomes
speed that will cause it to move upward forever, theoretically
1 GMm
coming to rest only at infinity. This minimum speed is called E = K + U = __
​   ​  mv2 – _____
​     ​

the escape speed from the surface of Earth. 2 R
A particle of mass m, on the surface of Earth, has gravi- For it to escape forever, E ≥ 0
tational PE equal to
1 GMm
fi  ​ __ ​  mv2 – _____
​      ​  ≥ 0
GMm 2 R
U = – _____

÷ 
​     ​ 
÷ 
_____ ______
R 2Gm 2GmR ____
fi  v ≥ ​ ​ _____
    ​ ​ = ​ ​ ______  ​ ​   = ÷   ​ 
​ 2gR 
To make it reach infinity, we must make its total energy R R 2

equal to or greater than zero. Thus, the minimum energy Minimum speed to escape the gravity is known as escape
that we need to impart to the particles to free it from the speed

÷ 
_____
2GM ____
ve = ​ _____​      =÷
​ ​     ​  
​ 2gR  ...(18)
R
Gravitation  11.17

Putting g = 9.8 ms–2 and R = 6.4 × 106 m gives Solution


ve 11.3 kms–1. Concepts
Mahabharata's Bhima could throw elephants at speed
large enough to make them escape Earth’s gravitational pull. For a particle to just escape gravity, its total energy must
The elephants never returned! be zero.
Equation (18) is applicable for any celestial object, not just K + U = 0  for escape
for Earth. Substituting mass of the moon M = 7.36 × 1022 kg
1 GMm
and radius of the moon R = 1.74 × 106 m in equation (18) fi ​ __ ​  mv2 – _____
​   ​ =
  0

gives escape speed from the surface of the moon. It is 2 3R
___
​   ​ 
close to 2.38 kms–1. 2
Escape speed on the Jupiter is close to 60 kms–1. [ 
R
​ Distance from centre = R + __
​   ​  =
2
3R
]
​ ___ ​  ​
2

÷  ÷ 
_____ ______
7.6  black holes 4GM GM
fi v = ​ _____
​      = 2 ​ ____
​ ​ 
  ​  2 ​  ◊ R ​ 
The great Indian scientist subrahmanyan Chandrasekhar 3R 3R

÷ 
___
predicted that a star having a mass more than few times gR
the mass of the sun would collapse under its own gravity, = 2 ​ ___ ​    
3

 ​ ​
after its fuel is exhausted, to become a superdense object of
vanishingly small size. Such objects are plentiful in the space. Example 17   Direction of projection
Equation (18) predicts that escape speed from such objects will not matter if v ≥ ve
will be extraordinarily high due to small radius (R). A particle is projected from the surface

÷ 
_____
2GM of a planet, making an angle q with ver-
ve = ​ _____
​      ≥ c for such objects.
​ ​   tical. Projection speed is equal to escape
R
speed. Will the particle escape? Discuss
Where c = 3 × 108 ms–1 is the speed of light. According
qualitatively.
to theory of relativity (Einstein, 1905), no material object can

÷ 
____
GM
have speed greater than that of light. Therefore, no object Solution  If projection speed is ve = ​ ____
​     
 ​ ​ 
, the total energy
can escape the gravity of such superdense stars. Even light R
cannot escape from such an object. Such objects were called of the projectile is zero. It is not
Black holes in 1969 by American scientist, John Wheeler. bound to the planet. It will escape
Chandrasekhar was awarded Nobel prize in 1983. to infinity. It takes a curved path,
as shown, and moves to infinity.
R
Example 16   A particle is located at a height h = __
​   ​  above However, depending on the angle
2 of projection, it is possible that
the surface of Earth. With what minimum speed should it be
the planet may intercept its path
projected up, so as to escape the gravity forever? R = radius
and the particle hits the planet.
of Earth.

Your Turn

Q.22  Potential energy of a rock piece on the surface of Q.23  Acceleration due to gravity on surface of planets A
a planet is – E0. Due to a blast, the rock piece acquires a and B are s and 2s respectively. Radii of A and B are R
kinetic energy of 2E0. and 2R respectively. Find the ratio of escape velocities from
(a) Will the rock piece escape out of the gravitational the surface of the two planets.
pull of the planet?
Q.24  A rocket is shot vertically upward from Earth’s sur-
(b) At some height, its KE is 1.6E0. What is its PE face. Find the distance from the centre of Earth that the
there? rocket reaches if the launcher gives it half the KE that is
(c) What will be KE of the particle when it is far away needed to escape the gravity forever? R = radius of Earth.
from the planet?
11.18  Mechanics II

8.  GRAVITATIONAL POTENTIAL 8.2  Potential due to a uniform thin spherical shell
A system of particles has fixed Consider a shell of mass m and radius R.
masses: m1, m2, m3 ... etc. In
presence of these fixed masses,
work needed to slowly bring
a mass M from infinity to P
is defined as PE of the mass
M. [Refer to equation (15). We
can also say that PE of M at
P is negative of work done by
external agent in slowly mov- What is the potential due to the shell at point P1 (at a
ing it from P to •]. distance r > R from centre)? It is the work done in slowly
• Æ P
UP = ​W​ext   ​ 
shifting a unit mass from • to P1 in the presence of the shell.
Agt
​         ​ Will it make a difference (to the work done) if the shell is
​ slowly ​
replaced by a point mass M at its centre? Answer is no. The
Gravitational potential (V) at point P due to fixed masses gravitational force due to a shell is same as that due to an
is defined as equal point mass at its centre. Thus, potential at P1 remains
UP unchanged if shell is replaced with a point mass.
VP = ​ ___ ​   ...(19)
M Gm
\ VP1 = – ​ ____​   (for r ≥ R)
r    ...(21)
–1
Its unit is J kg . It is a scalar.
Exactly on the surface (at P2)
Thus, the potential at a point may be defined as the work
Gm
done per unit mass by an external agent in slowly bringing VP2 = – ​ ____   
​   ...(22)
a particle from infinity to the given point. R
Now, what is the potential at P3 (a point inside the shell)?
8.1  Potential due to a point mass By definition
m is a fixed point mass. We already know that PE of another • Æ P3
VP3 = ​W​ext  ​ 
(for a unit mass)
Agt
mass M placed at a point P (at ​         ​
​ slowly ​
a distance r from m) is
• Æ P2 P2 Æ P3
= ​W​ext Agt
 ​ 
+ ​W​ext ​ 
Agt
  (for unit mass)
GmM
U = – _____
​         ​ ​         ​
​  r   
​  ​ slowly ​ ​ slowly ​

U
\  Potential at P due to m is V = __
P2 Æ P3
​    ​ = VP2 + ​W​ext Agt
​ 
M ​         ​
​ slowly ​
Gm
fi V = – ____
​  r   
 ​ ...(20) But no work is needed to move a mass from P2 to P3,
This is the work done by an external agent in slowly as there is no field inside the shell [Refer to article 5.1]. A
moving a unit mass from • to P. mass inside the shell will experience no force due to the
shell.
Note: (i) Potential obeys principle of superposition.
Potential at P in the shown figure is: P2 Æ P3
\ ​W​ext ​ 
​ = 0
Agt
​         ​
Gm1 Gm2 Gm3 ​ slowly ​
V = – ____
​  r  ​ 
 – ​ ____
r2 ​ 
 – ​ ____
r3 ​ 
  Gm
1 \ VP3 = VP2 = – ​ ____   
 ​ ...(23)
R
Potential at all inside points is same as surface
potential.

8.3  Potential due to a solid uniform sphere


Arguing exactly as above, we can easily say that
(ii) If we place a mass M at P, then its PE is given
by
U = MV
Gravitation  11.19 


Gm
V = – ​ ____
    ​  (for r ≥ R) ...(24)
3Gm ___
= – ​ _____
R3
 ​

  ​ ​  [ 
R2 __
2
 ​
  –
r2
​ 
2 ]
 ​   ​

Gm
And Vsurface = – ​ ____   
 ​ ...(25) \ Potential at P is
R
V = V1 + V2
For getting potential at an inside
point, we need to find the work done
Gmr3 _____ 3Gm
in bringing a unit mass from • to V = – ​ _____
3
 – ​  3 ​ (R2 – r2)
 ​ 
that inside point. This can be calcu- R 2R

( 
lated with the help of equation (6).
However, we will take a different
Gm __
V = – ​ ____
R3 2
3 1
 ​ ​ ​   ​  R2 – __ )
​    ​ r2  ​
2
...(26)
approach. We will use principle of
superposition for this calculation. Potential at the centre is obtained by putting r = 0 in the
Let us assume P is a point at a above equation.
distance r from the centre where we wish to find potential. 3 Gm
Vcentre = – ​ __ ​  ____
​     
​   ...(27)
Let us divide the sphere into two parts 2 R
(i) a solid sphere of radius r (shown as shaded part).
Example 18   Four particles, each of mass m, are kept at the
(ii) annular region between r £ x £ R.
vertices of a square of side length a. Find the gravitational
m 4 mr3
Mass of the first part is: m¢ = ______
​       ​ ◊ ​ __  ​ p r3 = ____
​  3 ​  field and potential at the centre of the square.
4
__ 3 R
​    ​ p R3 Solution
3
Potential at P due to solid sphere of mass m¢ is given by Concepts
equation (25) as
(i) Both field and potential obey principle of
Gm¢ Gm 2
V1 = – ​ ____ ​ = – ​ ____
r     ​  r superposition.
R3
(ii) We must be careful that field is a vector, whereas
Now, we will find potential (V2) at P due to the mass potential is a scalar.
in the region r £ x £ R. For this, we divide this mass into
concentric shells. Consider one such shell between radius x Field at O due to masses at A
and x + dx. and C cancel out. Similarly, field
due to masses at B and D also
cancel out.
Resultant field at O is zero.
a
AO = r = ___
​  __  ​ 
÷   
​ 2 ​
Gm
Potential due to each mass, at O is = – ​ ____
r    ​
__
÷    Gm
​ 2 ​ 
= – ​ ______
a    

Mass of the shell is \  Total potential at O is
__
m 3m x2 dx
dm = _____
​     ​   ◊ 4p x2 dx = _______
​   ​      Gm
4​÷2 ​ 
4
__ R3 V = – ​ _______
a    ​ 
​   ​  p R3
3
Note:  If we place a mass M at O, then its PE in pres-
Potential at P due to this shell is same as potential due
ence of the four fixed masses will be
to the shell at its own surface (equation (22)). __
   GmM
4​÷2 ​ 
Gdm 3Gm U = MV = – _________
​  a   ​ 
\  dV = – ​ _____​ = – ​ _____
x     ​ ◊ x ◊ dx


R3
Potential due to all the shells lying between x = r to Example 19   A tunnel to the centre
x = R is of the Earth

[  ]
R
3Gm 3Gm __ x2 R A tunnel is dug all along the radius
V2 = – ​ _____   ​ ​xdx = – ​ 
 ​
  Ú
​ _____  ​

  ​​ ​   ​   ​​​ ​
2 r of the Earth – from its surface to its
R3 r R3
centre. A ball is dropped in the tun-
nel. With what speed will it reach the
centre?
11.20  Mechanics II

GMm
Neglect any friction and assume that Earth is a uniform PE of the ball at the surface: Usurf = – _____
​     ​

R
solid sphere of mass M and radius R.
Potential at the centre of Earth:
Solution
3 GM
Concepts Vcent = – __
​   ​  ____
​     
​ 
2 R
(i) Assuming Earth to be a uniform solid sphere 3 GMm
PE of the ball at the centre: Ucent = – __
​   ​  _____
​     ​ 
allows us to use equations (26) or (27) for writing 2 R
potential at a point inside it. Energy conservation:
(ii) Potential energy of the ball (mass = m) at a point
KEcent + Ucent = KEsurf + Usurf
where potential is V is given by U = mV.
(iii) Mechanical energy is conserved. 1 3 GMm GMm
​ __ ​  mu2 – __
​    ​ _____  0 – _____
​     
​ = ​     ​

2 2 R R
GM
Potential at the surface of the Earth: Vsurf = – ____
÷ 
​     
​  ____
R GM GM
fi  u2 = ​ ____​   fi  u = ​ ____
   ​     
  
R R

​ ​
Your Turn

Q.25  Gravitational potential due to Earth at a point is Q.26  Is it right to say that PE of a particle at the centre
– 1.0 × 107 J kg–1 of Earth is more than the PE of a similar particle on its
(a) Is the point inside or outside the Earth surface?
(b) What is the PE of a 5 kg mass placed at this point?
Q.27  Find the gravitational potential at the centre of a ring
of mass M and radius R.

In Short: (m2, m3). The PE of these pairs does not change as


they are fixed. And, in solving a problem, the change
(i) Gravitational PE of a system of two point-masses is
in PE is important, not its absolute value. Thus, we
assumed to be zero when they are at • separation.
need not consider PE of interactions of m1, m2 and
(ii) Gravitational PE of a system of two point-masses, m3. What changes when M moves? It is energy of
separated by distance r, can be defined as: pairs (m1, M), (m2, M) and (m3 M).
– (Work done by gravitational force when the masses
are moved from • to separation r) =
work done by gravitational force when masses are
moved from r to • =
work done by external agent when masses are
moved slowly from • to r =
– (Work done by external agent when masses are
moved slowly from r to •)
Gm1m2
(iii) PE for a two-particle system is U = – ______
​  r   ​ 
(iv) PE of a multi-particle system is obtained by adding In such cases, concept of potential becomes
PE of each pair. important.
Potential at is PE of unit mass kept at P. Here, we
Gm1m3 ______
Gm1m2 ______ Gm2m3
U = – ______
​  r  ​ 
 – ​  r  ​ 
 – ​  r  ​ 
 ... are taking into account PE of pairs (m1, M), (m2, M)
12 13 23 and (m3, M) with M as unity.
(v) Let m1, m2, m3 be fixed masses and M is free to move Gm1 Gm2 Gm3
under influence of gravitational force due to fixed \  VP = – ​ ____
r1 ​ 
 – ​ ____
r2 ​ 
 – ​ ____
r3 ​ 

masses. In such cases, it is not necessary to worry
about energy of pair like (m1, m2), (m1, m3) or PE of a mass M at P is U = MVP
Gravitation  11.21

÷ 
_____
(vi) Potential due to a uniform spherical shell of mass M 2GM ____
v = ​ _____
​      =÷
​ ​ 
    ​ 
​ 2gR 
and radius R is R
GM Where g is acceleration due to gravity on the
V = – ____  ​  for r ≥ R
​  r    surface.
GM (xi) Earth has atmosphere. This is because the average
= – ____  ​  for r £ R
​     
R speed of gas molecules in the atmosphere of Earth
is much smaller than escape speed (11.3 kms–1).
Note:  The concept of field and potential may be
a bit difficult to understand at this stage. We will
come back to it in greater detail in the chapter of
electrostatics.

9.  SATELLITE
Moon is the only natural satellite of the Earth. However,
there are thousands of artificial satellites going around the
Earth. They are used for telecommunication, weather fore-
casting, spying, GPS (global positioning system) and many
(vii) Potential due to a uniform solid sphere of mass M
other applications.
and R is
GM To launch a satellite, a rocket is used to lift it to a desired
V = – ​ ____ ​   for r ≥ R
r    height and then another rocket is fired to impart it a veloc-

( 
ity parallel to the Earth’s surface. Depending on the speed
GM 3 2 __
= – ____
​  3 ​ ​ __
R
1
)
​    ​ R – ​   ​  r2  ​  for r £ R
2 2 given, a satellite goes around the Earth in a circular orbit or
an elliptical orbit.
Here, we will assume a satellite in circular orbit and
derive the equations for its orbital speed, period of revolution
and energy.

Orbital Speed
A satellite of mass m is orbiting around Earth, in a circular
orbit of radius r. Its orbital speed is v0. There is no engine
or rocket attached to the satellite. Once it is imparted a speed
v0, the gravitational pull of Earth provides it the necessary
centripetal force and it keeps revolving.
mv02 _____
GMm
​ ____ r   
  ​  2 ​ 
​ =  
r
____
(viii) Gravitational PE is always negative. It shows binding.
Two masses are said to be separated if they are far
fi ÷ 
GM
v0 = ​ ____
​  r   
​ ​    ...(28)

away from one another and their PE is zero. It means,


we need to increase the energy (i.e., do positive work)
on a system of two masses to separate them.
(ix) Amount of energy needed to separate a particle from
another and move them to infinity, is known as bind-
ing energy. Binding energy of a particle of mass m on
GmM
the surface of Earth (mass M, radius R) is ​ _____ ​. 
   
R
(x) Minimum speed of projection that takes a projectile
to infinity is known as escape speed. Escape speed
on the surface of a spherical celestial body is given If h is height of the satellite above the surface of Earth,
by r = R + h.
11.22  Mechanics II

÷ 
_____
\ ​ 
GM
v0 = ​ _____  
  ​ ​ ...(29)
9.1  Ellipitical Orbit
R+h
If the speed given to the satellite is different from v0 (but not
To establish a satellite in a circular orbit above the surface high enough to make it escape), the path is an ellipse.
of Earth, it must be imparted a speed given by the above Ellipse is a path traced by a
equation. point, which moves such that sum
Orbital speed (v0) is independent of the mass of the of its distance from two fixed
satellite. It is only the radius of the circular orbit that matters. points (F1 and F2) remains con-
Larger the radius of circular path, smaller is the speed. stant. r1 + r2 = a constant.
The fixed points, F1 and F2, are
Time period of revolution
known as foci (plural of focus).
Time needed to complete one revolution is the time needed Distance of each vertex (A and B) from the geometrical
by the satellite to travel a distance 2p r with speed v0. centre (O) of the ellipse is

÷ 
____
called its semi-major axis
2p r _____ 2p r r3
\  T = ____
​  v  ​ 
 = ​  ____  ​  = 2p ​ ____
​     
​ ​   ...(30) (a).
÷ 
0 GM
____ GM
​ ​  r   
​ ​   If distance of a focus
(F1 or F2) from O is c,
r3
or, T2 = 4p2 ____
​    ​   ...(31) c
then ​ __
a  ​ = e is known as
eccentricity of the ellipse.
Square of time period is proportional to cube of the radius
Value of e ranges from 0 to 1; when e = 0, the ellipse is a
of the circular path. Farther the satellite, higher is its time
circle and when e = 1, it is a straight line.
period.
Suppose a satellite is lifted above the surface of Earth to
Energy of a satellite point A and is given a velocity v parallel to the surface of
Earth. If the speed is exactly equal to v0 given by equation
Energy of a satellite is the sum of its kinetic and potential (28), the satellite will revolve in a circular path (shown as
energies. path 2 in figure). If v > v0, it will go in an elliptical path
GMm
U = – ​ _____r    
​ ...(32) with the centre of Earth (O) at a focus. If v is too large,
the satellite will move in an open path (like a parabola) and
1 1 GMm
and K = ​ __ ​  mv02 = __
​    ​ _____
​     ​   ...(33) move to infinity.
2 2 r
Total energy of the satellite is:
1 GMm
E = U + K = – ​ __  ​ ​ _____ 
   ​ ...(34)
2 r
Total energy is negative. It should be. The satellite is
bound to Earth.
Total energy is equal to half the potential energy and KE
is same as magnitude of total energy.
The above analysis is valid for circular orbit of any
satellite of any planet. It makes no difference whether the
satellite is natural or artificial.

If v < v0, once again, the satellite follows an elliptical path


(shown as path 3). If the speed is too small, the eccentric-
ity of the path increases and the path of the satellite will
intersect that of Earth. It will fall on Earth.
It can be proved that for elliptical orbit of a satellite, time
period and energy is still given by equations (31) and (34)
respectively, with r replaced by semi-major axis (a) of the
ellipse.
Gravitation  11.23 

Example 21   Distance of moon form the centre of the earth


is about rM = 4 × 105 km and its time period of revolution is
TM = 27 days. Find the time period of revolution of an earth’s
satellite that is in a circular orbit of radius r = 8,000 km.
Solution
Concepts
Equation (31), T 2 µ r3 is valid for moon also.

r3
T2 = 4p 2 ​ ____   

GM
a3 M is the mass of earth.
T2 = 4p2 ​ ____  ​  ...(35)
GM
4p2
Thus, ____
​      
​ is same for an artificial satellite as well as
1 GMm GM
and E = – ​ __ ​  _____
​     
 ​ ...(36) moon.
2 a
The last equation tells us that the total energy of a sat- \  T 2 µ r3
ellite depends only on the semi-major axis (a) and not on
eccentricity (e).
T2 ___
​ ___2  ​ =
TM rM
r3
( 
8 × 103 3
  ​  3  ​   fi  T 2 = (27d)2 ​​ ​ _______5 
4 × 10 )
 ​  ​​ ​

Example 20   The fastest satellite __


  
2​÷2 ​
(a) Find the maximum possible speed of a satellite of \  T = 27 × _____
​     ​ day = 1.83 h
1000
the Earth.
(b) What is the smallest time in which a satellite can Example 22   A satellite of earth has an orbital speed v0, in
complete a revolution around the Earth? a circular orbit. A rocket, on board, is fired to increase its
speed to v1 in a quick time. Find the minimum value of v1
Solution
for which the satellite will escape to infinity.
Concepts
Solution
(i) Speed is maximum for smallest radius of circular
Concepts
path. Smallest radius possible for an Earth’s satel-
lite is R (= radius of the Earth). Such satellites may The satellite will just escape if its total energy is made
be called as near-surface satellites. zero.
(ii) Time period is also smallest for a near-surface ____
satellite, as T 2 µ r 3. ÷ 
GM
v0 = ​ ____
​  r   
​ ​  
____

÷ 
GM For the satellite to just escape, its total energy must be
(a) v0 = ​ ____
​  r   
​ ​   made zero.

÷ 
K + U = 0

÷ 
____ ______
GM GM ___
vmax = ​ ____ ​ ​  = ​ ____   ​  8 kms–1 _____

÷ 
​      ​  2 ​  ◊ R ​ 
= ​÷gR  1 GMm 2GM __
R R ​ __ ​  mv12 – _____   0  fi  v1 = ​ _____
​  r   
​ = ​  r    =÷
​ ​       v0
​ 2 ​
2

÷ 
____
r3 It implies that KE of the satellite must be doubled.
(b) T = 2p ​ ____
​     
​ ​ 
GM
T is minimum if r = R Example 23   Weightlessness in a satellite

÷  ÷ 
____ ______ Inside an earth’s satel-

÷ 
__
R3 R2 R
T = 2p ​ ____ ​ ​ = 2p ​ ____
​      ​ = 2p ​ __
◊ R  
​    ​   ​ g ​ ​   lite, there is a ball sus-
GM GM pended with the help of
a spring balance. What is
84. 6 minutes
the reading of the spring
It means that no satellite of earth can go around the balance if mass of the
earth in a time less than 84.6 minutes. ball is m?
11.24  Mechanics II

Solution Now, if you fire a projectile using


____ some powerful launcher

÷ 
GM
Concepts so as to impart it a speed v = ​ ____
​     
​ ​ , the radius of curvature
R
Orbital speed of a satellite is independent of its mass. of the path becomes equal to radius (R) of the earth. The
projectile moves along path 4. When it reaches a point B,
If the satellite
____is in a circular orbit of radius r, its orbital it is still having same speed and follows the path of same
÷ 
GM
speed is v0 = ​ ____
​  r   
​ ​ .  curvature. The projectile will move in a circle. It is a satellite.
It is continuously falling towards the Earth, just like the ball
The ball, inside the satellite, is also having speed v0. (1), bullet (2) or shell (3), but it fails to hit Earth, as the
Imagine for a moment, that all the walls of the satellite Earth curves away from its path. Radius of Earth is smaller
(and the spring balance) are removed without disturbing the than (or nearly equal to) the radius of curvature of the path
ball. What will happen to the ball? It will keep revolving like of the satellite.
a satellite on its own! It has just the right speed for being
Every satellite (including the moon) is falling towards
a satellite. The gravitational pull of Earth on the ball is just
Earth (under gravity) but fails to hit Earth for the reason
equal to the necessary centripetal force, at speed v0.
explained above.
It means that the ball, inside the satellite, is moving in
a circle without any force from the spring. Tension in the Example 25   In the figure shown, O is the centre of the
spring is zero. The spring balance records zero weight. earth. Three identical satellites are going in paths shown in
Note:  An astronaut inside a satellite feels weightless. the figure. Path 3 is circular and
the other two paths are ellipti-
Example 24   The moon is continuously falling towards cal. The distance between verti-
Earth. Explain. ces of the elliptical paths 1 and
Solution  Imagine yourself at Mt. Everest (A). You throw a 2 is equal to the diameter of the
ball horizontally and it lands on ground, at 1. Now you fire circular path 3. Eccentricity of
a bullet and it lands at point 2. If you fire from a high-speed path 1 is 0.9 and that of path 2
cannon, the shell may reach as far as point 3. Note that with is 0.5.
increasing speed of projection, the radius of curvature of the Find the ratio of time periods and total energies of the
path (near A) is increasing. three satellites.
Solution
Concepts
Equations (35) and (36) tell you that the time period and
energy depend only on semi-major axis and are indepen-
dent of eccentricity.
Distance between vertices of an ellipse = 2a
\  All three paths have same semi-major axis.
\  T1 : T2 : T3 = 1 : 1 : 1
And  E1 : E2 : E3 = 1 : 1 : 1

Your Turn

Q.28  In the figure shown, Q.29  If orbital radius of a satellite is increased, what
lines 1–1, 2 – 2 and 3–3 are happens to its (a) potential energy (b) kinetic energy?
diameters of circular paths
around the earth. O is the centre
Q.30  A satellite at a height of 6R from the surface of
the earth has a time period of 24 h. Find the time period of
of the earth. Which circular path
another satellite at a distance of 3.5R from the centre of the
cannot be a path for a satellite
earth. R is the radius of the earth.
and why?
Gravitation  11.25

Q.31  An artificial satellite of Earth has an orbital speed How much energy was given to the satellite in the process?
equal to half the escape speed from the surface of Earth [R Given, g = 10 ms–2 and R = 6,400 km.
= radius of Earth] Q.33  A satellite is raised to a height R above the surface

÷ 
______
(a) Determine the height of the satellite above the surface 2 GM
of Earth. of Earth and given a velocity v = ​ __​   ​  ____
​     
​ ​  parallel to the

3 R
surface of Earth.
(b) An alien demon stops the satellite suddenly and
releases it, allowing it to fall freely. Find the speed (a) Will the satellite escape the gravity of Earth?
with which it hits the Earth. (b) What kind of path will the satellite follow? R is the
Q.32  A satellite of mass m = 2000 kg is lifted from the radius of Earth.
surface of Earth and placed in a circular orbit of radius 2R.

9.2  Geostationary Satellites h = r – R  36,000 km


Satellites are an important component of modern-day Syncom-2 was the first geosynchronous communication
communication. A TV station relays its signal to a satellite, satellite launched by NASA, in 1963. The first Indian geo-
which in turn broadcasts (spreads) it over a large area on the synchronous satellite was APPLE, launched in 1981. Since
surface of Earth. T is a TV station, which sends its signal to then, India has launched many INSAT, IRNSS and GSAT
satellite S. The satellite receives the signal, amplifies it and series of geosynchronous satellites.
re-emits it. Signal coming from satellite S can be received
on surface area ATB [SA and SB are tangents].

Higher the satellite, larger is the area that it can cover.


A more important consideration is the fact that the earth is 10. PLANETS AND KEPLER’S LAWS
rotating from west to east and its time period is 24  hours. If Everything discussed in article 9 is applicable for planetary
the satellite has a different time period, telecasting a cricket motion also. In all our discussions, M will represent mass
match is going to be very difficult. At a time the satellite will of the sun and m will represent the mass of a planet. The
be in a position so as to throw signal over the entire Indian gravitational pull of the sun provides the necessary centripetal
sub-continent, and at a different time, it may be in position force to the planets.
to provide signal to south America but not to India. Tycho Brahe, a great astronomer, made years of
To solve this problem, we need a satellite, which remains observations of the planetary motion (without using a
at rest, relative to a person on Earth. Such a satellite is telescope) and compiled extensive data. Kepler made three
possible only in equatorial plane (think why?) and should deductions, looking carefully at this data for years. These
have a time period of T = 24 hours. It must revolve from three empirical laws are now known as kepler’s laws. All the
west to east. Such a satellite is called a geostationary or three laws can be derived using Newton’s law. The three laws
geosynchronous satellite. hold equally well for satellites or any other body orbiting
4p2 3 ___ 4p2 ___ r3 around the sun.
T 2 = ____
​     
​ r = ​  g   
​ ​  2  ​ 
GM R 1.  The law of orbits: All planets move in elliptical orbits
Putting T = 24 h, R = 6.4 × 10  m and g = 9.8 ms–2, gives
6 with the sun at one focus.
r  42,000 km. The eccentricities of the planetary orbits are not large and
Thus, height of a geostationary satellite above Earth’s the orbits are nearly circular. The eccentricity of Earth’s orbit
surface is around the sun in only e = 0.0167.
11.26  Mechanics II

Consider a planet at point A in its elliptical orbit around


the sun (S). Velocity of the planet is v (Tangential to the
path) and its distance from the sun is r. In a small interval
of time, it moves to B, rotating by an angle D q.
Let velocity component perpendicular to r (= SA) be v^.
Angular velocity of the planet about S is
D q v^
w = ___
​   ​ = __
​  r ​   ...(i)
D t

Point A, where the planet is farthest from the sun, is Area swept in time D t is
known as Aphelion and point P, where the planet is closest 1
D A = area (SAB) area (SAC) = __
​   ​  (r) (AC)
to the sun in called perihelion. For Earth, we use special 2
terms apogee and perigee respectively.
1
= __
​   ​  r (r D q)
2.  The law of areas: The radius vector, drawn from the 2
centre of the sun to the planet, sweeps out equal area (in
1
the plane of the orbit) in equal time intervals. = ​ __ ​  r (v^ D t)
2
dA
___
​   ​ =
  a constant [The approximation, area (SAB) area (SAC) will
dt
Where A = area swept by radius vector of the planet. give very accurate result, as D t Æ 0 and D q is extremely
small.]
A planet moves from A1 to B1 in
a time interval D t and its position D A 1
\ Areal velocity, ___
​   ​ = __
​   ​  rv^ ...(ii)
vector sweeps area SA1B1 = D A1. The D t 2
same planet moves from A2 to B2 in Let us write the angular momentum of the planet
the same interval of time D t and its (mass = m) about the sun (S).
position vector sweeps area SA2B2 =
D A2. L = (mv^) r
D A1 = D A2
L
fi r v^ = __
​ m  ​ ...(iii)
Clearly, arc length A2B2 > arc length A1B1.
This implies that planet is moving at a higher speed in From (ii) and (iii),
the region A2B2 compared to its speed in region A1B1. This
implies that the planet is slowest when it is farthest from the D A ___ L
sun (at aphelion) and it will be fastest when it is closest to ​ ___ ​ =
  ​     
​ ...(37)
D t 2m
the sun (at perihelion).
Kepler’s second law is a direct outcome of law of Angular momentum of the planet about the sun must be
conservation of angular momentum. Here is how we can a constant, as the only force acting on it (the gravitational
prove this. pull of the sun) has no torque about S. Hence, right side of
equation (37) is a constant, implying that the areal velocity
is a constant.
3.  The law of periods: The square of the time-period of
any planet is proportional to the cube of the semi-major axis
of its elliptical orbit.
This law is nothing but equation (35).

Example 26   A planet going around a star in an elliptical


1
orbit has a semi-major axis equal to a and eccentricity e = ​ __ ​ .
2
Speed of the planet when it is nearest to the star is v1. Find
its speed when it is farthest from the star.
Gravitation  11.27 

Solution 4p 2 3
(a) T2 = ____
​    ​ a
GM
Concepts

[  ]
1
2 __
(i) At farthest and nearest points (i.e., at vertices of ​   ​ 
fi  a = ​​ ______  ​​3 ​
​  2 ​  
the ellipse), velocity is perpendicular to the posi- 4p

[ 
tion vector.
(ii) Angular momentum of the planet about the star is
conserved.
= ​​ ​ _______________________________
    
4 × (3.14)2
   
 ​  ​​ ​
]
6.67 × 10–11 × 2 × 1030 × (2.4 × 109)2 1/3

= [19.4 × 1036]1/3 = 2.7 × 1012 m


Now, r1 + r2 = 2a
fi r2 = 2a – r1 = 2.7 × 1012 × 2 – 8.9 × 1010
5.3 × 1012 m
r1
(b) r1 = (1 – e) a  fi  e = 1 – __
​ a ​ 
At perihelion (nearest point), distance of the planet from
the star is 8.9 × 1010
r1 = a – ea = (1 – e) a fi  e = 1 – ​ _________   ​ 0.97
2.7 × 1012
At aphelion (farthest point), distance of the planet from
Eccentricity is close to 1.0. The orbits is a long, thin
the star is
ellipse.
r2 = a + ea = (1 + e) a
Angular momentum at A = angular momentum at P Example 28   From a circular to an elliptical orbit
mv2r2 = mv1r1 A satellite of mass m is going around Earth in a circular orbit
of radius r. It has time period T1. A small rocket is fired
from the satellite, which decreases its speed by 4%, almost
fi v2 (1 + e) a = v1 (1 – e) a
instantaneously. Direction of motion remains unchanged.
fi ( 1– e
v2 = ​ ​ _____ 
1+e 1
  )
 ​  ​ v
Now the satellite moves in an elliptical orbit.
(a) Find the semi-major axis in terms of r.
(b) Find time period in terms of T1.
Example 27   Comet Halley
Solution
Since its first predicted return in 1759, Halley’s comet has
come back to us three times, in 1835, 1910 and most recently Concepts
in 1986. It is orbiting around the Sun with a time period
(i) In circular orbits, energy of the satellite is
of 76 years. In 1986, it was nearest to the Sun (at its peri-
GMm
helion) at a distance of r1 = 8.9 × 1010 m. Mass of the E = – ​ _____  
​.
   
2r
Sun = 2 × 1030 kg. GMm
In an elliptical orbit, energy is E = – ​ _____   ​

(a) Find the comet’s farthest distance from the Sun. 2a
(b) What is eccentricity of the orbit? (ii) The square of time period is proportional to cube
[76 years = 2.4 × 109 s, (19.4)1/3 2.7] of semi-major axis. here, if T1 is period in circular
orbit and T2 is period in elliptical orbit, then
Solution
T22 __ a3
Concepts ​ ___2 ​ =
  ​  3 ​ 
T1 r
(i) If r1 and r2 are the nearest and farthest distances (a) Rocket is fired at A to decrease the speed by 4%.
of the comet from the sun speed before firing of rocket
r1 + r2 = 2a ____

(ii) From the last example:


GM
v0 = ​ ____
​  r    ÷ 
​ ​  
r1 = (1 – e) a and r2 = (1 + e) a Energy of satellite at A is
GMm
E1 = – ​ _____     ​ 
2r
11.28  Mechanics II

PE just after rocket is fired is (the satellite is still


at A)
GMm
U2 = – ​ _____​ 
r   
\  Total energy after the rocket is fired is
– 0.54 GMm
E2 = K2 + U2 = __________
​  r      

If the semi-major axis of elliptical orbit is a then


GMm
E2 = – ​ _____
   

Just after the rocket is fired, speed is, v = 0.96 v0 2a
KE after rocket is fired is GMm GMm
fi  – 0.54 ​ _____ ​ = – ​ _____
r      ​   fi  a = 0.93 r

1
K2 = __ ( 
​   ​  m (0.96 v0)2 =
2
1
​ __
2
)
​   ​  mv 0  ​ ×
2
0.92
T22 a3
(b) ​ ___2  ​ = __
2a

​  3 ​   fi  T2 = T1 (0.93)3/2 = 0.89 T1

( 
T1 r

GMm
= 0.92 ​ _____
​     
2r

​  ​ )
Your Turn

Q.34   ‘‘Angular momentum of a planet about the Q.36  Two planets (P1 and P2) have equal mass. They
sun remains conserved but its KE does change.’’ Is this revolve around a common
statement true? Explain your answer. massive star S. P1 goes in a
Q.35  A planet is going around the sun. Its speed at circular orbit of radius r1 =
aphelion (A) is v1. Its distance from the sun in this position 108 km and has a time period
is r1. At some other point B, it is at a distance r2 from the of 2 years. Planet P2 moves in
sun and its velocity vector makes an angle q with the line an elliptical path with nearest
BS, as shown. Find speed (v2) at B. and farthest distances from
the star equal to r1 = 1 × 108
km and r2 = 1.8 × 108 km
(see figure).
(a) Find time period of P2.
(b) Which planet has greater speed at P?
(c) which planet has greater energy at P?

11. Mars orbiter Mission


(Mangalyaan)
Mass orbiter Mission (MOM) is a space probe orbiting
planet Mars since 24 september 2014. This was launched by
ISRO (Indian space research Organisation) on 5 November
2013. MOM was put into an orbit around Earth. over a span
of nearly one month, its orbit was raised six times by firing
rockets. The rockets were fired when MOM was nearest to
Earth in its path (at Perihelion). This was done to impart
maximum increase in KE of the spacecraft while using as
little fuel as possible.
Gravitation  11.29

Let the speed of spacecraft be v when a rocket is fired Finally, on 01 December 2013, a rocket was fired to
to impart a thrust in the direction of its motion. By burning impart enough energy for the probe to leave the gravity of
a certain quantity of fuel, the rocket can cause the velocity earth. MOM took a hyperbolic trajectory under influence of
to increase by some fixed amount D v. the sun’s gravity. The flight path of MOM intersected the
1 orbit of Mars at the moment when Mars was too close. At
k = __
​   ​  mv2
2 this moment, its speed was very large and it would have
1
__ escaped Mars gravity too. A rocket was fired to slow it down
D k = ​   ​  m (2v) D v = mv D v
2 to an adequate speed. It got captured into an orbit around
Thus, for a given D v, change in KE is maximum when Mars. Since then, it has sent many revealing photographs of
v is maximum and speed of the spacecraft it maximum at martian surface.
perihelion.

Your Turn
Q.37  Do you think that timing the escape of MOM from
the gravity of Earth was important? Why?

G m1m2
12. Binary stars m1r1w2 = m2r2w2 = _______
​  2 ​     ...(iv)
r
There are many pairs of stars, which revolve about their
Gm2 _______ Gm2 Gm
common centre of mass, under mutual gravitational attraction. fi  w2 = ​ ____
2
 ​
  = ​  3
    ​ = ​ ____
3
 ​ where M = m1 + m2
Many a times, observation of a star reveals that it is revolving r1r m
_______
2 r r
​     ​
but the companion star is invisible. Well, the companion m1 + m2

÷ 
____
is a black hole! Fact of the matter is that both – the earth GM
and the moon – are revolving about their common COM. fi  w = ​ ____ ​  3 ​ ​ 
  
The COM of the Earth – Moon system is located inside the r 3/2
2p 2p r
earth due to its large Time period of revolution, T = ___ ​   ​ = ______
​  ____ ​   ...(38)
w ÷    
​ GM  ​
mass. The centre of
the earth revolves on Kinetic energy of the system of binary stars:
1 1
a circle of relatively k = __
​   ​  m1(r1w)2 + __ ​    ​ m2 (r2w)2
small radius. 2 2
Consider a binary
star system comprising
= __
1 m
( 
rw
​   ​  m1 ​​ ​ _______
2
2
m1 + m2
   
2
) 1
( m1rw 2
 ​  ​​ ​ + ​ __ ​  m2 ​​ ​ _______
2 m1 + m2
    )
 ​  ​​ ​
two stars, of masses
m1 and m2, revolving
around their common
1 m1m2
= __
​   ​  ​ _______
2 m1 + m2
​     (  1
 ​  ​ r2w2 = __
2 )
​   ​  m r2w2 ...(39)

COM. Both have the m1m2


same time period and where, m = _______
​    
 ​ = reduced mass of the system
m1 + m2
are always located on
a diameter. Angular momentum of the system about COM:
r = separation between stars
L = (I1 + I2) w = (m1r12 + m2r22) w
r1 and r2 are distances of the two stars from their
COM. Substituting for r1 and r2 from (iii) and simplifying
m1r1 = m2r2 ...(i) gives
and r1 + r2 = r ...(ii) L = m r2 ◊ w ...(40)
Solving the above two equations, Effectively, we can replace this two-body problem with a
m2r m1r single-body problem in which a body of mass m is revolving
r1 = _______
​     ​ and r2 = _______
​    
 ​ ...(iii) in a circular orbit of radius r. The body experiences a
m1 + m2 m1 + m2
Gm1m2
Equation for centripetal force: force ______
​  2 ​  toward the centre.
r
11.30  Mechanics II

Miscellaneous Examples
Example 29   A thin uniform rod has mass M and length Force on m due to dM is
2L. A particle of mass M is kept at a distance a from the
rod on its perpendicular bisector. Find gravitational force on

GmdM
______
dF = ​  2 ​ 
Ma
Gm ​ ___
________________
 = ​ 
( 
​ sec2 q dq  ​
​     
L
  
 ​
  
)
the particle due to the rod. r (a sec q)2
Solution GmM
fi  dF = _____
​     ​ dq.

Concepts aL
(i) We will need to consider an infinitesimally small Component of this force along the line PO is
mass on the rod and write force on m due to that GmM
(ii) Adding forces due to all small masses making the dFy = dF cos q = _____
​     ​ cos q ◊ dq

aL
rod will give the answer.
By considering an identical element to the left of O, one
(iii) Considering identical elements on two sides of cen- can easily show that the resultant force on m will be along
tre (O) of the rod helps us to see that the resultant PO.
force is along the perpendicular bisector.
Resultant force is obtained by adding dFy due to all
(iv) Below, we have expressed our integration in terms elements.
of q. this keeps the integration simple, though one
q
can always express the entire thing in terms of x. GmM 0 cos q dq
\ Fy = Ú dFy = _____
​     ​ Ú
  0
aL –q0
GmM q
= _____
​     ​ [ 
  ​​ sin q ]​​ 0  ​ 
aL – q0

2GmM 2GmM ________ L


= ______​  ​ sin q0 = ______

    ​  ​ ​  ______

        ​
aL aL ÷ ​ a  + L2 
2
 ​
2GmM
\  Fy = _________
​  ______    ​ 
  2 + a2 ​ 
a ​÷L

Example 30   Potential energy of a particle of mass m at


M infinite distance from the earth (mass M) is assumed to be
Mass per unit length of the rod is l = __
​   ​ 
L GMm
zero. Its PE on the surface of earth is – ​ _____ ​ where R is
   
Consider an element of angular width dq as shown. R
radius of the earth. Find the change in PE of the particle
Length (dx) of the element can be expressed in terms of
q as follows. when it is raised to a height h (<< R) above the surface of
[Note that it is wrong to write dx = rdq. Why?] the earth.
Solution
dx
x = a tanq  fi ​ ___  ​ = a sec2 q
dq Concepts

fi dx = a sec2 q ◊ dq ( 
GMm
PE at a height h is ​ – _____
​ 
R+h
  ​  ​ )
Ma
Mass of element, dM = l dx = ___  ​ sec2 q dq
​     
GMm
U0 = – _____
​     
GMm
​  ; Uh = – ​ ______ 
  ​
L R (R + h)


1
D U = Uh – U0 = – GMm ​ _____
​     
R+h [ 
 ​ –
1
__
]
​   ​   ​
R

GMmh GMmh
= ________
​   
  ​ = _________
​      ​
R (R + h) 2
( 
R
h
R  ​ 1 + ​ __ ​   ​ )
gmh
_____
= ​      ​
h
1 + __​   ​ 
R
Gravitation  11.31 

h
when h << R, __
​    ​ << 1 KA + UA = KP + UP
R
\ D U mgh.
The result is as expected.
1
2 [  G 16Mm _____
​ __ ​  mu2 + ​ – ​ _______
2a
    
GMm
​ – ​     
8a ]
​  ​

Example 31   Distance between the centres of two stars is


10a. The masses of these stars are M and 16 M and their radii
[ 
G 16Mm ______
= 0 + ​ – ​ _______
8a
    
GM ◊ m
​ – ​ 
2a
​  ​
    ]
Simplifying this gives
are a and 2a respectively. A body is fired straight from the __ ____

÷ 
    ____
3​÷5 ​ GM
surface of the larger stars towards the surface of the smaller u = ​ ____ ​ 
 ​ ​  a   
​ ​  
star. What should be its minimum initial speed to reach the 2
surface of the smaller star?
Example 32   The earth is a uniform sphere of mass M and
Solution R
radius R. Assume that a massive spherical cavity of radius __
​   ​ 
4
Concepts is dug out, as shown in the figure. Find acceleration due
(i) Mistake that one can do in this problem is to apply to gravity and gravitational potential at a point P shown in
energy conservation, taking the speed of the body the figure.
on the surface of the smaller star to be zero.
The body can never reach the smaller star with zero
speed.
(ii) At points close to the larger star, its gravitational
pull is more powerful. Similarly, at points very
close to the smaller star, gravitational pull of the
smaller star will be much stronger than the pull of
the larger star. There is a point between them where
field is zero. Force on a body kept there will be Solution
zero. Once the projected body crosses this point, it Concepts
will be automatically pulled by the gravity of the
smaller star. (i) Both field and potential obey the principle of
superposition.
(iii) While writing potential energy of the body, we
must consider both the stars. (ii) Field (or potential) at P is equal to field (or poten-
tial) due to complete Earth (mass M) minus the
field (or potential) due to the spherical mass in the
cavity.

4
Mass of volume __
​   ​  p R3 is M.
3
Therefore, mass of volume __
4 R 3
​   ​  p ​​ __
3 (  ) M
​   ​   ​​ ​ is m = ___
4
​    ​ 
64

Let P be the point where a body experiences no force. To Acceleration due to gravity is equal to the field at a point.
the left of P, a body gets attracted towards the bigger star
GM
and to the right of P, a body will fall towards the smaller Field at P due to mass M is E1 = _____
​    ​ (Towards PO)
star. (3R)2
GM
G (16M) m _________
G (M) m Field at P due to mass m is E2 = _____
​    ​ 
​ _________
2
 ​ =
  ​ 
     ​ (CP)2
x (10 a – x)2
G M/64
= _______
​   ​ 
(towards PO)
10a – x __ 1 (9R/4)2
fi  ​ _______
x      ​   ​   fi  x = 8a
​ =
4
\ Field at P due to the cavitied sphere is
A body projected from point A slows down till it reaches
P. If it just manges to reach P (with almost zero speed), it GM GM 35 GM
E = E1 – E2 = ____
​  2 ​ – ________
​   2  ​ = ____
​    ​ ____
​  2 ​ 
will definitely hit the smaller star. To the right of P, pull of 9R 4 × 81R 324 R
smaller star is stronger.
Energy conservation,
11.32  Mechanics II

Potential can be calculated in similar manner, as Solution


GM
3R
GM/64
( 
V = V1 – V2 = – ​ ____ ​ – ​ – ______
​ 
9R/4
 ​  
  )
47 GM
​ = – ​ ____  ​ ​ ____
144 R
​ 
   Concepts
(i) A body (mass m) on the surface of Earth feels the
Example 33   A body is projected from the surface of the
gravitational
​_› pull of Earth and a centrifugal force.
earth in vertically upward direction. Velocity given to it is If ​F   ​ is the resultant of these_two forces, then accel-
​›
_
​› ​F   ​
just sufficient to take it to infinity. Calculate the time that eration of free fall is ​g   ​¢ = __
​ m ​ 
is taken to reach a height R above the surface. R is the
(ii) Latitude of a place is the angle that position vector
radius of the earth.
of the place relative to the centre of the earth makes
Solution with equatorial plane.
Concepts
____
(i) Speed of projection is escape speed = ÷    
​ 2gR ​
(ii) Using energy conservation, we can find speed (v)
at a distance r from the centre.
dr
(iii) Now, putting v = __​   ​ , separating the variables and
dt
integrating gives the answer.
Use of energy conservation greately simplifies the
mathematics.
____
  ​ 
Projection speed is u = ​÷2gR 
Let v be the speed at distance r from the centre.
Consider a particle of mass m at place P (at latitude f).
Conservation of energy gives: Particle rotates (with the Earth) in a circle of radius
1 GMm __ 1 GMm r = R cos f
​ __ ​  mv2 – _____ ​ = ​   ​  mu2 – _____
​  r   
  ​     ​

2 2 R Centrifugal force on it is
2GM
fi  v2 = _____
​  r   
​  + 2gR – 2gR Fc = mw2r
Resultant of gravitational force mg and Fc has been shown
[  2
​   u = 2gr  and ​      ] GM
____
R
​ = gR  ​ as mg¢
________________________________

2 2
  2 + Fc2 +    
mg¢ = ​÷(mg) 2 ◊ mg (Fc) ◊ cos (180 – f) ​
2 2GM _____
_____ 2GM ___ R R
___
fi  v = ​  r    ​ = ​  2 ​   ​  r ​ = 2g ​  r ​  ____________________
R fi  g¢ = ÷ ​ g  2 + (w2r)2    
– 2gw2r ◊ cos f ​
___ 1_ dr ___ 1_
  ​  ◊ R ◊ ​ ___
fi  v = ​÷2g    ​   fi ​ __ ​  = ÷    ​ ◊ R ◊ ​ ___
​ 2g    ​ 
​ r 
÷    ​ dt ​ r 
÷    ​ The term w4r2 is extremely small due to small angular
t
fi  ​Ú  ​ ​ dt = ​     ​ ​
_____1___
2R
  Ú ​  ​ ​ ÷
_
​ r 
   ​ dr
(  ​ 
2p
speed of Earth ​ w = _________    
24 × 3600 s )
rad
 ​ ​ ___ ​    ​
0    ​ R
R ​÷2g  _____________ __ 1
    ​ = ​​[ g – 2gw R cos  f ]​2 ​
​   ​ 
\  g¢ ​÷g  2 – 2gw2r cos f  2 2 2
2 1___ 3/2 2R
fi  t = __
​   ​  ______
​       ​ [r ​]​ ​  ​
3 R ​÷2g    ​ 
[  ]
R 1
__
2w2R ​   ​ 
= g ​​ 1 – _____ ​ cos2 f  ​2 ​
​  g   

2 ___ 1
= ___
​     ​ ____
​     ​ [(2R)3/2 – (R)3/2]
3R ​÷2g    ​  Using binomial expansion and neglecting higher order
terms

[ 
__

÷ 
__

]
÷   
​ 2 ​ __ R
= ___     –1) ​ __
​   ​ (2 ​÷2 ​ ​ g ​ ​   Rw2 cos2 f
3 g¢ g ​ 1 – _________
​  g    ​ = g – Rw2 cos2 f
​  

Example 34   Considering the angular speed of Earth to be D g


Fractional change = ___
​  g   ​ 
w about its rotation axis, find the fractional change in value
of acceleration of free fall due to rotation of Earth at a place Rw2 cos2 f
located at latitude f. Radius of the Earth is R. = – ​ _________
g   ​

Gravitation  11.33 

Example 35   Acceleration due to gravity at the poles


of Earth is g = 9.80000 ms–2. Find its value at a height
of 32 km above the north pole. Radius of Earth,
R = 6,400 km(exact).
Solution
Concepts
(i) Value of g has been gives up to 6 significant digits. The asteroid passes tangentially to the planet at P.
We must give answer up to 6 digits. Let its speed at P be v.
[  2h
]
(ii) g¢ = g ​ 1 – ​ ___ ​   ​ may not be useful, as we have
R
Angular momentum conservation about O:
neglected higher-order terms, which may affect the mvR = mud  fi  vR = u (2R)  fi  v = 2u ...(i)
answer if we need 6 significant digits. We must
include other higher terms as well. Energy conservation:


g
g¢ = ________
​    2  [  h –2
 ​ = g ​​ 1 + __ ]
​   ​   ​​ ​
K• + U• = KP + UP
h
​​ 1 + __ [ 
​   ​   ​​ ​
R ] R 1
2
1
fi ​ __ ​  mu2 + 0 = __
GMm
​   ​  mv2 – _____
2
​     
R


n (n – 1) 2
We know that (1 + x)n = 1 + nx + ​ _______  ​ x

  (2u)2 ____
u2 _____ GM 3 GM
2! fi  ​ __ ​ =
  ​   ​   ​  fi ​ __ ​  u2 = ____
 – ​      ​     
​ 
2 2 R 2 R
n (n – 1)(n – 2) 3
+ ​  _____________
÷ 
 ​
     x + ...
÷​ __​  3 ​  gR ​
_____ ____
3! 2GM 2
fi  u = ​ _____
​     ​ ​ 
  =  
3R
\
2h (– 2) (– 2 – 1) __
g¢ = g 1 – ___
​   ​ + ​ ___________
R 2
 ​
    
h 2
(  )
​​ ​   ​   ​​ ​
R
Example 37   A satellite is suddenly stopped in its circu-

(– 2)  (– 2 – 1) (– 2 – 2) __
+ ​  ___________________
   
1  ×  2  ×  3
 ​
h 3
(  )
   ​​ ​   ​   ​​ ​ + ...
R
lar orbit and released. Show that the time in which it will
1__
fall on the planet is ​ ____
   ​ times its original time period of
= g [1 – 2 × 0.005 + 3 × (0.005)2 – 4(0.005)3 + ...]   
4​÷2 ​
= g [1 – 0.010 + 0.000075 + very small terms] revolution. Assume that the original radius of circular path of
the satellite is large compared to the radius of the planet.
= 9.80000 × 0.990075 = 9.70274
Solution
Example 36   An asteroid of mass m is approaching a planet
Concepts ____

÷ 
of mass M (>> m) from a GM
large distance. The speed (i) For v0 = ​ ​ ____  ​ ​ path is circular. If v < v0, path
r   
of the asteroid is u and the becomes elliptical. If v is made very small, the
impact parameter is d = 2R, path will be an extended flat ellipse (eccentricity
where R is the radius of the close to 1). If v = 0, we can think that the straight
planet. The asteroid just line path (on which the satellite falls) is an ellipse
misses the planet. Find u. with eccentricity e = 1.
Solution (ii) Since planet is small, we can treat it like a point.
Concepts (iii) After the satellite is stopped, its path (a straight
line) is an ellipse with major axis equal to r. Here,
(i) Since M >> m, we can treat the planet to remain
r is the radius of its original circular path.
fixed.
(ii) Due to gravitational pull of the planet, the asteroid Path of the satellite is straight line path 3. It can be
takes a curved path. It just misses the planet. This regarded as an extreme case of elliptical path, with e = 1.
means it passes tangentially to the planet. Semi-major axis of this ellipse is
(iii) The angular momentum of the asteroid about the
r
centre of the planet is conserved as the only force a = __
​    ​ 
that it experiences passes through the centre. 2
(iv) Energy is conserved.
11.34  Mechanics II

Conservation of angular momentum:

mvr = m (v0 sin q) r0

÷ 
_____
v0r0 1 GMr0
fi vr = ____   fi  v = __
​   ​   ​    ​ ​ _____
​   ​ ​  
   ...(i)
2 2r 2
Conservation of energy gives:

1 GMm __ 1 GMm
​ __ ​  mv2 – _____ ​ = ​   ​  mv02 – _____
​  r   
  ​  r  ​ 

2 2 0

1 GMr0 ____ GM ____ GM GM


fi ​ ___  ​ _____
​   ​  ​ = ​   ​ – ____
 – ​  r    ​  r  ​ 

16 r2 4r0 0
If T1 is the original time period in circular orbit and T2
r0 1 3
is time period in path 3, then fi ​ ____  2 ​ – ​ __r ​  = – ​ ___  ​   fi  12r2 – 16r0 ◊ r + r20 = 0
16r 4r0
T2 2
​​ __
T1 (  ) r/2 3
​   ​   ​​ ​ = ​​ ___ (  )
​  r    ​  ​​ ​
( 
4 ± ​÷13 ​
   
fi r = ​ ​ _______
6
 ​  
 ​ r0
___

)
T1
fi T2 = ___
(  ( 
​  __  ​  ___ ___
  
​÷8 ​
Actually, the satellite will never complete ellipse 3 (which
\  rmin
   
4 – ​÷13 ​
= ​ ​ _______
6 ) 4+÷    
​ 13 ​
 ​ r0  and  rmax = ​ ​ _______
 ​  
6
 ​  
 ​ r0 )
is a straight line). It will hit the planet after completing half
the ellipse. Hence, required time is Example 39   A satellite of Earth is revolving in a circular
orbit of radius x. A gun in the satellite in aimed directly
T2 T1 T1 towards Earth. It fires a bullet with a velocity equal to half
t = __
​   ​  = ____
​  __  ​ = ____
​  __  ​  the orbital speed of the satellite, relative to the satellite. The
2   
2​÷8 ​   
4​÷2 ​
satellite is massive and its recoil can be neglected. Find the
Example 38   A planet is going around the sun (S). At a maximum and minimum distance of the bullet from the
certain instant, distance of the planet from the sun is r0 and centre of the earth during its subsequent motion.

÷ 
____
GM Solution
its speed is v0 = ​ ____
​   ​ ​ 
. Velocity

2r0
Concepts
vector of the planet makes an v0
(i) Velocity of bullet wrt the satellite is ​ __ ​  in radial
angle q = 30° with the radius 2
vector. Calculate maximum and direction, where v0 is the orbital speed of the satel-
____

÷ 
minimum distance of the planet GM
from the sun. M = mass of the lite. v0 = ​ ____
​  x   
​ ​ . 
sun. (ii) Velocity of bullet (wrt earth) is the vector sum of
v0
Solution satellite’s velocity v0 (tangential) and __
​   ​  (radial).
2
Concepts (iii) Angular momentum of the bullet about the centre
of the earth and its energy is conserved.
(i) Angular momentum of the planet about the sun is
conserved. ____
(ii) Angular momentum at aphelion and perihelion can
be written as mvr since v is ^ to r.

GM
v0 = ​ ____÷ 
​  x   
​ ​    ...(i)

In the position shown, angular momentum is Speed of the bullet immediately after it leaves the satellite
m (v0 sin q) r. is

÷  (  )
_________ __
(iii) Energy of the planet is conserved. v0 2 ___   
​÷5 ​
v = ​ v02 + ​​ ​ __ ​   ​​ 
​ ​ = ​   ​ v0
2 2
Let v be the velocity when it is perpendicular to the radius Let the speed of the bullet be u when its velocity is
vector (r). At this instant, the planet is either at maximum perpendicular to the position vector wrt the centre of the
or minimum separation from the sun. earth. Let its distance from the centre of the earth be r at
this instant.
Gravitation  11.35 

Energy conservation:

(  )
__
1 GMm 1 ÷     2 GMm
​ 5 ​
​ __ ​  mu2 – _____
​  r   
​=
  ​ __ ​  m ​​ ___
​   ​  v0  ​​ ​ – _____
​  x   


2 2 2
5 GM ___ v 2 GM
fi ​ __ ​  v02 – ____ ​ = ​   ​  – ​ ____
​  x     ​
r   
8 2
____ _____
GM
Substituting v0 = ​ ____ ÷  GMx
 ​ ​   and  u = ​ _____
​  x    ​  r    ÷ 
​ ​  

5 GM ____ GM __ x2 GM ____ GM
​ __ ​  ____
​  x     ​ = ​  2 ​  ____
​ – ​  x    ​     
​ – ​  r   
 ​
8 r 2x
fi 3r2 – 8x ◊ r + 4x2 = 0
__________


8x ± ÷   x ​
​ 64 2
– 48x
r = ​ ________________
2
    ​
    
2x
= 2x and ___
​   ​ 
Conservation of angular momentum: 6 3
2x
mur = mv0 ◊ x \ rmax = 2x, rmin = ___
​   ​ 
_____ 3
fi   
ur = v0 x = ​÷GMx 

  ...(i)
Worksheet 1
1. The force of gravitation is (a) 4.9 ms–2 (b) 0.98 ms–2
(a) repulsive (b) electrostatic (c) 0.49 ms–2 (d) 49 ms–2
(c) conservative (d) non-conservative 9. The distance of the centres of moon and earth is D.
2. Which of the following is the evidence to show that The mass of earth is 81 times the mass of the moon.
there must be a force acting on earth and directed At what distance from the centre of the earth, the
towards the sun? gravitational force on a partcile will be zero:
D 2D
(a) Deviation of the falling bodies towards east (a) ​ __ ​   (b) ​ ___ ​ 
(b) Revolution of the earth round the sun 2 3
4D 9D
(c) Phenomenon of day and night (c) ​ ___ ​   (d) ​ ___ ​ 
3 10
(d) Apparent motion of sun round the earth
3. The atmosphere is held to the earth by: 10. The value of ‘g’ at a particular point is 9.8 ms–2.
Suppose the earth suddenly shrinks uniformly to half
(a) winds (b) gravity its present size without losing any mass. The value
(c) clouds (d) None of these of ‘g’ at the same point (assuming that the distance
4. If the earth stops rotating, the value of free fall of the point from the centre of earth does not shrink)
accelaration at the equator will: will now be
(a) increase (b) remain same (a) 4.9 ms–2 (b) 3.1 ms–2
–2
(c) decrease (d) None of these (c) 9.8 ms (d) 19.6 ms–2
5. If the earth rotates faster than its present speed, the 11. Weight of a body of mass m decreases by 1% when
weight of an object will it is raised to height h above the earth’s surface. If
(a) increase at the equator but remain unchanged at the body is taken to a depth h in a mine, change in
the poles its weight is:
(b) decreases at the equator but remain unchanged (a) 2% decrease (b) 0.5 decrease
at the poles (c) 1% increase (d) 0.5% increase
(c) remain unchanged at the equator but decreae at 12. At what depth below the surface of the earth,
the poles acceleraton due to gravity g will be half its value
(d) remain unchanged at the equator but increase at 1,600 km above the surface of the earth [radius of
the poles the earth = 6,400 km]
6. If R is the radius of the earth and g is the accelera- (a) 4,352 km (b) 3,192 km
tion due to gravity on the earth’s surface, the mean (c) 1,563 km (d) None of these
density of earth is: 13. R is the radius of the earth and w is its angular veloc-
(a) 4p G/3gR (b) 3p R/4gG ity and gp is the value of g at the poles. The effective
(c) 3g/4p RG (d) p RG/12G value of g at the latitude l = 60° will be equal to:
7. Acceleration due to gravity on moon is 1/6 of the 1 3
(a) gp – __
​   ​  Rw 2 (b) gp – __
​   ​  Rw 2
acceleration due to gravity on earth. If the ratio of 4 4

(  )
re
___ 5
__
densities of earth (re) and moon (rm) is ​ ​   ​  ​ = ​    ​
rm
then radius of moon Rm in terms of radius of the
3
(c) gp – Rw  2 1
(d) gp + ​ __ ​  Rw 2
4
earth Re will be: 14. If both the mass and the radius of the earth decrease
5 1 by 1%, the value of the acceleration due to gravity
(a) ​ ___  ​ Re (b) ​ __ ​  Re will
18 6
3 1__ (a) decrease by 1% (b) increase by 1%
(c) ​ ___  ​ Re (d) ​ ____    ​ Re
18 2​÷3 ​   (c) increase by 2% (d) increase by 0.5%
8. If the mass of earth is 80 times of that of a planet 15. If the radius of earth is decreased by 4% and its
and diameter is double that of planet and ‘g’ on earth density remains same, then escape velocity from its
–2
is 9.8 ms , then the value of ‘g’ on that planet is: surface will

11.36
(a) remain same (b) incerease by 4% shaded areas A and B are also
(c) decrease by 4% (d) increase by 2% shown in the figure, which can
16. The escape speed for a projectile in the case of earth be assumed to be equal. If t1
is 11.2 kms–1. A body is projected from the surface and t2 represent the time for the
of the earth with a velocity which is euqal to twice planet to move from a to b and
the escape speed. The velocity of the body when it is d to c respectively, then
at infinite distance from the centre of the earth is: (a) t1 < t2 (b) t1 > t2
(a) 11.2 kms–1 (b) 22.4 kms–1 (c) t1 = t2 (d) t1 £ t2
__ __
    kms–1
(c) 11.2 ​÷3 ​     kms–1
(d) 11.2​÷2 ​ 23. A satellite is revolving around earth in a circular
orbit. The radius of orbit is half of the radius of the
17. The gravitational potential energy of a body of mass orbit of moon. Satellite will complete one revolution
‘m’ at the earth’s surface is – mgRe. Its gravitational in
potential energy at a height Re from the earth’s (a) 2 –3/2 lunar month (b) 2 –2/3 lunar month
surface will be (Here, Re is the radius of the earth)
(c) 23/2 lunar month (d) 22/3 lunar month
(a) – 2mgRe (b) 2mgRe
24. A particle falls on earth: (i) from infinity (ii) from a
1 1 height 10 times the radius of earth. The ratio of the
(c) ​ __ ​  mgRe (d) – ​ __ ​  mgRe
2 2 velocities gained on reaching the earth’s surface is
___ ___ ___ ___
18. A satellite of earth is moving in its orbit with a    
(a) ​÷ 11 ​  : ​÷10 ​    
(b) ​÷ 10 ​  : ​÷11 ​
constant speed v. If the gravity of earth suddenly (c) 10 : 11 (d) 11 : 10
vanishes, then this satellite will
25. Two satellites of same mass m are launched in the
(a) continue to move in the orbit with velocity v. same circular orbit of radius r around the earth
(b) start moving with velocity v in a direction tan- (mass M) so as to rotate opposite to each other.
gential to the orbit. If they collide inelastically and stick together as
(c) fall down with increased velocity wreckage, the total energy of the system just after
(d) be lost in outer space collision is:
19. A satellite is moving around the earth. In order 2GMm GMm
(a) – ​ ______​  
r    (b) – ​ _____
r    

to make it move to infinity, its velocity must be
increased by GMm GMm
(c) ​ _____ ​  
    (d) ​ _____ ​ 
   
(a) 20% 2r 4r
26. A solid sphere of radius
(b) it is impossible to do so
R/2 is cut out of a solid
(c) 82.8% (d) 41.4% sphere of radius R such
20. A planet of mass m is moving in an elliptical orbit that the spherical cavity
about the sun (mass of sun = M). The maximum and so formed touches the
minimum distances of the planet from the sun are r1 surface on one side and
and r2 respectively. The period of revolution of the the centre of the sphere
planet will be proportional to on the other side, as
(a) r13/2 (b) r23/2 shown. The initial mass
3/2
(c) (r1 – r1) (d) (r1 + 2)3/2 of the solid sphere was M. A particle of mass m is
21. A ball is dropped from a spacecraft revolving around placed at a distance 2.5R from the centre of the cavity
the earth at a height of 120 km. What will happen to O, as shown. What is the gravitational attraction on
the ball? the mass m?
GMm GMm
(a) It will continue to move with velocity v along (a) ​ _____
 ​  
  (b) ​ _____ ​ 

the original orbit of spacecraft R2 2R2
(b) It will move with the same speed tangentially GMm 23 GMm
to the spacecraft (c) ​ _____ ​  
  (d) ​ ____  ​ _____
​   ​  
8R2 100 R2
___
(c) It will fall down to the earth gradually 27. A body____B is fired with a velocity of magnitude ​÷gR   ​ 
(d) It will go very far in the space    
< v < ​÷2gR  ​ at an angle of 30° with the radius vector
22. The figure shows the motion of a planet around the of earth. If at the highest point, the speed of the body
sun in an elliptical orbit with sun at the focus. The

11.37
is v/4, the maximum height 33. A (non-rotating) star collapses onto itself from an
attained by the body is equal initial radius Ri, with its mass remaining unchanged.
to: Which curve in the
(a) R/2 figure best gives
(b) R__ the gravitational
(c) ​÷ 2 ​ R acceleration ag on
(d) none of these the surface of the
star, as a function
28. Suppose the gravitational force
of the radius of the
varies inversely as the nth power
star, during the collapse?
of distance. Then the time period of a planet in cir-
cular orbit of radius R around the sun will be pro- (a) a (b) b
portional to (c) c (d) d
34. A satellite revolves in the geostationary orbit but in
(  )
(a) ​R​
n + 1
​ ____
​   ​  
2 ​
 ​
(b) ​R​(  )n – 1
​ ____
​   ​  
2 ​
 ​
a direction east to west. The time interval between
​  ____
its successive passing about a point on the equator
(d) ​R​( 2 )​
n – 2
​   ​  
 ​
(c) Rn is:
29. A satellite is launched into a circular orbit of radius (a) 48 h (b) 24 h
R around the earth. A second satellite is launched (c) 12 h (d) never
into an orbit of radius (1.01) R. The period of the
35. A satellite of mass 5M orbits the earth in a circular
second satellite is larger than that of the first one by
orbit. At one point in its orbit, the satellite explodes
approximately
into two pieces, one of mass M and the other of
(a) 0.5% (b) 1.0% mass 4M. After the explosion, the mass M ends up
(c) 1.5% (d) 3.0% travelling in the same circular orbit, but in opposite
30. If the distance between the earth and the sun becomes direction. After explosion, the mass 4M is
half its present value, the number of days in a year (a) in a circular orbit
would have been (b) unbound
(a) 64.5 (b) 129 (c) elliptical orbit
(c) 182.5 (d) 730 (d) data is insufficient to determine the nature of
31. A spherical uniform planet is rotating about its axis. the orbit.
The velocity of a point on its equator is v. Due to 36. A satellite can be in a geostationary orbit around
the rotation of planet about its axis, the acceleration earth at a distance r from the centre. If the angular
due to gravity g at equator is 1/2 of g at poles. The velocity of earth about its axis doubles, a satellite
escape velocity of a particle on the pole of the planet can now be in a geostationary orbit around earth if
in terms of v is its distance from the centre is
(a) ve = 2v (b) ve = v r r__
__ (a) ​ __  ​   (b) ​ ____
   ​ 
(c) ve = v/2 (d) ve = ÷    v
​ 3 ​  2   
2​÷2 ​
r r
32. The escape velocity for a planet is ve. A tunnel is dug (c) ​ _____    ​   (d) ​ _____
   ​ 
along a diameter of the planet and a small body is (4)1/3 (2)1/3
dropped into it at the surface. When the body reaches 37. Satellites A and B are orbiting around the earth in
the centre of the planet, its speed will be orbits of radii R and 4R respectively. The ratio of
ve their areal velocities is:
(a) ve (b) ​ ___
__  ​ 
÷   
​ 2 ​ (a) 1 : 2 (b) 1 : 4
v__e
(c) ​   ​   (d) zero (c) 1 : 8 (d) 1 : 16
2

11.38
Worksheet 2
1. The magnitudes of the gravitational force on a (c) can be vertically above any place on the earth
particle at distances r1 and r2 from the centre of a (d) goes round the earth from west to east
uniform sphere of radius R and mass are F1 and 6. A satellite S is moving in an elliptical orbit around
F2 respectively. Then the earth. The mass of the satellite is very small
F1 r1 compared to the mass of the earth
(a) ​ ___  ​ = __
​   ​   if  r1 < R and r2 < R
F2 r2 (a) the acceleration of S is always directed towards
F1 r22 the centre of the earth
(b) ​ ___  ​ = __
​  2 ​   if  r1 > R and r2 > R (b) the angular momentum of S about the centre of
F2 r1
F1 r__1 the earth changes in direction, but its magnitude
(c) ​ ___ ​ = ​ r  ​   if  r1 > R and r2 > R remains constant
F2 2
(c) the total mechanical energy of S varies
F1 r22
(d) ​ ___ ​ =__
​  2 ​   if  r1 < R and r2 < R periodically with time
F2 r1 (d) the linear momentum of S remains constant in
2. Inside a hollow isolated spherical shell, magnitude
(a) gravitational potential is zero everywhere. 7. For a satellite to orbit around the earth, which of the
(b) gravitational field is zero everywhere. following must be true?
(c) gravitational potential is same everywhere. (a) It must be above the equator at some time
(d) gravitational field is same everywhere. (b) It cannot pass over the poles at any time
3. A geostationary satellite (c) Its height above the surface cannot exceed
is at a height h above 36,000 km ___
the surface of earth.    
(d) Its period of rotation must be > 2p ​÷R/g  ​ where
If earth’s radius is R R is the radius of earth
(colatitude at a place is 8. Two satellites s1 & s2 of equal masses revolve in
90°-latitude angle) the same sense around a heavy planet in co-planar
(a) The minimum circular orbit of radii R & 4R
colatitude on earth upto which the satellite can be (a) the ratio of period of revolution of s1 & s2 is
used for communication is sin–1 (R/(R + h)). 1 : 8.
(b) The maximum colatitudes on earth upto which (b) their velocities are in the ratio 2 : 1
the satellite can be used for communication is (c) their angular momentum about the planet are in
sin–1 (R/(R + h)). the ratio 2 : 1
(c) The area on earth’s surface that cannot receive (d) the ratio of angular velocities of s2 and s1 is
communication from this satellite is given as 2 : 5.
2p R2 (1 + sin q). 9. Figure shows the orbit
(d) The area on earth’s surface that can of a planet P around the
receive signal from this satellite is given as sun S. AB and CD are the
2p R2 (1 + cos q). minor and major axes of
4. When a satellite in a circular orbit around the earth the ellipse.
enters the atmospheric region, it encounters small air (a) Time needed for the
resistance to its motion. Then planet to travel half
(a) its kinetic energy increases the ellipse ACB is same as the time needed to
(b) its kinetic energy decreases travel the other half BDA.
(c) its angular momentum about the earth (b) If U is the potential energy and K is the kinetic
decreases energy, then |U| > |K| at both D & C
(d) its period of revolution around the earth (c) If U is the potential energy and K is the kinetic
increases energy, then |U| > |K| at D but not at C
5. A communication satellite (d) If U is the potential energy and K is the kinetic
(a) goes round the earth from east to west energy, then |U| > |K| at C but not at D
(b) can be in the equatorial plane only

11.39
Worksheet 3
1. A planet has uniform mass 7. 1 astronomical unit (AU) is average Earth – sun
density r and is rotat- distance. It was estimated that within a space of
ing about its axis with an radius r = 3 × 109 AU, centred at the centre of a
angular speed w. A straight galaxy, all celestical bodies have a total mass of 2.8 ×
smooth tunnel is dug across 1011 times that of our sun. Stars located at a distance
it, perpendicular to its axis. r from the centre of the galaxy were found to be
An object placed inside orbiting the centre with a period T = 3 × 108 years.
the tunnel, at any location, Assume that the entire mass of the galaxy within
does not experience accel- radius r can be assumed as a point mass at its centre
eration. Find w. for calculating the time period of a star and estimate
__
 a
2. A uniform ring of mass m lies at a distance of ​÷3 ​  the mass that must be present in the galaxy in a space
from the centre of a solid sphere of mass M. Axis of of radius r.
the ring is the diameter of the sphere. Both objects How much is this mass different from the estimate
have same radius a. Find the force applied by the made by visual observations as given in this problem?
ring on the sphere. This missing mass is dark matter.
8. The speed of a point on the equator of a planet is u
due to rotation of the planet about its own axis. Due
to rotation, the weight of a body at the equator is half
of its weight at the poles of the planet. Calculate the
speed with which a body on a pole shall be projected
from the pole so that it just manages to escape the
gravity of the planet.
3. A body of mass m is raised to a height h above
the surface of the earth and you use the equation 9. A ‘moon’ of a planet (mass = M) is orbiting in a
D U = mgh for calculating change in PE. At what circular orbit of radius r. It suddenly explodes into
value of h will the error in value of D U be 1%? three pieces in mass ratio 1:1:4. Immediately after the
explosion, one of the smaller fragments starts orbiting
4. Three stars, each of mass m, are located at the vertices
the planet in reverse direction in the same orbit. The
of an equilateral triangle of side length a. They
other smaller piece has zero initial velocity after the
revolve in circular orbit under mutual gravitational
explosion and falls straight towards the planet.
force while preserving the equilateral triangle. Find
the speed of each star. Also find the mechanical (a) Find the speed of the smaller piece when it is
energy of the system. about to hit the surface of the planet. Radius of
r
5. A scientist found that a planet of radius 3R is made the planet is __
​    ​. 
2
of two different materials. The inner core of radius (b) What happened to the third piece? Is is still a
R is a uniform dense liquid. Mass of the core is M. satellite?
The remaining part is solid with density half that of 10. Two particles A and B have masses m and 2m
the liquid core. Find the gravitational acceleration of respectively. They are held at separation r0 in space.
a particle at points A is given a velocity v0 along the line joining
(a) R and  (b) 3R from the centre of the planet. the two masses (away from B) and B is released
simultaneously. Find the range of velocity v0 for
6. A satellite is revolving around the earth in an equato-
which the two particles would remain bound under
rial plane from west to east direction. Its time period
their mutual gravitation.
is 8 hours. On 17 January 2018, at 8 am, the satellite
was exactly above a city Pontianak in Indonesia 11. Two identical particles are fixed at points B and
[Pontianak is a city, which lies on both the hemi- C. A third particle A is moved along perpendicular
spheres of Earth]. At what time and on which date the bisector of the line BC. It is moved from y = 0 to
satellite will once again be overhead Pontianak? Just y = •. Mass of the third particle is twice the mass
write the time and date nearest to 17/01/2018, 8 am. of particle at B (or C). The potential energy of the
three-particle system changes with distance (y) of A

11.40
from O as shown in the graph. Express value of U 16. A ring has mass M and radius R. Write gravitational
in terms of U0. potential due to the ring at
(a) its centre
(b) a point on its axis at a distance x from the
centre.
16a. A satellite of mass m is revolving in a circular orbit
of radius r around the earth (Mass M and radius R).
Due to atmospheric drag, it loses energy at a constant
rate w. It follows a spiral path and falls to earth.
Estimate the time in which it will fall to earth.
  17. A tunnel is dug along the diameter of the earth.
A particle of mass m is released from point P at
12. A projectile is fired from the surface of the earth at an a height h = R. The particle enters the tunnel and
angle q = 60° to the vertical. Its projection speed is u moves without friction.

÷ 
____
GM
= ​ ____
​     
​ ​  
. How high does the projectile rise? Neglect
R
rotation of the earth. Mass of the earth is M and its
radius is R.
13. Gravitational PE of a particle of mass m = 1 kg varies
with distance (r) from the centre of a planet, as
shown in the figure. Radius
of the planet is R. The par-
ticle is projected from the
surface of the planet with a (a) What is the maximum displacement of the par-
KE of 6 × 109 J. The maxi- ticle after it is released?
mum height attained by the (b) What is the maximum speed of the particle after
particle above the surface it is released?
is R. At what angle to the 18. The minimum and maximum distance of a satellite
vertical was the projectile from the centre of the earth are 2R and 4R respectively.
fired? Mass of the earth is M and its radius is R. Find:
14. At what angular speed of rotation will the surface (a) The minimum speed of the satellite in its
material on the equator of a planet be on verge of fly- path.
ing off the surface of the planet, if the planet is spher-
ical with radius 1,800 km and mass 3 × 1024 kg? (b) The radius of curvature of the path when it is
nearest to the earth.
15. A planet of mass M and a star of mass 2M are
revolving around their common COM under influ- 19. A satellite is moving in an elliptical orbit of
ence of mutual gravitational pull. Distance between semi-major axis a around a planet of mass M. Find
them is r. Find the total mechanical energy of the its speed when it is at a distance r from the centre
system. of the planet.

11.41
Answers Sheet
Your Turn
1. It becomes 4 times
2. Forces between building and us are relatively small. These tiny forces are not meaningful when we are overwhelmed
by the huge attraction of Earth. __
22 ÷     GMm
​________
2 ​
3. They have same acceleration. 4. 3.56 × 10  N 5(a). ​   ​ 

__ x2
÷     Gm2
​ 3 ​ GMm GMm
(b) ​ _______ ​  
  6. (a) 0 (b) _____
​  2 ​  
  7. ​ _____ ​ 

a2 x 2L2

2GM
8. (4GM)/(3x2) along DA 9. zero 10. ​ _____  ​   11.
p R2

÷ 
__
R R
12. 1.62 N kg –1
13. ​ __ ​   14. 2p ​ __
​ g ​ ​   84.6 minutes
2
3
15. ​ __ ​  R 16. (a) 1600 km (b) 11.5 km 17. Nearly 11 km
4

( 
____
6Gm2
) ÷ 
1 Gm ___
18. 39.2 ms–2 19. ​ _____​ ​ 1 – ___
a    ​  __  ​  ​ 20. ​ ____
​  r   
​ ​      ​  
21. ​÷gR 
÷   
​ 2 ​
vA 1
22. (a) Yes (b) – 0.4 E0 (c) E0 23. ​ __
vB  ​ =
__
​   ​  24. 2R
2
GM
25. (a) outside (b) – 3.14 × 108 J 26. No 27. – ​ ____​ 
  
R
28. 3-3 29. (a) Increases (b) Decreases 30. 8.48 h
___
31. (a) h = R (b) ÷    ​ 32. 9.6 × 1010 J
​ gR  33. (a) No (b) Elliptical 34. Yes
v1r1
35. ​ ______   ​   36. (a) 3.3 years (b) P2 (c) P2
r2 sin q

37. Yes, so that MOM intercepts the Mars orbit when the planet was there.

Worksheet 1
1. (c) 2. (b) 3. (b) 4. (a) 5. (b) 6. (c) 7. (a) 8. (c) 9. (d)
10. (c) 11. (b) 12. (a) 13. (a) 14. (b) 15. (c) 16. (c) 17. (d) 18. (b)
19. (d) 20. (d) 21. (a) 22. (c) 23. (a) 24. (a) 25. (a) 26. (d) 27. (b)
28. (a) 29. (c) 30. (b) 31. (a) 32. (b) 33. (b) 34. (c) 35. (b) 36. (c)
37. (a)

Worksheet 2
1. (a,b) 2. (b,c,d) 3. (a,c) 4. (a,c) 5. (b,d) 6. (a) 7. (a,d) 8. (a,b)
9. (b)

11.42
Worksheet 3

÷ 
______ __ ____

÷  – 3Gm2
4 ÷     GMm
​ 3 ​ GM _______
1. w = ​ __
​   ​  p GP ​   2. ​ ________ ​  
  3. 64 km 4. ​ ____
​  a   
​ ​  
, ​     
​ 
3 8a2 2a
GM 5 GM
5. (a) ____
​  2 ​ (b) __
​   ​  ____
​   ​   6. 8 PM, 17/01/2018 7. 2 × 1010 times mass of the sun
R 9 R2

÷ 
_____ _____
8. 2u
2GM
÷ 
9. (a) ​ _____
​  r   
​ ​  
(b) will escape
6 Gm
10. v0 < ​ _____
​  r  ​ ​ 
0
  

( ÷  )
__
R 2
11. 9U0 12. ​ __ ​   13. sin–1 ​ ​ __
​   ​ ​    ​ 14. 5.85 × 10–2 rad s–1
2 3


GM2
15. – ​ ____
r   
 ​
GM
16. (a) – ​ ____
R
  
GM
​ (b) – ​ ________
______    ​
÷​ R  2 + x2 
 ​
GMm __
16a. _____
​   
  
2W R r
1 1
( 
​ ​ ​    ​  – __ )
​   ​   ​

÷ 
____ __________

÷  (  )
____ GM 8R 2 __ 1
17. (a) 4R (b) ÷   ​  
​ 2gR  18. (a) ​ ____  (b) ___
​   ​ ​  ​   ​   19. v = ​ GM ​ __
​ r ​  –  
​ a ​ 
6R 3

​ ​
11.43
Chapter  12

Elasticity
“The finest steel has to go through the hottest fire’’
–Richard M. Nixon

1.  Introduction in a typical fashion, as shown


in the figure.
While dealing with solid bodies, we assumed them to be At some specific separa-
perfectly rigid. However, no object in real world is perfectly tion (r0), the potential energy
rigid. When an object is subjected to external forces, it
undergoes changes in size, shape, or both. The subject of (  dU
)
is minimum ​ ​ ___ ​   = 0  ​ and
dr
elasticity deals with the behaviour of those substances which the molecules are in equilib-
have the property of recovering their size and shape when rium. All molecules in a solid
the forces producing deformations are removed. This elastic will try to settle down at this
property is present in all solids to some extent. The extent separation (r0). If, by applying
to which the shape of a body is restored when the deform- some external forces, we try to
ing forces are removed depends on microscopic structure increase the separation between the molecules, the force turns
of the solid.
Study of elastic properties of material is of immense ( dU
attractive ​ ___
dU
)
​   ​ is positive and force F = – ​ ___ ​ is negative  ​,
dr dr
importance for engineering designs. when you ride a lift, trying to restore the original position of the molecules. When
you often come across messages like – “carrying capacity external forces decrease the inter-molecular separation, the
400 kg’’. The message may have a lot to do with the strength dU
forces amongst the molecules becomes repulsive (​ ___ ​  is
and elastic properties of the rope holding the elevator car. dr
negative for r < r0); again trying to push back the molecules
to their original separation.
2. Microscopic Reason For A large number of atoms come together and settle into
Elasticity their equilibrium position in a three-dimensional lattice,
to form a metallic solid. The interatomic forces can be
The property of a body to restore the original shape or to
modelled as being
oppose the deformation is known as elasticity. Steel is able
produced by tiny
to completely regain its natural shape after deforming forces
(and stiff) springs
are removed (provided deformation is small). We say that
holding these
steel is perfectly elastic. If a body has no tendency to regain
atoms. Increasing
its shape after deforming forces are removed, it is said to be
the atomic separa-
perfectly inelastic or plastic.
tion will create a
Clay, putty and dough are very close to being perfectly restoring attractive
inelastic. Most of the real-life objects are somewhere in force and decreas-
between. They try to regain their shapes after forces are ing the distance
removed but cannot regain their original shape completely. between the atoms
Some deformation remains. will cause a repul-
The potential energy of interaction of two neighbouring sive force between
molecules in a solid varies with separation (r) between them the atoms.
12.2  Mechanics II

You may note that, in materials not so rigid, like a rubber D L
Longitudinal strain, Œ = ​ ___   ​   ...(2)
pipe, the molecules make flexible chains, each chain being L
loosely bound to its neighbour. One may use ± sign to differentiate between tensile and
compressive strains.
3. STRESS AND STRAIN Strains is a dimensionless quantity. It has no unit.

When a body is subjected to deforming forces, three kinds 3.2  Shear Stress and Strain
of deformations can be seen. Associated with each of these
Consider a solid cylinder of cross-sectional area A, fixed
deformations, there are internal restoring stresses.
rigidly to the ground. A force F is applied on the top sur-
Stress characterises the strength of force that is causing face, in a direction tangential to the surface. Here, the force
deformation and strain is a measure of the amount of defor- is trying to cause
mation produced with respect to the size of the body. a deformation in
Let us take a look at three types of stresses and strains. which two paral-
lel surfaces have a
3.1  Longitudinal Stress and Strain tendency to slide
Consider a metal rod subjected to over one another.
two forces (F) of equal magnitude, Internal forces
acting at the opposite ends, so as develop to prevent this sliding.
to keep the rod in equilibrium. In the figure shown, at any horizontal cross-section of
The rod does not move but gets the cylinder, the two parts apply a tangential force on one
stretched. We say that the rod is another.
subjected to a longitudinal tensile Note that the cylinder shown in figure above is in equi-
stress. If we consider an inter- librium. There are forces acting on it due to the ground as
mediate cross-section of the rod, well.
the two sides pull each other with
Shear stress is defined as tangential force per unit area.
force F. This is internal restoring
In the figure shown, shear stress is
force. The longitudinal stress in
F
the rod is defined as ss = __ ​   ​ ...(3)
A
F
sl = __
​   ​ ...(1) Unit of shear stress is also
A Nm–2.
where A is the area of Shear strain is a measure of
cross-section of the rod. deformation produced when a body
Obviously, force F applied to the ends of a thick rod and is subjected to shear stress. We can
a thin rod will not have the same deforming effects. define it as displacement of a layer
AB divided by its distance from the
Force per unit area gives us a better idea of the deforma-
fixed layer CD.
tion-producing ability of the force. Unit of stress is Nm–2.
x
If the forces applied to the ends of the rod are so as Œs = __
​    ​ = tan q ...(4)
L
to compress it (see fig), we say that the rod is subjected
to longitudinal compressive stress. Shear strain is dimensionless.
At any cross-section, the two parts
3.3  Volume Stress and Strain
of the rod push against each other.
Stress in the rod is still defined by All bodies on the surface of
equation (1). Earth are subjected to atmo-
When the rod is stretched, it elon- spheric pressure (P0). When
gates. Its length (L) increase by D L. pressure on a body is changed
When the rod is subjected to a com- uniformly from all sides, we
pressive stress, its length decreases say that it is subjected to
by D L. In both cases, we define lon- Volume stress or Bulk stress.
gitudinal strain developed in the rod Change in pressure (DP) is
as fractional change in its length. defined as volume stress.
Volume stress, s v = D P ...(5)
Elasticity  12.3

For example, if you submerge a ball deep inside water, (AB) of the rod
the volume stress on it is D P = r gh, where r is the density that is inclined
of water and h is the depth. at an angle q to
When subjected to volume stress, the volume of a body the length of the
changes. Volume strain is defined as the fractional change rod. Write longitudinal and shear stress developed at the
in volume. It has no unit. section AB.
D V
Volume strain,  ŒV = ​ ___   ​   ...(6) Solution
V
Concepts
Example 1   A steel wire is 1 m long and another wire Force
is 2 m long. Both are stretched so as to elongate them by Stress = _____
​   ​ 
Area
1 mm. Can we say that the both wires are equally deformed? For longitudinal stress, we consider force that is normal
Give reasons. to the area and for shear stress, we consider component
Solution  Strain is correct measure of deformation. of forces tangential to the area.
For first wire, longitudinal strain is
Area of section AB is
D l1 _____1 mm S
Œ1 = ___
​   ​   = ​   = 10 –3
 ​  S¢ = ____
​     ​ 
l1 1m sin q
D l2 _____ 1 mm Normal and tangential
For second wire, Œ2 = ___
​   ​  = ​   = 0.5 × 10–3
 ​ 
l2 2m components of F are
First wire is more deformed. Fn = F sin q ; Ft = F cos q
Intermolecular distance have incrased more in the first \  Longitudinal stress at section AB is
wire. You can visualise this by considering two segments of Fn F sin q __ F
1 m length in the second wire. Each 1 m segment is elongated sl = ___
​   ​ = ______
​  = ​   ​  sin2q
 ​ 
S¢ S
____ S
by 0.5 mm only. ​     ​ 
sin q
Example 2   A uniform rod of cross-sectional area Shear stress at section AB is
S is pulled at its two ends by applying forces of equal
Ft F cos q __ F
magnitude acting along its length. Consider a cross-section ss = __
​    ​ = ______
​   ​ 
 = ​   ​  sin q ◊ cos q
S¢ S
____ S
​     ​ 
sin q

Your Turn

Q.1  A wire of length L = 1.0 m is pulled by applying


force of equal magnitudes at its two ends. Magnitude of
each forces is 100 N and area of cross section of the wire
is A = 10 mm2. The wire elongates by 2 mm. Find
(i) Longitudinal strain (ii) Longitudinal stress
Q.2  A cuboidal block is fixed to the ground. A tangential Q.3  A ball is taken deep inside the ocean to a depth of
2 km. Find volume stress on it.
force applied to the top surfaces deforms the block as shown,
with q = 1°. Find shear strain. [Density of sea water = 1.0 × 103 kgm–3, g = 9.8 ms–2]

4. HOOKE’S LAW Proportionality constant (E) is known as modulus of


elasticity.
For small deformations, stress in a body is proportional to We define three moduli of elasticity for three types of
strain. This is known as Hooke’s law. This is true for all stresses and strains.
three types of stresses and the corresponding strains. This
law was discovered by English scientist Robert Hooke, in 4.1  Young’s Modulus (Y)
the seventeenth century.
Consider a rod of length L and cross-section A, stretched
Stress µ Strain
by force F.
Stress = E (Strain) ...(7)
12.4  Mechanics II

F D L
Longitudinal stress, sl = __ ​   ​ (Stress)s = Ys (Strain)s = Ys ​ ___ ​ 
A L1
If the rod elongates by D L, D L
(Stress)cu = Ycu (Strain)cu = Ycu ​ ___ ​ 
then longitudinal strain is L2
D L \ (Stress)s = (Stress)cu
Œl = ___​      ​
L
D L D L
From Hooke’s law (for small \ Ys ​ ___ ​  = Ycu ___
​   ​ 
L1 L2
deformations)
Longitudinal stress µ longi- Ys 2 20
fi ​ ___ ​   = ​ ___  ​ =​ ___  ​ = ___
1
​   ​ 
tudinal strain L2 Ycu 1.1 11
sl
fi ​ __ Œl ​   = Y ...(8) Example 5    Three blocks of masses m 1 = 4 kg,
Proportionality constant Y is known as Young’s modulus m2 = 2 kg, m3 = 2 kg are connected with steel wires of
of elasticity. It is a constant for a given material. cross-sectional area A = 0.05 cm2, as shown in figure. The
incline is smooth and the pul-
We can write
sl ley has no mass. Length of the
FL
Y = __ ​ Œ  ​ = _____
​    ​
  ...(9) wire connecting m1 and m2 is
l A D L
2 m where as length of the other
If the rod is subjected to compressive stress, value of Y wire is 1 m. Find the strain in
does not change. the two wires when the sys-
tem is released to move. Take
Example 3   One end of a wire 2 m long and 0.4 cm2 in g = 10 ms–2 and Ysteel = 2 × 1011 Nm–2
cross-section is fixed to the ceiling of a room and a 5 kg load
is attached to the free end. Find the extension in the wire. Solution
Young’s modulus of the materials of the wire is Concepts
11 –2 –2
Y = 1.2 × 10 Nm . Take g = 10 ms We need to find tension in both wires using Newton’s law
of motion. Then, using equation (9) separately for the two
Solution wires will give the answer.
Concepts
Acceleration of the system is
Direct use of equation (9)
(m1 – m2 sin 30° – m3 sin 30°) g
a = ​ _________________________
         ​
m1 + m2 + m3
Tension in wire F = Mg = 5 × 10 = 50 N

FL
Using (9), D L = ___ ​ 
(50 N) (2 m)
​    ​ = __________________________
         ​ \ a = ​ 
(  1
​ 4 – 2 × __
2
______________________
   
1
​   ​  – 2 × __
2
 ​
)
​   ​   ​ × 10
   = 2.5 ms–2
AY (0.4 × 10  m2) (1.2 × 1011 Nm–2)
–4
4+2+2

= 2.08 × 10–5 m Let T1 = tension is string connecting m1 and m2.


m1g – T1 = m1a  fi  T1 = m1 (g – a)
Example 4   Two wires of equal cross-sections have lengths fi T1 = 4 (10 – 2.5) = 30 N.
L1 and L2. They are made of steel and copper respectively.
They are joined end-to-end and the combination is held T2 = Tension in other string
under a tension such that elongation in steel wire is same as T2 – m3g sin 30° = m3 a
that in the copper wire. Find the ratio of the original lengths
of the two wires. Young’s modulus of steel and copper are fi T2 = 2 (2.5 + 5) = 15 N.

Ys = 2.0 × 1011 Nm–2  and For wire having tension T1


Ycu = 1.1 × 1011 Nm–2 T1L1 30 × 2
D L1 = ____ ​ = ​ ____________________
​         11 ​ = 6 × 10–5m
  
AY – 4
(0.05 × 10 ) × 2 × 10
Solution
Concepts For wire having tension T2
Since, both wires have the same cross-section and tension, T2 L2 15 × 1
D L2 = ​ ____   
​ = ​ ___________________
       ​ = 1.5 × 10–5 m.
  
stress is same in both of them. AY 0.05 × 10–4 × 2 × 1011
Elasticity  12.5

Your Turn
Q.4  A metal wire is subjected to a tension, and the exten- briskly. Find the fractional amount your leg bones are
sion observed is x. If a wire of double the radius, made compressed by walking. Young’s modulus for bone is
of same material, is subjected to double the tension, find
extension. Y = 1 × 1010 Nm–2.

Q.5  A steel cable is used to suspend an elevator car. Its Q.7  Mass of block A in figure is 3 kg and that of block
length is 20 m when lift is at rest and diameter is 0.05 m. B is 6 kg. The cross-sectional area and Young’s modulus
Three people, having a total mass of 238 kg, enter the of the wire connecting the
elevator. Find the blocks are 0.005 cm2 and
2 × 10 11 Nm –2 respec-
(a) Stretch in the cable when lift is at rest
tively. Neglect friction and
(b) Stretch in the cable when lift is moving up with a find strain developed in
constant speed of 2 ms–1 [Ys = 2 × 1011 Nm–2] the wire when the system
Q.6  Your leg bones have a cross-sectional area of about is allowed to move. Take
9.5 cm2 and experience a force of 855 N when you walk g = 10 ms–2.

4.2  Shear Modulus or Modulus of Rigidity (h) Solution


Shear stress µ shear strain. Concepts
Proportionality constant (h) is Direct use of equation (10)
known as shear modulus of the
1 GPa = 109 Nm–2
material.
x
Shear strain = q = __
​    ​
Shear stress F/A F L
h = ​ __________
    ​ = ____
​   ​ = ___
​    ​   ...(10)
Shear strain x/L Aq Shear stress ___ F
Shear strain = ​ __________
 ​
    = ​    ​ 
h Ah
[For small deformations]
4000
Note that unit of h is also Nm –2
(or, Pascal) = ____________________
​    
     ​
(0.25 × 0.25) × 80 × 109

Example 6   A cubical metal block of side length L = 25 cm fi  Shear strain = 8 × 10–7
is fixed on a hard concrete floor. A force of 4000 N is applied x
fi  ​ __ ​   = 8 × 10–7
tangentially on the upper face. Shear modulus for the metal L
is h = 80 GPa. Find shear strain in the block and displace- fi  x = 8 × 10–7 × 25 cm = 2 × 10–5 cm
ment of upper surface. This is displacement of upper surface.

Your Turn

Q.8  A cube of side length 5 cm has its lower face dis- lower face, as shown. Find
placed by 0.2 mm when a tangential force of 8 N acts on its shear stress and modulus
of rigidity of the cube.
12.6  Mechanics II

4.3  Bulk Modulus (B) atmospheric pressure P0 = 1 × 105 Nm–2. By placing weights
on the piston, the pressure on water is increased to twice the
Volume Stress µ Volume strain
atmospheric pressure. Volume of water decreases by 50 cc.
Volume stress
fi  ​ ____________
    ​ = B (a) Find the bulk modulus of water.
Volume strain
(b) Find compressibility of water.
Constant B is known as Bulk modulus of the material.
If volume of a body changes by DV when pressure on it Solution
changes uniformly from all sides by D P, then Concepts
D P
D P D P (i) B = – V ​ ___  

B = – ​ ____ 
 ​ = – V ​ ___  
​ ...(11) D V
D V
___ D V While doing numerical calculations, we can always
​     ​ 
V do away with negative sign and take positive value
Negative sign is to ensure that we get a positive value for everything. Ultimately, what we want is a posi-
of B. Actually, change in volume (D V) is negative (i.e., tive value for B.
volume decrease) when pressure on a body is increased
(ii) Bulk stress is not pressure. It is the change in pres-
(i.e., D P is positive).
sure (D P)
For infinitesimally small changes, we can write
dP
B = – V ​ ___  ​ ...(12) (a) Volume stress, D P = 1 atmosphere = 1 × 105 Nm–2
dV
Unit of B is also Nm–2. D V 50 cc 50 × 10–6
Volume strain, ___
​     
​ = _____
​  3 ​ = ​ ________
 ​   
It is important to note that Bulk modulus is a term that is V 1 m 1
meaningful for solids, liquids, as well as gases. It is some-
D P 1  ×  105
thing that relates to elasticity of volume. All three forms \ B = ____
​     ​ = ________
​   
  ​ = 2 × 109 Nm–2
of matter have volume elasticity (i.e., tendency to regain D V
___ 50 ×  10 –6
​     
​ 
original volume). V
Young’s modulus and shear modulus have no meaning 1 1
for gases and liquids. (b) K = __
​    ​ = _______
​      ​ = 5 ×10–10 m2 N–1
B 2 × 109
Compressibility (K) of a material is defined as the recipro-
cal of the bulk modulus (B)
Note:
1 dV (i) Compressibility of mercury is much less than water.
K = __
​   ​  = – ​ ____  
 ​ ...(13)
B V dP It is only 3.7 × 10–11 m2 N–1
(ii) Gases do not have fixed value of Bulk modulus.
Example 7   Bulk modulus of water
Value of B for a gas depends on something known
as thermodynamic process.
A rigid tank contains 1m3 water. It is covered with a tight
fitting, light movable piston such that water in the tank is at

Your Turn

Q.9  Compressibility of water is 5 × 10–10 m2 N–1. Find Q.10  Bulk modulus of a certain kind of plastic is
change in volume of 1 litre water when pressure on it is 9.8 × 108 Nm–2. To what depth should a plastic cube be
increased by 15 MPa. taken in a lake so that its volume decreases by 0.1%?

5.  MEASURMENT OF YOUNG’S MODULUS the purpose of keeping the wire straight. The experimental
wire, having nearly the same length as the reference wire
The figure shows an experimental set-up to measure young’s is suspended from the fixed support, close to the reference
modulus of a wire. A long references wire, nearly 2 m long. wire. A vernier scale attached to the end of the experimental
is suspended from a fixed support. It has a scale attached wire can slide freely on the scale attached to the reference
to it and there is a heavy load below it. The load serves
Elasticity  12.7

wire. There is a hanger Change in diameter ___D d


attached to the vernier scale, Lateral strain = ​ ________________
  
   ​ = ​     ​ 
Original diameter d
which can carry weights.
Radius of the experi- Change in length D L
Longitudinal strain = ​  _______________
  
   ​ = ___
​     ​ 
mental wire is measured Original length L
using a screw gauge. You
Poisson’s ratio is
will do better to measure
D d/d
it at several locations and s = – ​ _____  ​   ...(14)
take their average. Some D L/L
initial weight (may be 1 kg) The minus sign is for convenience so that s comes out
is placed on the hanger, to positive.
make the experimental wire For most metals, s 0.25 to 0.35. For cork, s 0. It
straight. means the cork shows little lateral expansion when it is
Readings of the main compressed.
scale and vernier scale are
recorded. A known weight
Example 8   Incompressible material
(say 0.5 kg) is added to the hanger. The set-up is left for
about a minute to allow the elongation to happen. Reading A material whose volume does not change on (small) com-
of scale is recorded. The difference in this reading and the pression is called incompressible. What will be poisson’s
previous reading gives the extension in the wire. We go on ratio for an incompressible material?
adding weights and recording the extension. One should Solution
not exceed the breaking strength of the wire. For better Concepts
accuracy, one can repeat the experiment in reverse order by
decreasing the weight (0.5 kg at a time) and noting down the Volume = A ◊ L = p r2 L for a cylindrical solid of radius r
extension. and length L. It is compressed, L decreases but r increases.
If volume does not change, material is incompressible.
From the data obtained,
a graph of extension versus
load is plotted. The graph V = p r2 L
is a straight line passing Let a compressive force be
through the origin. applied on the material and
Slope of the graph (= its length reduce by D L and
tan q) is measured. It gives radius increase by D r.
Extension ___D L D V D r ___ D L
tan q = _________
​   ​  
= ​     ​ ​ ___   
​   = 2 ​ ___
Load Mg V r    ​ + ​  L    ​
Young’s modulus of the wire is calculated as D r D L
For DV = 0 ; 2 ​ ___ ___
r    ​ = – ​  L   ​ 
Stress _______ Mg MgL
Y = _____
​   
 ​ = ​     ​  = ______
​  2    ​ D r/r 1
Strain D L s = – ​ _____   ​ = ​ __ ​  = 0.5
p r2 ◊ ​ ___   ​  p r  D L \
D L/L 2
L
L
fi  Y = _______
​  2    ​   ...(13) Note:  For rubber, s 0.5
p r  tan q
7. LONGITUDINAL STRESS–STRAIN
6. POISSON’S RATIO CURVE FOR A METAL WIRE
When a cylindrical A metal wire of cross-sectional area A0 is stretched by
rod is stretched, its applying load to it. As the load is increased, it stretches
length increases. At more and more. We define stress as
the same time, its
Load
diameter decreases. s = ​ _____
 ​ 
 and a graph of stress versus strain is plotted.
The ratio of lateral A0
A typical graph appears as shown in the figure.
strain and longitudi-
nal strain is known When the strain is small, the stress is proportional to
as poisson’s ratio. strain. Stress–strain graph is a straight line and Hooke’s
12.8  Mechanics II

able to regain its original length. If the deforming force


is decreased gradu-
ally, the original
stress–strain curve
is not retraced. The
work done by rub-
ber cord when it is
returning to origi-
nal length is less
than the work that
was done to deform
it. The difference
law is being obeyed. This is true till point A, which is known
appears as heat.
as .
The phenomenon is known as elastic hysteresis.
Beyond A, stress is no longer proportional to strain. Graph
is not a straight line. However, the material is elastic up to Example 9   Why is rubber padding common in shock
B. This means that the material is able to regain its shape absorbers?
completely if the load is removed. Point B is the elastic limit.
Solution  Suppose a plate A is subjected to rhythmic vibra-
Beyond B, when the load is removed, the wire is unable
tion due to variable shock forces acting on it. Plate B is not
to regain its original length. Some permanent deformation
in direct contact with a, but there is a rubber padding inbe-
remains. Since it is often difficult to pin-point the elastic
tween. The rubber pad is compressed and released in every
limit, we define a point known as yield point (C) that is
cycle of vibration. In
close to elastic limit. Yield point is the point at which a
each cycle, amount of
permanent deformation of 0.2% [i.e., strain = 0.002] remains
energy transmitted to
in the wire if deforming load is removed.
B is less than amount
If the load is increased even further, the strain increases of energy received
at a much rapid pace. Beyond point D, from A. A large part
‘‘necking’’ is seen in the material. After of energy is dissipated
necking, all subsequent deformation takes by rubber due to elastic hysteries. B will have substantially
place in the neck and the material fails at less energy of vibration compared to A.
E, which is known as fracture point (or breaking point).
The stress corresponding to point D is known as ultimate
tensile stress (sU) of the material. It is most common number
9. Elastic Potential Energy
used to judge the strength of a material in tension. In natural state of a body, the molecules settle in their equi-
The graph takes a dip after D, indicating that the wire librium position and the potential energy corresponding to
continues to elongate even if the load decreases a bit. Yes, this position is minimum (Refer to the graph given in section
this is true. The wire elongates even if the load is decreased 2 of this chapter). When deformed, the molecular separation
a bit. But it does not mean that wire elongates even if stress changes and potential energy increases. This happens due to
is decreased a bit. By now, the cross-sectional area of the appearance of an internal forces, against whom work has to
wire has decreased substantially and the true stress is higher be performed to deform a body. The increase in potential
even if the load is decreased slightly. The graph takes a dip energy when body is deformed is known as elastic potential

the original cross-sectional area.


(  )
load
because we are still calculating stress as ​ ____
​   ​  
A0
 ​when A0 is
energy.
Let us calculate the elas-
tic potential energy of a wire,
If a large deformation takes place between elastic limit which is stretched by applying a
and fracture point, material is called ductile. If it breaks soon force .
after the elastic limit is crossed, it is called brittle.
Consider a wire of length L
and cross-section A. We pull it
8. LONGITUDINAL STRESS–STRAIN so as to stretch it slowly. When
CURVE FOR RUBBER extension is x, the tension force
is such that
If we load a rubber cord, it stretches a lot. Strain is high
but the cord remains elastic even when it has extended over F/A
Y = ____
​   ​   [Assuming A to remain constant]
few times its original length. If the load is removed, it is x/L
Elasticity  12.9

fi (  )
YA
F = ​ ___
​   ​  ​ x = kx
Equation (15) can also be written as

( 
L

Where
YA
k = ___
​   ​ is a constant
1
U = __
2
D L
​   ​  ​ AY ___
L)
​     ​   ​ D L
L 1
__
= ​   ​  (maximum value of stretching force) (extension)
Tension (F) changes with stretch (x) just like spring force. 2
...(18)
[fact is that we assumed that an ideal spring is one which
obeys Hooke’s law; implying that F µ x]. Example 10   A steel wire has length L and cross-sec-
Therefore, the elastic potential energy of the wire, when tional area A. It is stretched
its length is increased by D L is given as by gradually increasing force

(  )
1 1 YA till the strain becomes Œ1. The
U = __
​   ​  k (D L)2 = __
​   ​  ​ ___
​   ​  ​ (D L)2 ...(15) stress–strain graph for the entire
2 2 L
process is a straight line, as
This may be written as shown. Find the elastic potential

(  ) (  )
1 D L D L energy in the wire when strain
U = __
​   ​  ​ Y ___
​     ​   ​ ​ ___
​     ​   ​ (LA) is Œ1.
2 L L
1 Solution
fi U = __
​    ​ (stress) (strain) (volume of wire) ...(16)
2 Concepts
1
Energy stored in unit volume of the wire is known as Energy density, u = __
​   ​  × Stress × Strain
2
elastic energy density (u)

\
1
u = __
​   ​  × Stress × Strain ...(17)
Area under the given graph ​ = __ ( 1
2 )
​    ​ s1 Œ1  ​ gives u.
2 1
u = Area under graph = __ ​   ​  s1 Œ1
Unit of energy density (u) is Jm–3. 2
1
\ U = u × Volume = __ ​    ​ s1 Œ1 AL
2

Your Turn

Q.11  A metal wire is stretched by a small amount so as Q.12  A steel wire stretches by 2.0 mm when a load of
to produced a strain Œ in it. Y is young’s modulus of the 800 N is used to stretch it. Calculate the elastic potential
material. Write the elastic energy density in the wire. energy stored in the wire.

Miscellaneous Examples
4
Example 11   A solid sphere of radius R, made of a mate- Volume of sphere, V = __ ​   ​  p R3
rial of bulk modulus B, is surrounded by a liquid in a cylin- 3
If radius changes by D R, the corresponding change in
drical container. A massless piston of area A floats on the
volume is
surface of the liquid. A mass M is placed on the piston to
4
compress the liquid. Find fractional change in radius of the D V = ​ __ ​  p (3R2 D R)
sphere, assuming it to be small. 3
D V 3D R
Solution \ ​ ___   
​   = ____
​     


V R
Concepts
D P
Now, B = _____
​    ​ 
Change in pressure due to weight placed on the piston D V
– ​ ___   
​ 
Mg V
is D P = ___
​     
​. Mg
A 3D R
fi  ​ ____ ​  B = – ​ ___   
   ​
As per Pascal’s law, this change in pressure is transmit- R A
ted uniformly throughout the liquid. Therefore, D P is the D R Mg
volume stress on the solid sphere. fi ​ ___    ​   = – ​ _____   ​ 
R 3 B ◊ A
Negative sign indicates that R decreases.
12.10  Mechanics II

3.14 × 9.8 × 10
Example 12   A wire of length L = 10 m is suspended = ​  ________________________________
      ​ = 10–3 m
   
from the ceiling. The radius of cross-section of the 3.14 × 2 × 1011 × 5 × 10–4 × 9.8 × 10–4
wire changes linearly from a = 5 × 10–4 m at the top to
b = 9.8 × 10–4 m at the bottom. A load of mass m = 3.14 kg Example 13   A thin ring of radius R is made of a material
is attached to the lower end of the wire. Find increase in of density r and Young’s modulus Y. The ring is rotated
length of the wire. Young’s modulus for material of the wire rapidly about its centre in its own place at an angular speed
is Y = 2 × 1011 Nm–2. w. Find the small increase in its radius.

Solution Solution
Concepts Concepts
Tension is same at each cross-section but area is chang- We have learnt how to find tension in the ring in the
ing. Therefore, stress is different at different locations in chapter on circular motion. However, we will reproduce
the wire. the method here.
We will consider a small length dx of the wire, write Tension divided by cross-sectional area gives stress.
extension in it and then integrate to get the overall this stress is uniform throughout.
extension.
Consider an element of the ring that subtends a small
Radius of wire at a distance x from the top is angle Dq at the centre.
r = a + x tan q Length of element = R D q
b–a
= a + ​ ​ _____
L (  )
​  ​ x
   
Mass per unit length = (A ◊ 1) ◊ r = A r

Area of cross section


A = p r2

[  (  ) ]
b–a 2
= p ​​ a + ​ ​ _____
L
​  ​ x  ​​ ​
   

W
Stress = __
​   ​ 
A
If extension is dl in an element of length dx, then,
dl
Strain = ___
​   ​  \  Mass of element, D m = (Ar) (R D q)
dx
Tension on either side of the element = T
dl W W
\ ​ ___ ​   = ___ ​     ​  fi  dl = ___ ​    ​ dx Components of T towards centre provide the necessary
dx AY AY
centripetal force.
mg _____________ dx
\ dl = ___  ​ ​ 
[  (  ) ]
​          ​ q
Y p b–a 2 2T sin D ​__
   ​   = D mw2 ◊ R
​​ a + ​ ​ _____     
​  ​ x  ​​ ​ 2
L
D q
\  Total extension is Ú dl fi 2T ◊ ​ ___ ​    = Ar R D q ◊ w2R
2
L
mg _____________
D L = ___  ​ Ú​  ​  ​ ​ 
dx
Dq D q
[ 
​ For small Dq, sin ​ ___ ​  ​ ___ ​    ​ ]
[  (  ) ]
​          ​ 2 2
p Y 0 b–a 2
​​ a + ​ ​ _____ ​  ​ x  ​​ ​
   
L fi T = Ar w2R2
For integration, we can use substitution method.

(  )
T
Let t = a + ​ ​ _____
b–a
​  ​ x
    [at x = 0, t = a \  Stress in the ring is __
​   ​ = r w2 R2
L A
at x = L, t = b]
fi (  ) b–a
dt = ​ ​ _____
L
    ​  ​ dx

D l
(Strain) Y = r w2R2

fi ​ ___   ​  Y = r w2R2


 ​ ​  ​     
b

p Y ( b – a ) t
mg _____ L dt
\ L = ___
​     ​  ​ Ú​ ​    __
​​   ​ 

2
l
a
2p D R
fi ​ ______   Y = r w2R2
​ 
 ​ ​  ​       ​  ​ ​​  __
p Y ( b – a ) [ t ]
mg _____ L 1 b 2p R
= – ​ ___  ​   ​   ​​ ​​ 
a
r w2 R3
D R = ______
[  ]
mgL 1 1 mgL fi ​      ​

= – ​ _________
    ​ ​ __
​   ​ – __
​   ​  ​ = _____
​    ​  Y
p Y (b – a) b a p Y ab
Elasticity  12.11

dr dV D r D P
  Example 14   A block of mass 1 kg is fastened to one end drV + dVr = 0  fi ​ ___ ​ = – ​ ___ ​   i.e., ​ ___ ​  = ___
​     
​ 
2
of a wire of cross-sectional area A = 3 mm and is rotated r V r B
in a vertical circle of radius 0.2 m. The speed of the block 1 hrg B 1
fi ​ ____   ____
   ​ = ​   fi  hrg = ____
​      ​    ​ = _____
​     ​ 
at the bottom of the circle is 2 ms–1. Find the elongation 100 B 100 100 K
of the wire (in cm) when the block is at the bottom of 1 × 1 × 105
the circle. Young’s modulus of the material of the wire fi hrg =​  _____________
  
    ​
100 × 50 × 10–6
= 2 × 1011 Nm–2.
105
Solution fi h = ____________________
​         ​
5000 × 10–6 × 1000 × 10
Concepts
100 × 103
Change in length of the wire will be far less than 0.2 m = ​ _________
 ​  m = 2 km
50
because Young’s modulus is high. In such a scenario, we
need not worry about change in radius and assume that the Example 16   Increment in length due to own weight
block essentially swings in a circle of radius 0.2 m. A heavy rope of mass M and length L is hanged vertically.
The equation of centripetal force gives the tension in Find elongation in the rope due to its own weight. Assume
the wire. the cross-section to remain uniform at A.
Solution
Let the tension in the wire be T and W be the weight of
the block. Concepts
At the lowest point, the resultant force is T–W towards Tension is different at different points in the rope. Stress,
the centre of the circle and it must be equal to centripetal as well as strain, will be different at different points. A
force m u2/r. small segment at the lower end will suffer less extension
compared to an identical segment at upper end.
Thus, T – W = mu2/r
or, T = W + mu2/r Consider a small element of length dx in the rope at x
(1 kg) (2ms–2) distance from the lower end.
= (1 kg) (9.8 ms–2) + ____________ ​    
 ​ 
= 30 N
0.2 m Tension at the location of segment is
T/A TL
We have, Y = _____
​  ​  or,  D L = ___
    ​    ​
D L/L
30 N × (20 cm)
AY

M
(  )
T = ​ __
​   ​   ​ xg
L
= ​ _________________________
         ​

(  )
(3 × 10–6 m2) × (2 × 1011Nm–2) T M xg
Stress = __
​   ​ = ​ __
​   ​   ​ ​ ___ ​ 
= 5 × 10 –5 –3
× 20 cm = 10  cm A L A

Let increase in length of the element, be dy.


Example 15   Find the depth of lake at which density of
water is 1% greater than at the surface. Given compressibility dy
Strain in element is = ___
​    ​
of water is K = 50 × 10–6atm–1. dx
M xg
__
​   ​  ___
​   ​ 
Solution Stress _____ L A
Young’s modulus of elasticity Y = _____
​   
 ​ = ​    

Concepts Strain dy/dx
At high pressure, volume decreases and density increases.
Definition of Bulk modulus (or compressibility) will give
M xg
fi  ​ ___
​    ​  ​ ___
YL A
​   ​  dx = dy (  )
us change in volume when pressure is increased.
Extension in the entire wire is the summation of extension
For calculation of pressure, we can assume the density
in each such element.
of water to remain constant as the variation is not much. D l L
Mg MgL
Total extension in the rope, Ú​   ​  ​ ​dy = ____
​    ​ Ú​  ​  ​xdx = ____
​   ​ 
D P D V D P YLA 0 2AY
B = ______
​  ​  fi ​ ___   
    ​ = – ​ ___   
​  0
– D V/V V B
Note:  Since the stress is varying linearly, we may take
We know P = Patm + hrg the average stress as
Mass of a volume V of water is Average stress = 1/2 (Stress at top + stress at bottom).
m = rV = constant Strain = Average Stress/Y
Worksheet 1
8. The length of a metal wire is l1 when the tension in
1. Two wires A and B are made of same material. The it is T1 and is l2 when the tension is T2. The natural
wire A has a length l and diameter r while the wire length of the wire is
B has a length 2l and diameter r/2. If the two wires l1 + l2 ___
are stretched so as to produce the same strain in them, (a) ​ _____
 ​  
  (b) ​÷l 1l2 ​ 
2
the ratio of forces applied to A and B is
l1T2 – l2T1 l1T2 + l2T1
(a) 1/8 (b) 1/4 (c) ​ _________ ​

  (d) ​ _________ ​ 

(c) 4 (d) 8 T2 – T1 T2 + T1
2. A heavy, uniform rod is hanging vertically from a 9. A wire suspended vertically from one of its ends is
fixed support. It is stretched by its own weight. The stretched by attaching a weight of 200 N to the lower
diameter of the rod is end. The weight stretches the wire by 1 mm. Then,
(a) smallest at the top and gradually increases down the elastic energy stored in the wire is
the rod (a) 0.2 J (b) 10 J
(b) largest at the top and gradually decreases down (c) 20 J (d) 0.1 J
the rod 10. In determination of young’s modulus of elasticity of
(c) uniform everywhere wire, a force is applied and extension is recorded.
(d) maximum in the middle Initial length of wire is 1 m. The curve between
extension and stress is depicted. Then Young’s
3. The breaking stress of a wire depends on the modulus of wire will be
(a) material of the wire
(b) length of the wire
(c) radius of the wire
(d) shape of the cross-section
4. A rope, 1 cm in diameter, breaks if the tension in it
exceeds 500 N. The maximum tension that may be
given to a similar rope of diameter 2 cm is
(a) 500 N (b) 250 N
(c) 1,000 N (d) 2,000 N (a) 2 × 109 Nm–2 (b) 1 × 109 Nm–2
5. A wire can sustain a weight of 20 kg before breaking. 10
(c) 2 × 10 Nm –2
(d) 1 × 1010 Nm–2
If the wire is cut into two equal parts, each part can
11. Overall changes in volume and radii of a uni-
sustain a weight of
form cylindrical steel wire are 0.2% and 0.002%
(a) 10 kg (b) 20 kg respectively, when subjected to some suitable force.
(c) 40 kg (d) 80 kg Longitudinal tensile stress acting on the wire is
6. A wire elongates by 1.0 mm when a load W is (Y = 2.0 × 1011 Nm–2)
hanged from it. If this wire goes over a pulley and (a) 3.2 × 109 Nm–2
two weights W each are hung at the two ends, the
(b) 3.2 × 107 Nm–2
elongation of the wire will be
(c) 3.6 × 109 Nm–2
(a) 0.5 m (b) 1.0 m
(d) 4.08 × 108  Nm–2
(c) 2.0 mm (d) 4.0 mm
7. A heavy mass is attached to a thin wire and is 12. A cylindrical wire of radius 1 mm, length 1 m,
whirled in a vertical circle. The wire is most likely Young’s modulus = 2 × 1011 Nm–2, poisson’s ratio
to break m = p/10 is stretched by a force of 100 N. Its radius
will become
(a) when the mass is at the highest point
(a) 0.99998 mm (b) 0.99999 mm
(b) when the mass is at the lowest point
(c) when the wire is horizontal (c) 0.99997 mm (d) 0.99995 mm
(d) at an angle of cos–1 (1/3) from the upward 13. A uniform rod is rotating in gravity-free region with
vertical certain constant angular velocity. The variation of

12.12
tensile stress with distance x from axis of rotation is
best represented by which of the following graphs?
(d)

(a)

14. The load versus elongation graph for four wires of


same length and made of the same material is shown
in the figure. The thickest wire is represented by
the line
(b)

(c) (a) OB (b) OA


(c) OD (d) OC

12.13
Worksheet 2
1. A composite rod consists of a steel rod of length 3. Four rods A, B, C, D of __same__length and material
25 cm and area 2A and a copper rod of length 50 cm but of different radii r, r ​÷2 ​     and 2r respectively
  ,  r ​÷3 ​
and area A. The composite rod is subjected to an axial are held between two rigid walls. The rods just fit
load F. if the Young’s modulus of steel and copper between the walls without any stress in them.The two
are in the ratio 2 : 1. walls are brought slightly closer. If the rods do not
(a) the extension produced in copper rod will be bend, then
more. (a) the stress in the rods are in the ratio 1 : 2 : 3 : 
(b) the extension in copper and steel parts will be 4.
in the ratio 2 : 1. (b) the force on the rod exerted by the wall are in
(c) the stress applied to the copper rod will be the ratio 1 : 2 : 3 : 4.
more. (c) the energy stored in the rods due to elasticity
(d) no extension will be produced in the steel are in the ratio 1 : 2 : 3 : 4.
rod. (d) the strains produced in the rods are in the ratio
2. The wires A and B, shown in the figure, 1 : 2 : 3 : 4.
are made of the same material and have 4. A body of mass M is attached to the lower end of a
radii rA and rB respectively. The block metal wire, whose upper end is fixed. The elongation
between them has a mass m. When the of the wire is l.
force F is mg/3, one of the wires breaks. (a) Loss in gravitational potential energy of M is
(a) A breaks if rA = rB Mg l
(b) A breaks if rA < 2rB (b) The elastic potential energy stored in the wire
(c) It is difficult to predict which wire will break is Mg l
if rA = 2rB (c) The elastic potential energy stored in the wire
(d) The lengths of A and B must be known to is 1/2 Mg l
predict which wire will break (d) Heat produced is 1/2 Mg l.

12.14
Worksheet 3
1. A person’s femur (a long bone) has a diameter of 3.0 7. Wires P and Q are made of same material
cm and a hollow centre of 0.8 cm diameter. Length of and have cross-sectional radii of r1 and r2
the bone is 50 cm. It is supporting a load of 600 N. respectively. Two blocks and B are sus-
How much will it be shortened by this load? Young’s pended from the wires as shown. Masses
modulus of the bone is Y = 1.6 × 1010 Nm–2. of A and B are 3m and m respectively. It is
2. A catapult consists of two parallel rubber strings, observed that one of the two wires break.
each of length 10 cm and cross-sectional area 1 mm2. Find relation between r1 and r2 if
It is stretched by 6 cm and then released to project (a) Wire P breaks  (b) wire Q breaks.
a stone of mass 5.0 g. Assume that the entire elastic
energy is used to impart kinetic energy to the stone 8. A cylindrical pillar is made of two materials. The
and find the speed of projection of the stone. Young’s inner core, having radius r, has Young’s modulus
modulus for rubber is Y = 5 × 108 Nm–2. Y1 and the outer layer (between radius r and R) has
Young’s modulus Y2. A load of weight W is placed
3. A wire of length 1m has a circular cross section of on the cylinder. Find the fraction of load supported
radius 1 mm. When subjected to a load it stretches by the outer layer.
by x. The wire is melted and is then drawn into a
wire of square cross section having side length 2 mm.
Find the extension is the new wire when same load
is applied.
4. A wire has length L and cross-sectional area A. It is
suspended at one of its ends from a ceiling. Density
and Young’s modulus of the material of the wire are
r and Y respectively. Find the elastic strain energy
in the wire due to its own weight
5. A uniform block of mass M is suspended on three
vertical wires of equal lengths
arranged symmetrically, as
shown in figure. Middle wire 9. The breaking stress of aluminium is 7.5 × 108 dyne
is of steel and the other two cm–2. Find the greatest length of aluminium wire
wires are of copper. All wires that can hang vertically without breaking. Density of
have the same cross-section. aluminium is 2.7 g cm–3. [Given: g = 980 cm s–2]
Young’s modulus of steel is
10. A thin, uniform metallic rod of length 0.5 m and radius
twice that of copper. Find tension in steel wire.
0.1 m rotates with an angular velocity 400 rad s–1 in a
6. A light rod of length L = 2 m is suspended from horizontal plane about a vertical axis passing through
a ceiling by means of two vertical wires of same one of its ends. Calculate
length tied to the ends
(a) tension in the rod at a distance r from the rota-
of the rod. One of the
tion axis and
wire is made of steel
and has cross-sectional (b) the elongation of the rod.
area A. The other wire The density of material of the rod is 104 kgm–3 and
is made of brass and has the Young’s modulus is 2 × 1011 Nm–2.
cross-sectional area 2A. 11. A block of mass m and length l has cross-sectional
A weight is hung from area A. It is placed on a smooth horizontal surface
the rod at a distance x and pulled by applying a force F at its one end.
from the steel wire end. Young’s modulus of steel is Find out the elongation in block. Young’s modulus
twice that of brass. of block is Y.
(a) Find x such that both wires have equal strain.
(b) Find x such that the both wires have equal
stress.

12.15
Answers Sheet
Your Turn
1. (i) 0.002 (ii) 107 Nm–2 2. 0.0174 3. 1.96 × 107
x
4. ​ __  ​ 5. (a) 1.17 × 10–4 m (b) 1.17 × 10–4 m 6. 9 × 10–5
2
7. 2 × 10–4 8. 8 × 105 Nm–2 9. 7.5 ml 10. 100 m
1
11. ​ __ ​  Y Œ2 12. 0.8 J
2
Worksheet 1
1. (c) 2. (a) 3. (a) 4. (d) 5. (b) 6. (b) 7. (b) 8. (c) 9. (d)
10. (a) 11. (d) 12. (d) 13. (a) 14. (c)

Worksheet 2
1. (a, c) 2. (a, b, c) 3. (b, c) 4. (a, c, d)

Worksheet 3
p 2 r2 g2 AL3
1. 2.9 × 10–5 m 2. 84.9 ms–1 3. ​ ___ ​ x 4. ​ ________
   ​ 
16 6Y
4 r1 r1
5. Mg/2 6. (a) 1m (b) __
​   ​  m 7. (a) r2 > __
​   ​  (b) r2 < __
​   ​ 
3 2 2


(R2 – r2) Y2
8. ​ _______________
       ​
r2Y1 + (R2 – r2) Y2
9. 2.834 km
1
10. (a) 8p × 106 ​ __
4 [  ] 1
​   ​  – r2  ​ N (b) __
​   ​  × 10–3 m
3

F
11. ​ ____ 
  
2AY

12.16
Chapter  13

Miscellaneous Problems
based on chapters 11 and 12

Match the column 3. A steel rod of cross-sectional area 16 cm2 and two
identical brass rods of cross-sectional area 10 cm2
1. A star of mass M is a uniform sphere of radius R. each, support a mass of M = 5,000 kg, as shown. The
It is spinning about its axis with angular speed w0. steel rod is at the centre and the other two rods are
It begins to contract while maintaining its spheri- located symmetrically, bearing equal load. Given:
cal shape. The effect of contraction on quantities
in column I has been given in column II. Match
them.

Column I Column II
(a) Acceleration due to gravity at (p) Increases
poles
(b) Free fall acceleration at poles (q) Decreses

(c) Free fall acceleration at (r) Remains YS = 2 × 106 kgcm–2  and  YB = 106 kgcm–2
equator unchanged
(d) Binding energy of a particle (s) May increase Match entries in column I with those in column II
on pole or decreases
Column I Column II
2. Two identical particles are pro- (a) Ratio of stress in steel to that in brass (p) 1
jected from the surface of the Earth is close to
with same speed. Particle 1 is pro- 4
(b) Ratio of strain in steel to that in brass (q) __
​   ​ 
jected vertically up and Particle 2 is 3 
is projected tangentially. Neglect 2
(c) Ratio of compressive force in steel to (r) ​ __ ​ 
atmospheric resistance. 3
that in one brass rod is
32
(d) Ratio of change in length of steel to (s) ___
Column I Column II ​   ​ 
that in brass is 15
(a) Particles stop momentarily (p) particle 1
before falling back to the 4. In this problem, M refers to the mass of the Earth
ground and R is its radius. Match the items in column I to
(b) Attains more height (q) particle 2 the items in column II given below:
(c) Has higher angular momen- (r) Both 1 and 2
Column I Column II
tum about the centre of the _____

÷ 
earth (a) Escape velocity from a height R (p) ____
4GM
​ ​   ​ ​
  

above earth’s surface R
(d) May hit the surface of the (s) Neither 1 or 2
Earth normally.
13.2  Mechanics II

____

÷ 
(b) Minimum horizontal velocity (q) ___ path AB is W1 and work done in moving the same particle
GM
required for a particle at a height R ​ ​     
​ ​   from A to B along the path ACB is W2.
R
above earth’s surface to avoid falling 3. The differences in gravitational potential at A and B
on earth due to the sphere can be expressed as
____

÷  ( 
(c) Escape velocity on the surface of (r)
)
GM
​ ___
​    ​ ​  W1 + W2 W1 + W2
earth if earth’s diameter were shrunk 3R (a) – ​ ​ ________
   ​  
​ (b) ​ ________
   ​ 
to half of its present value 2m 2m
____

÷ 
(d) Velocity of a body thrown from the (s) GM W1 2 (W1 + W2)
​ ___
​    ​ ​  (c) ​ ___ (d) ​ __________
​ R​  
earth’s surface to reach a height of __ 2R m ​   m      ​
3
from the surface of the earth 4. The particle is slowly transferred from A to D along
the straight line AOD (assume a smooth tunnel
through the sphere). Take the direction of force (F)
PASSAGE-BASED PROBLEMS applied by the external agent on m to be positive,
​___›
Passage 1 when it is directed along OA​
​   . Which of the following
is the best representation of F versus r, where r is
Saturn’s ring has aroused curiosity since christiaan Huygens
distance from O. r is taken to be positive from O to
observed it in 1655 using a self-designed telescope. In 1859,
A and negative from O to D.
James clerk Maxwell demonstrated that a non-uniform solid
ring would not be stable and the ring must be composed of
numerous small particles, all independently orbiting Saturn.
The dense main ring extends from 8,000 km to 80,000 km (a) (b)
away from saturn’s equator, whose radius is nearly 60,000
km. The ring is composed of 99.9% water and ice with esti-
mated thickness of 10 m to 1 km. The ring contains particles
of radius ranging from 1 cm to 10 m. However, the total mass
of the ring has been estimated to be very small compared
to the mass of Saturn. (c) (d)
Acceleration due to gravitational pull of Saturn on its
surface at equator is 10 ms–2. Density of ice  Density of
water = 103 k gm–3.
Passage 3
1. Estimate the maximum KE of a particle in the ring
of Saturn A meteorite of mass m collides with a satellite, which was
(a) 1015 J (b) 108 J orbiting around a planet in a circular orbit of radius R. The
meteorite sticks to the satellite, which has mass 10 m. The
(c) 104 J (d) 1022 J
satellite now orbits the planet in an elliptical orbit. Mass of
2. The ratio of smallest time period of revolution of a the planet is M and the meteorite was moving towards the
particle around Saturn to the largest time period of centre of the earth before collision.
revolution of a particle around Saturn is nearly 5. What could be the maximum speed of the
(a) 0.8 (b) 0.1 meteorite?

÷  ÷ 
_______ _____
(c) 0.7 (d) 0.35 142 GM
_______ 3 GM
(a) ​ ​     ​ ​  
  (b) ​ _____
​     ​ ​ 

R R
Passage 2
÷​   R    ​ ​ ÷​   R   ​ ​ 
______ ____
63 GM GM
Consider a fixed uniform massive sphere of mass M and (c) ​ ______   (d) ​ ____  

radius R. O is the cen-


6. If the satellite has a minimum distance from the
tre of the sphere and R
AD is a straight line centre of the planet = ​ __ ​  after collision, the speed of
2
passing through O. OA meteorite before collision was:

÷  ÷ 
= OD. ABC is an equi- ____ ______
GM 58 GM
lateral triangle having (a) ​ ____
​     
​ ​    (b) ​ ______
​      
​ ​ 
R R
side length equal to R.

÷​   R   ​ ​   ÷​   R   ​ ​ 


_____ _____
Work done in slowly 3 GM 2GM
(c) ​ _____   (d) ​ _____  
moving a particle of mass m from A to B, along straight
Miscellaneous Problems based on chapters 11 and 12  13.3 

Passage 4 Passage 7
Two stars rotate about their common centre of mass as a Three wires AP, BP and CP are
binary system. Both move in circular orbits and their time made of same materials and have
period is 1 year. Mass of one star is double that of the other same cross sectional area. They are
and the lighter star has mass equal to one-third of the mass fixed to points A, B and C on a
of our sun. Distance between the Earth and the sun is R and ceiling and their free ends are knot-
mass of the sun is M. ted together at P. Lengths of the
7. Distance between the two stars is three wires are = CP = l2 and
(a) R (b) 1.2 R BP = l1. A weight W is suspended
at P and the wires get taut. Assume
(c) 1.5 R (d) 3R
the deformations to be small.
8. Kinetic energy of the smaller star is 13. Which of the following is true?
GM 2GM
(a) ​ ____   ​   (b) ​ _____
   ​  (a) Extension in wire AP is higher than extension
R R
in BP.
2 GM GM
(c) ​ __ ​  ​ ____ ​  
   (d) ​ ____ ​  (b) Extension in wire BP is more than extension in
9 R 9R
AP.
Passage 5
(c) Stress in wire AP is more than stress in BP.
Two satellites S1 and S2 revolve round a planet in co-planar (d) None of the above.
circular orbits in the same sense. There periods of revolu-
14. Tension in wire BP is
tion are 1 h and 8 h respectively. The radius of orbit of S1
is 104 km. At an instant, the two satellites are nearest to W W
(a) ​ _________ (b) ​ ________
(  ) (  )
  3   ​   2   ​
each other. Answer the following questions with regard to l1
__ l1
__
this instant. 1 + 2 ​​ ​   ​   ​​ ​ 1 + ​​ ​   ​   ​​ ​
l2 l2
9. Speed of S2 relative to S1 is
W W
(c) ​ _________ (d) ​ ________
(  ) (  )
(a) 104 kmh–1 (b) 2 × 104 kmh–1   2   ​   2   ​
l2 l2
(c) p × 104 kmh–1 (d) 2p × 104 kmh–1 1 + 2 ​​ ​ __ ​   ​​ ​ 1 + ​​ ​ __ ​   ​​ ​
l1 l1
10. Angular speed of S2 as observed by an astronaut is
S1 is Passage 8
2p
(a) p rad h–1 (b) ​ ___ ​ rad h–1 Consider a star and two planet system. The star has mass M.
3
The planets A and B have the same mass m, radius a and they
p p
(c) ​ __ ​  rad s
–1
(d) ​ __ ​  rad h–1. revolve around the star in circular orbits of radius r and 4r
3 3
respectively (M >> m, r >> a). Planet A has intelligent people
Passage 6 living on it and they have achieved a very high degree of
A body is released from a height R above the north pole of technological advancement. They wish to shift a geostation-
the Earth. Another identical body is released from a height ary satellite of their own planet to a geostationary orbit of
R above a city at latitude 30° N. There is no resistance to planet B. They achieve
motion of the bodies and consider no rotation of the Earth this through a series of
[R is radius of the Earth and M is mass of the Earth] high-precision maneu-
11. Relative speed of the two bodies at the instant they vers in which the satel-
R lite never has to apply
have travelled a distance __
​   ​  each, is
2 _____ brakes and not a single

÷  ÷ 
____
GM 2GM joule of energy is wasted.
(a) ​ ​ ____
  ​ ​    (b) ​ ​ _____   ​ ​  
R R S1 is a geostationary sat-

÷​   R    ​ ​
_____ ellite of planet A and S2
3GM
(c) ​ _____   (d) zero is a geostationary satel-
lite of planet B. Neglect
12. Distance between the two bodies at the instant they
R interactions between A
have travelled a distance __
​   ​  is and B.
2
3R R 15. If the time period of the satellite in geostationary orbit
(a) ​ ___ ​   (b) ​ __ ​ 
2 2 of planet A is T, then its time period in geostationary
(c) 2R (d) zero orbit of planet B is:
13.4  Mechanics II

(a) T (b) 4 T (a) I (b) II


(c) 8 T (d) Data insufficient (c) both I and II (d) neither I nor II.
16. If the radius of the geostationary orbit in planet A is 17. If planet A and B, both complete one revolution about
(  )
given by rG = r ​​ __
m 1/3
​   ​   ​​ ​, then the time in which the
M
their own axis in the same time, then the energy
needed to transfer the satellite of mass m0 from planet
geostationary satellite will complete one revolution A to planet B is
is
Gm m0 GM m0
I. 1 planet year = time in which planet revolves (a) ​ ______
 ​  
  (b) ​ ______
   
​ 
4r 4r
around the star
3GM m0
II. 1 planet day = time in which planet revolves (c) ​ _______
   
​   (d) Zero
about its axis 8r
Answers Sheet
Match the Columns
1. (a) p  (b) p  (c) q  (d) p 2. (a) p   (b) p   (c) q   (d) p
3. (a) q  (b) r   (c) s   (d) p 4. (a) q   (b) r   (c) p   (d) s

Passage-based Problems
1. (a) 2. (d) 3. (a) 4. (a) 5. (a) 6. (b) 7. (a) 8. (c) 9. (c)
10. (d) 11. (b) 12. (a) 13. (b) 14. (a) 15. (d) 16. (c) 17. (c)

13.5
Chapter  14

Past Years’ Questions

 CENTRE OF MASS AND MOMENTUM what distance should the second particle be moved, so
as to keep the centre of mass at the same position?
AIEEE/JEE Main Questions [AIEEE 2006]
m2 m1
1. A mass ‘m’ moves with a velocity ‘v’ and collides (a) ​ ___
m1 ​  d (b) ​ _______
    ​ d
m1 + m2
inelastically with another identical mass at rest. After
__ m1
collision, the first mass moves with velocity v/​÷3 ​    (c) ​ ___
m2 ​  d (d) d
in a direction perpendicular to the initial direction
of motion. Find the speed of the second mass after 5. A circular disc of radius R is removed from a bigger
collision. [aieee 2005] circular disc of radius 2R, such that the circumferences
__ __
   ) v
(a) (2/​÷3 ​ (b) v/​÷3 ​   of the discs coincide. The centre of mass of the new
__ disc is at a distance aR from the centre of the bigger
(c) v (d) ​÷ 3 ​  v disc. The value of a is [AIEEE 2007]
2. The block of mass M, moving on the frictionless (a) 1/3 (b) 1/2
horizontal surface, collides with
a spring of spring constant k (c) 1/6 (d) 1/4
and compresses it by length L. 6. A body of mass m = 3.513 kg is moving along the
The maximum momentum of the x-axis, with a speed of 5.00 ms–1. The magnitude of
block after collision is its momentum is recorded as [AIEEE 2008]
[aieee 2005] (a) 17.6 kg ms–1 (b) 17.565 kg ms–1
(c) 17.56 kg ms–1 (d) 17.57 kg ms–1
ML2
(a) Zero (b) ​ ____
  
​  7. A thin rod of length ‘L’ is lying along the x-axis,
k
___ with its ends at x = 0 and x = L. Its linear density
kL2
(c) ​÷ Mk ​L
  (d) ​ ___  
​ (mass/length) varies with x as k (x/L)n, where n can
2M
be zero or any positive number. If the position xCM
3. A bomb of mass 16 kg, at rest, explodes into two of the centre of mass of the rod is plotted against ‘n’,
pieces of masses 4 kg and 12 kg. The velocity of the which of the following graphs best approximates the
12 kg mass is 4 ms–1. The kinetic energy of the other dependence of xCM on n? [AIEEE 2008]
mass is [AIEEE 2006]
(a) 144 J (b) 288 J
(c) 192 J (d) 96 J
(a) (b)
4. Consider a two-particle system with particles having
masses m1 and m2. If the first particle is pushed
towards the centre of mass through a distance d, by
14.2  Mechanics II

(a) 0.2 Ns (b) 0.4 Ns


(c) 0.8 Ns (d) 1.6 Ns
(c) (d) 11. Statement-1: Two particles moving in the same
direction do not lose all their energy in a completely
inelastic collision.
Statement-2: Principle of conservation of momentum
8. A block of mass 0.50 kg is moving with a speed of holds true for all kinds of collisions. [AIEEE 2010]
2.00 ms–1 on a smooth surface. It strikes another mass (a) Statement–1 is true, Statement–2 is false
of 1.00 kg and then they move together as a single
body. The energy loss during the collision is (b) Statement–1 is true, Statement–2 is true;
Statement–2 is the correct explanation of
[AIEEE 2008] Statement–1
(a) 0.16 J (b) 1.00 J (c) Statement–1 is true, Statement–2 is true;
(c) 0.67 J (d) 0.34 J Statement–2 is not the correct explanation of
9. Consider a rubber ball freely falling from a height Statement–1
h = 4.9 m onto a horizontal elastic plate. Assume (d) Statement–1 is false, Statement–2 is true
that the duration of collision is negligible and the 12. This question has Statement I and Statement II. Of
collision with the plate is totally elastic. Then the the four choices given after the statements, choose
velocity as a function of time and the height as a the one that best describes the two statements.
function of time will be [AIEEE 2009]
Statement-1: A point particle of mass m moving
with speed u collides with a stationary point-particle
of mass M. If the maximum energy loss possible is
(a)
( 
1
given as f ​  ​ _____
2 mu2
  
) (  m
)
 ​  ​, then ƒ = ​ ______
​     
M+m
 ​  ​.

[JEE-Main 2013]
Statement-2: Maximum energy loss occurs when
the particles get stuck together as a result of the
collision.
(b)
(a) Statement-1 is true, Statment-2 is true,
Statement-2 is the correct explanation of
Statement-1.
(b) Statement-1 is true, Statment-2 is true,
Statement-2 is not the correct explanation of
(c) Statement-1.
(c) Statement-1 is true, Statment-2 is false.
(d) Statement-1 is false, Statment-2 is true.
13. A particle of mass m moving in the x direction with
speed 2v is hit by another particle of mass 2m moving
(d)
in the y-direction with speed v. If the collision is
perfectly inelastic, the percentage loss in the energy
during the collision is close to [JEE Main 2015]
10. The figure shows the position–time (x-t) graph of the (a) 62% (b) 44%
one-dimensional motion of a body of mass 0.4 kg. (c) 50% (d) 56%
The magnitude of each impulse is [AIEEE 2010]
14. Distance of the centre of mass of a solid uniform
cone from its vertex is z0. If the radius of its base is
R and its height is h then z0 is equal to:
[JEE Main 2015]
3h2 h2
(a) ​ ___ ​   (b) ​ ___  ​ 
8R 4R
3h 5h
(c) ​ ___ ​   (d) ​ ___ ​ 
4 8
Past Years’ Questions  14.3 

15. In a collinear collision, a particle with an initial speed


v0 strikes a stationary particle of the same mass. If
the final total kinetic energy is 50% greater than the
original kinetic energy, the magnitude of the relative
velocity between the two particles, after collision,
is: [JEE Main 2018]
v0
___ v0
__
(a) ​  __  ​   (b) ​   ​ 
÷   
​ 2 ​ 4
__ v__0
(c) ​÷ 2 ​  v0 (d) ​   ​ 
2
IIT JEE (Advanced) Questions
1. Statement-1: In an elastic collision between two
bodies, the relative speed of the bodies after collision (i) The speed of the block at point B immediately after
is equal to the relative speed before the collision. it strikes the second incline is
___ ___
[2007] (a) ​÷ 60 ​ ms–1 (b) ​÷ 45 ​ ms–1
because ___ ___
(c) ​÷ 30 ​ ms–1 (d) ​÷ 15 ​ ms–1
Statement-2: In an elastic collision, the linear
momentum of the system is conserved (ii) The speed of the block at point C, immediately
before it leaves the second incline is
(a) Statement-1 is true, Statement-2 is true; ____ ____
Statement-2 is a correct explanation for (a) ​÷ 120 ​ ms–1 (b) ​÷ 105 ​ ms–1
___ ___
Statement-1 (c) ​÷ 90 ​ ms–1 (d) ​÷ 75 ​ ms–1
(b) Statement-1 is true, Statement-2 is true; (iii) If collision between the block and the incline is
Statement-2 is NOT a correct explanation for completely elastic, then the vertical (upward)
Statement-1 component of the velocity of the block at point B,
(c) Statement-1 is true, Statement-2 is false immediately after it strikes the second incline is
___ ___
(d) Statement-1 is false, Statement-2 is true. (a) ​÷ 30 ​ ms–1 (b) ​÷ 15 ​ ms–1
(e) Statement-1 is false, Statement-2 is false ___
_
​› _
​› (c) 0    ms–1
(d) – ​÷15 ​
2. Two balls, having linear momenta ​p  1​  = p  and ​p  2​  = – p  ,
undergo a collision in free space. There is no external 4. If the resultant of all the external forces acting on
​_› ​_› a system of particles is zero, then from an inertial
force acting on the balls. Let ​p  1 ​¢ and ​p   ​¢2 be their final
momenta. Which of the following option(s) is(are) frame, one can surely say that: [2009]
NOT ALLOWED for any non-zero value of p, a1, (a) Linear momentum of the system does not
a2, b1, b2, c1 and c2. [2008] change in time.
​_› ​_›
(b) Kinetic energy of the system does not change
(a) ​p  1​¢  = a1  + b1  + c1  (b) ​p  ​¢ 1 = c1  in time.
​_› ​_› (c) Angular momentum of the system does not
​p  2​¢  = a2  + b2  ​p  ​¢ 2 = c2 
change in time.
​_› ​_› (d) Potential energy of the system does not change
(c) ​p  1​¢  = a1  + b1  + c1  (d) ​p  ​¢ 1 = a1  + b1 
in time.
​_› ​_›
​p  1​¢  = a1  + b1  – c1  ​p  ​¢ 2 = a2  + b1  5. Two small particles of equal masses start moving in
opposite directions from a point
3. Following three questions are based on the given A, in a horizontal circular orbit.
passage Their tangential velocities are v
A small block of mass M moves on a frictionless and 2v respectively, as shown in
surface of an inclined plane, as shown in figure. The the figure. Between collisions,
angle of the incline suddenly changes from 60° to 30° the particles move with constant
at point B. The block is initially at rest at A. Assume speeds. After making how many
that collision between the block and the incline is elastic collisions, other than that at A, will these two
totally inelastic (g = 10 ms–2) [2008] particles again reach point A? [2009]
(a) 4 (b) 3
(c) 2 (d) 1
14.4  Mechanics II

6. Look at the drawing given in the figure, which has 10. A ball of mass 0.2 kg rests on a vertical post of
been drawn with ink of uniform line thickness. The height 5m. A bullet of mass 0.01 kg, travelling with
mass of ink used to draw each of a velocity v ms–1 in a horizontal direction, hits the
the two inner circles, and each of centre of the ball. After the collision, the ball and the
the two line segments is m. The bullet travel independently. The ball hits the ground
mass of the ink used to draw the at a distance of 20 m and the bullet at a distance of
outer circle is 6m. The co-ordinates 100 m from the foot of the post. The initial velocity
of the centres of the different parts v of the bullet is [2011]
are: outer circle (0, 0); left inner circle (– a, a); right
inner circle (a, a); vertical line (0, 0); and horizontal
line (0, – a). The y-co-ordinate of the centre of mass
of the ink in this drawing is [2009]
(a) a/10 (b) a/8
(c) a/12 (d) a/3
7. Three objects, A, B and C, are kept in a straight
line on a frictionless horizontal surface. These have
masses m, 2m and m, respectively. The object A
moves towards B with a speed 9 ms–1 and makes an __
elastic collision with it. Thereafter, B makes com- (a) 250 ms–1    ms–1
(b) 250 ​÷2 ​ 
pletely inelastic collision with C. All motions occur (c) 400 ms–1 (d) 500 ms–1
on the same straight line. Find the final speed (in 11. A bob of mass m, suspended by a string of length l1,
ms–1) of the object C. [2010] is given a minimum velocity required to complete a
8. A block of mass 2 kg is free to move along the x-axis. full circle in the vertical plane. At the highest point,
It is at rest and from t = 0 onwards, it is subjected it collides elastically with another bob of mass m
to a time-dependent force F (t) in the x-direction. The suspended by a string of length l2, which is initially at
force F (t) varies with t, as shown in the figure. The rest. Both the strings are mass-less and inextensible. If
kinetic energy of the block after 4.5 seconds is the second bob, after collision, acquires the minimum
[2011] speed required to complete a full circle in the vertical
l1
plane, find the ratio ​ __ ​ . [2013]
l2
12. Consider regular polygons with number of sides
n = 3, 4, 5 ..., as shown in the figure. The centre
of mass of all the polygons is at height h from the
ground. They roll on a horizontal surface about
the leading vertex without slipping and sliding as
depicted. The maximum increase in height of the
(a) 4.50 J (b) 7.50 J locus of the centre of mass for each polygon is D.
Then D depends on n and h as [2017]
(c) 5.06 J (d) 14.06 J
9. A point mass of 1 kg collides elastically with a
stationary point mass of 5 kg. After their collision,
the 1 kg mass reverses its direction and moves with
a speed of 2 ms–1. Which of the following statement
(s) is (are) correct for the system of these two

( 
masses? [2010]


(a) Total momentum of the system is 3 kg ms–1
(b) Momentum of 5 kg mass after collision is

p
(a) D = h sin2 ​ __ (  )
​ n ​  ​
1
(b) D = h ​ _______
​     ​ 
p
(  )
cos ​ ​ __
n ​ 
  ​ )
– 1  ​


4 kg ms–1
(c) Kinetic energy of the centre of mass is 0.75 J

2p
(  )
(c) D = h sin ​ ___
p
(d) D = h tan2 ​ ___
​  n ​   ​ (  )
​    ​  ​
2n
13. A block of mass M has a circular cut with a friction-
(d) Total kinetic energy of the system is 4 J less surface, as shown. The block rests on the hori-
zontal frictionless surface of a fixed table. Initially,
Past Years’ Questions  14.5 

the right edge of the block is at x = 0, in a co-ordinate 2. A round uniform body of radius , mass M and
system fixed to the table. A point mass m is released moment of inertia I rolls down (without slipping)
from rest at the top-most point of the path, as shown, on an inclined plane making an angle q with the
and it slides down. When the mass loses contact with horizontal. Then its acceleration is: [AIEEE 2007] 
the block, its position is x and the velocity is v. At g sin q g sin q
that instant, which of the following options is/are (a) ​ ________  2

 ​ (b) ​ _________   
 ​
1 – MR /I 1 + I/MR2
correct? [2017]
g sin q g sin q
(c) ​ _________   
 ​ (d) ​ ________   
 ​
1 + MR2/I 1 – I/MR2
3. Angular momentum of the particle rotating with a
central force is constant due to: [AIEEE 2007]
(a) constant torque
(b) constant force
(c) constant linear momentum
(d) zero torque 
(a) The position of the point mass m is 4. Consider a uniform square plate of side ‘a’ and mass
__ mR ‘m’. The moment of inertia of this plate about an axis
    ______
x = – ​÷2 ​ ​      ​
M+m perpendicular to its plane and passing through one of
its corners is: [AIEEE 2008] 
(b) The velocity of the point mass m is

 ÷
(a) 5/6 ma2 (b) 1/12 ma2
______
2gR (c) 7/12 ma2 (d) 2/3 ma2
v = ​ ______
​  m  
   ​ ​
1 + __​   ​  5. A thin horizontal circular disc is rotating about a
M vertical axis passing through its centre. An insect is
(c) The x component of displacement of the centre at rest at a point near the rim of the disc. The insect
mR now moves along diameter of the disc to reach its
of mass of the block M is – ​ ______
    ​
M+m other end. During the journey of the insect, the angu-
m ____ lar speed of the disc [AIEEE 2011]
(d) The velocity of the block M is V = – ​ __ ​ ​÷2gR 
  ​  (a) continuously increases
M
14. A solid horizontal surface is covered with a thin (b) first increases and then decreases
layer of oil. A rectangular block of mass m = 0.4 (c) remains unchanged
kg is at rest on this surface. An impulse of 1.0 Ns (d) continuously decreases
is applied to the block at time t = 0 so that it starts 6. A pulley of radius 2m is rotated about its axis by a
moving along the x-axis with a velocity v(t) = v0e– t/t, force F = (20 t – 5 t2) N (where t is measured in sec-
where v0 is a constant and t = 4 s. The displacement onds) applied tangentially. If the moment of inertia
of the block, in meters, at t = t is _________. Take of the pulley about its axis of rotation is 10 kg m2,
e–1 = 0.37. [JEE Adv. 2018] the number of rotations made by the pulley before
its direction of motion is reversed, is
 ROTATIONAL MOTION [AIEEE 2011]
(a) more than 6 but less than 9
AIEEE/JEE Main Questions (b) more than 9
1. An annular ring with inner and outer radii R1 and R2 (c) less than 3
is rolling without slipping, with a uniform angular (d) more than 3 but less than 6
speed. The ratio of the forces experienced by the two 7. A mass m hangs with the help of a string wrapped
particles situated on the inner and outer parts of the around a pulley on a frictionless bearing. The
ring, F1/F2 is: [AIEEE 2005] pulley has mass m and radius R. Assuming pulley
(a) 1 (b) R1/R2 to be a perfect uniform circular disc, the accelera-
tion of the mass m, if the string does not slip on the
(c) R2/R1 (d) (R1/R2)2 pulley, is: [AIEEE 2011]
14.6  Mechanics II

(a) (2/3) g (b) g/3 4MR 2


MR__2
(a) ​ _____
__  ​  (b) ​ ______   ​ 
(c) (3/2) g (d) g    p
3​÷3 ​    p
32​÷2 ​
8. A hoop of radius R and mass m, rotating with an MR__2 4MR 2
angular velocity w0, is placed on a rough horizontal (c) ​ ______   ​   (d) ​ _____
__  ​ 
   p
16​÷2 ​    p
9​÷3 ​
surface. The initial velocity of the centre of the hoop
is zero. What will be the velocity of the centre of the 12. A particle of mass m is moving along the side of a
hoop when it ceases to slip? [JEE-Main 2013] square of side ‘a’, with a uniform speed v in the x – y
plane, as shown in the figure Which of the following
​_›
Rw0 Rw0 statements is false for the angular momentum ​L   ​about
(a) ​ ____
 ​  
  (b) ​ ____
 ​ 

3 2 the origin? [JEE-Main 2016]
Rw0
(c) Rw0 (d) ​ ____
 ​ 

4
9. A mass m is supported by a massless string wound
around a uniform hollow cylinder of mass m and
radius R. If the string does not slip on the cylinder,
with what acceleration will the mass fall on
release? [JEE-Main 2014]
​_›
mv
(a) ​L  ​  = – ​ ___
__ ​ R  when the particle is moving from
÷   
​ 2 ​
A to B.

[  ]
​_›
R
(b) ​L  ​  = mv ​ ___
​  __  ​ – a  ​  when the particle is moving
÷   
​ 2 ​
from C to D

[  ]
_
​›
R
(c) ​L  ​  = mv ​ ___ ​  __  ​ + a  ​  when the particle is moving
÷   
​ 2 ​
from B to C
​_›
mv
(d) ​L   ​ = ​ ___
__ ​ R  when the particle is moving from
÷   
​ 2 ​
2g g D to A
(a) ​ ___ ​   (b) ​ __ ​ 
3 2
13. A roller is made by joining two cones at their ver-
5g tices, O. It is kept on two rails, AB and CD,which
(c) ​ ___ ​   (d) g
6 are placed asymmet-
10. A bob of mass m, attached to an inextensible string rically (see figure),
of length l, is suspended from a vertical support. The with its axis perpen-
bob rotates in a horizontal circle with an angular dicular to CD and its
speed w rad s–1 about the vertical. About the point centre O at the centre
of suspension: [JEE-Main 2014] of the line joining AB
and CD. It is given
(a) angular momentum is conserved.
a light push so that
(b) angular momentum changes in magnitude but it starts rolling with
not in direction its centre O moving
(c) angular momentum changes in direction but not parallel to CD in the direction shown. As it moves,
in magnitude the roller will tend to: [JEE Main 2016]
(d) angular momentum changes both in direction (a) turn left (b) turn right
and magnitude (c) go straight
(d) turn left and right alternately
11. From a solid sphere of mass M and radius R, a
14. The moment of inertia of a uniform cylinder of length
cube of maximum possible volume is cut. Moment
l and radius R about its perpendicular bisector is I.
of inertia of cube about an axis passing through its
What is the ratio l/R such that the moment of inertia
centre and perpendicular to one of its faces is:
is minimum? [JEE-Main 2017]
[JEE-Main 2015]
Past Years’ Questions  14.7 

÷ 
__ __
37

3
(a) ​ __
​    ​ ​  
÷   
​ 3 ​
(b) ​ ___ ​  (a) ​ ___ ​ MR2 (b) 4 MR2
2 2 9
3__ 40
(c) 1 (d) ​ ___  ​  (c) ​ ___ ​ MR2 (d) 10 MR2
÷   
​ 2 ​ 9
15. A slender uniform rod of mass IIT JEE (Advanced) Questions
M and length l is pivoted at
1. A solid cylinder of mass m and radius R rolls down
one end so that it can rotate
an inclined plane of inclination q. Calculate the linear
in a vertical plane (see figure).
acceleration of the axis of cylinder. [2005]
There is negligible friction
at the pivot. The free end is 2. Two identical ladders, each of
held vertically above the pivot mass M and length L are resting
and then released. The angular on a rough horizontal surface, as
acceleration of the rod, when it makes an angle q shown in the figure. A block of
with the vertical is: [JEE-Main 2017] mass m hangs from P. If the sys-
tem is in equilibrium, find the magnitude and the
3g 2g
(a) ​ ___ ​  sin q (b) ​ ___ ​  sin q direction of frictional force at A and B. [2005]
2l 3l
3. If a particle is confined to rotate in a circular path
3g 2g
(c) ​ ___ ​  cos q (d) ​ ___ ​  cos q with decreasing linear speed, then which of the fol-
2l 3l lowing is correct? [2005]
​_›
16. Seven identical circular (a) ​L   ​ (angular momentum) is conserved about the
planar disks, each of centre
​_›
mass M and radius R are (b) Only direction of angular momentum ​L   ​ is
welded symmetrically as conserved
shown. The moment of
(c) It spirals towards the centre.
inertia of the arrange-
ment about the axis (d) Its acceleration is towards the centre.
normal to the plane and 4. From a circular disc of radius R and mass 9M, a small
passing through the point disc of radius R/3 is removed.
P is: [JEE-Main 2018] The moment of inertia of the
181 19 remaining disc about an axis
(a) ​ ____   2
 ​ MR
  (b) ​ ___ ​  MR2 perpendicular to the plane of
2 2
the disc and passing through
55 73
(c) ​ ___ ​ MR
2
(d) ​ ___ ​ MR2 O is: [2005]
2 2
17. From a uniform circular disc of radius R and mass 40
(a) 4 MR2 (b) ​ ___ ​  MR2
R
9M, a small disc of radius __
​   ​  is removed as shown 9
3 37
in the figure. The moment of inertia of the remaining 2
(c) 10 MR (d) ​ ___ ​  MR2
9
disc about an axis perpendicular to the plane of the 5. A block of mass m is held fixed against a wall by
disc and passing through centre of disc is: applying a horizontal force F. Line of action of F
[JEE-Main 2018] passes through the centre of the block. Which of the
following option is incorrect? [2005]

(a) friction force on the block = mg


(b) normal force will not produce a torque on the
block about its centre
(c) F will not produce torque on the block about
its centre
(d) normal reaction on the block = F
14.8  Mechanics II

6. A solid sphere plate. Each ball has mass m = 0.01 kg and n = 100
of radius R has balls hit the plate per second, per square metre of
moment of iner- plate area on the right half of the plate (shown as
tia I about its shaded area). The balls are distributed uniformly
geometrical axis. over the shaded area of the plate and all collisions
It is melted into a are elastic. Find the required speed of the balls just
disc of radius r and before they hit the plate, assuming all of them to be
thickness t. If its moving with the same speed. [2006]
moment of inertia 10. A small object of uniform density rolls up a curved
about the tangential axis (which is perpendicular to surface with an initial velocity v. It reaches up to a
plane of the disc), is also equal to I, then the value 3v2
of r is equal to: maximum height of ​ ___ ​ with respect to the initial
4g
[2006] position. The object is [2007]
2 2__
(a) ​ ____   ​ R
___ (b) ​ ___   ​ R
   
​÷15 ​   
​÷5 ​
__
3 ÷   
​ 3 ​
(c) ​ ____   ​ R
___ (d) ​ ____
___  ​ R
   
​÷15 ​    
​÷15 ​
7. A solid sphere is in pure rolling
motion on an inclined surface hav-
ing inclination q.
[2006] (a) ring (b) solid sphere
(a) frictional force acting on sphere is (c) hollow sphere (d) disc
f = m mg cos q
11. Statement–1: If there is no external torque on a
(b) f is dissipative force body about its centre of mass, then the velocity of
(c) friction will increase its angular velocity and the centre of mass remains constant. [2007]
decreases its linear velocity. Statement–2: The linear momentum of an isolated
(d) If q decreases, friction will decrease system remains constant.
8. A ball moves over a fixed track, as shown in the (a) Statement-1 is true, Statement-2 is true;
figure. From A to B the ball rolls without slipping. If Statement-2 is a correct explanation for
surface BC is frictionless and KA, KB and KC are the Statement-1
kinetic energies of the ball at A, B and C respectively, (b) Statement-1 is true, Statement-2 is true;
then: [2006] Statement-2 is NOT a correct explanation for
Statement-1
(c) Statement-1 is true, Statement-2 is false
(d) Statement-1 is false, Statement-2 is true.
12. Following three questions are based on the given
passage: [2007]
Two discs, A and B, are mounted coaxially on a
(a) hA > hC ; KB > KC (b) hA > hC ; KC > KA vertical axle. The discs have moments of inertia I
and 2I respectively, about the common axis. Disc A
(b) hA = hC ; KB = KC (d) hA < hC ; KB > KC is imparted an initial angular velocity 2w, using the
9. A rectangular plate of mass M = 3 kg of dimen- entire potential energy of a spring compressed by a
sions (a × b = 1 m × distance x1. Disc B is imparted an angular velocity
2 m) is hinged along w by a spring having the same spring constant and
one edge. The plate compressed by a distance x2. Both the discs rotate in
is maintained in hori- the clockwise direction.
zontal position by (i) The ratio of x1/x2 is:
balls moving in verti- (a) 2 (b) 1/2
cally upward direction __ __
(c) ​÷ 2 ​    
(d) 1/​÷2 ​
and colliding with the
Past Years’ Questions  14.9 

​_› ​_› ​_› ​_›


(ii) When disc B is brought in contact with disc A, they (a) ​v  ​ C – ​v  ​ A = 2 (​v  ​ B – ​v  ​ C)
​_› ​_› ​_› ​_›
acquire a common angular velocity in time t. The (b) ​v   ​C – ​v  ​ B = ​v  ​ B – ​v  ​ A

| ​_› ​_› | | ​_› |


average frictional torque on one disc by the other _
​ _
​ _
​ _
​›
during this period is: (c) ​ ​v  ​ C – ​v  ​ A  ​ = 2 ​ ​v  ​ B – ​v  ​ C  ​
(d) ​| ​v  ​ C – ​v  ​ A |​ = 4 ​| ​v  ​ B |​
› › ›
2Iw 9Iw
(a) ​ ____   
​   (b) ​ ____   
​ 
3t 2t 16. A block of base 10 cm × 10 cm and height 15 cm is
9Iw 3Iw kept on an inclined __ plane. The coefficient of friction
(c) ​ ____    ​   (d) ​ ____   ​    .  The inclination q of this inclined
between them is ​÷3 ​
4t 2t
(iii) The loss of kinetic energy during the above process plane from the horizontal plane is gradually increased
is: from 0°. Then [2009]
Iw2 Iw2 (a) at q = 30°, the block will start sliding down the
(a) ​ ___ ​  
  (b) ​ ___ ​ 
  plane
2 3
Iw2 Iw2 (b) the block will remain at rest on the plane up to
(c) ​ ___ ​  
  (d) ​ ___ ​ 
  certain q and then it will topple
4 6
13. Statement-1: Two cylinders, one hollow (metal) (c) at q = 60°, the block will start sliding down the
and the other solid (wood), with the same mass and plane and continue to do so at higher angles
identical dimensions, are simultaneously allowed to (d) at q = 60°, the block will start sliding down the
roll without slipping down an inclined plane from plane and on further increasing q, it will topple
the same height. The hollow cylinder will reach the at certain q
bottom of the inclined plane first. [2008] 17. A boy is pushing a ring of mass 2 kg and radius 0.5 m
and with a stick, as shown in
Statement-2: By the principle of conservation of the figure. The stick applies
energy, the total kinetic energies of both the cylin- a force of 2N on the ring
ders are identical when they reach the bottom of the and rolls it without slip-
incline. ping, with an acceleration
of 0.3 ms–2. The coefficient
(a) Statement-1 is true, Statement-2 is true; of friction between the
Statement-2 is a correct explanation for ground and the ring is large
Statement-1 enough that rolling always occurs and the coefficient
(b) Statement-1 is true, Statement-2 is true; of friction between the stick and the ring is (P/10).
Statement-2 is NOT a correct explanation for The value of P is: [2010]
__
Statement-1 18. Four solid spheres, each of diameter ​÷5 ​    cm and mass
(c) Statement-1 is true, Statement-2 is false 0.5 m are placed with their centres at the corners of
(d) Statement-1 is false, Statement-2 is true. a square of side 4 cm. The moment of inertia of the
14. If the resultant of all the external forces acting on system about the diagonal of the square is N × 10– 4
a system of particles is zero, then from an inertial kg m2, then N is. [2011]
frame, one can surely say that [2009] 19. A lamina is made by
(a) Linear momentum of the system does not removing a small disc of
change in time. diameter 2R from a big-
ger disc of uniform mass
(b) Kinetic energy of the system does not changes
density and radius 2R, as
in time.
shown in the figure. The
(c) Angular momentum of the system does not moment of inertia of this
change in time. lamina about axes passing
(d) Potential energy of the system does not change through O and p is I0 and Ip, respectively. Both these
in time. axes are perpendicular to the plane of the lamina. The
15. A sphere is rolling without slipping on a fixed hori- ratio Ip/IO to the nearest integer is: [2012]
zontal plane surface. In the figure, 20. Two solid cylinders P and Q of same mass and same
A is the point of contact, B is the radius start rolling down a fixed inclined plane, from
centre of the sphere and C is its the same height, at the same time. Cylinder P has
top-most point. Then, [2009] most of its mass concentrated near its surface, while
14.10  Mechanics II

Q has most of its mass concentrated near the axis. end of the rod at t = T and stops. The angular speed
Which statement (s) is (are) correct? [2012] of the system remains
​_›
w throughout. The magnitude
(a) Both cylinders P and Q reach the ground at the of the torque ​( ​| ​t    ​|​ )​ on the system about O, as func-
same time. tion of time, is best represented by which plot?
[2012]
(b) Cylinder P has a larger linear acceleration than
cylinder Q.
(c) Both cylinders reach the ground with the same (a) (b)
translational kinetic energy.
(d) Cylinder Q reaches the ground with larger
speed.
(c) (d)
21. The figure shows a system consisting of (i) a ring of
outer radius 3R rolling clockwise without slipping on
a horizontal surface with angular speed w and (ii) an 23. A small mass m is attached to
inner disc of radius 2R rotating anti-clockwise with a massless string, whose other
angular speed w/2. The ring and disc are separated end is fixed at P, as shown in
by frictionless ball bearings. The system is in the the figure. The mass is under
x–z plane. The point P on the inner disc is at a dis- going circular motion in the
tance R from the origin, where OP makes an angle x–y plane with centre at O
of 30° with the horizontal. Then with respect to the and constant angular speed w.
horizontal surface, [2012] If the angular momentum of
the system, calculated about ​_›
O and​_› P are denoted by ​L    ​0
and ​L    ​P respectively, then [2012]
​_› ​_›
(a) ​_L   ​0 and ​L   ​P do not vary with _
time.
​› ​›
(b) ​_L   ​0 varies with time while ​L   ​P remains constant.
​› ​_›
(c) ​L    ​ and ​L  P ​ both vary with time.
​_›0 ​_›
(d) ​L   ​0 remains constant while ​L   ​P varies with
time.
24. Two identical discs of same radius R are rotating
about their axes in opposite directions with the same
constant angular speed w. The discs are in the same
horizontal plane. At time t = 0, the points P and Q
(a) the point O has a linear velocity 3 Rw  are facing each other, as shown in the figure. The
(b) the point P has a linear velocity relative speed between the two points P and Q is v.
__
11   
​÷3 ​ In one time period (T) of rotaion of the discs, v as
​ ___ ​ R w  + ___
​   ​ R w  a function of time is best represented by [2012]
4 4
(c) the point P has a linear velocity
__
13   
​÷3 ​
​ ___ ​ R w  – ___
​   ​ R w 
4 4
(d) the point P has a linear velocity

( 
__
​ 3 – ___
÷   
​ 3 ​
) 1
​   ​  ​ R w  + __
4
​   ​  R w 
4
22. A thin uniform rod, pivoted at O, is rotating in the
horizontal plane with constant angular speed w, as (a) (b)
shown in the figure. At
time t = 0, a small insect
starts from and moves
with constant speed v with
(c) (d)
respect to the rod towards
the other end. It reaches the
Past Years’ Questions  14.11 

__
25. Following two questions are based on the given (c) It is w for case (a); & ÷     w for case (b)
​ 2 ​
passage (d) It is w for both the cases
The general motion of a rigid body can be considered (ii) Which of the following statements about the instan-
to be a combination of (i) a motion of its centre taneous axis (passing through the centre of mass) is
of mass about an axis, and (ii) its motion about correct?
an instantaneous axis passing through the centre of
(a) It is vertical for both the cases (a) and (b)
mass. These axes need not be stationary. Consider,
for example, a thin uniform disc welded (rigidly (b) It is vertical for case (a). and is at 45° to the
fixed) horizontally at its rim to a massless stick, as x–z plane and lies in the plane of the disc for
shown in the figure. case (b)
When the disc-stick system is rotated about the (c) It is horizontal for case (a); and is at 45° to the
origin on a horizontal frictionless plane with angular x–z plane and is normal to the plane of the disc
speed w, the motion at any instant can be taken for case (b)
as a combination of (i) a rotation of the centre of (d) It is vertical for case (a); and is at 45° to the
mass of the disc about the z-axis, and (ii) a rotation x–z plane and is normal to the plane of the disc
of the disc through an instantaneous vertical axis for case (b)
passing through its centre of mass (as is seen from 26. A uniform circular disc of mass 50 kg and radius
the changed orientation of points P and Q). Both 0.4 m is rotating with an angular velocity of 10 rad s–1
these motions have the same angular speed w in about its own axis, which is vertical. Two uniform
this case. circular rings, each of mass 6.25 kg and radius 0.2 m,
are gently placed symmetrically on the disc in such
a manner that they are touching each other along the
axis of the disc and are horizontal. Assume that the
friction is large enough such that the rings are at rest
relative to the disc and the system rotates about the
original axis. The new angular velocity (in rad s–1) of
Now consider two similar systems, as shown in the system is. [2013]
the figure: Case (a): the disc with its face vertical 27. In the figure, a ladder of mass m is
and parallel to x–z plane; Case (b): the disc with shown leaning against a wall. It is in
its face making an angle of 45° with x–y plane and static equilibrium, making an angle q
its horizontal diameter parallel to x-axis. In both the with the horizontal floor.
cases, the disc is welded at point p, and the systems The coefficient of friction between
are rotated with constant angular speed w about the the wall and the ladder is m1 and that
z-axis. [2012] between the floor and the ladder is m2. The normal
reaction of the wall on the ladder is N1 and that of
the floor is N2. If the ladder is about to slip, then
Case (a) [2014]
mg
(a) m1 = 0 m2 π 0 and N2 tan q = ___ ​   ​ 
2
mg
(b) m1 π 0 m2 = 0 and N1 tan q = ___ ​   ​ 
2
mg
Case (b) (c) m1 π m2 π 0 and N2 = ________
​      ​
1 + m1m2
mg
(d) m1 = 0 m2 π 0 and N1 tan q = ​ ___ ​ 
(i) Which of the following statements regarding the 2
angular speed about the instantaneous axis (passing 28. A horizontal circular platform of radius 0.5 m and
through the centre of mass) is correct? mass 0.45 kg is free to
__
(a) It is ÷    w for both the cases
​ 2 ​  rotate about its axis. Two
w massless spring toy-guns,
(b) It is w for case (a); and ___
​  __  ​ for case (b) each carrying a steel
÷   
​ 2 ​
ball of mass 0.05 kg, are
attached to the platform, at
14.12  Mechanics II

_
​› _
​›
a distance 0.25 m from the centre on its either sides, (a) The velocity ​v  ​  is given by ​v  ​  = (10  + 10  )
along its diameter (see figure). ms–1
​_›
Each gun simultaneously fires the balls horizontally (b) The angular momentum ​_› ​L  ​  with respect to the
and perpendicular to the diameter in opposite direc- origin is given by ​
L   ​ = – (5/3)   N m s
_
​› _
​›
tions. After leaving the platform, the balls have hori- (c) The force ​F   ​ is given by ​F   ​ = ( + 2  ) N
zontal speed of 9 ms–1 with respect to the ground. The ​_›
(d) The​_torque ​t   ​ with respect to the origin is given
rotational speed of the platform, in rad s–1, after the ›
by ​t   ​ = – (20/3) k N m
balls leave the platform is [2014]
33. Two thin circular discs of mass m and 4m, having
29. A uniform circular disc of mass 1.5 kg and radius
radii of a and 2a, respectively, are ___ rigidly fixed by a
0.5 m is initially at rest
massless, rigid rod of length l = ​÷24 ​     a through their
on a horizontal fric-
centres. This assembly is laid on a firm and flat sur-
tionless surface. Three
face and set rolling without slipping on the surface
forces of equal mag-
so that the angular speed about the axis of the rod
nitude F = 0.5 N are
is w. The angular momentum ​_› of the entire assembly
applied simultaneously
about the point ‘O’ is ​L   ​ (see the figure). Which of
along the three sides of
the following statement(s) is (are) true? [2016]
an equilaterial triangle
XYZ with the vertices on the perimeter of the disc
(see figure). One second after applying the forces,
the angular speed of the disc in rad s–1 is [2014]
30. The densities of two solid spheres A and B of the
same radii R, vary with radial distance r, as rA (r)
(  )
R
r
(  )R
r 5
= k ​ ​ __  ​  ​ and rB (r) = k ​​ ​ __  ​  ​​ ​, respectively, where k is
a constant. The moments of inertia of the individual
spheres about axes passing through their centres are
IB n (a) The magnitude of angular momentum of centre
IA and IB, respectively. If ​ __ ​  = ​ ___  ​,  the value of n of mass of the assembly about the point O is
IA 10
is [2015] 81 ma2 w
31. A ring of mass M and radius R is rotating with (b) The centre of mass of the assembly rotates about
angular speed w about a the z-axis with an angular speed of w/5 _
​›
fixed vertical axis passing (c) The magnitude of the z-component of ​L   ​ is
through its centre O with 55 ma2w
two point masses, each
(d) The magnitude of the angular momentum
of mass M/8, at rest at O.
of the assembly about its centre of mass is
These masses can move
17 ma2 w/2
radially outwards along
two massless rods fixed on 34. A uniform wooden stick of mass 1.6 kg and length
the ring, as shown in the l rests in an inclined manner on a smooth, vertical
figure. At some instant, the wall of height h (< l) such that a small portion of the
angular speed of the system is 8/9 w and one of the stick extends beyond the wall. The reaction force of
masses is at a distance of 3/5 R from O. At this instant the wall on the stick is perpendicular to the stick. The
the distance of the other mass from O is: [2015] stick makes an angle of 30° with the wall and the
2 1 bottom of the stick is on a rough floor. The reaction
(a) ​ __ ​  R (b) ​ __ ​  R of the wall on the stick is equal in magnitude to the
3 3
3 4 reaction of the floor on the stick. The ratio h/l and
(c) ​ __ ​  R (d) ​ __ ​  R the frictional force f at the bottom of the stick are
5 ​_›
5
32. The position vector ​r ​   of a particle of mass m is (g = 10 ms–2) [2016]
__ __
given by the following equation r (t) = a t3  + b t2  , h ÷   
​ 3 ​   
16​÷3 ​
(a) ​ __ ​  = ___
​   ​ , f = _____
​   ​  N
where a = 10/3 ms–3, b = 5 ms–2 and m = 0.1 kg. At l 16 3
__
t = 1 s, which of the following statement(s) is (are) h 3   
16​÷3 ​
true about the particle? [2016] (b) ​ __ ​  = ___
​    ​,  f = _____
​   ​ N


l 16 3
Past Years’ Questions  14.13 

__ __
h 3​÷3 ​      
8​÷3 ​ (c) mw2R sin w t  – mg 
(c) ​ __ ​  = ____ , f = ____
​   ​  ​   ​ 
 N
l 16 3 (d) – mw2 R cos w t  – mg 
__ __
h   
3​÷3 ​   
16​÷3 ​
(d) ​ __ ​  = ____ , f = _____
​   ​  ​   ​  N 36. A wheel of radius R and mass M is placed at the
l 16 3 bottom of a fixed step of height R, as shown in the
35. Following two questions are based on the given figure. A constant force is continuosly applied on
passage the surface of the wheel so that it just climbs the
A frame of reference that is accelerated with respect step without slipping. Consider the torque t about an
to an inertial frame of reference is called a non- axis normal to the plane of the paper passing through
inertial frame of reference. A co-ordinate system, the point Q. Which of the following options is/are
fixed on a circular disc, rotating about a fixed axis, correct? [ 2017]
with a constant angular velocity w is an example of
a non-inertial frame
​_› of reference. The relationship
between the force ​F   ​rot experienced by a particle ​_›of
mass m moving on the rotating disc and the force ​F   ​in
experienced by the particle in an inertial frame of
reference is
​_› ​_› ​_› ​__› ​__› ​_› ​__›
​F  ​rot
  = ​F    + 2m (​u 
 ​in  ​rot
  × w​
​  )  + m (​w​   × ​r ​ )  × w​
​   
_
​›
where ​u  ​ rot is the velocity of_​ the particle in the rotat-

ing frame of reference and ​r ​   is the position vector of (a) If the force is applied at point P tangentially,
the particle with respect to the centre of the disc. then t decreases continuously as the wheel
Now, consider a smooth slot along a diameter of climbs.
a disc of radius R rotating counter-clockwise, with (b) If the force is applied normal to the circumfer-
a constant angular speed w about its vertical axis ence at point X, then t is constant
through its centre. We assign a co-ordinate system (c) If the force is applied normal to the circumfer-
with the origin at the centre of the disc, the x-axis ence at point P then t is zero
along the slot, the y-axis perpendicular ​__›
to the slot and
the z-axis along the rotation axis (​w​   = w  ). A small (d) If the force is applied tangentially at point S then
​_› t π 0 but the wheel never climbs the step.
block of mass m is gently placed in the slot at ​r ​   =
(R/2)  at t = 0 and is constrained to move only along 37. Following two questions are based on the given
the slot. [2016] passage
One twirls a circular ring (of mass M and radius R)
near the tip of one’s finger, as shown in Figure 1. In
the process, the finger never loses contact with the
inner rim of the ring. The finger traces out the sur-
face of a cone, shown by the dotted line. The radius
of the path traced out by the point where the ring
and the finger is in contact is r. The finger rotates
with an angular velocity w0. The rotating ring rolls
without slipping on the outside of a smaller circle
described by the point where the ring and the finger
(i) The distance r of the block at time t is are in contact (Figure 2). The coefficient of friction
R R between the ring and the finger is m and the accelera-
(a) ​ __ ​  cos w t (b) ​ __ ​  (ew t + e –w t)
2 4 tion due to gravity is g. [2017]
R R
(c) ​ __ ​  cos 2w t (d) ​ __ ​  (e2w t + e– 2w t)
2 4
(ii) The net reaction of the disc on the block is
1
(a) ​ __ ​  mw2 R (ew t – e– w t)  + mg 
2
1
(b) ​ __ ​  mw2 R (e2w t – e– 2w t)  + mg 
2
14.14  Mechanics II

​_›
(i) The total kinetic energy of the ring is physical quantities of the​_› particle are mentioned: ​p  ​ 
1 is the linear momentum, ​L  ​  is the angular momentum
(a) M w 20 R2 (b) ​ __ ​  M w 20 (R – r)2
2 about the origin, K is the kinetic energy, U is the
3 potential energy and E is the total energy. Match
2 2
(c) M w 0 (R – r) (d) ​ __ ​  M w 20 (R – r)2 each path in List-I with those quantities in List-II,
2
(ii) The minimum value of w0 below which the ring will which are conserved for that path.
drop down is [JEE Adv. 2018]

÷  ÷ 
________ ________
g 2g LIST–I LIST–II
(a) ​ _______
​      ​ ​
  (b) ​ _______
​     ​ ​
  ​_› ​_›
m (R – r) m (R – ) P. ​r ​   (t) = a t iˆ + b t jˆ 1. ​p  ​ 
​_ ​_›

÷  ÷ 
_________ ›
3g
_________
g Q. ​r ​   (t) = a cos w t iˆ + b sin w t jˆ 2. ​L  ​ 
​_›
(c) ​ ________
​      ​ ​
  (d) ​ ________
​       ​ ​
  R. ​r ​   (t) = a (cos w t iˆ + sin w t jˆ) 3. K
2m (R – r) 2m (R – r)
​_› b
38. The potential energy of a particle of mass m at a S. ​r ​   (t) = a t iˆ + ​ __ ​  t2 jˆ 4. U
2
distance r from a fixed point O is given by V(r) = 5. E
kr2/2, where k is a positive constant of appropriate (a) P Æ 1, 2, 3, 4, 5; Q Æ 2, 5;
dimensions. This particle is moving in a circular orbit
R Æ 2, 3, 4, 5; SÆ5
of radius R about the point O. If v is the speed of the
(b) P Æ 1, 2, 3, 4, 5; Q Æ 3, 5;
particle and L is the magnitude of its angular momen-
tum about O, which of the following statements is R Æ 2, 3, 4, 5; S Æ 2, 5
(are) true? [JEE Adv. 2018] (c) P Æ 2, 3, 4; Q Æ 5;

÷ 
___ __

÷ ___
k k R Æ 1, 2, 4; S Æ 2, 5
(a) v = ​ ___
​     ​ ​   R (b) v = ​ __
​ m   ​ ​  R
2m (d) P Æ 1, 2, 3, 5; Q Æ 2, 5;

÷ 
___ mk R Æ 2, 3, 4, 5; S Æ 2, 5
​R2
   
(c) L = ​÷mk  (d) L = ​ ___
​   ​ ​   R2
2
39. Consider a body of mass ​_› 1.0 kg at rest at the origin  FLUID MECHANICS
at time t = 0. A force ​F   ​ = (a t î + b jˆ ) is applied on
the body, where a = 1.0 Ns–1 and b = 1.0 N. The AIEEE/JEE Main Questions
torque acting _
​›
on the body about the origin at time
t = 1.0 s is ​t   ​. Which of the following statements is 1. A jar is filled with two non–mixing liquids 1 and 2,
(are) true? [JEE Adv. 2018] having densities r1 and r2 respectively. A solid ball,
​_› 1 made of a material of density r3, is dropped in the
__
(a) |​t   ​ | = ​   ​  Nm
3 ​_›
jar. It comes to equilibrium in the position shown
(b) The torque ​t   ​is in the direction of the unit vector in the figure. Which of the following is true for r1,
+ k̂ r2 and r3? [AIEEE 2008]
​_› 1
(c) The velocity of the body at t = 1s is ​v   ​ = ​ __  ​
2
( î + 2 jˆ ) ms–1
(d) The magnitude of displacement of the body at
1
t = 1 s is __
​   ​  m
6
40. A ring and a disc are initially at rest, side by side, at
the top of an inclined plane which makes an angle
60° with the horizontal. They start to roll without
slipping at the same instant of time along the shortest (a) r3 < r1 < r2 (b) r1 < r3 < r2
path. If the time difference
__ ___ between their reaching the (c) r1 < r2 < r3 (d) r1 < r3 , r2
ground is (2 – ÷    )/​÷10 ​
​ 3 ​    s, then the height of the top
2. A ball is made of a material of density r, where
of the inclined plane, in metres, is __________. Take
roil < r < rwater with roil and rwater representing the
g = 10 ms–2. [JEE Adv. 2018]
densities of oil and water, respectively. The oil and
41. In the List-I below, four different paths of a particle water are immiscible. If the above ball is in equi-
are given as functions of time. In these functions, a librium in a mixture of this oil and water, which
and b are positive constants of appropriate dimen- of the following pictures represents its equilibrium
sions and a π b. In each case, the force acting on the position? [AIEEE 2010]
particle is either zero or conservative. In List-II, five
Past Years’ Questions  14.15 

1 + sin a 1 + cos a
(a) ​ ________  
​ (b) ​ ________   ​
1 – sin a 1 – cos a
(a) (b) 1 + tan a 1 + sin a
(c) ​ ________  
​ (d) ​ ________   ​

1 – tan a 1 – cos a

IIT JEE (Advanced) Questions


1. A U-tube is rotated about one of it s limbs with an
angular velocity w. Find difference in height H of the
liquid (density r) level, where diameter of the tube
(c) (d)
d << L. [2005]

3. Water is flowing continuously from a tap having


an internal diameter 8 × 10–3 m. The water velocity
as it leaves the tap is 0.4 ms–1. The diameter of the
water stream at a distance 2 × 10–1 m below the tap
is close to: [AIEEE 2011]
(a) 9.6 × 10–3 m (b) 3.6 × 10–3 m
(c) 5.0 × 10–3 m (d) 7.5 × 10–3 m
w 2L2 w L2
4. A uniform cylinder of length L and mass M, having (a) H = ​ _____
  
​   (b) H = ​ ____   

2g 2g
cross-sectional area A is suspended, with its length
vertical, from a fixed point by a massless spring, w 2L
(c) H = ​ ____   
​ (d) None
such that it is half submerged in a liquid of density 2g
s at equilibrium position. The extension x0 of the
2. Water is filled in a container upto height 3m. A
spring when it is in equilibrium is: (Here k is spring
small hole of area ‘a’ is punched in the wall of the
constant) [JEE Main 2013]
container, at a height 52.5 cm from the bottom. The

Mg
(a) ​ ___   
k (  LA s
​ ​ 1 – ____
​     
M )
​  ​
Mg
(b) ​ ___   
k (  LA s
)
​ ​ 1 – ​ ____ ​  ​
2M
cross-sectional area of the container is A. If a/A = 0.1,
then v2 is: (where v is the velocity of water coming
out of the hole. Take g = 10 ms–2) [2005]

​ ​  1 + ____
k ( M )
Mg LA s Mg (a) 50 (b) 51
(c) ​ ___    ​     
​  ​ (d) ​ ___   
​ 
k (c) 48 (d) 51.5
5. An open glass tube is immersed in mercury in such 3. Following three questions are based on the given
a way that a length of 8 cm extends above the mer- passage
cury level. The open-end of the tube is then closed A cylindrical tank has a hole of diameter 2r in its
and sealed and the tube is raised vertically up by bottom. The hole is covered with a wooden cylindri-
an additional 46 cm. What will be length of the air cal block of diameter 4r, height h and density r/3,
column above mercury in the tube now? (Atmospheric as shown. Initially, the tank is filled with water of
pressure = 76 cm of Hg) [JEE Main 2014] density r to a certain height. The level of water is
(a) 16 cm (b) 22 cm decreased gradually and it was found that the block
(c) 38 cm (d) 6 cm begins to move up when the height of water above
6. There is a circular tube in a ver- the top of the block is h1 (measured from the top of
tical plane. Two liquids, which the block). An external agent applies force to keep
do not mix and of densities d1 the block in place and the level of water is reduced
and d2, are filled in the tube. further. When the level of water is reduced to a point
Each liquid subtends 90° angle where its height from the base of the block is h2 (top
at the centre. Radius joining surface of the block is exposed to atmosphere) the
their interface makes an angle external force is no longer needed to keep the block
a with the vertical. Ratio (d1/d2) is: in place. [2006]
[JEE Main 2014]
14.16  Mechanics II

and/or Y. Match these statements to the appropriate


system(s) from Column-II. [2009]
Column–I Column–II
(a) The force (p) Block Y of mass M left on a fixed
exerted inclined plane X, slides on it with a
by X on constant velocity.
Y has a
(i) Value of height h1 is magnitude
2h 5h
(a) ​ ___ ​   (b) ​ ___ ​  Mg.
3 4
5h
___ 5h
___
(c) ​   ​  (d) ​   ​  (b) The (q) Two ring magnets Y and Z, each
3 2
(ii) Value of h2 is gravi- of mass M, are kept in frictionless
4h 5h tational vertical plastic stand so that they
(a) ​ ___ ​  (b) ​ ___ ​  potential repel each other. Y rests on the base
9 9
energy of X X and Z hangs in air in equilibrium.
h
__ 2h
___
(c) ​   ​   (d) ​   ​  is con- P is the top-most
3 3 tinuously point of the stand on
(iii) If height of water is lowered below h2, then: increasing. the common axis of
(a) cylinder will not move up and remains at its the two rings. The
original position whole system is in
(b) for h2 = h/3, cylinder again starts moving up a lift that is going
up with a constant
(c) for h2 = h/4, cylinder again starts moving up velocity.
(d) for h2 = h/5 cylinder again starts moving up (c) Mechanical (r) A pulley Y of mass m0 is fixed to
4. Statement-1: The stream of water flowing at high energy of a table through a clamp X. A block
speed from a garden hose pipe tends to spread like a the system of mass M hangs from a string that
fountain when held vertically up, but tends to narrow X + Y is goes over the pulley and is fixed
down when held vertically down. continuously at point P of the table. The whole
decreasing. system is kept in a lift that is going
Statement-2: In any steady flow of an incompress- down with a constant velocity.
ible fluid, the volume flow rate of the fluid remains
constant. [2008]
(a) Statement–1 is true, Statement–2 is true;
Statement–2 is a correct explanation for
Statement–1
(b) Statement–1 is true, Statement–2 is true; (d) The torque (s) A sphere Y of mass M is put in a
Statement–2 is not a correct explanation for of the non-viscous liquid X, kept in a con-
Statement–1 weight of Y tainer at rest. The sphere is released
(c) Statement–1 is true, Statement–2 is false about point and it moves down in the liquid.
P is zero.
(d) Statement–1 is false, Statement–2 is true
5. A cylindrical vessel of height 500 mm has an orifice
(small hole) at its bottom. The orifice is initially
closed and water is filled in it up to height H. Now
the top is completely sealed with a cap and the orifice
at the bottom is opened. Some water comes out from
the orifice and the water level in the vessel becomes (t) A sphere Y of mass M is falling with
steady with height of water column being 200 mm. its terminal velocity in a viscous liq-
uid X kept in a container.
Find the fall in height (in mm) of water level due to
opening of the orifice.
[Take atmospheric pressure 1.0 × 105 Nm–2, density
of water 1000 kgm–3 and g = 10 ms–2. Neglect any
effect of surface tension.] [2009]
6. Column-II shows five systems in which two objects
are labelled as X and Y. Also, in each case, a point P
is shown. Column-I gives some statements about X
Past Years’ Questions  14.17 

7. A thin uniform cylindrical shell, closed at both ends, List I List II


is partially filled with water. It is floating vertically
P. Lift is accelerating vertically 1. d = 1.2 m
in water in half-submerged state. If rC is the relative up
density of the material of the shell with respect to
water, then the correct statement is that the shell is: Q. Lift is accelerating vertically 2. d > 1.2 m
down with an acceleration
[2012] less than the gravitational
(a) more than half-filled if rC is less than 0.5 acceleration.
(b) more than half-filled if rC is more than 1.0 R. Lift is moving vertically up with 3. d < 1.2 m
constant speed.
(c) half-filled if rC is more than 0.5
S. Lift is falling freely. 4. No water leaks
(d) less than half-filled if rC is less than 0.5
out of the jar
8. Following two questions are based on the given
Code:
passage
(a) P-2, Q-3, R-2, S-4 (b) P-2, Q-3, R-1, S-4
A spray gun is shown in the figure where a piston
pushes air out of a nozzle. A thin tube of uniform (c) P-1, Q-1, R-1, S-4 (d) P-2, Q-3, R-1, S-1
cross-section is connected to the nozzle. The other
end of the tube is in a small liquid container. As the SURFACE TENSION AND VISCOSITY
piston pushes air through the nozzle, the liquid from
the container rises into the nozzle and is sprayed
out. For the spray gun shown, the radii of the piston AIEEE/JEE Main Questions
and the nozzle are 20 mm and 1 mm respectively. 1. A 20 cm long capillary tube is dipped in water. The
The upper end of the container is open to the atmo- water rises upto 8 cm. If the entire arrangement is
sphere. [JEE Adv. 2014] put in a freely falling elevator, the length of water
column in the capillary tube will be [AlEEE 2005]
(a) 8 cm (b) 10 cm
(c) 4 cm (d) 20 cm
2. If the terminal speed of a sphere of gold
(i) If the piston is pushed at a speed of 5 mms–1, the air (density = 19.5 × 103 kg m–3) is 0.2 ms–1 in a viscous
comes out of the nozzle with a speed of liquid (density = 1.5 × 103 kg m–3), find the terminal
(a) 0.1 ms–1 (b) 1 ms–1 speed of a sphere of silver (density = 10.5 kgm–3) of
–1 the same size in the same liquid. [AIEEE 2006]
(c) 2 ms (d) 8 ms–1
(a) 0.4 ms–1 (b) 0.133 ms–1
(ii) If the density of air is ra and that of the liquid r1, –1
(c) 0.1 ms (d) 0.2 ms–1
then for a given piston speed, the rate (volume per
unit time) at which the liquid is sprayed will be 3. A spherical solid ball of volume V is made of a
proportional to material of density r1. It is falling through a liquid
of density r2 (r2 < r1). Assume that the liquid applies

÷ 
___
ra _____
(a) ​ __
​   ​ ​      a r1 ​ 
(b) ​÷r a viscous force on the ball that is proportional to the
r1
square of its speed v, i.e., Fviscous = kv2 (k > 0). The

​     ​ ​
___
r terminal speed of the ball is [AIEEE 2008]
÷r
(c) ​ __
÷ 
1 __________
  (d) r1
Vg (r1 – r2) Vgr1
(a) ​ ​ __________ (b) ​ _____
a
    
     ​ ​     

9. A person in a lift is holding a water jar, which has a k k

÷ 
_____
small hole at the lower end of its side. When the lift Vgr1 Vg (r1 – r2)
(c) ​ _____
​     ​ ​    (d) ​ __________
      

is at rest, the water jet coming out of the hole hits k k
the floor of the lift at a distance d of 1.2 m from the 4. A capillary tube (A) is dipped in water. Another
person. In the following, state of the lift’s motion is identical tube (B) is dipped in a soap-water solution.
given in List I and the distance where the water jet Which of the following shows the relative nature of
hits the floor of the lift is given in List II. Match the liquid columns in the two tubes? [AIEEE 2008]
the statements from List I with those in List II and
select the correct answer using the code given below
(a)
the lists. [JEE Adv. 2014]
14.18  Mechanics II

respectively. Surface tension of the soap-water used


(b) to make bubbles is 0.04 Nm–1. Find the ratio nB/nA,
where nA and nB are the number of moles of air in
bubbles A and B, respectively. [Neglect the effect of
gravity.] [2009]
(c) 3. Following three questions are based on the given
passage
When liquid medicine of density r is to be put in
the eye, it is done with the help of a dropper. As
(d) the bulb on the top of the dropper is pressed, a drop
forms at the opening of the dropper. We wish to
5. Work done in increasing the size of a soap bubble estimate the size of the drop. We first assume that the
from a radius of 3 cm to 5 cm is nearly (surface ten- drop formed at the opening is spherical because that
sion of soap solution = 0.03 Nm–1): [AIEEE 2011] requires a minimum increase in its surface energy.
(a) 2p m J (b) 0.4 p m J To determine the size, we calculate the net vertical
(c) 4p m J (d) 0.2p m J force due to the surface tension T when the radius
of the drop is R. When this force becomes smaller
6. On heating water, bubbles being than the weight of the drop, the drop gets detached
formed at the bottom of the ves- from the dropper. [2010]
sel detach and rise. Take the
bubbles to be spheres of radius (i) If the radius of the opening of the dropper is r, the
R and making a circular contact vertical force due to the surface tension on the drop
of radius r with the bottom of the of radius R (assuming r << R) is:
vessel. If r << R and the surface (a) 2p rT (b) 2p RT
tension of water is T, value of r just before bubbles 2
2p r T 2p R2T
detach is: (density of water is rw) (c) ​ ______
   ​   (d) ​ ______
r   ​ 
R
[JEE Main 2014]
(ii) If r = 5 × 10–4 m, r = 103 kg m–3, g = 10 ms–2

÷  ÷ 
____ ____
rwg rwg T = 0.11 Nm–1, the radius of the drop when it detaches
(a) R2 ​ ____
​   ​ ​  
  (b) R2 ​ ____
​   ​ ​ 

3T 6T from the dropper is approximately

÷ 
_____

÷ 
____
2 ____
rwg 3rwg (a) 1.4 × 10–3 m (b) 3.3 × 10–3 J
(c) R ​ ​       ​ ​   (d) R2 ​ _____
​      ​ ​  
T T (c) 2.0 × 10–3 m (d) 4.1 × 10–3 m
(iii) After the drop detaches, its surface energy is
IIT JEE (Advanced) Questions
(a) 1.4 × 10–3 J (b) 2.7 × 10–6 J
1. Water is filled up to a height
h in a beaker of radius R as (c) 5.4 × 10–6 J (d) 8.1 × 10–6 J
shown in the figure. The den- 4. A glass capillary tube is of the shape
sity of water is r, the surface of a truncated cone with an apex angle
tension of water is T and the a so that its two ends have cross-sec-
atmospheric pressure is P0. tions of different radii. When dipped
Consider a vertical section in water vertically, water rises in it
ABCD of the water column through a diameter of to a height h, where the radius of its
the beaker. The force on water on one side of this cross-section is b. If the surface ten-
section by water on the other side of this section has sion of water is S, its density is r, and
magnitude: [2007] its contact angle with glass is q, the value of h will
|  2
(a) ​ 2P0 Rh + p R  rgh – 2RT   ​ | be (g is the acceleration due to gravity)
2S 2S
[2014]
| 
(b) ​ 2P0 Rh + R rgh2 – 2RT   ​ | ____
br g
____
(a) ​    ​ cos (q – a) (b) ​    ​ cos (q + a)
br g
| 
(c) ​ P0 p R2 + R rgh2 – 2RT   ​ | 2S
(  a
(c) ​ ____  ​ cos ​ q – __ ) 2S
(  a
​   ​   ​ (d) ​ ____  ​ cos ​ q + __ )
​   ​   ​
| 

(d) ​ P0 p R2 + R rgh2 – 2RT  ​ | br g 2 brg 2
2. Two soap bubbles A and B are kept in a closed cham- 5. Two spheres P and Q of equal radii have densities
ber where the air is maintained at pressure 8 Nm–2. r1 and r2, respectively. The spheres are connected by
The radii of bubbles A and B are 2 cm and 4 cm, a massless string and placed in liquids L1 and L2 of
Past Years’ Questions  14.19 

densities s1 and s2 and viscosities (c) The tangential (shear) stress on the floor of the
h1 and h2, respectively. They float tank increases with u0
in equilibrium with the sphere P (d) The tangential (shear) stress on the plate varies
in L1 and sphere Q in L2 and the linearly with the viscosity h of the liquid
string being taut (see figure). If
sphere P​_alone in L2 has terminal

 GRAVITATION
velocity ​v   ​P and Q​_alone in L1 has

terminal velocity ​v   ​Q, then AIEEE/JEE Main Questions
[2015] 1. A particle of mass 10 g is kept on the surface of a uni-


| 
(a) ​ 
​_›
____ |
​ ​v  ​ P  ​

h1
 ​ = ___
​   ​  
| 
(b) ​ 
​_›
____ |
​ ​v  ​ P  ​

h2
 ​ = ___
​   ​ 
form sphere of mass 100 kg and radius 10 cm. Find
the work to be done against the gravitational force
| 
​_›
​ ​v  Q
​_› ​_›
|
 ​   ​
h2
| 
​_›
​ ​v  Q
​_› ​_›
|
 ​   ​
h1
between them to take the particle far away from the
sphere (you may take G = 6.67 × 10–11 Nm2kg–2
(c) ​v  ​ P ◊ ​v  ​ Q > 0 (d) ​v  ​ P ◊ ​v  ​ Q <0
[aieee 2005]
6. Consider two solid spheres P and Q each of density 8 –9
(a) 6.67 × 10  J (b) 6.67 × 10–10 J
gm cm–3 and diameters 1 cm and 0.5 cm, respectively.
Sphere P is dropped into a liquid of density 0.8 (c) 13.34 × 10–10 J (d) 3.33 × 10–10 J
gm cm–3 and viscosity h = 3 poiseulles. Sphere Q 2. A planet in a distant solar system is 10 times more
is dropped into a liquid of density 1.6 gm cm–3 and massive than earth and its radius is 10 times smaller.
viscosity h = 2 poiseulles. The ratio of the terminal Given that the escape velocity from the earth is 11
velocities of P and Q is [2016] km s–1, the escape velocity from the surface of the
7. A drop of liquid of radius R = 10–2 m having surface planet would be [aieee 2008]
0.1
tension S = ​ ___ ​ Nm–1 divides itself into K identi- (a) 1.1 km s–1 (b) 11 km s–1
4p (c) 110 km s–1 (d) 0.11 km s–1
cal drops. In this process, the total change in the
surface energy is DU = 10–3 J. If K = 10 a then the 3. The height at which the acceleration due to gravity
value of a is [2017] becomes g/9 (where g = the acceleration due to
gravity on the surface of the earth) in terms of R,
8. A uniform capillary tube of inner radius r is dipped the radius of the earth, is: [aieee 2009]
vertically into a beaker filled with water. The water R
rises to a height h in the capillary tube above the (a) 2R (b) ​ ___
__  ​ 
÷   
​ 2 ​
water surface in the beaker. The surface tension of __
water is s. The angle of contact between water and (c) R/2 (d) ​÷ 2 ​  R
the wall of the capillary tube is q. Ignore the mass 4. The mass of a spaceship is 1000 kg. It is to be
of water in the meniscus. Which of the following launched from the earth’s surface, out into free space.
statements is (are) true? [JEE Adv. 2018] The value of ‘g’ and ‘R’ (radius of earth) are 10 ms–2
(a) For a given material of the capillary tube, h and 6400 km respectively. The required energy for
decreases with increase in r this work will be: [aieee 2012]
(b) For a given material of the capillary tube, h is (a) 6.4 × 10 Joules (b) 6.4 × 1011 Joules
10

independent of s (c) 6.4 × 108 Joules (d) 6.4 × 109 Joules


(c) If this experiment is performed in a lift going up 5. What is the minimum energy required to launch a
with a constant acceleration, then h decreases satellite of mass m from the surface of a planet of
(d) h is proportional to contact angle q mass M and radius R in a circular orbit at an altitude
9. Consider a thin square plate floating on a viscous of 2R? [JEE Main 2013]
liquid in a large tank. The height h of the liquid in 2Gm M Gm M
the tank is much less than the width of the tank. The (a) ​ ______
    ​   (b) ​ _____
  ​ 
3R 2R
floating plate is pulled horizontally with a constant
Gm M 5Gm M
velocity u0. Which of the following statements is (c) ​ _____
  ​   (d) ​ ______
    ​ 
3R 6R
(are) true? [JEE Adv. 2018]
6. Four particles, each of mass M are at the vertices of
(a) The resistive force of liquid on the plate is a square and move along a circle of radius R, under
inversely proportional to h the action of their mutual gravitational attraction. The
(b) The resistive force of liquid on the plate is speed of each particle is: [JEE Main 2014]
independent of the area of the plate
14.20  Mechanics II

÷  ÷ 
____ ________
GM __ GM
(a) ​ ____
​     
​ ​    (b) ​     ____
2​÷2 ​ ​     
​ ​ 

R R

÷​   R    ​ (1 + 2​÷2 ​  ) ​ ÷ 


____________ ____________
GM __ 1 GM __ (c) (d)
(c) ​ ____     (d) ​ __ ​  ​____
​        )   ​
2​÷2 ​
​ (1 +   
2 R
7. From a solid sphere of mass M
2. Statement-1: An astronaut in an orbiting space sta-
and radius R, a spherical portion
tion above Earth experiences weightlessness.
of radius R/2 is removed, as shown
in the figure. Taking gravitational Statement-2: An object moving around Earth under
potential V = 0 at r = •, the poten- the influence of Earth’s gravitational force is in a
tial at the centre of the cavity thus state of “free-fall”. [2008]
formed is: (G gravitational constant) (a) Both Assertion and Reason are true and Reason
[JEE Main 2015] is the correct explanation of Assertion
– 2GM – GM (b) Both Assertion and Reason are true and Reason
(a) ​ ______​  
    (b) ​ _____
  ​  is not the correct explanation of Assertion’
R 2R
(c) Assertion is true but Reason is false
– GM – 2GM
(c) ​ _____ ​  
    (d) ​ ______
    ​  (d) Assertion is false but Reason is true
R 3R
8. A satellite is revolving in a circular orbit at a 3. A thin uniform annular disc (see figure) of mass M
height ‘h’ from the earth’s surface (radius of earth has outer radius 4R and inner radius 3R. The work
R; h << R). The minimum increase in its orbital veloc- required to take a unit mass from point P on its axis
ity required, so that the satellite could escape from to infinity is: [2010]
the earth’s gravitational field, is close to: (Neglect
the effect of atmosphere). [JEE Main 2016]
____ ___
   
(a) ​÷2gR  ​    ​
(b) ​÷gR 
____ ___ __
   
(c) ​÷gR/2 ​    ​ (​÷2 ​
(d) ​÷gR      – 1)
9. The variation of acceleration due to gravity g with
distance d from the centre of earth is best represented
by (R = Earth’s radius): [JEE Main 2017]
2GM __ 2 __
(a) ​ _____     – 5)
​ (4​÷2 ​
   (b) – ​ _____
       – 5)
​ (4​÷2 ​
7R 7R
(a) (b)
GM 2GM __
(c) ​ ____ ​   (d) ​ _____ 
       – 1)
​ (​÷2 ​
4R 5R
4. Gravitational
__ acceleration on the surface of a planet
(c) (d)    /11g, where g is the gravitational acceleration on
is ​÷6 ​
the surface of the earth. The average mass density of
the planet is 2/3 times that of the earth. If the escape
speed on the surface of the earth is taken to be 11
IIT JEE (Advanced) Questions
km s–1, the escape speed on the surface of the planet
1. A spherically symmetric gravitational system of par- in km s–1 will be [2010]
ticles has a mass density [2008] 5. A satellite is moving with a constant speed ‘v’ in a
r for    r £ R circular orbit about the earth. An object of mass ‘m’
r =  ​ 0    ​

​ 0 for  r > R ​ is ejected from the satellite such that it just escapes
Where r0 is a constant. A test mass can undergo from gravitational pull of the earth. At the time of
circular motion under the influence of the gravita- its ejection, the kinetic energy of the object is:
tional field of particles. Its speed v as a function of [2011]
distance r (0 < r < •) from the centre of the system (a) (1/2) mv 2
(b) mv  2

is represented by (c) (3/2) mv2 (d) 2mv2


6. Two bodies, each of mass M, are kept fixed with
a separation 2L. A particle of mass m is projected
(a) (b)
from the mid-point of the line joining their centres,
Past Years’ Questions  14.21 

perpendicular to the line. The gravitational constant


is G. The correct statement(s) is (are) [2013]
(a) The minimum initial velocity of mass m to
escape_____ the gravitational field of the two bodies

is 4 ​÷GM/L  ​

11. A rocket is launched normal to the surface of the
(b) The minimum initial velocity of the mass m to Earth, away from the Sun, along the line joining
escape_____ the gravitational field of the two bodies the Sun and the Earth. The Sun is 3 × 105 times

is 2 ​÷GM/L  ​

heavier than the Earth and is at a distance 2.5 × 104
(c) The minimum initial velocity of the mass m to times larger than the radius of the Earth. The escape
escape the gravitational field of the two bodies velocity from Earth’s gravitational field is ve = 11.2

÷ 
_____
2GM km s–1. The minimum initial velocity (vs) required for
is ​ ​ _____
   ​ ​  
L the rocket to be able to leave the Sun–Earth system
(d) The energy of the mass m remains constant is closest to (Ignore the rotation and revolution of the
1 Earth and the presence of any other planet) [2017]
7. A planet of radius R = ___ ​    ​ × (radius of Earth) has the
10 (a) vs = 22 km s–1 (b) vs = 42 km s–1
same mass density as Earth. Scientists dig a well of –1
depth R/5 on it and lower a wire of the same length (c) vs = 62 km s (d) vs = 72 km s–1
and of linear mass density 10–3 kg m–1 into it. If the 12. A planet of mass M, has two natural satellites with
wire is not touching anywhere, the force applied at masses m1 and m2. The radii of their circular orbits
the top of the wire by a person holding it in place are R1 and R2 respectively. Ignore the gravitational
is (take the radius of Earth = 6 × 106 m and the force between the satellites. Define v1, L1, K1 and T1
acceleration due to gravity on Earth is 10 ms–2) to be, respectively, the orbital speed, angular momen-
[2014] tum, kinetic energy and time period of revolution of
satellite 1; and v2, L2, K2 and T2 to be the correspond-
(a) 96 N (b) 108 N ing quantities of satellite 2. Given m1/m2 = 2 and
(c) 120 N (d) 150 N R1/R2 = 1/4, match the ratios in List-I to the numbers
8. A spherical body of radius R consists of a fluid of in List-II. [JEE Adv. 2018]
constant density and is in equilibrium under its own LIST–I LIST–II
gravity. If P (r) is the pressure at (r < R), then the v1 1
correct option (s) is (are) [2015] P. ​ __
v2 ​   1. ​ __ ​ 
8
P (r = 3R/4) 63 L1
(a) P (r = 0) = 0 (b) ​ __________
    ​ = ___
​   ​  __
Q. ​   ​   2. 1
P (r = 2R/3) 80 L2
P (r = 3R/5) 16 P (r = R/2) 20 K1
(c) ​ __________
    ​ = ___
​   ​  (d) ​ _________   ​ = ___
​   ​  R. ​ ___ ​   3. 2
P (r = 2R/5) 21 P (r = R/3) 27 K2
9. A bullet is fired vertically upward with velocity T1
S. ​ __ ​   4. 8
v from the surface of a spherical planet. When it T2
reaches its maximum height, its acceleration due to
(a) P Æ 4; Q Æ 2; R Æ 1; S Æ 3
the planet gravity is one-fourth of its value at the
surface of the planet. (b) P Æ 3; Q Æ 2; R Æ 4; S Æ 1
__ If the escape velocity from the
planet is vese = v ​÷N    ​, then the value of N is (ignore (c) P Æ 2; Q Æ 3; R Æ 1; S Æ 4
energy loss due to atmosphere) [2015] (d) P Æ 2; Q Æ 3; R Æ 4; S Æ 1
10. A large spherical mass M is fixed at one position
and two identical point masses m are kept on a line
passing through the centre of M (see figure). The  ELASTICITY
point masses are connected by a rigid massless rod
of length l and this assembly is free to move along AIEEE/JEE Main Questions
the line connecting them. All three masses interact
1. If S is stress and Y is Young’s modulus of material of
only through their mutual gravitational interaction.
a wire, the energy stored in the wire per unit volume
When the point mass nearer to M is at a distance
is [AIEEE 2005]
r = 3l from M, the tension in the rod is zero for
(  )
2 2
M (a) 2S Y (b) S /2Y
m = k ​ ____
​    ​  ​. The value of k is [2015] 2
288 (c) 2Y/S (d) S/2Y
14.22  Mechanics II

2. A wire elongates by l when a load w is hanged from 105 Pa then the volume changed by 10%. The bulk
it. If the wire goes over a pulley and two weights, modulus is: [2005]
w each, are hung at the two ends, the elongation of 5
(a) 1.55 × 10 Pa 5
(b) 0.115 × 10 Pa
the wire will be (in mm) [AIEEE 2006] 5
(c) 1.4 × 10 Pa (d) 1.01 × 105 Pa
(a) l (b) 2l
2. One end of a horizontal thick copper wire of length
(c) zero (d) l/2 2L and radius 2R is welded to an end of another
3. Two wires are made of the same material and have horizontal thin copper wire of length L and radius
the same volume. However, wire 1 has cross-sec- R. When the arrangement is stretched by applying
tional area A and wire 2 has cross-sectional area 3A. forces at two ends, the ratio of the elongation in the
If the length of wire 1 increases by D x on applying thin wire to that in the thick wire is [2013]
force F, how much force is needed to stretch wire 2 (a) 0.25 (b) 0.50
by the same amount? [AIEEE 2009]
(c) 2.00 (d) 4.00
(a) F (b) 4F
3. In plotting stress versus strain curves for two
(c) 6F (d) 9F materials P and Q, a student by mistake puts strain
4. A man grows into a giant such that his linear dimen- on the y-axis and stress on the x-axis, as shown in
sions increase by a factor of 9. Assuming that his the figure. Then the correct statement(s) is (are)
density remains same, the stress in the leg will [2015]
change by a factor of: [JEE Main 2017]
1
(a) 9 (b) ​ __ ​ 
9
1
(c) 81 (d) ​ ___  ​ 
81
5. A solid sphere of radius r made of a soft material
of bulk modulus K is surrounded by a liquid in a
cylindrical container. A massless piston of area a
floats on the surface of the liquid, covering entire
cross section of cylindrical container. When a mass
m is placed on the surface of the piston to compress
the liquid, the fractional decrement in the radius of (a) P has more tensile strength than Q
(  )
the sphere, ​ __
dr
​  r ​   ​, is: [JEE Main 2018] (b) P is more ductile than Q
mg Ka (c) P is more brittle than Q
(a) ​ ___  ​ (b) ​ ___
mg 
Ka (d) The Young’s modulus of P is more than that
Ka mg of Q
 ​(c) ​ ____   
​ (d) ​ ____    ​
3mg 3Ka
IIT JEE (Advanced) Questions
1. When temperature of a gas is 20°C and pressure is
changed from P1 = 1.01 × 105 Pa to P2 = 1.165 ×
Answers Sheet
Centre of Mass and Momentum
AIEEE/JEE (Main) Questions
1. (a) 2. (c) 3. (b) 4. (a) 5. (a) 6. (a) 7. (a) 8. (c) 9. (c)
10. (c) 11. (b) 12. (d) 13. (d) 14. (c) 15. (c)
IIT JEE (Advanced) Questions
1. (a) 2. (a) 3. (i) (b) (ii) (b)  (iii) (c) 4. (a) 5. (c) 6. (a) 7. (4)
8. (c) 9. (a, c) 10. (d) 11. (5) 12. (b) 13. (b, c) 14. (6.30 m)

Rotational Motion
AIEEE/JEE (Main) Questions
1. (b) 2. (b) 3. (d) 4. (d) 5. (b) 6. (d) 7. (a) 8. (b) 9. (b)
10. (c) 11. (d) 12. (b,d) 13. (a) 14. (a) 15. (a) 16. (a) 17. (b)
IIT JEE (Advanced) Questions
2 1
1. ​ __ ​ g sin q 2. ​ __ ​  (M + m)g cot q 3. (b) 4. (a) 5. (b) 6. (a) 7. (c, d)
3 2
8. (b) 9. 10 ms–1 10. (d) 11. (d) 12. (i) (c)  (ii) (a)  (iii) (b) 13. (d) 14. (a)
15. (b,c) 16. (b) 17. (4) 18. (9) 19. (3) 20. (d) 21. (a,b) 22. (b) 23. (d)
–1 –1 –1
24. (a) 25. (i) (d)  (ii) (a) 26. 8 rad s 27. (c,d) 28. 4 rad s 29. 2 rad s 30. (6) 31. (d)
32. (a,b,d) 33. (b,d) 34. (d) 35. (i) (b)  (ii) (a) 36. (c) 37. (i) none  (ii) (a) 38. (b,c)
39. (a,c) 40. (0.75 m) 41. (a)

Fluid Mechanics
AIEEE/JEE (Main) Questions
1. (d) 2. (c) 3. (b) 4. (b) 5. (a) 6. (c)
IIT JEE (Advanced) Questions
1. (a) 2. (a) 3. (i) (c)  (ii) (a)  (iii) (a) 4. (a) 5. 6 mm
6. (a) (p, t)  (b) (q, s, t)  (c) (p, r, t)  (d) (q) 7. (a) 8. (i) (c)  (ii) (a) 9. (c)

Surface Tension and Viscosity


AIEEE/JEE (Main) Questions
1. (d) 2. (c) 3. (a) 4. (c) 5. (b) 6. none
IIT JEE (Advanced) Questions
1. (b) 2. (6) 3. (i) (c)  (b) (a)  (iii) (b) 4. (d) 5. (a,d) 6. (3) 7. (6)
8. (a,c) 9. (a,c,d)

Gravitation
AIEEE/JEE (Main) Questions
1. (b) 2. (c) 3. (a) 4. (a) 5. (d) 6. (d) 7. (c) 8. (d) 9. (d)
IIT JEE (Advanced) Questions
1. (c) 2. (a) 3. (a) 4. (3) 5. (b) 6. (b, d) 7. (b) 8. (b, c) 9. (2)
10. (7) 11. (b) 12. (b)

14.23
Elasticity
AIEEE/JEE (Main) Questions
1. (b) 2. (a) 3. (d) 4. (a) 5. (d)
IIT JEE (Advanced) Questions
1. (a) 2. (c) 3. (a, b)

14.24
Solutions
1. Centre of Mass
2. Momentum and Its Conservation
3. Miscellaneous problems on chapter 1 and 2
4. Torque and Equilibrium
5. Kinematics of Rotation
6. Rotational Dynamics
7. Miscellaneous Problems on Chapter 4 to 6
8. Fluid Mechanics
9. Surface Tension and Viscosity
10. Miscellaneous problems on chapters 8 and 9
11. Gravitation
12. Elasticity
13. Miscellaneous Problems based on chapters 11 and 12
14. Past Years’ Questions
Chapter 1 Centre of Mass
4. (i) xcm = 0
Your Turn
fi m1 x1 + m2x2 + m3x3 + m4x4 = 0
1. The two stars move in circles of radii r1 and r2 about fi 1 × 4 + 2 × 6 + 3 × (– 1) + 2 x4 = 0
the COM, as shown in figure.
fi x4 = –  6.5 cm
Radius of circular path of m
= distance of m from COM (r1) (ii) The fourth particle shall be placed at the COM
4 md 4d of the system of m1, m2 and m3.
= _______ ​   ​ = ___
   ​   ​ 
m + 4 m 5 1 × 4 + 2 × 6 + 3 × (– 1) 13
fi x = ​ ______________________
     ​   = ___
​   ​ cm
1+2+3 6
5. One can find xcm, ycm and zcm separately or can
4m directly use
CM ​_› 1 ​_›
  = ​    ​  S mi ​r ​
​r ​ cm __  i 
M
r2
​_› ​_› ​_›
​_› m1 ​r ​ 1  + m2 ​r ​ 2  + m3 ​r ​ 3 
​r ​ cm
  = ​ ____________________
      
 ​
m
r1 m1 + m2 + m3
1 ( + 4 + ) + 2 ( + + ) + 3 (2 – – 2 )
= _______________________________________
​         ​    
2. Consider the co-ordinate system as shown, with B as 1+2+3

( 
origin.
y
9  + 3 – 3
= ​ ___________
6
 ​
    
3
2
1
​   ​  + __
= ​ __
1
​   ​  – __
2
​    ​   ​ m
2 )
m 6. Consider origin at end A and x-axis along the rod.
An element of length dx has mass given by
A
3 x dx
a
2

2m
B a 4m
x L
x
dm = l dx = l0 ​ 1 + __
​    ​  ​ dx[  ]
Mass of the rod is the sum of masses of all such
a
x co-ordinate of mass m is ​ __ ​  and its y co-ordinate is elements.
__ 2 L

Ú dm = l0 ​Ú ​  ​ ​( 1 + __​ L ​  )​ dx = l0 L + ____


÷   
​ 3 ​ x l0L 3
= ___
​   ​ a \ M =  = ​ __  ​ l0 L
​   ​ 
2 a 2 2
2m × 0 + 4m × a + m × __ ​    ​ 0
_______________________ 2 9
    ​= ___
L

( 
xcm = ​      ​    ​  a = 0.64 a
2m + 4m + m
__
14 xcm = __
M
1 2
​     ​ Ú x dm = _____
​      
3l0 L 0 0 L
x
​ ​Ú ​  ​ x l  ​ 1 + __
5L
​   ​   ​ dx = ___
​   ​ 
9 )
  
​÷3 ​
___
2m × 0 + 4m × 0 + m × ​   ​ a
2 7. O is the common end. Both the rods can be replaced
ycm = ​ __________________________
       ​ = 0.12 a
7 m with a point mass placed at their respective centres (at
Distance of COM from origin is C1 and C2). Co-ordinates of COM of this two-particle
________________ system are
r = ÷  
​ (0.64 a)2
+ (0.12 a)2
    ​ = 0.65 a = 0.65 × 10 y

= 6.5 m
L
3. Let the line joining the particles be x-axis
m1 D x1 + m2 D x2
D xcm = _______________
​     ​
   M C2
m1 + m2
CM
m (6) + 3m (– 2)
= ​ ______________
       ​= 0 C1
x
4m O M L
S.4  Mechanics II

L
M ◊ ​ __ ​  + M (0)
2
___________ L
xcm = ​       ​  = __
​   ​ 
2M 4


M (0) + M ​ __
____________
ycm = ​ 
L
(  )
​   ​   ​
2
     ​ 
L
= ​ __ ​  \  Mass of remaining plate = 3 m
2M 4
Let C ¢ be the COM of the remaining plate. We can
Distance of the COM of the system from point O assume a point mass m at C and another point mass
is 3 m at C ¢. These two masses together make a system,

÷(   ) (  )
__________
L 2 L 2 __ L L
d = ​ ​​ __ __    __ ​  = ____
​   ​   ​​ ​ ​ = ​÷2 ​ ​ 
​   ​   ​​ ​ + ​​   ​  __
   ​  whose COM is at O.
4 4 4 2​÷2 ​    Refer to example 9 and the note given thereafter.
8. Let mass per unit area of the plate be s. R R
3 m ◊ x = m ◊ ​ __ ​   fi  x = __
​   ​ 
Mass of segment ABCF is 2 6
m1 = s ◊ a2 L
11. 2R + p R = L  fi  R = _____  ​
​     
Mass of segment of OFDE is 2+p
m2 = s (2a2) Let linear mass density of the wire be l. Mass of
straight wire AB is m1 = l (2R). Mass of curved seg-
ment ACB is m2 = l (p R). We can replace AB with
a point mass m1, placed at its centre O. ACB can
be replaced with a point mass m2, kept at a distance
2R
___
​  p ​ from O.
m2
2R
p CM
r1
O
A B
We can assume a particle of mass m1 at the COM of
ABCF at ​ __
a 3a
(  )
​   ​ , ​ ___ ​   ​ and another particle of mass m2 at
2 2
Distance of COM from O is

COM of FDEO at ​ a, __ (  ) a
​   ​   ​.
2 r1 = ​ 
2R
0 + m2 ​ ___
__________
  
(  )
​  p ​  ​
 ​  
2R
l ◊ p R. ___
​  p ​ 
____________
= ​          ​
(  )
2 __ a m1 + m2 l ◊ 2R + l . p R
(s a ) ​ ​   ​   ​ + (2 s a2) (a)
2 5
\ xcm = ​ ___________________        = __
 ​ ​   ​  a 2R
3 s a 2 6 = _____
​      ​
2+p
5
Similarly, ycm = __ ​    ​ a
6 13. COM is closer to the heavier side.
​_›
9. Let mass of ADC be m. Mass of ABC is 2 m. COM ​_› d ​r ​ 1 
2R 15. (i) ​v  ​1  = ____
​      ​ = 3  + 4t 
of a half-ring is at a distance ​ ___ p ​  from centre O
dt
​_›
[Refer to example 5]. We can assume At t = 1.0 s, ​v  ​ 1 = 3  + 4  cms–1
two point masses as shown. Taking O as
origin and downward direction as posi- ​_›
​_› d ​r ​ 2 
____
tive x-direction, we can write ​v  ​2  = ​      ​ = – 6  cms–1.
dt
2R
(2m) ​ ___ (  )
__________________
       
2R
​  p ​   ​ + m ​ – ​ ___ (  )
p ​   ​ ​_›
​__› ​_›
m1 ​v​  1 + m2 ​v  ​ 2 2 (3  + 4  ) + 3 (– 6  )
xcm = ​ 
2m + m
​ \ ​v  ​ cm = ​ ___________    ​  = ​  ________________
 ​
     
m1 + m2 2+3
2R
= ___ ​    ​
3p – 12  + 8 
= ​ _________
 ​  = (– 2.4  + 1.6  ) cms–1
2R 5
Thus, COM is below the origin O, at a distance ​ ___ ​  ​_›
3p ​_› d​v  1 ​
from it. ____
(ii) ​a  ​1  = ​      ​ = 4  cms–2
10. Let mass of removed part be m. (Area of the removed dt
p R2 ​_›
part is ____
​   ​ 
).
  Mass of the original plate (i.e., plate ​_› d​v  2 ​
4 ​a  ​2  = ____
​      ​= 0
without hole) will be 4m ( its area is p R2) dt
Solutions  S.5

​_› ​_›
​_› m1 ​a  ​ 1 + m2 ​a  ​ 2 8  + 0
​a  ​ cm = ​ ____________    ​ = ​ ______
   ​  = 1.6  cms–2
m1 + m2 5
_
​› ​_›
(iii) ​F  ​ ext = (m1 + m2) ​a  ​ cm = (5) (1.6)  g cms–2

= 8  dyne
16. Refer to example 17. The COM of the rod will fall
on a vertical line. The centre of the rod will finally
be at the initial position of lower end A. Lower end ​_› ​__› ​_›
​_› m (​v  ​A  + ​ v​ B  + ​v  ​C  )
L
moves to A¢, such that A A¢ = ​ __ ​ . 20. ​v  ​cm
  = ​ _______________      ​  =0
2 3m
B

B¢ A A¢
L
Find position 2
of the rod.

17. On the system of five blocks, there is no external


​_› ​_› ​_›
force. Hence, motion of the COM does not change Since ​v  ​ A, ​v  ​ B and ​v  ​ C are vectors of same magnitude
due to collision. Velocity of the COM before and at separation of 120° from one another, their resultant
after collision remains the same. is zero.
mu + 0 __ u The COM will not move.
vcm = ​ ______
   
​ = ​   ​ 
5m 5 21. Initial co-ordinates of COM are (0, 0, 0). COM has
​_›
​_›
initial velocity ​v  0 ​ = (120 ms–1)  and it continues to
– mv    + mv  v
18. ​v  ​cm
  = ​ ___________ = ​ __  ​ (–  + ) collision will not
     ​  move with an acceleration – g = – (10 ms–2) 
2m 2
change the motion of the COM of the system, as After D t = 3 s
there is no external force. xcm = 120 × 3 = 360 m
1 1
ycm = – ​ __ ​  gt2 = – ​ __ ​  × 10 × 32 = – 45 m
2 2
zcm = 0,
Since there is no movement of COM in z-direction.
2 _​ ›
__ 3 _​ ›
__
​   
 ​ 
M  ​
r ​
  A + ​   ​  M ​r ​
  B
​_› 5______________ 5
​r ​ cm
  = ​         ​
M
2 3 ​_›
360  – 45  + 0  = __ ​   ​  (300  + 24  – 48  ) + __
​   ​  ​r ​  B
5 5
​_› 5 2
__ __
fi ​r ​  B = ​   ​  (360  – 45  ) – ​   ​  (300  + 24  – 48  )
19. Length of base of the wedge is 3 3
__ __
   ◊ ​÷3 ​
h cot 30° = 10​÷3 ​      = 30 m. = (400  – 91  + 32  ) m
22. If the hemisphere moves to the left by a distance
If block B moves by x to the right, then horizontal dis- x, the horizontal displacement of the block will be
placement of A to the left is h cot 30° – x = 30 – x (R sin q – x).
Since the system (A + B) has no horizontal force Since COM of the system will not experience any
acting or it, the COM will have no horizontal horizontal displacement,
displacement.
m (R sin q – x) = M ◊ x
mA xA = mB xB
mR sin q
fi x = _______
​    

fi m (30 – x) = 5m ◊ x  fi  x = 5m m+M
S.6  Mechanics II

Worksheet 1

1. Particles can be located in any direction from the


origin, hence rcm £ r. For example, if a large number
of particles are spread uniformly on the surface of a
sphere, rcm = 0.
2. AB is replaced by a point mass (m), placed at its
centre D. AC is replaced by a point mass (m), placed
at its centre E.
B
m
D

130°
A G

23. Mass of chain, M = l p R E


m
2R
Height of the COM from O is ycm = ___
​  p ​  C

\ COM of the system is at G, equidistant from D


2R and E.
\ U = Mg ycm = l . p R ◊ g ◊ ​ ___
p ​  __
l l l ÷    l
​ 3 ​
AB = __
​    ​  ; AD = __
​    ​  ; AG = __
​    ​  cos 30° = ___
​   ​   
= 2l gR2 2 4 4 8
3. Replace each brick with a point mass at its centre.
24. vrel = 5 – 2 = 3 ms–1

m1m2 4×2 4  ​ 2
l
m ​ __
2
l
​    ​   ​ + m ​ __ (  ) ( 
​    ​  + __
2
l

) ( 
​    ​   ​ + m ​ __
2
l
​    ​  + __
    ​
l l
​    ​  + __
4 )
​    ​   ​
2
m = _______  ​ = ​ _____ 
 ​ = __
( 

​ 
m1 + m2
  
4+2 3
​   ​  kg

xcm = ​ 
+ m ​ __
l
​    ​  + __
2 4 6 2
_______________________________
    
l l
​    ​  + __

​    ​  + __
l
​ ​   ​   )
  ​​
(  )
1 1 4 4m
\ kwrt cm = __ 2
​   ​  m vrel = __
​   ​  ​ __
​    ​  ​ (3)2 = 6 J
2 2 3 25l
= ___ ​   ​ 
24
25. Momentum of a system in COM frame is always
zero. 4. COM lies on the x-axis, as x-axis is the line of sym-
metry. Let s = mass per unit area.
Mass of circle = p r2 s ; Mass of square = 4 r2 ◊ s

(p r2 ◊ s) r + (4r2 ◊ s) (3r)
xcm = ​ ___________________
     
   ​
p r2s + 4r2 ◊ s

p + 12
= ​ ​ ______ ​ 
p+4
 ​ r (  )
​ 
1
m ​ __ (  ) (  ) (  ) (  )
1
​   ​   ​ + m ​ __
2 2
5. xcm = ___________________________
      
1
​   ​   ​ + m ​ __
2
3
​   ​   ​ + m ​ __
​    ​  ​
2
  ​ =
3
__
​   ​ 
4m 4
From symmetry, xcm = ycm = zcm
6. Distance of COM for H end is

0 + 1.27 × (35.5 m)
= _________________
​    
  
  ​Å
(35.5 + 1) m

This is a number slightly less than 1.27 Å
Solutions  S.7

7. Mass of each quarter piece – 1, 2, 3 and 4 – is m. 3R


fi x0 = – ​ ___ ​ 
COM of (1 + 2) is at O and COM of (3 + 4) is at 14
P. Thus, we can assume a point mass 2m at O and Similarly, for y co-ordinate.
another mass 2m at P. 12. If mass per unit length be l, then
 pRl (0) +  p Rl (0)  +
1  ​    ​
(  ) (  )
3 4   
2R 2R
O p Rl ​ ___
​  p ​   ​ + p Rl ​ ___
​  p ​   ​
     ​ 
R
2
P hcm = ​ ​ ____________________
  ​ = __
​ p ​
4 ◊ p Rl

Distance of COM from O is


0 + 2m (OP) ___ OP __ L  ​
L
m ​ __
2 (  ) ( 
L L
​   ​   ​  + m ​ __
​   ​  + __
5 2   ) ( 
​   ​   ​ + m ​ __
L L __
​   ​  + __
5 5 2
    ​
L
)
​   ​  + ​   ​   ​
xcm = ​ __________
(  ) (  )

     
​= ​   ​ 
 = ​   ​  L L L
2m + 2m 2 4 + m ​ __ ​   ​  + __ ​   ​   ​ + m ​ __ ​  ​ ​  
5 2 2
9. Let s = mass per unit area. Mass of the ring element 14. xcm = ​ ​_________________________________
         ​​
5m
shown is dm = s . p rdr.
23L
= ​ ____ ​ 
50
__ __
÷    l
​ 3 ​   l 
​÷3 ​
15. x ◊ (7m) = ___
​   ​ ◊ m 

  fi  x = ___
​   ​ 
2 14
16. Just before breaking, velocity of the projectile is hori-
zontal. Just after breaking the COM will still have
horizontal velocity only. It means the two pieces have
no vertical motion after the projectile breaks. They
will hit the ground at same time.
Mass of complete disc is 17. Since 60 kg mass (i.e, heavier mass) moves to the left,

R2 the plank (along with B) must move to the right so
s p
M = ​Ú ​   ​d​  m = s p ​Ú ​ ​   ​rdr = ___
​   ​  (R22 – R21) as to keep COM at the original position.
  R1 2
Let the displacement of plank (and B) be x towards
Úy dm 1
R2
2r
the right.
ycm = ​ _____​ =
    ​    ​ ​Ú ​ ​ ​ ​ __
__  ​  ◊ dm
M M R1 p Displacement of 40 kg mass = (x + 4) m towards the
right.
R2
1
     ​ 2s ​Ú ​ ​   ​ r2dr
= ​ ___________ Displacement of 60 kg mass = (4 – x) m towards the
s p 2
___ left
   2 – R21)
​   ​ (R R1
2
m1D x1 + m2D x2 + m3D x3 = 0
4(R32 – R31)
= ​ __________
  
    ​ \   (50 + 90) x + 40 (x + 4) – 60 (4 – x) = 0
3p (R22 – R21)
1
10. The COM of the original rectangle and the COM fi x = __
​   ​  m
3
of the removed part (unshaded rectangle) are joined. 18. ycm = 0
Extend this line. The COM of the remaining piece
m 3m
will fall on this line. ​ __ ​  × 15 + ___
​   ​ × y = 0  fi y = – 5 cm
4 4
11. Let mass of each removed circle be m.
Mass of the complete circle (without hole) = 16 m ; 19. Mutual interaction cannot change velocity of COM.
since its area is 16 times that of each small circle. v2 sin 2q 10 2
×1
20. Range = _______
​  g    ​ = ​ _______
   ​ 
 = 10 m
Mass of remaining piece = 14 m. 10
Now, xcm = 0 for the complete disc. The complete COM of the entire system (Platform + boy + stone)
disc is made of a 14 m mass having x-co-ordinate will not get displaced horizontally.
x0, a mass m having x co-ordinate 3R and a mass m \  (100 kg) (x) = (1 kg) (10 m)
having x co-ordinate 0.
\  14m ◊ x0 + m (3 R) + m (0) = 0. 1
fi  x = ___
​    ​  m = 10 cm
10
S.8  Mechanics II

​_› 2  + 2  Let displacement of B be x (¨), then displacement of


21. ​v  ​ cm = ​ _______ = ( + ) ms–1.
 ​  A will be (8R – x) towards right, for COM to remain
2
at fixed position
​_› + + 0 __ 1
​a  ​cm
  = ________
​   ​  = ​   ​  ( + ) ms–2. M ◊ x = 2M (8R – x)
2 2
​_› ​_› 16R
Since and ​v  ​ cm ​a  ​ cm
are in same direction, the COM fi x = ____
​   ​ 

will continue to move along straight line. 3
​_› ​_ ​_› 27. 3M ◊ x = M (2 – x)  fi x = 0.5 m

​_› ​F   ​net _______
M ​g  ​ + ​R  ​ 
22. ​a  ​ cm = ____
​      ​ = ​      ​  28. A completely filled shell as well as a completely
M M
empty shall have their COM at the geometrical
24. B remains fixed. centres.
\  A must move 60 cm to the right, to meet B. To R
29. COM is at a distance ​ __  ​ from the centre of the
keep COM at a fixed position, the plank moves 60 cm 2
earth.
to the left, as masses of B and plank are same.
w1(0) + w2(R) mg ◊ R
Thus, the right end of the plank comes at the location xcg = ​ ____________
   = ​ ______ 
 ​   
 ​ = R
w1 + w2 0 + mg
of B.
25. Horizontal force = 0. 30. In COM frame, momentum is always zero. _
​›
\  There is no motion of the COM in horizon- \  Momentum of the other particle must be – ​P  ​ 
tal direction. The block has acceleration in vertical 31. As the car A moves from P to Q (and B moves from
direction. Thus, COM has a vertical component of P to S) the COM moves from P to O.
acceleration.
Note that (M + m) g > N, Where N is normal force
by the ground and (M + m) is the total mass of the
two-block system.
26. Mass of B = M
Mass of A = 2M

2M M
R
R
mvy + mvy
A B vcm = ​ ________
   ​ 
= vy
2m
10 R
Where vy = velocity component of A and B along
y-direction.
certainly vy < v.
32. Friction on wedge will act rightward. Hence, COM
2R will move rightward. In vertical direction, there is
an unbalanced force on the system (as the block has
a vertical acceleration) Thus, COM has a vertical
motion also.
Solutions  S.9

see that the ball will stop when the string makes 30°
Worksheet 2 with vertical, as shown in second figure.
1. If mass density continuously increases or decreases, Let displacement of the plank be x (¨).
then COM will be on the heavier side.
Mx = m (1 ◊ sin 30° + 1 ◊ sin 30° – x)
2. F π O implies that acm π O
1
3. fi 4x = 1 (1 – x)  fi  x = __ ​   ​  = 0.2 m
5
A O B
B and C have equal and opposite momentum.
Thus mB vB = mC vC ...(i)
30°
If speed of either B or C is known, we can easily
find the speed of the other.
1 1 1 sin 30°
And ​ __  ​ mBvB2 + __
​   ​  mCvC2 = Q (Given) ...(ii)
2 2 A1
0
O1 B1
Solving (i) and (ii), will give vB and vC if Q is given.
If the decay is like A Æ B + C + D
​_› ​_› ​_› 30°
Then, ​P  ​B  + ​P  ​C  + ​P  ​D
  = O

The above equation


​_› ​_› has too many unknowns –
directions of ​P  ​ B and ​P  ​ C apart from their magnitudes x
​_›
will decide the value of ​P  ​ D
All unknowns cannot be found even if total KE is 6. In all cases, Fext = 0
known. \  COM will not move.
4. C is the COM of rod AB. Its co-ordiantes are ​ __
R R
2 2 (  )
​   ​  , __
​   ​   ​. 7. Net force on the (man + balloon) system is zero.
gravity is balanced by forces due to air (buoyancy).
Mass of ring is assumed at O and mass of rod is a
point mass at C. COM of the system is between O COM of the system will remain unmoved even when
and C on the line OC. the man climbs.
8. According to the question, the COM of the system
Also, xcm = ycm
moves in a circle of radius R.
Thus, (A) and (B) are possible co-ordinates of COM, \  Fext = Macm = 3m ◊ w2R
depending on masses of the ring and the rod.
y 9. acm = g (Ø)
B If v1 cos q1 = v2 cos q2, Then x component of velocity
of the COM will be zero. COM will rise and fall on
C a vertical line. COM can never move in horizontal
line since there is a vertical acceleration.
45°
x 1
O A 10. Ko = Kcm + __ ​   ​  mv2cm
2
Since vcm does not change in absence of external
1
force, the system always has __ 2
​   ​  mvcm (= Ko – Kcm)
2
5. When the ball stops, the plank must also stop so amount of energy in ground frame. Also, Kcm change
that the velocity of COM remains zero, in horizontal due to internal interactions.
direction. From conservation of energy, it is easy to
S.10  Mechanics II

2R
= _____
​      ​
Worksheet 3 p+4
2m ◊ R 2 (2l R) R
1. The COM lies on y-axis. An element, subtending an zcm = ​ _______  ​ = __________
  ​       ​
angle d f has length Rdf and its mass is M + 2m l p R + 4l R
4R
dm = l Rdf = _____
​      ​
p+4

÷ (  ) (  )
where l = linear mass density. y co-ordinate of this _________________ __
2R 2 4R 2 2​    R
÷5 ​
element is y = R cos f \ rcm = ​  ​​ _____
​      ​  ​​ ​+ ​​ _____
​      ​  ​​ ​ ​ = ​ ______ 
    ​
p+4 p+4 p+4
q /2
Ú y dm 3. Initially, AP = R tan 45° = R
–q
\ ycm = 2 l R 2 q /2 fi  COM of the rod is at P. Taking O at origin,
=
Ú cos f df the COM of the system of rod plus cylinder has x
l ◊ Rq
l Rq -q /2
co-ordinates given by
R q R sin q/2
= __
​   ​ ◊ 2 sin ​ __ ​  = _______
​   ​   m (0) + m R cos q
q 2 q/2 xcm = ​ ______________
    
​ 
2m
q
Check: when __
​   ​  Æ O ;  ycm Æ R = ____
R
​  __   ​   ...(i)
2   
2​÷2 ​
q p 2R
when __
​   ​  Æ __
​   ​  ;  ycm Æ ___
​  p ​  In position when the end B touches the cylinder, let
2 2 L
< B¢ O¢ A¢ be f; clearly, tan f = ​ __ ​ = 2. COM of the
R
rod (at P¢) is at a horizontal distance R sin f from
end A¢.

2. Mass of semicircle is M = l p R


It can be replaced with a point mass at C1 such that
2R
OC1 = ___
​  p ​ . Mass of each straight segment

m = l 2R
y

C1 M

x
O
C2 x co-ordinate of P¢ (with O as origin) is
2m
O¢ A¢ – x – R sin f (where x is displacement of
z cylinder)

We can assume 2m mass at C2 (COM of the two = R sec f – x – R sin f


straight segments) such that OC2 = R.
__ 2R 3R
Obviously, xcm = O = ÷     R – x – ___
​ 5 ​ ​  __ ​ = ___
​  __ ​ – x
  
​÷5 ​   
​÷5 ​
2R 2R
M ◊ ​ ___
p ​  l ◊ p R ___
​  p ​ 
_______
ycm = ​     __________
 ​ = ​        ​ COM of the system is still given by (i)
M + 2m l p R + 4l R
Solutions  S.11

R
\ ​ ____
__
3R
m ​ ___
  
​÷5 ​
  ​  ______________
   ​ =
( 
​  __ ​ – x  ​ – mx
       ​
) 6. (i) COM has initial velocity ucm = 4 ms–1 and it
continues to move with this velocity.
  
2​÷2 ​ 2m Displacement of man relative to platform = 4 m.
__ __
  )   R
    – ​÷5 ​
(3​÷2 ​ Time taken to move from one end to another is
fi x = ​ ___________
 ​ 
___   4  m
   
2​÷10 ​ t = ______
​    ​ 
= 2 s
2 ms–1
4. Hemisphere (shell) can be replaced with a point \  Displacement of COM of the system is
R
mass M, kept at a distance ​ __ ​  from O on the vertical D xcm = ucm ◊ t = 4 × 2 = 8 m.
2
symmetry line.
(ii) Let displacement of the platform be Dx
Displacement of the man will be (Dx + 4)
m1Dx1 + m2Dx2
Dxcm = ​ _____________
  
  
 ​
m1 + m2

25 (D x) + 75 (Dx + 4)
fi 8 = ​ __________________
  
 ​  
25 + 75
fi Dx = 5 m
The liquid is like a solid hemisphere and it can
be replaced with a point mass 2M at a distance Displacement of the man = 5 + 4 = 9 m
3R
___ (iii)  Dx = 5 m
​   ​ from O.
8 7.
COM of the whole system will stay at rest. Let x be
\  Distance of COM of the system from O is the leftward displacement of the car when all balls
have been collected into the bucket.

xcm = ​ 
3R
2M ​ ___ (  ) (  )
_______________  
R
​   ​  ​ + M ​ __
8
​   ​   ​
2
    ​ = ___
5
​    ​ R
Mass of man + car = M; mass of all balls = m.
Displacement of balls = (L – x) to right.
2M + M 12
mL L
5R 7R \  m (L – x) = M ◊ x  fi  x = ______​  ​ = ______
    ​       ​
Height of COM from the floor is = R – ___ ​   ​ = ___
​   ​  M+m M
12 12 1 + ​ __
m  ​

5. Let s = mass per unit area Obviously, x < L.
s r2
Mass of removed square, m = s l2 = ____
​   ​   8. Let the displacement of wedge be x towards
2 right. Displacement of B = (x – l) towards right.
r2
Mass of remaining disc, M = s ​ (p r2 – __
​   ​   ​
2 (  ) Displacement of A relative to wedge (on the incline)
is l.
(  1
= s ​ p – __ )
​   ​   ​ r2
2
\  Displacement of A in horizontal direction
= l cos q + x
Let COM of remaining disc be at a dis- y The COM of the entire system suffers no displace-
tance y from O, as shown. ment in horizontal direction.
If the square hole is refilled, the two m \ Dxcm = 0
masses m and M make a complete disc r
fi m1Dx1 + m2Dx2 + m3Dx3 = 0
having its COM at O. 2
O
r fi M ◊ x + m (x – l) + m (l cos q + x) = 0
\ My = m ​ __  ​  y
2 ml (1 – cos q)
M
fi x = ​ ___________
( 
      

1
fi s ​ p – __
2 ) s r2 __r
​   ​   ​ r2 ◊ y = ____
​   ​ ◊ ​ 
2 2

    ​ 

M + 2m
9. Consider a hemispherical shell of radius r and thick-
r r ness dr.
fi y = ________
​       ​ = ________
​       ​
(  1
__
)
4 ​ p – ​   ​   ​
2
2 (2p – 1) Mass of shell, dm = (2p r2 ◊ dr) r
r
= (2p r2 dr)ro ​ __  ​
​ 
r
\  Co-ordinates of COM are ​ 0, – ________
   
2 (2p – 1)
 ​  ​ (  ) = _____
2p  ro 3 
​     
R
​ r ◊ dr

R
S.12  Mechanics II

5L
and OQ = ___
​   ​ 
6

 ​
1 L
0.25m ​ __ (  )
​   ​  ◊ ​ __ ​  cos 30°  ​  +  1.25 m ​ __
2 6  
( 
1 5L
)
​   ​  . ​ ___ ​ cos 30°  ​
2 6
      ​
(  ( 
  

​ ​
L __
__ L
) 5L __
___ L
)
+ 2m ​ ​   ​  + ​   ​   ​ cos 30°  +  m ​ ​   ​ + ​   ​   ​ cos 30°
6 2
ycm = ______________________________________
     
6 2
   
  ​​
2m + 1.5m + m
__
r   
79​÷3 ​
COM of the shell is on y-axis at a distance ​ __  ​  from = _____ ​   ​ L 0.63 L
2 216
O. We can replace the shell with a point a mass on
r 11. COM of liquid in the cone is at a height
y-axis at y = ​ __  ​.  The entire hemisphere is made of 3h 3
2 ___
​   ​  = __
​   ​  × 4R = 3R from the apex.
infinite such shells. We can replace them all by point 4 4
masses on y-axis. Height of COM from the base of cylindrical container
is h1 = 3R + 2R = 5R.
Rr 2pro R 4 1 4
Ú dm
2◊R 0
Ú r dr Mass of liquid, m = __
​   ​  p R2 (4R) ◊ d = __
​    ​ p R3 ◊ d
02 2
\ ycm = = = __
​   ​  R 3 3
R 2pro R 3 5
Ú dm Ú r dr Taking base of cylinder as reference level, the PE of
0 r 0 liquid is
10. Let the mass x of the mid-segment lie on left slope 4 20p 4 
and mass (1.5 m – x) lie on the right slope. U1 = mgh1 = ​ __ ​  p R3 ◊ d ◊ g ◊ 5R = ​ ____
 ​ 
 R ◊ d ◊ g
3 3
Equilibrium is possible only when mass of rope on Height of liquid column in cylinder will be given
both sides is same. by
2m + x = m + 1.5 m – x 1
p R2 ◊ ho = __ ​   ​  p R2 (4R)
fi x = 0.25 m 3
4R
fi ho = ___
​   ​ 
3
ho 2R
\  Height of COM above base, h2 = __
​   ​ = ___
​   ​ . Final
2 3
PE of the liquid is
4 2R 8p
U2 = mgh2 = __
​   ​  p R3 . d . g . ​ ___ ​ = ​ ___ ​  R4 ◊ d ◊ g
3 3 9
Work done by gravity = Negative of change in PE
= – (U2 – U1) = U1 – U2

of mid segment.
1
Length OP has mass 0.25m, which is ​ __ (  )
​   ​   ​th the mass
6
20 8
= ​ ___
​   ​ – __
3
​   ​   ​ p R4 ◊ d ◊ g
9 (  )
L 52p 4 
\ OP = __
​   ​  = ​ ____
 ​ 
 R ◊ d ◊ g
6 9
Chapter 2 Momentum and Its Conservation

Your Turn Speed just after hitting the floor,


____ ___________
50   ​ = ​÷2  × 10 × 1.25 ​
1. Pbullet = _____
–1
​    ​ × 500 = 25 kg ms–1 v = ​÷2gh     = 5 ms
1000
Pcycle = 80 × 2.5 = 200 kg ms–1 Change in momentum of the ball is
[   9 kmh–1 = 2.5 ms–1] 50 3
|D P| = mu + mv = _____ (10 + 5) = __
​    ​   ​   ​  kg ms–1
1000 4
\ Pcycle > Pbullet
The smallest time for which the ball could have been
1 1 in contact with the floor is
2. k = __
​   ​  mv2 = ___
​    ​ (mv)2
2 2m
____ D t1 = 0.11 – 0.01 = 0.10 s
fi  
mv = ​÷2mK ​ D P

\  Maximum average force Fmax = ___
​     ​
____ D t1
fi P =  
​÷2mK ​

0.75
= ____


Since, k is same for both but m is larger for the ​   ​ = 7.50 N
0.1
truck, the truck will have higher momentum.
_
​› The largest time for which the ball could have been
3. (i) ​P  ​ i = m (u cos q)  + m (u sin q)  in contact with the floor is
​_›
At top, ​P  ​ f = m (u cos q) 
D t2 = 0.11 + 0.01 = 0.12 s
​___› _
​› ​_›
D P ____ 0.75
\ ​D P​   = ​P  ​ f – ​P  ​i  = – mu sin q \  Minimum average force, Fmin = ___
​    ​ = ​   ​ 
D t2 0.12
= mu sin q (Ø)
= 6.25 N
\  We can say that average force on the ball was
in range 6.25 N £ F £ 7.50 N.

or , F = (6.88 N ± 0.37 N)

​ where ​ __________ [ 
6.25 + 7.50
2
 ​
    = 6.88  ​ ]
dP
6. F = ___
​   ​ = slope of P–t graph.
dt
7. D P = mv (Æ) = – mv (¨) = 2mv (Æ)
_
​›
(ii) ​P  ​ i = m (u cos q)  + m (u sin q)  p R
​_›
D t = ___
​  v   ​ 
Just before landing, ​P  ​ f = m (u cos q)  – m (u sin q) 
​___› D P 2mv2
_
​› ​_› \  FaV = ___
​   ​ = _____
​      

\ ​DP​ =   ​P  ​ f – ​P  ​ i = – 2m u sin q ◊ D t p R
= 2mu sin q (Ø) 8. Mass of air incident per second = r (volume incident
per second) = r Av
4. (i) Pi = (1500 kg) × (15 ms–1) = 22500 kg ms–1 Change in momentum of the incident mass = (r Av) ◊ v
dp
Pf = (1500 kg) × (5 ms–1) = 7500 kg ms–1 per second. Thus force F = ___ ​   ​ = r Av2
dt
|D P| = 22500 – 7500 = 15000 kg ms–1 If speed is doubled, the force will become 4 times.
|D P| 15000 9. Refer to example 4. Mass of water accumulated in
Fav = ____
​      
​= ______
​   = 105 N
 ​  the container at time t is m t.
D t 0.15
1 \  Weight of water in container, W = m t ◊ g
(ii) Loss in KE = ​ __ ​  × 1500 (152 –52) = 1.5 × 105 J
2 Apart from this weight, the falling water also exerts
5. Speed before hitting the floor, a thrust force on the container.
____
____ _________ Speed of water hitting the container, v = ÷    
​ 2gH 
u = ÷   ​ = ​÷2  × 10 × 5 ​
​ 2gH    = 10 ms–1
S.14  Mechanics II

Mass of water hitting the container in unit If Jt = impulse applied by table on the chain, then
time = m. ____
  ​ (Ø)
Jt + Jg = 0  fi  Jt = – M ​÷2gL 
Water particle does not bounce back. ____
\  F = rate of change of momentum = M ​÷2gL      ​ (≠)
____ Same impulse is applied by the chain on the table in
  ​ 
= m ​÷2gH  downward direction.
____
\  Reading of scale = m ​÷2gH    ​ + m gt 16. Let the string apply an impulse J to both the balls
10. Gun exters force on bullets, causing their momentum so that both of them acquire same velocity after the
to change. Bullets exert equal force on the gun in string is taut. Let the final velocity of the two balls
opposite direction. The army person has to apply be v.
same force on the gun to hold it. For ball of mass m:
Force = (number of bullets fired per second) mv (Æ) – mu (Æ) = J (¨)
× (change in momentum of each bullet)
fi mv – mu = – J ...(i)
20 50
= ___
​   ​ × _____
​    ​ × 1000 = 250 N For ball of mass 2m:
4 1000
2mv (Æ) = J (Æ)  fi  2mv = J ...(ii)
12. Fnet = mg sin 30° = 10 N
2mu
\ J = Fnet ◊ t = 10 × 2 = 20 Ns From (i) and (ii): J = ____
​   ​ 

3
13. J = D P = Pf – Pi = mv – mu = 1000 (10 – 20)
= – 10000 Ns
Negative sign indicates that impulse is opposite to
the direction of motion.
14. Let the speed of the block immediately after the hit
be v.
Friction on the block, when it is moving is
f = m mg,
f
\  Retardation is a = __ ​ m   ​ = mg = 2 ms–2
Using v2 = u2 + 2as
0 = v2 – 2 × 2 × 1  fi  v = 2 ms–1

[  ]
It means that the stick imparts a velocity of 2 ms–1
2 2
to the block. ​_› ​_›
17. Impulse, ​J  ​   = ​Ú ​    ​​F  ​  dt = ​ 2 ​Ú ​    ​t dt  ​  = 4  Ns
\ J = mv = 10 × 2 = 20 Ns 0 0

[During the interaction time of the stick and the ​_› ​_› ​_› ​_› ​_› ​_›
​P  ​ f – ​P  ​ i = ​J  ​   fi ​P  ​ f = ​P  ​ i + ​J  ​ 
block, the impulse of friction is negligible.]
15. Change in momentum of the chain ​_›
fi 1 (​v  ​f ) = 1 (2  + 3 ) + 4 
D P = Pf – Pi = 0 – 0 = 0 ​_›
fi ​v  ​f  = (2  + 7  ) ms–1
Final state refers to the position when the complete
chain falls to the table. ​_› ​_› ​_› _
​› _
​›
18. ​P  ​ f = ​P  ​ i + ​J  ​  = m ​u  ​  + (m ​g  ​)   t

÷ 
___
2L
Time of fall t = ​ ​ ___
g ​ ​  
1
19. Impulse J = Area under F – t graph = __
​    ​ × 8 × 20
Impulse of gravitational force on the chain is 2
____
   
Jg = Mgt = M ​÷2gL 
​ (Ø) = 80 Ns
Solutions  S.15

u
Pi = mu = 4 kg ms–1 m __
​   ​ 
2 mu
fi v0 = ​ ______    ​ = ________
​      ​
Pf = Pi + J = 84 kg ms–1 M + m 2 (M + m)
Pi2 42 Note that x component of velocity of the bullet
Ki = ​ ___  ​ = _____
​      ​ = 8 J
2m 2 × 1 (wrt ground) is less than u cos 60°. Relative to the
ground, the bullet comes out of the barrel at an angle
Pf2 842
___
Kf = ​    ​ = _____
​    
 ​ = 3528 J higher than 60° with horizontal.
2m 2×1 23. Let the velocity of two blocks be vA (¨) and vB (Æ)
W = Kf – Ki  (WE theorem) after the spring opens completely.

= 3520 J
20. (i) Let the gun recoil with velocity v. Momentum
of gun + bullet system is conserved and remains
zero.
Momentum conservation gives:
mu
\  Mv = mu  fi  v = ___ ​   ​  2m vA = mvB
M
(ii) Energy released = KE of gun + KE of bullet fi 2vA = vB ...(i)
+ Heat + light + sound Energy is also conserved.
1
KE of gun + bullet = __
2 (  ) mu 2 1
​   ​  M ​​ ___
​   ​  ​​ ​ + ​ __ ​  mu2
M 2
1 1 1
​ __ ​  (2m) vA2 + ​ __ ​  mvB2 = __
​   ​  kx2
2 2 2

1
= __
2 [  ]
​   ​  mu2 ​ 1 + __
M
m
​   ​   ​
1 1
fi ​ __ ​  (2m) vA2 + __
1
​   ​  m (2vA)2 = ​ __  ​ kx2
​   ​  mu  ​[ 1 + __
2 2 2
​   ​  ]​
1 m
\ Energy released > __ 2

÷ 
___
2 M k
fi  6m vA2 = kx   fi  vA = ​ ___
2
​     ​ ​   ◊ x
21. Let velocity of bullet be vb (Æ) and that of the gun 6m

÷ 
___
be v (¨). k
And vB = 2 ​ ___ ​ ​  ◊ x
​      
6m
24. Total momentum after explosion = 0
fi  m (2  + 5  – 6  ) + m (– 4  + 3  + 2  )
​_›
+ 2m ​v  ​  = 0
Given u (Æ) = vb (Æ) – v (¨) _
​›
fi ​v  ​  = (–  – 4  + 2  ) ms–1
fi u = vb + v  fi  vb = (u – v) __________ ___
\  v = ​÷1  2 + 42   
+ 22 ​ = ​÷21 ​
   ms–1
Conservation of momentum gives
25. Sum of momentum of the three particles is zero.
mu
m (u – v) = Mv  fi  v = ______
​     ​ 26. Let velocity of the man relative to the ground be
M+m
vm and the velocity of the car relative to the ground
22. Velocity of shell wrt the gun is be vc.
__
​_› u ÷   
​ 3 ​
​u  ​   = u cos 60°  + u sin 60°  = __
​   ​  + ​ ___ ​  u  v (¨) = vm (¨) – vc (Æ)
2 2
Let recoil velocity of the canon be v0(–   ). Velocity fi v = vm + vc  fi  vm = v – vc (¨)
of shell relative to the ground is Momentum conservation gives:
__

( 
​_› _ ÷   
​ 3 ​
u
)
​›
​v  ​   = ​u  ​  + v0 (–  ) = ​ __
​   ​  – v0   ​ + ___
​   ​  u  M vc – mvm = (M + m) u
2 2
fi M vc – m (v – vc) = (M + m) u.
Applying conservation of momentum in x-direction,

( 
(M + m) u + mv mv
u
we get m ​ __ )
​   ​  – v0   ​ = Mv0
2
fi vc = ​ _____________
M+m
    ​ = u + ______
   ​ 
M+m
   ​
S.16  Mechanics II

mv 33. The ball moves towards B and the cart remains sta-
\  Change in velocity of the car is ______
​     ​
M+m tionary. Ball hits the cart (wall at B) and exchange
of velocity takes place. The ball comes to rest and
27. mvm = (M + m) u  fi  vm = ​ ______
M+m
​  m    ( 
​  ​ u ) the cart begins to move towards right with velocity
28. Refer to example 15. u. Now the wall at A comes from behind and hits
the ball.
There is no effect along the line of motion.
Time = time for ball to travel from A to B ( Cart not
29. (M + m) v = Mu [Conserving momentum in horizontal moving) +
direction]
Time for cart to travel a distance L (ball not
Mu
fi v = ______
​     ​ moving)
M+m
Mu L L ___ 2L
Change in velocity Dv = v – u = ______ ​      ​ – u = __
​ u ​ + __
​ u ​ = ​  u ​ 
M+m
mu 34. Let v1 and v2 be final velocities after collision.
= – ​ ______     ​
M+m
mv1 + mv2 = mv
30. (i) Let final velocity of the two objects, when slip-
ping stops, be v. Momentum of the system is fi v1 + v2 = v ...(i)
conserved
\ (M + m) v = mu  fi  v = ______ ​ 
M+m
mu
    ​
And ( 
v1 – v2
– ​ ​ ______ ​  
v–0 )
​ = e


1
2
1
(ii) KEi = ​ __ ​  mu2 ; KEf = __ ( 
mu 2
​   ​  (M + m) ​​ ______
2
​ 
M+m
   )
 ​  ​​ ​
fi v2 – v1 = ev
(1 + e) v
...(ii)

1 m2 u2 Solving (i) and (ii) v2 = ​ _______


 ​  and
= ​ __  ​ ​ ______ 

 ​ 2
2 M+m (1 – e)v
v1 = ​ _______
 ​  
1
Loss in KE = ki – kf = __
​    ​ mu2 ​ 1 – ______
2
m
​     
M+m
 ​  ​ [  ] 1 1
2
3 1
Given __
​   ​  mv12 + ​ __ ​  mv22 = __
​   ​  ◊ ​ __ ​  mv2

1 mM
= __
​   ​  ​ ______
2 M+m
​     ​  ​ u2 (  ) 2 2 4 2
(1 – e)2 2 (1 + e)2 v2 __
3
\  Work done by friction is ​ _______   + ​ ________
 ​ v
   ​  = ​    ​ v2

4 4 4

1 mM
2 M+m ( 
Wf = – ​ __  ​ ​ ______
​     )
 ​  ​ u2 fi  (1 – e)2 + (1 + e)2 = 3
1
31. When the smaller block is at P, the horizontal com- fi  2 + 2e2 = 3  fi  e2 = ​ __ ​ 
2
ponent of velocity of m is same as horizontal velocity 1__
___
of M. Using conservation of momentum in horizontal fi  e = ​    ​ 
÷   
​ 2 ​
direction, we get
35. (i) Relative speed of approach = 2 + 1 = 3 ms–1. Let
mu
(M + m) v = mu  fi  v = ​ ______    ​ velocity of ball be v (¨) after collision. Velocity
M+m of heavy wall does not change.
32. (i) In head on elastic collision of two bodies, veloc-
Relative speed of separation = Relative speed
ities get exchanged.
of approach
\ vA = 0 ;  vB = u
v – 1 = 3  fi  v = 4 ms–1
1
\ KA = 0 ;  KB = ​ __ ​  mu2 (ii) (Relative speed of separation) = 0.5 (Relative
2
speed of approach)
(ii) If collision is perfectly inelastic, final velocity
of both block is same (v) fi  v – 1 = 0.5 × 3
u v = 2.5 ms–1
2mv = mu ; fi  v = __ ​    ​ fi 
2

(  )
1 1 u 2 1 36. Using equation (13), velocity of first ball after colli-
\ Ki = ​ __ ​  mu2 ; Kf = __ ​   ​  (2m) ​​ __
​   ​   ​​ ​ = __
​   ​  mu2 sion is (Note: u2 = 0)
2 2 2 4
1
\  Loss in KE = Ki – Kf = __
​   ​  mu2
4
( 
m – eM
v1 = ​ ​ _______ ​  
m+M )
​ u
Solutions  S.17

For v1 to be negative, m < eM


m
fi ​ __ e ​  < M
5m
fi ​ ___ ​ < M
4
37. Let initial velocity of m be u. Using equation (13),
velocities of the two balls after collision are

( m – e (2m)
v1 = ​ ​ _________   
m + 2m
  )
​  ​ u =
u
__
​   ​ 
6
From conservation of momentum:
m2v2 = m1u

( 
m (1 + e)
v2 = ​ ​ ________ ​ 
m + 2m
  ) 5u
​ u = ___
​   ​ 
12
fi (  )
m1
v2 = ​ ___
​ m   ​  ​ u
2

e = – ​(​  ______  ​ )​ = – ​(​  _____ 


u – 0)
v –v
1 2 0–v 2
Time needed by second ball to hit the wall is \ u – u  
1 2
 ​  ​
d 12d
t1 = __
​ v   ​  = ____
eu = v   fi  eu = ​( ___
​   ​ 
​ m  ​  )​ u
5u m 1
2 fi 2
2
In this time, the first ball gets closer to the wall by m1
fi e = ___
​ m  ​   ...(i)
2d
x1 = v1t1 = ___
2
​   ​ 
5 Similarly for collision between m2 and m3
Now, after colliding with the wall velocity of second m2
5u e = ___
​ m  ​   ...(ii)
ball becomes v2¢ = 0.25 v2 = ___
​   ​  3
48 m1 m2 _____
Balls approach with relative speed From (i) and (ii), ___
​ m  ​ = ​ ___
m3 ​   ; Hence m2 = ÷
​m  1 m3 ​ 
2
u 5u 40. For B to rise upto a height of h = 3.2 m, the minimum
= v1 + v2¢ = __
​   ​  + ___
​   ​ 
6 48 speed of B must be
Time needed to cover distance x1 is ____ __________
  ​ = ​÷2  × 10 ×  
vB = ​÷2gh  3.2 ​ = 8 ms–1
2d
___
​   ​  Using equation (13) with u2 = 0
_______ 5 2.22d
t2 = ​  = _____
   ​  ​  u   
​ 
u ___
__ 5u (1 + e) m1u1 (1 + e) m × 10
​   ​  + ​   ​ 
6 48 vB = ​ __________
    ​  fi  8 = ​  ____________
     ​ 
m1 + m2 2 m
\  Distance from the wall where the collision occurs
3
is fi  e = __
​   ​ 
5
5u 2.22d
= v2¢ ◊ t2 = ​ ___ ​  ◊ ​ _____    ​ = 0.23 d
48 u 41. The line of impact (n-line) is z-axis. Velocity com-
ponent parallel to the wall does not change.
38. The ball is at the top point of its trajectory while it
hits the wall. It is travelling hori- \ vx = 2 ms–1
zontally and the collision is head
on. There is no impulse on the ball The z-component of velocity changes direction and
in vertical direction during impact. its magnitude is |vz| = e |uz|
Its vertical motion has no change.
Therefore, time of flight from A (i) |vz| = 3 [ e = 1]
to the wall (top of the trajectory ​_›
= time of flight from the wall to the ground). \ ​v  ​   = 2  – 3 
Velocity after impact = e ux = 0.5 ux (ii) |vz| = 0 [ e = 0]
x
\  Horizontal ground covered after impact = __
​    ​ ​_›
2 \ ​v  ​   = 2 
39. For collision between m1 and m2
u1 = u ;   u2 = 0 (iii) |vz| = 0.5 × 3 = 1.5 ms–1 [ e = 0.5]
​_›
and v1 = 0 \ ​v  ​   = 2  – 1.5 
S.18  Mechanics II

__
42. u1n = 2 u sin 60° = u ; u1t = 2 u sin 60° = ÷  u
​ 3 ​

u2n = u

Along the n-line, we have a head-on collision of two


equal masses. Velocities get exchanged.
\  Velocity of B after impact is u along n-line, as 3 10
fi  v × ​ __ ​   = 2  =  v = ___
​   ​  ms–1
shown. 5 3
Component of v0 along n line is
3
v0 sin q = 2 × __
​   ​  = 1.2 ms–1
5

\  (  (  )
10
e = – ​ ​ __________
0 – 10
    ​  )
1.2 – ​ – ​ ___ ​   ​
3
​ = 0.453

45. Y component of velocity does not change.


\  vy = u sin 30°
__
   u
Component of velocity of A are v1t = u1t = ​÷3 ​ And |vx| = e |ux| = e u cos 30°
v1n = u (in direction shown) vy 1
\  ​ __ __
vx ​  = ​ e ​ tan 30°
\  Velocity of A is
__________
__ 1 1
​ u  2 + (​÷3 ​
v1 = ÷      2
  u)  ​ = 2u fi  tan 60° = __
​ e ​ tan 30°  fi  e = __
​   ​ 
3
u 1
tan a = ____
​  __    
​= ___
​  __  ​   fi  a = 30°
÷    u
​ 3 ​  ÷   
​ 3 ​
\  Direction of v1 makes 90° with original direction
of motion
43. In elastic collision, angle
of incidence (a) = angle of
reflection (b)
Since a + b = 90°
\  a = b = 45°
46. Horizontal velocity of the projectile remains
\  q = 45° unchanged throughout. The vertical component of
velocity gets reduced at B.
44. The ball will move along the same line (n-line). Let
its velocity be v after impact, as shown. Momentum uy = |​ vertical component of velocity at A |​
conservation along horizontal direction gives: = ​| vertical component at B just before collision |​
m ux = Mv0 – mvx After collision at B; vy = euy
fi  1 (10 sin 37°) = 4 × 2 – (1) (vx) 2ux uy 2ux vy 2eux uy
Range: L1 = _____
​  g   ​  and  L2 = _____
  ​ = ______
​  g    ​  g   ​ 
fi  vx = 2 ms–1
L1 1
\  ​ __ ​   = __
​   ​
But  v sin q = vx L2 e
Solutions  S.19

47. Let velocity of the second piece be v, making an


angle q with x-direction as shown. Worksheet 1
Px = mu 1. Law of conservation of momentum
u
fi m ​ __ ​  + mv cos q = mu 2. Read the question carefully. It says ‘‘sum of
2
u magnitudes of momenta ...’’
fi  v cos q = __
​   ​   ...(i)
2 ​_› ​_› ​_› ​_›
And  Py = 0 When ​F  ​ext
  = 0 ; ​P 
 ​1  + ​P  ​2  + ​P  ​3  + ... = 0
​_› ​_› ​_›
fi 
u
mv sin q = m ​ __ ​   fi  v sin q = __
2
u
​   ​  
2
...(ii) |  | |  | |  |
But ​ ​P  ​ 1  ​ + ​ ​P  ​ 2  ​ + ​ ​P  ​ 3  ​ ... will be a positive number
Squaring (i) and (ii) and adding them if particles are moving. This sum may change also.
P2
(  )
1
v 2 = 2 ​​ __
u 2 u
​   ​   ​​ ​  fi  v = ___
​  __  ​  3. K = __
​   ​  mv2 = ___
​    ​ 
2 ÷   
​ 2 ​ 2 2m
Dividing (ii) by (i) gives tan q = 1  fi  q = 45° If KE is made 4 times, P becomes twice. It means
P increase by 100 %
P2
4. K = ___
​    ​ 
2m
dK P
fi ​ ___  ​ = __
​    ​
dp m
P
For small changes D k __ ​ m ​  D P

|  |
P D P
_____
dm D k ​  m    ​  2 D P
48. Fth = ​ u ​ ___ ​  ​ ___ _____
\ ​     ​ = ​  2  ​   = _____
​      ​

dt k P P
​ ___  ​ 
Rocket just lifts if Fth = mg 2m
dm
fi  ​ u ___
|  |
​   ​  ​ = mg
dt
D P
\ ___
P
1 D k
 ​ × 100 = __
​      ( 
​   ​  ​ ​ ___   ​ × 100  ​ =
2 k ) 1
__
​   ​  × (0.2%)
2
fi  ​ ___
dm
|  |
500 × 10
​   ​  ​ = ​ ________
dt 2000
= 2.5 kgs–1
 ​ 

5.
P
K = ___
2
​  fi  P = ÷
​       
​ 2mk 


= 0.1%
____
dv
49. m ◊ ​ ___ ​ =
  Fth – mg 2m
dt ________
P1 ÷ ​ 2  × 1 × k 
 ​ __ 1
fi  500 × 5 = Fth – 500 × 10 \ ​ ___ ​ =
  ​  _________
________   ​ = ​   ​ 
P2 ​ 2  × 4 × k 
÷  ​ 2
fi  Fth = 7500 N

|  | |  |
v
dm dm 7500 6. Angular speed w = __
​   . ​
\  ​ u ___
​   ​  ​ = 7500  fi ​ ___
​   ​  ​ = _____
​   ​ = 3.75 kgs–1 R
dt dt 2000 2R
Distance of COM from O is ___
​  p ​ 
50. velocity of falling sand in horizontal direction = u
(due to inertia). Linear momentum of train
\  Relative velocity of the mass getting separated
from the cart = 0.
2R
P = Mv0 = M ​ ___ ( 
​  p ​ w  ​ )
\  Thrust force on the cart = 0
\  Acceleration = 0
2R v
= M ​ ___
2Mv
( 
​  p ​  ◊ ​ __  ​  ​ = ____
R
​  p ​ 
  )
51. Refer to example 31 7. Speed just before hitting the floor,
dv dm ____ __________
m ​ ___ ​ =
  – u ​ ___ ​  u = ÷   ​ = ÷​ 2  × 10 ×  10 ​
​ 2gh 
dt dt
__
dm
___     ms–1
= 10​÷2 ​
fi  dv = – u ​  m ​ 
3
v m/2 On collision, it loses __ ​   ​  th of its KE. It means it has
dm
fi  ​Ú ​  ​dv = – u ​Ú ​  ​   ___
​​  m ​  1
4
0 m only ​​ __ ​​ ​th of its original KE. This is possible if speed
1 4
fi  v = – u ln ​ __ ​  = u ln2 = 2ln2 halves.
2
S.20  Mechanics II

u __
\ v = ​ __ ​  = 5​÷2 ​
    ms–1 12. Speed at top point, before explosion = v cos q.
2
Momentum conservation gives:
Impulse = change in momentum
m
mv cos q = 0 + __
​   ​  v1
= mv (≠) – mu (Ø) 2
= (mv + mu) ≠ fi v1 = 2v cos q
__
50 __ __   
3​÷2 ​ 13. Impulse (F D t) = change in momentum. when
= _____
​    ​ [5​÷2 ​   ]  = ____
    + 10​÷2 ​ ​   ​ 
 = 1.06 Ns impulse is added vectorially to initial momentum,
1000 4
we get the final momentum.
1 1 _
​› _
​› _
​› _
​›
8. D k = __
​   ​  m (v22 – v12) = __
​   ​  m (​v  ​ 2 – ​v  ​ 1) (​v  ​ 2 + ​v  ​ 1)
2 2
1 ​_› ​_› ​_› 1 ​_› ​_›
= ​ __ ​  D ​P  ​  ◊ (​v  ​ 2 + ​v  ​ 1) = __
​    ​ I ◊ (​v  ​ 1 + ​v  ​ 2)
2 2
9. After a collision, the ball travels a distance 2d for next
collision with the same wall. Hence, time intervals Sum of two vectors will be larger if angle between
between two successive collision with a wall is them is smaller.
2d \ P2 > P1
T = ​ ___
v0 ​ 
1 v0 14. Mv = 0 + (M – m) v1
Frequency of collision with one wall, n = __
​   ​ = ___
​    ​ 
T 2d
Mv
\ Force = (Change in momentum in one collision) fi v1 = ______
​     ​
M–m
× (frequency of collision)
15. Speed of the (block + bullet) combined system after
v0 mv02
= 2mv0 ​ ___  ​ = ____
​     
 ​ the hit is given by momentum conservation.
2d d u
2mv = mu  fi  v = ​ __ ​ 
10. Sum of momentum of particles moving in 2
2 2
v u
perpendicular direction is \ H = ___
​    ​ = ___
​   ​ 
____________ __ 2g 8g
P = ÷   2 + (mv)
​ (Mv) 2
   ​ = ÷     mv
​ 2 ​
16. Let the fragments have mass m each. Momentum of
Momentum of the third particle is exactly equal and each of the three fragments has magnitude.
opposite to P so as to keep the total momentum _____
zero.   o ​ 
P = ​÷2m E
If they move along x-, y- and z-direction then
​_› ​_›
P  + P  + P  + ​P  ​4  = 0  fi ​P  ​4  = – P ( + + )
__ _____
\ P4 = ÷    P =
​ 3 ​    o  
÷​ 6m E ​
P24
KE, K4 = ​ ___  ​ = 3E0
2m

Minimum energy released = sum of KE of the three \  Sum of KE of all fragments = 6E0
particles 17. Momentum conservation:
1 1 P2 (M + Nm) v0 = N mv
= __
​    ​ mv2 + __
​   ​  mv2 + ______
​     ​ 
2 2 2 (2m)
N mv
fi v0 = ​ _______
   ​
2m v 3 2 2 M + Nm
= mv2 + _____
​  ​ = ​ __ ​  mv2
  
4m 2 18. Change in momentum of each ball is
11. Momentum conservation along horizontal direction: D P = 2 mu cos 60° (≠)
100
(500 + 50) v = 500 × 10 fi v = ____
​   ​ ms–1 20
= 2 × _____
1
​    ​ × 5 × __
1
​   ​  = ___
​    ​ Ns
11 1000 2 10
Solutions  S.21

Similarly, Velocity after third impact, v3 = e3u


\  Height gained after third impact,
v32 e6 u2
h3 = ___
​   ​ = ​ ____   
​ 
2g 2g
e6 ◊ 2gh
fi  h3 = ______
​     ​ = e6 ◊ h

2g
Force on table F = n ◊ D P 25. let v0 = speed of (bag + sphere) combined system
1 after impact.
= 20 × ___
​    ​ = 2 N
10 Energy conservation gives:
where n = number of balls hitting per second ____
1
If N = normal reactions of each leg ​ __ ​  (M + m) v02 = (M + m) gh  fi v0 = ​÷2gh 
  ​ 
2
4N = F + W  fi  4N = 2 + 0.2 × 10 Momentum conservation for the collision event
gives:
fi  N = 1 N
19. When momentum of a system is zero, the particles
in it may be moving. The system can have KE. Your

M + m ____
mv = (M + m) v0  fi  v = ​ ​ ______
m    (   ​ 
​  ​ ​÷2gh 
  )
ceiling fan has no momentum but it has KE. 26. Speed of remains practically unchanged. Speed of
separation = speed of approach
When KE = 0, it means all the particles in the system
are at rest. fi  4 – v1 = 6 – 4
20. Final common velocity (v1) is given by fi  v1 = 2 ms–1
6mv1 = m (5v) + 5m (v) 1
27. KE before impact = __
​   ​  mu2
2
5v
fi v1 = ___
​   ​  1
3 KE after impact = 0.8 × __ ​   ​  mu2
2
Impulse applied by m on 5m = change in momentum
1 1
of the ball of mass 5m fi  ​ __ ​  mv2 = 0.8 × __​   ​ mu2
2 2
___
2v 10 mv
fi J = 5m (v1 – v) = 5m ​ ___ ​ = _____
​   ​ 
  fi     ◊ u = 0.89 u
v = ​÷0.8 ​ 
3 3
\  e = 0.89
21. Refer to example 25
​_› 28. In head-on elastic collision of two equal masses,
22. ​P  ​   = A   cos kt – A   sin kt there is exchange of velocity. If M2 is at rest, it will
​_› get entire KE of M1 and M1 will come to rest.
​_› d ​p   ​
​F  ​   = ___
​   ​ = – Ak   sin kt – Ak   cos kt 29. Momentum conservation:
dt
3m ​_› m
​_› ​_›
2 2
​ ___ ​ ​v  ​   = 2m  + __ ​   ​  
Now ​F  ​  ◊ ​P  ​  = – A k sin kt ◊ cos kt + A k sin kt ◊ cos kt = 0 2 2
4 1
23. Velocity of the heavy object will not change much. fi v = __ ​   ​   + __
​   ​  
3 3
Relative speed of approach = 10 + 2 = 12 ms–1

÷ 
______ ___
16 1 ____ ÷     –1
​ 17 ​
Relative speed of separation = 12 ms–1  [ e = 1] fi  v = ​ ___
​   ​ + __
​   
 ​ ​  = ​   ​ 
 ms
9 9 3
\  v – 2 = 12
fi  v = 14 ms–1
____
   
24. Velocity before first collision u = ​÷2gh 

Velocity after first collision, v1 = eu (≠)
The ball will return back for second collision while ____
moving with velocity v1 (Ø) 30. Speed before collision is u = ÷   ​ 
​ 2gh 
1
Velocity just after second impact, v2 = ev1 = e2u ____
  ​ 
Speed after collision is v = ​÷2gh 
2
S.22  Mechanics II

____ ____
DP = mv – (– mu) = mv + mu = m ​( ​÷2gh   2 ​ + ÷​ 2gh )​ Spring PE = loss in KE
  1 ​  
31. Force applied by falling particles on the scale is
1
2
1
​ __ ​  kx2 = ​ __ ​  mv12 + __
2
1
2
1
​   ​  mv22 – __ ( 
mv1 + mv2 2
​    ​ (m + m) ​​ ​ _________
2 m+M
  
  )
​  ​​ ​
F = (number of particles hitting per second) (change

÷ 
_________
in momentum of each particles) mM
Simplifying gives: x = (v1 – v2) ​ ​ ________    ​ ​

(M + m) 
1
= 100 × 2mu = 200 × _____ ​     ​  
u Alternate:
1000
____ _________ 1 1

1
= ​ __ ​  ​÷2gh 
    1
​ = __
​   ​  × ​÷2  × 10 × 2 ​  = 1.265 N K = __
​   ​  (M + m)v CM 2
+ __
​   ​  m v 2rel
5 5 2 2
1.265
\  Reading of the scale = _____ ​   ​ kg 
  = 0.1265 kg During maximum compression, kinetic energy is
10 1
simply __​   ​  (m + m) v CM 2
.
= 126.5 g 2
​_› ​_› ​_› ​_› ​_› ​_› 1 1
32. m1 ​v  ​ 1 + m2 ​v  ​ 2 + m3 ​v  ​ 3 = m1 ​u  ​ 1 + m2 ​u  ​ 2 + m3 ​u  ​ 3 \  ​ __ ​ kx2 = __
​   ​  m v 2rel
2 2
_ m2 _​ › m3 _​ › m2 _​ › m3 _​ › mM
kx2 = ______
​›
fi v1 = ​u  ​ 1 + ___
​ m  ​ ​u  ​ 2 + ___
​ m  ​ ​u  ​ 3 – ___
​ m  ​ ​v  ​ 2 – ___
​ m   ​ ​v  ​ 3 fi ​     ​ ◊ (v – v2)2 
1 1 1 1 m+M 1

÷ 
_________
= 20  + 2 (20 ) + 4 (20  ) – 2 (10  + 5  ) – 0 mM
fi x = ​ ________
​      ​ ​ (v – v2)

= 20  + 20  + 70  k (m + M) 1
____
36. Speed of m1 before collision is u = ÷   ​ 
​ 2gh 
33. rB = 2rA
Speed of combined mass after collision is
\  Volume VB = 8VA

÷ 
___

÷ 
____
h gh
\  Mass mB = 8 mA v = ​ 2g __   ​ ​ = ​ ___
​    ​   ​ ​  
4 2
If mA = m  then  mB = 8m Momentum conservation for collision:
Let velocity of A be v1 and that of B be v2 after m1u = (m1 + m2) v

÷ 
collision ___
____ gh
mu = mv1 + 8mv2  fi  u = v1 + 8v2 ...(i)   ​  = (m1 + m2) ​ ​ ___ ​ ​  
fi  m1 ​÷2gh 
2
v1 – v2
And – ​ ​ _____ 
u–0 (  )
 ​  ​ = 0.5  fi v2 – v1 = 0.5u ...(ii) fi  2m1 = m1 + m2  fi  m1 = m2
v
From (i) and (ii) 37. mv = (100 m)v0  fi  v0 = ____ ​     ​ 
100
h
v1 + 8v2 = 2v2 – 2v1  fi  3v1 = – 6v2 39. Time required by the body of mass m to fall through __
​   ​ 
2
is

 ÷ (  )
_____
v1 h
fi ​ __
v2 ​ =
  – 2 2 ​ __
÷ 
​    ​  ​ __
2
_____ h
t = ​ ​  g      = ​ __
​ ​  ​ g ​ ​ 
Negative sign tells us that v1 and v2 are oppositely
directed. The second body of mass 2m is projected with a

|  |
h
v1 velocity so as to reach height __ ​    ​ in time t
​ __
​ v  ​  ​ = 2 2
2 1
\  x = ut + __
​   ​  at2 gives
2
34. Impulse = ÚFdt = area under the graph.
÷ 
__

(  )
h 1 h ___
​ __ ​ =   u ​ __
​ g ​ ​  – __
​   ​  g ​ __
​ g ​  ​  fi u = ÷   ​ 
​ gh 
35. At maximum compression, both the blocks have 2 2
same velocity (Say v). h
Speed of this body when it is at height ​ __  ​ is given
Momentum conservation gives: 2
by
h
(m + M) v = mv1 + mv2 u22 = u2 – 2g ​ __ ​  = gh – gh = 0
2
mv1 + mv2 fi  u2 = 0
fi  v = ​  _________
  
 ​
m+M Speed of first body at the instant of collision is
Solutions  S.23

÷ 
____
h ___
u1 = ​ 2g ​ __ ​ ​  = ÷   ​ 
​ gh 
2
Velocity of combined mass after collision is given
as:
1 ___
3mv = mu1  fi  v = __   ​ 
​   ​  ​÷gh 
3
2un = 3vn
h
Now this mass falls through a height ​ __ ​  with accelera-
2 Where un = u cos q = velocity of 2 kg mass before
tion g. Its speed before hitting the ground is
collision along n-line.
h gh 10 gh
v02 = v 2 + 2g ​ __ ​  = ___
​   ​ + gh = ​ _____
 ​ 
  vn = velocity of 3 kg mass after collision
2 9 9 along n-line.
_____

​÷10gh  ​ 2
\  v0 = ______
​   ​   fi vn = ​ __ ​  un
3 3
40. Velocity of approach = component of u along n-line fi (Speed of separation after collision)
= u cos q 2
= __​   ​  (speed of approach before collision)
3
2
\  e = ​ __ ​ 
3
44. u1 = velocity of ball before collision
u2 = velocity of car

Velocity of separation = component of v along n-line


= v sin q
v sin q
\  ​ ______   ​ 
= e ...(i)
u cos q
Now, the velocity component along the incline does
not change. Relative speed of approach
v
\  u sin q = v cos q  fi ​ __
u ​ = tan q
u1 u2 __
= ___ ​  __  ​ + ___   
​  __  ​ = 4​÷2 ​
÷    ÷
​ 2 ​   
​ 2 ​
\  e = tan2 q
There is no change in velocity of the massive cart.
41. In elastic collision, speed will not change. Taking
If v1n = velocity component of the ball along n-line
help from the figure in last solution,
after collision, then
D P = 2mu cos q [along n-line] u2 __
v1n – ___   ) 
  e ◊ (4​÷2 ​
​  __  ​ =
____ ÷   
​ 2 ​
  ​  ◊ cos q
= 2m ​÷2gh  __ __ __
fi      + 2​÷2 ​
v1n = 4​÷2 ​     = 6​÷2 ​
  

42. Let velocity of ball 2 be v2 after collision Velocity component along the incline has not changed
for the ball
v 4
2mv = 2m ​ __  ​ + mv2  fi v2 = __ ​   ​  v u __
3 3 \  v1t = ___   
​  __  ​ = 2​÷2 ​

(  )
÷   
​ 2 ​
v
__ 4v
​    ​ – ___
​   ​ 
( 
_____________
__ __ __
Now
v1 – v2
_______
e = – ​ ​ u – u  
1 2
 ​  
3
​ = – ​ ​ )
______
v–0
3
 ​   ​= 1 \    ÷2 ​
v1 = ​÷(6​   )  2 + (2​   )  2 ​ = 4​÷5 ​
÷2 ​
       ms–1

45. z component of velocity has no reason to change.


\  collision is elastic Momentum conservation along x direction gives
43. m1 will move along t-line if it loses all its momen- MvAx + 2Mvx = 0  fi  vAx = – 2vx
tum along n-line. Conservation of momentum along ​_›
n-line \  ​v  ​ A = – 2vx  + u 
S.24  Mechanics II

1 1
46.
r/2 1
sin q = ___
​   ​ = __
​   ​  y = uyt + ​ __  ​ ayt2  fi  15 = 20 t – __
​    ​ × 10 × t2
2r 4 2 2
Exchange of velocity takes place along n-line. fi t2 – 4t + 3 = 0  fi  t2 – 3t – t + 3 = 0

\  Velocity of B = uAn fi t (t – 3) –1 (t – 3) = 0

÷ 
___
15 fi t = 1 s, t = 3 s
= v cos q = v ​ ___
​   ​ ​  
4 Obviously, our time is t = 1 s. (The projectile would
have been at same height at t = 3 s had there been
no roof.)
\  total time of flight = 2 s
\  x = ux × 2 = 30 m
52. Initial momentum = 0.
\  Both masses must come to rest simultaneously
47. Change in momentum of system of two particles so that momentum is zero. When they come to rest
= Impulse of external forces (together), the compression in the spring must be x0
= (m1 + m2) g (2t0) for energy to remain conserved.
10 × 14 + 4 × 0
48. vCM = ______________
​     = 10 ms–1
  
 ​
10 + 4
49. Velocity just before explosion = ux = 50 cos 53°
Radius of curvature of the path (R1) is given by

mu2x
​ ____ ​ = mg
R1
ux2
fi  R1 = __
​ g ​  Since, the COM stays at rest
After explosion, the velocity of moving fragment \  m1x1 = m2x2 ...(i)
= 2ux
And x1 + x2 = 2x0 ...(ii)
\  Radius of curvature of its path
Solving (i) and (ii) gives x1 and x2
(2ux)2
R2 = _____
​  g   ​ = 4R1 53. At the point of maximum compression, both M and
m will have same velocity.
50. From example 26 mu
\  (M + m) v = mu  fi  v = ______
​     ​
1 M+m
tan b = __
​ e ​ tan a
1 1
x ___ 1 (1.5 – x) ki = __
​   ​  mu2 ; kf = __
​   ​  (M + m) v2
fi ​ __ ​ =
  ​    ​ ​ _______
   
​  2 2
l 0.5 l
fi x = 3 – 2x  fi  3x = 3
1
= __
2
​ 
M+m
mu 2
​   ​  (M + m) ​​ ______
   ​  ​​ ​ (  )
fi x = 1.0 m
m2u2
= ________
​    
 ​
2 (M + m)

Loss in KE = spring PE (U) = ki – kf


1
= __
m
​   ​  mu2 ​ 1 – ______
2
​     
M+m [ 
 ​  ​ ]
1 mM
= __
​   ​ ​  _____
2 M+m
​     ( 
 ​  ​ u2 )
–1
51. ux = 25 cos 53° = 15 ms and uy = 25 sin 53°
U M
= 20 ms–1 \  ​ __  ​ = ______
​     ​
ki M + m
Time taken to reach the roof is given by
Solutions  S.25

54. Momentum conservation: \  Velocity of the particle relative to the man is


​_› ​_› ​_›
20 20
10 v + _____
​    ​ × 100 = _____
​    ​ × 500 ​v  ​21
  = ​v  ​2  – ​v  ​1  = – v2 sin (wt)  + [v2 cos (wt) – v1] 
1000 1000
fi v = 0.8 ms–1 \  Momentum of particle as observed by the
man is
The block travels 0.2 m and comes to rest. It’s retar- ​_› ​_›
dation is given by ​P  ​ 21 = m​v  ​ 21 = – mv2 sin (wt)  + m [v2 cos (wt) – v1] 
0.64
02 = (0.8)2 – 2a × 0.2  fi a = ​ ____ ​ = 1.6 ms–2 p R p × 5 __ p
0.4 58. Time of first collision is t1 = ___
​ u  ​ = ​ _____  = ​   ​  s
 ​ 
1 10 2
\  Friction = ma fi m mg = ma
For collision between A and B:
a
fi m = __
​ g ​ = 0.16 Speed of approach = 10 ms–1
After collision relative speed of separation
55. If B is very massive (i.e., a is large), A will rebound = e × 10 = 5 ms–1
with speed nearly equal to v0. It will hit the wall and
rebound back to hit B again. If, after colllison with B, \  Time (after first collision) for second collision
A has speed larger than B, then it will rebound back is
from the wall and will chase B with speed greater 2p R
t2 = ____
​   ​ 
 = 2p s
than B; hitting it again. 5
5p
In the critical case, speed of A, say v (¨) is equal to \  Required answer is t = t1 + t2 = ​ ___ ​  s
speed of B (equal to v (Æ)). In this case, A will not 2
be able to catch B after rebounding from the wall. 59. Due to impact, velocity normal to the incline is
For this critical case lost; velocity component parallel to incline does
v0 not change. Velocity after impact = v cos q (up the
a Mv – Mv = Mv0 fi v = _____ ​       ​ incline) Height to which particle rises is proportional
a–1
to its KE after impact; which decrease with increas-
Since e = 1
ing q.
\  speed of separation = speed of approach 60. Using equation (11), we get velocity of 1 and 2 after
collision as
fi  2v = v0
2v0
fi  ​ _____  
a–1
 ​ = v0 fi a – 1 = 2  fi a = 3
( 
m–M
v1 = ​ ​ ______ 
m+M ) ( 
M–m
 ​  ​ v = – ​ ​ ______ 
m+M )
 ​  ​ v

2mv
57. In time t, q = wt and v2 = ______
​    
 ​
m+M
Velocity of the particle is
_
​› After collision of 2 and 3, velocity of 3 is
​v  ​ 2 = – v2 sin q  + v2 cos q 

= – v2 sin (wt)  + v2 cos (wt) 
(  )
​ 
2M
v3 = ​ ______
m+M 2
   ​  ​ v

v = ​( ______ ​  ​ ​  ______


​_›

m + M ) ( m + M ) (m + M)
Velocity of the man ​v  ​1  = v1  2M 2mv 4m Mv
fi 3 ​      ​   ​  ​ = ​ ________
    
2
  ​

According to the question |v1| = |v3|


4m Mv
fi ​ ________

(m + M)2

M–m
 ​ = ​ ​ ______ 
m+M (  )
 ​  ​ v  fi 4Mm = M2 – m2

M 2
fi ​​ __ (  ) M
​ m ​   ​​ ​ – 4 ​ __ (  )
​ m ​   ​ – 1 = 0
______
M ± ​÷16
  + 4 ​
4__________   __
fi  ​ __
m ​   = ​   ​
    =2±÷   
​ 5 ​
2
M __
\  ​ __
m ​   = 2 + ÷   
​ 5 ​
S.26  Mechanics II

61. For conservation of momentum along x-axis Horizontal__ component of impulse received by the
2mv0 cos q = mv [v0 = speed of each piece]    . Same horizontal impulse is applied by
ball = m​÷3 ​
the ball on the wedge towards left. The back support
v
fi  v0 = ______
​     ​  applies equal impulse on the wedge towards right to
2 cos q prevent it from moving.
  cos q < 1 64. In first case, momentum will not change much in
x-direction during collision. Impulse of friction is
v
\  v0 > ​ __  ​ negligible.
2
In second case, there is impulse on the system in
62. vn and vt are velocity components perpendicular and
both x- and y-directions. The ground will impart a
parallel to AB. After collision, velocity is v1. Its com-
significant normal impulse during collision. Friction
ponents are:
will also be impulsive in nature as it depends on the
v1n = e vn normal force.
v1t = vt 65. As mass (snow) gets added to each cart, their velocity
will decrease to conserve momentum, in horizontal
direction. When Ram removes snow and throws it
sideways, it will not make any difference to velocity
of his trolley. [Refer to Example 14 and 15]
66. Relative velocity of the mass getting separated (in
horizontal direction) is zero.
dm
\  Thrust force, Fth = u ​ ___ ​ = 0
dt
Mass of wagon at time t is m0 – l t
​_›
d​v   ​ ​_›
\ (m0 – l t) ​ ___ ​  = ​F  ​ 
dt
Now v1n is actually parallel to BC. During second dm dm
67. Fth = u ​ ___ ​   fi 210 = 300 ___
​   ​ 
collision, it remains unchanged. dt dt
dm
v1t is normal to BC. fi ​ ___ ​ =
  0.7 kgs–1
dt
\  After collision,
v2n = e v1t = e vt 68. (5 × 103 + 103) v = 5 × 103 × 1.2  fi v = 1 ms–1
dm dm
And v2t = v1n = e vn. 69. Fth = u ​ ___ ​ = 980 ​ ___ ​ 
dt dt
\  Both v2t and v2n have become e times their
dv
original value, and their directions is reversed. M ​ ___ ​   = Fth – Mg
​_› dt
\  Final velocity = – e ​v  ​ 
63. vf = find speed of ball after collision. Since, velocity
dm
fi  4000 × 19.6 = 980 ​ ___ (  )
​   ​  ​ – 4000 × 9.8
dt
component parallel to the incline does not change
dm
__ fi  ​ ___ ​  = 120 kgs–1
\  3 cos 60° = vf cos 30°  fi vf = ÷    ms–1
​ 3 ​  dt
Solutions  S.27

Worksheet 2
2. If both come to rest it will violate law of conservation
of momentum. Hence, (A) is wrong.
If the heavy ball remains still and the moving ball
changes its velocity then also law of conservation of This is impulse applied by the floor on the ball.
momentum gets violated. Hence, (D) is wrong
3. In explosion, chemical energy will get converted into vy euy e ◊ u sin q
tan f = __
​ v  ​  = ___
​ u  ​ = _______
​   ​ 

= e tan q
KE, sound, heat, etc. Thus, final KE will be greater x x u cos q
than initial KE. ______ __________________
5. Let v be speed of the two blocks immediately after Speed, v = ​÷v  x2 + vy2 
 ​ = ÷  
​ (u cos q) 2
(e u sin q)2 ​
+   
the bullets hits block B. ____________________________
For B: 2mv – mu = – J   2 q + sin    
= u ​÷cos 2
 q + e2 sin2 q – sin2q ​
For A: 3mv = J ______________
u = u ​÷1  – (1 – e2) sin
  
2
 q ​
Solving the above two equation gives v = ​ __  ​ and
3mu 5
J = ____
​   ​ 
  1
__
​   ​  mv2
5 KEf 2 (u cos q)2 + (e u sin q)2
Impulse on ceiling is 2J = ____
6mu
​   ​ 
 because tension in ​ ____  
​ = _____
​   ​ = ​ __________________
     
 ​   
5 KEi 1
__ u2
​   ​  mu2
the string connected to ceiling is twice the tension 2
in the string connecting the blocks. = cos2q + e2 sin2 q
6. Momentum is always zero.
L 9. For first bead, we will use work-energy theorem
7. Time when block 1 hit 2 is t1 = __
​ v ​ 
to find its speed just before collision with second
bead.
____
1 2Fd
​ __ ​  mu2 = Fd  fi u = ​ ____
2
​  m   
 ​ ​  ÷ 
Since collision is head-on elastic and masses are
The two blocks stick and the common velocity equal, there will be exchange of velocities. The first
becomes: bead will come to rest after collision.
v Second bead begins to move at speed u, hits the third
v1 = __
​   
2 bead and comes to rest. Now bead 2 is at a distance
d from its original position and is at rest. In the mean
 ​ Time, after first collision, when second collision time, bead 1 gets accelerated due to force F and again
takes place is hits bead 2. Just before collision, speed of bead 1 is
L 2L again u. It will again come to rest. After this, it will
t2 = __
​ v   ​  = ___
​  v   ​ once again get accelerated for a distance d.
1

\  Total time when (n – 1) collision takes place Thus, bead 1 travels a distance d as its speed increases
th

is uniformly from 0 to u. ____


T = t1 + t2 + ... + t n – 1 0 + u __
Average speed = ​ _____
2
 ​ 
1 2Fd
2 ÷ 
 = ​   ​  ​ ____
​  u   
​ ​  
L L (n – 1) n 10. Speed of approach = u + v
= __ ​ v ​  [1 + 2 + 3 + ... + (n – 1)] = ​ _________
    ​ 
2v
Speed of separation = v1 – u
mv __
___ v
vcm = ​ nm  ​ = ​ n ​ where v1 = velocity of the ball after the impact.
Since impact is elastic
This remains constant throughout.
\ v1 – u = u + v  fi v1 = 2u + v
8. D P = mvy + m uy
D Pball = mv1 + mv = 2m (u + v)
= m e uy + m uy
2m (u + v)
= (e + 1) m uy = (e + 1) mu sin q \ Fav = ​ _________
    
D t
S.28  Mechanics II

1 1
D k = kf – ki = __​    ​ mv12 – __
​   ​  mu2
2 2
1
= ​ __ ​  m [4u + v + 4uv – u2]
2 2
2
1
= ​ __ ​  m [3u2 + v2 + 4uv]
2
11. When C hits A, exchange of velocity takes place; C
comes to rest and A acquires velocity u. At maximum
compression of the spring, both A and B will have
same velocity.

÷ 
_______
__
4v20
___ 2 ÷   
​ 7 ​
___
\ Speed of each bead = ​ u + ​   ​ ​
    = ​   ​ v0
9 3
13. Acceleration for first 3 s is,
15 ms–1
a = _______
​   = 5 ms–2
 ​ 
3 s
\  F = ma = 10 × 5 = 50 N
(m + m) v = mu
According to the question, a body of mass 10 kg
u
fi v = ​ __ ​  moving at 15 ms–1 collides head-on with a 25 kg mass
2 and sticks to it. Final velocity of combined mass is
\ 
1
2
u 2
KE = ​ __ ​  (2m) ​​ __
​   ​   ​​ ​ =
2 (  ) 1
​ __ ​  mu2
4
5 ms–1.
Let velocity of 25 kg mass before collision be v.
1 1 1
Loss in KE = ​ __ ​  mu2 – __
​   ​  mu2 = __
​    ​ mu2 10 × 15 + 25 × v = (10 + 25) × 5
2 4 4
This energy is stored in the spring as PE fi  v = 1 ms–1

÷ 
___ 14. (A) the sum of momenta of three particles must be
1 1 m
\  ​ __ ​  kx2 = __
​    ​ mu2  fi  x = ​ ___
​     ​ ​  ◊ u zero. In three-body case also, sum of momenta
2 4 2k
will be zero if the three momenta are in same
mv0 + mv0 ___ 2v0 plane.
12. vcm = ​ _________
     
​ = ​   ​  
3m 3 (B) Momentum of conservation and sum of KE
Let u be the velocity of beads relative to the hoop gives two equations. These can be solved in
just before collision. two-body case but cannot be solved in three-
In y-direction, the beads and the hoop will have same body case, as number of unknowns will be 3.
2v0 (C) In three-body case, particle can move in any
velocity equal to vcm = ___
​   ​  . direction in a plane. If we do not know direction
3
In ground frame, speed of each bead is of motion of some of these particles, there will
be too many unknowns.

÷ 
_______
_______ 4v02 16. Refer to equation (12)
​ 
= ÷ u2 + ​v​cm
2
​ ​​  =
   ​ u2 + ___
​   ​ ​



9
Energy conservation gives: (  ) m1 – m2
v1 = ​ ​ _______ 
 ​  ​ u
m1 + m2 1
1
2 (  4v02
​ __ ​  m ​ u2 + ___ )
​   ​  
9
1
  ​ × 2 + __
2 (  )
2v0 2 1
​   ​  m ​​ ___
​   ​    ​​ ​ = __
3
​    ​ mv02 × 2
2 = ​( ​ _______ 
m – 2m
m + 2m
 ​ )​ u = – ​ __ ​ 
u
3 1
1

v0
fi u = ___
​  __  ​ 
÷   
​ 3 ​ \ ​| v1 |​ < ​| u1 |​
Solutions  S.29

Loss in KE of first particle = __


1
2
1 u1 2​
​    ​ mu21 – __ (  )
​   ​  m ​​ __
2
​   ​   ​ ​
3
When ball from B to A reaches first, it decreases the
speed of trolley A. Now the ball projected from A will
1 8 carry less momentum (as it is travelling at smaller
= ​ __ ​  mu1 × __
2
​   ​ 
2 9 speed) and will cause less change in speed of B.
8
Fractional loss = __
​   ​  When ball from A to B misses the trolley, speed of
9 B remains unchanged. But speed of A decreases due
KE lost by first particle must be equal to KE gained to ball landing on it.
by the second particle, as the collision is elastic.
20. The COM of the system of box + bomb will not get
18. For A: mv1 = mu – J displaced.
J 21. When the rod is vertical, let the velocity of cart be
fi  v1 = u – __
​ m  ​ 
u (Æ). From conservation of momentum, in horizon-
For B: mv2 = J tal direction, velocity of bob must be 6u (¨).
Energy conservation:
J
fi  v2 = __
​ m   ​
1 1
2J
___ ​ __ ​  6mu2 + __
​   ​  m (6u)2 = mgl
v2 – v1 ​ m  ​ – 2J 2J 2 2
e = ​ _____
u    

​ = ​ ______ ​ = ___
u    ​ mu  ​ – 1 = ___
​   ​ – 1
P

÷ 
___
gl
fi u = ​ ___
​    ​ ​  
21

÷ 
___
7
\ Relative speed = 6u + u = 7u = ​ ___
​   ​ ​   
3

22. Due to recoil of the cannon, the horizontal component


of velocity of the cannon ball is less than vrel cos q.
But its vertical velocity is vrel sin q. Thus, actual angle
of projection is larger than q. [Refer to example 22].
Thus, the trajectory shown in (C) is possible.
19. When a ball falls on a trolley, it brings a momentum 23. Both observers are in inertial frames. They can
with it which is opposite to that of the trolley. Thus, use work energy theorem and impulse-momentum
the speed of the trolley decreases when a ball lands equation in their usual form.
on it.
R
When the balls are projected simultaneously, they 24. For l = ​ __ ​ , the projectile hits the wall while moving
2
have equal magnitude of momentum and will cause
horizontally. Its velocity gets reversed and it retraces
equal change in velocity of trolleys.
R
its path. For l < ​ __ ​ , the projectile will spend less time
2
in air before hitting the wall and will spend more
time in air after hitting the wall.
The overall time period does not change as collision
does not change the vertical component of velocity.
R
For l > ​ __ ​ , the projection will spend more time while
2
moving towards the wall.
\  while coming back it will travel a horizontal
distance < l.
\  It will fall between point P and the wall.
S.30  Mechanics II

u 3
D Pbullet = mv – mu = m __
​   ​  – mu = – ​ __  ​ mu.
Worksheet 3 4 4
3
1. Let the cross-sectional area of the beam be A. In = ​ __ ​  mu (¨)
time D t, all the particles inside the cylindrical volume 4
shown in figure will hit the surface. B exerts an impulse on the bullet and the bullet
exerts same impulse on the block towards right. This
impulse is
3
J = __
​   ​  mu [Ans. to (ii)]
4
For block B
u
Impulse due to rope + Impulse due to bullet = m ​ __ ​ 
4
3
__ mu
___
fi  JB + ​   ​  mu = ​   ​ 
4 4
Mass of particles inside the volume shown
1 1
m
= ​ __ ​  ◊ v D t = D m (say). This D m mass suffers a fi  JB = – ​ __ ​  mu = __
​   ​  mu (¨)
d 2 2
momentum change of 2D mv in time interval D t. 1
\  Impulse applied by B on rope = __ ​    ​ mu (Æ)
m 2
2 ◊ ​ __ ​  v D t ◊ v
D P _____
___ 2D mv __________
\  F = ​   ​ = ​ 
D t D t
​ = ​ 
   
d
D t
      
m
​ = 2 ​ __
d (  )
​   ​   ​ v 2 4. Only B moves (along y-direction) till the string
becomes taut. Position of A and B is as shown when
the string is about to get taut (fig. a) After the string
2. Refer to example 3 gets taut, both the particles move such that they have
Force on the roof = (Number of stones hitting per same velocity component along the length of the
second) × (change in momen- string.
tum of one stone)
Number of stones hitting the roof in 1 second,
n = 2000 × 100 = 2 × 105

Mass of one stone,


4
m = __
3
4
​   ​  p r3 ◊ d = __ (  )
0.5 3
​   ​  × 3.14 × ​​ ____
3
​    ​  ​​ ​ × 900
100
= 4.71 × 10– 4 kg
Change in momentum of one stone on hitting the
roof is A will move along AB¢ since it receives impulse in
D Pone = mv = 4.71 × 10– 4 × 20 this direction and originally it was at rest. Let its
velocity be v. Component of velocity of B along
= 9.42 × 10–3 kgms–1 thread = v. Let velocity of B perpendicular to the
thread be v¢.
\  Force, F = n D Pone = 2 × 105 × 9.42 × 10–3
= 1884 N
3. Let the final velocity of the system be v. From
conservation of linear momentum
u
4mv = mu  fi  v = __​   ​ 
4
Block A is set into motion due to the impulse it
receives from the rope.
mu
\  JA = mv = ___
​   ​   [Ans. to (iii)]
4
Impulse applied by B on the bullet is reason behind
change in momentum of the bullet.
Solutions  S.31

Since B does not receive any impulse perpendicular 1 1 1


​ __ ​  (5) v2 + __
​   ​  (1) u2 = __
​    ​ k x02
to the string, its velocity will not change in that 2 2 2
direction.
fi 5v 2 + 25v 2 = 100 (1)2  fi  30 v2 = 100
u
\ v ¢ = u cos q = __
​    ​

÷ 
___
2 10
fi v = ​ ___
​   ​ ​  ms–1
Applying momentum conservation along the thread. 3
mv + mv = mu sin q  fi  2v = u sin 60° 7. Time of flight of the ball after impact is

÷ 
__ ___
÷   
​ 3 ​ 2h
fi v = ___
​   ​  u T = ​ ___
​  g ​ ​  
4 __
÷   
​ 3 ​ If velocity of ball after impact is v1 (¨), then
Velocity of A is vA = ​ ___ ​ u in a direction making 60°
4 d = v1T
with x.

÷ 
___
________ d g
fi v1 = ​   ​ = d ​ ___
__ ​    ​ ​  
​ v  2 + (v¢)2 
Velocity of B is vB = ÷  ​ T 2h

÷(   ) (  )
___________
__ __ Let v2 (Æ) be velocity of wedge after impact.
   2
​÷3 ​ 1 2 ÷   
​ 7 ​
=​ ​​ ​ ___ ​  ​​ ​ + ​​  
​ __ ​   ​​ ​ ​ ◊ u = ​ ___ ​ u Momentum of ball + wedge system is conserved in
4 2 4 horizontal direction during the collision. Note that
__
÷   
​ 3 ​ spring is relaxed and during the short interval of
Impulse on A is J = mvA = ​ ___ ​ mu making 60° with collision, it does not apply any impulse.
4
x direction.
\ Mv2 = mv1

(  ) ( 
__ __

) ÷ 
_
​›
___
  
​÷3 ​   
​÷3 ​ md g
​J  ​   = ​ ___
​   ​ mu cos 60°  ​  + ​ ___
​   ​  mu sin 60°  ​  fi v2 = ​ ___ ​ ​ ___
​    ​ ​  
4 4 M 2h

( 
__
÷
= ​ ___
  
​ 3 ​ 3
​   ​ mu  ​  + ​ __
8 ) (  )
​    ​ mu  ​ 
8
The wedge compresses the spring and comes to
rest when the spring is fully compressed. Let the
maximum compression be x. Conservation of energy
5. Let impulses by the wall and the floor be J1 and J2 gives:
respectively. 1 1
​ __  ​ Mv22 = ​ __ ​  k x2
Jx = J1 sin q (Æ) 2 2

÷  ÷ 
___ ______
M g
 v
fi m ​ __ ​  (Æ) – mv (¨) = J1 sin q (Æ) fi x = ​ ​   ​ ​   ◊ v2 = md ​ ______
__ ​      ​ ​ 
2 k 2h k M
8. Refer to example 16
v
fi m ​ __  ​ + mv = J1 sin q After the man jumps out, velocity of the rear car will
2
mv
3 mv be v1 = u – ______
​      ​
fi ​ __ ​  ​ ____  ​ 
= J1 M+m
2 sin q Total momentum of the two car + man system is
conserved. If v2 is velocity of front car + man after
he lands into it, then

(M + m) v2 + Mv1 = (2M + m) u.


Mmv
fi (m + m) v2 = (2M + m) u – Mu + ______
​    
 ​
M+m
Net impulse in y-direction should be zero.
Mmv
fi v2 = u + ________
​      ​
3 mv
\ J2 = J1 cos q = __
​   ​  ____
3
​    ​ cos q = __
​   ​  mv cot q (M + m)2
2 sin q 2
9. The bob of the pendulum hits the wall at B with
6. Let velocity of A be v (Æ) and that of B be u (¨) at a horizontal velocity (v) given by conservation of
the moment they get separated. Momentum conserva- energy.
tion gives: ___
1
​ __ ​  mv 2 = mgl  fi  v = ÷   ​ 
​ 2gl 
5v = 1 ◊ u  fi  5v = u ...(1) 2
Energy conservation gives: The bob rebounds with speed
S.32  Mechanics II

Let velocities be v1 and v2


After collision in positive direction (see figure).

(  ) (  )
m1 – m2
v1 = ​ ​ _______ 
m1 + m2 1
​ 
2m2
 ​  ​ u + ​ _______  
 ​  ​ u
m1 + m2 2

= (​  ​ _______    ​) + (​  _______
m + 2m ) ÷ m + 2m ) ÷
m – 2m ____ 4m ____
 ​  ​ (– ​ 2gH  ​    
    ​)
 ​  ​ (​ 2gH   
___
  ​ 
v1 = ev = e ​÷2gl  5 ____
= __
​   ​  ÷   ​ 
​ 2gH 
3
Now the bob rises to some height h and then again
comes back to hit the wall at B. Before hitting,
its velocity is v1 (¨) and after the second hit, it
becomes
​  ( 
2m1
Similarly v2 = ​ _______ ) ( 
m2 – m1
 ​  ​ u + ​ ​ _______ 
  
m1 + m2 1
 ​  ​ u
m2 + m1 2 )
1 ____
___ = – ​ __ ​  ​÷2gH 
  ​ 
v2 = ev1 = e 2
  ​ (Æ)
​÷2gl  3
The process continues and the speed of bob after nth Negative sign tells us that v2 is directed to left. Height
hit is raised by bob of mass m is given as
___
vn = en ​÷2gl   ​  mg H1 = __
1
​   ​  mv12
If the bob rises to a height hn after n collisions, 2
then 25
fi H1 = ___
​   ​ H
1 9
mg hn = ​ __ ​  mv 2n
2 H
Similarly, H2 = __ ​   ​ 
vn2 e2n ◊ 2gl 9
fi hn = ​ ___ ​ = ​ _______​ = l ◊ e2n
   
2g 2g 11. (i) Let velocity of combined mass (bullet + block)
be v just after the collision.
\ (  ) (  )
2 2n
hn = ​​ ___
  
​÷5 ​
4 n
​  __  ​  ​​ ​ l = ​​ __
​    ​  ​​ ​ ◊ l
5
Conservation of momentum along horizontal
direction gives
l–h mu
From geometry, ​ ____  ​ = cos q
    (M + m) v = mu  fi  v = ______ ​    
 ​
l M+m
fi 1 – ​ __ ​ =
  cos q Loss in mechanical energy during collision
l is same as loss in KE in the context of the
4 n
fi 1 – ​​ __ (  )
​   ​   ​​ ​= cos q
5
question.
1 1
1 \ loss = ​ __ ​  mu2 – __
​   ​  (M + m) v2
For q < 60°, cos q > __
​   ​  2 2
2
4 n
fi 1 – ​​ __ (  )
​   ​   ​​ ​ >
1 4 n
​ __ ​   fi ​​ __ (  )
​   ​   ​​ ​ <
1
​ __ ​ 
1
= __
2
1
​   ​  mu2 – __
2 M+m
mu 2
​   ​  (M + m) ​​ ______
​     ​  ​​ ​ (  )
5 2 5 2
Mmu2
= ________
​    
 ​
This condition is satisfied for n ≥ 4 2 (M + m)
\ Minimum number of collisions needed = 4 (ii) For completing the circle
____
10. Speed of the two balls just before collision is   ​ 
v ≥ ​÷5gL 
____
u1 = u2 = ÷   ​ 
​ 2gH 
(  )
mu ____ + m ____
fi ​ ______
     ​   fi  v ≥ ​ ​ M
 ​ ≥ ​÷5gL  ______
m      ​ 
​  ​ ​÷5gL 
M+m
12. The vertical component of velocity does not change
due to impact. Hence, total time of flight from A to
2uy 2u sin q
B to C to A is T = ___
​  g   ​ = _______
​  g    ​ 
While travelling from A to B, the horizontal com-
ponent of velocity of the ball is u cos q and while
moving back from B to A it is eu cos q.
Solutions  S.33

\ tA Æ B + tB Æ A = T 14. It will be useful to refer to example 23.


t-line is line along the floor and n-line for each
d d 2u sin q
​ ______ + _______
   ​  ​  = ​ _______
   ​   g    
​ impact is vertical (along y).
u cos q eu cos q
Horizontal component of velocity remains constant
d
_______ 2u sin q ______ d = ux = u cos q. Vertical component of velocity:
fi ​  = ​ _______
   ​   g   ​ – ​     ​ 

eu cos q u cos q   After first collision, v1 = e uy = e u sin q
1 = e2 u sin q
fi e = ​ __________
2
     ​     After second collision, v2 = ev1
u  sin 2q
_______   After nth collision, vn = en u sin q
​     

​ –1
d ◊ g
Total time of flight
13. Velocity at top is
2
2u sin q 2e u sin q 2e  u sin q
ux = 24 cos 60° = 12 ms–1 T = ​ _______
g    

​ + ​ ________
g    

​ + ​ ________
g    ​ + ... •

terms
Velocity of one fragment after explosion is vertically
up. Let it be v2. 2u sin q
= ​ _______
g    ​ [1 + e + e2 + ... + • terms]

2u sin q _____
= _______
​  g   
1
​ ​ ​     

1–e (  )
 ​  ​   [Ans. to (ii)]

Total horizontal displacement is

x = ux ◊ T = ​ ________
g    (  )
u2 sin2 q _____ 1
​ ​ ​     
1–e
 ​  ​  [Ans. to (i)]

F or conservation of momentum in vertical direction, Launch angle after nth collision is given as:
the other fragment must have a downward component vn
of velocity equal to v2. tan qn = __
​ u  ​  fi  tan qn = en tan q
x
Let horizontal component of velocity of the second
fi qn = tan–1 (en tan q)
fragment after explosion be v1.
For momentum to be conserved along horizontal 15. (i) Let velocity of men after jump be vm (Æ) and
direction that of car be vc (¨)
u (Æ) = vm (Æ) – vc (¨) fi vm = u – vc.
mv1 = 2m ux  fi  v1 = 24 ms–1
Momentum conservation gives:
According to the problem AB = 45 m
2mu
2m (u – vc) = mvc  fi  vc = _______
​     ​
fi 24 t = 45 2m + M
(ii) When only one man has jumped, let velocity of
15
Æ t = ​ ___ ​  s = 1.875 s car be v1 (¨).
8
Velocity of man (who has jumped) is vm = u – v1
u2 sin2 60° 242 × 3
H = ​ _________
    ​ = ​ _________
   
  ​ = 21.6 m Momentum conservation gives:
2g 2 × 10 × 4
mu
For second fragment, time of flight is t = 1.875 s m (u – v1) = (M + m) v1  fi  v1 = ​ _______
    ​ (¨)
M + 2m
1
\ v2 × 1.875 + __
​   ​  × 10 × 1.8752 = 21.6 Now, the car is moving with velocity v1. The
2 second man jumps out and its velocity changes
fi v2 = 2.55 ms–1 to v2 (¨).
v¢m = u – v2 (Æ)
\ Velocity of 1st particle = v2 = 2.55 ms–1
Momentum conservation gives:
Velocity of second particle
______ __________ (M + m) v1 (¨) = Mv2 (¨) + m (u – v2) (Æ)
​  +
=÷ v12 v22 
 ​ =   2 + 2.55
÷​ 24   
2 –1
 ​= 24.1 ms
fi (M + m) v1 = Mv2 – m (u – v2)
S.34  Mechanics II

fi (M + m) v2 = mu + (M + m) ​ _______
​ 
mu
M + 2m
  
 ​  ​ (  )
mu mu
fi v2 = ​ ______
    ​ + _______
​      ​
M + m M + 2m
16. Collision of C with A and that with B is identical.
Both A and B will move along LOI with same
speed.
Mv0 = mvx ...(i)
Momentum (for the system of three discs) is con-
served along any direction. We will conserve it along Velocity component of the ball along the t-line
x direction. (parallel to incline does not change).
From simply geometry, we can find angle between u \ vx cos q – vy sin q = u sin q ...(ii)
and LOI.
D C1 C2 C3 is equilateral
\ q = 30°
And ( 
v1n – v2n
e = – ​ ​ _______  
 ​  ​
u1n – u2n  )
fi ​ ( 
vx sin q + vy cos q + v0 sin q
1 = – ​ ______________________
   
– u sin q – 0
 ​ 
   ​ ) ...(iii)

2mu
Solving (i), (ii) and (iii) gives: v0 = _______
​     ​
m + 2M
From energy conservation:
1 1
​ __ ​  kx2 = __ ​   ​  Mv02
2 2

÷  ) ÷ 
___ ___

Momentum conservation along x direction gives fi


M
x = ​ __
k ( 
​ 
2mu
​   ​ ​  v0 = ​ _______
m + 2M
 ​  
M
  ​ ​ __
​   ​ ​  
k
2mv cos q = mu 18. Retardation of 2 kg block while going up is
u u u a1 = g (sin q + m cos q) = 10 (0.05 + 0.25 ×1)
fi v = ​ ______ = _____
   ​  ​   __   ​ = ___
​  __  ​ 
2 cos q     ​÷3 ​
​÷3 ​   
2 ___
​   ​  = 3 ms–2   [ cos q  1 for small angle]
2
Velocity of 2kg mass just before collision is given
( 
vcn – van
For A and C: e = – ​ ​ ________
ucn – uan  
​   ​ ) by
v02 = 102 – 2a1s = 100 – 2 × 3 × 6 = 64
___u
​  __  ​ 
0–v
= – ​ ​ _________
  
u cos q – 0 ( ÷   
​ 3 ​
 ​  ​ = ____
2
​  __  ​ = __
÷    3
​___
3 ​
u ​   ​ 
​   ​  ) \ v0 = 8 ms–1
Its acceleration while coming down is
2
a2 = g (sin q – m cos q) = 10 (0.05 – 0.25) = – 2 ms–2

1 1
( 
Loss in KE = ​ __ ​  mu2 – 2​  ​ __ ​  mv2   ​
2 2 ) Negative sign shows that the block retards.
Let its velocity after collision be v1 (down the
1
2 ÷
u 2 1
= ​ __ ​  mu2 – m ​​ ___
​  __  ​  ​​ ​ = __
  
​ 3 ​ 6 (  )
​   ​  mu2 plane).
Then 12 = v12 + 2a2 ◊ s
1
__ fi v12 = 1 + 2 × 2 × 6  fi  v1 = 5 ms–1
​   ​  mu2
6 100
% loss = ​ ______   ​ × 100 = ____
​   ​ 
 = 33.3% Let velocity of M after collision be v2 (up the
1
__ 2 3 plane).
​    ​ mu
2
02 = v22 – 2 × a1 × (0.5) fi v22 = 2 × 3 × 0.5 = 3
17. Let velocity of wedge be v0 (Æ) after collision. __
Horizontal and vertical velocity components of the     ms–1
fi v2 = ​÷3 ​
ball after collision are vx and vy as shown. Relative speed of approach = v0 = 8 ms–1
__
Momentum conservation along x direction gives: Relative speed of separation = (5 + ÷    ) ms–1
​ 3 ​
Solutions  S.35

will be no change in vertical motion of the balls.


Their time of flight will not change.

2u sin 30° ________
50 50
\ ​ ________
g    ​  = ​  + _________
   ​  ​     ​ 
u cos 30° e u cos 30°
  fi u = 37.5 ms–1
__
5+÷   
​ 3 ​
\ e = ​ ______
 ​    dv dM
8 21. M ​ ___ ​ =
  – u ___
​   ​ 
Momentum conservation gives: dt dt
m t
__ 26
    – 2 × 5 = 2 × 8  fi  M = ___ dM dM a
M​÷3 ​ ​  __  ​ kg fi ​   ​   fi ​Ú  ​ ___
M a = – u ___ u ​ ​Ú0 ​  ​ dt
​  ​ = – ​ __
÷   
​ 3 ​ dt M0 M

(  )
1
___
19. (i) In figure, sin q = ​    ​ = 0.42. Let the critical M at
2.4 fi ln ​ ___
​    ​  ​ = – ​ __
u ​ 
M0
allowed velocity be u0. As the string gets taut,
the velocity component of ball along the length at
__
of thread becomes zero. fi M = M0 ​e– ​ 
​ u ​ ​

22. Let v = velocity of wagon at time t


m = mass of wagon at t = m0 + bt
Relative velocity of mass being added; u = 0 – v (Æ)
= v (¨)


dm dm
Fth = u ​ ___ ​ = bv (¨) ​ ___
dt [ 
​   ​ = b is positive  ​
dt ]
This happens due to impulse J applied by the \ Net force on the wagon = F + Fth
string on the ball. dv
m ​ ___ ​   = F – bv ...(i)
\ J = mu0 cos q dt
J 3 dv
fi u0 = ______ = ​ ______
​     ​      ​ fi (m0 + bt) ___
​   ​   = F – bv
m cos q 2 × 0.9 dt
_________ v t
​ 1  – (0.42)2 
[   cos q = ÷  ​ = 0.9] dv
fi ​Ú  ​ ​ ​ ______
     ​ 
dt
​ = Ú​  ​  ​ _______   ​
0 F – bv 0 m 0 + bt
1.65 ms–1
1 1
(ii) Velocity component perpendicular to OA¢ fi – ​ __ ​ ​​[ ln (F – bv) ]v​​0 ​​  = __
​   ​ ​​[ ln (m0 + bt) ]t​​0 ​​ 
remains unchanged. b b

) (  )
\ After the string gets taut, velocity of ball is
perpendicular to OA¢ and is equal to fi (  F
ln ​ ______
​     
F – bv
m0 + bt
 ​  ​ = ln ​ ​ _______
m0 ​  
 ​

u0 sin q = 1.65 × 0.42 = 0.69 ms–1


F m0 + bt
1 fi ​ ______
    ​ = ​ _______
m0 ​  
\ Loss in KE = __ ​   ​  m (u20 – u02 sin2 q) F – bv
2
1
__
= ​    ​ × 2 × ((1.65)2 – (0.69)2) 2.25 J fi ​ 
Ft
v = _______
    ​
2 m0 + bt
20. From symmetry the
two balls will collide bFt
F – _______
​      ​
at P such that AQ = dv
___ F – bv
______ m + bt
___________
0
From (i) ​   ​ = ​  m    ​ = ​    
    ​
BQ = 50 m. Ball pro- dt m0 + bt
jected from A follows
the trajectory ABPCA. Fm0
Due to collision, there \ a = _________
​      ​
(m0 + bt)2
S.36  Mechanics II

23. Retardation of 2 kg block a = m g = 2 ms–2 Let the object and astronaut meet after time t.
12t
Velocity of 2 kg block just before collision is given Displacement of astronaut x1 = v1t = ___
​   ​ 
11
by v02 = u2 – 2ax
In this time the object travels a distance = 8 + 8
fi  v02 = 12 – 2 × 2 × 0.16 fi v0 = 0.6 ms–1 + x1 with a constant speed v2.

Let v1 and v2 be velocities of 2 kg and 4 kg blocks \ 16 + x1 = v2t


immediately after collision.
12t 120 108
2v1 + 4v2 = 2 × 0.6 fi 16 + ___
​   ​  = ____
​   ​  t  fi  16 = ____
​   ​  t
11 11 11
fi v1 + 2v2 = 0.6 ...(i)
16 × 11
fi t = ​ _______
 ​ 

= 1.6 s
( 
v1 – v2
And – ​​ _______ 
0.6 – 0 )
​ = 1 fi v2 – v1 = 0.6
 ​   ...(ii)
108

25. The block will stop climbing at the instant its velocity
–1
Solving (i) and (ii) gives: v2 = 0.4 ms  ; v1 = – 0.2 ms . –1 relative to the wedge becomes zero.

Retardation after collision = m g = 2 ms–2


\ Displacement (x1) of 2 kg block toward left after
collision is given as

02 = 0.22 – 2 × 2 × x1 fi x1 = 0.01 m
Displacement (x2) of 4 kg block towards right after
collision is given as:
Let the velocity of the system (wedge + block) at
02 = 0.42 – 2 × 2 × x2 fi x2 = 0.04 m this instant be v (¨).
\ Final separation = x1 + x2 = 0.05 m = 5 cm Conservation of momentum gives

24. Let the astronaut recoil with speed v1 and speed of (M + m) v = mu ...(i)
the object be v2.
Energy conservation gives:
Given v2 + v1 = 12
1 1
fi v2 = 12 – v1 ​ __ ​  (M + m) v2 + mgh = __
​   ​  mu2 ...(ii)
2 2
Momentum conservation gives:
Solving (i) and (ii) for u and putting m = h m
5v2 = 50 v1  fi  12 – v1 = 10 v1 gives

÷  ( 
12 __________
fi v1 = ___
​   ​ ms–1
11
12 120

1
__
)
​    ​   ​ ​
u = ​ 2gh ​ 1 +  
h
And v2 = 12 – ___
​   ​ = ____
​   ​ ms–1
11 11
Chapter 3 Miscellaneous problems on chapters 1 and 2

5. Both the truck and the tempo apply equal force


  match the column on one another. Due to large mass of the truck, its
m1 acceleration is smaller.
1. (A) If ​ ___
m2 ​ Æ 0  fi m1 Æ 0 KE of the system decrease in collision.
Then v2f Æ 0  and  v1f = – v1i KE of the temo cannot be determined using the given
information. KE of the tempo can even increase if
v1f the initial velocity of the truck was very high.
\ ​ ___
v1i  ​ = – 1 6. (A) Exchange of velocity has taken place.
m1 (B) Both objects stick and move together
when ___
​ m   ​ Æ •  fi  m1 Æ • (C) Velocity of body 2 is unchanged and that of
2
body 1 has reversed.
then v2f Æ 2v1i  and  v1f = v1i
(D) Speed of both gets reduced and direction of
v1f motion is reversed. There is loss in KE.
\ ​ ___
v1i  ​ = 1 7. Slope of x-t graph gives velocity.
(A) Both fragments are moving in the same
\  Correct graph is (B) direction. Net momentum is not zero. An exter-
(B) From above discussion, the answer is – 1 nal impulsive force must have acted.
v2f (B) The two pieces have equal and opposite
m1
(C) When ___
​ m  ​ Æ • , ___
​ v   ​ Æ 2 velocities. If mA = mB, then momentum is
2 1i
conserved.
v2f m1
(D) ​ ___ ___
v1i  ​ Æ 0 if m2 is massive  fi  when ​ m2  ​ Æ 0
(C) Same as (A)
(D) vA > vB
v2f m1 For conservation of momentum, mA vA = mB vB
​ ___ ___
v1i  ​ Æ 2 if m2 is light  fi  when ​ m2  ​ is large Therefore, mA < mB
(E) Similar to (D)
3. (A) Sum of momentum of two pieces cannot be
(F) Net momentum is not zero.
towards right
8. In (C), the monkey and the block have the same
(B) Here also, sum of momentum cannot be towards
mass. If the monkey accelerates up, the block will
right
also accelerate up, as the same force acts on both of
(C) For momentum to be towards right, them.
m1v1 > m2v2  fi m1> m2 [Since v1 < v2] In (D), both the blocks will always move with equal
and opposite velocities.
(D) In this case, resultant momentum can be towards
right for  m1 = m2  or m1 > m2  or m1 < m2.
4. Both energy and momentum are conserved. A has
  Passage-based Problems
maximum speed in the initial state, as neither B nor
the spring are having energy at this instant. Entire Passage 1
energy is KE of A. 1. The friction forces on the two blocks are equal and
At some point, A will come to rest, as it is slowing opposite. Their resultant is zero. Thus, linear momen-
down. Its minimum speed is zero. At maximum tum of the system of two blocks is conserved.
compression, both blocks are having equal velocity of When one of the blocks stops, friction stops acting on
u
__
​   ​ . it. The right block stops after 5 s. After this, friction
3 acts only on the left block and is directed leftward.
KE can never be zero for the system, as it will mean Now, momentum of the system is not conserved.
that momentum has become zero. 2. COM stops moving after both the blocks stop.
Retardation of each block, a = 2 ms–2
S.38  Mechanics II

Right blocks stops at t = 5 s and the left block stops Conservation of momentum, for first collision,
at t = 10 s. gives:
Initial velocity of COM, Mv0 = Mv1 + mv2 ...(1)
20 × 1 – 10 × 1
u = ______________
​   ​
     = 5 ms–1 (Æ) Here, v1 and v2 are velocities of the cart and the bead
2
Displacement of COM in 5 s is x1 = 5 × 5 = 25 m. immediately after collision.
v2 – v1
Retardation of COM between t = 5 and t = 10 s is Coefficient of restitution is 1: ​ ______
v0 ​ 
 = 1 ...(2)
1× 2 + 0
acm = _________
​   ​    = 1 ms–2 Solving (1) and (2) gives:
2
M–m m–M
Further displacement of COM before it stops is given v1 = ​ ______ 
 ​ v = – ​ ______ 
 ​ v  ;
by M+m 0 M+m 0
1 2M
x2 = 5 × 5 – __
​   ​  × 1 × 52 = 12.5 m v2 = ______
​    
 ​ v
2 m+M 0
mv0
\  Total displacement of COM = 25 + 12.5 = \ v1c = v1 – vc = – ​ ______
   
​  and
m+M
37.5 m.
Mv0
v2c = v2 – vc = ______
​     

Passage 2 m+M
3. Let leftward direction be positive 6. Relative speed after collison = relative speed before
collision
vb – v4 = – 7
L
__
vb = v4 – 7 \  t2 – t1 = ​ v   ​ 
0
Spring does not exert any force, as it is relaxed. 7. Distance travelled by the cart till first collision = L
During the event of jump, there is no compression Distance travelled between first and second collision
in the spring. Momentum of the 4 kg block and the = (t2 – t1) |v1|
boy remains zero during the jump. Thus,
L m–M m–M
0 = 10 (v4 – 7) + 4v4 = __
​ v   ​  ​ ______ 
 ​ v0 = ​ ______ 
 ​ L
0 M + m M+m
70
v4 = ___
​   ​ = 5 and vb = – 2.0 ms–1 Total distance is obtained by adding the above two
14
distances.
4. When velocity of 1 kg block is maximum, its accel-
eration will be zero. Passage 4
fi  spring will be in its natural length. 8. As the cylinder moves down, the vertical component
Conservation of momentum: of the momentum of the system changes. At the bot-
tom, the cylinder only has horizontal momentum. And
20 – v1
4 × 5 = 1v1 + 4v4  fi v4 = ​ ______
 ​  Æ momentum is conserved in horizontal direction.
4
9. Initial energy of the system = mgR
Conservation of energy:
final energy when the cylinder reaches the bottom
1
__ 1 1
​   ​  × 4 × (5)2 = ​ __ ​  × 1 × v12 + ​ __ ​  × 4 × v42 1 1
2 2 2 of the track = ​ __  ​ mv2 + __
​   ​  mv 21
5v21 2 2
1
100 = v12 + __ ​   ​  (400 + v12 – 4v1)  fi ​ ___ ​   = v1 where v (towards right) is the velocity of ‘m’ and v1
4 4
(towards left) is the velocity of ‘M’ relative to the
fi v1 = 0.8 ms–1 ground.
Passage 3 1 1
\  mg (R – r) = __
​   ​  mv2 + ​ __ ​  Mv 21 ...(i)
5. Initially, the cart moves to right with velocity v0 and 2 2
the bead is at rest. Velocity of COM of the system Initial momentum of the system = 0
is
Mv0 Final momentum when cylinder has reached bottom
vc = ______
​    
 ​ of the track B = mv – Mv1
m+M
First collision occurs when the cart moves through a \  mv – Mv1 = 0 ...(ii)
distance L. The bead hits the left wall.
Solutions  S.39

mv
\  v1 = ___
​   ​ Substituting in (i) gives from question,
M


1
mg (R – r) = __
1
​   ​  [m] [v]2 + __
2 [  ]
mv 2
​   ​  [M] ​​ ___
2
​   ​  ​​ ​
M

1
2 ( 
1
k ​ __ ​  (3m) u2 = ​ __
2
1
​    ​ mv 21 + 2 × __
2 )
​   ​  mv2  ​

[ 
3ku2 = v 21 + 2v2 ...(iv)
1
= __
2
1 m2v2 __
​    ​ mv2 + __
​   ​  ​ ____
2 M
  
v2 m2
​ = ​   ​  ​ m + ___
2
​   ​   ​
M ] Substituting for v from (iii)
1
or,
2g (R – r)
v2 = ​ ________
[  m  ​ 
__
]
​ 1 + ​   ​   ​
[  2gM (R – r)
= ​ ​ __________
M+m
  
    ]
​  ​
3ku2 = v 21 + __
Solving for v1
​    ​ (v 21 + 9u2 – 6v1u)
2
M _______
v1 = u [1 + ÷   – 1) ​ 
​ 2 (k ]

[ 
2gM (R – r) 1/2
v = ​​ ​ __________
M+m
  
    ]
​  ​​ ​
For u = 500 ms–1 and k = 1.5

v1 = 500 [1 + ÷  
​ 2 (1.5
__________
1) ​] = 1000 ms–1
–   
Substituting in (ii) gives:
Passage 6

M [ 
m 2gM (R – r) 1/2
v1 = __
​   ​ ​​ ​ __________
M+m
  
    ] [  2g (R – r) 1/2
​  ​​ ​ = m ​​ ​ _________ 
M (M + m)
  ]
 ​  ​​ ​ Sol : 12,13. After time t mass of the car with water is
mt = (m0 + mt) kg. Let at that momentum speed of
the car be v.
Passage 5 \  mt ___
dm
​ dv ​   = Fext + vrel ___
​   ​ 
dt dt
10. Let the mass of the shell be 3m. The mass of each
fragment is m. dv
As ( 0 + mt) ___
​   ​   = 0 – (u sin q + v) m
The particle with maximum velocity must be moving dt
v   t
in the forward direction. dv dt
fi  ​Ú ​  ​ __________
​      ​ = – Ú​  ​  ​​ ​  _______
m0      ​
By law of conservation of momentum,
3mu = mv1 – mv2 cos q2 – mv3 cos q3
v0 (u sin q + v) 0 ___
(  )
​ ​   ​ + t  ​
m

​( ___
[  ]
​   ​ + t )​
3u = v1 – v2 cos q2 – v3 cos q3 m 0

fi ln ​  ​ _________ 
v1 = 3u + v2 cos q2 + v3 cos q3

Also, mv2 sin q2 = mv3 sin q3


...(i)

...(ii)
(
u sin q + v

u sin q + v0 ) m
 ​  ​ = – ln ​ ​ _______    
m0/m
​  

u sin q + v m0
fi ​ _________   ​ = ________
  ​      ​
u sin q + v0 (m0 + m t)

​ 
m0
fi  v = u sin q ​ _______
  
m0 + m t (  m0v0
 ​ – 1  ​ + ​ ________
  
(m0 + m t)
 ​ )
​ 
m t
fi  v = – u sin q ​ _______  
m0 + m t (  m0v0
 ​  ​ + ​ ________)
  
(m0 + m t)
 ​

Passage 7
If v1 is to be maximum,
Sol: 1 4. Let v be the leftward velocity of wagon (relative to
q2 = q3 = 0 earth). Let u be the velocity of pendulum in a frame
11. From (ii), if q2 = q3 fixed to the wagon. Then, u cos b is the relative hori-
zontal velocity of the bob and u sin b is its vertical
v2 = v3 = v (say) velocity. Let vx and vy be the absolute horizontal and
vertical downward velocities of the bob.
Equation (i) becomes
fi vx = u cos b – v  and  u sin b = vy
v1 = 3u + 2v
There is no external force on the system in the hori-
v = (v1 – 3u)/2 ...(iii) zontal direction.
S.40  Mechanics II

Therefore, by the principle of conservation of Passage 8


momentum,
17. Each shot of mass m (say) leaves the cannon with a
0 = m (u cos b – v) – Mv relative velocity u and a frequency n; the rate of loss
M of mass of the system is given as
fi  u cos b – v = ​ __
m ​  ◊ v
(M + m) v
fi  u = ​ ________  


m cos b

1
15. Kinetic energy of bob = __
​   ​  m (v 2x + v 2y)
2 dm
r = ​ ___ ​ = mn
By the conservation of energy, dt
dm
fi  the thrust force, R = u ​ ___ ​ = mnu
dt
For the cannon to accelerate, this force shall be larger
than the friction
mnu > m M0g

fi  n > m M0g/(mu)

18. The canon can recoil later, as its mass goes on


1 decreasing and therefore, limiting friction force on
mg l(1 – cos a) = mg l(1 – cos b) + __
​   ​  Mv2
2 it also goes on decreasing.
1 19. The net force acting on the system after a time t
+ ​ __ ​  m [u cos b – v)2 + u2sin2b]
2
fi F = R – f  fi  m¢ a = mnu – m N
or,  2mg l (cos b – cos a)
where f is the force of friction
2
M2v2 m sin b (M + m)2 v2
= Mv2 + m ​ _____
 ​

  + ​ ________________
    ​
   where N = m¢ g
m2 m2 cos2b Thus, m¢ a = mnu – m m¢ g ...(i)

{ 
2 2

M
= Mv2 ​ 1 + __ } (M + m)  v
​ m ​   ​ + ​ __________
  
m cos2b
sin2b
  ​ 
m¢ = mass of the cannon with the shots remaining
inside it after a time t
m¢ = M – mnt ...(ii)
M (M + m) 2 (M + m)2 v2sin2b mnu
= ​ _________
   

​ 
v + ​ ______________
        ​ using (i) and (ii), a = ________
​     ​ – m g
m m cos2b M0 – mnt

[ 
Integrating both sides
fi 2 2m2gl (cos b
v = ​ ______ 
M+m

– cos a) cos2b
 ​ ​ ​ _________________
    
M + msin2b
    ​  ​
] v

​​ 
t
dt
​Ú ​   ​dv = mnu ​Ú ​    ________  ​ – m g ​Ú ​   ​dt
   
t

÷  [ 
________________________ 0 0 M 0 – mnt 0

\ 
2m2gl (cosb
v = ​ ______
​ 
M+m


– cosa) cos2b
 ​ ​ ​ ________________
        
M + m sin2b
    ​  ​ 
] [ mn
1
v = mnu ​​ – ​ ___   
​ 
l n |M0 – mnt| 
t
]
​​0 ​​  – mgt

16. In this particular case, when b = 0, fi ( 


M0 – mnt
v = – u ln ​ ​ ________
M0 )
​ – m gt
 ​  

÷ 
_____________
a

÷ 
_______________

( 
2 sin ​ __ ​ 
2

)
2 2
2m  gl (1 – cos a) 2m  gl 2 M0
v = ​ ______
​   ​ ​ _________

        = ​ ______
​ ​  ​    ​ ​ _______
         
​ ​ = u ln ​ ________
​      ​  ​ – m gt
M+m M M+m M M0 – mnt

÷ 
_________
gl Since after a time t, the cannon + remaining shots
a
v = 2m sin ​ __ ​  ​ _________
​       ​ ​. has mass M
2 (M + m) M
fi M0 – mnt = M
fi v = u ln (M0/M) – m gt
Solutions  S.41

Passage 9 (r v) u = Mg
20,21. Refer to example 25 in the chapter.
r p d2 2
Let A get deflected by angle q due to collision with B. fi  ​ _____
 ​ 
 u = Mg ...(i)
4
Velocity of B = velocity component of A along
LOI, which gets imparted to B Power = KE imparted to air
vB = u sin q 1 1 p d2 3
= ​ __ ​  (r V) u2 = __
​    ​ r ◊ ​ ____
 ​ 
 u
Velocity of A = velocity component of A ^ to LOI 2 2 4

(  )
before collision
r p d2 3 _____
r p d2 _____4Mg 3/2
vA = u cos q = _____
​   ​  u = ​   ​  ​​ ​     ​  ​​ ​
8 8 r p d2
(Mg)3/2
= ​ ______
___ ​ 
d ​÷p r 
   ​

Passage 11
24,25. Impulse of friction is

Jf = k Ú v dt = k ◊ L

Speed of____
A when it reaches the bottom of the slide
  ​ 
is u = ​÷2gH 
____
After hitting C, the disc A moves along x-direction.   ​ 
Momentum = mu = m ​÷2gH 
During collision, it will impart its velocity, along
LOI, to C and retain its velocity perpendicular to For the system to stop
LOI. It means that LOI is along y-direction. C begins ____
  ​ 
Jf = m ​÷2gH 
to move in y-direction.
____
vC = vA sin q = u cos q sin q   ​ 
kL = m ​÷2gH 

and vA¢ = vA cos q = u cos2 q k2L2


fi H = ​ _____2 
 ​ 
u 1 1 2gm
Given, vA¢ = ​ __ ​   fi  cos2 q = __ ​   ​  fi cos q = ___ ​  __  ​ 
2 2   
​÷2 ​
​_› u Passage 12
__
\  ​v   ​C = ​   ​ 
2 Sol. The entire process is like a head-on collision of two
_
÷    ​ ÷
​​ 2 ​
1
​v  ​ B = ​  u __    ​ ​ ___
​›
[ 
    ​÷2 ​
​ 2 ​
1
​  __  ​  – ___
]
​  __  ​    ​ = __
  
u u
​   ​  – __
2
​   ​ 
2
blocks with coefficient of restitution = h
[Refer to example 24 in the chapter]

Passage 10 m1 (1 + h)
\ vB = ​ _________ ​ u 

  [Ans. to Q.27]
m1 + m2
22. Lift force on helicopter = momentum imparted to
blown air in unit time At the instant of maximum deformation,
\ Mg = V r ◊ u V = volume of air pushed vA = vB = v (Say)
down per second
Mg m1u
fi V = ___
​   ​  (m1 + m2) v = m1u  fi  v = _______
​    
 ​
r u m1 + m2

23. Volume of air pushed down in unit time can also be Energy absorbed by the thread is
written as


d 2
V = p ​​ __ (  )
​    ​  ​​ ​ ◊ u
2
1
2
1
= ​ __ ​  m1u2 – __
2
​ 
m1u
​   ​  (m1 + m2) ​​ _______
m1 + m2(  )
  
2
 ​  ​​ ​

​[ 1 – _______ ​   ​  ​  _______


For take off:
m + m ] 2 (m + m )
1 m 1 mm
= __  ​  ​ = __
1 1 2
Momentum imparted in unit time = Mg ​   ​  m1u2 ​     ​     ​  ​ u 2
2 1 2 1 2
S.42  Mechanics II

Passage 13 of wedge is v0. We know that (from our discus-


sion in constraint-related problems in the chapter of
Sol. Every time the disc climbs a slide (and slides down
Newton’s law of motion)
it), it imparts a KE to the slide. Height attained on
B will be maximum when it climbs on it for the first vy 1
time. ​ ______   tan q  fi  vy = ___
   ​ = ​  __  ​ (vx + v0) ...(i)
vx + v0 ÷   
​ 3 ​
Let velocity of A be v1 (¨) when C reaches the
bottom. Let velocity of C be 1 (Æ) at this instant. Conservation of momentum in horizontal direction
gives
Conservation of momentum gives Mv1 = mu1 ...(i)
1 1 Mv0 = mvx  fi  4v0 = vx ...(ii)
Conservation of energy: __ ​   ​  Mv12 + __
​   ​  mu21 = mgh
2 2
From (i) and (ii),
fi  Mv21 (  )
Mv1 2
+ m ​​ ____
​  m   
​  ​​ ​ = 2mgh

÷
__
  
​ 3 ​   
4​÷3 ​
v0 = ​ ___ ​ vy  and  vx = ____
​   ​ 
 v
__

5 y

÷ 
_________ 5

÷ 
______
2m2gh 2Mgh
fi  v1 = ​ _________
​      ​ ​  and  u1 = ​ ______
  ​    
 ​ ​
  Conservation of mechanical energy gives
M (M + m) M+m
1 1
Now C climbs on B. At maximum height, C stop ​ __ ​  Mv20 + ​ __ ​  m (v 2x + v y2) = mgy
2 2
moving on B. It means both have same velocity; say
(  ) ( 
__
v2 (Æ).
Momentum conservation: (M + m) v2 = mu1
÷
fi 4 ​​ ___
   2
​ 3 ​
​   ​ vy  ​​ ​ +
5
48
​ ___ )
​   ​ v 2y + v y2  ​ = 2gy
25

) ÷ 
______ 10
fi v y2 = ​ ___ ​ gy
(  m 2Mgh
fi  v2 = ​ ______
​       ​  ​ ​ ______
​   
 ​ ​

  17
M+m M+m

÷  ÷ 
_____ _____
10 dy 10
Energy conservation: mgH + __
1
​   ​  (M + m) v22 = __
1 fi vy = ​ ___
​   ​  mu21 ​   ​  gy ​   fi ​ ___ ​ = ​ ___
​   ​  gy ​ 
2 2 17 dt 17

(  )2Mgh

÷ 
1 m ____ t
fi  mgH = – ​ __ ​  (M + m) ​​ ______
h
 ​  ​​ ​______
2
​      ​   
  ​ dy 100
2 M+m M+m fi ​Ú ​    ​ ​  __  ​ = ​ ____
___  ​Ú ​    ​dt
​   ​ ​ 
0 ÷ ​  
y  ​
  17 0

1
2 (  ​ 
2Mgh
+ ​ __ ​  m ​ ______
M+m )    ​  ​ __
fi ​÷ h ​  = ____
5
​  ___
÷
_______ ___
  × 17 ​  ____
​ 0.5
◊ t  fi  t = ​ _________
   ​    ​  
   
​÷8.5 ​
= ​   ​  s
[ 
M mgh
= ​  ______
M+m

m
_______
 ​ ​ 1 – ​ 
(M + m)
     ​  ​
] ÷
​  
17 ​  5 5

Passage 15
2
M
fi  H = ​ ________
    ​ ◊ h There is no change is magnitude of horizontal com-
(M + m)2 ponent of velocity due to impacts.
2a
Finally, the disc will slow down such that its speed 33. \  T = ___​ u  ​   fi  ux = ___ ​ 2a ​ 
x T
will becomes slightly less than the wedge it is chas-
2a 2a
ing. It will no longer be able to climb the wedge. fi  u cos q = ​   ​   fi  u = ______
___ ​     ​ 
T T cos q
At this instant, the entire energy will be shared
a a
between A and B as disc will have negligible KE 34. In this case, T = __ ​ u   ​   fi  ux = __ ​   ​
(due to m << M). x T

÷ 
____
a
1 2 __
__ 1 2 mgh
\ ​    ​ Mv + ​   ​  Mv = mgh  fi  v = ​ ____
​     
​ ​   fi  u = ​ ______    ​ 
2 2 M T cos q

Passage 14 Passage 16
Sol. Let velocity compo- ​_› 30  + 20  – 20  + 50  – 50 
35. ​v  ​ cm = ​ _________________________
   
 ​   
nents of the block, 3
after it has descended
= (20  – 10 ) ms–1
a distance y, be vx and
​_›
vy, as shown. Velocity ​v  ​cm
  will not change due to internal interactions.
Solutions  S.43

​_›
\  Velocity of shell before explosion = ​v  ​ cm For A to topple, x > 2d
\  KE of shell before explosion
4 u2
fi ​ ___  ​ ___
​    ​  > 2d
1 1 75 m g
= __ 2
​   ​  Mv cm = __
​   ​  × 3 × (202 + 102) = 750 J
2 2
5 _____
fi u > __  
​   ​  ​÷6m gd 


36. KE at given instant is 2
1 1 1
K = ​ __ ​  × 1 × (302 + 202) + __
​    ​ × 1 × (202) + __
​   ​  5 _____
2 2 2 \  umin = u0 = __  
​   ​  ​÷6m gd 


2
× 1 × (502 + 502) _____
= 3,350 J  
39. When u = 2u0 = 5 ​÷6m gd  ​, the flying object will have

velocity towards left (after collision) given by (iii)
KE at given instant is higher than initial KE by
as
3340 – 750 = 2600 J.
1
37. KE = KEwrt com + __​    ​ Mv cm 2
3u _____
2 v1 = ___  
​   ​ = 3​÷6m gd ​ 
5
KEwrt com can become zero if all the particles stop
1
moving due to attraction. But, __ ​    ​ Mv2CM will remain It will fly like a horizontal projectile and its range
2 will be
unchanged

÷  ÷ 
1 ___ ___
\ Minimum KE possible after explosion = __ ​   ​  Mv2CM 2d _____ 2d ___
2 x = v1 ​ ​ ___
g  ​ ​
   
 = 3 ​÷6m gd 

  ​ ___
​  g  ​ ​
     ​
 = 6d ​÷3m 

= 750 J
Passage 18
Passage 17
4
38. Let v1 and v2 be velocities of the flying object and Sol. Velocity of approach = v cos q = 10 × __
​    ​ = 8 ms–1
5
block B just after collision.
Relative speed of separation = e × 8 = 4 ms–1
mu = mv1 + 4mv2  fi  u = v1 + 4v2 ...(i)
Let velocity of disc A (after collision with the ball)
Since, collision is elastic be v1 along LOI and velocity of ball be v2 along
LOI.
\ ( 
v1 – v2
– ​ ______
​ 
u–0
 ​   )
​ = 1  fi  v2 – v1 = u ...(ii)
v1 – v2 = 4 ...(i)

3u 2u
Solving (i) and (ii); v1 = – ​ ___ ​  , v2 = ___
​   ​  ...(iii) Conserving momentum along LOI
5 5
After collision, B begins to move with velocity v2. Mv1 + Mv2 = M × 8
The upper block A does not move. There is no hori-
zontal force on it! fi  v1 + v2 = 8 ...(ii)
B retards due to friction from the floor. Solving (i) and (ii) v1 = 6 ms–1, v2 = 2 ms–1
Friction on B = m (4m + 2m) g = 6 m mg Velocity of ball along t-line remains unchanged
6m mg 3
Retardation of B = ______
​   
  ​= __
​   ​  m g
3
vt = v sin 37° = 10 × __
​   ​  = 6 ms–1
4m 2 5
______ ___
\  Displacement of B till it stops is given by \ ​ 2  2 + 62 ​ 
vball = ÷    ms–1
= ​÷40 ​
v2 = u2 + 2ax
Disc A hits head-on, disc B with a velocity of

(  ) (  )
2u 2 3 v1 = 6 ms–1 and the collision is elastic.
fi 0 = ​​ ___
​   ​   ​​ ​ – 2 ​ __
​    ​ m g  ​ x
5 2 \  Disc B acquires a velocity of 6 ms–1 and A comes
to rest.
4 u2
fi x = ___
​    ​ ___
​    ​ 
75 m g
Chapter 4 Torque and Equilibrium

Your Turn \ t = 60 N-m along negative z.


​_› _
​›
(2 + 2  )m ; ​F  ​  = 3 
1. ​r ​    = 6. Torque of mg about O is
_
​› ​_› ​_›
R
​t    ​ = ​r ​   × ​F  ​  = 6 N-m. t = mg ◊ ​ __ ​ 
2
2. Component of__ force perpendicular to op is u2 sin2q
    N.
F cos 30° = 10​÷3 ​ = mg _______
​     
​ 
__ __ 2g
\ t = dF^ = 1 × 10​÷3 ​
    = 10​÷3 ​
    N-m ( )
​_› ​___› = mu2 sin q cos q
or, ​r ​    = ​   
OP​ 7. Friction produces an anti-clockwise torque about C.
​_› _
​›
Angle between ​r ​   and ​F  ​  is 60°. To balance this, N must produce a clockwise torque
__ about C. The correct line for N is 3.
÷   
​ 3 ​ __
\ t = r F sin 60° = 1 × 20 × ___     N-m
​   ​ = 10​÷3 ​ 8. For horizontal equilibrium, it is necessary that
2
3. Torque about C: horizontal force at A = force at B (= Fx). For verti-
cal equilibrium Fy = 600 N
t = Fx + Fy ( )
tA = 0
= F (x + y) = FL ( )
Where L is the length of the rod. Thus, value of Fx (6 sin 53°) = 200 (3 cos 53°) + 400 (4 sin 53°)
torque is independent of choice of point C. fi Fx = 275 N
F

C
x y

4. Torque due to string tension F1 is t1 = F1R ( )


Torque due to F2 is t2 = F2R ( )
Torque due to Mg is zero as line of Mg intersects
the axis at O. The axis (the support) at O must by 9. For horizontal equilibrium,
applying a force on the pulley. But this force is acting FH = T cos 37° ...(i)
at O. Whatever be its direction, it has no torque.
For vertical equilibrium,
\  Total Torque = 0.
FV + T sin 37° = 150 ...(ii)
Taking torque about D to be zero
(T sin 30°) × 1 = 50 × 0.7 + 100 × 1.4 ...(iii)
Solving (i), (ii) and (iii) gives the answer.

5. t1 = F1 (AD) = 20 N-m, directed along negative z


[This is same as saying clockwise if the observer is 10. Balancing torque about the centre point C,
looking downwards] 15g (x) = 10g (2.5)
t2 = F2 (AB) = 40 N-m along negative z. 5
fi x = __
​   ​  m
t3 = 0 ; t4 = 0 ; t5 = 0 3
Solutions  S.45

Worksheet 1
1. Weight acts at COM. Its line is the rotation axis. It
has no torque.
2. Resultant force is
_____________
  2 + (400)
F = ​÷(300) 2
    ​ = 500 N

4
11. N1 + N2 = Mg And tan q = __
​   ​   fi  q = 53°
3
fi 30g + 25g = Mg Torque about O is
fi 55g = Mg t = (400R +200R + 100R) ( ) + 200 R ( )
tA = 0
= 500R ( )
fi Mg ◊ x = N2 (160 cm) 400 N
F
Force shown in option (C) will
25
fi x = ___
​   ​ × 160 cm = 72.7 cm have exactly the same effect
55
on the wheel as the given sys- q
\  Required answer is 160 – 72.7 = 87.3 cm tem of forces. 300 N
N1 N2 3. Torque applied by the person balances the torque due
to weight about the corner.
G b
\ t = W __
​   ​  A
A 2 b
Torque due to weight (about A) is 2
Mg
x clockwise. The person must pro-
duce an anti-clockwise torque.
14. Taking the edge of the table to be origin, the COM of
the system of two bricks will have its x co-ordinate Note:  Actually, the fingers apply
W
given by large friction on the book and


(  )
L
M ​ – ​ __ ​   ​ + M (0)
4
xcm = ______________
​       ​ 
L
= – ​ __ ​ 
this friction causes a torque since size of fingers is
not exactly zero. Shaded portion
2M 8 in the figure is thumb. It exerts
It means COM is well within the support region. The friction ( f ), which produces a
system will be in equilibrium. torque fr about A. The book has
a tendency to rotate about a point
15. Block will slide if tan q > m. The cube will be on the that is slightly inside from corner
verge of toppling if vertical line through its COM (C) A – somewhere around centre of
passes through its edge A. In this the thumb.
case:
4. For equilibrium, torque about any point = 0.
AM
tan q = ____
​   ​ = 1 Let us take torque about B
CM
2 × 3 = 6 × x  fi  x = 1m
fi q = 45°
4N
\  B lock will slide before toppling if tan q ≥ m for 2N
q < 45°.
D
fi m £ 1 A B
x
16. Displacement of the two balls are related as:
6N
m1x1 = m2x2
5. In option C:
​_›
Since the COM of the system will not move in ​_r ​    = – b  + o – c
absence of external forces. ​›
​F   ​ = a  _
The COM of the entire system remains exactly above ​_› _
​› ​›
\ ​t    ​ = ​r ​   × ​F  ​  = – ab  + ac 
the pivot point.
\ tz = – ab
S.46  Mechanics II

6. Torque about the centre of the rod = 0 L L ___ 3L


\ Smax = __
​   ​  + __
​   ​  = ​   ​ 
l l 4 2 4
N1 ​ __  ​   = N2 ​ __  ​  10. The COM will accelerate. we will take t = 0 about
4 4
COM.
fi N1 = N2
NB ◊ a = NA ◊ a  fi  NB = NA
N2
W
N1 \ NB = NA = __
​   ​ 
2
11. Balancing torque about A gives
C
N(AB) = mgR
Mg
_________
7. Balancing torque about COM of the beam N÷   2 – R2 ​ =
​ (2R)  mgR

l
N1 ​ __
2 4 (  l
​    ​  – __ ) l
​    ​   ​ = N2 ​ __ (  l
) N1 4
​    ​  – ​ __  ​   ​  fi ​ ___ ​ = __
2 6 N2 3
​    ​

mg
N = ​ ___
÷
__ ​ 
  
​ 3 ​
8. Due to symmetry: N1 = N2  and  f1 = f2 12. About A:
N1 + N2 = 2Mg Torque due to W = torque due to P

fi N1 = N2 = Mg = 200 N W ◊ r = P sin q (AB)


AB
P W ___
​   ​  ◊ sin (p – 2q) = P sin q (AB)
2
sin2q
fi P = W ​ ______    
​ = W cos q
2 sin q
C
N1 N2
Mg Mg P sin q q
60° P

Q f1 f2 R B q
Considering rotational equilibrium of board PQ about
P, gives
r
l
N1 l cos 60° = Mg ​ __  ​  cos 60° + f1 l sin 60°
(p

2

2q

__
)

1 Mg   
​÷3 ​ 100
Mg ◊ ​ __ ​ =
  ___  + f1 ___
​   ​  ​   ​   fi  f1 = ____
​  __ ​ N W A
2 4 2 ÷   
​ 3 ​
9. For Brick 2 to remain in equilibrium above Brick 1, 13. Torque about A is zero
L
the maximum overhang of 2 over 1 can be ​ __ ​ . For the
2 mg ◊ (AE) = T ◊ (AB)
entire system to remain stable and not topple about
A, COM of the two-brick system shall be to the left
of edge A. In critical case, when COM is at A, taking
A to be the origin, we can write
L /2

2
1
A
x

( 
L
M ​ __ ) L
​   ​  – x  ​ = M ◊ x  fi  x = __
2
​   ​ 
4
Solutions  S.47

mg ◊ (AG cos 45°) = T ◊ l f = F

( 
__

÷    l ___
​ 2 ​
mg ​ ___
​   ​ . ​ 
3 ÷
1
)
  __  ​  ​ = T.l

  
​ 2 ​

2
f = ​ __ ​  Mg
3
mg But f £ mN
fi T = ___
​   ​ 
3 2 2
\ ​ __ ​  £ m Mg  fi ​ __ ​  £ m
For translational equilibrium, hinge at A must apply 3 3
a vertical force (F) such that 17. To balance torque due to friction (f) about C, the
2mg effective normal force must pass through left of C.
F + T = mg  fi  F = ____ ​   ​ 
  Thus, the left half exerts more force on the incline
3
compared to its right half.
14. For translational equilibrium:
N = Mg cos 30° ...(i) N
f
and T + f = Mg sin 30° ...(ii) C
Torque about C = 0
fi TR = fR  fi  T = f
Mg
1
From (ii), f = ​ __ ​  Mg sin 30°
2
18. Lowering the centre of gravity is the basic physical
But f £ mN
principle that is used in the show. If the pole is heavy
1 and flexible, the COM of the system of the man +
\ __
​   ​  Mg sin 30° £ m Mg cos 30°
2 pole can be below the rope as well.
1 1__ 19. String tension T = mg
fi ​ __ ​  tan 30° £ m  fi ​ ____
   ​ £ m
2   
2​÷3 ​ Component of ‘T’ parallel to incline plane will be
T T sin q = mg sin q.
N
C
f
Mg sin 30°

Mg cos 30°
Mg
30°

15. Vertical line through COM will fall outside the base
region if the system shown in (C) is tilted slightly [This is not shown in the figure]
towards right.
Net force on cylinder along the incline is zero.
16. Just when the cube is about to rotate about A, the
normal force will pass through point A. In critical \ f = Mg sin q + mg sin q ...(i)
case, torque of F about A is equal to torque due to
Mg about A. [Friction has no torque about A] For zero torque about C

2Mg T ◊ R = f ◊ R  fi  T = f (= mg)


3b b
\ F ◊ ​ ___ ​  = Mg ​ __ ​   fi  F = ____
​   ​ 

4 2 3 Substituting in (i) gives; mg = Mg sin q + mg sin q
N M sin q
fi m = _______
​   
 ​

1 – sin q

C
F 20. When the torque due to F about A just becomes equal
b/2 3b to the torque due to Mg, the cylinder is on the verge
4
of moving. normal force
f A at B = 0 and normal force
Mg and friction at A are of no
2
If the cube is not to slide when force F = ​ __ ​  Mg concern if we are consid-
3 ering torque about A.
is applied,
S.48  Mechanics II

Worksheet 2 Worksheet 3
1. For vertical equilibrium of entire system, 2N = 2W 1. m1(3l) = m2 l fi  3m1 = m2
fi N = W (m1 + m2) = m3l fi  (4m1) (3l) = m3l
Considering equilibrium of part AB only, we can eas- fi  12m1 = m3
ily see that there will be a horizontal force on it at A
And (m1 + m2 + m3) (3l) = m4l
and it will be equal to T acting towards left. As we
have already concluded that N = W, there cannot be a fi (m1 + 3m1 + 12m1) (3l) = m4l
vertical force on AB at A. Value of T will depend on
fi 48m1 = m4
the distance of Point P from A. Larger the distance
AB, smaller will be T. 96
fi m1 = ___
​   ​ = 2 kg
48
A
2. tA = 0
f (AB) = Mg (AC sin 45°)

W W
f (AB) = Mg ​ ___ ( 
AB 1__
​   ​  ◊ ​ ___
2 ÷   
​ 2 ​ )
  ​  ​
N P q N Mg
T T fi f = ____
​  __  ​   ...(i)
  
2​÷2 ​

B C
A N1
C f
2. T = mg.
N2
As mg increases, tension produces more and more B
torque about A. Normal force moves to right and in Mg
extreme case (when the block is about to topple), the
contact is only at A. Normal force acts at A. Cylinder
will not tilt as long as: 45°

h Mg M
Mg __
​   ​  ≥ T ◊ h  fi ​ ___ ​ 
 ≥ mg  fi ​ __ ​  ≥ m Net force in vertical direction = 0
4 4 4
fi N2 cos 45° + f sin 45° = Mg
N2 ___ Mg
fi ​ ___ __ 
 ​ + ​   ​  
  = Mg
  
​÷2 ​ 4
3Mg
fi N2 = ____
​  __ ​ 
  
2​÷2 ​
Since f £ mN2
Mg 3Mg 1
\ ​ ____  ​  £ m ◊ ​ ____
__  __ ​   fi ​ __  ​ £ m
3. For equilibrium, Fnet = 0 and tnet = 0   
2​÷2 ​   
2​÷2 ​ 3
3. The COM lies vertically below the peg. If linear mass
density = l, then
mass of OA is m1 = l L
mass of OB is m2 = 2l L
Solutions  S.49

For COM to lie on y axis:


q
m1x1 = m2x2
R–r

L
(l L) ​ __ (  ) 2L
​   ​  sin q  ​ = (l 2L) ​ ___
2 (  )
​   ​  ◊ cos q  ​
2
fi tan q = 4 sin q = R – r
R+r
fi q = tan–1 4
4. For horizontal equilibrium: From (i) and (ii): mN1 + mN1sin q = N1 cos q
T2 cos b = T1 cos 30° fi m (1 + sin q) = cos q ...(iii)
__
÷   
​ 3 ​
fi T2 cos b = ___
​   ​ T1 ...(i) R–r
From geometry: sin q = ​ _____   ​
2 R + r ___
For vertical equilibrium: ________
   ​
2​÷Rr 
\ ​ = _____
​ 1  – sin2 q  
cos q = ÷ ​    ​
T1 sin 30° + T2 sin b = Mg R+r
\  Equation (iii) becomes
T1 ___
fi ​ __ ​  + T2 sin b = Mg
2
...(ii)
( 
R–r
m ​ 1 + ​ _____ 
R+r ) 2​÷Rr 
 ​  ​ = _____
​ 
R+r
   ​
  ​

÷ 
__
r
fi m = ​ __ ​    ​ ​ 
R
7. There is no friction between roller and the rod. The
normal force by the roller on the rod is perpendicular
to the rod. Write equation of horizontal and vertical
equilibrium
Taking torque about C to be zero:


3L
(T1 sin 30°) ​ ___ (  ) L
​   ​   ​ = Mg ◊ ​ __ ​ 
4 4
2 Mg
fi T1 = ​ _____  ​    ...(iii)
3
Mg
Equation (i) becomes T2 cos b = ___ ​  __ ​   ...(iv)
÷   
​ 3 ​
f = N2 sin q ...(i)
2Mg
Equation (ii) becomes T2 sin b = ____
​   ​  
  ...(v) N1 + N2 cos q = Mg ...(ii)
3
Dividing (v) by (iv) gives tA = 0 gives
2__
tan b = ​ ___   ​ 
L
N2 (AC) = Mg ​ __ ​  cos q
÷   
​ 3 ​ 2
6. Fig. shows forces on the smaller roller. h
____ L
N2 ​      ​ = Mg __​   ​  cos q
sin q 2
N1
q MgL
r
fi N2 = ____​      ​ sin q ◊ cos q
O 2h
f1 N2 Putting this value in (i) and (ii) gives:
f2 MgL
mg f = ____ ​ sin2 q cos q  and
​     
2h
In critical case, frictions f1 and f2 are
f1 = mN1  and  f2 = mN2
N1 = [  L
2h ]
​ 1  – ​ ___  ​ sin q cos2 q  ​

Since f £ mN1
For horizontal equilibrium:

N1 cos q – mN2 – mN1sin q = 0 ...(i) \  ___


2h
L
( 
L
​    ​ sin2 q ◊ cos q £ m ​ 1 – ___
​    ​ sin q ◊ cos2 q  ​
2h )
Taking torque about centre (O) to be zero L sin2 q ◊ cos q
fi m ≥ ​ _______________
       ​
mN1 r – mN2r = 0  fi  N1 = N2 ....(ii) 2h – L sin q ◊ cos2 q
S.50  Mechanics II

8. For equilibrium of M, 9. There is no horizontal or vertical acceleration.


T = Mg \ N = Mg
For torque on pulley to be zero about its centre O, and f = F

T (OQ) = mg (OP) fi mN = F
fi mMg = F
TR = R sin q
For rotational equilibrium: torque about A = 0
fi mg = mg sin q
1 fi  Mg (OG sin q) = F (R – R sin q)
fi ​ __ ​   = ____
​      

M sin q 3R
fi  Mg ​ ___ ​ sin q = mMg R (1 – sin q)
8
fi  3 sin q = 8m – 8m sin q
3 sin q
P O fi  m = __________
​    ​
  
Q 8 (1 – sin q)
q

mg T
T

mg
Chapter 5 Kinematics of Rotation

Your Turn Acceleration of the COM is g (Ø) as only external


force on the stick is mg and aCM depends only on
( 
18 × 2.54
1. a = w2R = (20)2 ​ ​ ________
100
 ​    )
m  ​ = 182.9 ms–2 external force.
_
​› _
​› _
​›
2. Let w1 and w2 be the angular speed of larger and \ ​a  ​ A = ​a  ​ ACM + ​a  ​ CM
smaller pulleys respectively.
fi aA = 4 (Ø) + 10 (Ø) = 14 ms–2 (Ø).
For no slipping, speed of a point on the circumfer-
ence of the pulling = speed of the load. 6. Instantaneous centre of rotation lies at ​_the point​_›
of

intersection of perpendiculars drawn to ​v  ​ A and ​v  ​ B.
fi w1r1 = w2r2 = v
Obviously D ABC is equilateral D.
fi w1(0.2) = w2(0.1) = 10
vA = w (AC)
fi w1 = 50 rad s–1;  w2 = 100 rad s–1. v
v = w L  fi  w = __
​    ​
3. q = 2 turns = 4p radian L
x = Rq = 4p R meter Note:  Velocity of the centre of the rod (D) is w (CD) along
4. The stick must be vertical at the instant mentioned. a direction perpendicular to CD; i.e., along BA.
If it is not so (as shown in first figure), then velocity
of A or B cannot be zero since velocity of A or B is
the vector sum of v0 and wr.

7. wR = 2 × 0.5 = 1 ms–1
vB = 0 is possible if situation is as shown in the v > wR
second figure. For vB to be zero, wr = v0 = 2 ms–1 Hence, the disc is slipping.
\   vA = v0 + wr = 4 ms–1 Velocity of the contact point is v – wR = 2 ms–1 in
forward direction.
5. In COM frame, the stick is rotating uniformly. Point
v 4
A is going in a circle of radius r (= CA = 1m). 8. w = __
​   ​  = __
​    ​ = 4 rad s–1
R 1
\  Acceleration of A wrt COM is
vA = wr + v = 4 × 0.5 + 4 = 6 ms–1
aACM = w2r = 42 × 1
_________ ______ __
= 4 ms–2 (Ø)   2 + v2 
vB = ​÷(wr)  ​ = ​÷2  2 +  42      ms–1
 ​ = 2​÷5 ​
vC = v – wr = 4 – 2 = 2 ms–1.
S.52  Mechanics II

9. Relative to centre, point A is performing uniform


circular motion. Worksheet 1

aAC = w2r (Ø) = 42 × 0.5 = 8 ms–2 (Ø) 1. Particles like P, A, B, C ... have the same speed.
Particles like P and Q have the same velocity.
Acceleration of the centre is zero
\ aA = 8 ms–2 (Ø)
10. (i) vpc = wR = v (Ø)
​_› ​_› ​_›
​v  ​p  = ​v  ​pc
  + ​v  ​c 
__
\ vP = ​÷2 ​
   v

2. Refer to the figure in previous solution. In the plane


(ii) The wheel may be considered to be in pure represented by circle ABCP, no two particles can
rotation about the contact point (A). have the same velocity (direction and magnitude
Speed of P is vP = wr1 both same). Speed of all the particles on the cylinder
PABDEQP is same.
Speed of Q is vQ = wr2
3. All points have velocity along tangents. Thus, body
Since r1 > r2 v
is in pure rotation about its centre. w = ​ __ ​ 
\ vP > vQ R
4. l is a line perpendicular to vB and m is line perpen-
dicular to vA. Intersection of l and m at O is the
instantaneous centre of rotation.
w (OA) = vA
L 2v
w ​ __ ​   = v  fi  w = ___
​   ​ 
2 L
Speed of the centre (P) is vp = w(op)
2v L
= ___ ​   ​  ◊ ​ __ ​  = v
L 2
Note:  Velocity of P is ^ to op.

5. It depends on the values of v and wR.


If v = wR there is no slipping.
If v < wR then the contact point moves back-
wards, etc.
Solutions  S.53

6. Velocity of P wrt centre (C) is vpc = wR = v in tan-


gential direction Velocity of P in ground frames is
_
​› _
​› _
​›
​v  ​ P = ​v  ​ PC + ​v  ​ C
______ __
\ vP = ​÷v  2 + v2 
 ​ = ÷   v 
​ 2 ​
v
And tan q = ​ __v ​ = 1  fi  q = 45°
Acceleration of P wrt C is
v2
apc = w2R = __​   ​  towards centre C. 8. COM will move in a circle with centre at O.
R
Acceleration of C itself is zero Immediately after the string is cut, there is no speed.
​_› ​_› ​_› ​_›
Thus, COM has no radial acceleration. COM has
\ ​a  ​ p = ​a  ​ pc + ​a  ​ c = ​a  ​ pc tangential acceleration, which is directed vertically
​_› down.
\  ​a  ​ p is horizontally along PC.
9. Speed of any point (like P) on the chain is w1r1 or
w2r2
\ w1r1 = w2r2
w1(0.2) = (200 rpm) (0.5)
fi w1 = 500 rpm

7. vAC = wR = v = 25 ms–1
Direction of vAC is as shown.
​_›
​v  ​ AC = (25 sin 37°)  + (25 cos 37°)  10. Speed of a point on the circumference is
v = 2 ms–1 = wR. There is no acceleration of the
= 15  + 20 block. It means there is no angular acceleration of
Velocity of point A wrt ground is the pulley. A point on the circumference has radial
​_› ​_› ​_› acceleration only.
​v  ​ AC = ​v  ​ AC + ​v  ​ C = 15  + 20  + 25
v2 22
= (40  + 20 ) ms . –1 a = ​ __ ​  = ​ ___  ​ = 40 ms–2
R 0.1
S.54  Mechanics II

v = wR = 3.8 × 103 × 0.05


Worksheet 2
= 1.9 × 102 ms–1
1. w = w0 + a t
5. Separation between points A and B cannot change.
fi w = 2 + 2 × 10 = 22 rad s–1
1
q = w0t + __​   ​  a t2
2
1
fi q = 2 × 10 + __ ​   ​  × 2 × 102 = 120 rad
2
120 \ vAx = 6 cms–1
= ____
​   ​ rotations 20
2p Let velocity of A wrt B = vAB
Angular velocity is same for all points. vAB
Then, w = ___
​      ​
Acceleration of a point is the vector sum of radial and l
tangential accelerations. It is not in radial direction. vAB
2. Smallest speed is for the fi 0.4 = ​ ___ ​   fi  vAB = 0.2 ms–1.
0.5
particle in contact with
the ground. It is 0 in pure vAB = vAy – vBy
rolling. Highest speed fi 0.2 (Ø) = 0.1 ms–1 (Ø) – vBy
is for the top point. It is
fi  vBy = 0.1 ms–1 (≠) = 10 cms–1 (≠)
v + wR = 2v for a point
10 5
on the top. Velocity of any Velocities of A and B are as shown. tan q = ___
​   ​ = __​   ​ 
point relative to C is wR in _ _6 3
​› ​›
tangential direction. For_​ the l is a line ^ to ​v   ​A and m is a line ^ to ​v   ​B and

point shown in figure ​v   ​PC their intersection point C is instantaneous centre of
is equal to wR = v in the rotation.
direction shown. BM
____
​     
​  = tan q
Velocity of P wrt ground is obtained by adding d
​_› ​_› ​_›
velocity of centre (​v  C ​ = ​v   ​) to ​v  PC
 ​ . Obviously, speed
25 cm __ 5
of P is v. fi ​ _____ ​  = ​    ​  fi  d = 15 cm.
   
d 3
3. The wheel can be thought to be in pure rotation
about it contact point M. speed
of any point is wr, where r is the
distance from M. C
ACB is an arc of radius R with A R B
centre at M. All particles in the
shaded region will have speed M
less than wR = v.
4. Time taken by light to travel from the wheel to the
mirror and back is
6. For pure rolling, a = R a.
2L 2 × 500 1
Dt = ___
​   ​ = ​ _______8  ​ = __
​   ​  × 10 –5 s \ Acceleration of centre is a = (10 cm) (2 rad s–2)
C 3 × 10 3
In this time interval, the wheel rotates through an = 20 cms–2. Acceleration of top point A wrt centre is
angle R a (Æ) = 20 cms–2 (Æ)
2p \ Acceleration of A is
Dq = ____
​    ​ rad
500
Dq 2p aA = 20 + 20 = 40 cms–2 (Æ)
\ w = ___
​   ​ = _____________
​        ​
D t 1
500 × __
​   ​  × 10 –5 7. If disc I rotates by an angle q, it wraps x = R q
3
length of the thread. Disc II will move up by
= 3.77 × 103 rad s–1  3.8 ×103 rad s–1
x R q
Linear speed of a point on the edge is ​ __  ​ = ___
​   ​  
2 2
Solutions  S.55

3v0
Worksheet 3 \ vA = ___
​   ​  
2
1. (i) If disc rotates through q, a length x = Rq
dvA 3 dv0
unwinds and the centre falls through x. fi ​ ___ ​  = __
​   ​  ___
​   ​ 
dt 2 dt
x = Rq
3
dx
fi ​ ___ ​ =
dq
  R ​ ___ ​   fi  v = Rw fi a = __
​   ​  a0 [a0 = acceleration of the centre]
dt dt 2
fi 4 = 0.2 w fi  w = 20 rad s–1 2
fi a0 = __
​   ​  a
(ii) Vertical part of thread has no speed. Point C of 3
the disc must also be at rest.
4. Initial velocity of centre = u (≠)
vc = wR (≠) + v (Ø) = 0
2u

​_›
Velocity of A wrt centre is ​v ​  ACM = wR(Æ) ​ ___ –1
g ​   = 2 s  fi  u = 10 ms .
= v (Æ).
In two seconds, the stick rotated once. Hence, angular
Actual velocity of A is obtained by adding veloc-
​_› speed of the stick is
ity of the centre v (Ø) to the velocity ​v  ​ACM

______ D q 2p rad
__ __ w = ​ ___ ​ = ______
​   = p rad s–1.
 ​ 
​ v  2 + v 2 
\  vA = ÷  ​= ÷
​ 2 ​     ms–1
    v = 4​÷2 ​ D t 2 s
​_›
Direction of ​v  ​A  makes 45° with horizontal. p
In 0.5 seconds, the stick rotates by ​ __ ​  radian. It is
2
vertical in this position. Velocity of ends A and B
wL
​   ​  =  ​p   ​ 
1
relative to the centre of the stick is ___
​   ​  = p × __
2 2 ​2​
rad s–1 in the directions shown. Velocity of A and B
relative to ground is obtained by adding the velocity
of centre to this.
The COM moves with acceleration g (Ø).
\ Velocity of COM at t = 0.5 s is

v = u – gt = 10 – 10 × 0.5 = 5 ms–1 (≠)

÷  (  ) ÷  (  )
__________ ________
wL 2 p 2
2. Speed of belt = speed of a point on circumference \ vA = vB = ​ v + ​​ ___  ​    ​​ ​ ​ = ​ 52 + ​​ ​ __ ​   ​​ 
​     2
​ ​
of the cylinders 2 2

fi v = w1R1 = w2R2 = 5.24 ms–1


w1R1
fi w2 = _____
​   ​  
R2
3. Let velocity of the centre of the spool be v0 and its
angular velocity be w.
v0 = wR
Velocity of point B is
v0 v0 3v0
vB = wr + v0 = __
​   ​  r + v0 = __
​   ​  + v0 = ___
​   ​  
R 2 2
It means that the thread
at B is moving at a speed
3v0
___
​   ​  . Block A must also be
2
moving at this speed.
Chapter 6 Rotational Dynamics

Your Turn 8. Distance of centre of rod BC from the COM of the


triangle is
1. (a) Ix = m1r12 + m2r22 + m3r23
l
= 0 + 20 × 62 + 30 × 32 r = BD ◊ tan 30° = ____
​   __ ​ 
  
2​÷3 ​
= 990 gcm2
(b) Iy = 10 × 22 + 0 + 30 × 42 D is COM of rod BC. MI of rod BC about an axis
through D and perpendicular to the plane of the
= 520 gcm2
figure is
(c) Iz = 10 × 22 + 20 × 62 + 30 × (32 + 42) ml2
ICM = ___
​   ​ 
= 1510 gcm2 12
MI of rod BC about a parallel axis through COM of
the triangle is given by parallel axis theorem

  r =
______
​÷ 32
+ 42 ​ 

ml2
I = ICM + mr2 = ___
12 (  )
l __ 2
​   ​ + m ​​ ​ ____
   ​  ​​ ​ =
2​÷3 ​  
ml2
___
​   ​ 
6
 .

ml2
MI of all three rods will be thrice of this = ___
​   ​ 
 .
2
A

2. I = Irod + Iparticles
m (2l)2
= ______
​   + ml2 × 2 m
 ​  m CM
12
 
7
= __ ​   ​  ml2 r
3 30°
B C
3. All particles are at a distance R from the axis. D

4. All particles are at distance R from the axis. 9. Consider an element of length dx as shown. Mass of
5. The disc can be divided into 12 sectors of 30° each. the element is
Each sector has identical mass distribution about the dm = l dx = l0 x dx
given axis. Hence, each sector has same MI.
\ dI = x2 dm = l0 x3 dx
11 11
\ I = ___
​   ​ × Idisc = ___
​   ​ × 1.2 = 1.1 kgm2. L
L4
12 12 \ I = Ú dI = l0 ​Ú ​    ​x3 dx = l0 ​ __ ​ 
0 4
6. Disk with smaller density will have larger radius for
the two disks to have same mass. Thus, it’s MI will
be larger. x dx
7. I = IAB + IBC + ICD + IDA x=L
x=o
ml2 ml2
= 0 + ___  + ml2 + ___
​   ​  ​   ​ 

3 3
2
5ml 2
= ____
​   ​   10. I = ​ __ ​  MR2
3 5
Axis DI DR
​ ___   ​ = 2 ​ ___   ​
B C I R
DI DR
( 
​ ___   ​ × 100 = 2 ​ ___
I )
​     ​ × 100  ​ = 2 × 0.1 = 0.2%
R
11. MI will be minimum about an axis passing through
A D COM of the system.
[About any other axis I = ICM + Md2 (> ICM)]
Solutions  S.57

mL ML 16. a = R a ...(i)


r1 = ______
​   ​  and  r2 = ______
   ​     ​
m  +  M m+M For block:  mg sin q – T = ma ...(ii)
MmL2
\ ICM = Mr21 + mr22 = ________
​    
 ​ (M + m) For pulley:  TR = I a
(M + m)2
1
TR = __
( 
​   ​  MR2 ◊ a
= ​ ______
​ 
M+m
Mm
   ​  ​ L2 ) 2
1
T = ​ __ ​  Ma ...(iii)
ICM 2
Adding (ii) and (iii) gives
CM
m g sin q
M
r1 r2 a = ______
​     ​
M
m + __​   ​ 
2
12. Let r = density of sphere
4 3M
M = r __ ​    ​ p (a3 – b3)  fi  r = __________
​      ​ T
3 4p (a3 – b3) a
I = ​I​sphere of ​ – ​I​sphere of ​ a T
​        ​ ​        ​
​ radius  a ​ ​ radius b ​

[  ] [  ]
q
2 4 2  4 sin
= __ ​   ​  ​ r ◊​__
​   ​  ​ r ◊ ​ __ ​  p a3   ​ a2 – __    ​  p b3   ​ b2 mg
5 3 5 3 q

  ​ 
5 ( a   –  b )
= ​ __ ​  ​( __
​   ​  p r )​ (a – b ) = ___
5 5
2 4 2M a______
5 –b
5 17. The relevant equations are:
​   ​ ​    ​  ​
5 3 3 3
a = Ra ...(i)
1 1
13. For solid cylinder = ​ __ ​  MR2  fi  k1 = ___
Mk12 ​  __  ​ R m1g – T1 = m1a ...(ii)
2 ÷   
​ 2 ​
2 2
For cylindrical shell Mk2 = MR   fi  k2 = R T2 – m2g sin q = m2a ...(iii)

k1 1 T1 R – T2 R = I ◊ a


\ ​ ___  ​ = ___
​  __  ​ 
k2 ÷ I
  
​ 2 ​ T1 – T2 = ___
​  2   ​  a ...(iv)
R
2 2
14. I = ​ __ ​  mr2 = __
​    ​ × 2 (0.5)2 = 0.2 kgm2 Add (ii), (iii) and (iv) to get the desired result.
5 5
t = F ◊ r = 10 × 0.5 = 5 Nm T2
a
t 5
\ a = ​ __ ​ = ___
​    ​ = 25 rad s–2 a T2 T1
I 0.2
T1
Using w = w0 + a t ; w = 0 + 25 × 3 = 75 rad s–1
q a
sin
15. (i) Torque on the rod about it’s rotation axis through mg
A is m1g

l
t = mg ​ __  ​  sin q 19. Net torque on (disk + particle) system is
2
t = mg R sin q
\ Using Ia = t gives
MI about horizontal axis
a
ml 2
l through O is
​ ___ ​ ◊ a =

  mg ​ __  ​  sin q
3 2 1 O
__ I = __
​   ​  MR2 + mR2 R
3 g g
    __
3​÷3 ​ 2 q
fi a = ​ __ ​  __
​   ​  sin 60° = ____
​   ​ 
 ​   ​  mg R sin q
2 l 4 l t P R sin q
\ a = __
​   ​ = __________
​       ​
(ii) In vertical position, line of mg passes through A.
I M
​ __ ( 
​   ​  + m  ​ R2
2 ) mg
There is no torque about A.
2mg sin q
\ a = 0 = __________
​      ​
(M + 2m) R
S.58  Mechanics II

Speed of particle immediately after release = 0. 1 a Ma


Equation (ii) can be written as T = __ ​    ​ M ​ __  ​ = ___
​   ​ 

\  Radial acceleration = 0 2 2 4
Substituting in (iv) gives:
2mg sin q
Tangential acceleration at = R a = ________
​   

 ​ Mg – ​ ___ ​ 
Ma
 = Ma  fi  a = ___
4g
​   ​ 
(M + 2m) 4 5
20. (i) Torque about O 22. When v > wR, contact point has velocity in forward
t = mgl direction. Friction is in backward direction. Torque
due to friction increases w.
Moment of inertia about rotation axis through
23. wR = 2v
O is
1 Contact point has a velocity towards left. Friction
I = ICM + Md2 = ___ ​    ​  M (4l)2 + Ml2
12 acts towards right.
7 f
= __ ​   ​  Ml2 a = __
​    ​ 
3 M
t Mgl 3g \  Velocity at time t is
\ a = __​   ​ = _____
​  = ___
  ​  ​   ​ 
I __ 7 7l f
​   ​  Ml2 v0 = v + __
​     ​ t ...(i)
3 M
N fR 2f
t
a
a = __
​   ​ = ​ ______ = ____
   ​  ​    ​ 
I __ 1 2 MR
​   ​  MR
O 2

\  Angular velocity at t is
Mg aCM
2ft
w0 = w – a t = w – ____ ​    ​ 
(ii) COM will move in a circle of radius l about O. MR
It will have no speed immediately after release, 2v 2ft
= ___ ​   ​ – ​ ____  ​   ...(ii)
hence no radial acceleration. R MR
Tangential acceleration of COM is When pure rolling (i.e, no sliding) begins
3g v0 = w0R
at = l ◊ a = ___
​   ​ (Ø)
7
f 2ft
fi v + __
​    ​  t = 2v – ___
​   ​ 
(iii) Mg – N = M ◊ aCM M M
3Mg ____4Mg 3ft
fi  N = Mg – ____ ​   ​ 
 = ​   ​ 
.  fi ​ ___ ​  = v
7 7 M
21. For upper disc: Putting this value of time in (i) gives
a1
1 R
TR = __
​   ​  MR2 ◊ a1 f v 4v
2 v0 = v + __
​    ​  t = v + __
​    ​ = ___
​   ​ 
M 3 3
1
fi T = __
​   ​  M (R a1) ...(i)
2 T
T
For lower disc [about its COM]
1
TR = __
​   ​  MR2 ◊ a2 R
2
a2
1
T = __
​   ​  M (R a2) ...(ii) a
2 Mg

From (i) and (ii) a1 = a2.


Linear acceleration (a) of the disc results due to
unwinding of thread from both the discs.

\ a = Ra1 + Ra2 = 2Ra2 ...(iii)


For translation:  Mg – T = Ma ...(iv)
Solutions  S.59

24. (a) Refer to Example 23. adisc > aring. F


a
g sin q g sin q 5g sin q
(b) a = ______
​    = ______
 ​  ​   ​ = _______
  ​   ​   2F
R
k 2
___ 2
__ 7
1  + ​  2  ​  1 + ​   
 ​
R 5 a

÷ 
_________
10
\ v = 0 + 2aL  fi  v = ​ ___
2 2
   ​
​   ​ gL sin q 
7

÷ 
_________ f
v 1 ___ 10
w = __
​ r ​ = __    ​
​ r ​  ​ ​   ​ gL sin q 
7
f ◊ 2R = 2MR2 ◊ a
Mg sin q Mg sin q Mg sin q
25. f = ________
​  2  ​ = _______

  ​   = ​ _______

 ​  ​   fi 2f = M (2Ra)
MR
____ 1 + 1 2
​     ​ + 1 fi 2f = Ma  [   a = a (2R)] ...(ii)
I
f £ mN F
Solving (i) and (ii) f = ​ __ ​ 
3
Mg sin q 1
\ ​ _______  ​   £ m mg cos q  fi ​ __ ​  tan q £ m 30. Let acceleration of centre of the cylinder be a and
2 2 its angular acceleration be a.
1
fi ​ __ ​   £ m For no slipping a = Ra ...(i)
2
Speed of block = speed of top point of cylinder
26. There is no friction. There is no torque about centre.
There is no rotation. = twice the velocity of centre of
27. Friction is kinetic. the cylinder

f = m N = m Mg cos q
\ Ma = mg sin q – m mg cos q
fi  a = g (sin q – m cos q)

t f ◊ R 5m Mg cos q 5 m g cos q


And a = ​ __ ​ = ​ ______ = __________
  ​  ​       ​ = ​ _________
       

I 2 2M ◊ R 2R Thus, acceleration of m must be twice that of the
​ __  ​ MR 2
5 cylinder = 2a.
28. Let friction (f) be in forward direction. T + f = Ma ...(i)
F + f = Ma ...(i) 1 1
F
TR – f R = __​   ​  MR2 a  fi  T – f = __
​    ​ Ma ....(iii)
2 2
1
__ 2
And FR – fR = ​   ​  MR  ◊ a
2 a And  mg – T = m (2a) ...(iv)
a
1 Solving the above equations gives
fi F – f = __
​   ​  M (Ra)
2 f 4mg
a = ________
​      ​
1 3M + 8m
fi F – f = __
​   ​  Ma ...(ii)

( 
2

Solving (i) and (ii) a = ___


4F
​     

1
31. K = __
2
1
​   ​  ICM w2 + __
​   ​  M​v2​CM
2
1 __
__
  ​ = ​   ​ ​
2 2
1
) 1
  ​    ​ MR2   ​ w2 + ​ __ ​  Mv2
2
3M
29. Net torque due to two forces is 2FR ( ) + F (2R)( ) = 0. 1 1 3
= __
​   ​  M (Rw)2 + __
​    ​ Mv2 = __
​   ​  Mv2.
The spool will have a linear acceleration towards left. 4 2 4
It must have an anticlockwise angular acceleration to 1
__
avoid slipping. Thus friction is to the right. kT ​    ​ Mv2
__ 2
________ Mv2
32. ​   ​ = ​    ​ = _______
  ​  2    ​ 
= 1 [   Rw = v]
kR 1
__ 2
2F – F – f = Ma ​   ​  ICM ◊ w2 MR  w
2
F – f = Ma ...(i)
f ◊ 2R = Ia
1
33. I = ​ __ (  )
​   ​  ml2   ​ × 3 = ml2
3
S.60  Mechanics II

1 1 It does not dissipate mechanical energy. Volume of


\ k = ​ __ ​  Iw2 = __
​   ​  (ml2) w2

34.
2 2
Wt = t D q = 20 × (2 × 2p) = 80 p J.
R
cylinder with radius ​ __ ​  becomes
2
1 th
​​ __ (  )
​   ​   ​​ ​ the original.
4
M
Thus, mass of the unrolled cylinder is __ ​   ​ .
\ KE = Wt = 80 p J 4
35. P = t ◊ w
P is maximum when w is maximum, i.e, at the end
of two rotations.
Final w can be calculated as:
1 160 p
​ __ ​  Iw2 = 80 p  fi  w = _____ ​ = 160 rad s–1
​  p   

2 M R 7
Loss in PE = MgR – __ ​   ​  g ​ __ ​  = __ ​   ​  MgR.
\ Pmax = (20 Nm) (160 rad s–1) = 3200 watt. 4 2 8
3
37. (a) Energy conservation. KE of a purely rolling cylinder is __ ​   ​  mv2
4
gain in KE = Loss in PE 3 M 2 ___ 3
\  KE of rolling part = __ ​   ​  __
​   ​  ◊ v = ​    ​  Mv2
1
2
L
​ __ ​  IA w2 = Mg ​ __
​   ​  +
2 [  L
__
]
​   ​  sin 30°  ​
2 3
Energy conservation: ___
4 4
​    ​  Mv2 = __
7
​   ​  MgR
16

16 8
1 ML2 2 __ 3
fi ​ __ ​  ____
÷ 
​   ​ w

  = ​   ​  Mg L _____
2 3 4 14gR
fi  v = ​ _____
​   ​ ​ 
  

÷  ÷ 
____ ___ 3
g g g
fi w = ​ __
​   ​  ​ __  
​ ​ = 3 ​ ___
​    ​ ​  
2 L 2L 40. At B, KE of the ring = Mg (H – h)
1 1
fi  ​ __ ​  Mv2 + __ ​   ​  I w2 = Mg (H – h)
2 2
1 1
fi  ​ __  ​ Mv2 + __
​   ​  MR2 w2 = Mg (H – h)
2 2
fi  Mv2 = Mg (H – h)
________
\  
v = ​÷g (H – h) ​

Now, the ring is a horizontal projectile projected


from B at speed v. [Remember that rotational KE of
the ring will not change once it is in air. There is no
torque on it]

÷  ÷ 
___ ___
(b) In vertical position, COM of the rod has a radial 2h ________ 2h _________
acceleration (towards A) given by \  CD = v ​ ​ ___
g  ​ ​
   
 = ​÷g (H – h) ​
  ​ ___
​  g  = ÷
 ​ ​
  ​  
2h (H – h) ​

​_›
9
a = w2 ​ L    ​ = __
​   ​  g 42. Distance of line of ​P  ​  from point of suspension is
​2 ​ 4 always l. Hence, L is maximum when P (i.e, speed)
If hinge force is F, then is maximum. Speed is maximum at lowest position.
1 ___
9 13
F – Mg = Ma  fi  F = Mg + ​ __ ​  Mg = ___
​   ​  Mg ​ __ ​  mu2 = mgl  fi  u = ÷   ​ 
​ 2gl 
4 4 2
___
38. Work done by friction = change in KE of the ball \    ​
Lmax = mu ◊ l = ml ​÷2gl 

\ Wf = kf – ki 43. (a) At highest point, momentum (P) is horizontal.


Perpendicular distance of line of P from O is

10
7
= ___
5v 2
​    ​  M ​​ ___ (  )
​   ​   ​​ ​ –
7
1
__ 1
​   ​  Mv2 = – ​ __ ​  Mv2
2 7
\
equal to maximum height (H).
L = mux ◊ H  [using L = r^P]
39. As the carpet unrolls, it loses PE and gains KE. The
part that unrolls, lies on the floor without any KE. u2 sin2 q _____________
mu3 cos q ◊ sin2q
= m(u cos q) _______
​      
​ = ​       
​ 
There is no sliding (rubbing) and friction is static. 2g 2g
Solutions  S.61

aRT2 MR2 aRT2


fi Iw = _____   fi ​ ____
​   ​     = _____
 ​ ◊ w
  ​   ​ 

2 2 2
aT 2
fi w = ​ ____ 
 ​
MR
50. Velocity (v) acquired by the COM is independent of
h. Angular speed (w) about centre will be maximum
when the angular impulse is maximum. This happens
(b) When the projectile is about to hit the ground when line of force is at maximum distance from the
its velocity is u directed at an angle q to the centre.
horizontal.
w = 0 (and hence KE is minimum) when line of
Using L = P^ ◊ r action of the force passes through COM.
L = (P sin q) ◊ R 51. Moving to equator means moving farther from the
where R = horizontal range. rotation axis of the Earth. This will increase the
u2 sin 2q moment of inertia of the Earth. Angular momentum
= (mu sin q) _______ ​  g    ​  of the Earth associated with its spin about its rotation
3 2
2mu  sin q ◊ cos q axis cannot change due to internal interactions. This
= ______________ ​  g    
​  implies that angular speed (w) will decrease and
44. This is a fixed axis rotation. hence the Earth will take more time to complete one

[ 
rotation.
MR2
\  L = I ◊ w = ​ ____
​   ​ 
2
R 2
 + M ​​ __
2 (  ) ] 3
​   ​   ​​ ​  ​ w = __
​   ​  MR2w
4
53. I0w0 = Iw

1
45. L = Lspin + Lorbitals = __
​   ​  MR2w + MvR
2
MR2
​ ____
2
 ​ w

MR2
  0 = ​ ____
​   ​ 
2 [ 
 +  mR2   ​ w ]
( 
1 1 3
= __ ​   ​  MR (wR) + MvR = __
2
​   ​  MvR + MvR = __
2
​   ​  MvR
2 fi ​ 
M
w = ​ _______
  
M + 2m 0 )
 ​  ​ w

[ wR = v]

46. L = Lspin + Lorbitals = ICM ◊ w ( ) + MvR ( )


MvR
= MR2w ( ) + MvR ( ) = ____ ​   ​ (

  ) + MvR ( )
3
2
= __ ​   ​  MvR ( )
3 54. Due to inertia, Velocity of ball before and after the
2 2
47. L = Lspin + Lorbital = __
​    ​ MR2w + 0 = __
​   ​  MR ◊ v release are equal. Angular momentum of the ball wrt
5 5 the rotation axis is same before and after the release.
The line of velocity of COM passes through A. Therefore, w for boy + platform system will also not
Dt change in order to conserve angular momentum.
48. Angular impulse = Ú​  ​  ​ ​ t dt = area under the graph 55. (a) This is similar to previous problem. Angular speed
0
of the disc will not change.
1
= __
​   ​  × 0.0 1 × 20 = 0.1 N.m.s (b) Vertical velocity of the chip just after separation,
2
u = wR (≠).
DL = 0.1
u2 w2R2
0.1 \ h = ​ ___ ​ = _____
​      

Iw = 0.1  fi  w = ___
fi ​   ​  2g 2g
0.4
56. Tension in the cord does not produce any torque on
= 0.25 rad s–1.
the block about the centre of the circle.
49. Torque t = RF = aRt Angular momentum of the block about centre remains
T T conserved.
aRT2
Angular impulse = Ú​  ​    ​t dt = aR ​Ú ​    ​t dt = _____
​   ​ 
. 
0 0 2 mv1r1 = mv2r2 ...(i)

aRT2 Tension in the cord when radius is r2 is


\ D L = ​ _____
 ​ 

2
S.62  Mechanics II

mv22 59. Conservation of momentum tells us that COM of the


T2 = ​ ____
r2 ​  
  ...(ii) rod will move parallel to u with a velocity v given

[  ]
T2r2 __​ 12 ​   by
From (i) and (ii); mv1r1 = m ​​ ____ ​  ​​ ​ ◊ r2
​  m    u
2mv = mu  fi  v = ​ __ ​ 

[  ]
2
[ 
mv12 r21 __​ 13  ​
]
1
__
4  ×  16 × 0.25 ​ 3 ​ 
fi r2 = ​​ ______
​   ​    ​​ ​ = ​​ ​ ____________
 ​ 
     ​​ ​ = 3 m. Let angular speed of the rod be w.
T2 600
We will conserve angular momentum about a point
57. Speed of the ball after falling through L is fixed to the table which is just beneath the centre C
____
   
u = ​÷2gL 
​ of the rod.

Angular momentum of the ball + pulley system about Li = Lf


O just before and after the string gets taut remains
L
same. m (u sin q) ​ __ ​ =
  Lspin + Lorbital
2
2
L (2m) (L )
fi  mu sin q __
​   ​  = ________
​   ​ w

 + 0
2 12
3u sin q
fi w = _______
​   ​    [vcm is along the line
L
passing through the
origin (just below C)]

[Note:  Weight of the ball (mg) has a torque about


O. But this torque has negligible impulse during the
short duration in which the string gains tension]
1
muR = mvR + __ ​   ​  MR2 ◊ w
2
1
mu = mv + __
​   ​  M (Rw)
2
60. (a) Conservation of momentum:
1
mu = mv + __
​   ​  Mv [   v = Rw] mu
2 (M + m) v = mu  fi  v = ______
​     ​
M+m
2mu
\ v = ​ _______
    ​ (b) We will conserve angular momentum about
2m + M
COM of the combined system.
58. Conservation of angular momentum about the rotation
axis of the disc gives: Li = Lf

( 
u 1
m ___

​  __  ​  R + ​ __  ​ MR2 w = muR
÷   
​ 2 ​ 2
[KE of the bullet is
halved. This means its
u__

2
mu (h – R) = ​ __
​   ​  MR2 + mR2   ​ w
5 )
speed becomes ​ ___  ​]  mu (h – R)
÷   
​ 2 ​ fi w = ​ ____________   
   ​
( 
__ __
÷    (÷
​ 2 ​ 
fi w = ​ ____________
    – 1) mu
​ 2 ​
MR
     ​ 
2
​ __ )
​   ​  M + m  ​ R2
5
Solutions  S.63

Using perpendicular axis theorem


Worksheet 1
Iz ML2
I + I = Iz  fi  I = __
​   ​  = ____
​   ​ 

1. Square of side 2a has its mass particles placed at 2 12
largest distance from the axis.
Ma2
9. ICM = ____
​   ​ 
2. Mass of loop, m = r L 12
L Ma2
Radius of loop, r = ___
​     ​ \ I = ____
​   ​ + Md2
2p 12
1
ICM = __
(  )
​   ​  mr2 Ma2 a 2 7
2 = ____
​   ​ + M ​​ ___
​  __  ​  ​​ ​ = ___
​    ​  Ma2
12 ÷   
​ 2 ​ 12
(  )
3
1 3 3 L 2 3rL
Ixx = __
​   ​  mr2 + mr2 = __
​    ​ mr2 = __
​   ​  r L ​​ ___
​    ​ ​ ​ = _____
​  2 ​ 
2 2 2 2p 8p

3.
ML2
I1 = ____
​   ​  
12
L 2
 ; I2 = Mr2 = M ​​ __
​ p ​ ​ ​ (  )
I1 ___ p2
\ ​ __ ​ =
  ​    ​ < 1
I2 12
4. If a disc of mass M is folded about its diameter
(perpendicular to AB), distances of particles from line
AB do not change.
10. Two masses lie on axis. They do not contribute to
\ I = MI of a disc about diameter (AB)
MI. 4 masses are at separation
__ a from the axis and
1
= __ ​   ​  MR2 2 masses are at separation ÷     a.
​ 2 ​
4 __
5. If I CM is MI abut a line through centre of the rod that \   I = 4 ◊ ma2 + 2 ◊ m (​÷2 ​
    a)2 = 8 ma2.
is parallel to z-axis, then MI about any other parallel 11. MI is minimum about an axis through COM.
axis is
I = 2x2 – 12x + 27
I = ICM + Mr2
dI
where r = distance of the axis I is minimum if ___
​    ​ = 0
from the line through COM. If dx
I is to remain constant, then r fi 4x –12 = 0  fi  x = 3
must remain constant. Thus the
locus is a circle with its centre 12. COM is farthest from B
at the centre of the rod \  IB is maximum
6. MI of upper ring I1 = mR2 + mR2 = 2mR2 rot 500 × 2p ____50 p
14. w = 500 ____
​    ​ = ​ ________
 ​ 
   rad s–1.
= ​   ​ 
MI of lower ring min 60 3
1 Angular retardation produced due to braking can be
I2 = __
​   ​  2
+ m (3R)2 = 9.5 mR2
2 calculated as
\ I = I1 + I2 = 11.5 mR2 50 p
w = w0 + a t  fi  0 = ____ ​   ​   – a × 15
3
( 
2
7. I = ​ __
​   ​  MR2 + MR2   ​ +
3 ) (​  __​ 23 ​  MR
2
) 34
+ (3R)2   ​ = ___
​   ​  MR2
3 fi  a = ____
10 p
​   ​ 
9
 rad s–2
8. MI about an axis through junction point and Friction, f = mN = 100 m.
perpendicular to the figure is Torque tf = mN ◊ r = 100 m × 0.25 = 25 m Nm
1 1
ML2 ML2 ____ ML2 Moment of inertia, I = __ ​   ​  mr2 = ​ __ ​  × 15 × (0.25)2
Iz = ____
​   ​ 
 + ​ ____ ​ 
 = ​   ​ 
  2 2
12 12 6 15
= ___
​   ​ kg m2
Consider the given axis and one more axis 32
perpendicular to it in the plane of the figure. MI about Using t = I a
both these axes must be same due to symmetry. 15 10p
25m = ___​   ​ × ____
​   ​  
  fi  m = 0.065
32 9
S.64  Mechanics II

15. To loosen the nut we need a certain minimum 5 5 4


OA = __
​   ​  ◊ sin q = __
​   ​  × __
​   ​  = 1
torque. 4 4 5

[ 
6 × 8 × sin 30° = F × 16  fi  F = 1.5 N
1
16. k = __
​   ​  Iw2 = __
1 L
​    ​  (Iw)2 = __
2
​    ​

5/3 4
tan q = ___
​   ​ = __
5/4 3
4
5 ]
​   ​   fi  sin q = ​ __  ​  ​

2 2I 2I \ Angular momentum of the particle about O is

÷ 
__
L12 L22
L1 I1 1__ L = mv (OA) = 2 × 8 ×1 = 16 kg m2 s–1.
\ ​    ​ = ​    ​   fi ​ __ ​  = ​ __
___ ___ ​    ​ ​  = ​ ___  ​ 
2I1 2I2 L2 I2 ÷   
​ 2 ​
L
22. tA = Mg ​ __ ​  cos q
17. For no rotation, the force must be applied at the COM 2
of the body. Thus P is COM.
ML2 L
Distance of COM from C is ​ ____  ​ ◊ a =

  Mg __ ​   ​  cos q
3 2
2ml + m ◊ 2l __ 4l
x = ​ __________
      
​ = ​   ​  3g
3m 3 fi a = ___
​    ​ cos q
2L
18. For no rotation f = mg sin q
\ tf = f R = mg R sin q
tf
\ Angular retardation, a = __
​   ​ 
I
mg R sin q 2g sin q
= ________
​  = ​ _______
 ​ 
      ​ 
1
__ R
​   ​  mR2
2
Using, w = w0 + a t Acceleration of end B immediately after release is
tangential acceleration perpendicular to the rod
( 
2g sin q)
0 = w – ​ _______
​ 
R
      ) ​ 
Rw
​  ​ t  fi  t = _______
  ​ 
2g sin q
3g
aB = a L = ​ ___ ​ cos q.
2
Once the cylinder stops spinning, it will begin to roll
down the incline.
1 1
2
5
R 2
23. I2w2 = I1w1  fi ​ __ ​  M ​​ __
2 (  ) 2
​   ​   ​​ ​ w2 = __
​   ​  MR2 w1
5
19. ​ __ ​  ICM ◊ w2 = 0.4 ​ __ ​  Mv2
2 2 fi w2 = 4w1
2 2
fi  ICM w = __ 2
​    ​ ◊ M (wR)2  fi  ICM = __ ​   ​  MR2 Angular speed becomes 4 times. It means it will
5 5
1
__
​    ​ ICM w2
2
_______________ 1
1 th
take ​​ __
4 (  )
​   ​   ​​ ​ of 24 hr to complete one rotation.
20. f = ​        ​ = ________ ​       ​ [ v = wR] ​_›
1
__ 1 MR2
​   ​  ICM w2 + __ ​   ​  Mv2 1 + ____
​   ​

24. ​r ​   = 3 cos q + 4 sin q 
2 2 ICM ​_›
​_› d ​r ​   dq dq
f is maximum when ICM is maximum. ​v  ​   = ___
​   ​ = – 3 sin q ___
​   ​  + 4 cos q ​ ___ ​ 
dt dt dt
Maximum value of ICM for a ring is equal to MR2
dq
\  fmax = __
1
​   ​  ​ ___ ​  = w = angular speed about O.
2 dt
21. The line of motion is as shown. perpendicular dis- ​_›
tance of line of motion from O is \ ​v  ​   = (– 3 sin q + 4 cos q ) w
_
​› ​_› ​_›
​L  ​   = ​r ​   × (m​v  ​) 
= m (12 cos2 q + 12 sin2 q ) w = 12 mw

\ 0.6 = 12 × 0.01 × w

fi w = 5 rad s–1
\ q = wt = 5t rad
Solutions  S.65

28. t is constant.
\ a is constant.
\ w = a t
P t = t ◊ w = t a t
Thus power µt. Graph of P vs t is a straight line
25. Friction does not dissipate energy in pure rolling passing through origin.
when the cylinder stops it has no KE. 29. Hinge force is equal and opposite to applied force
(F) as
1 2Mg sin q
​ __ ​  kx2 = Mg x sin q  fi  x = ________
​       
​ Fnet = 0   [ aCM = 0]
2 k
Hinge force has no torque about centre. Applied force
26. Angular momentum is conserved about O.
has a torque. Thus w increases.
Li = Lf
30. Process B has more inertia, therefore less
a
Mv __
​   ​ =
  I0w acceleration.
2
31. When the cylinder moves up its velocity (v) decreases.
a 2
Mv __
​   ​ =
  __
​   ​  Ma2 ◊ w to maintain pure rolling, w must also decrease. For
2 3 this friction is up providing an anti clockwise torque.
3v We already know that friction is up the plane when
\ w = ___
​    ​ moving down.
4a

27. mg – T1 = ma1 32. Initially (when stretch in spring is small), mg sin q is


\ T1 = mg – ma1 ...(i) larger than spring force kx and the cylinder accelerates
down. Friction must be up the plane to provide the
And T2 – mg = ma2 necessary angular acceleration for rolling. As the
T2 = mg + ma2 ...(ii) spring stretches beyond equilibrium position, spring
From (i), T1 is less than mg and from (ii), T2 is larger forces exceeds mg sin q. Now the cylinder retards. It
than mg. must also experience angular retardation. For this the
friction assumes a direction down the incline.
\ T2 > T1
33. MI about rotation axis through P is
Note:  Torque due to T1 is larger than torque due to
I = IPQ + IQR
T2. Thus, pulley rotates clockwise
[T1R > T2r]
1
= __
1
​   ​  ml2 + ​ ___
3 [ 
​    ​ ml2 + mr2   ​
12 ]
1
= __
1
​   ​  ml2 + ___
3
l2
​    ​  ml2 + m ​ l2 + __
12 (  5
​   ​   ​ = __
4 )
​   ​  ml2
3

Energy conservation:
1 l l 1 5 2 2
​ __ ​  Iw2 = mg ​ __  ​  + mg ​ __  ​   fi ​ __ ​  __
​   ​  ml w = mgl
2 2 2 2 3

÷ 
____
6 g
fi w = ​ __
​   ​  __
​   ​ ​ 

5 l
S.66  Mechanics II

34. F – f = ma ...(1) Speed of point Q is


FR 1 vQ = (AQ) w
​ ___ ​ + f ◊ R = ​ __ ​  mR2 ◊ a
2 2
For C: vC = (AC) w
1 1
​ __ ​  F + f = __
​    ​ ma ...(2)
2 2 For P: vP = (AP) w
[ Ra = a] Since, AQ > AC > AP
Eliminating a between (1) and (2) gives \ vQ > vC > vP
F – f = F + 2f  fi  f = 0.

Note:  The applied force itself ensures a = Ra and


therefore friction will adjust itself to zero.
4
35. Friction at both surfaces is kinetic and directed 37. M1 = __
​   ​  p R3 ◊ (2d)
opposite to the contact point. 3
In case (a) 4
M2 = ​ __ ​  p (2R)3 (d) = 4M1
3
fa = N2  fi  mN1 = N2 ...(i)
k1 = k2
And f + N1 = mg  fi  mN2 + N1 = mg ...(ii)

÷  ÷ 
___ __
L12 L22 L1 M1 1 1
fi ​ ____  ​ =
  ____
​    ​   fi ​ __ ​  = ​ ​ ___  ​ ​ = ​ __
​    ​ ​  = __
​    ​
2M1 2M2 L2 M2 4 2
38. Since, the ball begins its motion with pure rolling,
friction on it is zero. Thus there is no friction on the
plank. It will stay at rest.
39. Angular momentum is conserved.
   L2
K = __ ​    ​
mg 2I
Solving (i) and (ii) N1 = ______
​  2     ​ After folding of arms, new KE is K¢ = _____
L2 K
​    ​ = __
​   ​ 
m   +  1 2 (2I) 2
mmg 40. On each object, angular impulse is F.R.t. Thus change
\ fa = mN1 = ______
​  2    
 ​ in angular momentum is same for all of them.
m   +  1
41. Conservation of angular momentum (about A)
Figure shown for case (b) clearly shows that the gives:
sphere will definitely move towards right and there
will be no contact with the wall.
N2 = f = 0

\ fb = mN1 = mmg
fa 1 9
\ ​ __ ​ = ______
​       ​ = ​ ___  ​ 
fb m2 + 1 10

36. Since, the disc is rolling without sliding we can


consider it to be in pure rotation about contact Li = Lf
point. m 1
​ __ ​  v0L = __
​   ​  mL2 ◊ w
4 3
Solutions  S.67

3 v0
fi w = ​ __ ​  __
​   ​ 
4 L
Velocity of end B after collision is:
3
vB = wL = __​   ​  v0
4
relative speed of separation of end B
and the particle after collision
e =
relative speed of approach of the 46. When tortoise moves from A to C the MI of the
particle before collision system about rotation axis through O decreases.
3
__
​   ​  v0 As it moves from C to B the MI increases. During
4 3
= ​ ____
v0 ​   = __
​   ​  the entire course of motion angular momentum of
4
the system is conserved. Thus, angular speed first
42. Tension is always perpendicular to the direction of increases when tortoise moves from A to C and then
velocity of the ball. Hence, it decreases as MI increase when it moves from C to
tension does not perform B. It is easy to see that the variation is not linear.
work on the ball. KE of the
ball does not change. But 47. A massless rod will not apply a force perpendicular
tension has a torque about to its length.
O. Thus angular momentum \  Velocity of the particle immediately after
of the ball about O does change. impact = v. And velocity of the other particle
43. COM of the (hoop + particle) system is at a dis- immediately after impact = 0. Angular velocity of
v
R moving end wrt the stationary end is w = ​ __ ​ 
tance ​ __ ​  from O (at C). After collision COM moves l
2 48. Li = Lf (about centre of disc)
with velocity v0 given by:
v (I + mR2) w = Iw0 + mvR
2mv0 = mv  fi  v0 = __ ​    ​
2
(I + mR2) w – mvR
fi w0 = ​  _______________
       ​
I
49. Li = Lf (about C)

Ml2 l
​ ____ ​  ◊ w = mv ​ __  ​ 
12 3
fi Mlw = 4mv ...(i)
We will conserve angular momentum about a point
attached to the table that is just below C. Since, collision is elastic
R \  Velocity of approach = velocity of separation
Li = Lf  fi  mv __ ​   ​  = IC ◊ w
2 l
w __
[ 
​    ​   = v
R R 2
(  ) (  ) ]R 2
fi mv ​ __ ​   = ​ mR2 + m ​​ ​ __ ​   ​​ ​ + m ​​ ​ __ ​   ​​ ​  ​ w
2 2 2
3
3
Putting this in (i) gives, m = __
​   ​  M.
4
v
fi w = ___
​    ​ 
3R
44. Due to impact, linear motion of A will get trans-
ferred to B(head on elastic collision of equal masses).
There will be no change in angular speeds of the two
spheres as there is no angular torque on them.
45. There is no torque about rotation axis, hence angular
momentum is conserved.
Only force that performs work is gravity, thus 50. Immediately after impact, the velocity of the centre
mechanical energy is conserved. is
J
mu = J  fi  u = __ ​ m  ​ 
S.68  Mechanics II

Worksheet 2
1. In first case the man will continue to have tangential
speed (wr) due to inertia.
Thus angular momentum of the man wrt centres does
not change. Hence angular speed of the platform will
also not change. In second case, after the man steps
in, (due to friction) he will acquire rotation. Since
MI of the system increases, w must decrease
2. Friction remains constant throughout.
Using equation (9)
Angular speed about the centre is given by angular-
impulse momentum theorem. Mg sin q Mg sin q 2Mg sin q
f = _______
​  2  
 ​ = _______
​   
 ​ = ​ ________
 ​  
R
___ 3
__ 5
ml2 l ​  2  ​ + 1 ​   
 ​ + 1
​ ____ ​  ◊ w = J ◊ ​ __  ​  K 2
12 2
2Mg sin q

6J
w = ___
​    ​ But f £ mN  fi ​ ________  ​   £ m Mg cos q
ml 5

Angle by which the rod will rotate in time ​ 2  ​ tan q £ m


fi  __
5
p ml
t = ____
​   ​ is 3. MI about any tangent will be same.
12J
6J p ml p \ I1 = I2 = I4
q = wt = ___
​    ​  ◊ ​ ____ ​ = __
​   ​ 
ml 12J 2 From perpendicular axes theorem:
Velocity of point P wrt C at this time is Iz = I1 + I2 = I1 + I4
l J
vpc = w ​ __  ​  = __
​     ​ Distance of centre from O is:
6 m _______ __
\  Velocity of P relative to ground is r = ÷
​R  2 + R2 
 ​ = ÷  R
​ 2 ​

_________ __ J mR2 5
  pc)2 + u 
vP = ​÷(v 2
 ​ = ÷     __
​ 2 ​ ​ m    \ Iz = ICM + mr2 = ____  + m (2R2) = __
​   ​ 
2
​   ​  mR2
2

4. During uphill motion v decrease and w increases.
Thus there will be a point where v = wR. During slid-
ing, friction is kinetic. Its value is mN. Once sliding
ceases, static friction adjusts to a value given by
equation (9).
​_› ​_› ​_›
5. Angular momentum about O is ​L  ​  = ​r ​   × (m​v  ​) 
As the particle rotates, the plane
​_› ​_›
containing ​r ​   and ​_​v  ​  changes.

Thus direction of ​L   ​ changes._
​›
However, magnitude of ​L  ​ 
about O does not change.
​_›
Vertical component of ​L   ​ about
O is same as angular momen-
tum about line OC.
Lz = mvr (Directed up in the
diagram)
Lc is also mvr directed along Lz.
Solutions  S.69

6. Refer to Section 4.2. (i.e., it will remain zero). Thus, the body will finally
come to rest while having no spin.
9. For hoop: k1 = kT + kR = Mv12
3
For cylinder: k2 = kT + kR = __ ​   ​  Mv22
4
Since, k1 = k2
__
3 ÷   
​ 3 ​
\ v12 = __
​   ​  v22  fi  v1 = ___
​   ​ v2  fi  v1 < v2
For a ring (or a hollow cylinder) the force produces 4 2
F __
acceleration a = ​ __  ​  v1   v  2 ___
​÷3 ​   
​÷3 ​
__
M ​L​hoop
spin
​ ​ = ICM ◊ ​ __ ​  = MR2 ◊ ​ ____   ​ = ​   ​ Mv2R
and angular acceleration produced is R 2R 2
FR F F v2 1
a = ​ ____2  ​ = ____
​    ​   fi  a R = __
​    ​  ​  ​ = ICM ​ __ ​  = __
​L​cylinder
v 1
​  2 ​  = __
​    ​ MR2 __ ​    ​ Mv2R
MR MR M spin R 2 R 2
fi a = R a.
\ ​L​hoop ​ ​ π ​Lcylinder
​spin​  ​
FR FR2 spin
For any other object: a = ____
​  2  ​  fi  Ra = ​ ____2 
 ​ 
MK MK Since, both have same KE, they will attain same
R2
fi Ra = a ​ ___
​  2  ​  ​
K (  ) height on an incline. But they will have different
retardations.
g sin q g sin q
\ Ra > a a1 = ______
​      ​ = ​ ______  ​  
k
___
2 2
7. Assume the situation shown in figure. 1 + ​  2  ​ 
R
F – f = Ma ...(i) g sin q 2
a2 = ​ ______   ​ = __ ​   ​  g sin q
1 3
fR – Fr = MK  ◊ a 2
1 + ​ __ ​ 
2

R
r
f – F ​ __ (  ) MK2  
​    ​  ​ = ​ ____
R2
 ◊ a
 ​  ...(ii) Time needed by hoop to stop is given by
v1    v2
​÷3 ​ 
__
   v2
​÷3 ​ 
__

(i) + (ii) gives t1 = __


​ a   ​ = ________
​    ​  = ______
​     

g sin q g sin q

[  ]
1
2 ◊ ​ ______ ​   

R
r
F ​ 1 – __[  ] K2
​    ​  ​ = M ​ 1 + ___
​  2 ​   ​ a
R
2
v2 _______ v2 3 v2
r For cylinder, t2 = ​ __ a2  ​ = ​ __      ​ = __
​   ​  ______
​      ​
a is positive as 1 > __ ​    ​. 2
​   ​  g sin q
2 g sin q
R 3
\ t1 > t2

10. In case (i) there is no rotation about COM.


In case (ii) there is rotation. In both cases speed
acquired by COM is same.

12. I1 + I3 = Iz  But  I1 = I3


Iz
\  rotation is clockwise and the thread will wind \  I1 = I3 = __
​   ​ 
up if rotation is clockwise. 2
Iz
8. Friction applied by the table cloth will provide a Similarly, I2 = I4 = __
​   ​ 
​_›
2
linear as well as angular velocity to the body. But ​_›
d ​L  ​
14. ​ ___ ​ = ​t  ​ 

the friction force has no torque about a point like
dt ​_›
A. Thus angular momen- ​_› ​_› ​_› ​_›
d ​L   ​
tum of the body about A Since ​t   ​ (= ​A  ​  × ​L 
​_ ​) › is perpendicular to ​L  ​,  thus ​ ___ ​ is
dt
is zero [Lspin + Lorbital = 0]. perpendicular to ​L  ​. 
Angular momentum about _
​›
_
​› _
​›
A will be conserved after d ​L  ​ 
___
​   ​ is perpendicular to both ​L  ​  and ​A  ​. 
the table cloth is removed dt
S.70  Mechanics II

​_› ​_› 3
It means d ​L  ​  is perpendicular to ​A  ​.  = __
​   ​  MvR ( ) [ Rw = v]
​_› _
​› 2
It means component of ​L  ​  along ​A  ​  does
not change. 1
And LA = Lspin + Lorbital = __
​   ​  MR2w ( ) + MvR ( )
15. Consider any particle on the 2
rod.​_ Its angular momentum
› ​_› ​_›
is ​L 
​_›
  ​ = ​r ​   × (m ​v   ​). Direction
​_›
of ​L   ​ is perpendicular to ​r ​   as
​_›
well as ​v   ​.
In the position​_shown, point P

has velocity (​v   ​) directed out _
​›
of the plane of figure. Thus ​L  ​  1 1
is in the direction shown. = __
​   ​  MvR ( ) + MvR ( ) = __
​    ​ MvR ( )
2 2
As the rod rotates, vertical component of L does not
change but horizontal component of L is rotating. \ |LB| = 3 |LA|
Horizontal component of vector L is changing
(in direction). There must be a torque in the direc- 20. When COM moves by S, the point of application of
tion of change of L. F moves by 2 S.
16. As seen in Q.7 of this worksheet, when F1 is applied,
\ WF = F (2S)
acceleration will be towards right.
When F3 is applied, it gives 7
KE of rolling sphere is ___
​    ​ Mv2
an anticlockwise torque. There 10

÷ 
_____
is no tendency of translation. 7 20FS
Friction will act towards left \ ​ ___  ​ Mv2 = 2FS  fi  v = ​ _____
​  ​ ​ 

  
10 7M
to oppose the tendency of slip-
page. Thus, the yo–yo will 21. L = Lspin + Lorbital
move towards left.
Both Lspin and Lorbitals are maximum when the sphere
Line of F2 passes through point of contact A. There reaches the lowest position.
is no torque about A. Friction will adjust to a value
equal to horizontal component of F2 and there will mg sin q
Friction on incline plane is f = _______
​   
 ​
be no motion. R2
___
1 + ​  2  ​
17. If a0 is acceleration of the plank, then for no slip- K
ping, tangential acceleration of the contact point must
Friction is maximum where sin q is maximum.
be a0.
a + r a = a0  fi  a < a0 Slope is maximum at the top, hence friction is
maximum at the top.
Friction does perform work on
the sphere. The contact point When the sphere reaches the bottom, its speed is
(A) when the friction acts is given by
not at rest. Work done by fric- 10 mg
7 mv2 _____
tion on the sphere is equal to ​ ___  ​ mv2 = mgR  fi ​ ____   
​ = ​   ​  
10 R 7
change in KE of the sphere.
Friction does negative work on the plank. mv2
For circulation motion COM: N = mg = ​ ____    ​ 
On the system (sphere + plank) as a whole, static R
friction does not perform work. 17
fi N = ___
​   ​  mg
19. LB = Lspin + Lorbital 7
22. Friction is kinetic and it dissipates KE.
1
= ​ __ ​  MR2w + MvR The force applied by the axes on the discs do not
2
have zero torque about the point of contact.
Solutions  S.71

Worksheet 3
1. 1g and 99 g particles are at a distance of 49 cm from
the axis. 2 g and 98 g particles are at a distance of
48 cm from the axis; and so on.

M
dm = __
​   ​  dx.
L
Perpendicular distance of the element from the axis
is
\ I = (1 + 99) × 492 + (2 + 98) × 482 + (3 + 97)
r = x sin q
× 472 + ... + (49 + 51) × 12 + 50 × 02
= 100 [492 + 482 + 472 + ... + 22 + 12] \ (  )
M
dI = ​ __
M
​   ​  dx  ​ (r2) = __
L
​   ​  sin2 q x2dx
L
49 (49 + 1) (2 × 49 + 1)
= 100 × ____________________
L
​     
 ​    M 1
6 \ ​   ​  sin2 q ​Ú ​    ​ x2dx = __
I = __ ​   ​  ML2 sin2 q
L 0 3
Sum of squares of first n natural
4. Area of plate = (2a)2– a2 = 3a2

n (n + 1) (2n + 1)
numbers is S = ​ ______________  ​   Mass per unit area, s = ___
   ]  M
​  2  ​ 
6 3a
= 4.04 × 106 g-cm2 Mass of removed part, m = s . a2 = __
M
​   ​ 
2 3
= 0.404 kgm M
Mass of plate before making of hole = M + __ ​   ​ 
2. Consider a shell of radius x and thickness dx. 3
4M
___
M0 = ​   ​   
Volume of shell, dv = 4p x2 dx 3
For a square plate of mass M0, moment of inertia
Mass of shell, dm = r ◊ dv
about its side is
= r0 ​ 1 + ​    ​  ​ 4p x  dx
x
__
R (  )
2

I = ICM + M0a2 = _______


M0(2a)2
​   + M0a2
 ​ 
( 
12
2
__
3
2 2
__
3
x
__
R
4
)
dIshell = ​   ​  (dm) x = ​   ​  r0 ​ 1 + ​   ​   ​ 4 p x dx
16Ma2
4 4 4M 2 ______
= __ ​   ​  M0a2 = __
​    ​ ◊ ​ ___ ​ 
 a = ​   ​   
8pr0 4
_____ 8pr0 5
_____ 3 3 3 9
= ​   ​   x dx + ​       
​ x dx
3 3R
R
8pr0 R 4 8pr0 R 5
\ I = ​Ú   ​ ​   ​dIshell = _____
​   ​  ​Ú ​    ​x dx + _____
​     ​ ​Ú ​    ​x dx

x = 0 3 0 3R 0
44
= ___
​   ​  pr0R5
45 For square of side a [the hole] and mass m, moment
of inertia about AB will be
Ihole = ICM + md2

ma2 13 13 M
= ____
​   ​ + ma2 = ___
​   ​  ma2 = ___
​   ​  ◊ ​ __ ​  a2
12 12 12 3

13Ma2
= ______
​   ​ 

36
\  MI of the plate with hole about AB is

3. Consider an element as shown. Mass of element, 16 13


= I – Ihole = ​ ___
​   ​ – ___
9 (  17
​   ​   ​ Ma2 = ___
36 )
​   ​  Ma2.
12
S.72  Mechanics II

5. (a) As the drum rotates, it winds the string and X fi T1 – 10 = 0.2 a ...(ii)
moves up.
For B:
a = retardation of X ; a = angular retardation 26 26
of Y. m2g – T2 = m2a2  fi ​ ___ ​  g – T2 = ___
​   ​ (0.15 a)
7 7
Both X and Y retard and eventually stop. fi  260 – 7 T2 = 3.9 a ...(iii)
For X: mg sin q – T = ma ...(i)
Solving (i), (ii) and (iii) gives
1
For Y: T ◊ r = ​ __ ​  Mr2 ◊ a
2 a = 21.4 rad s–2 ; T1 = 14.28 N ; T2 = 25.22 N
1
fi T = ​ __ ​  M (r a) a1 = 0.1 × a = 2.14 ms–2 ;
2
1 a2 = 0.15 × a = 3.21 ms–2.
fi T = ​ __ ​  Ma ...(ii)
2
7. After the right string
Solving (i) and (ii) gives is cut, forces on the
1 rod are-its weight
mg sin q 0.5  ×  10 × __ ​   ​ 
_______ ___________ 2 __ 5 (Mg) and tension
a = ​    ​ = ​      ​  = ​   ​  ms–2
M
__ 2
__ 3 at end A. Both the
m + ​   ​  0.5 + ​   ​ 
2 2 forces are vertical.
5 Hence, COM can only have vertical acceleration.
and T = ​ __ ​  N
3 Let acceleration of COM be a (Ø) and angular accel-
(b) Initial velocity of x is u = wr = 10 × 0.5 = 5 ms–1 eration of the rod be a.
up the plane.
Mg – T = Ma ...(i)
Using v2 = u2 + 2a s
ICM ◊ a = tCM
5
0 = 52 – 2 × __
​   ​  × s
3 ML2 L
​ ____ ​  
 ◊ a = T ◊ ​ __ ​ 
fi s = 7.5 m 12 2
6. Let a be angular acceleration of the pulley in clock- MLa
_____
fi ​   ​  
 = T ...(ii)
wise sense. 6
L
Acceleration of B will be  a2 = aa (Ø) Acceleration of end A in COM frame = ​ __ ​  a (≠).
2
Acceleration of A will be  a1 = ba up the incline. String is inextensible and acceleration of end A in
For pulley: vertical direction shall be zero, in ground frame
L
T2a – T1b = Ia \ aA = 0  fi  a (Ø) + __ ​   ​  a (≠) = 0.
2
0.15 T2 – 0.1 T1 = 0.11 a
La
fi 15 T2 – 10 T1 = 11 a ...(i) fi ​ ___ ​  = a
2
Ma
put in (ii) ​ ___ ​  
 = T
3
3g
put in (i) a = ​ ___ ​ 
4
M Mg
And T = __​   ​  a = ​ ___ ​ 
. 
3 4
8. In equilibrium both the springs apply a force
Mg
F = ___
​   ​ 
 on the rod.
2

For A:
T1 – m1g sin q = m1a1
1
T1 – 2 × 10 × __
​   ​   = 2 (0.1 a)
2
Solutions  S.73

ft
When right spring is cut, the spring force at A cannot After time t: v = v0 – at = v0 – __
​     ​
change suddenly. [String tension can change sud- M
denly but not the spring tension]. 5ft
w = w0 – a t = w0 – _____
​      ​
Mg 2MR
\ Ma = Mg – ___​   ​ 
  ft
2 v becomes zero at time given by v0 – ​ __  ​ = 0.
g M
fi a = ​ __ ​  ft
2 fi  ​ __  ​  = v0 ...(i)
M
Writing t = I a about COM,
(a) At this time w is also zero
ML2 L
​ ____ ​  
 ◊ a = F ◊ ​ __ ​  5ft 5 v0
12 2 fi  w0 – _____
​     ​ = 0  fi  w0 – __
​   ​  ​ __ ​  = 0.
2MR 2 R
ML2 Mg __ L 3g
fi ​ ____ ​  
 a = ___    ​   fi  a = ​ ___ ​ 
​   ​ ◊ ​ 
  v0 2
12 2 2 L fi  ​ ____  ​  = __
​    ​.
w0 5
Acceleration of the end, in COM frame is
(b) For sphere to be rotating w > 0.
L 3g
a ​ __ ​   = ___
​   ​ (≠). 5ft 5 v0
2 2 fi  w0 > _____​    ​   fi  w0 > ​ __  ​ __
​   ​   [using (i)]
g 2MR 2 R
Acceleration of COM is __ ​    ​ (Ø)
2 2 v0
3g g fi ​ __ ​  > ____
​    ​ 
5 w0R
\ aA = ___ ​   ​ (≠) + __​   ​  (Ø) = g (≠).
2 2 12. a0 = acceleration of plank.
9. Slipping ceases when velocity of a point on the a = acceleration of sphere
circumference of the disc becomes v. a = angular acceleration of sphere
18
wR = v  fi  w = _____ ​    ​  = 240 rad s–1
0.075
mMg
Friction on disc f = m N = ____ ​   ​  
2
10
= 0.25 × 4 × ___ ​   ​ = 5N
2
1 1 For no slipping
MI of disc I = __
​   ​  MR2 = __
​    ​ × 4 (0.075)2
2 2 Ra + a = a0 ...(i)
= 0.01125 kg-m2 For sphere: f = ma ...(ii)
2
t f ◊ R 5________ × 0.075 fR = __ ​   ​  mR2 ◊ a
\  Angular acceleration, a = __ ​   ​ = ___
​      ​ = ​   ​  5
I I 0.01125 2
__
fi f = ​   ​  m (Ra) ...(iii)
= 33.33 rad s –2 5
For plank: F – f = Ma0 ...(iv)
Using w2 = w02 + 2 a q
Calculating a, Ra and a0 from (ii), (iii) and (iv) and
(240)2 = 0 + 2 × (33.33) q putting into (i) gives:
fi q = 864 rad. 5f f f
F
864 ​ ___  ​ + __ ​     ​ = __
​    ​ – __
​    ​ 
\  Number of turns = ____ ​   ​ = 137.6 2m m M M
2p
10. Refer to Article 4.2 in the text. The ring will roll
without sliding even if there is no friction.
5
fi f ​ ___ [ 1     ​ + __
​    ​ +  ​​m
2m
1
] __ 2Fm
_______
​ ​ M  ​   ​ = ​ M ​   fi  f = ​ 7M +   m
  ​

11. The contact point has velocity in forward direction.


Hence, friction is towards left. But f £ m mg
f
Retardation: a = __ ​    ​  2Fm 2F
M \ ​ _______     ​ £ m mg  fi ​ _________      ​ £ m
fR 5f 7M + m (7M + m) g
t__ ______
Angular retardation a = ​   ​ = ​      ​ = _____
​    ​ 
I __2 2MR
​   ​  MR2
5
S.74  Mechanics II

2F F
13. Let acceleration of two cylinders be a. aA = ___
​ m ​ + ______
​       ​
M+m
Acceleration of plank A = horizontal acceleration of
top point of the two cylinders = 2a. (b) Angular speed of disc at time t is
2Ft
w = a t = ​ ___
mr ​ 
1 1
It’s rotational KE is: kR = __
​   ​ ​  __
2 2 (  F2t2
​   ​  mr2   ​ w2 = ____
​  m   )
​ 

Linear speed of system at time t:


Ft
v = at = ______
​    
 ​
M+m
Translational KE at t is:
Friction forces at various surfaces are assumed to be
f1 and f2 as shown.

1
2 ( 
​ 
M+m
Ft
kT = ​ __ ​  (M + m) ​​ ______
   )
2
 ​  ​​ ​

1 F2 2
For A: F – 2f1 = M (2a) ...(i) = ​ __ ​  ______
​     ​
2 M+m

[ 
For B: f1 + f2 = ma ...(ii)

and
1
f1r – f2r = ​ __ ​  mr2 ◊ a
2
\ k = kR + kT = F2t2 ​ __
1 1
​ m   ​ + ________
​     
2 (M + m)
 ​  ​ ]
15. (a) The COM moves in a circle of radius (R + r).
1
fi f1 – f2 = ​ __ ​  ma   [ ra = a] ...(iii)
2
3
Adding (i), (ii) and (iii) F = ​ 2M + __ ( 
​   ​  m  ​ a.
2 )
2
fi a = ​ ________
    ​
4M + 3m
4F
\ Acceleration of A is = ________
​      ​ Let v be speed of the COM of the ball at angular
4M + 3m
position q.
Using v2 = u2 + 2as
7
​ ___  ​ mv2 = mgh
÷ 
________
2 8FL
________ 8FL
________ 10
v = 0 + ​      ​ fi v = ​ ​     ​ ​

4M + 3m 4M + 3m 7
​ ___  ​  v2 = g (R + r) [1 – cos q]
14. (a) Torque on disc about its central rotation axis 10
is 10
fi v2 = ​ ___ ​  g (R + r) [1 – cos q] ...(i)
t = F ◊ r 7
t F ◊ r 2F If the ball loses contact at this position, then
\ a = ​ __ ​ = _____
​     ​ = ___
​ mr  ​ N=0
I 1
__
​   ​  mr2
2 mv2
fi mg cos q = ​ _____  
 ​
We have used t = Ia about an axis through R+r
COM and it is perfectly alright even if COM 10 g
is accelerated. fi g cos q = ​ ____
 ​ 
 [1 – cos q ] [using (i)]
7
Linear acceleration of the (disc + mount) system 10
fi 17 cos q = 10  fi  cos q = ___ ​   ​ 
is 17

[ 
F
a = ​ ______


M+m
  
 ​
Acceleration of point A wrt centre of the disc

10
7
17 – 10 ___
(b) From (i) v2 = ​ ___ ​  g (R + r) ​ ​ ______
17
 ​  ] 10
 ​= ​   ​ g (R + r)
17

÷ 
__________
is 10g (R + r)
2F \ v = ​ ​ _________
 ​ ​
    
  
ra = ​ ___
m ​ 
17

÷ 
__________
\  Acceleration of point A relative to the v 1 10g (R + r)
\ w = ​ __r ​ = ​ __r ​  ​ ​ _________
 ​ ​
    
  
ground is 17
Solutions  S.75

16. Height of point O from the surface cannot change.


Mechanical energy is conserved.
3R
17. Distance of COM from O is x = ___
​   ​ 
8
Torque about A is
3MgR
tA = Mg ◊ x = ______
​   ​   
8
Moment of inertia about
the instantaneous axis 8M
Mass of cavity filled sphere, M0 = M + m = ___
​   ​ 

through A is 7
KE of given sphere = (kinetic energy of cavity filled
7
IA = ​ __ ​  MR2 R
5 sphere) – (kinetic energy of sphere of radius ​ __ ​ 
2
[It makes no difference occupying the cavity space)
whether it is a complete For writing KE we will consider pure rotation about
sphere or half sphere instantaneous axis through contact point A.
when we are writing MI
about axis through A. Why?]
tA 15 __ g
\
1 7
2 5 0 ( 
KE = ​ __ ​ ​  __ ) 1 7 __
​    ​ M  R2   ​ w2 – __
​   ​  ​ __
2 5 (  (  ) )R 2
​   ​  m ​​ ​   ​   ​​ ​  ​w2
2
\ a = ​ __  ​ = ___
​   ​ ​   ​  7 8M 2 ___ 7 M
IA 56 R = ___
​    ​  ◊ ​ ___ ​ v
  – ​    ​  ◊ ​ __ ​  v2 [where Rw = v]

10 7 40 7
COM has a vertically downward acceleration given
31
by x a [you can think of COM rotating about O. Or = ___
​   ​ Mv2.
40
else, think of COM rotating about A. Acceleration of
COM will be ^ to line joining A to COM. Component 19. Speed of the rim of the pulley = speed of the
of this acceleration in vertical direction is xa] block.

\ Mg – N = M (xa) fi wR = v

(  ) (  )
3R 15 __ g where w = angular speed of the pulley and
fi  N = Mg – M ​ ___
​   ​  ​ ​ ___
​   ​ ​    ​  ​
8 56 R v = speed of the block.
403
____ When the block falls by a distance x, its PE decreases
= ​   ​ Mg.
308
f 1
Horizontal acceleration is a = ​ __  ​ (this is also accel- by mgx. PE in the spring increases by ​ __  ​ kx2 and the
M 2
eration of point O). block and the pulley both gain kE.
For pure rolling a = Ra. 1 1 1
\ ​ __ ​  mv2 + __
​   ​  Iw2 + __
​   ​  kx2 = mgx
2 2 2
f 15g 15Mg
\ ​ __  ​   = ​ ____ ​   fi  f = _____
M 56
​   ​ 
56
  1
2
1 1
fi ​ __ ​  mv2 + __
​   ​  ​ __
2 2 (  v 2 1 2
​    ​ MR2   ​ ​​ __ ) (  )
​   ​   ​​ ​ + __
R
​   ​  kx = mgx
2
15Mg 403
But f £ m N  fi ​ _____  £ m ​ ____ ​  Mg.
 ​  Put the values of m, M, k and x and simplify to get
56 408
v = 2.4 ms–1.
fi 0. 27 £ m
20. MI of the body about rotation axis is
M 8M
18. Density r = ______________
​      3 ​ = _________​      ​ ml2
I = 0 + ml2 + ​ ___ ​ where
4 3 __
__ 4 __ R
(  )
​   ​  p R – ​   ​  p ​​ ​   ​   ​​ ​
3 3 2 ( 
4
7 ​ __ )
​   ​  p R3  ​

4
3

  m = mass of each rod.

3 = __ ​   ​  ml2.
3
Mass of removed part i.e., mass of a sphere of
Loss in PE when the body becomes vertical is
R
radius __
​   ​   is l 3
2 = mgl + mg ​ __  ​  = __
​   ​  mgl

(  )
2 2
4 R 3 4 R3 ________ 8M M
m = ​ __ ​  p ​​ ​ __ ​   ​​ ​ r = __
​    ​ p ___
​   ​ ​     ​  = __
​   ​ 
(  )
3 2 3 8 7 Centre of one rod falls through l and the centre of
4
7 ​ __ 3
​   ​  pR    ​ l
3 the other rod falls through __ ​    ​ 
2
S.76  Mechanics II

Conservation of energy: Mechanical energy of the sphere is conserved as it


1 3 rolls up.
​ __ ​  Iw2 = ​ __  ​ mgl
2 2 7 7
\ ​ ___  ​ mv 2 = ___
​    ​  mg (R – r) + mg 2 (R – r).

÷ 
__

( 
10 10
1 4 2 2 __
fi ​ __ ​ ​  __ ) 3 3 g
​    ​ ml    ​ w = ​   ​  mgl  fi  w = __
​   ​  ​ __
÷ 
​   ​ ​  . __________
2 3 2 2 l 27 27
fi v 2 = ___
​   ​ g (R – r)  fi  v = ​ ___
​   ​ g (R   
– r) ​
7 7
21. Let the hammer impart the blow
at end A. 25. Energy conservation gives w when the rod is
horizontal.
Impulse = DP
1 L 1 ML2 2 L
​ __ ​  IAw2 = Mg ​ __ ​   fi ​ __ ​  ____   = Mg __
​   ​ w
  ​   ​ 
\ J = mv ...(i) 2 2 2 3 2

÷ 
___
Angular impulse about COM 3g
l fi w = ​ ___
​   ​ ​  
is J ◊ ​ __  ​  L
2
Now we will find angular acceleration of the rod in
l
J ◊ ​ __  ​ =
  DL horizontal position.
2
IA ◊ a = tA
l ___ ml2 6J 6mv ___ 6v
fi J ​ __  ​ =
  ​   ​  w  fi  w = ___
​    ​ = ​ ____   
​ = ​     ​ ML2 L
2 12 ml ml l ​ ____  ​ ◊ a =

  Mg ​ __ ​ 
3 2
l l
\ vA = w ​ __  ​  + v = 4v ; vB = w ​ __  ​  – v = 2v (¨). fi
3 __
__ g
a = ​   ​  ​   ​
2 2 2 L
22. Lspin = Angular momentum about COM (C)
= mvr + mvr = 2mvr.
Lorbital = 0 since COM is not moving.
23. In case (A) the applied
force provides an
anticlockwise torque.
To ensure pure roll-
ing friction will act
towards left so as L
COM of the rod is moving in circle of radius ​ __ ​  with
to provide linear 2
acceleration. angular acceleration a and angular speed w. Radial
acceleration of COM is horizontal.
f = Ma ...(i)
L 3g
R ar = w2 __
​   ​  = ___
​   ​ 
F ​ __ ​  – fR = Mk2 ◊ a 2 2
2
F
fi ​ __ ​  – f =
2
Ma
___
​   ​ 
2 [ 
 ​
R2
k2 = ___ ]
​   ​   and  Ra = a  ​ ...(ii)
2
Horizontal component of hinge force is responsible
3Mg
for this acceleration Fx = Mar = ____
​   ​ 
2

F 3Ma F
Adding (i) and (ii): __ ​   ​  = ​ ____   fi  a = ___
 ​   ​    ​  Tangential acceleration of COM is vertically down-
2 2 3M wards, equal to
Similarly, one can solve for case B. L 3g
at = __
​   ​  ◊ a = ___
​   ​ 
24. COM of the sphere moves in a circle of radius (R – r) 2 4
and it will complete the circle if it has a minimum 3Mg
_______
\ Mg – Fy = ____
​   ​   
speed of v0 = ÷   – r) ​ 
​ g (R at the top. 4
7 [Fy = vertical component of hinge force]
KE of a sphere rolling with speed v0 is ___
​    ​ mv02.
10
Mg
\  Minimum KE of the sphere at the top of the path fi Fy = ___
​   ​ 

4 _______ ___
7
is ___
​    ​  mg (R – r).
10 \ Hinge force is F = ÷
​F  x2 + Fy2  ÷    
​ 37 ​
 ​ = ​ ____
4
 ​ Mg.


Solutions  S.77

26. During the fall, the system gains kE at the expense


of potential energy.

u 1 u__2 1 u2 u2 __
y = ___
​  __  ​  t – __
​   ​  gt2 = ​ ____   ​ – __
​   ​  g ​ __2 ​  = ___     – 1)
​   ​ (​÷2 ​
÷   
​ 2 ​ 2 ÷    g 2 g
​ 2 ​  2g

Figure shows initial and final positions. \  Position vector relative to O is


Loss in PE is ​_› u__2 ˆ ___ u2 __
   ​ ____
​r ​ =     – 1) jˆ
  ​ i + ​   ​ (​÷2 ​

(  ( 
÷    g
​ 2 ​  2g
R R
DU = mg ​ __
​   ​  + __
4 4 )5R 5R
​   ​   ​ + mg ​ ___
​   ​ + ​ ___ ​  ​
4 4 ) ​_› ​_› ​_› ​_›
​L  ​   = ​r ​   × ​P  ​  = m (​r ​   × ​v  ​) 
​_›

[ 
= 3 mgR
  Centre of the disc falls through __
5R 5R 5R
A falls through ___
​   ​ + ​ ___ ​ = ​ ___ ​ 
R
​   ​  and particle at
2
u2 ___
= m ​ ____
​  __    
÷    g ÷
​ 2 ​ 
u
​ ​  __  ​ (1 – ÷
  
​ 2 ​
__
​ 2 ​
u2 __
  )  – ___
2g
u__
    – 1) ​ ___
​   ​  (​÷2 ​   ​  ​k̂
  
​÷2 ​ ]
4 4 2 mu__3
Moment of inertia of the disc-mass system about line = – ​ _____   ​ 

   g
2 ​÷2 ​ 
PQ is

[ 
28. The entire rod is rotating about point O. Note that
I = Idisc + Iparticle
mR2
= ​ ____
​   ​ 
4
R 2
 + m ​​ __
4 (  ) ] (  ) 5R 2
​   ​   ​​ ​  ​ + m ​​ ___
​   ​  ​​ ​
4
any particle in the rod remains at a fixed distance
from O when the rod moves. Let angular speed of
15
= ___
​   ​ mR2 the rod be w when it becomes horizontal.
8
Gain in kE = Loss in PE
1
​ __ ​  Iw2 = 3mgR
2
where w = angular speed in final position

÷ 
____
15 16g OC = OC1 = R cos 45°
fi ​ ___ ​ mR2w2 = 3mgR  fi  w = ​ ____ ​   ​ ​ 

16 5R h1 = OC ◊ sin 45°

(  )
5R ____
Speed of particle at A is v = w ​ ___   ​ 
​   ​  ​ = ​÷5gR  Loss in PE of the rod is
4
Since, the particle is rotating in a circle of DU = Mg (h2 – h1)

5R = Mg [R cos 45° – R cos 45° ◊ sin 45°]
radius ___
​   ​ 

[ 
__
4
27. ux = ___
u
​  __  ​  ; uy = ​ ___
÷   
​ 2 ​
u__
  ​ 
  
​÷2 ​
= MgR ​ ___
÷
1
   2
​ 2 ​
1
​  __  ​ – __
2 ]
   – 1)
MgR (​÷2 ​
​   ​   ​ = ​ ____________
 ​     

u MI of the rod about an axis passing through O


At t = __
​ g ​ ; perpendicular to the plane of the figure is

( 
__
vy = ___
÷
u
  
​ 2 ​
u
​  __  ​ – g . ​ __
g ÷
1
 ​ = – u ​ 1 – ___
  
​ 2 ​ )
​  __  ​  ​
​ 
   R)2
M (​÷2 ​ 
I = ________
12
 ​  + M (OC1)2

​_›
\ ​v  ​   = __
u
​   ​  iˆ – u ​ 1 – ___
2
1
( 
​  __  ​  ​ jˆ
  
​÷2 ​ ) MR2
= ____
​   ​ 
6 ÷
R 2 2
 + M ​​ ___
​  __  ​  ​​ ​ = ​ __ ​  MR2
  
​ 2 ​ 3 (  )
1
Co-ordinates at time t = ​ __
u KE of the rod when it is horizontal is k = ​ __  ​ Iw2
g ​ are 2
1
= __
​   ​  MR2 w2
u u2 3
x = ___
​  __  ​  ◊ t = ____
​  __   ​ 
÷   
​ 2 ​ ÷    g
​ 2 ​  Gain in KE = loss in PE
S.78  Mechanics II

( 
__
1    – 1
​÷2 ​
​ __  ​ MR2 w2 = MgR ​ ​ ______ ) Adding (ii) and (iii)

[ 
 ​    ​

]
3 2 I1 I2r1 I1w1 I2w2

÷ 
__________
w0  ​ __
​ r   ​ + ____ ____ ____
__
3(​     – 1) __
÷2 ​ g ​  2 ​  ​ = ​  r  ​ 
 + ​  r  ​ 

fi w = ​ ​  ________
 ​    
 ​   ​  1 r2 1 2

[ 
2 R
 ​ Velocity of centre: v = (OC1) w = ___

= __
1 __
  ÷2 ​
​    ​ ​÷3(​
÷
R
​  __  ​  ◊ w
  
​ 2 ​
___________
\
I1w1r2 + I2w2r1
w0 = ​ ​ _____________
  
  
I1r22 + I2r12
 ​  ​ r2
]
[ 
    – 1) gR
  

]
2 w0r1 I1w1r2 + I2w2r1
 ​ 29. Top point of roller moves with a speed that is twice And w = ____  = ​ ​ _____________
​  r  ​    
    ​  ​ r1
that of the centre.
2 I1r22 + I2r21
Each roller moves by L when the plank moves by 31. Particle moves from P1 to P2 in time t.
2L.
Let speed of centre of each roller be v. q = wt.
3
KE of each roller = __ ​   ​  Mv2.
4
Speed of plank = 2v.
1 3
KE of plank + rollers = ​ __  ​ M (2v)2 + 2 × ​ __  ​ Mv2
2 4
7
= __
​   ​  Mv2
2
Loss in PE = Loss in PE of plank + (loss in PE of
each roller) × 2
= Mg (2L sin 30°) + [Mg (L sin 30°)] × 2
Co-ordinates of the particle are:
= MgL + MgL = 2MgL.
x = R cos wt
7
From conservation of energy: __ ​   ​  Mv2 = 2MgL y = R sin q = R sin wt
2

÷ 
____
4 z = a.
fi v = ​ __
​   ​  gL  
​ ​_›
7
\ ​r ​ =
   R (cos wt)  + (R sin wt)  + a 
30. Let kinetic friction between the two cylinders be f
​_›
and t be the time in which they stop slipping. Let Velocity: ​v  ​   = – v sin q + v cos q
final angular speed of first cylinder be w0 and that
of the second cylinder be w. = Rw [– sin (wt)  + cos (wt)  ]
(a) Angular momentum about O is
_
​› ​_› ​_› ​_› ​_›
​L  ​  = ​r ​   × (m​v  ​)  = m (​r ​   × ​v  ​) 
= mR2w [(cos wt) ◊ cos wt + sin2 wt]  – mRaw
sin (wt) – mRaw cos (wt) 
= mR2w  – mRaw [cos (wt)  + sin (wt)  ]
When there is no slipping
(b) Lz = mR2w
w0r1 = wr2 ...(i)
Lz can be written by _simple observation._
Using angular-impulse momentum theorem for both ​› ​›
Note:  (i) Magnitude of ​L   ​ is constant (ii) ​L   ​ vec-
cylinders, we can write:
tor has a fixed z component and its component in
– fr1t = I1w0 – I1w1 xy plane rotates with angular velocity w.
I1w0 ____ I1w1 32. (a) Conservation of momentum gives:
fi – ft = ____
​  r  ​ 
 – ​  r  ​  
  ...(ii)
1 1 2v0
3mv = 2mv0  fi  v = ___ ​   ​  
And fr2t = I2w – I2w2 3
L
I2w0r1 ____
I2w2 (b) The COM of the system is at a distance __
​   ​  from
ft = ______
​  2 ​  – ​  r  ​  
  [using (i)] ...(iii) 3
r2 2 end A. We will conserve angular momentum
Solutions  S.79

about a point fixed on the table which is just


beneath the COM of the system (at C).
Li = LB + LA + LP
2L L
= mv0 ​ ___ ​  ( ) + mv0 ​ __ ​  ( ) + 0
3 3
L
= mv0 ​ __ ​  ( )
3

[ m << M, hence we are neglecting contribution


of m in moment of inertia. MI of cube about its edge
2
is __
​   ​  ma2
3
mv
\ w = ____
​    ​   ...(i)
2Ma
After collision, the cube is set into rotation about A
and its mechanical energy remains conserved. For
topping it must be able to reach the position shown
in 3rd figure (In this position COM is at maximum
height. PE is maximum).
After collision, angular momentum about our 1
\ ​ __  ​ IA ◊ w2 = Mg (h2 – h1)
origin is 2

(  ) (  )
1 8 mv 2 __
Lf = Lspin + Lorbital = Iw + 0 fi ​ __ ​ ​  __
​   ​  Ma2   ​ ​​ ____     – 1) a
​  ​​ ​ = Mg (​÷2 ​
​     
2 3 2Ma

[  (  ) (  ) ] __ 1
__
L 2 2L 2 M ​   ​ 
= ​ 2m ​​ __
​   ​   ​​ ​ + m ​​ ___
​   ​   ​​ ​  ​ w fi v = __
​ m ​  ​​[ 3ag (​÷2 ​ 
    –  1) ]​​2 ​
3 3
v0 3
Using Li = Lf gives w = ___
​     ​ ( ) 34. (a) KE of rolling cylinder = ​ __ ​  mv02. Let its speed
2L 4
(c) vA = velocity of COM + velocity wrt COM be v immediately after it is on incline.

wL 2v0 ___ v0 L __ v0
= v – ___
​   ​  = ___
​   ​  – ​     ​ __
​   ​  = ​   ​ 
3 3 2L 3 2
2L
vB = v + w ​ ___ ​ = v0
3
Alternate: A massless rod will not exert any force
on A in the direction of motion (^ to rod), during
collision.
\  Momentum of (A + P) system is separately Gain in KE = loss in PE
conserved. 3 3
​ __ ​  mv2 – __
​   ​  mv02 = mgR (1 – cos q)
v0 4 4
\  mv0 = 2mvA  fi  vA = __
​   ​ 

÷ 
2 _________________
4
fi  v = ​ v02 + __
​   ​  gR (1 –  
cos q) ​ ...(1)
Velocity of B remains unchanged, i.e, vB = v0. 3
33. Angular momentum is conserved about rotation axis (b) The COM of the cylinder is instantaneously
through , during collision. rotating about P with speed v.
mv2
L before collision = L after collision \ ​ ____   
​  = mg cos q – N
R
4a 2 mv2
mv ​ ___ ​   = __
​   ​  M (2a)2 ◊ w fi N = mg cos q – ____​     
​ 
3 3 R
S.80  Mechanics II


mv02 __
= mg cos q – ____
R
4 4
 ​ – ​   ​  mg + __
​     
3
​   ​  mg cos q
3
1
2
1
fi ​ __  ​ mR2 w0 = ​ __
2 [  v
​    ​ mR2 __
​   ​  + mvR  ​ × 2
R ]
[using (i)] [When pure rolling starts, v = wR]
For no jump, N ≥ 0
w0R
7 4 mv02 fi v = ​ ____
 ​ 

fi ​ __ ​  mg cos q – __
​   ​  mg – ____ ​ ≥ 0
​      6
3 3 R v0
37. Before collision, w0 = ​ __ ​ . After collision, velocity of

÷ 
______________ R
7gR
fi v0 £ ​ ____
​   ​   4    ​ gR ​
 cos q  – ​
   the ball is ev0 (¨) and its angular speed is w0 ( ).
3 ​3 ​
35. Speed of COM is given by Mu = J
fi 0.6 u = 6  fi  u = 10 ms–1.
Angular speed about COM is given by
ICM ◊ w = angular impulse about COM
1
​ ___  ​ ML2 ◊ w = J ◊ (0.05 m)
12
1
fi ___
​    ​ × 0.6 × 0.32 ◊ w = 6 × 0.05
12
200
fi w = ____  rad s–1
​   ​ 
3
As the rod is kicked off the edge, its angular speed
(w) becomes constant. There is no torque about Note that w0 do not change during collision.
COM.
Friction converts this rolling with sliding motion into
Velocity of COM increases in vertical direction due pure rolling. when pure rolling starts v = wR.
to Mg.
Conserving angular momentum about point A:
After 1.0 s the vertical component of velocity of
Li = Lf
COM is
v = gt = 10 ms–1. 2 2
m (ev0) R ( ) + __
​    ​ mR2 w0 ( ) = mvR  ( ) + __ ​   ​  mR2w  ( )
______ 5 5
__
\ vCM = ​÷v  2 + u2      ms–1.
 ​ = 10​÷2 ​ 2 2 Rw0 = v0
fi ev0 – __ ​   ​  v0 = v + __ ​   ​  v
5 5  and Rw = v
1 2 1
KE = __
​    ​ MvCM + ​ __ ​  ICM ◊ w2 2 7
2 2 fi 0.7 × 14 – ​ __ ​  × 14 = __ ​   ​  v
5 5
(  )
1 __ 1 1 200 2
= __   )  2 + __
​   ​  0.6 × (10​÷2 ​ ​   ​  × ___
​    ​ × 0.6 × 0.32 × ​​ ____
​   ​  
 ​​ ​ fi 1.4 – 0.8 = 0.2 v
2 2 12 3
fi 3 ms–1 = v
= 70 J ____
38. Just before impact, velocity of COM is v0 = ÷   ​ 
​ 2gh 
36. Consider a point A on the surface that is to the left
of both discs. Angular momentum of the system is Let v be the velocity of COM and w be angular
conserved about this point. [Refer to example 45] velocity of the rod after impact.
J = impulse applied by the floor.
J = Pf – Pi
J = mv – (– mv0) = m (v + v0) ...(i)
Angular impulse about COM = Change in angular
momentum about COM
Li = Lf
L mL2
fi J __ ____
​   ​ w ;  using (i)
[ 
​   ​  cos q =
1
​ __ ​  mR2 w0 =
2
1
​ __ ]
​   ​  mR2 w + mvR  ​ × 2
2
2
Lw
12

fi (v + v0) cos q = ___
​   ​    ...(ii)
6
Solutions  S.81

block has no acceleration and the RF attached


to it is inertial.

m (v1 + v2)2
\ N – mg = ​ __________
  
   ​
R–r
where N = normal force by block on cylinder.

Since, collision is elastic, 0.5 × (2 + 1.5)2


N = 0.5 × 10 + ​  _____________
 ​
    = 16.67 N.
Relative speed of approach of point A wrt floor = 0.525
relative speed of separation of A wrt the floor.
(c) Considering vertical equilibrium of the block.
wL
fi v0 = ___
​   ​  cos q + v ...(iii) N1 = Mg + N [N1 = normal reaction of floor]
2
Solving (ii) and (iii) gives = 1 × 10 + 16.67 = 26.67 N.
____
6v0 cos q    
6​÷2gh 
​ cos q 40. MI of the combination of rods about rotation axis
w = ____________
​          ​ = ​ ____________
       ​
L (1 + 3 cos2q) L (1 + 3 cos2 q) through P is


ML2 _____
I = ____
​   ​ 
3
2ML2
 + ​   ​ 
12
3L 2
 + 2M ​​ ___ (  )
​   ​   ​​ ​ = 5ML2
2
(a) Conservation of angular momentum about P
gives

Mu (2L) = [5ML2 + M (2L)2] w


Note:  Alternatively one can get equation (ii) where w = angular speed just after impact.
by applying conservation of angular momentum
2u
about A. fi w = ___
​   ​ 
L
39. (a) Let velocity of the centre of the cylinder and the
Now, the KE of the system just sufficient to raise it
block be v1 (Æ) and v2 (¨) when the cylinder
to horizontal position.
reaches the bottom. w is angular velocity of
cylinder. Loss in KE = gain in PE

(  ) (  )
For no slipping: wr – v1 = v2 ...(i) 1 L 3L
​ __ ​  [5ML2 + M (2L)2] w2 = Mg ​ __
​   ​   ​ + 2Mg ◊ ​ ___
​   ​   ​
2 2 2
+ Mg ◊ (2L)

9 4u2 11
fi ​ __ ​  ML2 ___
​  2 ​  = ___
​   ​ MgL
2 L 4
  

÷ 
_____
11
Momentum conservation along horizontal fi u = ​ ___
​   ​  gL ​ 
72
direction: mv1 = mv2 ...(ii)
(b) Let velocity of the object be v after impact.
Energy conservation:
Angular momentum conservation
1
2
1 1
mg (R – r) = ​ __ ​  mv12 + __
​   ​  ​ __
2 2 (  )
​   ​  mr2  ​ w 2 about P gives:
1 Mu (2L) = Mv (2L) + Iw
+ ​ __ ​  Mv22 ...(iii)
2 fi 2MuL = 2MvL + 5ML2w
Solving the above equation gives
fi 2u = 2v + 5wL ...(i)
v1 = 2 ms–1.  and  v2 = 1.5 ms–1.
Relative speed of separation of end
(b) In RF attached to the block, the COM of the Q and the object after collision
cylinder is moving in a circle of radius (R – r) = e (relative speed of approach of the
at a speed of v1 + v2. At this point in time, the object and end Q before collision)
S.82  Mechanics II

fi w (2L) – v = eu ...(ii) 1 L
fi ​ __ ​  Iw2 = Mg __
2
3L
​   ​  + 2Mg ​ ___
2 (  )
​   ​   ​
2
Eliminating v between (i) and (ii) gives:


2u (1 + e)
w = ​ ________
    ​   ...(iii)
1
2 [ 
2u (1 + e) 2 __
fi ​ __ ​  5ML2 ​​ ​ ________
9L
    ​   ] 7
​​ ​ = ​   ​  MgL
2

÷ 
9L ______
1 567 gL
After collision: fi u = ______
​       ​ ​ ______
​   ​ ​ 


(1 + e) 20
Loss in KE = gain in PE
Chapter 7 Miscellaneous Problems on Chapters 4–6
​_›
It is given that ​v  ​1  = +
  match the column
\ v = 1  and  wR = 1
1. (a) MI about axis through C and perpendicular to Also, v2 = (v + wR)  = 2 
the plane of the frame is
Thus the cylinder is in pure rolling motion.
​_› ​_›
(b) ​v  1 ​ = + and ​v  ​3  = –  , implies that the point
1 and 3 are getting closer (one is moving to
right and the other is moving to left). Distance
between two points cannot change in a rigid
body.
(c) and (d) can be understood on similar lines.
​_›

[ 
4. (a) ​r ​   = 3t  + 4 
ml2
Iz = ​ ___
12
l 2
​   ​ + m ​​ __ (  ) ]
​    ​   ​​ ​  ​ × 4
2 ​v  ​  = ___
​_›
d ​r ​  
​   ​ = 3 
_
​›

dt
4
= ​ __ ​  ml2 Distance (r) from origin is increasing at a
3
constant rate of 3 ms–1.
Now,  Ix + Iy = Iz _
​›
​_› d ​v   ​
(b) Acceleration, ​a  ​  = ___
​   ​ = 0
4 2 dt ​_› _
fi 2Ix = __​    ​ ml2  fi  Ix = __ ​   ​  ml2 ​_› ​_› ​r ​   . ​v   ​
​›
3 3 (c) Angle between ​r ​   and ​v  ​  given by cos q = ____
​  r v ​ 
\ IAB = Ix + (4m) (PC)2 = __
2
​    ​ ml2 + 4m ◊ ​​ ___
3 (  )
l 2
​  __   ​  ​​ ​
  
​÷2 ​ \  cos q = ___________
​  _______
(​  ÷​ 9t
9t

  + 16 ​  
2
3t
    ​ = ________
​  _______
)​3 ÷​ 9t
    ​
  + 16 ​ 
2
8 2
__
= ​   ​  ml ​_›
3 Component of velocity perpendicular to ​r ​   is
1 2 __ 1 5
(b) I = 0 + ​   ​  ml + ​   ​  ml2 + ml2 = __
__ ​   ​  ml2 ________
12
3 3 3 ​ = ________
v^ = v sin q = 3 ​÷1  – cos2 q   ​  _______
   
 ​
(c) As found in (A), Ix = __
2
​    ​ ml2   + 16 ​ 
÷​ 9t 2

(  )
v^ 12
(d) I = Iz + (4m) ​​ __
l 2 7
​    ​   ​​ ​ = __
​   ​  ml2 \ Angular velocity is, w = __
​  r ​  = _______
​  2      ​
2 3 9t   +  16
2. Total acceleration is always directed towards the cen- \  w decreases with time.
v2 _
​› ​_› ​_›
tre of the disc and has a magnitude a = __​   ​ . (d) ​L  ​  = m (​r ​   × ​v  ​)  = – (12m) 
R
5. First, write the direction of motion of the contact
point. Friction will be directed opposite to this.
7. (a) F – f = Ma ...(i)
And fR – F ◊ r = I ◊ a

Fr I
f – ___
​   ​   = ___
​  2   ​  ◊ (R a)
R R
Path of point P (in ground frame) is a cycloid. At
Fr ___
___ I
point A, its normal acceleration is directed vertically f – ​   ​   = ​  2   ​  ◊ a ...(ii)
R R
down, i.e, towards the centre of the disc.
At point B, when it is touching the ground, its
tangential acceleration is vertically up. Note that at
this point, normal acceleration must be zero as speed
of the point is zero.
​_›
3. (a) ​v  1 ​ = v  + wR
Where v is velocity of centre and wR is velocity
of point 1 relative to the centre.
S.84  Mechanics II

(i) + (ii) gives: F ​ 1 – __ ( 


r
) (  I
​    ​  ​ = ​ M + ___
R )
​  2   ​   ​ a.
R   Passage-based Problems
r
Since ​ __  ​ < 1, a is positive. yo-yo moves to the
R Passage 1
right. Putting the value of a in (i) gives friction
1.
f. It is also positive, which implies that friction
is correctly shown in the above diagram.

For rotational equilibrium: F ◊ r = f ◊ r  fi  F = f.


...(i)
– r
 cosq = ___
​   ​  For translational equilibrium along the incline:
R
f + F sin q = mg sin q ...(ii)
When line of F passes through the contact point
(A), it will product no torque about A. Friction
also does not have torque about A. Thus, there
fi [  ] 3
F ​ 1 + __
3
​   ​   ​ = mg × __
5
3
​   ​   fi  F = ​ __  ​ mg.
5 8
is no rotation. For no sliding, there should not
2. The above equation (i) holds true for any direction
be any translation as well. Hence, f is to left
of applied force F.
and is equal to F cos q.
When angle q is greater than that shown in the
above figure, it means line of F passes from right
of A. This produces an anti-clockwise torque
about A. The body has anti-clockwise angular
acceleration. It must have leftward acceleration
for pure rolling. Hence, friction is to the left.
When q is less than that shown in the above
figure, line of F passes from left of A. This pro-
duces a clockwise torque. Body has clockwise For translational equilibrium:
angular acceleration. It must move to the right
for pure rolling. F// + f = mg sin q

8. Mg – T = Ma ...(i) Where F// = F cos a is component of F parallel to the


incline.
I
and T ◊ r = I ◊ a fi T = ​ ___2   ​  ◊ (Ra) ...(ii)
R \  F cos a + F = mg sin q
I
From (i) and (ii), Mg = ​ M + ___
( 
​  2   ​   ​ a
R ) [   R a = a]
Obviously, F is minimum when cos a = 1
(fi  a = 0°)
MR2 3mg
mg sin q ____
For disc, I = ____  and for ring, I = MR2
​   ​  \ Fmin = _______
​   ​  = ​   ​ 
2 2 10
\ aring < adisc, irrespective of the radius. 3. In first question:  N + F cos q = mg cos q
If both the objects are disc (or ring) then aA = aB 4 3 4 1
fi N = __
​   ​  mg – __ ​    ​ mg ◊ ​ __  ​ = __
​   ​  mg
Higher the acceleration, higher will be the distance 5 8 5 2
through which the object falls. It means higher accel- 3
And f = F = __ ​   ​  mg
eration implies higher length of thread unwound. 8
3 1 3
Since f £ mN  fi ​ __  ​ mg £ m. __ ​    ​ mg  fi __
​   ​  £ m
8 2 4
Similarly, find for second question.
Solutions  S.85

Passage 2 3
fi ​ __ ​  tan q £ m for the man to not slip.
2
4. For slowly rolling the sphere (i.e., with no accelera-
tion), we have \  For both to move without slipping
F + f = mg sin q ...(i) 3
m ≥ ​ __  ​ tan q
2
And Fh = fR
Fh Passage 3
fi f = ___
​   ​   ...(ii)
R 7. In the co-ordinate system shown, the co-ordinates of
Substituting in (i) gives the COM of the card board are
Fh mg sin q
F + ​ ___ ​ = mg sin q  fi  F = _______
​   
 ​

R h
1 + __
​   ​ 
R
F is minimum when h is maximum, i.e., equal
to R.

a
ycm = ​ __ ​ 
2
__a a
​    ​ × m + __
​    ​ × m + 0 × m
2 ___________________
2 a
xcm = ​     
    ​ = ​ __  ​
5. Both have the same mass. For same acceleration, 3m 3
friction on both will be the same.
The board is on verge of toppling if the vertical line
6. For slow motion of the man: through the COM passes through O.
x 2
tan a = __
​ y ​ = __
​   ​ 
3
8. For no sliding, m ≥ tan a
2
For no sliding before toppling m ≥ ​ __ ​ 
3
9. Acceleration = g sin a
1
Using S = ut + __ ​    ​ at2
2

÷ 
______
1 2a
mg sin q + F = f1 ...(i) a = ​   ​  g sin a ◊ t   fi  t = ​ ______
__ 2
​     ​ 

2 g sin a
For sphere:
Passage 4
F + f2 = mg sin q ...(ii)
Torque needed to lift the block is indepen-
And FR = f2R dent of choice of wrench. It is Mg R to just
fi F = f2 ...(iii) lift the block. To accelerate the block, torque
Solving the above equation gives should be higher.
Torque produced by wrench on the nut
mg sin q
F = _______
​   ​    is t = force × arm length.
2
For a fixed t, force is inversely propor-
mg sin q mg sin q tional to length.
\ f2 = _______
​   ​     fi  m mg cos q ≥ _______
​   ​   
2 2
1 Passage 5
fi m ≥ __
​   ​  tan q
2 When the cylinder reaches the bottom of the hill, its velocity
3
And from (i), f1 = __ ​   ​  mg sin q is v (it has no spin as the hill is smooth) such that
2
1 ____
3
__ ​ __ ​  Mv2 = Mgh  fi  v = ÷   ​ 
\ ​    ​ mg sin q £ m mg cos q ​ 2gh 
2 2
S.86  Mechanics II

Friction slows dows the cylinder and imparts it a spin. It


also accelerates the plank. Finally, pure rolling starts.
vC – wr = vP ...(i)
\ vC > wr   [Ans to 13]

v0
\ v = v0 + wr = v0 + __
​   ​  ◊ r
R
[   For pure rolling v0 = wR]

Conservation of linear momentum gives fi ( 


R+r
v = v0 ​ ​ _____
R
   ​  ​ )
Mv = MvC + MvP  fi  v = vC + vP ...(ii)

From (i) and (ii), 2vC = v + wr ....(iii)


q
17. tan ​ __ (  ) R
​   ​   ​ = __
2
​ x ​ 

  ​. 
Adding (i) and (ii) gives 2vP + wr = v = ​÷2gh 
____
\ x = R cot ​ __
q
​   ​   ​
2 (  )
[The above relations give answer to Q.15] dx
fi ​ ___ ​   = – R cosec2 ​ __
dt (  ) [  ]
q 1 ___
​   ​   ​ ​ __
2 2 dt
dq
​   ​  ​   ​   ​

​     ​ sin  ​( __
​   ​  )​
dq 2v q
fi – ​ ___ ​   = ___
0 2
dt R 2
dq
– ​ ___ ​   = angular speed of the bar.
dt
When the cylinder is sliding, friction on it is leftwards. dq
​ ___ ​ is negative, as q is decreasing.
f dt
Retardation,  a = __
( 
​    ​ 
M
fr
\
2v
w = ​ _____
​    
R+r
q
2 )
 ​  ​ sin2 ​ __ ​ 
Angular acceleration,  a = ​ _____    ​ 
1
​ __ ​  Mr2
2
2f
fi a = ​ ___  ​ 
Mr
ft
Speed after time t is, vC = v – at = v – __ ​    ​ 
M
2ft
Angular speed at time t is, w = a t = ___
​     

Mr
For pure rolling condition, (iii) must hold.
Passage 7
\ (  M
ft
2 ​ v – __ ) 2ft
​    ​   ​ = v + ___
​   ​ 
M
18. If acceleartion of centre of a roller is a0, then
acceleration of the rod will be aR = 2a0.
4ft ft v 19. One can find f1 and f2 by solving the following set
fi ___
v= ​   ​   fi ​ __   ​ = __
​    ​
M M 4 of equations:
ft 3 3 ____
​     ​ = v –  ​v    ​ =
\ vC = v – __ __
​   ​  v = __   ​ [Ans to 14]
​   ​  ​÷2gh 
M ​4 ​ 4 4

Passage 6
16. Let speed of centre be v0 and angular velocity of the
yo-yo be w.
Horizontal velocity of the string is same as velocity
of point P.
Solutions  S.87


m
mg sin q – 2f2 = m (2a0)
m
...(i) 1
= __
1
​   ​  m (2v20) – __
2
1 2 2 ___
​   ​  mv02 – __
2
​   ​  ​ __
2 5 (  ) (  )
5v0 2
​   ​  mr    ​ ​​ ​  r   ​   ​​ ​
​ __ ​  g sin q + f2 – f1 = __
​   ​  a0 ...(ii) 9
2 2 = – ​ __ ​  mv02
2

1 m 2 
f1r + f2r = ​ __ ​  ​ __
2 2 (  )
​   ​   ​ r ◊ a Passage 9
m 24 to 26.
fi f1 + f2 = __
​   ​  a0 ...(iii)
4 After touching the cylinder, the bullet moves down
20. For no slipping, both must slide with an acceleration by 0.2 m while it travels 48 m horizontally
of g sin q. The cylinders do not spin. 1
Time of fall Æ 0.2 = ​ __ ​  gt2
2
Passage 8
t = 0.2 s
21. Vertical component of velocity after impact is For horizontal motion
vy = v0  [ collision is elastic] vt = 48  fi  v = 240 ms–1 [Ans to (24)]
Angular momentum of bullet before hitting
L = mvR = 0.05 × 800 × 0.25 = 10 kg ms–2

[  ]
D t
Impulse of normal force is JN = 2m v0 ​ = Ú​  ​   ​   ​N dt  ​
0
Impulse of friction is

[  ]
Dt
JF = m JN ​ = Ú​  ​  ​   ​mN dt  ​
0 Angular momentum conservation
mJN mvR + Iw = 10
\ mvx = Jf  fi  vx = ​ ____
m   ​ = 2mv0
2p
fi 0.05 × 240 × 0.25 + I × ___
​   ​ = 10
Range of the ball after impact is 4
2vxvy 2mv20 fi I = 4.46 kg-m2  [Ans to 25]
R = ​ _____​ = ____
g    ​  g   
​ 
Moment of inertia of the cylinder with content can
2v02 be written as
If R = ___ ​   then  m = 1.
​  g   
_______ _________ I = (MI of solid wooden cylinder of radius R)
22. Impulse J = ​÷J  N2 + J f2 ​ 
= ​÷J  N2  +  m2JN2 ​  – (MI of solid wooden cylinder of radius r)
__ __ + (MI of ice)
  J  N = 2​÷2 ​
= ​÷2 ​     mv0
1 1 1
\  4.46 = ​ __ ​  msolid R2 – __
​   ​  mhole r2 + ​ __ ​  mice r2
23. Angular impulse about centre is = Jf ◊ r 2 2 2

\ Iw – Iw0 = – mJN ◊ r 1
4.46 = ​ __ ​  × 600 × p × (0.25)2 × 1 × (0.25)2
2
2
​ __ ​  mr2(w – w0) = – 2mv0r [ m = 1; = 2mv0] 1
5 – ​ __ ​  × 600 × p × (0.25)2 × 0.98 × (0.23)2
2
5v0 5v0 5v0
fi  w = w0 – ___
​  r   ​ = ___
​  r   ​ – ___
​  r   ​ = 0 + __
1
​   ​  mice × (0.23)2
2
1
Change in KE = __
1
​   ​  m (vx2 + vy2) – ​ __
2
1
[ 
​   ​  mv20 + ​ __ ​  Iw02   ​
2 2 ] Solving, mice = 127 kg [Ans. to 26]
S.88  Mechanics II

Passage 10
27and28.  For a board to be in equilibrium, weight on left
and right ends are related as:

(  )
2l
WL ​ __ (  )
l
​   ​   ​ = WR ​ __
3
​    ​   ​  fi  2WL = WR
3
\  Joker A must apply 60 kg force on the first board
(i.e, NLA = 60 kg).
Since weight of joker is 80 kg
2m (0) + m (l) __l
\ NRA = 20 kg CO = ​ ___________
      
​ = ​    ​ 
3m 3
For balance of second board Torque on the structure (about C)
NLB = 2 × 20 = 40 kg l
t = 3mg __
​    ​  sin q = mg l sin q
\ NRB = 40 kg 3
Using t = I a
For balance of third board, NLC = 2NRB = 80 kg
Thus, joker C will have to put his entire weight on mg l sin q = (3ml2) ◊ a
the third board. 1 g g 3 g
fi a = __
​   ​  __
​   ​  sin q = __
​    ​  × __
​    ​ = __
​    ​ 
3 l 3l 5 5l
Passage 11 Immediately after release, speed of each particle
29and30.  For no slipping, rate of increase of speed of = 0.
a point on the circumference of the drum must be Each particle has tangential acceleration only.
equal to the rate of increase of speed of a point on
the circumference of solid cylinder.

Acceleration of A is ^ to CA and acceleration of D


is ^ to CD.
g
\  Ra = r a0 [a0 = angular acceleration aD = a ◊ l = ​ __ ​ 
5
Ra of solid cylinder]
fi  ao = ___
​  r   ​   Radial force by the rod
4
Forces on the solid cylinder are as shown. Fr = mg cos q = __
​   ​  mg
5
For no translation: f = mg sin q Tangential force (^ to the rod) is given by
For rotation about its COM: mg sin q – Ft = maD
t = I a0
3 mg 2mg
Ft = __
​   ​  mg – ___
​   ​ = ____

1 Ra
f ◊ r = ​ __ ​  mr2 ​ ___
2
(  )
​  r   ​   ​

5 5
​   ​ 
5

_______
2mg ______
1
fi  mg sin q ◊ r = __
Ra
​   ​  mr2 ___
2g sin q
​  r   ​   fi  a = ​ _______
r    ​   r2 + F2t 
\  F = ​÷F  ​ = ____  ÷
​   ​ 
5
2
 ​ = ___
​ 2  2 + 12  ​  __  ​ mg
2   
​÷5 ​

Passage 12 Passage 13
31&32.  COM of the structure is at O. 33. Angular momentum of the ball about B (a point fixed
to the table) will remain conserved, as it moves on
If AC = CB = CD = l, then the table.
Solutions  S.89

Lf = 0, since the ball stops.

\ Li = 0
This will be true if line of impact passes through B.
The impulse given to the ball has no angular impulse
about B. Thus q = 45°.
34. Let the ball acquire a velocity u due to impact.
Let angular retardation of A = a. Then, retardation
Friction is kinetic, f = mN = mMg of B = 2a.
Retardation, a = mg Slipping ceases when angular speed of A (and C)
Using v2 = u2 – 2as becomes w ( ) and that of B becomes w ( )
O = u2 – 2 mg ◊ s For A: w = w0 – a t
u2 For B: – w = w0 – 2a t
s = ____
​     

2mg 2w0
Solving: a t = ​ ____
 ​ 

But the initial speed is given by Mu = J 3
J w0
fi u = ​ __  ​  \ w = w0 – a t = ___
​   ​ 
M 3
J2 Passage 15
\ s = ​ ______     

2M2mg
38. Position when dust particle (A) reaches origin (O) is
Passage 14 as shown.
35. To keep the axles stationary, forces must be applied
on them. Those external forces can produce torque
about a selected point.
Hence answer is (D).
36. Friction forces acting on each cylinder are as shown.
Each cylinder experiences the same torque (t = 2fR)
about its axle. Thus, each of them has the same
angular retardation.

3
39. Kinetic energy of rolling cylinder is __
​   ​  mv2.
4
Therefore,
3 3
​ __ ​  mv2 + mgR = __ ​   ​  mv02
4 4

÷ 
________
4gR
fi v = ​ v02 – ____
​   ​ ​ 
   
3
Passage 16

Slipping will cease when each comes to rest. 40. L = ​L​ chair ​ + Lwheel
​        ​
​student ​
37. In this case, torque on B is twice that of torque on = 0 + Lspin + Lorbital
A or C.
= 0 + I0w0 + 0 = I0w0 along positive z-direction.
S.90  Mechanics II

41. After the wheel is turned upside down, its spin


angular momentum reverses direction. To conserve
angular momentum, the system of student and chair
begins to rotate with angular velocity w, such that
​_› ​_›
​​L  ​​  chair ​ + ​L  ​ wheel = (I0w0) 
​        ​
​student ​
​_› ​_›
When body A is at the top, centre C is not having
fi  I1w  + [​L  ​spin
  + ​L  ​orbital
  ] = (I0w0)  any acceleration. Relative to C, A is moving in circle
at speed v.
fi  I1w  + [– (I0w0)  + (I0 + ma2) w  ] = (I0w0) 
If N1 = Normal force between A and C
fi  (I1 + I0 + ma2) w = 2I0w0 Mv2
N1 + Mg = ____
​     
 ​ ...(ii)
2I0w0 2I0 w0 6w0 R
fi  w = __________
​   2 ​ = ​ __________
      ​ = ____
   ​   ​ 
I1 + I0 + ma I0 16
4I0 + I0 + __
​   ​ 
3w0 3
= ​ ____
 ​ 

8

Passage 17
42&43. Let v = velocity of centre when the body is at highest
point. Forces on hoop are as shown.
2v = Velocity of body at this instant. Apply For N2 > 0
conservation of energy:
Mg > N1
[Energy of rolling hoop = Mv2 and speed of A when Mv2
it is at lowest point is 0] fi  Mg > ____
​     
​ – Mg [Using (ii)]
R
1 Mv2
Mv02 = Mv2 + __
​   ​  M (2v)2 + Mg (2R). fi  2Mg > ____​     ​   fi  2gR > v2
2 R
fi  3v2 = v20 – 2gR v02 2gR
fi  __
2gR > ​   ​  – ____
​   ​ 

÷ 
_______
2 3 3
0 – 2gR
v_______ ____
fi  v = ​ ​   ​ ​  

  ...(i)
3    
fi ​÷8gR 
​ > v0
Chapter 8 Fluid Mechanics

\ Force on base is
Your Turn
F = PS = 3 × 104 × (0.5 × 0.5) = 7,500 N
1. Hg is 13.6 times heavier than water. (b) Pressure at the interface of the two liquids is
\  Mass of Hg in the beaker = 0.5 × 13.6 = 6.8 kg h
P1 = r g ​ __ ​  = 104 Nm–2
2. Consider a volume V (in cm3) of the alloy, which has 2
mass m (in g).
Mass of Al in alloy is m1 = 0.1 m
M1 (0.1 m)  g
Volume of Al in alloy is V1 = ___
​   ​ = ​ __________
     ​
r1 (2.7) g/cm3
m
= ___
​    ​ cm3
27
Mass of Cu in alloy is m2 = 0.9 m
m2 0.9 m m
Volume of Cu in alloy is V2 = ___​   ​ = ​ _____
 ​ 
 = ___
​    ​ 
3 r2 9 10
cm
Pressure decrease linearly from 3 × 104 to 1 × 104 Nm–2
Volume of alloy, V = V1 + V2 when one travels a height 0.5 m through the lower liq-
uid. Then it decrease linearly from 1 × 104 Nm–2 to
m m
fi V = ___
​    ​ + ___​    ​  h
zero when one moves from y = ​ __ ​  to y = h in upper
27 10 2
liquid.
m 1 270
fi r = __
​   ​  = ________
​     ​  = ____
​   ​  6. Average pressure on upper half of the wall is (see
V ___ 1 1
___ 37
​    ​  + ​    ​  solution of last Q)
27 10
= 7.3 gcm–3 0 + P1 0_______+ 104
Pav = ​ ______
 ​  = ​   ​  = 5,000 Pa
2 2
4. Pressure at depth h is P = P0 + rgh where P0 is
atmospheric pressure. Average pressure on lower half of the wall is
The eardrum experiences force due to water that is
1 ×  104 + 3 × 104
inward. The air inside the ear exerts a force on the P¢av = ​  _______________
 ​
     = 2 × 104 Pa
eardrum in outward direction. Net force on eardrum 2
h
is difference of the two forces. Area of half wall is, S = ​ __ ​  a = 0.5 × 0.5 = 0.25 m2.
Pressure at 10 m depth is 2
\  Required force is
P = P0 + rgh = P0 + 103 × 10 × 10
= P0 + 105 Nm–2 F = Pav ◊ S + P¢av ◊ S = (5,000 + 20,000) × 0.25
Inward force due to water = 6250 N
F1 = P D S = (P0 + 105) D S 7. Horizontal force due to water is
Outward force due to air in the middle air (behind F = (Pav) (area of wall in contact with water)
the eardrum) is

\
F2 = P0 D S
Fnet = F1 – F2 = (105) (D S)
( 
0 + r gh
= ​ ​ _______
2
 ​   ) 103 × 10 × 102 × 30
 ​ (hL) = ​  ________________
2
 ​
    

= 105 × (1 × 10–4) = 10 N = 1.5 × 107 N


5. (a) Pressure at bottom is Weight of the wall, W = volume × density × g
h h 1
P = r g __ ​    ​ + 2r ◊ g ​ __ ​  fi W = __
​   ​  H (a + 2 a) ◊ L × 3000 × 10
2 2 2
3 3 1
= __ ​   ​  r gh = __ ​   ​  × 2000 × 10 × 1 = __ ​   ​  × 12 (3 a) × 30 × 3000 × 10
2 2 2
= 3 × 104 Nm–2 = 162 a × 105 N
S.92  Mechanics II

As per the question: W = 10 F


fi  162 a × 105 = 15 × 107  fi  a = 9.26 m

8. (a) P = r gh = 103 × 10 × 0.2 = 2,000 Nm–2


Force on base due to water is
F1 = P ◊ S = 2,000 × (20 × 10–4) = 4 N
(b) Consider equilibrium of water in the pot.
Base surface exerts a force of F1 = 4N (≠) on
water.
Weight of water is W = (0.5 l) (1 kgl–1) × g = Guage pressure at 1 is P1 – P0 = roil g (60 cm)
5 N (Ø)
The curved wall must exert an upward force of 1N
on the water body for its equilibrium.
2
= ​ __ (  )
​   ​  × 103  kgm–3  ​ (10 ms–2) (0.6 cm)
3

10. g is smaller at equator; therefore, h will be higher at = 4,000 Nm–2


equator. 14. Let us write pressure at the bottom of the tube. It can
11. Inside a freely falling lift, geff = 0 be written starting from point 1 and it can also be
written starting from point 2. The two values should
P0 be same.
From equation (7): h = _______
​     ​ =•
rHg ◊ geff
Hg will rise as much as the tube permits.
12. Let us express pressure in unit of height of Hg–column.
Pressure of air in the tube is P1.

P0 + rHg ◊ g ◊ h = P + PHg g (20 cm)

\ P0 + rHg ◊ g (50 sin 30° cm) = P + rHg g (20 cm)

fi P = P0 + rHg g (25 – 20 cm)


5
= 1 × 105 + 13600 × 10 × ​ ____(  )
​     ​  m  ​= 1.068 × 105 Nm–2
100

15. PA = (P0 + 10 cm) of Hg column


PB = P0 – 10 cm of water column
P2 = P3 10
= P0 – ____
​    ​ cm of Hg column
P1 + 72 cm = 76 cm 13.6
10
fi P1 = 4 cm height of Hg \  PA – PB = 10 + ____
​    ​ cm of Hg
13.6
= (0.04 m) × (13600 kgm–3) × (9.81 ms–2)
= 10.74 cm of Hg
= 5337 Nm–2
13. P1 = P2 16. Pressure below left piston = pressure below right
piston
P0 + roil g (60 cm) = r0 + rw g (40 cm)
0.5 g + F1 2g + F2
P0 + ​ ________
 ​ = P0 + ​ _______
   ​ 

roil 2 A1 A2
fi  ​ ___ ​ =
  ​ __ ​ 
rw 3
5 + 10 20 + F2
Pressure at 1 is fi  ​ ______  = ​ _______
 ​    ​  
2 10
P1 = P0 + roil g (60 cm)
fi F2 = 55 N
Solutions  S.93

17. Force on larger piston needed to just move the load 3 4


23. ​ __ ​  VrL g = Vr1 g  fi  rL = __
​    ​ r1
is 4 3
F2 = Mg + f = 100 × 10 + 20 = 1020 N 24. ma = FB – mg
Pressure in the liquid must increase by fi  Vd a = Vrw g – Vd ◊ g
where d = density of wood.
F2 1020 N
D P = ​ ___ ​ = _______
​    ​  d g
A2 100 cm2 fi  d (g + a) = rw g  fi ​ ___   ​ = _____
​      ​
rw g + a
\  Force on smaller piston is given by 10 5
fi  r ◊ d = ______
​   ​ = ​ __ ​  
  
F1 – f 10 +  2 6
​ _____   D P
 ​ =

A1 25. When the log is completely submerged, buoyancy on


( 
1020 N
F1 – 20 = ​ _______
​   
)
​ (10 cm2) = 102 N
 ​  
m
it is FB = Vrg = ​ __
d(  )
​   ​   ​ r g

( 
100 cm2

fi F1 = 122 N
​ 
120 kg
fi  FB = ​ _________
600 kgm–3
  ​ ​ 103
 ​  
) (  )m
kg
___
​  3  ​  ​ ◊ g = 200 ◊ g

18. (a) PB – PA = r gy = 103 × 10 × 4 = 4 × 104 Pa


According to the question
(b) PD – PA = r ax = 103 × 4 × 2 = 8 × 103 Pa
Weight of log + Additional weight = FB
(c) PC – PA = r gy + r ax = 4 × 104 + 8 × 103
fi  120 ◊ g + mg = 200 g
4
= 4.8 × 10 Pa
fi  m = 80 kg
\ PA – PC = – 4.8 × 104 Pa 26. Let x = mass of copper in jewellery. (in gram)
20. Using result obtained in example 12: (50 – x) = mass of gold (in gram)
1 Volume of jewellery = volume of copper + volume
PB – PA = ​ __ ​  r w2 ◊ L2
2 of gold
fi PB = PA + __
1
​   ​  r w2 L2
2
x
fi  V = ​ ___ ( 50 – x
​    ​ + ​ ______
10 20 )
 ​ in cm3
 ​  
1
= 1 × 105 + __ ​   ​  × 103 × 202 × 12 Buoyancy = Loss in weight
2
= 3 × 105 Pa fi  Vrw g = 4 ◊ g

21. P3 = P2 + r gH = P0 + r gH x


fi ​ ___ (  50 – x
​    ​ + ​ ______
10 20 )
 ​ × (1) × g = 4 ◊ g [rw = 1 gcm–3]
 ​  
And  P3 – P1 = raL x 50 – x
g fi ​ ___  ​ + ​ ______  = 4  fi  x = 30 g
 ​ 
10 20
fi  P0 + r gH – P1 = r __
​   ​  L
2 27. For equilibrium of the balloon,
L
fi  P1 = P0 + r g ​ H – __ ( 
​   ​   ​
2 ) FB = 800 g ...(i)

for P1 = P0, we must have L = 2H Let the mass of balloon be m after it is reduced.
Buoyancy does not change as volume has not changed
22. Buoyancy = Loss in weight and it displaces same amount of air as earlier.
Vrw g = 0.03 ...(i) ma = FB – mg  fi  m (a + g) = 800 g

And Vd ◊ g = 0.1 N ...(ii) 800 × 10


fi  m = ​ ________ ​

  = 667 kg
2 + 10
Vdg 0.1
\ ​ _____  ​  
= ____
​    ​ 
Vrw g 0.03 \  D m = 800 – 667 = 133 kg
10 1 28. Consider an ice block (without gold) floating in
fi d = ___
​   ​  rw = ​ __ ​  × 104 kgm–3
3 3 water.
S.94  Mechanics II

____________
It displaces (ABCD) volume of water. When it melts, = ​÷2  × 9.8 ×  
19.6 ​
is makes water, which has volume exactly equal to
= 19.6 ms–1
(ABCD).
As the ball enters the liquid, it experiences buoyancy
(FB) apart from its weight (W).
Now acceleration (a) of the ball is
FB – W
a = ​ ______
m   ​ (upward)

rw
V rw g – V ​ ___ ​  ◊ g
2
= ​ _____________  ​
     = g (≠)
r
V ​ ___ ​ 
w
2
When the ice block has gold embedded in it, it will
have to displace more water to balance the weight of The ball retards and stops at C.
the gold. Now volume of water displaced is (CDFE).
For motion from B to C: v2 = u2 + 2ax
In fact, the volume (ABEF) is 19.6 times the volume
of gold piece as relative density of gold is 19.6. It fi  02 = (19.6)2 – 2 × 9.8 × y  fi  y = 19.6 m
means we need 19.6 cm3 water to balance the weight
of 1cm3 gold. 32. Volume of iron (the shaded part in diagram) is
When ice melts, it forms water to fill the volume mass of iron 6000/9.8
V1 = ​ ____________
    ​ = _________
   ​   ​ = 0.078 m3


ABCD. The volume ABEF will remain unoccupied. density of iron 7.87 × 103
Thus, the level of water will fall.
When placed in water
29. Area of cross-section of tube A = p r2 = 3.14 × 0.82
= 2.0 cm2 Buoyancy = loss in weight
Weight added = Increase in buoyancy due to displace- (V1 + V2) rg = 2000 [V2 = volume of cavity]
ment of extra water
fi  (0.078 + V2) × 103 × 9.8 = 2,000
fi D mg = (A × 2 cm) rw ◊ g


(  ) g
D m = (2 × 2 cm3) × ​ 1 ​ ____
cm3
   ​  ​ = 4 g
fi  V2 0.12 m3
33. Initially, FB + FS = W
30. If m mass of water is removed and poured into the
tube, the tube will sink further. The extra mass of fi  Vrg + kx1 = Vdg
water that it will displace will be equal to m (as
increase in mass of tube is m). It will make no [d = density of solid and r = density of liquid]
difference to level of water in the beaker.
\  kx1 = V(d – r) g ...(i)
  If you are rowing a boat and you drink some water
from the lake, has the level of water in the lake When the container is accelerated, geff = g + a
changed? Obviously, no. In equilibrium: FB¢ + FS¢ = W ¢ [In RF of container]
31. Speed of the ball when it reaches B (at water surface)
is fi  Vr geff + kx2 = Vd geff

fi  kx2 = V (d – r) geff ...(ii)


x2 geff
Dividing (ii) by (i) gives: __​ x  ​  = ___
​  g   ​ 
1
x2 10 + 2
fi ​ ____   ​ = ​ ______   fi  x2 = 3.6 cm
 ​  
3 cm 10

34. Centre of buoyancy is O. A slight tilt


in clockwise sense will cause the two
forces (FB and W) to form a clock-
____ wise couple. This will further rotate
  ​ 
v = ​÷2gh  the sphere.
Solutions  S.95

35. Centre of buoyancy is at mid-point of the submerged Lift force is F = D P A


part. For COM to be at the same location, mass of = 4,095 × (10 × 2) = 81,900 N
the particle shall be m.
41. Loss in pressure energy (per unit volume) = gain in
36. A1v1 = A2vP = A3v3 KE per unit volume
1
0.5 × 0.1 = 1 × vP = 0.05 × v3 fi  (3.5 –3) × 105 = ​ __ ​  r v2
2
fi  vP = 0.05 ms–1 ; v3 = 1 ms–1 1
fi  0.5 × 105 = ​ __ ​  × 103 × v2
37. Due to high speed of air 2
outside the house, pressure
fi v = 10 ms–1
is substantially lower than
inside pressure. The differ- 42. A1v1 = A2v2 = 1 cm3 s–1
ence in pressure causes an
upward force on the roof. 1 cm3 s–1 ___________
1 cm3 s–1
\  v1 = ________
​  2
 ​ 

= ​       ​ = 25 cms–1
38. Air, in immediate contact 4 mm 4 × 10–2 cm2
with the train, is moving at
1 cm3 s–1
speed equal to the speed of and  v2 = ________
​  = 50 cms–1
 ​ 

the train. In the narrow space between man’s body 2 mm2
and the train, speed of air is high. This results in Using Bernoulli’s equation
drop in pressure and the atmospheric pressure from
1 1
behind pushes the man towards the train. PA + ​ __ ​  r v 21 = PB + __
​   ​  r v 22
2 2
40. P1 and P2 are pressure at the top and bottom of
1
the wing. Height difference between two points is fi  PA – PB = __
​   ​  P (v22 – v12)
negligible. 2
1
1 1 = ​ __ ​  × 103 (0.52 – 0.252) = 93.75 Pa
\  P2 + __
​   ​  r v22 = P1 + __
​   ​  r v12 2
2 2
44. Speed of efflux will depend on height difference
1 between surface of water and end A.
fi  P2 – P1 = __
​   ​  r (v12 – v22)
2
h = H – l sin q
1
fi  D P = ​ __ ​  × 1.3 × (1202 – 902) = 4,095 Nm–2 ____ ____________
2 \ v = ​÷2gh   
  ​ = ​÷2g (H – l sin q) ​
  
S.96  Mechanics II

Volume conservation gives:


Worksheet 1 x
Ax = 16 Ay  fi  y = ___
​    ​ 
1. In a cylindrical vessel, force on base is equal to 16
weight of liquid in it. Now, P1 = P2
2. In case (C), the slant container wall exerts downward P0 + rw g (70 cm) = P0 + rHg ◊ g ◊ (x + y)
force on the liquid, thus force on the base increases. rHg
or, height of liquid will be maximum in vessel (C). fi  70 cm = ___
​   ​ (16y + y)
rw
Therefore, pressure at bottom is maximum in (C).
70
3. Pressure is normal thrust per unit area. fi ​ ________
     ​ cm = y  fi  y = 0. 3 cm
13.6 × 17
\  P = rg (H – h)
11. Vsub ◊ rl ◊ g = Vsolid rs ◊ g
4. When air is pumped out, pressure at the top decreases.
1
Pascal’s law says that pressure decreases by same fi  Vsub = Vsolid × __ ​   ​ 
3
amount everywhere.
\  One-third of its volume is submerged and two
5. Reason is same as in question 4.
third is exposed
If atmospheric pressure increases, buoyancy force on
12. Loss in weight = Buoyancy
a floating wood piece will not change.
60 – 40 = Vrw g
6. Pressure increases
as one moves hori- fi 20 = Vrw g ...(i)
zontally from point Also, 60 = Vrs g ...(ii)
1 to 2. Thus, force rs 60
Dividing (ii) by (i) gives ___
​   ​ = ___
​    ​ = 3
on a small segment rw 20
of the top surface
13. Let V = volume of the sphere
close to point 2 is V
higher than force on Weight of the sphere = weight of ​ __ ​  volume of water.
2
a segment of same When the sphere is about to sink (it is completely
size that is close to submerged); Weight of sphere + Weight of water in
1. Thus resultant force is nearer to 2. it = Weight of V volume water.
A2
7. Force gets multiplied by ___ (  )
20 2
​   ​ = ​​ ___
A1
​   ​   ​​ ​ = 400
1 of water.
V
\  Weight of water in sphere = weight of ​ __ ​ Volume
2
\  mg × 400 = 1500 g  fi  m = 3.75 kg 14. Weight loss = Buoyancy
8. 3P = P + rgH  fi  rgH = 2P For same loss in weight buoyancy must be same
When the height of water is lowered by a fifth, the 15. Let mass of A be 36 g and that of B be 48 g.
4H
new height is h = ​ ___ ​ . Pressure at bottom of the tank
5
is
4 8P 13P
P¢ = P + rgh = P + __ ​   ​  rgH = P + ___
​   ​ = ​ ____
 ​ 

5 5 5
10. If area of one side is A then the area of other limb
is 16 A. When water is added, let the mercury level
fall by x in the thinner tube. Mercury level moves
up by y in the other limb.

F1 and F2 are buoyancy forces on the two objects.


For equilibrium of the balance
Tension T1 = tension T2
\  36 g – F1 = 48 g – F2
Solutions  S.97

36 48
fi  36 g – ___ ​   ​ × 1 × g = 48 g – ___
​   ​ × 1 × g = 600 g + 40 g + F = 640 g ◊ + F
9 d
36 gram If the ball is released it will float with 20% of its
[F1 = V1 rw g = _______​  × 1gcm–3 × g;
 ​  volume outside water and 80% immersed in water if
9 gcm–3 volume of ball is V then
similarly F2 is written. d = density of second object]
0.8 V rw g = 40 g ...(i)
fi  d = 3 gcm–3
16. M = 1.5 kg To submerge the remaining ball the pin must apply
Initially, Mg – FB = Ma a force that is equal to additional buoyancy when
remaining 20% is pushed into water.
Mg Mg
fi  FB = Mg – ___
​   ​ 
 = ___
​   ​ 

2 2 \ F = 0.2 Vrw g = 10 g [from (i)]
\ Reading = 640 g + 10 g = 650 g
= 650 gram wt.
19. Pressure just below the piston is
12. g
P1 = P0 + ____
​     ​   [A = 800 cm2]
A
Pressure at same horizontal level in the left limb is
P2 = P0 + rgh
Since, P2 = P1
After the mass is decreased to m, the buoyancy force 12.g
has not changed as volume remain same. \ rgh = ____
​     
​ 
A
Mg g 12 kg
\  FB – mg = ma  fi ​ ___ ​   – mg = m ​ __ ​  h = ​ _______________________
(  )
2 2 fi   
     ​ = 0.15 m
kg
M
__ ​ 103 ​ ___3  ​  ​ × (800 × 10–4 m2)
fi  m = ​   ​  = 0.5 kg m
3
\  Decrease in mass = 1.0 kg 20. In a cylindrical container, if P is pressure at bottom
17. Weight of the hemispherical bowl = Buoyancy and A is area of base then
2 PA = Weight of content in the cylinder
fi W = ​ __ ​  p R3 ◊ rl ◊ g [R = radius = 0.5 m]
3 \ P1 A = Weight of water
2
\  Mass of bowl, M = __ ​   ​  p R3 rl P2 A = Weight of water + block.
3
Volume of material in the bowl is \ (P2 – P1) A = Mg
rl 21. Let F1 = downward force
M 2
V = ​ __  ​ = __
​   ​  p R3 __
​   ​ on top surface due to liquid
rs 3 rs
pressure.
2 1.2
= ​ __ ​  p (0.5)3 × ___ ​   ​  F2 = upward force on curved
3 20 lower face due to liquid.
If inner radius of the bowl is r, then volume of Buoyancy = F2 – F1
material is
2 Vrg = F2 – p R2 . (rgh)
V = __​   ​  p (R3 – r3)
3 fi F2 = Vrg + p R2 rgh
2 2 1.2
\  ​   ​  p (R3 – r3) = __
__ ​    ​ p (0.5)3 × ___
​   ​  22. Metal coin has high density
3 3 20
compared to water. To balance its weight, a large
fi  0.53 – r3 = 0.53 × 0.06 volume of water is displaced by the wooden block.
When the metal coin is removed, the cube moves up
fi  r3 = 0.53 × 0.94 fi  r = 0.5 × (0.94)1/3
and level of water goes down. If the coin is placed
= 0.5 × 0.98 inside water, it will displace volume of liquid equal
\  Inner diameter = 2r = 0.98 m to its own volume, which is not much.
18. Reading = weight of vessel and water + weight of ball 23. Let volume of ice in kerosene be V1 and that in water
+ vertical force needed to keep the ball sunk (F) be V2.
S.98  Mechanics II

Buoyancy = Weight 28. Upward acceleration of the ball inside water is given by
V1rk g = V2 rw g = (V1 + V2) rice g ma = FB – W


rk
___
​   ​ +
rw
V2
___
(  ) V2 rice
​   ​   = ​ 1 + ___
V1
​   ​   ​ ___
V1 rw
​   ​ 
Vra = V s g – Vrg  fi  a = ​ __
s
​   ​  – 1  ​ g
r (  )
Speed gained when it reaches the surface is given

(  )
V2 V2 by
fi 0.8 + ___
​   ​   = ​ 1 + ___
​   ​   ​ × 0.9 v2 = 02 + 2 ah  fi  v2 = 2 ah ...(i)
V1 V1
V V2 Outside the water, the ball moves under gravity.
​ ___  ​ (1 – 0.9) = 0.1  fi ​ ___ ​ = 1
2
fi Height attained above surface of water is
V1 V1

( 
24. Proceed as in last question. v2 ah
25. Considering equilibrium of the liquid, we can easily
H = ___
​    ​ = ___
2 g
s
​  g ​ = ​ __ )
​   ​  –  1  ​ h
r
see that force on it due to the container balances its
29. For equilibrium
weight.
2 ◊ T cos 45° + W = FB
26. The situation has been shown in the figure. Initially,
3 segments of the tube contain 20 cm3 volume of
water in each of them.
When the new liquid is poured in the left column, the
water moves to the right. Final situation is as shown
in second figure.

__ 4
fi ​÷ 2 ​  T + Mg = ​ __ ​  p R3 rw ◊ g
3
4
__
​   ​  p R3 rw g – Mg
3
fi T = ​ ______________ __ ​     
÷   
​ 2 ​
30. Let length of the block be L. compression in the
L
spring = __
​   ​ 
rgh = rwg ◊ (60 cm) 3
fi 4 rw gh = rwg (60 cm)
L
For equilibrium: k ​ __ ​  + (AL) ​ __
3
r
(  )
​   ​   ​ g = AL ◊ r ◊ g
3
fi h = 15 cm
fi k = 2 rAg
\  Length of new liquid in the tube = 15 + 20 = 35 cm
31. For equilibrium, force on dumbbell = 0
\ Volume = 35 cm3
3
27. ​ __ ​ of the volume of the combined cylinder is submerged.
4
\ (  )
V
2 ​ __
​   ​  r    ​ = (3 M + m) g
2
Buoyancy = Weight fi Vrg = (3 M + m) g ...(i)
3 V V
​ __ ​  Vd g = __
​   ​  d1g + __
​   ​  d2g For rotational equilibrium, let us make torque about
4 2 2
the centre of right ball to be zero.
3 d
fi d1 + d2 = ___
​   ​  V
2 mg ◊ l + Mg ◊ d = __
​   ​  rg d
2
Since, d1 > d2 V
fi (Vr – 3 M) l = __ ​   ​  rd – Md [Using (i)]
2
3 d
\ 2 d1 > ___
​   ​  (Vr – 2 M)d
2 fi l = ​ __________    ​
2(Vr – 3 M)
3 d
fi d1 > ___
​   ​ 
4
Solutions  S.99

32. Force due to liquid pressure on any small element on 37. Pressure at top-right corner = 0
the surface of the cylinder is normal to the cylinder. This is because there is no air and the liquid tends
It is radial and will have no torque about O. to move backward.
Therefore, F = 0. L L
\ Pcentre = rg ​ __ ​  + ra __
​   ​ 
[Applying any force F will cause the gate to rotate]. 2 2
Which force balances the horizontal force on the gate 39. A1v1 = A2v2 + A3v3
due to liquid pressure? Obviously, hinge force.
A × 3 = A × 1.5 + 1.5 A ◊ v
33. Bucket falls down with an acceleration
fi v = 1 ms–1
2 mg –  mg __ g
a = _________
​   ​    = ​   ​  40. Continuity equation requires v1 = v2.
3 m 3
[m = mass of block] 41. A1v1 = A2v2
\  Gauge pressure at the bottom of the bucket is
10 × 1 = 5 × v2
(  )
g
P = r ◊ geff h = 103 × ​ g – ​ __ ​   ​ × 0.15
3 fi v2 = 2 ms–1
3 2______
× 10
= 10 × ​   ​  × 0.15 = 103 Pa
3
= 1 kPa
34. Conceptually similar to Q.21.
35. Let side length of cube be a.
Bernoulli’s equation:
1
Volume ADE = ​ __ ​  × AE × ED × a
2 1 1
P1 + ​ __ ​  r ​v2​1​ =
​  P2 + __
​   ​  r ​v2​2​​ 
2 2
1 1
= ​ __ ​  a ◊ (a tan q) ◊ a = __
​   ​  a3 tan q 1 1
2 2 2000 + ​ __ ​  × 103 × 12 = P2 + ​ __ ​  × 103 × 22
2 2
fi P2 = 500 Pa
42. Bernoulli’s equation
1 1
​ __ ​  r v22 + P0 + 0 = ​ __ ​  r v12 + P0 + rgh
2 2

1
Given volume, ADE = __ ​   ​  a3
3
1 1
\ ​ __ ​  tan q = __
​    ​
2 3
2
fi tan q = __
​    ​
3
If f is acceleration of the tank then,
f __ 2 2 g \ v22 = v12 + 2 gh = 12 + 2 × 10 × 0.15
fi ​ __ ___
g  ​ = ​ 3  ​  fi  f = ​  3 ​ 
\ v2 = 2 ms–1
36. The question has been created to test your
Continuity equation:
understanding of the contribution made by atmospheric
pressure. A2v2 = A1v1
Consider the equilibrium of the piston. A2 × 2 = 10–4 × 1  fi A2 = 5 × 10–5 m2
____
F + P0 A = P A   ​ 
43. Speed of efflux at square hole is v1 = ​÷2 gy 
fi F + P0 A = [P0 + rg (H + h)] A Volume flow rate from square hole is
fi F = rg (H + h) A
S.100  Mechanics II

____
  ​ 
Q1 = A1v1 = L2 ​÷2 gy  45. Fluid has more potential energy at A and more
______ pressure energy at B.

Speed of efflux at the circular hole is v2 = ​÷2 g (4y) ​   47. Bubbles start forming at the tip when pressure there
____
  ​ 
= 2 ​÷2 gy  is equal to atmospheric.
Volume flow rate from circular hole is Therefore, pressure in the liquid at the level of
____ hole is
   
Q2 = A2v2 = p R2 2 ​÷2 gy ​
P = P0 + rgh
Given Q1 = Q2
Applying Bernoulli’s equation between points just
L
fi L2 = 2p R2  fi  R = ​ ____    ​ 
___ inside the hole and outside it, gives
÷    
​ 2 p ​ 1
44. P1 – P2 = r Hg ◊ g ◊ D h P + 0 + 0 = P0 + __ ​   ​  r v2 + 0
2
D P = r Hg ◊ g ◊ D h 1
__
fi P0 + rgh = P0 + ​   ​  r v2
2
____ __________
Note that flowing liquid that fills up some space in fi    
v = ​÷2 gh ​ = ​÷2  × 10 ×   0.2 ​ = 2 ms–1
manometer will have no effect in D P as rHg >> r. ____
   
48. Initially, v = ​÷2 gh ​
Range R µ v
For range to double, the speed of efflux must double.
Let extra pressure on water surface be D P for speed
of efflux to be 2v.
1
P0 + D P + rgh = P0 + ​ __ ​  r (2v)2
2
____
fi D P = 2 r ​​( ​÷2 gh 
  ​  )2​​ ​ – rgh = 3 rgh
= 3 × 103 × 10 × 10 = 3 × 105 Nm–2

49. Pressure just beneath the piston is


Mg 10 × 10
P = P0 + ​ ___   
​ = P0 + ​ __________
    ​
  Speed does not change as cross section is uniform. A 1000 × 10–4
Height difference between point 2 and 1 is
= (P0 + 103) Nm–2
h = d ◊ tan q
Using Bernoulli’s equation,
1 1
P1 + __
​   ​  rv2 + 0 = P2 + __
​   ​  r v2 + rgh
1
P + rgh = P0 + __
​   ​  rv2
2 2 2
\ P1 – P2 = rg (d tan q) 1
fi P0 + 103 + 103 × 10 × 0.5 = P0 + __
​   ​  × 103 × v2
rd 2
rHg ◊ g ◊ D h = rg ◊ d ◊ tan q fi D h = ___
​    ​  tan q
rHg fi v2 = 12  fi  v = 3.46 ms–1
Solutions  S.101

Our solid cylinder will also experience same force on


Worksheet 2 curved face. Vertical force due to surrounding liquid
1. When the block is submerged, the liquid applies a on semi-cylindrical liquid element is
buoyancy (F) on the block. Due to this reading of Fv = Weight of semi-cylindrical liquid
the spring balance will decrease. p R2
= ​ ____  L r  ◊ g
 ​ 
From Newton’s third law, the block applies F on 2
the liquid in downward direction. Thus, reading of Same vertical force will act on solid cylinder also.
balance B will increase. Torque is zero since force due to liquid on any surface
2. Siphon works only when discharge end is below the element of the cylinder is radial.
level of water in the tank. It means h3 > 0. 6. Weight of coin = Vdg
Pressure at 2 can be calculated using Bernoulli’s Volume of water having same weight as the coin is
equation
Vdg d
1
P2 + __
1
​   ​  rv2 + rgh3 = P3 + __
​   ​  r v2 + 0 D V = ​ ____ ​  = V ​ __ ​
rg r
2 2
fi P2 = P0 – rgh3 When the coin is removed, the block will rise up so
as to reduce the volume of submerged part by D v.
With increase in h1, pressure decrease at the top. d Vd
Siphon will not work if h1 is too large such that \ D l A = V ​ __ ​  fi  D l = ___
​    ​
r rA
pressure at the top becomes zero.
3. A = area of tube To find change in level of water, let us assume that
volume of submerged part of the block is V1 and V2
Volume flow rate = Av
before and after the coin is removed. Also, assume
Momentum of water particles that hit the wall in time D t that volume of water in the container is V0.
is = (Av D t r) v = Ar D t v2
Ar D t v2 Before coin is removed
Force = rate of change of momentum = ​ _______    
​ 
D t
= Arv2
\ F µ v2
If v is doubled, F becomes four times.
Kinetic energy of water coming out in D t time is
1
D k = __
​   ​  (Av D t r) v2
2
D k 1 Volume ABCD = V0 + V1
​ ___ ​ =   __
​   ​  Ar v3
D t 2 fi A1 H = V0 + V1 ...(i)
If v is doubled, rate of loss of becomes 8 times After the coin is removed from the block and placed
5. Average pressure between the top and bottom of the in the tank
cylinder is Volume PQRS = V0 + V2 + V
rg ◊ 3 R
Pav = rgR + ______​   ​   = 2 rgR
2 fi A1h = V0 + V2 + V ...(ii)
Consider a semi-cylindrical (i) – (ii) gives A1 (H – h) = V1 – V2 – V
water element (as shown)
inside a tank full of water. fi A1 D h = (V1 – V2) – V
The element of water is in
equilibrium. Weight of coin
= ​ _____________
rg       ​– V
Horizontal force on face
ABC due to surrounding
liquid = force on flat rectangular face represented
Vdg
= ____
d
​   ​ – V = V ​ __
rg
​    ​ – 1  ​
r (  )
by AC due to surrounding liquid.

FH = 2R ◊ L ◊ rav = 2 R . L 2 rgR = 4 rg LR2


\
V d
D h = ___
A1 r (  )
​    ​ ​  __
​    ​ – 1  ​
S.102  Mechanics II

7. Volume of liquid that overflows = volume of the ball So removing the block and filling the dark shaded
(V). When ball is submerged, it experiences buoyancy portion with liquid will not change the situation.
equal to FB = Vrg. It applies same force on the liquid
13. Px + rw g (175 cm) – rHg g (112 cm) + rw g (75 cm)
in downward direction. Thus, the ball applies a force
FB = Vrg on the liquid downward. But same weight – rHg g (88 cm) – rw g (150 – 88 cm) = Py
of the liquid has overflown also. Thus, reading will
not change. fi Px – Py = rHg g (200 cm) – rw g (188 cm)
In second case, reading is = 13600 × 9.8 × 2 – 1000 × 9.8 × 1.88
R2 = W + mg – Weight of liquid that overflows = 266560 – 18424 = 248136 Nm–2
= W + mg – Vrg  248 Nm–2
____ _________
8. v2 = ÷   ​ = ​÷2  × 10 × 5 ​
​ 2gH    = 10 ms –1
14. Let M be mass of ship and m be mass of cargo. Let
v2 A2 ​r2​2​​  the volume of sea water displaced initially be V0.
1
v1 = ____
​   ​  = v2 __
​  2 ​  = 10 × __
​   ​  = 2.5 ms–1 Cross-sectional area of the ship is A.
A1 ​r​1​​  4
rs = density of sea water, rr = density of river water
Pressure at the location of vertical tube can be cal-
culated using Bernoulli’s equation. (M + m) g = V0 rs g  fi  M + m = V0 rs ...(i)
1
P + __
​   ​  r v12 + 0 = P0 + 0 + rgH When cargo is removed in river, the ship rises by Y,
2
1
fi P = P0 + rgH – ​ __ ​  r v12 it means
2
mg = AY rr  fi  m = AY rr ...(ii)
1
fi rgh + P0 = P0 + rgH – ​ __ ​  r v12
2 Putting in (i) gives  M = V0 rs – AY rr ...(iii)
2
v1 2.5 2
After unloading of cargo, when the ship is back in
fi h = H – ​ ___   ​ = 5 – ______
​    
 ​ = 4.69 m
2 g 2 × 10 sea water
10. As the level of water falls, air inside the tank expands Mg = (V0 + AX – AY – AZ) rs g
and pressure at the surface of water decreases. Flow
fi V0 rs – AYrr = V0 rs + (AX – AY – AZ) rS  [using (iii)]
will stop when (P1 = P2) (= P0).
fi Yrr = rs (Y + Z – X)
rs Y
fi ​ __ ​  = ________
​       ​
rr Y + Z – X
15. Liquid A applies force normal to the cylinder wall
which cancels out. Liquid A does not apply force
on cylinder. But it does not mean that liquid A has
no role to play. Its pressure increases the pressure in
11. At large depths, due to large hydrostatic pressure, liquid B. For finding h, consider equilibrium.
the size of balloon with decrease substantially. It will
displace less water and buoyancy will fail to balance Buoyancy = weight
the weight of the system. It will sink further. A hB ◊ rB ◊ g + A hA rA ◊ g = A (h + hA + hB) d ◊ g
12. Equilirbium is maintained for d £ dw. As long as the fi 0.8 × 1.2 + 1.2 × 0.7 = (h + 1.2 + 0.8) × 0.8
block floats, it applies a force on the liquid that is
equal to weight of displaced liquid. fi h = 0.25 cm
When cylinder is depressed by h, the resultant force
on it is upward and is equal to change in buoyancy.
Length of cylinder inside A does not change but its
length inside B increases by h.
\ F = A h rB g

The block exerts downward force that is equal to fi ma = A h rB g


weight of dark shaded volume of liquid (= Buoyancy). fi A (h + hA + hB) d ◊ a = A h rB ◊ g
Solutions  S.103

0.25 × 1.2 g
fi a = ​ _________________
      ​ g = __
​    ​ Worksheet 3
0.8(0.25 + 1.2 + 0.8) 6
16. Volume of water displaced must have weight equal 1. Initially, spring is relaxed. Buoyancy balances the
to the bowl weight.
\ Vr g = mg \  Fraction of volume submerged = ___
d
​     ​ = 0.8
rw
m
fi V = __
​   ​ 
r 20% Volume of the cube is outside water.
m
Volume of steel in bowl V0 = __ ​   ​  fi  3 cm × 0.2 = 0.6 cm height is outside water.
d
When submerged completely, volume of water When the cube is completely submerged, it compresses
displaced is V0 the spring by 0.6 cm = 0.006 m
\ ma = mg – FB For equilibrium:
mrg Additional weight (W) + weight of block = Buoyancy
ma = mg – V0 rg = mg – ____ ​     
​ 
d + spring force
\ (  ) r
a = g ​ 1 – ​ __ ​ 
d
W + (3 × 10–2)3 × 800 × 10

= (3 × 10–2)3 × 1000 × 10 + 50 × 0.006
fi  W = 0.35 N
2. Let mass of Cu = x g
Mass of Zn = (12.9 – x) g
x 12.9 – x ___ x 12.9 – x
Volume of alloy = ___ ​ + ​ _______
​           = ​     ​ + ​ _______
​   ​ 

rcu rzn 8.9 7.1

Loss of weight in water = Buoyant force


x
\  (12.9 – 11.3) g = ​ ___ ( 
12.9 – x
​     ​ + ​ _______
8.9 7.1
 ​ rw ◊ g
 ​   )
x 12.9 – x
fi  1.6 = ___
​     ​ + ​ _______
 ​  
  [\  rw = 1 gcm–3]
8.9 7.1

Solving gives x = 7.6 g

3. (a) Vsub ◊ rHg ◊ g = V rFe ◊ g

Vsub rFe 7.2


fi  ​ ____
  ​  = ___
​    ​ = ____
​    ​ = 0.53
V rHg 13.6
fi  53% of volume is submerged and 47% is outside
Hg.
\  Height of block above Hg is 0.47 × 5 = 2.35 cm
(b) Let y = height of water level for equilibrium.
Weight = Buoyancy
fi  W = weight of displaced water + weight of
displaced Hg.

fi  x2 ◊ rFe g = x2 y rw g + (x – y) x2 rHg ◊ g

y rHg
rFe
fi ​ ___ ​ =
rw
y
__ ( 
​ x ​ + ​ 1 – __ )
​ x ​  ​ ___
​    
rw

S.104  Mechanics II

fi 103 × 10 (h – 0.8) = 1.6 × 105 – 1.0 × 105


= 0.6 × 105
fi h = 6.8 m
6. (a) If side length is x, volume of iron used to make
the box is
VFe = 6x2 ◊ t [t = 1mm = thickness]
Mass of box m = rFe VFe = 6rFe x2 t
y y y 6.4
fi  7.2 = __
​ x ​ + 13.6 – 13.6 __
​ x ​  fi ​ __ ​ = ____
​    ​  With increasing x, mass of box increase as m
12.6 µ x2. But with increasing x, its overall volume
6.4 increases as V µ x3. Therefore, larger box will
fi  y = 5 × ____
​    ​ = 2.54 cm have a large volume so as to displace sufficient
12.6
quantity of water needed for floating.
4. (a) Weight and buoyancy are forces on the ball.
A large box will not sink.
rl Æ 0 implies that there is no buoyancy and (b) For the box to float while remaining completely
the ball is falling freely.
Submerged: Buoyancy = weight
KE gained when ball falls through 8 cm is given
to be 4 J. x3 ◊ rw g = 6 rFe x2 t ◊ g
\  KE = mgh r Fe
fi  x = 6 t ◊ ​ ___ ​ = 6 × (1 mm) × 8
fi  4 = m × 10 × 0.08  fi  m = 5 kg rw
(b) Density of the ball is 1.5 gcm–1. This is because = 48 mm = 4.8 cm
it experiences no net force when placed in the
liquid of density rl = 1.5. The graph shows that For x > 4.8 cm, the box floats with its some part
its KE is nearly zero even after it has travelled remaining outside water.
through 8 cm. [It travelled with a speed nearly 7. Let volume of sphere be V0 and Volume of foam in
equal to zero and never accelerated] it be V.

Volume of concrete = V0 – V
For floatation: Buoyancy = weight
V0 rw g = (V0 – V) rC g + VrF g

If density of liquid = 3.0 gcm–3, net force on the rC rF


fi  V0 = (V0 – V) ​ ___  ​ + V ​ ___  ​
ball will be mg upwards. It will move up with rw rw
acceleration g. 2
fi  V0 = (V0 – V) × 2.4 + V × 0.3  fi V = __ ​    ​ V0
\ After moving a distance 8 cm in upward 3
direction, its KE will be 4 J (as in (a).) 2 1
Thus, ​ __  ​ rd volume is foam and remaining ​ __  ​ rd is
5. For water to leak, the seal must move 3 3
up. It means force due to water on concrete.
the seal must become greater than
mc rc ◊ Vc 2.4 × V0/3
its weight. In critical case- \ ​ ___
mF  ​ =
_____
​  =
 ​  ​ __________
     ​ = 4
rF VF 0.3 × 2V0/3
PA = Mg + P0A0
fi P × 0.01 = 1 × 105 × 0.015 + 10 × 10 8. Let density of cube be d.
fi P = 1,6 0,000 Nm–2 = 1.6 × 105 Nm–2 Vdg = W
fi P0 + rg (h – 0.8) = 1.6 × 105 fi  (0.1 m)3 × d × 10 = 5
Solutions  S.105

fi  d = 0.5 × 103 kgm–3 11. Initially spring is compressed by x0 such that


Density of cube is less than that of liquid. It can float kx0 = P0A
in the liquid such that Where P0 = atmospheric pressure, A = area of
d 0.5 5 piston.
vsub rl g = vd ◊ g  fi  vsub = v ◊ ​ __  ​  = v × ___
​   ​ = __
​   ​  v
rl 0.8 8 If the device is kept at a location where pressure is
Thus, the cube will float with its vertical edge remain- P then the spring will get further compressed by x
5 such that
ing submerged up to length = __ ​    ​ × 10 = 6.25 cm
8
The tank contains liquid up to much greater depth. PA = k (x0 + x)  fi  (P – P0) A = kx
Thus the cube is floating without touching the base kx
of the tank. fi  gauge pressure = P – P0 = ​ __ ​ 
A
If height of liquid in tank is h, then, (a) Least count of x = 1 mm
Volume (ABCD) = Volume of \  Smallest gauge pressure that can be
liquid in the tank + volume of recorded is
submerged part of cube
kx 100 × 1 × 10–3
= __
​   ​  = ​  ____________
  
   ​= 1,000 Nm–2
\  15 × 15 × h = 15 × 15 A 1 × 10–4
× 8 + 6.25 × 10 × 10
= 1 kPa
97
fi  h = ___
​   ​ cm = 10.8 cm (b) At depth of 2 m, the gauge pressure is
9
= r gh = 103 × 10 × 2 = 2 × 104 Nm–2
9. Both buoyancy and weight act at the centre of the
rod. = 20 kPa
For rotational equilibrium of the \  Further compression in spring (= displace-
rod, torque on it about O must be ment of piston) = 20 mm = 2 cm
zero. 12. A >> a, therefore level of water in the tank will
L practically remain unchanged.
T ◊ L cos q = ​ __ ​  sin q ◊ (W – FB)
2 At every moment, half the candle will be submerged
when length becomes 18 cm, after 1 hour half the
2T
fi  tan q = ​ ______
    ​ length (= 9 cm) will be above water. Thus, the top of
W – FB the candle has moved down by 1 cm only.
q is maximum when T is maximum. 13. Consider a water volume represented by ABC inside
\ tan qmax = a tank full of water. Horizontal force by surrounding
water on face AC = Horizontal force by surrounding
2 × 45
​ _________________________________________
   
       ​ water on face AB.
0.012 × 1 × 2000 × 10 – 0.012 × 1 × 1000 × 10

3
= ​ __ ​ 
( 
0 + r gh
\  Fx = ​ ​ _______
2
 ​   ) 1
 ​ hb = __
​   ​  r gbh2
2
4
fi qmax = 37°
10. Refer to Example 12.


1
2 (  )
2L 2
PB – P0 = ​ __ ​  r w2 ​​ ___
​   ​   ​​ ​
3
where P0 = pressure at rotation axis
1
fi  And  PA – P0 = __
L 2
​   ​  r w2 ​​ __
2
​   ​   ​​ ​
3 (  )
Taking difference of the two equations:

[  ]
1 4 1 1 Vertical force by surrounding water on face AC =
PB – PA = __
​   ​  rw2 L2 ​ __
​   ​  – __
​   ​   ​ = __
​   ​  r w2 L2 Weight of volume ABC.
2 9 9 6
1 1
fi  Fy = __ ​    ​ h ◊ (h tan q) b r g = __
​   ​  r ghb2 tan q
2 2
S.106  Mechanics II

\  Resultant force on AC is 15. (a) Refer to Example 25. Range is maximum when
_______ ________ H + h ___ 5h
  x2 + Fy2 
F = ​÷F
1
 ​= __ ​ 1  + tan2 q ​
​   ​  r gbh2 ÷
2
  y = ​ _____
2
 ​ 
 = ​   ​ 
6
H + h ___ 7h
1 (b) Range is maximum if y = ​ _____
 ​ 
 = ​   ​ 
= __
​    ​ r gbh2 sec q 2 6
2
But y cannot be larger than h.
Same force will be applied by water on the wall.
\ Hole is at the bottom of the tank for range to
Alternate (by direct integration) be maximum
Consider a strip of width dz in the wall, as shown. \ y=h
Area of the strip dA = b ◊ dz 16. (a) Area of cross-section of the pipe (plug) is
Depth, y = z cos q  fi  dy =  ◊ cos q ...(i)
A = p r2 = 3.14 × (0.02)2 = 1.26 × 10–3 m2
Pressure at the strip, P = rgy
Outward force on the plug due to water pressure
is
F = PA = rgh ◊ A = 103 × 9.8 × 6 × 1.26 × 10–3
= 74 N
Friction must be equal to 74 N to prevent the
plug from moving.
____ _________
  ​ = ​÷2  × 9.8   
(b) Speed of efflux v = ​÷2gh  × 6 ​
= 10.84 ms–1
Volume of water flowing out per second
= Av = 1.26 × 10–3 × 10.84
Force on the strip (normal to wall) is
= 1.36 × 10– 2 m3 s–1
dF = P ◊ dA Water flowing out in 24 h is
= rgy ◊ bdz = rg b sec q ◊ ydy [using (i)] = 1.36 × 10–2 × 24 × 60 × 60 = 1,175 m3
h ____
\
1
F = rg b sec q ​Ú ​  ​ydy = ​ __ ​  rg bh2 sec q   ​. 
17. When height of water is h, rate of outflow is a ​÷2gh 
0 2 When the rate of outflow becomes equal to the rate
14. Tension in thread is of inflow, the level of water becomes constant.
V a02 v2
T = W – FB = V (3r0) g – __ ​   ​  r0 g ____
2   ​ = a0v  fi  h = ​ _____
a ​÷2gh   
 ​ 
5 2g a2
= ​   ​  V r0 g __
2 18. (a) Apply Bernoulli’s equation between E and D
where r0 = density of water and 3 r0 is density of 1
P0 + __ ​   ​  rv2 + 0  fi  P0 + 0 + rg (5 m)
4 2
the ball. V = __
​   ​  p r3 = volume of the ball
3 fi  v2 = 2 × 10 × 5  fi  v = 10 ms–1
COM of the rod is at a distance ​ ​  ____
brim on right side of it.
(  )
y– x
2
 ​  
 ​ from the (b) Speed at all points inside the pipe is same.
Using Bernoulli’s equation between B and D
For equilibrium, torque about the brim (the support) gives
must be zero. 1 1
PB + __ ​    ​ rv2 + rg (6.5 m) = P0 + __
​    ​ rv2 + 0
(  )
5 y–x 2 2
​ __ ​  V r0 g ◊ x = mg ​ ​ ____ ​  
 ​ 5
2 2 fi  PB = 1 × 10 – 900 × 10 × 6.5
4
fi  5 × __ ​   ​  × p × (0.5 × 10–2)3 × 1 × 103 × x = 41.5 × 103 Nm–2
3
= 4.4 × 10–3 (y – x) = 41.5 kNm–2
x 4.4
fi  5.22 x = 4.4 (y – x)  fi ​ __y ​ = ____ ​    ​ = 0.46 (c) PA = PE – rg (1 m) = 1 × 105 – 103 × 10 × 1
9.62
= 0.9 × 105 Nm–2
Solutions  S.107

1
(d) PA¢ + ​ __ ​  rv2 = PA fi
10 min
t = ______
​  __ ​   = 7.07 min
2
÷   
​ 2 ​
1
fi  PA¢ = 0.9 × 105 – __
​   ​  × 900 × 102 22. Let speed of efflux be v.
2
Applying Bernoulli’s equation between a point just
= 4.5 × 104 Nm–2 outside the hole and a point just inside it, we get
1
(e) From solution of part (a) it is evident that v is (P0 + rg ◊ 2h + 2r ◊ g ◊ h) + 0 = P0 + __ ​   ​  rv2
independent of r. 2
____
1 fi    
v = ​÷8gh  ​
19. W = DU + D K = rg (h2 – h1) + __ ​   ​  r (v22 – v12)
2 Volume of liquid flowing out per second = Sv
v1 Mass of liquid flowing out per second = 2r (Sv)
Since, A2v2 = A1v1  \ v2 = __
​   ​  = 0.5 ms–1
2 Momentum gained by the outgoing liquid per second
1 = (2r sv) v
\ W = 103 × 10 × (5 – 2) + __
​   ​  × 103 × (0.52 – 12)
2 \  Force experienced by the tank horizontally
= 3 × 104 – 0.375 × 103 = 29625 J towards left is
1 1 Fthrust = 2rsv2 = 16 rsgh
20. (a)  P1 + ​ __ ​  rv12 = r2 + __
​   ​  rv22
2 2 Mass of the tank and its content
1
fi D P = P2 – P1 = __ ​   ​  r(v12 – v22) = M + (2h A)r + (2h A) (2r)

( 
2
= M + 6h rA

)
2
1 Q Q2
= ​ __ ​  r ​ ___
​  2 ​ – ___
​  2 ​   ​ For friction to be able to prevent sliding
2 A1 A2
mN ≥ Fthrust

m (M + 6h rA) g ≥ 16 rsgh


16 rsh
fi m ≥ ​ _________  ​

M + 6h Ar

A2
23. A1v1 = A2v2  fi  v1 = ___
​    ​ v
where Q = A1v1 = A2v2 = volume flow rate (m3/s) A1

(  ) (  ) P1 = P0 ;  and  P1 – P2 = rg (h2 – h2)


2
1 1 1 rQ
\ D P = ​ __
​   ​  rQ2  ​ ___
​  2  ​ – ​ __
​  _____
 ​  ​  2 ​  
 ​
2 A1 2 A2
fi  P2 = P0 – rg (h1 – h2)
1
D P = a ​ ___2  ​ – b 1
\  P0 + __
1
​   ​  rv12 + rgh1 = P2 + ​ __ ​  rv22 + 0
A1 2 2

(  )
2
1 1 rQ A 2
where a = ​ __ ​  rQ2  and  b = ​ __ ​  ​ ____
 ​  1
fi  P0 + __
​   ​  r ​​ ___
2
​   ​ v  ​​ ​ + rgh1
2 2 A22 2 A1
(a) D P = 0  when  A1 = A2 1
= P0 – rg (h1 – h2) + __ ​   ​  rv2
2

[  (  ) ]
1 1
\ ​ ___2  ​ = 8  fi  A2 = ____   ​ = 0.35 m2
​  __
A2   
2​÷2 ​ 1 A2 2
fi  rg (2h1 – h2) = __
​   ​  rv2 ​ 1 – ​​ ___
​    ​  ​​ ​  ​
2 A1
600 × 103
(b) From graph, slope is = ​ _________
(  )
 ​  = 75 × 103
8 (2h1 – h2) A2 2
fi  2g ​ ________
 ​ = 1 – ​​ ___
  ​   ​   ​​ ​
1 75 × 103 × 2 2 A1
\ ​ __  ​ rQ2 = 75  fi  Q2 = ​ ___________  ​
     v
2 103

÷ 
______________
A2 2g (2h1 – h2)
Q = 12.25 m3s–1 fi  ​ ___ ​ =
  ​ 1 – ​ ___________
 ​ ​
      
A1 v2
21. Refer to Example 26

÷ 
____
__ t H/2 24. P2 = P3
  ​   fi ​ ______
t µ ​÷H  = ​ ____
   ​  ​     
​ ​  
10 min H P1 + r0gh = P0  fi  P1 = P0 – r0gh
S.108  Mechanics II

1 1
fi ​ __ ​  rv2 + P0 – r0gh = P0  fi ​ __  ​ rv2 = r0gh
2 2

÷ 
______
2r0 gh
fi v = ​ ______
​   ​ ​ 


r

÷ 
__________________
2 × 103 × 10 × 10–2
= ​ __________________
​   ​ ​
  
      12.4 ms–1
1.3
In simple language, the loss in pressure = gain in KE
per unit volume by air
1
\ D P = __ ​   ​  rv2
Bernoulli’s equation 2
1 where D p = amount by which pressure at 1 is less
​ __ ​  rv2 + P1 = P0 than atmospheric pressure
2
1
\  r0gh = __ ​   ​  rv2
2
Chapter 9 Surface Tension and Viscosity

8. Pressure at a depth of 10 m inside water is


Your Turn
1. F = S L = (0.075 Nm–1) (0.1 m) = 7.5 × 10–3 N P = P0 + rgh = 1.013 × 105 + 103 × 101 × 10
2. As the disc is pulled up, the water surface clings to
= 2.013 × 105 Nm–2
the outer periphery and the surface tension force acts
downward on the disc. In the extreme case the water Excess pressure inside air bubble is
surface becomes tangential (i.e., vertical) to the disc.
If the disc is moved up further, the liquid surface 2 S 2 × 0.072
leaves it. D P = ___
​  r   ​  = ​ ________–3
  ​ = 1440 Nm–2
0.1 ×10
\  Pressure inside the bubble
= 2.013 × 105 + 1440
= 2.0274 × 105 Nm–2

9. Thickness of wall of A is more, so that it contains


more liquid. But thickness of wall is always much
\  Downward force on disc due to surface tension smaller compared to radius of the bubble and is of no
is concern in deciding the pressure inside a bubble.
S
F = 2p R ◊ S = 2 × 3.14 × 0.05 × S 10. Excess pressure at A: D PA = __
​ r ​ 
fi 3 × 10–2 = 2 × 3.14 × 0.05 × S S
Excess pressure at B: D PB = ​ __ ​ 
fi S = 0.096 Nm –1 R
\  Difference in pressure at A and B is
3. Spherical
4. A = 4p r2 = 4 × 3.14 × (0.01)2 = 1.256 × 10–5 m2 1 1
( 
= S ​ ​ __r ​  – __ ) (  1
​   ​   ​ = 0.07 × ​ _______
R
​ 
2 × 10
1
 ​ – ​ _______
  –3     
4 × 10–3 )
 ​  ​

\ ( 
m
J
U = S ◊ A = ​ 0.47 ___ )
​   2 ​  ​ (1.256 × 10–5)
1
= 70 × __
​   ​  = 17.5 Nm–2.
–6 4
= 5.9 × 10  J
5. R = 1 cm = radius of original drop 4 S 4 S
11. P0 = ___
​   ​   fi  R = ___
​   ​ 
r = radius of each of the smaller droplets formed R P0
n = 1000 Surface energy = surface area × S

(  )
1
__
4 4 R3 ​   ​  R
n ◊ ​ __ ​  p r3 = __
​   ​  p R3  fi  r = ​​ _____
​    ​  ​​3 ​ = ___
​    ​ = 0.1 cm
3 3 1000 10 = 2 × 4 p R2 × S [There are two surfaces]
Increase in surface energy is
2 2
 A = n ◊ 4p r – 4 p R = 36 p × 10 –4
m2
4 S 2
= 8 p ​​ ___ (  ) S3
​   ​  ​​ ​ ◊ S = 128 p  ​ ___2  ​ 
P0 ​P​0​​ 

\  Increase in surface energy is
2 S cos q __________________
2 × 0.07 × cos 60°
D U = S ◊ D A = 0.075 × 36 × 3.14 × 10–4 15. h = ​ _______
   
​ 
= ​     
    ​= 1.43 × 10–2 m
rrg –3 3
0.5 × 10 × 10 × 9.8
= 0.85 × 10–3 J
= 1.43 cm
7. Work done = Increase in surface energy
2 S cos q
= S . D A = S ◊ (2 l x) 16. Vertical rise, h = _______
​      ​ = 3.57 cm

rrg
[There are two surfaces] h
l = ______
​     ​ = 7.14 cm
\ W = 0.03 × 2 × 0.1 × 10–3 = 6 × 10–6 J sin 30°
S.110  Mechanics II

2 S cos q 2 × 0.5 × cos 120°


17. h = _______
​      ​ = ​  ______________________
     
     ​ Worksheet 1
rrg 1 × 10–3 × 13.6 × 103 × 9.8
2. The thread will get pulled by liquid on side B. It will
= – 3.75 × 10–3 m = – 3.75 mm become convex towards B and concave towards A.
2S 1
Negative sign indicates that mercury level falls inside 4. h = ____ ​  fi  h µ __
​      ​ g ​
Rrg
the tube by 3.75 mm
2 S Inside a satellite, geff = 0
18. Pressure just below mercury surface is P0 + ___
​   ​ .
R
\ h = •
Pressure at the same level can also be written as
P0 + rgh. For a tube of finite length, water will rise to full
length.
2 S 2 S 6. There are two surfaces
\ P0 + ___   P0 + rgh  fi  R = ____
​   ​ = ​      ​
R rgh
\ U = 2 × 0.02 × 5 = 0.2 J
19. Due to viscous forces between different layers. mass
20. Velocity of top layer of oil in contact with the block 7. Density of mixture is r = _______
​    ​ 
volume
is v = 2 ms–1.
Velocity of lowest layer touching the incline = 0 m+m 2 × 0.8
\ r = ​ ________
m  
m  ​ = ​ ______
   = 0.89 gcm–3
 ​ 
___ ___
​    ​ + ​    ​  1.8
2 ms–1
\  Velocity gradient = ______
​  –3   = 2000 s–1
 ​  0.8 1.0
10  m
2 S hrrg
For constant velocity h = ____
​     ​  fi  S = _____
​   ​ 

rrg 2
Fv = mg sin q

fi (  )
v
h A ​ __
1
​   ​  ​ = 50× 10–3 × 10 × __
d
​   ​ 
2 \
( cm
g
(5 cm) (0.1 cm) ​ 0.89 ____
S = ​  ______________________________
      ​
) (   
cm
​   3 ​  ​ ​ 980 ___
S )
​  2 ​  ​

2
fi h ◊ (20 × 10–4) (2000) = 0.25
= 218 dyne cm–1
fi h = 0.06 Nsm–2 = 0.6 poise 2 S
8. P – ___
​  r   ​  = P0
21. At terminal speed, acceleration = 0  fi  Fv = mg
2 S
22. v0 µ r2 fi P – P0 = ___
​  r    ​

If volume is made 8 times, radius doubles 2 × 0.07

)
= ​ __________
  
   ​ = 500 Nm–2
( 
–3
4 0.28 × 10
​   v = __
​   ​  p r3  ​
3
9. Work done = increase in surface energy
\  Terminal speed becomes 4 times.
or, W = (increase in surface area) S
or, 2 × 10–4 = 2 × [0.1 × 0.06 – 0.08 × 0.0375] × S
fi S = 3.3 × 10–2 Nm–1.
10. Vertical rise still remains 50 cm.
__
\  Length of liquid column in tube = 50 ÷     cm
​ 2 ​
11. As the ring tries to move up the water surface,
two surfaces are formed – one touching the inner
circumference and the other touching the outer
circumference. Extra weight is needed to overcome
the surface tension force on the ring.

\  S (2p r + 2p R) = mg.


Solutions  S.111

mg 7.48 × 10–3 × 9.8 \  F µ v


S = ________
​      ​ = ​  ________________________
        ​
2p (r + R) 2 × 3.14 [8 × 10–2 + 9 × 10–2] \  Force is tripled if v is tripled.
17.
Density of metal >> density of air.
= 0.068 Nm–1 \  Let us neglect buoyancy. [Even if you consider
mg buoyancy, the result will remain unchanged]
12. 2 l S = mg  fi  l = ___
​    ​ Net force on the sphere is zero when it travels at
2 S
terminal speed (v0).
v 0.07
13. Velocity gradient = __
​   ​ = _____
​    = 70 s–1
 ​  Air friction is f µ p r2 ◊ v2
d 0.001
\ f = k r2 v2
At terminal speed, f = mg
4
fi k r2 v02 = ​ __ ​  p r3 ◊ d ◊ g
3
v
F = hA ​ __ (  )
​   ​  ​ = 1 × (100 × 10–4) × 70 = 0.7 N
d
fi v0 µ r1/2

18. Let radius of new drop be R and that of original drops


14. Velocity first increases and then becomes constant. be r each.
Therefore, slope of s-t graph increases and then
becomes constant. 4 4
​ __ ​  p R3 = 2 ◊ ​ __ ​  p r3  fi  R = (2)1/3 ◊ r
Option (D) appear to be the best choice. 3 3
15. Pressure of air in the tube is higher than atmospheric Terminal speed v µ (radius)2
pressure by v0 ___ R2
D P = rgh \ ​ __
v  
 ​ = ​  2
 ​ = (2)2/3 = (4)1/3
4 s r
Excess pressure inside bubble is D P = ___ ​  r   ​ 
\ v0 = (4)1/3 v
The pressure of air inside the bubble and the entire
19. Surface tension does not change
tube is same.
4 s rghr 20. Gauge pressures at B and A are
\ ​ ___ r   ​ =
  rgh  fi  s = _____
​   ​ 

4 2S
PB = ___
​    ​ 
16. Let gap between the shaft and the sleeve be d. 3R
2S 2S ___ 2S 8S
and PA = PB + ___
​   ​ = ___
​    ​ + ​   ​ = ___
​     ​
R 3R R 3R
PA 4
\ ​ ___  ​ = __
​   ​ 
PB 1

h
21. PA = P0 – rg ​ __ ​ 
2
h
\ PA – P0 = – rg ​ __ ​ 
2

2 S cos q
Viscous force = – rg ​ _______ ​ 
2 (Rrg)
v
FV = hA ​ __ ​
d S cos q
= – ​ ______
   


Where A = area of shaft in contact with fluid. R
Negative sign indicates that pressure at A is less than
To maintain constant speed, external force must be atmospheric pressure.
equal to Fv in magnitude.
S.112  Mechanics II

22. P2 = P1
Worksheet 2
2S 2S
P0 – ___
​  r   ​ + rgh = P0 – ___
​   ​  1. When air bubble rises, the surrounding hydrostatic
R pressure decreases. The bubble expands. Excess
___
rg [ 
2 S __
1 __ 1
]
fi h = ​    ​ ​ ​ r ​  – ​   ​   ​
R
pressure inside it decreases with increasing radius.
Therefore, pressure inside the bubble will decrease.

[ 
2. Speed increases and becomes constant after some
= ​ 
× 7.5 × 10–2 _______
2____________
103 × 10
      ​ ​ ​ 
1
     ​ – ​ 
1 × 10–3 1.5 × 10–3
1
_________
   
]
 ​  ​ time.
\  KE increases and attains a constant value.
If v0 is terminal speed, then
= 5 × 10–3 m = 5 mm
mg FB
bv0 + FB = mg  fi  v0 = ___ ​     ​ – ___
​   ​ 
23. Initially, the meniscus if flat, i.e., radius of curvature b b
is •. As the tube is pulled out, R decreases
\  Terminal speed depends on m as well as b.
(i.e., surface gets curved). After the tube length
becomes sufficient, the radius of curvature becomes 3. When the spheres are moving at ter-
R = r cos 0° = r and remains constant. minal speed (v0), net force on the
4 S
___ system = 0
24. Excess pressure in bubble, D P1 = ​   ​ 
R
fi m1g + m2g = F1 + F2
Let the radius of drop formed be r. Conserving the
volume of liquid, we Where F1 and F2 are viscous forces
get 4 4
\ ​ __ ​  p r13 r g + __ ​   ​  p r23 rg =
4 3 3
​ __ ​  p r3 = 4p R2 ◊ d
3 6p h r1 v0 + 6p h r2 v0
fi r = (3R2d)1/3
2 rg
fi    v0 = __ ​   ​ ​  ___ ​  [r12 – r1r2 + r22]
Excess pressure 9 h
inside the drop is
[ 
​ \ r13 + r23 = (r1 + r2) (​r2​1​​  – r1 r2 + ​r2​2​)​    ​ ]
2 S ________ 2S Considering the motion of lower ball:
D P2 = ​ ___
r   ​  = ​ (3R2d) 
1/3
 ​ 
T + F2 = m2 g  fi  T = m2g – F2
D P2 ________
\ ​ ____ ​ =
2S
  ​  2  
DP1 (3R d) 1/3
 ​ 
4S
R
× ___
R 1/3
​    ​ = ​​ ____
​      
24 d
​  ​​ ​(  ) fi
4
T = __
​   ​  p r23 rg – 6p h r2v0
3
2 rg
25. ma = mg – FB [as viscous force is zero initially] ...(i)
= __
4
[ 
​    ​ p r23 rg – 6 p h r2 ​ __ ​ ​  ___ ​  ​ ​r2​1​​  – r1 r2 + ​r2​2​  ​
3 9 h
]
Maximum velocity is terminal velocity (v0). 4
​   ​  p rg ​[ r23 – r2 r12 + r1r22 – r23 ]​
= __
6p h r v0 + FB = mg 3
4
fi 6p h r v0 = mg – FB = ma [using (i)] = __
​   ​  p rg r1 r2 [r2 – r1]
3
Sphere of large radius will be at lower position. If
2
when velocity is __
​   ​  v0 r1> r2, then
3
4
T = __
​   ​  p rg r1r2 [r1 – r2]
( 


2
ma1 = (mg – FB) – ​ 6p h r __ )
​   ​  v0  ​
3
3
4. If pressure of air in the tube is P0 then
2
= ma – __
​    ​ ma P0 – ___
2 S 2 S
​ r  ​ + rgh = P0 – ___
​ r  ​ 
3 1 2

\
2
a1 = a – __
1
​   ​  a = __
3
​   ​  a
3

2 S 1
h = ___
rg r1 [ 
​   ​ ​ __
​    ​  – __1
]
​ r   ​   ​    0.028 m
2

Note that h does not depend on P0.


Solutions  S.113

5. Terminal speed is given by


Worksheet 3
F F
6p ha v0 =F  fi  v0 = ​ _____ = k ___
   ​  ​     
​.
6p h a h a 1. Surface tension of water inside the
loop decrease due to adding of
Even if the force change direction, ultimately the detergent.
sphere will move in the direction of force with speed
Force due to pure water on an
v0.
element of thread (DF1) is larger
6. Terminal speed in water is given by than force on the element due to
2 detergent mixed water. Therefore, thread becomes
2 rg
v0 = ​ __ ​  ___
​     ​ (d – r) circular.
9 h
2. Same as in Q.1
2 × (10–3)2 × 10  
= ​ ______________    ​ [1 × 104 – 103] = 20 ms–1
   3. FB = W + Fst
9 × 10–3
The sphere speed v0 = 20 ms–1 on reaching the sur- (a2 ◊ h) rg = mg + 4a ◊ S
face of water.
mg 4S
____ fi h = _____  ​ + ____
​  2    ​    ​ 
\   ​  = 20  fi  h = 20 m
​÷2gh  a  rg arg
If h is doubled v0 = 10 ms–1 4. The liquid forms two surfaces as the ring is lifted.
____ One surface touches its outer edge and the other
\   1 ​  = 10  fi  h1 = 5 m
​÷2gh surface touches its inner circumference. In extreme
case, when the ring is just about to leave the liquid
g surface, the tangential force of surface tension is
If g is made __
​   ​  vertical.
6
20 10 \ F = mg + (2p r ◊ S) × 2
v0 = ___
​   ​ = ___
​   ​ ms–1
6 3
5. Radius of meniscus in two arms are

÷ 
_______
10 10 10 r1 r2
\ ​ 2 .​ ___ ​ . h2 ​  
= ___
​   ​   fi  h2 = ___
​   ​ m R1 = ______
​  = 2 r1 and R2 = ______
   ​  ​     ​ 
= 2r2
6 3 3 cos 60° cos 60°
Pressure at the bottom of the tube can be written as:
2 S 2 S
P0 – ___
​   ​ + rgh1 = P0 – ___
​   ​ + rgh2
R1 R2

fi rg (h2 – h1) = S ​ __( 


1
​ r   ​  –
2
__1
)
​ r   ​   ​
1

S (r1 – r2)
fi h2 – h1 = ​ ________ ​ 

rg r1 r2

6. Volume of submerged part of plate is

0.77
V = 20 × ____  × 0.2 = 1.54 cm3
​   ​ 
2
Buoyancy force, FB = Vrg
= (1.54 cm3) (1 g/cm3) (980 c ms–2) = 1509 dyne
Water touches the plate along a rectangular periphery
having total length L = 2 (20 + 0.2) = 40.4 cm
\ Downward pull due to surface tension is

Fst = 73 × 40.4 = 2949 dyne


Apparent weight = (8.2) g + Fst – FB
S.114  Mechanics II

6
= 8.2 × 980 + 2949 – 1509 = 9476 dyne fi 600 = 103 × 10 × h fi h = ____ ​     ​  m = 6 cm.
100
9476 10. The common wall will be flat as the pressure on
= _____
​   ​ gm wt = 9.67 gm wt.
980 boths sides is same.
11. Contact angle is 90°. This means that the tube wall
7. P1 = P2 is conical with its apex at the centre of curvature of
2 S 2 S the spherical meniscus.
P0 – ​ ___ ___
r1 ​ + rgh = P0 – ​ r2 ​ 

r
From figure: __
​    ​ = sin a
R


2S 1 __
h = ___
​    ​ ​ __
rg r1 r2 (  1
​    ​  – ​    ​   ​ ) fi R = _____
r
​     ​ 
sin a

( 
2 S
P1 = P0 – ___
2 × 0.07 ________
= ​ ________
3

  ​ ​ ​ 
10 × 9.8 1.5 × 10
1
  –3 ​  
1
– ​ _______
   ​  
3 × 10–3

) \


​   ​ 
R
2 S
P2 = P1 + rgh = P0 – ___
​   ​ + rgh
R
= 4.73 × 10–3 m = 4.73 mm
But P2 = P0
8. r = radius of each small drop
2 S 2S sin a
1 \ h = ​ ____   
​ = _______
​      

4 4 – ​ __  ​ Rrg rrg
n ◊ ​ __ ​  p r3 = ​ __ ​  p R3  fi  r = n​ ​ 3  ​◊ R
3 3
2S
\ increase in surface energy is given by 12. Capillary rise = ____
​    ​ = h
Rrg
g
D U = [n ◊ 4p r2 – 4p R2] S (a) Rise will become 2h if geff = __ ​   ​ 
2
[ 
= 4p ​ n ◊ ​n​ 3 ​ R2 – R2  ​ S
2
– ​ __  ​
] The elevator must move down with acceleration
g
a = ​ __ ​  (Ø)
2
h
= 4p R2 ​ ​n​3 ​ – 1  ​ S [  1
​ __ ​ 
] (b) For rise to be __
​   ​ , geff = 2g
2
The elevator must move with an acceleration g(≠)
9. Refer to example 15
(c) In this case, geff = 0
In extreme case, the lower surface will become
\  Capillary rise Æ •
hemispherical and its radius will be r = 0.5 mm.
Water will rise to the top and its surface will become
Pressure just inside the lower surface will be
flat (R = •).
2 S 13. Here, buoyancy is larger than weight. The air bubble
P2 = P0 + ___
​  r   ​  = P0 + 300 Nm–2
goes up. Since density of air (in bubble) is very small
Pressure below upper surface is compared to density of liquid, the weight of bubble
can be neglected. Terminal velocity is attained when
2 S buoyancy gets balanced by viscous force.
P1 – ___   P0 – 300 Nm–2
​  r   ​ =
4
6p h rv0 = __
​   ​  p r3 r ◊ g
But P2 – P1 = rgh 3
Solutions  S.115

2r2rg __ –3 2 3 0.085
fi v0 = _____
​     
2 (0.4 × 10 ) × (0.9 × 10 ) × 9.8
​ = ​   ​  × ​  _________________________
       ​    fi h × 0.1 × _______
​    
 ​ = 0.01 × 9.8
9h 9 0.15 3 × 10–3
fi h = 0.034 Nsm–2
= 2.1 × 10–3 ms–1 = 0.21 c ms–1
16. A particle will be seen suspended if it has not
14. (a) Fv = 6p h r v = (6 × 3.14) × (0.8) × (1 × 10–3) travelled 2 cm in 10 minute, Speed for such particles
× (0.5 × 10–2) must be
2 cm 2 × 10–2 __ 1
= 7.5 × 10–5 N v £ ​ _________
    ​ = ​ _______ = ​   ​  × 10–4 ms–1
 ​ 

10 minute 600 3
4
(b) Buoyancy; FB = ​ __ ​  p r3 r g since, particle quickly attains terminal speed, we can
3
assume them to travel at constant terminal speed.
4
= __
​   ​  × 3.14 × (1 × 10–3)3 1
\ Particle having terminal speed £ ​ __ ​  × 10–4 ms–1 will
3 3
× 1260 × 9.8 remain suspended.
1
= 5.2 × 10–5 N v0 £ __
​   ​  × 10–4
3
At terminal speed, 2 g 1
​ __ ​  r2 ​ __ ​  (d – r) £ __
​   ​  × 10–4
FV + FB = mg 9 h 3
fi 6 p h rv0 + 5.2 × 10–5 = 50 × 10–6 × 9.8 1 9 × 10–3
fi r2 £ __
​   ​  × 10–4 × ​ ___________________
   
    ​
3 2 × 9.8 × (1.8 – 1) × 103
fi 6 × 3.14 × 0.8 × 10–3 × v0 = 43.8 × 10–5
fi v0 = 0.029 ms–1 = 2.9 cms–1 = 0.191 × 10–10
15. When speed is constant (i.e., a = 0), we must have
fi r £ 0.44 × 10–5 m
mg = T  and  T = Fv
v
\ FV = mg  fi  h. A ​ __ ​ = mg \ rmax = 4.4 m m.
d
Dmax = 2 × 4.4 = 8.8 m m
Chapter 10 Miscellaneous Problems on Chapters 8 and 9
5. Effective acceleration of free fall in an RF attached
  match the column _________ __
​ g  2 + (2g)2 ​ 
to the container is ÷ =÷    g.
​ 5 ​ 
1. (A) When A and C are opened, water flows out 1
geff makes angle q with vertical, where tan q = __
​   ​ 
through C and pressure above water surface remains 2
atmospheric pressure
(C) Water flows out initially and air in the bottle
expands. Pressure of air drops. When pressure just
inside the hole at C drops to atmospheric pressure
the flows stops.
2. (A) FA = (r gh) ◊ p R2 = Force on top surface
(B) FB = [r g (2h) + 2r gh] p R2 = 4p r gR2h (A) Water surface is perpendicular to geff.
= Force on bottom surface
(C) Force by A + Force by B
1
\  a = q = tan–1 ​ __
​   ​   ​
2 (  )
(B) Effective buoyancy is in a direction opposite to
= (Pav​)1​ ​ (h ◊ 2R) + (Pav​)2​ ​ (h ◊ 2R) geff. String will be along geff.

​ 
2 ( 
r gh + r g ◊ 2h
= ​ ____________
 ​ 
     )
​ (h ◊ 2R)
\ 

b = 90° – q
tan b = cot q = 2

( 
r g2h + (r g ◊ 2h + 2r ◊ g ◊ h)
+ ​ ​ ______________________
   
2
 ​     ​ (h ◊ 2R) )
= 9r ◊ g Rh2

(D) FB – FA = 3p  r gR2h

3. (A) In bucket B, the volume of water displaced


(= extra water that is there is bucket A) is equal
to weight of the block. (C) Points A and B lie on a equi-pressure line. This
line is parallel to liquid surface.
(B) Clearly, the block is an additional weight in B.
2
(C) Weight of the object is more than the water tan a = __
​ x ​
displaced.
1 2
4. Pressure varies with depth as: P = rgh fi ​ __ ​  = __
​ x ​  fi  x = 4 cm
2
Pressure increases in horizontal direction opposite to
acceleration, as P = ra
At the right extreme top, the pressure is zero, but it
increases as one moves to the left.
Average pressure on ABFE is ​ ​ _______ [ 
0 + ra
 ​   ]
 ​. Force on
this surface is
2

1
(D) As explained above, f = a = tan–1 ​ __
​   ​   ​
2 (  )
[  ]
0 + ra 2 (r 3) ____
r 2 mg
​ ​ _______ ​ [ ] = ____
 ​     ) = _____
​   ​ (
  ​   ​ a
   = ___
​   ​  6. Speed of efflux v = ​÷ 2gh   ​ it is not dependent on
2 2 2 2 density but depends on g. Range of liquid jet will

[  ]
0 + ra change if container is accelerated since time of flight
Average pressure on BDHF is ​ ​ _______
 ​  
 ​. Force on will depend on g but v will depend on geff
2
this surface is Time needed to empty the tank is given by

÷ 
___

[ 
0 + rg 2
​ ​ _______
2 ] rg
 ​ [ ] = ___
 ​  
r ()3
​   ​  (3) = _____
2 2
mg
  = ___
​   ​ (g)
  ​   ​ 
2

A 2H
t = ___
​     ​ ​ ___
Ao g
​     ​ ​ 

It depends on Ao. It does not depend on density but


Similarly, for the other part. will change if geff changes.
Solutions  S.117

d Bubble detaches when force due to striking air


8. (A) Volume of displaced liquid Vsub = V ◊ ​ ___  ​ 
rw molecules exceed this surface tension force.
Mass water formed due to melting of ice = mass
b2
of ice = Vd. fi  p b2 r v2 ≥ 2p S ◊ ​ __ ​ 
R
\  Volume of water formed due to melting ice 2S
____
d fi  R ≥ ​  2  ​ 
is = V ___
​    ​  r v
rw
\  Volume of water formed fits exactly in the Passage 3
space occupied by ice block inside water. Sol. Reading of balance immediately after the sugar lump
is put into it is R0 = Mg + FB
(B) It can be argued as above.
[Water applies buoyancy on sugar lump. The lump
(C) If r > rw then volume of submerged part will
pushes the water with the same force in downward
be less compared to Vsub in water. Melting of
direction]
ice will produce larger volume of water than the
m
volume occupied by ice block in liquid. Level \ R0 = Mg + Vsugar ◊ r g = Mg + __
​   ​  r g ...(i)
d
will rise. At time t1, the reading is
(D) It can be argued as above.
R1 = ​ M + __ (  m
)
​   ​   ​ g + Vsugar r g
2
  Passage-based Problems = ​ M + __ (  m
)
​   ​   ​ g + ___
2
m
​    ​ r g
2d
Passage 1 mg 1 __ m
= Mg + ___ ​   ​ + __ ​   ​  ​   ​  r g ...(ii)
2. When air is expelled out of the large tube, pressure 2 2 d
inside it drops. This is the reason why the liquid rises
in the tube. The hydrometer floats in this sample of = Mg + ____
mg r __
​     
d [  1 d __
]
​ ​ ​   ​  __
2 r 2
1
​    ​ + ​    ​  ​ ...(iii)
liquid d
Since ​ __  ​ > 1, therefore quantity inside the bracket​
3. For rotational equilibrium it is necessary that the r

Passage 2
COM lies below the centre of buoyancy. 1 d
[  1
]
= ​ __  ​ ​ __  ​  + ​ __  ​  ​ in the above equation is greater
2 r 2
than 1.
4. Volume of air striking the bubble wall per second Thus, comparing (i) and (ii) gives: R1 > R0.
is = vA
At time t2, reading is
Mass of air striking the wall per second = vAr
Rate of change of momentum of air particles per R2 = (M + m) g
second = (vAr) v = Arv2 mg mg
= Mg + ___ ​   ​ + ___
​   ​   ...(iv)
\  Force = Arv 2 2
[   A = p b ] 2 2
Comparing (ii) and (iv) gives R2 > R 1​
= p b2 r v2
5. Surface tension force on bubble is acting tangentially
over the circumference of length 2p b.
[ 
r
Since __
​   ​ < 1  ​
d ]
\ R2 > R1 > R0

The reading of the balance must have increased


continuously.

Passage 4
9. For stable equilibrium, centre of mass must be below
the centre of buoyancy (C).
C is at mid-point of submerged part and centre of
Resultant surface tension force is towards right in the mass (G) is below it.
diagram. When the wood piece is horizontal and complelety
submerged, centre of buoyancy shifts to geometrical
b b2
Fst = S ◊ 2p b . sin q = S ◊ 2p b ◊ ​ __ ​  = 2p S ​ __ ​  centre of the wood piece.
R R
S.118  Mechanics II

Passage 5
12. Initially, weight of the box is balanced by buoyancy.
When water level inside the box becomes x, the
box must sink further by x to balance the additional
weight of water inside it.

The difference in height (y) between the level of water


outside and inside the tank remains unchanged.
x + y + (h – x) = H  fi  y = H – h
13. Consider two points 1 and 2, which are just above
For translational and rotational equilibrium, the point
and below the hole respectively. Using Bernoulli’s
of action of weight (W) of the wood piece must be
equation:
to the left of FB and F0, as shown. Hence, (c) is 1
correct. P1 + ​ __ ​  r v2 = P2 + 0
2
1
10. FB = F0 + W ...(i) fi  P0 + r g x + __ ​   ​  r v2 = P0 + r g (x + y)
2
Balancing torque about G gives
fi v2 = 2gy = 2g (H – h)

(  )
_________
L
F0 ​ x + __
​   ​   ​ = FB ◊ x \ v = ÷  
​ 2g (H – h) ​

2
This speed is independent of x.
L _________
fi  F0 x + F0 __
​   ​   = AL r0 g ◊ x 14. Volume flow rate = vA = A ​÷  2g (H –   h) ​ = a
2
constant
F0L
fi  (ALr0 g – F0) x = ​ ____  ​ 
  The box will sink when x = h
2
F0L Volume of water inside the box = abh
fi x = ​ ____________     ​ abh
2 (ALr0 g – F0) \  Time needed, t = ___________
​  _________
    ​


A ​÷2g (H – h) ​

Passage 6
To know the force applied by water on the slant
face of the wooden wedge, consider a volume ABC
of water inside a water tank. Force applied by sur-
rounding liquid on face AC will be same as force by
water on the given wedge.

11. In horizontal position: W = FB – F0 = ALr0 g – F0


In vertical position: Buoyancy = W
Horizontal force on AC = Force on AB by surrounding
fi Alr0 g = ALr0 g – F0
liquid.
F0 Fx = F
l = L – _____
( 
fi ​    ​ 
Ar0 g

0 + r gh
Fx = (P av) (hb) = ​ ​ _______
2 ) 1
 ​ hb = __
 ​   ​    ​ r gbh2
2
Solutions  S.119

÷ 
______

÷ 
_____
Vertical force 2 (AE) 2×1
\  Range AB = v ​ ______
​  g   ​ ​ 
  = 4.96 ​ ​ _____
 ​ ​

  
Fy = weight of water in volume ABC 10
1 1 = 2.22 m
= ​ __ ​  b (h ◊ h tan q) r g = __​    ​ r g b h2 tan q
2 2 \  OB = 2 cos 30° + 2.22 = 3.95 m
1
15. N = Fx = __ ​   ​  r gbh2 a 2 1
2 20. tan a = __
​ g ​ = ___
​    ​ = __
​   ​ 
1 10 5
16. Fy = __
​   ​  r gbh2 tan q
2
Passage 8
17. Friction on left wall, f = mN (Ø)
Sol. Object is floating while remaining completely
In critical case, when the wedge is about to slide up: submerged. Its density is equal to that of the water.
f + mg = Fy Force on slant surface = Buoyancy [As told in the
question]
1 1
m ​ __ ​  r gbh2 + mg = ​ __ ​  r gbh2 tan q fi  Downward force on top surface due to
2 2

÷ 
____________ atmospheric pressure = upward force on bottom
2m
fi h = ​ ___________
​       ​ ​ surface due to water.
r b (tan q – m)
fi  P0 ◊ p r22 = (P0 + r gh) p r12
Passage 7 fi  105 r22 = (105 + 103 × 10 × 30) r12
18. Water surface has to remain horizontal, as accelera-
tion due to gravity is downwards. fi  r22 = 4r21  fi  r2 = 2r1 ...(i)
19. Volume of water in the tank is 8 m3.
Let DE = x
2
Then OC = x + 2 tan 30° = x + ___ ​  __  ​ 
÷   
​ 3 ​

r1 r2
From geometry: tan 53° = __ ​ x ​  = ______
​       ​
x + 30
r1 __ 4 3
fi  ​ __ __
x ​   = ​ 3 ​   fi  x = ​ 4 ​  r1
Volume of water can be expressed in terms of x as: r1 ______ r2
And ​ __ x ​   = ​ x +     ​
V = 2 (area of trapezium OCDE) 30


1
(  2
= 2 ◊ ​ __ ​  ◊ 2 ​ x + x + ___ ) (  2
​  __  ​  ​ = 2 ​ 2x + ___ )
​  __  ​  ​
fi r1x + 30r1 = r2x

(  )
2 ÷   
​ 3 ​ ÷   
​ 3 ​ 3 3 3
fi  ​ __ ​  r12 + 30r1 = (2r1) ​ __
​   ​  r1   ​  fi ​ __ ​  r1 = 30

( 
4 4 4
÷
2__
\ 2 ​ 2x + ​ ___  ​  ​ = 8
  
​ 3 ​ ) fi  r1 = 40 m
1__
fi x + ​ ___   2  fi  x = 1.42 m
  ​ =
÷   
​ 3 ​
Vertical height difference between D and E is
__
÷   
​ 3 ​
h = x cos 30° = 1.42 × ___
​   ​ = 1.23 m
2
____ ____________
\  Speed of efflux: v = ÷   ​ = ​÷2  ×  10 × 1.23 ​
​ 2gh    
If the body is inverted, the force on two flat surfaces
= 4.96 ms–1 are no longer equal. Force on slant surface will
increase in magnitude. This can be understood simply
AE = 2 sin 30° = 1 m
by observing that pressure increase with depth and
S.120  Mechanics II

the inverted object has more surface area exposed to \  Volume flow rate through entire cross-section
higher pressure. is

(  ) [  ]
r0
x2 r20 __ r02
Passage 9 Q0 = 2p v0 ​Ú  ​ ​   ​​ x – ​ __
r0  ​ 
  ​ 
d x = 2p v0  ​ __
​   ​
  – ​   ​   ​
0 2 3
23. Just outside the mouth of the pitot tube, at A, the
gas has speed and just inside, there is no speed. 1
= __ ​   ​  p v0 r20
Therefore, pressure at a point inside the tube (at A) 3
is higher than outside pressure. This is the reason the D P
26. Q depends on ___ ​  , h and r.
​     
liquid is pushed down in the right arm. l

(  )
24. Apply Bernoulli’s equation between points just out- D P a b c
side the tube and just inside it (at A). Let Q = k ​​ ___ ​   ​​ ​ h  r ;   k is a constant
​     
l
Taking dimension;

[  ]
a
M L–1 T –2
[L3 T –1] = ​​ ________
​      ​  ​​ ​ [ML –1 T –1]b ◊ Lc

L
[L3 T –1] = [M a + b L– 2a + c – b T – 2a – b]
1
P1 + __
​   ​  r v2 = P2 + 0
2 Equating the dimension of M, L and T, we get
1
fi  ​ __ ​  r v2 = P2 – P1 0 = a + b ; 3 = – 2a – b + c ; – 1 = – 2a – b
2

÷ 
______
2r0 On solving, a = 1, b = – 1 and c = 4
1
fi  ​ __ ​  r v2 = r0 gh  fi  v = ​ ___
​   ​ 
 gh 
   ​
2 r D P r4
\ Q = k _____
​   ​    [In fact, this equation is known
\  Volume flow rate is lh
as Poiseuille’s equation]

÷ 
______
2r0 gh Q0l0h0
Q = S v = S ​ ______
​   ​ ​ 

  \ DP = ______
​  4 ​   
r k r0
Passage 10 Passage 11
25. Cross-section of the tube has been shown. Speed of Sol. Let angular speed be w after time t.
flow along the axis is v0 and speed of fluid at tube Speed of fluid layer in contact with inner cylinder is
wall is zero. v = wr.

Speed increases linearly from 0 to v0 over a distance


Speed of layer in contact with outer cylinder = 0.
r0.
v0 wr – 0 wr
\  Velocity gradient = __​ r  ​  \  Velocity gradient is = ​ _____ 
 ​ = ​ _____
   ​
0 R–r R–r
Speed at a distance x from the axis is Viscous force on a strip of width D x on the surface
of inner cylinder is
( 
v0
u = ​ __ __x
r0 ​  (r0 – x) = v0 ​ 1 – ​ r0  ​   ​ ) (  ) wr
D fv = h D A ​ ____
​    
R–r
 ​  ​
\  Volume flow rate through shaded portion of the
= hL D x ​( ____
wr
R – r)
cross-section is
​     ​  ​
dQ0 = dA ◊ u = (2p xdx) v0 ​ 1 – __
x
( 
​ r   ​   ​
0 ) Torque due to this force (about rotation axis) is
Solutions  S.121

​ 
R–r
wr2
Dt = D fv ◊ r = h L ​ _____  ​  ​ D x
   (  ) Passage 12
29. Viscosity depends on temperature. We must know the
Total torque due to force on all such element is temperature at which we are measuring it.

(  ) (  )
2 2
When the ball is not able to attain terminal speed
wr wr
t = Â D t = h L ​ _____
​   ​  ​ Â D x = h L ​ _____
   ​   ​  ​ 2p r
   before reaching P, t1 > t2 [Ball is still accelerated].
R–r R–r We want to ensure that t1 = t2. For this, the ball should
3
wr
= 2p h L ______
​    
 ​ quickly attain terminal speed after it is dropped. A
(R – r) ball of smaller radius has smaller terminal speed and
= k w it is more likely to attain terminal speed before reach-
ing P. Highly viscous liquid ensures that terminal
2p h Lr3
Where, k = _______
​   ​
  speed is attained quickly.
(R – r)
d
Angular retardation of the cylinder is 30. Terminal speed, v0 = __
​    ​ 
t0
t kw 2k
a = __
​   ​ = _____
​      ​ = ____
​  2  ​  ◊ w 2 
I 1
__
​   ​  mr2 mr 2 g r0 (s – r) __ d m
2 fi ​ __ ​  ​ _________
 ​     = ​    ​   [s = _____
​     ​  = density of
9 h t0 4
__ 3
​    p
 ​   r ball]
dw 2k 3 0
\ ​ ___ ​ =
  – ​ ____2  ​  ◊ w 2
dt 2 t0gr0
mr fi h = __​   ​  _____
​     ​ (s – r)
9 d

[ 
w t
dw 2k
​Ú  ​ ​ ___
​   ​  = – ​ ____2  ​ ​Ú ​  ​ dt 2

]
fi 2 t0gr0 _____ m
w0 w mr 0 = __
​   ​  _____
​     ​ ​ ​       ​ – r  ​
9 d 4
__
2k ​   ​  p r03
2k w – ​ ____  
 ​◊ t 3
fi ln w – ln w0 = – ​ ____2  ​  ◊ t  fi ​ ___  ​ = ​e​ mr2  ​
mr w0
2k
– ​ ____  ​  ◊ t
fi  w = w0 ​e​ mr2 ​
Chapter 11 Gravitation

Your Turn
1
1. F µ __
​  2  ​ 
r
r
If you put __ ​    ​  in place of r, value of F will become
2
four times.
G Msun Mearth
4. F = ​ ___________
 ​
    
r2
We have studied in the chapter of vector that
6.67 × 10–11  × 2 × 1030 × 6 × 1024
= _______________________________
​          
 ​ resultant of five such symmetrical forces is
(1.5 ×1011)2 zero.
​___›
= 3.56 × 1022 N If particle at A is removed, the force along OA​
​   
gets removed.
GMm
5(a). F = _____
​  2 ​ 
  Resultant _​ __› of other four forces must be F
x _______ along AO​
​    .
__
Resultant, F0 = ÷   2 + F2 
​F  ​ = ÷    F
​ 2 ​ 7. A common mistake is to replace the rod with a
__
÷     GMm
​ 2 ​ point mass at its centre.
= ​ ________ ​ 

x2

There is no such principle in physics. Only


spherical masses behave like point masses
placed at their centres.
Consider an element of length dx as shown.
M
Gm2 Mass of this element is dm = ​ __ ​  dx
(b) F = ____
​  2 ​ 
  L
a Force applied by this element on m is
Resultant is along y-direction as x component cancel
G (dm) m GMm ___ dx
out.
__ dF = ________
​   ​  = _____
​     ​ ​  2 ​ 
x2 L x
Gm2 ___ ÷   
​ 3 ​
F0 = 2F cos 30° = 2 ◊ ​ ____  ​ ◊ ​ 

   ​ 
a 2 2 2L
__ dx
÷    Gm
​ 3 ​  2 \ F = _____​ Ú​  ​  ​ ​ ___
​      ​   ​ 
= _______
​  2 ​    L L x2
a
[Note: at A x = L, at B x = 2L]

GMm __
= – ​ _____
L

   
1 2L
[  ] GMm ___
​ ​​ ​ x ​  ​​L​  ​ = – ​ _____
L
1
[  1
​ ​ ​    ​ – __
   
2L L ] GMm
​   ​  ​ = _____
​  2 ​ 
2L

8. Field due to the mass at B and C cancel out. Field


at D is only because of mass at A.
__
÷   
​ 3 ​
AD = ___
​   ​ x
6. (i) The five force acting on the particle at O are in 2
direction shown. Each has a magnitude
GM 4 GM
\  E = _______
​  __    ​ = __
​   ​  ____
​   ​ 
 along DA

GmM
F = _____
​  2 ​ 
x

(  )
÷
​​ ___
   2
​ 3 ​
​   ​  x  ​​ ​
2
3 x2
Solutions  S.123

9. Field due to elements at diametrically opposite ends Note that time period of rotation of the Earth is the
cancels out. length of the day.
10. From symmetry, field is along y-direction. Putting R = 6.4 × 106 m and g = 9.8 ms–2 in the above
For element shown, expression gives T 84.6 minutes.

M 15. Value of acceleration due to gravity at a depth d is


dm = ​ ___  ​ (R dq)
p R
(  d
g ​ 1 – __ )
​    ​  ​ and its value at height h is ________
​ 
g
  2   ​.
M
= __
​ p ​  dq
R h
​​ 1 + __ (  )
​   ​   ​​ ​
R
Given: g  ​ 1 – __(  d
)   g
​    ​  ​ = ​ ________
  2   ​

Gdm
dE = ​ _____
R2
 ​ 

R
( R
​​ 1 + __ )
​   ​  ​​ ​
R
3
GM fi d = ​ __ ​  
= ​ ____2 
 ​ dq 4
p R
g h 5
16. (a) ________
​    2 ​  = 0.64 g  fi  1 + __
​   ​  = __
​   ​ 
Resultant field is

p /2 p /2
(  h
__
)
​​ 1 + ​   ​   ​​ ​
R
R 4

GM R
E0 = Ú dE cos q = Ú cos q dq fi  h = __​   ​  = 1600  km
– p /2 p R2 – p /2 4
(b) For small height
2GM
= _____
( 
​  2 ​ 
p R
2h
g¢  g ​ 1 – ___
​   ​   ​
R )
GM
12. E = ​ ____
R2
 ​
  =
6.67 × 10–11 × 7.36 × 1022
________________________
​     
(1.74 × 106)2
 ​
  
2h
\ g (1 – 0.0036) = g ​ 1 – ___
​   ​   ​
R (  )
2h
= 1.62 N kg–1 fi  ​ ___ ​ =
  0.0036
R
GM 6400 × 0.0036
13. On surface, Esurface = ____​  2 ​  fi  h = ​ _____________
 ​
     = 11.5 km
R 2
At distance r from centre (inside Earth) g
17. At height h above a pole, g¢ = ​ ________
  2   ​

GM
E = ____
​  3 ​  ◊ r
R
h
​​ 1 + __
​   ​   ​​ ​
R (  )
1 Effective acceleration of free fall at equator is
E = __
​   ​  Esurface
2
geq = g – w2R
GM 1 GM R
\ ​ ____ ​  ◊ r = __
​   ​  ____
​   ​   fi  = __
​   ​  As per the question
R3 2 R2 2
g
​ ________
  2   ​ = g – w2R
(  )
Depth of such a point from the surface of Earth is
h
R ​​ 1 + __
​   ​   ​​ ​
h = R – r = __
​   ​  R
2
g ​[ 1 – ___
​   ​  ]​ = g – w R
14. Man on the equator will feel no weight if the effec- 2h 2

tive free fall acceleration (g¢) becomes zero, putting R
g¢ = 0 in equation (8) gives
[Since effect of rotation is small at equator, the value

÷  ÷ 
__ __
g 2p g of h must also be small]
w2R = g  fi  w = ​ __
​   ​ ​    fi ​ ___ ​ = ​ __
​   ​ ​  
R T R Substituting data in above expression gives
h  11 km.
÷ 
__
R
T = 2p ​ __
​ g ​ ​   Note: Just by going through example 8 and question
16 one can guess the answer.
S.124  Mechanics II

____
4
r ◊ ​ __ ​  p R3 23.    
v = ​÷2gR 

M
___ 3
_______ 4
 = __
​   ​  p G r ◊ R

÷ 
18. g = G ​  2  ​  = G ◊ ​   ​  __________
R R2 3 vA 2s R 1
\  ​ __ __________   ​ ​ = __
vB  ​ = ​ ​ 2 (2 s) (2R)
   ​   ​ 
2
\  g µ r ◊ R
GMm
24. PE on surface = – ​ _____
   ​

Since r and R for planet are twice that of Earth, value R
of g on the surface is four times that on Earth. GMm
KE needed to escape = _____
​     ​

R
19. Wg = – D U = Ui – Uf
1 GMm
KE imparted = __
​   ​  _____
( 
​     ​ 

Gm ◊ 2m _______
= ​ – ​ _______
a   
Gm ◊ 3m ________
​ – ​  a   
G 2m ◊ 3m
​ – ​  __    
   a
​÷2 ​

​  ​ – ) 2 R
GMm
\  Total energy of rocket = – _____
​     ​+

1 GMm
__
​     ​​ _____
   ​ 
R 2 R
( 
Gm ◊ 2m _______
​ – ​ _______
a   
Gm ◊ 3m ________
​ – ​  a   
G 2m . 3m
​ – ​  a    ​  ​ )
1 GMm
= – ​ __ ​  _____
​      

2 R
(  ÷ )
1__ ____
= 6 ​ 1 – ​ ___
Gm2
  ​  ​ ​  a   
  
​ 2 ​
​  When at maximum distance (x) from the centre, its
energy is purely potential energy.
20. Both must have equal speed in order to conserve GMm 1 GMm
\  – _____ ​ = – __
​  x   
  ​   ​  _____
​     ​  fi x = 2R

momentum. Let their speed be r. 2 R

Ki + Ui = Kf + Uf 25. (a) Potential at the surface,


GM GM
1 1 Gm ◊ m Vsurf = – ​ ____​ = – ____
   ​  2 ​  ◊ R = – gR
0 + 0 = ​ __ ​  mv2 + __
​   ​  mv2 – ______
​  r    
​ R R
2 2
____
= – 9.8 × 6.4 × 106
fi  ÷ 
Gm
v = ​ ____
​  r   
 ​ ​  = – 6.27 × 107 J kg–1
At a distance x from the centre (outside the
21. Ki + Ui = Kf + Uf Earth)
GM
V = –  ​ ____​ 
x   
GmM __ 1 GmM
0 – ​ _____
    ​    ​ mv2 – _____
​ = ​      

2R 2 R Since ​| – 1 × 107 |​ < ​| – 6.27 × 107 |​
[Initial distance of the body from the centre of the
\ x > R
Earth is R]

÷ 
(b) U = mV = – 5 × 6.27 × 107 = – 3.14 × 108 J
÷ 
____ ______
GM GM ___
\  v = ​ ____ ​ ​  = ​ ____
​      ​  2 ​  ◊ R ​    ​ 
= ​÷gR  27. Potential due to a small mass dM on the ring is
R R
GdM
dV = – ​ _____​ 
   
22. (a) Total mechanical energy = – E0 + 2E0 = E0. R
This is positive. Hence the rock piece will All elemental masses are at a distance R from the
escape out of the gravitational pull. centre of the ring.
G GM
(b) Mechanical energy is conserved. \  V = Ú dV = – ​ __ ​  Ú dM = – ____
​     
​ 
R R
K + U = E0
28. Force on a satellite always acts towards the centre of
fi  1.6E0 + U = E0  fi  U = – 0.4 E0 the Earth. If the path has 3-3 as diameter, this force
is not towards centre of the circular path.
(c) At infinity, U = 0
GMm 1
\ K = total energy = E0 29. U = – _____ ​  fi  U µ – __
​  r   
  ​ r ​ 
Solutions  S.125

÷ 
____ ____

÷ 
\ U will have decreased magnitude with negative GM GM
33. (a) Velocity for circular orbit, v0 = ​ ____ ​ ​  = ​ ____
​  r    ​   ​ ​ 

sign if r is increased. 2R
\  U increases Velocity for escape (Refer to example 22)

÷ 
GMm ____
k = _____
​     ​  fi  k will decrease
  __ GM
2r v1 = ÷    v0 = ​ ____
​ 2 ​  ​     
​ ​  
R
T2 2
30. ​​ __
T1 (  ) (  )
r2 3
​   ​   ​​ ​ = ​​ __
​ r   ​  ​​ ​
1 2 GM
Since v = ​ __
÷ 
​   ​  ​ ____
3 R
______
​ ​  is less than v1, the satellite
  

fi  (  )
r2 3/2
T2 = ​​ __ (  )
3.5 R 3/2
​ r  ​   ​​ ​ ◊ T1 = ​​ _____
1
​  ​​ ​ ◊ (24 hr)
​     
7R
will not escape.

(b) Speed v > v0


\  T2 = 8.48 hr
\  Path is elliptical.

÷ 
____ _____
31. (a)
GM
v0 = ​ ____
​  r    ÷ 
​ 
R+h
GM
​ ​  = ​ _____  
 ​ ​
  34. Only force on planet is gravitational pull of the sun.
It has no torque about the centre of the sun. Hence,
ve
Given v0 = ​ __ ​  angular momentum is conserved.
2
Force is not perpendicular to the velocity of the planet

÷  ÷ 
_____ _____
GM 1 2GM (except at perihelion and aphelion). Thus, work done
fi ​ _____
​   ​ ​ = __
  
  ​    ​ ​ _____​ ​    fi  R + h = 2R
​     
R+h 2 R by force is not zero in a finite displacement. Thus,
KE keeps changing.
fi  h = R
35. Angular momentum conservation
(b) Energy conservation gives v1r1
mv1r1 = m (v2 sin q) r2  fi  v2 = ______
​    ​ 
1 GMm
​ __ ​  mv2 + ​ – ​ _____
2 R
    (  )
GMm
​  ​ = 0 + ​ – ​ _____
2R
  ​  ​ (  ) r1 + r2
r2 sin q

36. (a) For P2, a = ​ ______ = 1.4 × 108 km


 ​ 
GM ___ 2
fi  v 2 = ____  ​  fi  v =
​         ​
÷​ gR 

(  ) ( 
R
32. Energy given = (Energy of the satellite in the orbit)
– (Energy of the object when it was on the Earth)
T2 2
​​ __
T1
1.4 × 108 3
​    ​   ​​ ​ = ​​ ​ ________
1 × 108
 ​ 

  )
​​ ​ fi T2 = (2 yr)(1.4)3/2

3.3 years
GMm
= – _____
​ 
2 (2R)

GMm
– ​ – _____
 ​  ​     
R
​  ​ =
  (  ) 3 GMm
__
​   ​  _____
4 R
​     ​  (b) Speed of P2 is larger
(c) PE of both P1 and P2 are same at P but KE of
3 3
= __
​   ​  mgR = __
​   ​  × 2000 × 10 × 6.4 × 106 P2 is larger.
4 4
= 9.6 × 1010 J \  E2 > E1
S.126  Mechanics II

10. The whole mass of Earth can be assumed to be con-


Worksheet 1 centrated at its centre. Thus, shrinking of Earth will
not make any difference.
1. Gravitational force is conservative. Work done by it


does not depend on path.
2. Gravitational force provides the necessary centripetal
2h
11. At height h, g1 = g ​ 1 – ___
​   ​   ​
R [  ]
(  )
force 2g
3. In absence of gravity, the gas molecule will escape. Decrease in g is D g1 = ​ ​ ___ ​   ​ h [This is given as
R
1%]

[  ]
4. Due to rotation, acceleration of free fall is g¢ =
h
g – w2R. If w becomes zero g¢ increases. At depth h, g2 = g ​ 1 – __
​   ​   ​
R
5. g¢ = g – w2R at equator but there is no effect of
rotation at poles
4
g
Decrease in g is D g2 = ​ __
​    ​  ​ h
R (  )
G ◊ ​ __ ​  p R3 r
GM 3 Obviously, the answer is 0.5%
6. g = ____
​  2 ​   fi  g = ​ _________   ​   
R2
[ 
R
h
12. g ​ 1 – __ ] 1
​    ​  ​ = __
g
​   ​  __________
[ 
​       ​
fi  ​ 
3g
r = ______
4p G R
   ​ 
R 2 1600 2
​​ 1 + _____
​ 
6400 ]
 ​  ​​ ​

4 h 8 17
G ◊ ​ __ ​  p R3r 1 – _____  ___
​     ​ = ​    ​   fi  h = ___
​   ​ × 6400 km
3 6400 25 25
7. g = ​ _________  ​     fi  g µ Rr
R2 = 4352 km
gm
​ ___
ge ​ =
Rmrm
_____
​ 
Rere
 ​  
gm re
fi  Rm = Re ​ ___
e
(  ) (  )
​ g  ​   ​ ​ ___
​   ​  ​
rm
13. From example 34

g¢ = g – R w2 cos2 f = g – ____
Rw2
​   ​  
​    ​ )​ ​( __
= R  ​( __
4
6 3 ) 18
1 5 5
​   ​   ​ = ___
​    ​  R G M D g D M D R
14. g = ____
​  2 ​   fi ​ ___ ____ ​ – 2 ◊ ​ ___   
e e

R g   ​ = 1 ◊ ​  M    R
​ 
GM4GM
8. g = ____
_____
​  2 ​ =
​  2 ​  
  [D = diameter] D g D M D R
R D \ ​ ___ ____ ​ × 100 – 2 ___
g   ​ × 100 = ​  M    ​     
​ × 100

(  ) (  )
R
M gp Mp De2 Mp De 2
\  g µ ___
​  2  ​   fi ​ __
ge  ​ =
___
​  2 ​  ◊ ​ ___  ​ = ​ ___
​    ​  ​ ​​ ___
​    ​  ​​ ​ = (– 1%) – 2 (– 1%) = + 1%
D Dp Me Me Dp

÷ 
__________
4
2G __
​   ​  p R3r
÷ 
_____
1 1 GM 3
= ___
​    ​ × 22 = ___
​    ​  ____
15. v = ​ 2 ​     
​ ​   _________
= ​ ​         ​ ​ 
80 20 R R
ge 9.8
\  gP = ___
​    ​ = ___
​   ​ = 0.49 ms–2 \  v µ R
20 20
1 GMm
9. Let P be the point where force (on a body of mass M) 16. If v is escape speed, then __
​   ​  mv2 – _____
​     ​= 0

2 R
due to Earth and the moon are equal and opposite.
1 GMm
fi  ​ __ ​  mv2 = _____
​     ​
  ...(i)
G (81 m) M _______
G m ◊ M 2 R
​ _________
 ​ =
  ​ 
    
 ​
x2
(D – x)2 Let speed at • be u when the body is projected with
speed 2v.
D – x __ 1 9D
fi  ​ _____  ​   ​   fi  x = ___
​ =
x    ​   ​  1 1 GMm
9 10 ​ __ ​  mu2 = ​ __ ​  m (2v)2 – _____
​     
​ 
2 2 R
1 4 1
fi  ​ __ ​  mu2 = __
​   ​  mv2 – __
​   ​  mv2 [Using (i)]
2 2 2
__
fi     v
u = ​÷3 ​ 
GMm
17. Given, U = – mgRe  fi  – ​ _____  
​ = – mgRe
   
Re
GMm 1
\  At height h = Re, Uh = – ​ _____   
​ = – ​ __ ​  mgRe
2Re 2
Solutions  S.127

18. Switching off the gravity means that there will be no 2GMm
\  Total energy = PE = – ​ ______ 
r   ​
centripetal force on the satellite. It will move along
a straight line. 4
26. Volume of solid sphere = __
​    ​ p R3 = v
19. The satellite will just manage to escape if its speed 3

1
is increased so as to make its total energy zero.
GMm
Volume of removed part = __
4
3
R 3 V
​   ​  p ​​ __ (  )
​   ​   ​​ ​ = ​ __ ​ 
2 8
​ __ ​ mv2 – _____
​  r   
​ = 0

2 M
_____ \  Mass of removed part = __ ​   ​ 
÷ 
2GM __ 8
fi  v = ​ _____
​  r    =÷
​ ​       v0 = 1.414 v0
​ 2 ​
Force on m = Force due to solid sphere – Force due
____ to mass that was present inside the cavity region

satellite.
GM
Where v0 = ​ ​ ____
r    ÷ 
  ​ ​ is the orbital speed of the M
G ​ __ ​  m
GMm 8 23 GMm
Percentage increase in speed needed is 41.4% = _____
​  – ______
 ​  ​    ​ = ____
​    ​ ​ _____
 ​ 

(2R)2
(2.5R) 2 100 R2
r1 + r2
20. Semi-major axis of elliptical path, a = ​ ______
 ​ 
  27. Conservation of angular momentum gives
2
T 2 µ a3  fi  T µ a3/2 mv
\  ​ ___ ​  ◊ r = m (v sin 30°) R.
4
\  T µ (r1 + r2)3/2
fi  r = 2R
21. The ball is moving with the satellite. After its release,
due to its inertia, it will have the same velocity as \  Height attained, h = 2R – R = R.

(  )
the satellite. It will continue to revolve around the 1
__
GMm GM ​ 2 ​ 
planet under gravitational pull of the planet. 28. mw2 R = ​ _____ ​   fi  w = ​​ _____
n    ​  n + 1 
 ​  ​​ ​
R R 
23. 1 lunar month = time period of revolution of the
moon. 1
\  w µ ______
​     ​  fi  µ ​R​ (  )
n + 1
​ ____
​   ​  
2 ​
 ​
n + 1
(  )
( 
​ ____

) (  )
​   ​  
 ​
T 2 r/2 3 ​R​ 2 ​
\ ​​ ____________
​       ​  ​​ ​ = ​​ ___
​  r    ​  ​​ ​
1 lunar month
29. T µ r3/2

( 8 )
1
__
1 ​   ​ 
​​ __
fi  T = ​   ​   ​​2 ​ lunar month = 2– 3/2 lunar month D T 3 ___
\ ​ ___   ​ = __
T 2
D r D T
T
3 D r
​    ​ ​     ​   fi ​ ___   ​ × 100 = __
​   ​  ​ ___
2 r
(  )
​     ​  × 100  ​
24. When a particle falls from •, its KE on reaching the
3
surface of Earth is = __
​   ​  (1%) = 1.5%
2

GMm
k1 = loss in PE = 0 – ​ –  ​ _____
R
    (  GMm
​  ​ = _____
​     
R )

​ ...(i) T
30. ​ ________
365 days
r/2 3/2
 ​​ ___
   ​ = (  )
​  r   ​   ​​ ​
When it falls from a height h = 10R, its KE on
365
reaching Earth is \  T = ____
​  __ 
 ​ days = 129 days
  
2​÷2 ​
GMm
k2 = Loss in PE = – ​ _____   
11R
GMm
​ – ​ – ​ _____
R
   ​  ​ (  ) g
31. ​ __ ​ =
  g – w2R
2

÷ 
___
10 GMm g g
= ​ ___ ​ _____
​     ​   ...(ii) fi  w R = ​   ​   fi  w = ​ ___
2 __ ​    ​ ​  
11 R 2 2R
Velocity of a point on the equator is
1
__
​    ​ mv 12

÷ 
___
k__1 ___ 11 2 11 gR
  ​    ​  fi ​ _____ ​ 
\  ​   ​ = = ___
​    ​ v = wR = ​ ___
​   ​ ​  
k2 10 1
__ 2 10 2
​   ​  mv 2
2 ____

÷ 
___
v1 11    
ve = ​÷2gR 
​ = 2v
fi  ​ __
v2  
 ​ = ​ ​ ___ ​ ​  
10
÷ 
_____
2GM
32. ve = ​ _____
​     
​ ​  
25. Both have same speed, as they are in the same orbit. R
after collision, the combined mass will have zero Let speed of the body at the centre of the tunnel be
velocity to conserve momentum. v. From conservation of energy, we get:
S.128  Mechanics II

1 3 GMm GMm
​ __ ​  mv2 – __
​   ​  _____
​     ​ = – ​ _____
    
​ Worksheet 2
2 2 R R

[ 
​ Potential at the centre = – ​ __ ​  ____
3 GM
​     
2 R ]
​  ​ 1. if a

particle is inside the sphere
F µ r

÷ 
____
GM GM v \  (A) is correct
v  = ____ ​   fi  v = ​ ____  ​ ​ = ___
2 e
fi  ​      ​      ​  __  ​ 
R R ÷   
​ 2 ​ 1
If a particle is outside the sphere F µ __
​  2  ​ 
GM 1
33. ag = ​ ____ ​ 
  fi  ag µ ___
​    ​
  r
R2 R2 \  (B) is correct
34. Time period of satellite = 24 h 3. S is satellite. Any signal sent from it can reach up
Earth and the satellite are orbiting in opposite direc- to a point P on the surface of Earth. Colatitude at P
tions. Both will rotate by 180° (in opposite direction) is q.
in 12 hours. The satellite will once again be above R
sin q = _____
​      ​
the same point on Earth. R+h
35. From momentum conservation, we can find the speed
of the fragment of mass 4M.
3
5Mv0 = – Mv0 + 4Mv  fi  v = __
​   ​  v0
2
__
If speed of a satellite is made more than ÷     times
​ 2 ​
its orbital speed, it will escape.
36. Angular speed of Earth doubles. This implies that its
time period becomes half (= 12 h).
T 2 µ r3
Area of the spherical cap PCP is given by
T/2 2
\  ​​ ___
​     
T (  ) (  )
​   ​​ ​ =
R 3
​​ __
r
​ r ​   ​​ ​  fi  R = _____
​  1/3
(4)
   ​  A = 2p R2 [1 – cos (90 – q)]

= 2p R2 [1 – sin q]
dA L mv0r 1
37. Areal velocity ___
​   ​ = ___
​    ​ = ____
​     
​= __
​   ​  v0r
dt 2m 2m 2 The area that cannot be captured is
____
A¢ = 4p R2 – 2p R2 [1 – sin q] = 2p R2 [1 + sin q]

1 GM
= __
2 ÷ 
​   ​  ​ ____
​  r   
​ ​   
1 _____
◊ r = __
2

​   ​  ​÷GMr 
​ 
4. Due to air resistance, the mechanical energy of the
_ satellite decreases. But its KE increases, as orbital
\  areal velocity µ ​÷r 
  ​ 
speed increases for smaller radius of path.

÷ 
___
R 1 5. A satellite that is not above the equator cannot remain
\  Required ratio = ​ ___
​     ​ ​ = __
​   
4R 2 continuously over a point on Earth. Similarly, a satel-
 ​
lite that is rotating opposite to the Earth’s spin cannot
remain above a point. A communication satellite,
must always remain above a particular point.
6. Angular momentum is directed perpendicular to
the plane of rotation and is fixed in magnitude and
direction.
7. Centre of the Earth is focus of any path of satellite.
Such a path will definitely pass over the equator. The
smallest time period is for a satellite, which is near
to the surface of Earth.

÷ 
__
R
Tmin = 2p ​ __
​ g ​ ​  


T1
T2 (  )
R 3/2
8. (A) ​ __ ​  = ​​ ___
​     ​  ​​ ​ =
4R
1
__
​   
8
 ​
Solutions  S.129

÷ 
___
v1 4R __ 2
(B) ​ __ ___
v2 ​  = ​ ​  R ​ ​  = ​ 1 ​  Worksheet 3
____

÷ 
GM _____ 1. Consider the object (mass = m) at a distance x from
(C) L = mvr = m ​ ____
​  r    ◊ r = m ÷
 ​ ​    
​ GM r ​  the centre inside the tunnel. For it to experience no
_ acceleration, net force on it (in RF of the planet)
\  L µ ​÷r 
  ​ 
must be zero.

÷ 
__
L1 1 1 fi  Gravitational force = centrifugal force
\  ​ __ ​   = ​ __
​   ​ ​   = __
​    ​
L2 4 2 fi mg¢ = mw2x
2p
(D) w = ___
​   ​  fi  g¢ = w2x
T

(  ) ÷ 
____
w1 GM GM
fi ​ ___ ​ =
T2 8
__
​   ​ = __
​   ​  fi ​ ____
​  3 ​  ​ x = w2x  fi  w = ​ ____
​  3 ​ ​ 

w2 T1 1 R R

÷ 
__________
 4
9. Along ACB, the planet is relatively farther (from sun) G ◊​__
   ​  p R3 ◊ r
÷ 
______
3
__________ 4
as compared to its distance when it travels along fi  w = ​ ​    ​ ​
     = ​ __
​   ​  p Gr  

BDA. It means along ACB the planet is slower. R3 3
Time needed in ACB > time needed in BDA 2. Force applied by the ring on the sphere = force
Since planet is bound to the sun, its total energy is applied by the sphere on the ring. For finding force
negative. on the ring due to the sphere, we can replace the
\  |U| > k at all points. latter by a point mass placed at its centre.

Consider two small masses, dm each, on the ring at


diametrically opposite ends. Force on each element
due to M is
GMdm
dF = ______
​  2 ​  
r
Components of the two forces along x-direction will
add and their components perpendicular to it will
cancel out
\ Total force on the ring
GM
F = Ú dF cos 30° = ____  cos 30° Ú dm
​  2 ​ 
r
[Note that r and cos 30° are same for every element
selected on the ring]
GM
\  F = ____
​  2 ​ 
 cos 30° ◊ m
r __ __
GMm ÷     ​÷________
​ 3 ​     GMm
3 ​
= _________ ​  2     ◊
 ​  ___
 ​   ​
  = ​   ​ 

(a + 3a ) 2 2 8a2
3. The equation D  = mgh assumes a constant value of
g and therefore, it gives a value of D  that is larger
than the general value (i.e., correct value).
S.130  Mechanics II


GMm
D Ucorrect = – _____
​ 
R+h
  ( 
GMm
 ​ – ​ – ​ _____
R
   ​  ​
  ) 5. (a) Acceleration due to gravity has the same value
as gravitational field intensity.

[ 
At r = R, field is contributed by the core only.
1
= GMm ​ __
R R+h
1
​    ​ – _____
​     
GMmh
 ​  ​ = ​ ________ 
R (R + h)
  ​ ] The solid part can be divided into many shells
and each shell creates zero field at the point (as
According to the problem, the point lies inside each shell).
GM
DU = 1.010 (D Ucorrect) \  g1 = ​ ____  ​ 
R2
GMmh
fi  mgh = 1.010 ◊ ​ ________ 
  ​ (b) In this case, the entire mass can be assumed to
R (R + h) be at the centre.

fi 
R
( 
g = 1.010 g ​ _____
​     
R+h )
R+ h
 ​  ​  fi ​ _____
R
​ = 1.010
    Volume of core = __
4
​   ​  p R3 = V (say)
3
4 4
h h Volume of solid part = ​ __  ​ p (3R)3 – ​ __  ​p R3
fi  1 + ​ __ ​   = 1.010  fi ​ __ ​  = 0.010 3 3
R R = 8V.
fi  h = 0.010 R = 0.010 × 6400 km = 64 km \  Mass of solid part = 4M
[   density is half that of core]
4. All the stars move in a common circle of radius r
about the centroid O. Total mass = 5M
a a
r = ________
​  = ___
   ​  ​  __  ​  G 5M
2 sec 30° ÷   
​ 3 ​ \  g2 = ​ _____2 ​ 
(3R)
5 GM
fi  g2 = ​ __ ​  ____
​   ​ 
9 R2
6. P1 is the city and S1 is the location of the satellite at
given time.

Force on the star at A is


FA = 2F cos 30°  (along AO)
__ __
   ÷
​÷3 ​
Gm ◊ m ___    Gm2
​ 3 ​ 
= 2 ______    ​ = ​ _______
​  2 ​ ◊ ​ 
   ​ 
  In the next 8 hours, the satellite will make one
a 2 a2 full rotation and will be back to S1. But the
This force provides the necessary centripetal force to
city (P) will have rotated by 120° during this
the star A.
mv2 _______
fi  ​ ____
r    
​ =

÷
​ 
   Gm2
​ 3 ​ 
__
 ​   
  ___
fi ​ 
v2    Gm
​÷3 ​ 
______
a   ​ = ​  a2 ​ 
___

__
24
8
period ​ ___( 
​    ​ × 360 = 120  ​. )
a2
(  )
​  __  ​  360 °
____ ÷   
​ 3 ​ In next t hours, the satellite rotates through ​ ____
​   ​ t 

  ​
÷ 
GM 8
v = ​ ____
(  )
fi  ​  a   
​ ​   360 °
and P rotates through ​ ​ ____ ​ t  ​ .
Mechanical energy of the system 24
For S to be above P –
E = U + K
3Gm ◊ m
= – ​ _______
a   
1
​ + 3 ◊ ​ __
​   ​  mv 2  ​
2 (  ) (  ) (  )
360
24
360 °
​   ​  t  °​ = ​ ____
120° + ​ ____ ​   ​ t 
8

  ​

3Gm2
= – ​ _____
a   

​+
3 Gm
__
2
(  )
​    ​ m ​ ____
​  a   
3 Gm2
​  ​ = – ​ __ ​  ____
2 a
​     
​  fi  t = 4 h
Solutions  S.131

\  Satellite will be exactly overhead P at 10. Momentum of the system will remain conserved.
position S2. A will slow down and B will speed up. At a point they
Total time elapsed = 8 + 4 = 12 h will have the same velocity (at separation r). After
this they will start getting closer. Thus, the system
\  Time in clock = 8 PM on 17 January 2018. will be bound if r is not infinity.
4p2 v0
7. For the Earth: (1 yr)2 = _____
​     ​ (1 AU)3 ...(i) 3mv = mv0  fi  v = ​ __ ​ 
6  Ms 3

(  )
Ms is mass of the sun. Gm (2m) 1 1 v0 2 G (m) (2m)
– ​ ________r0 ​  + __
​   ​  mv02 = ​ __ ​  (3m) ​​ __
​   ​   ​​ ​ – _________
​  r   ​ 
For the star in the galaxy: 2 2 3
4p2 2Gm2
Gm (2m) _____ 1
(3 × 108 yr)2 = ____ ​ (3 × 109 AU)3
​      ...(ii) fi ​ ________ 

​ = ​  ​ __ ​  mv 20
G M r  r0 ​ 
 –
3
Ms 2Gm2 1
From (i) and (ii): (3 × 108)2 = ___
​   ​ 27 × 1027 For r < •, _____
​  r  ​ 
 > ​ __ ​  mv 20
M 0 3

÷ 
_____
fi  M = (3 × 1011) Ms 6Gm
fi  v0 < ​ _____
​  r  ​ ​ 
  
0

D M = 3 × 1011 Ms – 2.8 × 1011 Ms = 0.2 × 1010 Ms GMM


11. When A is at •, U0 = – _____​       

2a
= 2 × 1010 Ms Where M is mass of particles at B and C and 2a is
8. Effective value of ‘g’ at equator is separation between them.
g¢ = g – w2R When A is at y = 0,

÷ 
___ GM (2 ) ________
GM(2M)
g g GMM ________
fi  ​ __ ​ =
  g – w2R  fi  w = ​ ___
​    ​ ​   U = – ​ _____
   ​ – ​  a    ​ – ​  a   ​ 
2 2R 2a

÷ 
____ – 9 GMM
1 = ________ ​       

\  u = wR = ​ __
​   ​  gR ​  2a
2
____ = 9U0
Escape speed is ve = ÷    
​ 2gR 
​ = 2u. 12. At highest point, velocity (v) will be perpendicular
____

÷ 
GM to the position vector OP.
9. Orbital speed, v0 = ​ ____
​  r   
 ​ ​   ...(i)

Masses of the fragments are


m1 = m, m2 = m, m3 = 4m
After explosion: v1 = – v0, v2 = 0, v3 = ?
Momentum conservation: 6mv0 = – mv0 + 4mv3
7
fi  v3 = ​ __ ​  v0
4
A satellite
__ will escape if its velocity is increased to
v1 = ÷     v0.
​ 2 ​
__
dr
In this case v3 > ÷     v0
​ 2 ​ [When v is ^ r to r, it implies that __
​   ​ = 0 and r has
dt
Therefore, the third piece will escape the gravity of stopped growing]
the planet. Angular momentum of the projectile about O is
Energy conservation for m2: conserved.
\ LA = LP
GMm __ 1 GMm
– ​ _____  + ​   ​  mv2 = – _____
 ​  ​  r   


r/2 2 (m u sin q) R = mvr
_____

÷ 
2GM __
v = ​ _____
​  r    =÷
​ ​ 
      v0  [From (i)]
​ 2 ​ fi  (u sin 60°) R = vr ...(i)
S.132  Mechanics II

Energy conservation gives: 2Mr 2r r


15. r1 = ​ _______
    ​ = __
​   ​  ; r2 = ​ __  ​ 
2M + M 3 3
GMm __ 1 GMm __ 1
– ​ _____
r   ​ + ​    ​ mv2 = – ​ _____
   ​ + ​    ​ mu2

2 R 2 G (M) (2M)
Mr1w2 = 2Mr2w2 = __________​   ​
     ...(i)
r2
( 
__
GMm __
fi – ​ _____
r   
1
2
÷
​ + ​   ​  m ​​ ___
    uR 2
​ 3 ​
​   ​ ​ ___
2 r )
GMm __
   ​  ​​ ​ = – ​ _____
R
   
1
​ + ​   ​  mu2
2 And
2GM _____
w2 = ​ _____2
2 GM _____
 ​ = ​ 
2r

3 GM
 ​ = ​  3 ​ 

r1 r ​ __ ​  ◊ r 2 r
3
GMm __ 3 GMm GMm __ 1 GMm
fi – ​ _____ 
r   ​ + ​   ​  _____   = – ​ _____
​   ​ R
     ​ + ​   ​  _____
​     ​  
8 r2 R 2 R

[  ÷  ]
____
GM
​ u = ​ ____
​     
​ ​   ​
R

1 3 __ R 1
fi – ​ __r ​  + __
​   ​  ​  2  ​  = – ​ ___   ​
8 r 2R

fi 4r2 – 8Rr + 3R2 = 0


G (M) (2M) __ 1 1
R 3R \ E = – ​ __________ + ​   ​  M r12 w2 + ​ __  ​ (2M) r22 w2
r     ​ 
fi r = __
​   ​  and ___
​   ​  2 2

(  ) (  )
2 2
3R
Obvious answer is ___
R
​   ​ . Height above surface = ​ __ ​ 

2GM2 __
= – ​ _____
r   

1 2GM2 __
2
( 
​ + ​   ​  ​ _____
​  r   
2r
) (  )
3
1 2GM2 __r
​  ​ ​ ​   ​   ​ + __
​   ​  ​ _____
2
​  2 ​  
r
 ​ ​ ​    ​   ​
3
2 2
GMm 2GM2 2 GM2 1 GM2 GM2
13. Usurface = – 8 × 109 J = – ​ _____
   ​
  = – ​ _____
r   
​ + __
​   ​  ____
​     
​ + __
​   ​  ____ ​ = – ​ ____
​  r    r   
​ 
R 3 r 3
GMm
PE at height R is U = – _____ ​      ​ = – 4 × 109 J 16. (a) All particles in the ring are at distance R from
2R the centre.
Look into the figure of last solution
G dm G GM
\  V = – Ú _____​ = – __
​      ​   ​  Ú dm = – ____
​     
 ​
R R R
KP + UP = KA + UA

1
​ __  ​ mv2 – 4 × 109 = 6 × 109 – 8 × 109
2
_______
\ v2 = 2 × 109  fi  v = ​÷2  × 109 ​ 
ms–1
________
1
At surface, ​ __ ​  mu2 = 6 × 109  fi u = ÷   × 109 ​ 
​ 12 ms–1 (b) All particles in the ring are at distance
2 ______
Conservation of angular momentum: r=÷
​R  2 + x2 ​ 
from the given point

mu sin q ◊ R = mv (2R) G dm GM


\  V = – Ú ​ _____​ = – ​ ____
r    ​ 
r   
________ _______
​÷ 12 × 109 ​ 
sin q = 2 ​÷2  × 109 
 ​ GM
= – ​ ________
______   ​ 

÷ 
__
2 ÷​ R  2 + x2 ​ 
fi  sin q = ​ __
​    ​ ​ 
3 GMm
16a. Energy of the satellite in its orbit is E1 = – ​ _____
( ÷  )
__  

   
2 2r
fi  q = sin  ​ ​ __
​   ​ ​    ​ –1
3
14. g¢ = g – w2R = 0

÷  ÷  ÷ 
__ ____ _____________________
g GM 6.67 × 10–11 × 3 × 1024
fi  w = ​ __
​    ​ ​  = ​ ____  = ​  ​ ____________________
​  3 ​ ​     
     ​ ​
  
R R (1.8 × 106)3

= 5.85 × 10–2 rad s–1


Solutions  S.133

Its energy when it gets close to Earth (i.e., it is 18. (a) Let v1 be maximum and v2 be minimum
revolving in a circle of radius R) is speeds.
GMm mv1 (2R) = mv2 (4R)
E2 = –  ​ _____
  ​ 
2R
fi v1 = 2v2 ...(i)
\  Loss in energy D E = E1 – E2
1 GMm 1 GMm
​ __ ​  mv12 – _____ __
​   ​  mv22 – _____
( 
​     ​ =
  ​     ​
  ...(ii)
GMm __
= _____
​   ​ 
2
1 1
 ​ ​   ​  – __
R r
​   ​   ​ ) 2 2R 2 4R

÷ 
____
GM
Solving (i) and (ii) gives v2 = ​ ____
​ ​  ​    ​  – ​ __ ​   ​.
​   ​ ​  
  and
2w ( R r )
D E _____ GMm __ 1 1
\  t = ​ ___ ​ = ​ 
w       6R

÷​   3R   ​ ​ 


_____
2GM
17. (a) From conservation of energy, it follows that the v = ​ _____
1  
particle will come out of the tunnel from the
other side and reach height h. (b) If r is the radius of curvature of point A, then
\  Maximum displacement = 2R + 2R = 4R
mv21 GMm 4v 21 R2 8R
(b) Particle has maximum speed when it reaches the ​ ____
r   
​ = _____
​   ​ 
  fi  r = ______
​     
​= ___
​   ​ 
centre (O). After this, the particle retards. (2R)2 GM 3

kP + UP = k0 + U0 19. Total energy of a satellite in elliptical orbit is


GMm
GMm __ 1 3 GMm –  ​ _____
  ​ 
0 – ​ _____ 
  ​ = ​    ​ mv2 – __
​   ​  ​ _____
   ​  2a
2R 2 2 R

[ 
1 GMm GMm
\  ​ __  ​ mv2 – _____ ​ = – ​ _____
3 GM
​ Note:  Potential at centre = – ​ __ ​  ____
2 R
​     
 ​  ​ 2
​  r   

] 2a
  
__________

÷ 
2 __ 1
÷  (  )
_____
2GM ____ fi  v = ​ GM ​ ​ __r ​  –  
​ a ​  ​ ​
fi  v = ​ _____
​      =÷
​ ​      
​ 2gR ​
R
Chapter 12 Elasticity

Your Turn ​ 
(20 N) 
= __________________________
     ​ = 2 × 10–4
  
 (0.005 × 10 m2)2 × 1011 Nm–2
–4
D  2 mm
1. (i) Œl = ___
​     ​ = _______
​  3     
​ = 0.002
L 10  mm x 0.2 × 10–1 cm
8. Shear strain = __
​   ​  = ​ ___________
 ​
    
= 0.004
F 100 N L 5 cm
(ii) s l = __ ​   ​ = ___________
​     ​ = 107 Nm–2
  
A 10 × 10–6 m2
F 8 N
Shear stress = __
​   ​ = ​ ____________
    ​ = 3,200 Nm–2
(  )
x p p  
2. Œs = __
​   ​  = tan q = tan 1° = tan ​ ____
​     ​  ​ ____
​     ​  A 0.05 × 0.05 m2
L 180 180
= 0.0174 Shear stress _____ 3200
\  h = ​ __________
    ​ = ​    = 8 × 105 Nm–2
 ​ 
[for small angles tan q q] Shear strain 0.004
3. Volume stress = increase in pressure = r gh 1 D V
9. K = __
​    ​ = ___
​   ​ 
3
= 10 × 9.8 × 2000 = 1.96 × 10  Nm 7 –1 B VP

FL fi D V = KVP
4. x = ___
​    ​
AY
On doubling the radius, A becomes 4 times. = (5 × 10–10 m2 N–1) (1000 ml) (15 × 106 Nm–2)
Therefore, doubling F and radius will make value of
x half. = 7.5 ml
d2
5. Area of cross-section of the cable, A = p ​ __ ​  D V ___
4 D P D P
10. B = ____
​     ​   fi ​ ___   
​ = ​     
 ​
D V V B
(  )
0.05 2 ___
​     
​ 
\  A = 3.14 ​​ ____  ​​ ​ = 0.002 m2
​   ​   V
2
D V 0.1
(a) Additional tension in the cable when people ​ ___   
​   = ____
​    ​ = 1 × 10–3
V 100
enter the elevator is
F = (238 kg) × (9.8 ms–2) = 2332 N D P
\  ​ ___   
​   = 1×10–3  fi  D P = 9.8 × 105
B
FL 2332 × 20
D L = ___
​    ​ = ​ ______________
      ​
AY 0.002 × 2 × 1011 fi  r gh = 9.8 × 105
= 1.17 × 10–4  m.
fi  103 × 9.8 × h = 9.8 × 105
(b) Tension remains unchanged if the lift moves
with constant speed. fi  h = 100 m
6. Strain = fractional change in length
1

D L Stress
= ___
​   ​ = _____
​     ​
  11. u = __
​   ​  × Stress × Strain
L Y 2

855
= ____________________
​       ​ = 9 × 10–5
   1 1
= ​ __ ​  × (Y ◊ strain) ◊ strain = __
​    ​ Y Œ2
(9.5 × 10–4) × (1 × 1010) 2 2

7. 3g – T = 3a  and 1
12. U = __
​   ​  × (maximum stretching force) (extension)
2
T = 6a
1
Solving, T = 20 N = ​ __ ​  × 800 × 2 × 10–3 = 0.8 J
2
Stress
Strain = _____
​      ​

Y
Solutions  S.135

Stress D l Stress


Worksheet 1 10. Strain = _____
​      ​  fi ​ ___   ​  = _____
  ​     ​

Y l Y
l
1. Same material implies same Y. \  D l = (Stress) __ ​    ​
Y
(Strain)A = (Strain)B
Graph between D l and (stress) is a straight line with
(Strain)A ________
(Stress)B l
​ ________
    ​  = ​  ​   fi  (Stress)A = (Stress)B
    slope = __
​    ​
Y Y Y
FA FB FA 4 4 mm – 2 mm 1m
fi  ​ ___ ​  = ___​    ​   fi ​ ___  ​ =​ __ ​  fi ​ ___________________
   
  6  ​ = ___
​     ​
A A/4 FB 1
6
(8 × 10 – 4 × 10 ) Nm –2 Y

2. Tension (and hence, stress) is maximum at the top. fi  Y = 2 × 109 Nm–2


Longitudinal strain is maximum at the top.
4. A material fails if breaking stress is applied to it. 11. V = p r2 ◊ l
What it means is that it is stress (and not the force), D V D r D l
which decides the breakage of the rope. Both ropes ​ ___   
  2 ___
​ = ​  r   ​  + ___
​     ​ 
V l
fail when stress reaches breaking stress.
D V D r D l
500 N ​ ___   
​  × 100 = – 2 ​ ___ ___
r   ​  × 100 + ​  l   ​  × 100
F V
​ ___  ​  = _____
​     ​ = F = 2,000 N

4A A [Radius decrease]
D l
5. Solution to the previous question has the clue. fi  0.2 = – 2 × 0.002 + ___ ​     ​  × 100
l
6. Tension in the wire in both cases is W.
D l
___
7. Tension is maximum at the lowest position. fi  ​     ​ =   0.204
l
8. Let natural length be l0. Stress = Y (strain)
Stress l1 – l0 T1
Strain = _____
​      ​   fi ​ _____  ​ 
 = ___
​    ​  = 2 × 1011 × 0.204 = 4.08 × 108 Nm–2
Y l0 AY
T1l0 D l F 100 1
fi  l1 – l0 = ____
​   ​   ...(a) 12. ​ ___   ​  = ___
​    ​ = ________________
​         ​ = ___
​    ​ × 10–3
AY l AY p (10–3)2 × 2 × 1011 2p

T2l0 D r/r
Similarly, l2 – l0 = ____
​   ​   ...(b) Passion’s Ratio = ____
​   ​ 
AY D l/l
D r ___ p  ___ 1
l1 – l0 l_____2 – l0 fi  ​ ___ – 3
r   ​   = ​ 10  ​  ◊ ​ 2p  ​ × 10  = 0.5 × 10
–4
From (a) and (b): ​ _____ ​ 
 = ​   ​   
T1 T2
T2 l1 – T1 l2 fi  Dr = 0.00005 mm.
fi l0 = ​ _________  ​ 

T2 – T1 \  New radius = 1mm – 0.00005 mm
1
9. U = __
​   ​  = 0.99995 mm
2
(Final value of force stretching the wire) × (extension) 13. Go through solution given to Q.9 in Worksheet 3.
Look at equation (i) in the solution.
1
= __
​    ​ × 200 × 0.001 = 0.1 J 14. Thickest wire will have the highest load for a given
2
strain.
S.136  Mechanics II

3. Strain in all rods are same. Therefore, stress is same


Worksheet 2 in all of them.
1. Tension is the same everywhere. Cross-sectional areas are in ratio
\  Stress in Cu wire is more. Obviously, strain AA : AB : AC : AD = 1 : 2 : 3 : 4
is higher for Cu wire, as it has a smaller Young’s
FA FB FC FD
modulus as well. ​ ___  ​ = ___
​    ​= ___
​    ​ = ___
​    ​
AA AB AC AD

\  FA : FB : FC : FD = 1 : 2 : 3 : 4
1
Energy stored, U = ​ __ ​  F ◊ D l  fi  U µ F
2

Cu wire has more strain and longer original length. \  UA : UB : UC : UD = 1: 2 : 3 : 4
Therefore, it has more extension.
4. When a block attached to a spring is released from
2. Tension in A > Tension in B its unstretched position, it will go down to stretch the
mg 4 spring by x such that
TA = mg + ___ ​   ​ = __
​    ​ mg 2mg
3 3 1
​ __ ​  kx2 = mg x  fi  x = ____ ​    ​ 

1 2 k
And TB = __
​   ​  mg  fi  TA = 4TB
3 Ultimately, due to air friction, etc., the block will
When rA = rB, Stress in A is more. Therefore, it settle down in its equilibrium position, stretching the
breaks spring by

(  )
TA TB mg x
(Stress)A = ______ ; (Stress)B = ______
​   2 ​   ​     ​  x0 = ___
​     ​ ​ = __
​    ​  ​
p (rA) p (rB)2 k 2
x
Thus, energy equal to mg ​ __  ​  is lost forever. This
(Stress)A > (Stress)B if 2
energy is mainly lost as heat in overcoming viscos-
TA
​ ___2 ​ >
rA

TB
rB
TA
​ ___2 ​   fi ​ ___  ​ >
T B
(  )
rA 2
​​ __
​ r  ​   ​​ ​
B
ity of air.
In case of the wire (which is just like a very stiff
spring), similar thing happens. We may not be able
fi  (  )
rA 2
4 > ​​ __
​ r  ​   ​​ ​  fi  2rB > rA
B
to observe oscillations as in the case of a spring due
to small amplitude. Ultimately, the system settles in
equilibrium where half the loss in gravitational PE
When rA = 2rB, both wires have the exact same
of the block is lost as heat.
stress.
Solutions  S.137

4. Stress at distance x above the lower end


Worksheet 3 is
xArg
1. Cross-sectional area supporting the load is Stress = _____
​     

​ = x r g
A
A = p ​ ​​ __
2 [ (  ) (  ) ]
3 2 0.8 2
​   ​   ​​ ​ – ​​ ___
​   ​    ​​ ​  ​ cm2
2
Elastic potential energy stored in an ele-
ment of length dx is
FL
D L = ___
​    ​ 1
AY dU = __
​   ​  (stress) (strain) (volume)
2

(  )
Substituting the values (in SI unit) will give the 2 2
1 x r g 1 r  g  A 2
answer = __
​   ​  (x r g) ​ ____
​     ​  ​ (A dx) = __
​   ​  ______
​     ​ x dx
2 Y 2 Y
YA
2. Force constant of each cord is k = ___
​     ​ 
l 2 2 L
r2g2AL3
1 r  g  A
\  U = Ú dU = ​ __ ​  ______
​     ​ ​Ú ​    ​ x2dx = _______
​     


5 × 108 × 1 × 10–6 2 Y 0 6Y
fi k = ​ ________________
 ​   = 5 × 103 Nm–1
  
0.1
5. Strain is same in all wires.
Elastic energy stored in both cords is \ (stress)s = Ys ◊ (strain)


1
E = ​ __(  )
​   ​  k D l2  ​ × 2
2
(stress)cu = Ycu ◊ (strain)
(stress)s ___ Ys 2
\ ​ ________   ​    ​ = __
 ​ = ​    ​
fi E = 5 × 103 × (0.06)2 = 18 J (stress)cu Ycu 1
Ts __ 2 1
1 1 5 fi ​ ___  ​ =
  ​   ​   fi  Tcu = __
​    ​ Ts
\ ​ __ ​  mv2 = 18  fi ​ __ ​  × _____
​     ​ × v2 = 18 Tcu 1 2
2 2 1000
Also 2 Tcu + Ts = Mg
fi v = 84.9 ms–1
Ts + Ts = Mg
3. Let the length of original wire be l1 Mg
Ts = ​ ___ ​ 

2
Stress x F
Strain = _____
​      ​ or,
;  __
​    ​  = ____
​     
​ 6. Let tension in two wires be Ts and TB. For rotational
Y l1 A1Y
equilibrium:
Fl1 Fl1 TS ◊ x = TB = TB (2 – x) ...(i)
fi x = ____
_____
​     
​=
​  2    
​ ...(i)
A1Yp r Y
(a) (Strain)S = (Strain)B
When the wire is re-drawn to have square section of
side length a, its length (l2) will be given by volume (Stress)S ________
(Stress)B
conservation fi  ​ _______
   ​  = ​      ​ 
YS YB
p r2
l2 . a2 = p r2 ◊ l1  fi l2 = ____ ​  2 ​  ◊ l1 ...(ii) TS TB
a fi  ​ ___  ​  = ___
​    ​   [   YS = 2YB and AB = 2AS]
2A 2A
Stress x F
Strain = _____ ​   fi __
​   ​  = ____
2
​      ​    ​ 
Y l2 A2Y fi TS = TB ...(ii)
Fl2 Fl2
fi x2 = ____
​     
​= ____
​  2   ​  Solving (i) and (ii) gives: x = 1m
A2Y a  ◊ Y
(b) (Stress)S = (Stress)B
p r2
F ​ ____
​  2 ​ l1  ​
a
= ​ _________   

​ = ______
​ 
(  )
p F r2 l1
   
​ 
TS
​ __ ​   =
A
TB
___
2A
TB
​    ​   fi  TS = ___
​   ​  
2
...(iii)
a2 Y a4 Y 4
Solving (i) and (iii) gives: x = ​ __  ​ m
But a = 2r 3
7. TP = 4mg ; TQ = mg
p Fl1 ___ p 2 A wire will break if stress in it exceeds the breaking
\  x2 = ___
​    ​ ___
​  2   ​ = ​   ​  x [Using (i)]
16 r Y 16 stress. Since both wires are made of the same mate-
rial, they have same breaking stress.
S.138  Mechanics II

(a) For P to fail:


(Stress)P > (Stress)Q
4mg mg r2 1
​ ____  ​  > ​ ____2  ​   fi ​ __
r1 ​  > ​ __ ​ 
p r12 p r2 2

(b) For Q to fail:

(Stress)Q > (Stress)P


Tension in the rod at distance r from axis is
r2 __ 1 r1
fi  ​ __ __
r1 ​   < ​ 2  ​  fi  r2 < ​ 2 ​ 
L
1
T = ​Ú ​   ​rAw2rdr = ​ __ ​  rAw2 [L2 – r2] ...(i)
r 2
8. A1 = p r2 ; A2 = p (R2 – r2) [One can also assume the entire mass of the seg-
ment of the rod between r and L to be located
s1 A1 + s2 A2 = W ...(i)
at its COM.

[ (  ) ]
Strain in both sections is same.
1 1 2
s1 s2 ​   ​  × 104 × p × 10–2 × (400)2 ​ ​​ __
T = __ ​   ​   ​​ ​ – r2  ​
2 2
​ ___ ​   = ___
​   ​   ...(ii)
[ 
Y1 Y2

​ 
WY2
Solving (i) and (ii) gives s2 = __________

    ​

1
= 8p × 106 ​ __
4 ]
​   ​  – r2  ​ N

A1Y1 + A2Y2 (b) Now, if dy is the elongation in the element of


length dr, at position r, then strain
s2 A2 __________
A2Y2
Required fraction is: _____
​     
​ = ​   
    ​ 2
W A1Y1 + A2Y2 dy stress ___ T 1 rw
​ ___  ​ = _____
​     ​ = ​    ​ = __
  ​    ​ ____
​     ​ [L2 – r2]
dr Y AY 2 Y
(R2 – r2) Y2
= ​ _______________
       ​ So the elongation of the whole rod
r2 Y1 + (R2 – r2) Y2
L
rw2 1 rw L
2 3
9. Let L be the greatest length of the wire that can hang ​   ​ ​Ú ​    ​(L2 – r2) dr = __
D y = ____ ​   ​  ______
​      ​ 
2Y 0 3 Y
vertically without breaking.
Mass of wire m = cross-sectional area (A) × length 4
1 10 × (400)  (0.5)
2 3
1
(L) × density (d), = __
​   ​  × ​ _______________
   ​
   = ​ __  ​ × 10–3 m
3 2 × 1011 3
Weight of wire = mg = AL dg
F
The top of the wire experiences maximum tension 11. Acceleration of the block is a = __
​ m  ​
equal to weight of the wire. Wire is likely to break Tension at a distance x from the rear end is
at the top.
LA dg
Breaking stress = _____
​     ​ = Ldg
A
fi  7.5 × 108 = L × 2.7 × 980
7.5 × 108
fi  L = ​ ________   ​ cm = 2.834 × 105 cm m
2.7 × 980 T = m¢a, where m¢ = __
​   ​  x = mass of segment of length

x.
= 2.834 km
10. (a) Tension in the rod at a distance r from the axis Fx
fi T = ___
​   ​ 
of rotation will be equal to the centripetal force 
needed by all elements between x = r and x = L. Tdx
Elongation in element of length dx is = ____
​   ​ 
,
Consider an element of length dr, at a distance AY
r from the axis of rotation, as shown in figure.  
Tdx Fxdx ____ F
The centripetal force acting on this element will Total elongation, d = Ú​  ​  ​ ____
​   ​ = Ú​  ​    _____
​​   ​ = ​      

o AY o AY 2AY
be = dmrw2 = (rAdr) rw2.
Chapter 13 Miscellaneous Problems based on Chapters 11 and 12
FS (Stress)S ◊ AS __
4 16 ___ 32
​ ___  ​ = ___________
​     ​ = ​    ​ × ___
   ​   ​ = ​   ​ 
  match the column FB (Stress)B ◊ AB 3 10 15
2 2
1. I0w0 = Iw fi ​ __  ​ MR2 ◊ w0 = __
​    ​ Mr2 ◊ w 4. O is centre of the Earth.
5 5 Particle is projected from
fi (  )
R 2
w = ​​ __
​ r ​   ​​ ​ ◊ w0. Thus, w decreases. A and its smallest distance
from O is R (when the par-
Free-fall acceleration at equator is ticles reaches B). If smallest
distance is less than R, the
g¢ = g – w2r = ​ ____
GM
r 2
 ​  [ (  ) ]
R 2 2
 – ​​ ​​ __
​ r ​   ​​ ​ w0   ​​ ​ ◊ r particle will hit the Earth.

GM _____ R4w20
= ____
​  2 ​   – ​  3 ​    mv ◊ 2R = muR  fi  2v = u ...(i)
r r
1 GMm __ 1 GMm
As the planet contracts, r decreases. The second term in ​ __ ​  mv2 – _____
​     ​ = ​    ​ mu2 – _____
  ​     ​

(  )
2 2R 2 R
1
above expression ​ a ​ __3  ​   ​ increases more rapidly com-
GM
r (2v)2 – v2 = ____
(  )
fi  ​     
​ 
1 R
pared to the first term ​ a __ ​  2  ​   ​. Thus, g¢ decreases.

÷ 
____
r GM
fi  v = ​ ____
​     
There is no effect of rotation at poles. 3R

​ ​
\  Acceleration due to gravity at poles = free fall
GM   Passage-based Problems
acceleration at poles = ​ ____ ​ 
 .
r2
with decrease in r, gpole will increase.
Passage 1
1. Each particle is like a satellite of the Saturn. Every
GMm
Binding energy = _____
​  r   

  particle experiences gravitational pull of the planet
and force on it due to the particles in the ring can
With decrease in r, binding energy will increase be neglected, as the ring has a very small mass.
2. At maximum height, Particle 2 will have a velocity The particles closest to the planet have highest speed.

  [ 
in a direction perpendicular to its radius vector. It is given by

÷ 
_____ _________
Particle 2 has angular momentum = muR, where u is

÷
GM GM
its speed of projection. It cannot hit the surface of the v0 = ​ _____
​   ​ ​ = ​ ________
  
  ​       ​ ​
Earth normally, as this will violate the law of conser-
R+h h
R ​ 1 + __ ]
​   ​   ​

 ÷  ÷
R
vation of angular momentum. [Angular momentum _____ ___________
of a radially moving body about the centre of the gR 10 × 6 × 107
Earth will be zero] = ​ _____
​        ​ ​ = ​ ​ ___________
  
   ​ ​ 
h 8
1 + __
​   ​  1 + ___
​    ​ 
3. Change in length of steel = Change in length of R 60
brass
= 2.3 × 104 ms–1.
D l D l
\  (Strain)S = ___
​   ​  ; (Strain)B = ___
​   ​  Mass of heaviest particle = mass of an ice ball of
30 20
radius 10 m
(Strain)S __ 2
fi ​ ________  ​ = ​   ​ 
4
m0 = __
4
​   ​  p r3 ◊ d = __
​   ​  × 3.14 × (10)3 × 103
(Strain)B 3 3 3

(Stress)S = 4.19 × 106 kg


_______
​      ​ 
YS 1
fi ​ ________ 
2
 ​ = __
​   ​  \ KEmax = __
​   ​  m0v02
(Stress) 3 2
________
​ 
B
    ​ 
YB 1
= __
​   ​  × 4.19 ×106 × (2.3 × 104)2
2
(Stress)s __ 2 YS __ 2 4
fi ​ ________  ​ = ​   ​  ___
​    ​ = ​   ​  × 2 = __
​    ​
(Stress)B 3 YB 3 3 = 1.1 × 1015 J
S.140  Mechanics II

2. The nearest particles are going in a circle of radius


r1 = 68,000 km and the farthest ones are in orbit of
radius r2 = 60,000 + 80,000 = 1,40,000 km.
T1 2
\ ​​ __ (  ) (  ) (  )
r1 3
​   ​   ​​ ​ = ​​ __
T2
68 3
​ r   ​  ​​ ​ = ​​ ____
2
​    ​  ​​ ​ = 0.12
140
T1
\ ​ __ ​ =
  0.35
T2
3. Work done in moving the mass from A to B along
1
any path will be same. fi ​ ____
   ​ (v 2 + 100 v02) < 2v02
121
\ W1 = W2
÷ 
____
____ GM____
fi 2
v < 142 v20  fi  v < ​÷142 ​ 
    v0 = ÷    ​ ____
​ 142 ​ ​     
​ ​  
​  A Æ B
W1 = W ​ext Agt ​ = UB – UA = m (VB – VA) R
​        ​
​slowly ​
6. Let speed at minimum distance be u1
W1
\ VA – VB = – ​ ___
W 1 + W2
________
m ​ = – ​ ​  2m    ​  
​ (  ) ​___›

R
mu1 __
u1 10 ____
  muy ◊ R  fi ​ __ ​  = ___
​   ​ =
2 2
____
GM
​   ​ ​ ​     
11 R
​ ​  
÷ 
÷ 
4. From A to O, the gravitational force is along AO​ .
​    ____
20 GM
Force by external agent is equal and opposite. Thus, fi u1 = ___
​   ​ ​ ____
​     
​ ​  
1 11 R
F is positive. F decreases as __
​  2  ​  when m is outside
r Conservation of energy
the sphere. Inside the tunnel, F µ r. At centre, F
GMm 1 GMm 1
= 0. From O to D, gravitational force is in posi- – ​ _____ ​ 
 + __
​   ​  mu21 = – ​ _____​ +
    __
​   ​  mu2
tive direction. Force by external agent is in negative R/2 2 R 2
direction. Substituting for u1 and u______
we get an equation in v.

÷ 
5. Momentum conservation gives: 58GM
______
solving this gives: v = ​ ​      ​ ​ 

11m ux = mv R
7. r1 + r2 = r
v
fi  ux = ___
​    ​  m1r1 = 2mr2
11
2r r
11m uy = 10m ◊ v0 Solving, r1 = __
​   ​  and r2 = ​ __  ​ 
3 3

÷ 
____
10 10 GM
fi uy = ___
​   ​ v0 = ___
​   ​ ​ ____
​     
 ​ ​ 
11 11 R

For star of mass m:


Gm (2m)
mw2r1 = ________
​   ​ 

r2
Speed after collision is
2r 2Gm
______ fi ​ __ ​  w2 = _____
​  2 ​   
u = ​÷u  2x + u2y 
 ​ 3 r
3Gm
_________ fi w2 = _____
​  3 ​    ...(i)
​    ​ ​÷v  2 + 100 v
1
= ___ 2
  0  ​ r
11
Gme M
For the satellite to avoid escaping For the Earth mewe2R = ​ ______  ​ 

__ R2
    v0
u < ​÷2 ​ GM
fi we2 = ____
​  3 ​   ...(ii)
fi u2 < 2v02 R
Solutions  S.141 

According to the problem w for the star system is


same as that of the Earth.
\  From (i) and (ii)
3Gm GM
​ _____ ​ =

  ​ ____ ​ 
r3 R3
Given M = 3m
\ r = R
3Gm ____ GM
8. From (i) w2 = _____
​  3 ​ 
 = ​  3 ​ 
r R
\  KE of the star of smaller mass is

÷ 
_____
1 2GM
K1 = ​ __ ​  m (wr1)2 fi v = ​ _____
​     ​ ​ 

2 R
1 M 2r 2 _____
= ​ __ ​  ◊ ​ __ ​  ◊ ​​ __
2 3 3 (  ) 3GM
​   ​   ​​ ​ ◊ ​  3 ​ 
r
  Angle between velocities of two bodies is 60° at this
instant.
2 GM __ 2 GM \  Relative speed is
= ​ __ ​  ____ ​ = ​   ​  ____
​      ​     
​ 
÷ 
___________________ _____
9 r 9 R 2GM
vr = ​÷v  2 + v2 + 2v
    v = ​ _____
v cos 120° ​ =
​     
 ​ ​ 
2
9. T  µ r  3 R
12. Distance between the bodies = chord length A1B1
T2 2
\ ​​ __
T1 (  ) (  )
​   ​   ​​ ​ =
r2 3
​​ __
​ r  ​   ​​ ​
1 3R
= ​ ___
2 (  3R
​   ​ sin 30°  ​ × 2 = ___
​   ​ .
2 )
8 2
fi ​​ __
1 (  ) (  )
r2 3
​   ​   ​​ ​ = ​​ ___
​  4  ​  ​​ ​
10
13. After the weight is added, the joint P moves to P1
Extension in BP is PP1 = D l1 (say)
fi r2 = 4 × 104 km
2p
Speed of S1 is v1 = w1r1 = ​ ___ (  )
​    ​  ​ (104 km)
1 h

= 2p × 104 kmh–1.

2p
Speed of S2 is v2 = w2r2 = ​ ___ (  )
​    ​  ​ (4 × 104 km)
8 h
= p × 104 kmh–1. Drop a perpendicular from P on the line AP1.
\  The position of the satellites when they are Extension of wire AP is
nearest is as shown in the figure. 1 = D l2 (say)
4 –1
Relative speed = |v1 – v2| = p × 10 kmh . [This is because AQ  AP]
10. Angular speed Since extension are small,
Relative speed –APB  –AP1B = q (say)
w = ​ ____________
  
   ​
Distance S1S2
P1Q = PP1 cos q  fi  D l2 = D l1 cos q ...(i)
p × 104 __ p
= ​ _______4   ​ = ​   ​  rad h–1.
3 × 10 3 \  D l1 > D l2. It means extension in BP is more.
R 14. For BP, Stress = Y strain
11. Speed of each body at height __ ​   ​  can be calculated
2 T1 D l1 D l1
using conservation of energy as \  ​ __ ​ =
  Y◊ ​ ___ ​  
  fi  T1 = YA ​ ___ ​ 

A l1 l1
1 GMm GMm D l2 Dl2
​ __ ​ mv2 – _____
​    – _____
 ​ =
  ​     ​
  T2
2 R/2 R For AP, ​ __ ​ =
  Y ​ ___ ​  
  fi  T2 = Y A ___
​   ​ 
A l2 l2
S.142  Mechanics II

T1 D l1 __l2 1 _____ 1 Let T = 1 planet day


\  ​ __ ​ =
  ___
​    ​ ​   ​  = _____
​     ​  
◊ ​     ​ 
T2 D l2 l1 cos q cos q
T0 = 1 planet year

(  )
2 
fi  T2 = T1 cos q ...(ii) 4p 2 3 ____4p 2 __ m
Now T 2 = ____
​     ​  ​ ​   ​   ​ r3 = T 30  fi T = T0
​ rG = ​    
Gm Gm M
Therefore, T1 > T2
17. The energy of any geostationary satellite is the sum
For vertical equilibrium of kinetic energy of satellite, interaction energy of
T1 + 2T2 cos q = W ...(iii) satellite and its own planet and interaction energy of
satellite and star. Both planets have same mass and
fi  T1 + 2 ◊ T1 cos3 q = W   [using (ii)] same length of day. Geostationary satellite–planet
system will have same interaction energy in either
W
fi T1 = ​ __________
     ​ planet. Also, kinetic energy of both satellites will be
1 + 2 cos3 q same. But the satellite-star system will account for
the energy difference.
W
= ​ _________
(  )
  3   ​
l1 GM m0
__
1 + 2 ​​ ​   ​   ​​​ Ui = – ​ ______
   ​ + U satellite – planet
l2 2r
15. The time in which the planet rotates about its axis is GM m0
Uf = – ​ ______ ​  + U satellite – planet
not given for either planet. 2 (4r)
16. For geostationary satellite, time period = 1 planet day 3GM m0
Emin = Uf – Ui = _______
​      ​ 
8r
Chapter 14 Past Years’ Questions

Centre of Mass and Momentum


AIEE/JEE Main Questions
1. Momentum conservation along x and y directions
gives:
mv = mv2 cos q
fi v = v2 cos q ...(a)
v__
___ Mass of remaining part = 3M [Area = 3p R2]
And m ​    ​ =  mv2 sin q
÷   
​ 3 ​ R
\ 3M ◊ x = M ◊ R  fi  x = __ ​   ​ 
v__ 3
fi ​ ___   ​ =
  v2 sin q ...(b)
÷   
​ 3 ​ 6. We must pay attention to significant digits.
P = m ◊ v
M = 3.513 kg (4 significant digits)
v = 5.00 ms–1 (3 significant digits)
P = 17.565 kg m s–1 17.6 kgms–1
The final answer must be rounded off to keep three
significant digits.
7. For n = 0, linear density is a constant. The COM lies
L
Squaring and adding the two equation gives at x = __
​   ​ .
2
v2
v22 (sin2 q + cos2 q) = v2 + __ ​   ​ 
3
2__
___
fi v2 = ​    ​ v
÷   
​ 3 ​ For any other value of n, the rod is heavier towards
2. The block compresses the spring and then again its end at x = . The COM moves to the right
L
returns to its original position (where the spring is of x = __
​   ​ .
relaxed). At this position, the KE of the block is 2
As n becomes larger and larger, the linear density
maximum (equal to its original KE). This is when
becomes higher and higher. The COM moves to the
the momentum of the block is maximum too.
right for every rise is value of n. In extreme case of
KE when spring is relaxed = PE when spring has n Æ •, the rod becomes extremely massive at its
maximum compression. right end and has near zero density at locations close
1 1 to x = 0. The COM moves very close to x = L.
\ ​ __ ​  Mv2 = __
​    ​ KL2
2 2 2
8. (0.5 + 1) v = 0.5 × 2  fi  v = __ ​   ​  ms–1

÷ 
___
K ____ 3
fi v = ​ __
​   ​ ​  L  fi  Mv = ÷   ​  
​ MK  ◊ L
M
3. The pieces must have equal and opposite momenta
1
Energy loss = __
1
​   ​  × 0.5 × 22 – __
2
2 2
​    ​ × 1.5 × ​​ __
2 (  )
​   ​   ​​ ​
3
\ 4v = 12 × 4  fi  v = 12 ms–1 = 0.67 J
1 9. In all options, h is positive. This implies that upward
\ KE = __
​   ​  × 4 × 122 = 288 J
2 direction has been considered positive. When the ball
4. m1D x1 = m2D x2 begins to fall, its speed increases linearly. Velocity is
downward (negative). As soon as it hits the floor, its
With D x1 and D x2 in opposite directions. velocity becomes upward (positive) without change
m1 in magnitude. There is no time lag in velocity becom-
\  D x2 = ___
​ m  ​  ◊ d ing + ve from – ve, as the duration of collision is
2
negligible.
5. Mass of removed part = M [Area = p R2]
S.144  Mechanics II

By doing this much analysis only we can choose the


answer (C).
However, it is recommended that you plot h versus
t graph on your own.
10. Slope of x – t graph is velocity.
Velocity just before t = 2 s is v1 = slope of graph
2
= __
​   ​  = 1 ms–1 Momentum conservation gives
2
mv1 + mv2 = mv0
Velocity just after t = 2 s is v2 = slope = – 1 ms–1
fi v1 + v2 = v0 ...(ii)
Impulse acted at t = 2 s and changed the velocity 2
from to v1 to v2. Now, (v1 + v1) = v20

\ ​| Impulse |​ = |​ m (v2 – v1) |​ = 0.4 × 2 = 0.8 Ns or 1.5v02 + 2v1v2 = v02  fi 2v1v2 = – 0.5 v02 ...(iii)

Similar impulse acted on the body at Now, (v1 – v2)2 = v21 + v22 – 2v1v2
t = 4, 6, 8s  ...
= 1.5 v20 + 0.5v02
11. To conserve momentum, there must be some veloc- __
or    v0
v1 – v2 = ​÷2 ​ 
ity after collision. This implies that there cannot be
a complete loss of kinetic energy. Iit Jee (Advanced) Questions
12. Maximum energy is lost in perfectly inelastic 1. Both statements are true but only statement 2 cannot
collision. lead to conclusion in statement 1.

)
​_› ​_›


1
2
2
Loss = ​ __ ​  m vrel = __
( 
1 mM
​    ​ ​ ______
2 m+M
​     ) ( 
1
 ​  ​ v2 = f ​ __
​   ​  mv2  ​
2
2. ​P  ​ 1¢ + ​P  ​ 2¢ = 0
This is possible in (b) and (c) but not possible in (a)
M and (d).
where f = ​ ______
    ​
M+m 3. (i) Velocity of the block just before reaching B can
be calculated using energy conservation.
13. Let velocity after collision be v = vx  + vy  1
​ __ ​  mu2 = mgh1
2 2
3m vx = m (2v) fi  vx = __
​    ​ v u2 = 2gh1 = 2g (BD tan 60°)
3 __ __
2 = 2 × 10 × ÷     × ​÷3 ​
​ 3 ​     = 60
And 3m vy = 2mv fi  vy = __
​    ​ v ___
3 \ u = ÷    ms–1
​ 60 ​
______ __
v = ​÷ 
  
2​÷2 ​
\ vx2  ​ = ____
+ vy  ​   ​ 
 v Component
___
of u along BC is u cos 30°
3  
= ​÷45 ​ ms–1

Loss in energy

(  )
__
1 1 1    2
2​÷2 ​
= ​ __ ​  m (2v)2 + __
​   ​  m (v)2 – __
​    ​ (3m) ​​ ____
​   ​ 
 v  ​​ ​
2 2 2 3

5
= ​ __ ​  mv2
3
5
__
​   ​  mv2
3
_________________
% loss = ​         ​ × 100 56%
1
__ 1
​   ​  m (2v)2 + __ ​   ​  2m (v)2
2 2

1 1 This remains unchanged and the velocity


15. ​ __ ​  m(v 21 + v 22) = 1.5 ​ __ ​  mv02 component normal to BC becomes zero after
2 2
collision.
or, v21 + v22 = 1.5 v02 ...(i) (ii) Height of B (above C) is
__
    tan 30° = 3 m
h2 = 3​÷3 ​
Solutions  S.145

Energy conservation between B and C gives 8. Area under F – t graph = Impulse = Change in
1 1 ___ momentum
​ __ ​  mv2 = mgh2 + __   ) 2
​   ​  m (​÷45 ​
2 2
1 1
fi v2 = 2 × 10 × 3 + 45 = 105 \  P = ​ __ ​  × 4 × 3 – __
​   ​  × 1.5 × 2 = 4.5 kg m s–1
2 2
____
\    ms–1
v = ​÷105 ​ [Note that the straight-line graph tells you that F is
2N at 4.5 s]
(iii) If collision is elastic, velocity component
u sin 30° gets inverted and component u cos 30° P2 _____ 4.52
down the incline remains unchanged. Vertical \  k = ___ ​     
​ = ​    
 ​ = 5.06 J
___ ___ 2m 2 × 2
component of velocity is ​÷ 15 ​  cos 30° – ​÷ 45 ​ 
sin 30° = 0. 9. Let initial speed of 1 kg mass be u before collision
and speed of 5 kg mass be v after collision.
1 ◊ u = – 1 (2) + 5v
fi u = – 2 + 5 v ...(i)
Speed of approach = speed of separation
fi u = 2 + v ...(ii)
–1 –1
Solving (i) and (ii) gives: v = 1 ms  ; u = 3 ms
\ Pinitial = 1 × 3 = 3 kg m s–1
1 × 3
5. First collision occurs at B, as the particle having vcm = ​ _____ 
 ​ = 0.5 ms–1
1+ 5
speed 2 u covers double the distance travelled by the
particle having speed v. 1 2 1
​ __ ​  Mvcm = __
​   ​  × 6 × 0.52 = 0.75 J
2 2

÷ 
___
2h
10. For ball: 20 = v1 ​ ___
​  g ​ ​  

÷ 
_____
2×5
fi  20 = v1 ​ ​ _____   fi  v1 = 20 ms–1
 ​ ​  

10

÷ 
___
2h
For bullet: 100 = v2 ​ ​ ___
g ​ ​    fi  v2 = 100 ms
–1

At B, the velocities get exchanged and the next Momentum conservation


collision will occur at C. Once again, velocities are
0.01v = 0.2 × 20 + 0.01 × 100 fi v = 500 ms–1
exchanged and the particles meet at A. ___
6. ycm 11. Speed of first bob at top of its trajectory is ​÷ gl1 ​ 
Exchange of velocity takes place during collision.
6m (0) + m (a) + m (a) + m (0) + m (– a) ___
= ​ __________________________________
           ​ ___
= ​    ​  \  Second bob acquires speed ___ ÷
​   1____
gl  ​ after collision.
6m + m + m + m + m 10
  2 ​ 
  1 ​ = ​÷5gl
According to the question: ​÷gl
7. Speed of B after elastic collision between A and B
l1
is fi ​ __ ​ =
  5
l2
( 
​ 
2m
v = ​ _______
  
2m + m )
 ​  ​ × 9 = 6 ms–1
12. In the figure, a triangle has been shown, but it can
by any polygon. Sum of all interior angles

= (n – 2) p

After inelastic collision of B and C, speed is


2m ◊ v
\
1 (n – 2)p
q = __
​   ​  _______
2
​  n    ​ = ​ __( 
p p
​   ​  – __
2 n )
​   ​  ​
v0 = _______
​    ​ = 4 ms–1

2m + m
S.146  Mechanics II

Rotational Motion

AIEE/JEE Main Questions


1. In an RF attached to the centre, both particle are
performing circular motion with speeds

h h v1 = wR1  and  v2 = wR2


Clearly (from first figure): h1 = ____
​      ​ = __________
​       ​
sin q sin ​ __ p __ p
( 
​   ​  – ​ n ​  ​
2 ) v12
Accelerations are: a1 = ___
​    ​ = w2R1
R1
h
= _____
​   p  ​ 
cos ​ __ a2 = w2R2
n ​

[ 
Accelerations in ground frame are also a1 and a2
\
1
D h = h1 – h = h ​ _____
​   p  ​ – 1  ​
cos ​ __
n ​ ] because the centre has no acceleration.
F1
\ ​ ___ ​ =
a1
__
​ a  ​  =
R1
___
​   ​ 
13. COM of the system does not move. Let displacement F2 2 R2
of the block be d (¨). (Assuming the particles to be of equal masses)
Displacement of the point mass = (R – d) (Æ)
Ma2
4. I0 = ____
​   ​ 

mR 6
Md = m (R – d)  fi  d = ______
(  )
​     ​ (¨) Ma2 a 2
M+m IA = I0 + Mr2 = ____

​   ​   + M ​​ ___
​  __  ​  ​​ ​
6 ÷   
​ 2 ​
mR
______
\  Displacement of COM of the block = – ​     ​ 2
M + m = __
​   ​  Ma2
3
x co-ordinate of point mass is same at co-ordinate
of the right edge of the block.
For finding velocities, first we use conservation of
momentum.
mv + mV = 0 ...(i)
Energy conservation gives: 5. As insect moves towards centre, MI decreases and
hence angular speed increases. When the insect
1 1
​ __ ​  mv2 + __
​   ​  MV2 = mgR ...(ii) moves away from the centre, MI increases and angu-
2 2
lar speed decreases.
Solving (i) and (ii) gives velocity (v) of particle as
6. Torque, t = r F = 2 (20t – 5t)2
well as velocity of the block (V).
t
14. Speed gained by the black due to the impulse is \  Angular acceleration, a = __
​   ​ = 4t – t2
I
J 1 w
vo = __
​ m   ​ = ___
t
​    ​ = 2.5 m/s dw
0.4   4t – t2  fi  ​Ú ​  ​   ​dw = ​Ú ​    ​(4t – t2) dt
\  ​ ___ ​ =
dt 0 0
Given, v = vo e– t/t [vo = velocity at t = 0] t3
fi  2 __
w = 2t – ​   ​ 
dx 3
or, ​ ___ ​ =
  2.5 e –t/t
dt w = 0  when  t = 6 s
x t
or, ​Ú ​    ​dx = 2.5 ​Ú ​    ​e – t/t dt dq
o o Now  w = ___
​   ​ 
dt
t
or, x = – 2.5t [​  e–t/t  ]​o
dq t3
\  ​ ___ ​ =
  2t2 – __
​   ​ 
dt 3
or, x = – 2.5 × 4 [e–1 – eo]

(  )
q 6
t3
= 10[1 – 0.37] = 6.30 m fi  ​Ú ​    ​dq = ​Ú ​    ​​ 2t2 – __
​   ​   ​ dt
0 0 3
Solutions  S.147

q = 36 rad 12. For the particle moving along AB

( 
​_› ​_›
)
36 _ R R
2p n = 36  fi  n = ___
​›
fi  ​    ​ < 6 ​L  ​   = m (​R  ​  × ​v  ​)  = m ​ ___
​  __  ​  + ___
​  __  ​    ​ × (v  )
2p ÷   
​ 2 ​ ÷   
​ 2 ​
8. Angular momentum conservation about A gives: mvR
= – ​ ____
__ ​ 
2
mR w0 = mvR + mR ◊ w 2  ÷   
​ 2 ​
fi  mR2w0 = mvR + mvR    [   v = wR] Similarly, one can find angular momentum for other
segments of the path.
w0R
fi  v = ​ ____
 ​ 
  Though the question was intended to have one correct
2 answer, it has two correct answers, viz (b) and (d)
13. For a moment, assume that the double cone does
not deviate and its centre O continues on a straight
line. After moving some distance, the left cone will
be touching rail AB on a circle of radius r1 and right
cone will be touching the rail CD on a circle of radius
9. mg – T = ma ...(i) r2 with r2 > r1. If angular speed of the cone is w and
2 
it is rolling, then speeds of the center C1 and C2 of
For cylinder: TR = mR ◊ a the circles touching the rails will be
fi  T = ma  [ Ra = a] ...(ii) v1 = wr1  and v2 = wr2
From (i) and (ii); a = g/2 Clearly v2 > v1
10. Particle (P) moves in a circle forming a conical
pendulum. Angular momentum about O is
​_› ​_› ​_›
​L  ​   = ​r ​   × (m ​v  ​) 

The double cone will steer towards left. [What will


happen if the centre of right wheel of your car moves
faster than the centre of left wheel? The car steers to
the left. If the left tire of the car bursts suddenly, the
car will steer to the left on its own causing serious
​_› ​_› ​_› ​_› accident].
The magnitude of ​r ​ ,  ​v  ​  and angle between ​r ​   and ​_​v  ​
  
› 14. Assuming the mass to be constant,
does not change with time. Hence, magnitude of ​L  ​ 
m
is constant.
_ _ p R2l r = m  fi R2 = ____ ​    ​ 
​› ​› p r l
But plane containing
​_› ​r ​   and ​v  ​  is changing. Hence,
direction of ​L  ​  is changing. Now, the said MI is

( 
11. Diagonal of cube = 2R
​÷ 3 ​  a = 2R
__
ml2 mR2 __
I = ​ ___ ​ + ____
12
​   ​ 
4
m l2
 = ​   ​  ​ __
4 3 )
​   ​  + R2  ​

(  )
2R
a = ​ ___
__ ​  m l2 ____
= __
​   ​  ​ __
m
​   ​  + ​    ​ ​ ...(i)
÷   
​ 3 ​ 4 3 p l r
M
Density, r = ​ _____    ​ 
4 dI
__
​   ​  pR3 For minium I, __ ​    ​ = 0

(  ) ( 
3 dl
2R M
) 2M
(  )
3
3 ___ ______ _____ m 2l
​ = ​  __   ​  fi  ​ __ ​  ​ __ m 2l m
Mass of Cube m = a  ◊ r = ​​ ​  __ ​  ​​ ​ ​ ​     ​  
4 ​   ​  – _____ ​ ​ = 0  fi ​ __ ​  = _____
​  2      ​  2     

  
​÷3 ​  ​   ​ p R
  3 ÷
​    ◊
3 ​   p 4 3 p l r 3 p l  r
​3 ​
2
2
4MR2 2l p R  l r
ma
Momenta of inertia is I = ____
​   ​   = ______
​  __     
​ fi ​ __ ​   = ______ ​  2     


6    ◊ p
9​÷3 ​  3 p l r
S.148  Mechanics II

÷  (  )
__ mg 1
l2 3 l 3 fi f sin q = ​ Mg + ___​   ​  ​ cos q – __
​    ​ Mg cos q
or, ​ ___2  ​  = __
​    ​  fi ​ __  ​ = ​ __
​    ​ ​  2 2
R 2 R 2
1
__
fi f = ​   ​  (m + m) g ◊ cot q
One may choose to express equation (i) in terms of 2
R, instead of l. Note that by considering torque about P, we have to
eliminate the need to think about force applied by
15. I a = t
BP on AP.
Ml2 l 3 g p R2
​ ____   = Mg ​ __  ​  sin q  fi  a = __
 ​ ◊ a
  ​   ​  __
​   ​  sin q 4. Area of removed part = ____ ​   ​   
3 2 2 l 9
16. Moment of inertia of the entire system about an axis \  Mass of removed part = M
passing through O is (O is COM of the system) I = Iwhole – Iremoved



1
2 [ 1
ICM = ​ __ ​ MR2 + 6​ __
2 ] 55
​   ​  MR2 + M(2R)2  ​ = ___
​   ​  MR2
2
1
= __
1
​   ​  (9M) (R)2 – ​ __
2
R 2
​   ​  (M) ​​ __
2 [  (  )
​   ​   ​​ ​ + M ​​ ___
3 (  ) ]
2R 2
​   ​  ​​ ​  ​
3
IP = ICM + 7 M(3R)2 = 4MR2
55 181 5. For horizontal and vertical equilibrium F = N and
= ​ ___ ​  MR2 + 63 MR2 = ____   2
​   ​ MR
  f = mg. For rotational equilibrium about the centre C,
2 2
1 line of normal force (N) must pass below C. This will
17. Area of removed part is ​ __ ​  of the area of the original
9 ensure that torque due to f gets balanced by torque
disc
due to N.
\ mass of removed part = M
I = Icompleted disc – Iremoved disc
1
= __
2
1 R 2
​   ​ (9M)R2 – ​ __ [  (  ) (  ) ]
​    ​M​​ __
2 3
2R 2
​   ​   ​​ ​ + M​​ ___
​   ​  ​​ ​  ​ = 4 MR2
3
IIT JEE (Advanced) Questions

g sin q g sin q 2 g sin q


1. a = ________
​  2 2    ​ = ______
  ​   ​ = _______
  ​   ​   
k /k + 1 1
__ 3
​   ​  + 1
2 1 2
6. ​ __ ​  Mr2 + Mr2 = ​ __ ​  MR2
2. Normal force at A and B are same due to symmetry. 2 5
Friction at A is towards right and at B it is towards 2
fi r = ____
​  ___    ​  
R
left. Both friction forces must have same magnitude    
​÷15 ​
for horizontal equilibrium of the system.
Thickness has no role to play.
For vertical equilibrium
7. Friction is not dissipative in pure rolling. Friction
2N = 2Mg + mg provides torque about centre and increases angular
mg speed.
fi N = Mg + ___
​   ​ 
2 mg sin q – f = ma ...(i)
2 2
And f ◊ r = __
​   ​  mr2 ◊ a  fi  f = __
​    ​ ma ...(ii)
5 5
2
__
Solving (i) and (ii) gives: f = ​   ​  mg sin q ...(iii)
7

Consider rotational equilibrium of segment AP and


taking torque about P to be zero gives: If q decreases, friction will decrease. Static friction
L force is given by (iii). It is not equal to m N.
Mg __
​   ​  ◊ cos q + f ◊ L sin q = N ◊ L ◊ cos q
2
Solutions  S.149

8. Energy of the ball is conserved. From A to B, rolling


is pure and friction is static. There is no dissipation
of energy. From B to C, there is no friction.
KE of the ball is maximum at B since PE is least
there. At B, the total KE of the ball is not translational.
A part of it is rotational also. This rotational energy
does not change from B to C, as there is no friction.
At C, the ball has same rotational energy as at B,
though its linear speed is zero. Thus, total energy at
C is not potential energy.
hC < hA (ii) Let final common angular velocity be w0
9. Consider a strip of width dx on the plate as shown. (I + 2I) w0 = I ◊ 2w + 2I ◊ w
Area of strip = a dx. Number of balls hitting the strip 4
per second = (a dx) n. fi w0 = __
​   ​  w
3
For disc A:
Change in angular momentum is


4
|D L| = I ◊ 2w – I ​ __ (  )
​   ​  w  ​ =
3
2
__
​    ​ Iw
3
Since angular impulse = D L
2 2Iw
\ t ◊ t = __
​   ​  I w  fi  t = ____
​     
​ 
Change in momentum of each ball = 2mv 3 3t
Force on the strip, dF = (2mv) (an dx) (iii) Loss in KE = ki – kf

]
Torque due to this force about the hinged side is

dt = x ◊ dF = 2mv an x dx [ 
1
= ​ __
2
1
​   ​  I (2w)2 + __
​    ​ (2I) w2  ​ –
2
1
(  )
4
​ __ ​  (3I) ​​ __
2
2
​    ​ w  ​ ​
3
b
3 1
Total torque t = 2mv an ​Ú ​   ​x dx = ​ __ ​  mv ab2n = __
​   ​  Iw2
b 4 3
__ ​   ​ 
2
13. On an incline, asolid > acylinder
This torque balances the torque due to weight
Thus, statement I is false.
3 b 15. If vB = v then vC = 2v and vA = 0
\  ​ __ ​  mv ab2n = Mg ◊ ​ __ ​ 
4 2 16. The block will begin to slide when tan q > m
2 Mg 2 3 × 10 __
fi  v = ​ __ ​  ​ _____ = __
  ​  ​   ​  × ​ ________________
       ​ fi  tan q > ​÷3 ​ 
    fi  q > 60°
3 m abn 3 0.01 × 1 × 2 × 100

= 10 ms–1

10. Energy is conserved in pure rolling


1
2
1 v 2
​ __ ​  mv2 + __
​   ​  I ​​ __
2 R (  ) 3v2
​   ​   ​​ ​ = mg ​ ___ (  )
​   ​  ​
4g
1
fi  I = __
​   ​  mR2
2
\  Body is disc.
The block will topple, if vertical line through COM
1 1 1 1 passes from outside the base. Critical case has been
12. (i) ​ __ ​  I (2w)2 = ​ __ ​  kx12  and ​ __ ​  (2I) w2 = __
​   ​  kx22
2 2 2 2 shown in the figure.
x1 __
\ ​ __ x2 ​   = ÷   
​ 2 ​
AB
tan q = ___
​    ​ =
10/2
____
​    ​ =
2
__
​   ​ 
BC 15/2 3
S.150  Mechanics II

This angle is much less than 60°. Thus, the block


will topple even before it begins to slide.
17. Though not stated explicity, in the context of the
problem, we assume that 2 N is the normal force
applied by the stick on the ring.
N1 = 2 N

As the ring rolls, the stick applies a kinetic frciton


force on it (if we assume that the stick is having no

[ 
vertical motion). However, there is no slipping at the
ground surface.
1
Ip = ​ __
​   ​  4M (2R)2 + (4M) (2R)2  ​
2 ]
[ 
MR
– ​ ____
2
]
 + M (4R2 + R2)  ​
​   ​ 
2
37
= ___ ​   ​ MR2
2
Ip 37/2
​ __ ​   = ____
​   ​  3
I0 13/2
N1 – fs = ma fi  2 – fs = 2 × 0.3 20. Energy is conserved in pure rolling
fi  fs = 1.4 N 1 1
​ __  ​ Mv2 + ​ __ ​  Iw2 = Mgh
And (fs – fk) R = Ia fi  fs – fk = m (R a) 2 2
1 1
fi  1.4 – fk = 2 × 0.3 fi  fk = 0.8 N fi  ​ __ ​  M (Rw)2 + ​ __ ​  Iw2 = Mgh
2 2
fk 0.8
\  m = ___
​    ​ = ___
​   ​  = 0.4 1
N1 2 fi  ​ __  ​ [MR2 + I] w2 = Mgh
2
\  P = 4 \  w is larger for smaller I. Cylinder Q has

) [ 
lower I.
18. ( 
2
I = 2 ​ __
5
2
​    ​ mr2  ​ + 2 ​ __
5 (  ) ]
a 2
​    ​ mr2 + m ​​ ___
​  __  ​  ​​ ​  ​
÷   
​ 2 ​
21. The centre (O) must have a velocity towards the right,
equal to
= 9 × 10– 4 kgm2 v = 3R ◊ w
​_›
\ N = 9 \ ​v  ​ 0 = (3Rw) 

Velocity of P wrt O is as shown.


19. Mass of bigger disc = 4M
​_› w w
Mass of removed part = M ​v  ​ r = – ​ __ ​  R sin 30°  + __
​   ​  R cos 30° 
2 __
2

1 1
I0 = ​ __ ​  4M (2R)2 – ​ __
2 [ 
​   ​  MR2 + MR2  ​
2 ] wR
= – ​ ___ ​   + ___
4
÷
​  
3 ​

​   ​ wR 
4
__
13 ​_› ​_› wR ÷   
​ 3 ​
= ___
​   ​  MR2 fi ​v  ​p  – ​v  ​0  = – ​ ___ ​   + ___
​   ​ wR
2 4 4
Solutions  S.151

__
_
( 
​›
)
wR ÷
fi ​v  ​ p = ​ 3Rw – ​ ___ ​    ​ + ___
4
  
​ 3 ​
​   ​ wR 
4
25. Imagine yourself at the origin of a co-ordinate sys-
tem. You always see the same face of the disc (just

​   ​ Rw )​ 
​   ​ Rw )​  + (​  ___
__ like we always see only one face of the moon). As the
= ​( ___
11  
​÷3 ​   COM of the disc completes one rotation, the disc is
4 4 back to its original orientation (in both cases). Thus,
22. At time t, angular speed about COM is same as w.
x = vt 26. Conservation of angular momentum about vertical
axis gives

Moment of inertia of the system about rotation axis


(through O) is I = I0 + m (vt)2
Where I0 = MI of the rod.
50 (0.4)2
​ ________
2
 ​  ​  [ 
50(0.4)2
 × 10 = ​ _______
2
 ​  ]
 + 2(6.25) (0.2)2 × 2  ​ w

\  Angular momentum at time t is fi w = 8 rad s–1


L = Iw = [I0 + mv2t2] w 27. The ladder is about to slip. This means that friction
is limiting.
L is changing as I is increasing. An external torque
is needed to change L. [ If there is no torque, the
rod will slow down as the MI increases.]
dL
\ t = ​ ___ ​ = mv2w (2t)
dt
\  Torque increases linearly with time. For t > T,
no torque is needed.
23. Go through solution to​_›Q.10 in AIEEE/JEE Main
questions.
​_› Obviously, ​L   ​P Keeps changing direc-
tions. ​L  0 ​ is along the z-axis always and has fixed
magnitude of mvr.

when m1 = 0 (in option (A) and (D))


N1 = f2  and  N2 = mg

Torque about A = 0
l
fi  N1 l sin q = mg __ ​    ​  cos q
2
mg
24. One can easily observe that relative velocity will fi  N1 = ___
​   ​ cot q
2
become zero twice in every rotation. Thus, (a) is the
mg
obvious choice. fi  N1 tan q = ___
​   ​ 
2
when m2 = 0
There is no force to balance N1. Hence, equilibrium
is not possible.
when m1 π 0, m2 π 0

N1 = m2N2
S.152  Mechanics II

N2 + m1N1 = mg
mg
fi  N2 + m1 m2 N2 = mg  fi N2 = ________
​      ​
1 + m1 m2
28. Conservation of angular momentum gives
MR2
2mvr = ____  w
​   ​ 
2

2
Its MI is dI = ​ __ ​  r (4p x2 dx) x2
3
R R
2 2 k 4p
\ IA = ​Ú ​    __
​​   ​  rA (4p x4dx) = __
​    ​ ____ ​ ​Ú ​    ​x5 dx
​     
0 3 3 R 0

8p k R6
= ____
​   ​ ___
​   ​ 
3R 6
R
2
Similarly for B; IB = Ú​  ​    ​ __
​    ​ rB (4p x4 dx)
0 3
1
4 × 5 × 10–2 × 9 × __​   ​  R
4mvr
_____ _________________ 4 8p k 8p k R
10
fi w = ​  2 ​ 
= ​    
    ​ = ​ ____5 ​ Ú​  ​    ​x9dx = ____
​  5 ​ ​ ___ ​ 
MR 1
__
–2
45 × 10 × ​   ​  3R 0 3R 10
4
–1
= 4 rad s IB 6
\ ​ __ ​  = ___
​    ​ 
IA 10
29. t = 3 (F sin 30°) R
31. Angular momentum is conserved. Initial MI = MR2
(point masses at centre)
3R
MI when one mass is at ___ ​   ​ from O and the other is
5
at x is

(  ) __​ M8 ​  x
M 3R 2
I = MR2 + __
​   ​  ​​ ___
8 5
​   ​  ​​ ​ + 2

​[ MR + __
​   ​  ​​( ___ ​   ​  x ]​ ◊ ​ ___ ​ = MR  ◊ w
8 5 )
M 3R M 8w2
​   ​  ​​ ​ + __
2 2  2
8 9
4
fi  x = ​ __ ​  R
5
​_› 10
Angular impulse 32. ​r ​   = a t3  + b t2  = ​ ___ ​ t3  + 5t2 
3
= t t = 3FR t sin 30° _
​›
​_› d ​
r ​

​v   ​ = ​ ___ ​ = 10t2  + 10t 

1
= 3 × 0.5 × 0.5 × 1 × __
​   ​  dt
2 ​_›
3
__ At t = 1.0 s, ​v  ​  = (10  + 10  ) ms–1
= ​   ​  N ms

[ 
8
]
​_› ​_› ​_› 10
3 ​L  ​   = m (​r ​   × ​v  ​)  = 0.1 ​ ___
​   ​   + 5    ​ × [10  + 10  ]
Iw = ​ __ ​  3
8
10 5
1 3 = ​ ___ ​  – 5  = – ​ __ ​   N ms
fi ​ __ ​  × 1.5 × (0.5)2 w = ​ __  ​ 3 3
2 8 _ ​_›
​›
d ​v   ​
fi  w = 2 rad s–1 Force ​F  ​   = m ___
​   ​ = m (20t  + 10  )
dt
​_›
30. Consider a shell of radius x and thickness dx. At t = 1 ; ​F  ​  = 0.1 (20 + 10  ) = (2  + ) N.
Solutions  S.153

( 
​_›

​_› 10
t = ​r ​   × ​F  ​  = ​ ___
3 )
​   ​  + 5   ​ × (2  + ) 81
= ___
    2
81​÷24 ​
​   ​  m wa l = ______
​   ​   ma w [(A) is incorrect]
___

5 5
10 20 ​_›
= ​ ___ ​   – 10  = – ​ ___ ​   Nm Direction of ​L  ​0  is along OS.
3 3 ​_›
33. Speed of centre (A) of smaller disc is vA = wa To find LZ we need to add z components of ​L   ​0
​_›
[Rolling without slipping] and ​L  ​S  .
Note that the entire rod (and centres of the discs) is LZ = L0 cos q – Ls sin q
rotating about axis OS that is perpendicular to the
On substituting the value
rod. Let angular speed of the assembly about OS be
w1. Speed of point A can also be written as LZ 80 ma2w
34. N + N cos 60° = mg
2mg
fi N = ____
​   ​  
  ...(i)
3
And  f = N sin 60°
__
mg 16​÷3 ​   
= ___
​  __ ​ = ​ _____
 ​ 
 N
÷   
​ 3 ​ 3

vA = w1 (OA) = w1l
a
\ w1l = wa  fi  w1 = w ​ __ ​ 
l
​__›
Angular velocity w​​   1 is directed along OS. Component
of this velocity in z direction is angular velocity
about z-axis
a w a OA
\ wZ = w1 cos q = w ​ __ ​  cos q = ___
​     ​  ​ ___ 
 ​
l l OP
________
Taking torque about A to be zero.
wa l w [  ]
= ​ ___   ​  ​ ___   ​ = __   [
​   ​ ​ OP =   2a + a2 
​÷24  ​ = 5a  ​ l h
l 5a 5 mg __
​    ​  cos 60° = N ◊ ​ ______
   ​ 
[(B) is correct] 2 sin 60°
__
mgl ____ 2mg ___2h h   
3​÷3 ​
COM of the assembly is at C such that fi  ​ ____
 ​    ​  __  ​   fi ​ __ ​  =
 = ​   ​  ____
​   ​ 
4 3 ÷   
​ 3 ​ l 16
0 + 4m.l __ 4l
AC = ​ _______​ 
   = ​   ​  ​_› ​_›
5m 5 ​_› ​__› ​__› ​_› ​__›
35. (i) ​F  ​ rot = ​F  ​ in + 2m (​v  ​ rot × w​
​  )  + m (​w​   × ​r ​ )  × w​
​   
Angular momentum about COM (or the angular
momentum associated with spin) is The third term is directed radially outward (actually
​___› it is the centrifugal force).
Ls = Icm ◊ w (Direction is along ​   as
BO​ shown) ​__› ​_›
​w​   × ​r ​ =
   (w  ) × (r  ) = wr 
= ​ ____
​   ​ 
2 [ 
ma2 _______
4m (2a)
 + ​ 
2
 ​  
17
 ​ w = ___
​   ​  ma2
2
2

] ​__› ​_› ​__›


​    = (wr  ) × (w  ) = w2r 
(​w​   × ​r ​ )  × w​
[(D) is correct] The second term is a force that is perpendicular to
Angular momentum associated with motion of COM the edge of the slot [i.e., along y direction] and gets
about O (which we have usually called Lorbital in our balanced by the normal force of the slot wall.
discussion in the book) is Fin is zero as mg is balanced by normal force of the
L0 = (5m)  (vcm) (OC) = 5m (w1 ◊ OC) (OC) floor of the slot.
\  In rotating frame,
9l 2
a __
__
(  ) [ 
4l __
__ 9l
= 5m w ​   ​  ​​ ​   ​   ​​ ​ ​ OC = l + ​   ​  = ​   ​   ​
l 5 5 5 ] dv
m ​ ___ ​ =
dt
  mw2r ...(a)
S.154  Mechanics II

​_›
v r ​_› ​__› d​r ​  
dv ​    = 2m ___
2m ​v  ​ rot × w​ ​   ​ × w
fi  v ​ ___  ​ = mw2r  fi ​Ú ​  ​vdv = w2 ​Ú   ​   ​r dr dt
dr 0 R/2
R
= 2m __
​   ​  w (ewt + e– wt)  × (w  )
v2 ___
fi  ​ __ ​ =
2
w2 R2
  ​   ​ ​ r2 – ___
2
​   ​   ​
4 (  ) 4

= – ​ ___ ​ 
mR 2 wt
 w (e + e– wt) 

÷ 
______ 2
R2
fi  v = w ​ r2 – ___
​   ​ ​   The normal force by the slot wall balances this
4
1
Ny = ​ __ ​  mRw2 (ewt + e– wt)
  w ​ r  – ___
______ \
÷ ​ R4 ​ ​ 
2 2
dr
fi  ​ __ ​ = 2

dt The normal force by the floor balances the weight
r t
dr \  NZ = mg
fi  ​Ú   ​   _______
​​  ______
     ​ = w ​Ú ​   ​dt
R2
÷  1
R/2 0
2 ___
​ r – ​   ​ ​ 
  \  N = ​ __ ​  mRw2 (ewt + e– wt)  + (mg) 
4 2

[  (  ÷  ) ]
______ r 36. The question is not clear.
R2 R
fi  ​​ ln ​ r + ​ r – ___ 2
  ​  __​​ ​ ​  = wt
​   ​ ​   The official answer key states the answer as (c) or
4 ​ 2 ​ 
(cd). I am not attempting any solution as I see lots

[  ]
[Refer a Calculus text for integration] of ambiguity.

÷ 
______
37. Once again, the question is ambiguous. In official
R2
r + ​ r2 – ___​   ​ ​ 
  answer key, first question was awarded bonus marks
4
fi  ln ​ ​  __________
 ​      ​ = wt and answer to the second question was given as
R
__
​   ​  (A).
2

÷ 
______
R2 R
fi  r + ​ r2 – ___   = ​ __ ​  ewt
​   ​ ​  
4 2

fi 
R2
r2 – ___
R
​   ​   = ​​ __
4 (  2
)
​   ​  ewt – r  ​​ ​
2
R2 R2
fi  r2 – ___
​   ​   = ​ ___ ​ e2wt + r2 – R r ewt
4 4
R2
fi  r R ewt = ​ ___ ​ (1 + e2wt)
4
R
fi  r = __
​   ​  (ewt + e– wt) ...(b) However, let us discuss the problem as it appears
4 interesting.
Alternatively, one can decide the answer by care- (i) In the figure shown, the axis of cone (traced by
fully looking at the options. Options (a) and (c) are finger) passes through O. Contact point moves on
incorrect as they tell that r decreases with time. The circle S. As the contact point moves from A to A1,
unbalanced centrifugal force causes the block to the centre of the ring moves from to C to C1.
move radially outward. Clearly,  – AOA1 = – COC1
R
In option (C), r = ​ __ ​  (ewt + e– wt) This implies that the centre of mass (C) of the ring
4 rotates about O with an angular speed w0. (equal to
d2r R the speed of contact point).
​ ___2 ​  = ​ __ ​  w2 (ewt + e– wt)
dt 4 In fact, the whole ring is in pure rotation about axis
through O.
d2r R Rw2
at t = 0, ___
​  2 ​ = __
​   ​  w2 (1 + 1) = ____
​   ​ 
  1 1
dt 4 2 \  KE = __ ​   ​  I0 × w02 = __
​   ​  [MR2 + M (R – r)2] w02
2 2
This is correct from equation (a) [   Distance, OC = R – r]
(ii) Horizontal normal reaction (in y direction) is equal This is the correct value of KE.
to the term We may also write KE as
Solutions  S.155

__

÷ 
1 2 1
KE = ​ __ ​  MV cm + __
​   ​  Icm w02 mv2 k
2 2 ​ ____   
  kR fi v = ​ __
​ = ​ m  ​ ​   ◊ R
R
1 1 ___
= __
​   ​  M [(R – r) w0]2 + __
​   ​  [MR2] w02 Angular momentum L = mvR = ÷ ​ ◊ R2
   
​ mk 
2 2 ​_›
​_›
​F   ​
1 39. Acceleration ​a  ​  = __
​ m ​  = t î + jˆ 
= __
​    ​ M [(R – r)2 + R2] w02
2 ​_› v t
d ​v   ​
(ii) As discussed above, the COM of the ring is rotating ​ ___ ​  = t î + jˆ   fi ​Ú ​   ​dv = ​Ú ​    ​(t î + jˆ)dt
in a circle of radius (R – r) with angular speed w0. dt o o
​_› 2
t
Forces on it are: or, ​v   ​ = __
​   ​  î + t jˆ 
2

( 
​_
Normal force by finger (N), Friction by finger (f) ​_›

)

​r ​   t
d​r ​   t2 ​_› t2
and weight (Mg) ​   ​  î + t jˆ   fi ​Ú ​  ​ d​r ​ ​
fi ​ ___ ​  = __    = ​Ú ​    ​​ __
​   ​ î + t jˆ   ​dt
dt 2 o o 2
For vertical equilibrium:
_
​› t2 t2
f cos q = N sin q + Mg. fi ​r ​    = ​ __ ​  î + __
​   ​   jˆ 
6 2
fi  f cos q – N sin q = Mg
Centripetal force is
...(a) ​_› ​_› ​_›
t3
\ ​t    ​ = ​r ​   × ​F  ​  = ​ __
6 ( 
t2
​   ​  î + __ )
​   ​  jˆ   ​ × (tî +  jˆ )
2

f sin q + N cos q = Mw02 (R – r) ...(b) = – ​ ​   ​ k̂  ​ N-m


t__3
3 (  )
Dividing (b) by (a) gives At t = 1s
​_› 1
​v  ​   = __ ​   ​  ( î + 2jˆ) m/s
w02 (R – r) f sin q + N cos q 2
​ ________
g    ​  = ​ _____________
       ​
f cos q – N sin q ​_› 1 ​_› 1
​t    ​ = – ​ __ ​  k̂  fi |​t   |​ = __ ​   ​  N-m
In limiting case f = m N 3 3

÷ 
40. Length of the incline is

( 
_______________________
g
\  w0 = ​ ______
​     
m sin q + cos q
 ​​ ​  _____________
         
(R – r) m cos q – N sin q
 ​ ​  )
g sin q
s = h/sin q

Acceleation a = ​ _______   
 ​
I
1 + ____
​   2 ​ 
mR
g sin q
aring = ______
​   ​   
2

We can have answer to this question only if q is 2g sin q


given. If we assume that q is small (which is not adisc = ​ _______  ​ 

3 ___

÷ 
stated in the question) then 1 2 2s
s = ​   ​  at    fi  t = ​ __
__ ​ a ​ ​  
sin q Æ 0  and  cos q Æ 1 2

÷  ÷ 
______ ______

÷ 
________ 4s 3s
g tring = ​ ______
​     ​ ;  tdisc = ​ ______
  ​     ​

\  w0 = ​ _______
​       ​
  g sin q g sin q
(R – r) m

÷  ÷ 
_________ _________ __
4s 3s   
2 – ​÷3 ​
kr 2
\ ​ _________
​       ​ ​ – ​ _________
  ​       ​ ​ = ​ ______
  ___ ​ 

38. Potential energy V = ___
​   ​  10 sin 60° 10 sin 60° ÷    
​ 10 ​
2
dV 3
Force F = – ___
​   ​ = – kr or h = __
​   ​  = 0.75 m
dr 4
Negative sign indicates that the force is attractive. 41. P represents a path:
This force provides the centripetal force. x = a t  and  y = b t
S.156  Mechanics II

y
\ ​ __x ​ =
b
__ (  )
b
​   ​   fi y = ​ __
a
​   ​  ​ x
a
3. Bernoulli’s equation:
1 1
​ __ ​  r v2 + P0 + 0 = __
​   ​  P (0.4)2 + P0 + rg (0.2 m)
This is a straight line. 2 2
dx d2x fi  v2 = 4.16
And ​ ___ ​   = a  fi ax = ___
​  2 ​ = 0
dt dt fi  v = 2.04 ms–1
dy d2y Continutiy equation:
​ ___ ​   = b  fi ay = ___
​  2 ​ = 0
dt dt d2 (8 × 10–3)2
p ​ __ ​  ◊ v = p ​ _________
 ​   
× 0.4
Thus the particle is moving on a straight line with 4 4
constant velocity. fi  d2 = 12.8 × 10–6  fi  d 3.6 × 10–3 m
\ P, L, K, U and E are constants. 4. FS + FB = Mg
Similarly, one can show that Q represents an elliptical AL
kx0 + ___
​   ​ s g = Mg
path. For such a path L and E are constants (think 2
of motion of a planet).
R is a circular path. L, K, U and E are constant
fi 
Mg
x0 = ___
​     
k [ L s A
​ ​ 1 – _____
​     
2M
​  ​ ]
Path S is parabolic
b
x = a t;  y = __
​   ​  t2
2
vx = a;  vy = bt
________
  2 + b2 t2 
\ v = ​÷a  ​
\ P and K are not constant.
ax = o;  ay = b
› _

\ ​F  ​   = mb jˆ

( 
​_› ​ › __
dV = – ​F  ​  ◊ dr​
P
​   = – (mbjˆ) ◊ ​ a dt î + __ )
​   ​  (2 + dt) ĵ   ​
2
5. This problem makes use of Boyle’s law, which we
will study in heat and thermodynamics section.
= – mb2t dt

mb 2t2
\ U = – _____
​   ​   + Uo
2
[Uo = a constant of integration]

Total energy E = K + U
1 mb2t2
= __
​   ​ m (a 2 + b2t2) – _____
​   ​  + Uo
2 2
1
or, E = ​ __ ​  ma2 + Uo = a constant.
2

Fluid Mechanism
Air in the tube is sealed at atmospheric pressure (P0).
AIEEE/JEE Main Quesitons Its volume is V0 = (A ◊ 8) where A = cross-sectional
area. When the tube is moved up, the air expands
1. In liquid 1, the ball sinks. It implies r3 > r1. The
and pressure drops to P. Therefore, Hg rises in the
ball floats in liquid 2 with only a part submerged.
tube.
\  r2 > r3 Let final length of air column be x (in cm).
2. This can be easily understood from pervious Final volume of air, V = A ◊ x
question. Using Boyle’s law, P1V1 = P2V2
Solutions  S.157

P0 (A ◊ 8 ) = P (A ◊ x) \  Centripetal force on considered cylindrical


76 × 8 volume of liquid is
fi  P = ​ ______
x    ​  

where P0 = 76 cm of Hg.

L
FC = (r AL) w2 ​ __
2 (  )
1
​   ​   ​ = __
​   ​  r AL2w2
2
Now pressure at point 2 must be atmospheric
pressure.
\ P + (54 – x) = 76
[pressure is being written in unit of height of Hg
column]

76 × 8
fi ​ ______
x   ​ + 54 – x = 76
Solving this equation gives x = 16 cm
6. – BOD = – DOC = 90°
h1 = R sin a 1
\  P2A – P1A = __
​   ​  r AL2w2
h2 = R cos a 2
h3 = R(1 – cos a) 1
fi  P2 – P1 = __
​   ​  r L2w2
h4 = R(1 – sin a) 2
1 w2L2
fi  r gH = __
​   ​  r L2w2  fi  H = _____
​     
​ 
2 2g

2. h = 3.000 – 0.525 = 2.475 m


Speed of efflux = v
Speed of the water at the surface in the tank = v0
Av0 = av
av
fi v0 = ___
​   ​   ...(i)
A

We will write pressure at A from two sides and equate


them.
d1g h4 = d2g (h1 + h2) + d1g h3

fi d1(1 – sin a) = d2 (sin a + cos a) + d1 (1 – cos a)

fi d1 [cos a – sin a] = d2[sin a + cos a]

d1 1________
+ tan a Bernoullis’s equation between 1 and 2.
fi ​ __ ​ =
  ​   
 ​
d2 1 – tan a 1 1
P0 + __
​   ​  r v2 + 0 = P0 + ​ __ ​  r v02 + rgh
2 2
Alternatively, we can check the options under
extreme conditions. For example, if d2 Æ 0, then a
[  ]
a2
fi  v2 ​ 1 – ___
​  2  ​   ​ = 2gh


d1
Æ 45°, __
​   ​ Æ • and the expression given in (c) also A

÷ 
d2 ______ ______________

÷
tends to infinity when a Æ 45°. 2gh 2 × 10 × 2.475
fi  v = ​ ______
​   2

  ​ ​ = ​ ​ ______________
  
    ​ ​ = 50
a
___ 1 – (0.1)2
IIT JEE (Advanced) Questions 1 – ​  2  ​ 
A
1. Consider a cylindrical volume of liquid as shown.
3. (i) When height of water is h1, net force on the
Mass of liquid = r AL. COM of this liquid cylinder
block is zero. Area of cross-section of the block
L
is at a distance ​ __ ​  from the rotation axis. is
2
A0 = p (2r)2 = 4p r2
S.158  Mechanics II

Area of lower face that is exposed to atmosphere 6. In situation (S), the sphere is denser than liquid. The
is sphere moves down (its PE decrease) and an equal
A1 = p r2 volume of liquid moves up (its PE increases).
Because sphere is denser, the PE of system (x + y)
decrease. Since liquid is non viscous, there is no loss
in mechanical energy and it remains constant.
  In option (t), density of sphere is larger than liquid.
Force by liquid (Buoyancy + viscous force) on sphere
is Mg upward.
Area of lower face that is in contact with Mechanical energy is dissipated due to viscosity.
water is 7. Let Vw = Volume occupied by water in the shell
A2 = A0 – A1 = 3p r2 Va = Volume occupied by air in the shell
\ P2A2 + P0A1 = P1A0 + W Vm = Volume of the material of the shell
For equilibrium: FB = W
[P0 + r gh1 + rgh] [3p r2] + P0 p r2
r (Vw + Va + Vm)
= [P0 + rgh1] [4p r2] + 4p r2 ◊ h ​ __ ​  g ​ _____________
 ​     rw g = Vmrm g + Vwrw g
3 2
5h
fi  h1 = ___ ​   ​  [Neglecting weight of air inside the shell]
3
(ii) When height of water is h2, cylinder is again in
equilibrium.
rm
fi  Vw + Va + Vm = 2Vm rC + 2Vw ​ rC = ___
​    ​  ​
rw [  ]
r
\  P0 ◊ 4p r2 + __ ​   ​  ◊ 4p r2hg fi  Vw = Vm (1 – 2rC) + Va
3
= (P0 + rgh2) (3p r2) + P0 ◊ p r2 1
If  rC > __
​   ​   then  Vw < Va
4h 2
fi  h2 = ___
​   ​  1
9 If  rC < __
​   ​   then  Vw > Va
2
(iii) for h < h2, the cylinder will not move up.
8. (i) A1v1 = A2v2
5. Initially, air above the water is at atmospheric pres-
sure (P0). As water flows out, the air expands and fi  p (20 mm)2 (5 mm s–1) = p (1 mm)2 v2
pressure (P) drops. When pressure in the tank at bot-
tom (point 1) becomes equal to outside atmospheric fi  v2 = 2000 mms–1 = 2 ms–1
pressure the flow of water stops. (ii) Due to high speed of air in the nozzle, the
pressure drops and liquid is forced up by the
atmospheric pressure. Let P = pressure inside
the nozzles.
1 1
P + __
​   ​  ra v22 = P0 + ​ __ ​  ra v12
2 2
[Pressure inside the tube P0 and speed
v1 = 5 mms–1 0]
1
P + rg (200 mm) = P0 \  P = P0 – __
​   ​  ra v22
2
fi  P = 1.0 × 105 – 103 × 10 × 0.2
= 98 × 103 Nm–2
Using Boyle’s law: PV = P0V0
98 × 103 × A (500 – 200) = 105 A (500 – H)
fi  H = 206 mm
\  Required answer is 206 – 200 = 6 mm
Solutions  S.159

Neglecting h (see figure), we can write for liq- 5. Work done is nearly equal to increase in surface
uid as energy. [When the size increases, the walls of the
1
P0 – P = __
​   ​  rl v2 bubble are pushed outward against the atmospheric
2 pressure. But this is being done by pushing in more
1
__ 1
__ air. The expansion of air itself is negligible]
2
fi ​   ​  ra v2 = ​    ​ rl v2
2 2
\ W = [8p R2 – 8p r2] S = 8p [(5 × 10–2)2

÷ 
___
ra
fi v = ​ __
​   ​ ​   ◊ v2 – (3 × 10–2)2] × 0.03
rl

÷ 
___
ra
\  Volume flow rate of liquid µ ​ __
​   ​ ​   = 384 p × 10–6 J = 0.384 p m J 0.4 p m J.
rl
9. Speed of efflux 6. The surface tension force on the table acts along a
____ circular line of length 2p r, as shown in figure.
  ​ 
v = ​÷2gh 
Resultant is vertical having magnitude.

÷  ÷ 
___ ___
2H ____ 2H
\ d = v ​ ___
​  g   ​ ​ = ÷   ​ ​ ___ r r2
​ 2gh  ​  g   ​ ​  Fst = T ◊ 2p r sin q = T . 2p r ◊ ​ __  ​ = 2p T __
​   ​ 
___ R R
  ​ 
d = 2 ​÷hH  The bubble detaches when buoyancy becomes just
d does not depend on g. Only when the lift falls larger than this surface tension force. For writing
freely, there will be no efflux. buoyancy, we can consider the volume of displaced
4
water as ​ __ ​  p R3 since r << R
3

Surface Tension and Viscosity

AIEEE/JEE Main Questions


1. geff = 0
In such situation, water can rise up to infinite height. 4 r2
\  ​ __ ​  p R3rwg = 2p T __
​   ​ 
Sine tube length is 20 cm only , water rises to the 3 R

÷ 
top of the tube. _____
2rwg
2 r2
2. v = __
​   ​  __
​   ​  (d – r) g fi  r = R ​ _____
2
​     
​ ​ 

9 h 3T
vs d_____ s – r None of the given answers is correct.
\ ​ __ vg ​ = ​  dg – r   ​
IIT JEE (Advanced) Questions
[ 
10.5 – 1.5.
fi  vs = 0.2 ​ ​ _________ 
19.5 – 1.5
1
 ​  ​ = 0.2 × __ ]
​   ​  = 0.1 ms–1
2 1. Force due to pressure is

3. At terminal speed, acceleration = 0


fi  F = 0
( 
P0 + P0 + rgh
F1 = Pav ◊ A = ​ ​  ___________
2
 ​ 
    ​ 2Rh. )
fi  Fviscous + FBuoyancy = w = 2P0 Rh + Rrgh2

fi  kv2 + Vr2g = Vr1g Force due to surface tension is F2 = 2RT


\  Resultant force is F = |​ F1 – F2 |​
÷ 
__________
Vg (r1 – r2)
fi  v = ​ ​ __________    ​ ​ 
    
k
4. Surface tension of soap water is less. Therefore, cap-
| 
= ​ 2P0 Rh + Rrgh2 – 2RT  ​ |
illary rise is less. 2. This question makes use of the ideal gas equation
S.160  Mechanics II

PV = nRT 4. The problem is based on finding the geometrical


At a given temperature, n µ P ◊ V relation between the radius (b) of the tube and radius
(R) of curvature of meniscus surface.
Pressure of air inside bubble A is
4S 4 × 0.04
PA = P0 + ___
​    ​ = 8 + ​ _______ ​ 

RA 0.02
= 16 Nm–2
Pressure inside bubble B is
4S
PB = P0 + ___
​    ​ = 12 Nm–2
RB

​ ___
nA _____

 ​ =
PA VA ___
​  ​

  =
PA ___
​  ◊
​ 

   ​​ ​  (  )
RA 3 ___
16 __
1 __ 1
nB PB VB PB RB  ​  ​​ ​ = ​ 12 ​ × ​ 8 ​  = ​ 6 ​ 
nB
\  ​ ___
nA  ​ = 6

3. The calculation of surface tension force on the drop


(due to dropper) is similar to Q.6 in AIEEE/JEE Main
Problems.

a
– QAS = 90° – ​ q + __
​   ​   ​
2 (  )
r – QAS + b = 90°
(i) Fst = 2p r T ◊ sin q = 2p r T __
​    ​
R
a
2p r2 T \  b = q + __
​   ​ 
= ​ ______     ​  2
R


2p r2 T
(ii) ​ ______
   
4
​ = mg = ​ __ ​  p R3rg
b
R
a
2 ( 
\  ​ __ ​   = cos ​ q + ​ __ ​   ​ )
R 3
b
fi R = __________
​       ​
3 r2T
fi  R4 = ​ __ ​  ____
2 r ◊ g
​     
​ (  a
cos ​ q + __ )
​   ​   ​
2
– 4 2 2S
3 (5 × 10 ) × 0.11
= ​ __ ​  ×  ​ _______________
  
 ​
   P1 = P0 – ​ ___ ​ 
2 R
103 × 10
2S
P2 = P1 + Pgh  fi  P0 = P0 – ​ ___ ​ + rgh
R 1.4 × 10–3 m R

(iii) Surface energy, U = T ◊ 4p R2 = 2.7 × 10– 6 J


fi 
2S
h = ​ ____  ​ = ​ ____________
a
__
2
     ​ 
( 
2S cos ​ q + ​   ​   ​ )
Rrg brg

5. Obivously, r2 > s 2 and r1 < s1.


For equilibrium of the system of two spheres

FBP + FBQ = WP + WQ

s1 Vg + s2 Vg = r1Vg + r2Vg

fi  s1 – r2 = r1 – s2 ...(i)

When P is alone in L2, it will move up (since density


of L2 is even larger than L1). It will attain a terminal
velocity in upward direction.
Solutions  S.161

2r2
vP = ____
​    ​ (s2 – r1)
9h2

For Q in L1
This force is directly proportional to A and inversely
2r2 proportional to h.
vQ = ____
​    ​ (r2 – s1) downward.
9h1 FV uo
​   ​ = h ​__
Shear stress = ___    ​ 
​_› ​_›
A h
Since ​v  ​P  and ​v  ​Q
  are oppositely directed, This is proportional to h
their dot product is negative.

|  | |  |
vP h1 (s2 – r1) h1 Gravitation
And ​ ___
​ v  ​  ​ = ___
​   ​ ​ ​ ________ 

 ​  ​ = ___
​   ​   [Using (i)]
Q h2 (r2 – s1) h2
AIEEE/JEE Main Questons
r2(d – P)
6. v  µ ​ ________
   ​ 
GMm
1. PE = – ​ _____
   ​ 
h R

(  ) ( 
rP 2 h2 d – r1
)
vP GMm
\ ​ ___ ​​ __
​ r  ​  ​​ ​ ​ ___ ​  ◊ ​ ​ _____  \  Work needed to free the particle = _____
​     ​

vQ ​ = Q h1 d – r2
 ​  ​ R
100 × (10 × 10–3)
= 6.67 ×10–11 × ​ _______________
(  ) ( 
 ​
    

2 2 2
= ​​ __
​   ​   ​​ ​× __
1 3
8 – 0.8
​   ​  × ​ ​ ______ 
8 –1.6
 ​  ​ ) = 6.67 × 10–10  J
0.1

÷ 
=3 ______

÷ 
________
2GMp 2G 10 Me
7. Conservation of volume gives 2. v = ​ ​ _____ ​ ​ 
     = ​ ________
​  = 10 × 11 = 110 kms–1
 ​ ​ 
  
Rp Re/10
4 4
k ◊ ​ __ ​  p r3 = __
​   ​  p R3  fi  k r3 = R3 ...(i) 3. Acceleration due to gravity at height h is
3 3
g g
Ui = S ◊ 4p R2 ​ ________
  2   ​ = __
​   ​ 

Uf = k (S ◊ 4p r2)
h
​​ 1 + __ ( 
​   ​   ​​ ​
R )9

h
Given k (S . 4p r2) – S ◊ 4p R2 = 10– 3 fi  1 + ​ __  ​ = 3  fi  h = 2R
R
4p S [kr2 – R2] = 10– 3 GMm
4. Energy needed is = _____​     
​ = gmR
R
fi 
0.1 __​ 1 ​ 
4p ___ [  ]
​   ​ ​ ​k​3 ​ – 1  ​ R2 = 10– 3
4p
= 10 × (1000) × (6400 × 103) = 6.4 × 1010 J

​ from (i) r2 = ___


R2
​  2/3  ​  ​
k [  ] GMm
5. E = Ef – Ei = – ​ _____ 
2 (3R)
 ​  ( 
GMn
– ​ – ​ _____
R

    ) 5 GMm
​  ​ = __
​   ​  _____
6 R
​     ​ 

a/3 a/3
fi  10 – 1 = 100 fi  10 = 101 6. Force on mass at A due to masses at B, C and D
are
fi  a 6
2s cos q GM2
8. Capillary rise h = ​ ________
   ​  FC = _____
​    ​ 
rrg (2R)2
For a given material of the tube q is constant. there- GM2 G ◊ M2 GM2
fore, h decreases with increase in r. FB = FD = _____
​   
 ​ 
= _______
​  __     ​ = ____
​   ​ 
(AB)2     R)2
(​÷2 ​ 2R2
h is dependent on s.
In a lift that is accelerating up, the effective accelera-
tion is greater than g. Hence, h decreases.
h is not proportional to q; it is proportional to
cos q.
uo
9. Velocity gradient = __
​   ​ 
h
uo
viscous force is FV = h A __ ​   ​ 
h
S.162  Mechanics II

Resultant force on mass at A is along AC.

( 
_________
F = ​÷F   B2 + F  
D
2
 ​ + F C =
GM2 ___
____
​   ​ 
R2 ​÷2 ​
1
   4
1
 ​ ​  __  ​ + __
​   ​   ​ )
GM2 __
= ____     + 1)
​  2 ​ (2​÷2 ​
4R
This force provides centripetal force.

Mv2 GM2 __
​ ____
  
 ​ = ____     + 1)
​  2 ​ (2​÷2 ​
R 4R
3. Consider a ring of radius x and with dx.

÷ 
____________
1 GM __
fi  v = __ ​   ​  ​ ____
​           
​ (2​÷2 ​ + 1) ​ Mass of ring element is
2 R
M
M dm = ____________
​        ​ ◊ 2p xdx
7. Removed part has mass = __ ​   ​  p (16R2 – 9R2)
8
Vcentre of cavity = ​V​due to M​ – ​V​ __ M​ 2 M
due to ​   ​  = __
​   ​  ___
​    ​  x dx
8 7 R2
= – ​ ____
GM __
R 3
3
2 [ 1 R 2
 ​ ​ ​   ​  R2 – __
​    ​ ◊ ​​ __
2 2 (  ) ] [  3 GM/8
​   ​   ​​ ​  ​ – ​ – ​ __ ​  _____
2 R/2
​  ]
 ​  
 ​ Entire mass dm is at a distance r from point P,
where
________
GM
= – ​ ____     ​ r = ÷   2 + x2 
​ 16R  ​
R
8. For near surface satellite, orbital speed is \  Potential at P due to dm is
___
v0 =   ​ 
​÷gR 
____
  ​ 
Escape speed is v = ​÷2gR 
__ ___
\  D v = (​÷2 ​   ​ 
    – 1) ​÷gR 

9. Variation inside the spherical earth is linear.

Iit jee (Advanced) questoins


1. When circular path is 1.

____
​  x   
​ =
  ​ 
4
Gm ​ __
​   ​  p x3 ◊ r0   ​
mv 2 _____________
3
 ​    
(  )
x2 Gdm 2 GM __________ xdx
dV = – ​ _____​ = – ​ __ ​  ____
r    ​   ​ ​  ________
     ​
[Force is only due to mass inside sphere of 7 R2 ​÷16R   2 + x2  ​
radius x] Potential due to given disc is

÷ 
______
4p Gr0 4R
fi  v = ​ ______
​   ​ ​ ◊ x



2M xdx
 ​ ​Ú ​ ​   __________
V = – ​ ____2  ​​  ________
     ​
3 7R 3R ​÷16R   2 + x2  ​
\  v µ x
For integration take 16R2 + x2 = t
When path is 2 (x > R) fi  2x dx = dt

mv
​ ____
4
Gm ​ __ ( 
​   ​  pR3 ◊ r0   ​
3 2
​   = _____________
) fi 
1
xdx = __
​   ​  dt
2
x    ​ 
x2
 ​    
And t = 25R2  when  x = 3R

÷ 
________
4p Gr0R3 ___ t = 32R2  when  x = 4R
1__ 1
fi  v = ​ ________
​   ​ ​ ◊ ​ 

     ​   fi  v µ ___
​  __  ​  32R2
3 ​ x 
÷    ​ ​ x 
÷   ​ 
2GM __ 1 dt
\  V = – ​ _____   Ú  ​ ​   __
 ​ ​   ​ ​
2 2
​​  _  ​ 
7R 25R2 ​
÷   ​ 
t 
Solutions  S.163

_____ _____
2GM [ ]​
= – ​ _____ ​ ​  ​÷32R
  2 ​ – ​÷25R
  2 ​  
7R2
2GM __
= – _____
​          – 5]
​ [4​÷2 ​

7R
PE of unit mass at P is V.
PE of unit mass at • is zero.
2GM __
\  W = 0 – V = _____
​          – 5)
​ (4​÷2 ​

7R
__ __
GMP   
​÷6 ​     GMe
​÷6 ​ Force on a small element of length dx of the wire
4. ​ _____
 ​ 
 = ___
​   ​ g = ___
​   ​ ​ ____
 ​ 

RP2 11 11 Re2 is
GM
4
__ __ __4 dF = (dm) g = (l dx) ​ ____ ​ x
​   ​  p RP3 rp ​   ​  p Re3 re R3
3________ ___     _______
​÷6 ​ 3
fi  ​   ​  

= ​   ​ ​   ​    Total gravitational force on wire
Rp2 11 Re2

[  ]
R
__ __ l GM l GM __1 16R2
  
​÷6 ​ 2 ÷   
​ 6 ​ F = _____  Ú​   ​   ​x dx = _____
​  3 ​   ​    ​ ​ R2 – ​ _____
​  3 ​   ​  
 ​
fi  Rp rp = ___
​   ​ Rere  fi  Rp ​ __ ​  = ___
​   ​ R R ___ R 2 25
11 3 11 e 4R
​   ​ 
5
__
Rp 3​÷6 ​ 9 l GM
  
fi  ​ ___  ​ = ____
​   ​   ...(i) = ___
​    ​ _____
​     
​ 
Re 22 50 R

÷ 
___________
4 External agent must apply equal force on the wire in
2G ​ __ ​  p  3 ◊ r
÷ 
_____
2GM
_____ 3
__________ radially outward direction to hold it in place
Escape speed, v = ​ ​     
​ ​  
= ​ ​         ​ ​ 
R R 9 l GM
__ \  Fext = ___
​    ​ _____
​     
​ 
\  v µ R ​÷r 
   ​ 50 R
1
Volume of planet is ​ _____   
 ​ times of volume of
__ 1000

÷ 
__ __
vp Rp ​÷r  p ​  3​÷6 ​
    __ 2 3
Earth.
\  ​ __
ve  ​ =
______
​  __ ​  ____
= ​   ​  ◊ ​ ​   ​ ​   = ___
​    ​  Me
  e  ​ 22 3
Re ​÷r 11 \  Mass of planet M = _____
​    ​ 
1000
3
fi  vp = ___
​    ​ × 11 = 3 km s–1 9 l Me 9l  GMe
11 \  Fext = ___
​    ​ _________
​      ​ = ​ _____  ​ ____
​     
​ 
50 R 5000 Re
1000 × ​ ___  ​
__ e
5. Escape speed = ÷    v
​ 2 ​  10
__
1
k = __ 9 × 10– 3 9 × 10– 3
\      v)2 = mu2
​   ​  m (​÷2 ​ = ​ _______ × (geRe) = ​ _______
 ​ 
  × 10 × 6 × 106
 ​ 

2 5000 5000
6. Energy of mass m is conserved as gravitational force = 108 N
is conservative.
8. Consider a cylinder of fluid extending from surface
(A) to a point (B) at a distance r from the centre. Cross
section of the cylinder is D S. Pressure at B is

Weight of cylinder AB
P = ​  ___________________
       ​
1 GMm _____ GMm D S
For m to just escape, __
​   ​  mu2 – _____
​     ​ – ​     
  ​= 0

2 L L

÷ 
____
GM
fi  u = 2 ​ ____
​     
​ ​  
L
7. Acceleration due to gravity at a distance x from the
centre is
GM
g = ____
​  3 ​  ◊ x
R
S.164  Mechanics II

To Find weight of cylinder, consider an elemental 11. Ms = 3 × 105 Me


length dx as shown.
r = 2.5 × 104 Re
dW = r (D S dx) g
Acceleration due to gravity at distance x from the
centre is
4
G __​   ​  p R3 ◊ r
GM 3 4
g = ____
​  3 ​  ◊  = ​ __________   ​  x = __
  ​   ​  p Gr ◊ x
R R3 3
For escaping, the total energy of projectile in the
4 gravitational field of sun + earth shall be zero.
\  dW = __
​   ​  p Gr2 D S xdx
3
1 GMsm ______GMem
R ​ __ ​  mvs2 – ______
​  r   ​ – ​  ​ = 0
   
4 2 2 Re
​   ​  p Gr2 D S ​Ú ​  ​xdx = __
\  W = __ ​   ​  p Gr2 D S (R2 – r2)
3 r 3
1 G  ×  3 × 105 Me GMe
fi  ​ __ ​  vs2 = ​ _____________
    ​ +
   ____
​     
​ 
W 2 2 2.5 × 104 Re Re
\  P = ___ ​ = __
​      ​   ​  p Gr2 (R2 – r2)
D S 3

÷ 
______
Using the above expression you can check which of 2 GMe ___ ___
fi vs = ​ ______
​  ◊ ​÷13 ​
​ ​  
        = ÷    ve
​ 13 ​ 42 kms–1
the given options are correct. Re

÷ 
9. When bullet is at a distance r from the centre of the ____
GM
planet, force on it is 12. Orbital speed is v = ​ ____
​     
​ ​  
R

÷ 
GMm ___
F = _____
​  2 ​    v1 R2
r \ ​ __
v2 ​ =
  ​ ___
​   ​ ​  = 2
Force becomes one fourth when r = 2R. R1
(R = radius of the planet). This ratio is correctly matched in option (b). Just by
When the bullet is fired with speed v, it goes upto a looking at this one can say that the correct answer
distance 2R from the centre of the planet. is (b)
1 GMm GMm
\  ​ __ ​  mv2 – _____
​      ​  = – _____
​     
​    Elasticity
2 R 2R

÷ 
____
GM ve AIEEE/JEE Main Questions
fi  v = ​ ____ ​ ​    fi  v = ___
​      ​  __  ​ 
R
(  )
÷   
​ 2 ​
1 1 S S2
__ 1. u = __
​   ​  × stress × strain = __
​   ​  S ​ __
​   ​  ​ = ​ ___  ​ 
fi     v
ve = ​÷2 ​  2 2 Y 2Y
10. Let acceleration of both point masses be a towards 2. Tension in the wire is still W.
the centre of sphere [Masses are connected with rod.
\  Extension = l
Therefore, they will move together]
For the middle mass 3. Wire 1: cross section = A, length = 3L
GMm _____ Gmm Wire 2: cross section = 3A, length = L
ma = _____
​  2
 ​ – ​  2 ​  
  [Tension = 0] ...(i)
(3l) l Stress
For wire 1: ​ _____
   
​ = Strain

Y
For right most mass
F D x AYD x
fi ​ ___   ​ = ___ ​   ​   fi  F = _____
​     ​
  ...(i)
GMm _____ Gmm AY 3L 3L
ma = ​ _____2
 ​ 
+ ​  2 ​  
  ...(ii)
(4l) l F¢ D x
For wire 2: _____
​    ​  = ___
​     ​
3A ◊ Y L
From (i) and (ii)
3AY D x
GMm ______Gm ◊ m _____
GMm _____
Gmm fi  F¢ = _______
​      ​ = 9F [from (i)]
​ _____ ​ –
  ​  2 ​ 
   = ​  2 ​ 
+ ​  2 ​ 
  L
9l2 l 16l l 4. Volume (or mass) increases by a factor = 93
7M Cross sectional area of leg increases by a
fi  m = ____
​    ​ 
288 factor = 92
Solutions  S.165

93 Let cross-section of the two wires be 4A and A.


\  Stress increases by a factor = __
​  2 ​  = 9
9 For thick wire:
4
5. V = ​ __ ​ p r3 Stress
3 Strain = _____
​      ​

Y
4
fi DV = ​ __ ​ p(3r2Dr) DL1 _____
3 F FL
fi  ​ ____ ​ =
  ​     ​   fi  DL1 = ​ ____   

DV 3Dr 2L 4A ◊ Y 2AY
\ ​ ___   ​ = ​ ____ ​ 
r   
V
For thin wire:

DV/V
P
K = ​ _____   ​   fi
Dr
​ ___
r   ​ =
mg
____
​     
3Ka
[  mg
​ ​   P = ​ ___ ]
a   ​  ​ DL2 F FL
​ ____
   
​ = ___
​     ​  fi  DL2 = ___
​    ​
L AY AY
IIT JEE (Advanced) Questions
DV DL2
1. Volume strain ___
​     ​ = 0.1 \  ​ ____ ​ =

2
__
​   ​ 
V DL1 1
Volume stress DP = 0.155 × 105 Pa. 3. For P fracture happens at higher stress.
0.155 × 10 5 \  P has more tensile strength.
\  B = ​ __________
 ​ = 1.55 × 105 Pa
    
0.1 In the graphs, the end points are fracture points.
2. Tension is same in both wires. Let is be F. P is more ductile as it fractures after a lot of strain.
It can be stretched into a wire.

You might also like